Sie sind auf Seite 1von 320

Universidade Estadual do Cear

REITOR
Francisco de Assis Moura Araripe

VICE-REITOR
Antnio de Oliveira Gomes Neto

PR-REITOR DE ADMINISTRAO
Luis Carlos Mendes Dodt

PR-REITOR DE PLANEJAMENTO
Vladimir Spinelli Chagas

PR-REITOR DE POLTICAS ESTUDANTS PRAE


Joo Carlos Holanda Cardoso

PR-REITORA DE GRADUAO
Josefa Lineuda da Costa Murta

PR-REITOR DE PS-GRADUAO E PESQUISA


Jos Jackson Coelho Sampaio

PR-REITORA DE EXTENSO
Celina Magalhes Ellery

UECEVEST
NCLEO DE AO COMUNITRIA
Zoraide Braga Nogueira Marques

COORDENAO PEDAGGICA
Abnza Pontes de Barros Leal

COORDENAO ADMINISTRATIVA
Magali Mirian Milfont Tefilo

COORDENAO PEDAGGICA POR REA


Eddie William de Pinho Santana Biologia
Eveline Solon Barreira Cavalcanti Qumica
Francisco Agileu Lima Gadelha Histria
Francisco Jos Pereira Lngua Portuguesa
Jos Stenio Rocha Fsica
Maria Ivonisa Alencar Moreno Matemtica
Maria Liduina dos Santos Rodrigues Espanhol e Ingls
Rejanny Mesquita Martins Rosa Geografia

SECRETRIAS
Daniela Cludia Matos dos Santos
Fabiana Moraes Frota

APOIO DE SECRETARIA
Antnio Albert Vidal Almeida

Apostilas UECEVEST mod3.indb 1 06/02/2011 09:56:58


PROFESSORES ORGANIZADORES
Lngua Portuguesa Rodrigo Alves Patricio
Francisco Jos Pereira (Coordenador) Rogrio dos Santos Andrade
Emanoel Pedro Martins Gomes Wendel Macedo Mendes
Esdras Pereira Anto
Eulidiane Morais da Silva Geografia
Francisco Roque Magalhes Neto Rejanny Mesquita Martins Rosa (Coordenadora)
Jefrei Almeida Rocha Naiana Paula Lucas dos Santos
Marcos Alberto Xavier Barros Robson Almeida Machado
Nathalia Barreto de Queiroz Washington Bezerra de Oliveira
Nathalia Mugnaro
Qumica
Lngua Estrangeira Eveline Solon Barreira Cavalcanti (Coordenadora)
Maria Liduina dos Santos Rodrigues (Coordenadora) Alan Ibiapina de Andrade
Igor Augusto de Aquino Pereira Celso Pires de Araujo Junior
Ivana Roberta Siqueira Marreio Everardo Paulo de Oliveira Junior
Janaina Rodrigues Freitas Joo Rufino Bezerra Neto
Karoline Matos Monteiro Levy Bruno Correia Bezerra
Mrcio Freitas de Alcntara Regina Amanda Franca Almeida
Wallysson Gomes Pereira
Matemtica
Maria Ivonisa Alencar Moreno (Coordenadora) Histria
Artur Teixeira Pereira Rejanny Mesquita Martins Rosa (Coordenadora)
Anderson Douglas Freitas Pedrosa Flavio da Conceio
Francisco de Paula Rego Carvalho Jose Ren de Franca Silva
Hudson de Souza Felix Vicente Gregrio O. M do Amaral
Redomarck Barreira Cunha Wendell Guedes da Silva
Rafael Pereira Eufrazio Waldejares Silva de Oliveira
Wesley Liberato Freire
Waldeglace Rodrigues Pereira Biologia
Eddie William de Pinho Santana (Coordenador)
Fsica Andr Luiz B..S. Brasilino
Maria Ivonisa Alencar Moreno (Coordenadora) Antonio Carlos Nogueira Sobrinho
Adriano Oliveira Alves Camylla Alves do Nascimento
Dimitry Barbosa Pessoa Donisethi Teixeira Llis Jnior
Francisco de Assis Leandro Filho Karoline Soares Garcia
Paulo Vicente de Cassia L. Pimenta Maria da Conceio de Souza
Pedro Augusto Martins Sarnento Michael Robert Martins Rocha

Copyright 2011 Curso Pr-Vestibular UECEVEST

Todos os direitos reservados. proibida a reproduo total ou parcial desta edio, por qualquer meio ou
forma seja mecnica ou eletrnica, fotocpia, scanner, gravao, etc , nem apropriada ou estocada em
sistema de banco de dados, sem a expressa autorizao do Curso Pr-Vestibular UECEVEST.

UNIVERSIDADE ESTADUAL DO CEAR UECE


Pr-Reitoria de Polticas Estudants PRAE
Curso Pr-Vestibular UECEVEST
Fone: (85) 3101.9658
Av. Parajana, 1700 Campus do Itaperi 60.740-903
Fortaleza Cear

O presente material uma ao conjunta da Secretaria de Educao do Estado do Cear SEDUC, com a
Universidade Estadual do Cear UECE, atravs do Convnio de Cooperao Tcnica Cientfica n 07/2009.

Apostilas UECEVEST mod3.indb 2 06/02/2011 09:56:59


SUMRIO

Gramtica ....................................................................................................................................... 05

Literatura ........................................................................................................................................ 21

Redao ........................................................................................................................................... 39

Ingls ............................................................................................................................................... 53

Espanhol ......................................................................................................................................... 61

Geografia ......................................................................................................................................... 69

Histria Geral I .............................................................................................................................. 115

Histria Geral II ............................................................................................................................. 127

Histria do Brasil ............................................................................................................................ 149

Matemtica I .................................................................................................................................... 167

Matemtica II .................................................................................................................................. 199

Fsica I ............................................................................................................................................. 217

Fsica II ............................................................................................................................................ 233

Qumica Geral ................................................................................................................................ 247

Qumica Orgnica .......................................................................................................................... 265

Fsico-Qumica ............................................................................................................................... 275

Biologia I ......................................................................................................................................... 287

Biologia II ....................................................................................................................................... 303

Apostilas UECEVEST mod3.indb 3 06/02/2011 09:56:59


Apostilas UECEVEST mod3.indb 4 06/02/2011 09:56:59
P R - V E S T i B U l A R

GRAMTICA

01 GRAMTICA.indd 5 06/02/2011 20:25:17


Caro(a) Aluno(a),

O mdulo que voc tem em mos possui contedos relacionados s Matrizes de Referncia para a rea de
Linguagem, Cdigos e suas Tecnologias, do Exame Nacional do Ensino Mdio (ENEM). Abaixo, h a indicao
das competncias da rea e de suas habilidades que, neste mdulo, so contempladas, e, em seguida, dos objetos
de conhecimento associados s Matrizes.

Competncia de rea 1 Aplicar as tecnologias da comunicao e da informao na escola, no trabalho e
em outros contextos relevantes para sua vida.
H1 Identificar as diferentes linguagens e seus recursos expressivos como elementos de caracterizao dos sis-
temas de comunicao.
H4 Reconhecer posies crticas aos usos sociais que so feitos das linguagens e dos sistemas de comunicao
e informao.
H3 Relacionar informaes geradas nos sistemas de comunicao e informao, considerando a funo social
desses sistemas.

Competncia de rea 6 Compreender e usar os sistemas simblicos das diferentes linguagens como
meios de organizao cognitiva da realidade pela constituio de significados, expresso, comunicao e
informao.
H18 Identificar os elementos que concorrem para a progresso temtica e para a organizao e estruturao
de textos de diferentes gneros e tipos.

Competncia de rea 7 Confrontar opinies e pontos de vista sobre as diferentes linguagens e suas ma-
nifestaes especficas.
H21 Reconhecer em textos de diferentes gneros, recursos verbais e no-verbais utilizados com a finalidade de
criar e mudar comportamentos e hbitos.
H22 Relacionar, em diferentes textos, opinies, temas, assuntos e recursos lingusticos.
H23 Inferir em um texto quais so os objetivos de seu produtor e quem seu pblico alvo, pela anlise dos
procedimentos argumentativos utilizados.
H24 Reconhecer no texto estratgias argumentativas empregadas para o convencimento do pblico, tais como
a intimidao, seduo, comoo, chantagem, entre outras.

Competncia de rea 8 Compreender e usar a lngua portuguesa como lngua materna, geradora de
significao e integradora da organizao do mundo e da prpria identidade.
H25 Identificar, em textos de diferentes gneros, as marcas lingusticas que singularizam as variedades lingus-
ticas sociais, regionais e de registro.
H26 Relacionar as variedades lingusticas a situaes especficas de uso social.
H27 Reconhecer os usos da norma padro da lngua portuguesa nas diferentes situaes de comunicao.

Objeto de conhecimento
Estudo dos aspectos lingusticos em diferentes textos: recursos expressivos da lngua, procedimentos
de construo e recepo de textos organizao da macroestrutura semntica e a articulao entre ideias
e proposies (relaes lgico-semnticas).
Estudo dos aspectos lingusticos da lngua portuguesa: usos da lngua: norma culta e variao lingus-
tica uso dos recursos lingusticos em relao ao contexto em que o texto constitudo: elementos de
referncia pessoal, temporal, espacial, registro lingustico, grau de formalidade, seleo lexical, tempos e
modos verbais; uso dos recursos lingusticos em processo de coeso textual: elementos de articulao das
sequncias dos textos ou construo da microestrutura do texto.

01 GRAMTICA.indd 6 06/02/2011 20:25:17


GR A M T I C A

SinTAxE Frases imperativas: o emissor expressa uma ordem, um


pedido, uma splica ou um conselho. Vem acompanhada de
Parte da gramtica que estabelece as relaes de combinao um vocativo:
(ordenao, dependncia e concordncia) entre as palavras. Ex.: Chiquinho, sai da, peste! (Mrio de Andrade)

Frases optativas: o emissor expressa um desejo, uma


Anlise sinttica
possibilidade:
Ex.: Assim eu quereria o meu ltimo poema! (Manuel Bandeira)
Por que estudar anlise sinttica?
Para conhecer melhor como se estruturam e se articulam as Frases imprecativas: o emissor expressa uma imprecao, isto
frases em nossa lngua, para o aperfeioamento de nossa escrita e , uma splica insistente por meio de maldio:
para o estudo de outros assuntos gramaticais. Ex.: Um raio que te parta, maldito! M lepra te consuma,
coisa ruim! Uma cascavel que te morda a lngua, co danado!
Conceitos essenciais (Bernardo Guimares)
Em uma anlise sinttica podemos ter:
Orao
Frase ideia que se organiza em torno de um verbo.
a reunio de palavras que expressam uma ideia completa, Ex.: Tudo comea com o pagamento da dvida.
constitui o elemento fundamental da linguagem, no precisa ne-
cessariamente conter verbos. (Revista Vida Pessoal, 12/99, p.07)
Ex.: Final de ano, incio de tormento. (Revista Nova Escola, 11/00)
diCA
Frase nominal O verbo pode estar elptico (no aparece, mas existe pois est
aquela que tem o seu ncleo significativo concentrado num subentendido no contexto)
nome (substantivo, adjetivo, numeral ou advrbio). Ela se carac- Ex.: O Jeca-Tatu de Monteiro Lobato fez tanto sucesso quanto
teriza por no apresentar verbo que indique movimento ou ao, (fizeram) os Fradinhos que Henfil lanou nas pginas do Pasquim.
pois traduz uma viso esttica da realidade. No entanto, na frase (Revista poca, 24.05.99, p.06)
nominal, pode aparecer verbo de ligao, pois este apenas funcio-
na como um elo entre o sujeito e a qualificao ou estado dados Perodo
a ele. Observe os exemplos: o conjunto de 2 ou mais oraes. O perodo pode ser:
simples constitudo por apenas uma orao.
Ex. 1: A cabea agora na pedra, sem o palet. E o dedo sem a Ex.: Macunama o heri com muita preguia e sem nenhum
aliana.(Dalton Trevisan) carter. (poca, 24.05.99, p.7)
Ex. 2: Em redor, tudo parado. Esttico. No silncio da madru-
gada, nem o piar de um pssaro (Lygia Fagundes Telles) composto constitudo por mais de uma orao.
Ex.: Ns no podemos fingir /que as crianas no tm incons-
SAibA mAiS ciente. (Nova Escola, 11/00)
So os verbos de ligao os que, na frase nominal exprimem
uma viso esttica do ser. Os mais comuns so: ser, estar, tornar-
se, permanecer, continuar, ficar e parecer.

Frases verbais TERmOS DA ORAO


So aquelas que tm um ncleo significativo concentrado em
um verbo, que no de ligao, ou em uma locuo verbal. In- Sujeito
dicam movimento ou ao, pois traduzem uma viso dinmica Leia: Um trecho da msica Esquadros, de Adriana Calcanhoto
da realidade. Eu ando pelo mundo prestando ateno
Ex. 1: No volte sozinha para casa, de noite. Em cores que eu no sei o nome
(Carlos Drummond de Andrade) (...)
Ex .2: Todos caminhavam rumo aos fogos de artifcio. Eu presto ateno no que meu irmo ouve
E como uma segunda pele, um calo, uma casca, uma cpsula
As frases quanto ao sentido protetora
Observando os elementos que as constituem, j sabemos que Eu quero chegar antes pra sinalizar o estar de cada coisa
as frases podem ser nominais ou verbais, de acordo com seu n- ( ...)
cleo de significao. Pela janela do quarto, pela janela do carro Pela tela, pela janela
Alm dessa classificao, podemos ainda analis-las a partir de (Quem ela, quem ela?)
seu sentido. De acordo com esse critrio teremos: Eu vejo tudo enquadrado
Frases declarativas: aps a constatao de um fato, o emissor Remoto controle
ou autor do enunciado faz uma declarao: (...)
Ex.: Tudo vale a pena se a alma no pequena.(Fernando Pessoa)
No verso: Eu ando pelo mundo prestando ateno
Frases interrogativas: o emissor formula uma pergunta. Quem anda? eu = sujeito.
Ex.: Que bichos so estes? (Menotti Del Pichia)
Chamamos de sujeito o termo a respeito do qual damos al-
Frases exclamativas: o emissor revela um estado emotivo: guma informao. Seu ncleo (palavra mais importante) pode
Ex.: J madrugada! Puxa, que pernada! (Menotti Del Pichia) ser um substantivo, um numeral, um pronome ou uma palavra
substantivada. Outra maneira de se achar o sujeito procurando

UECEVEST 7

01 GRAMTICA.indd 7 06/02/2011 20:25:18


G R A M TICA

a palavra ou expresso que concorda em nmero e em pessoa com AtenO


o verbo. Muitas vezes tambm, o sujeito o ponto de partida para Os verbos indicadores de fenmeno da natureza, na ocorrn-
que ocorra a ao expressa pela verbo ao qual ele se refere. cia de sujeito inexistente, tm que estar no seu uso denotativo
Ex.: O Jeca-Tatu de Monteiro Lobato fez tanto sucesso quanto (literal, sentido primrio). Caso contrrio, a orao ter sujeito,
(fizeram) os Fradinhos que Henfil lanou nas pginas do Pas- como no exemplo abaixo:
quim. (Revista poca, 24.05.99, p.06) Ex. Ontem, choveu canivete na festa. Sujeito: canivete
Sujeito da 1 orao: O Jeca-Tatu de Monteiro Lobato
Ncleo do sujeito: Jeca-Tatu (substantivo) Fazer, ser, estar indicarem tempo cronolgico.
Ex.: Faz meses que ele no aparece.
Tipos de sujeito J uma hora da tarde.
Simples Est quente em So Paulo.
Composto
Oculto, elptico ou desinencial ObS.:
Indeterminado O verbo ser, impessoal, concorda com o predicativo, po-
Inexistente ou sem sujeito dendo, assim, aparecer na 3 pessoa do plural.
Ex: J so trs horas da tarde.
Sujeito Simples
Aquele que possui apenas um ncleo. haver com sentido de existir, de ocorrer, e quando indicar
Ex.: Livros ganham as prateleiras dos supermercados. tempo decorrido.
(poca, 24.05.99, p.124) Ex.: Havia mulheres na sala.
Ncleo: livros H trs anos ele partiu.

Sujeito Composto Ver outras ocorrncias do sujeito inexistente em sala com


Aquele que possui mais de um ncleo. o professor.
Ex.: Jogadores e torcedores reclamaram da arbitragem.
Ncleo: Jogadores e torcedores AtenO
Os verbos impessoais sempre ficaro na 3 pessoa do singular
Sujeito oculto, elptico ou desinencial (havia, faz...) Observar que esses verbos impessoais tambm po-
Aquele que no vem expresso na orao, mas pode ser facil- dem aparecer conjugados, ou seja, com pessoas (1, 2 ou 3/ sig.
mente identificado pela desinncia do verbo. Ou pl.), mas nesses contextos eles passam a ter sujeito expresso.
Ex.: Aonde vou, o que quero da vida?
(Estado de Minas, 02.07.00, p.21)
E x E R C C i O
Apesar de o sujeito no estar expresso, pode ser identificado
nas duas oraes: eu. Nesse caso, recuperou-se o sujeito pela de- 01. Um esparso tilintar de chocalhos e guizos morria pelas que-
sinncia do verbo com o qual ele concorda. bradas. Qual o sujeito e o tipo de sujeito desta orao?
a) Um esparso tilintar de chocalhos e guizos / simples.
Sujeito indeterminado b) Um esparso tilintar de chocalhos e guizos / composto.
Aquele que no se quer ou no se pode determinar. c) Um esparso tilintar / simples.
Ex.: Vive-se melhor em uma cidade pequena. d) Um esparso tilintar / composto.
Absolveram o ru. e) Chocalhos e guizos / composto.
Falaram mal de voc.
02. Marque a nica opo em que o sujeito no indeterminado.
Se tiver contexto nestes dois ltimos exemplos, o sujeito po- a) Falaram mal daquela moa.
der ser elptico: eles. b) Mataram um guarda.
c) Vive-se bem aqui.
AtenO d) Precisa-se de professores.
O sujeito pode ser indeterminado em trs situaes: e) Vendem-se carros usados.
verbo na terceira pessoa do plural sem sujeito expresso: Telefo-
naram por engano para minha casa. 03. Veja o texto:
verbo na terceira pessoa do singular acompanhado do pronome Congresso internacional do medo
SE (ndice de indeterminao do sujeito): Acredita-se na exis- Provisoriamente no cantaremos o amor, que se refugiou
tncia de polticos honestos. mais abaixo dos subterrneos. Cantaremos o medo que, esteriliza
Se o verbo for transitivo direto e o se for partcula apassiva- os abraos, no cantaremos o dio porque esse no existe, existe
dora, o sujeito no ser indeterminado, pois estar expresso na apenas o medo, nosso pai e nosso companheiro, o medo gran-
orao. Ex. Aluga-se uma casa na praia. (Sujeito paciente - uma de dos sertes, dos mares, dos desertos, o medo dos soldados, o
casa na praia). medo das mes, o medo das igrejas, cantaremos o medo dos di-
tadores, o medo dos democratas, cantaremos o medo da morte e
Sujeito inexistente ou orao sem sujeito o medo de depois da morte, depois morreremos de medo e sobre
A informao contida no predicado no se refere a sujeito nossos tmulos nascero flores amarelas e medrosas.
algum. Ocorre orao sem sujeito quando temos um verbo im- (Carlos Drummond de Andrade)
pessoal. O verbo impessoal quando:
Marque V ou F caso as afirmativas sejam verdadeiras ou falsas.
indica fenmenos da natureza (chover, nevar, amanhecer, etc.) a) ( ) O poeta utiliza vrias vezes a forma verbal cantaremos,
Ex.: Anoiteceu muito cedo. que tem sujeito oculto ns.
Choveu muito no Rio de Janeiro este ms. b) ( ) Em no cantaremos o dio porque esse no existe,

8 UECEVEST

01 GRAMTICA.indd 8 06/02/2011 20:25:18


GR A M T I C A

temos o pronome esse como sujeito do verbo existe e o 09. (EEAR) Como poderia haver relaes entre o predador e o
dio como objeto direto desse mesmo verbo. consumidor, se no houvesse o comrcio?
c) ( ) O sujeito da frase existe apenas o medo classificado a) sujeito composto c) sujeitos inexistentes
como oculto ( ele ) e apenas o medo o objeto direto do b) sujeito simples d) sujeitos compostos
verbo existir.
d) ( ) Em depois morreremos de medo o sujeito a palavra 10. (EEAR) O tique-taque do relgio diminui, os grilos come-
depois ( simples) que exerce a funo de um substantivo, e am a cantar. E madalena surge no lado de l da mesa. Digo
de medo o objeto indireto do verbo morrer. baixinho: - Madalena! H no texto:
e) ( )Flores amarelas e medrosas nascero sobre nossos a) trs sujeitos
tmulos a ordem direta da orao sobre nossos tmulos b) cinco sujeitos
nascero flores amarelas e medrosas que tem como sujeito c) quatro sujeitos
simples flores amarelas e medrosas. d) trs sujeitos, sendo um oculto

04. Indique o tipo de sujeito de cada orao abaixo: 11. (EEAR) Quando acabou o espetculo, cada famlia entrou
a) No choremos, amigos, a mocidade. no seu carro; as poucas que no tinham esperavam uma estiada.
A funo sinttica das palavras sublinhadas, respectivamente, :
a) sujeito objeto direto
b) Corriam por aqueles dias boatos da revoluo. b) sujeito adjunto adverbial
c) sujeito adjunto adnominal
d) adjunto adnominal adjunto adverbial
c) O homem, a fera e o inseto, sombra delas, vivem livres de
fome e fadigas. 12. (EEAR) Assinale a alternativa em que aparece orao sem
sujeito:
a) Esperanas haver sempre.
b) Comearam cedo, as aulas este ano.
d) No chores, meu filho. c) No se brinca com facas e armas de fogo.
d) Inventaram um novo pra-quedas, os homens da aeronutica.
e) A maioria das pessoas imagina que o importante, no 13. (EEAR) Em qual alternativa o sujeito se acha posposto ao
dilogo, a palavra. verbo?
a) D muitas volta o mundo.
b) E tu crs na liberdade, filho?
05. (AMAN). As granadas explodindo entre os restolhos secos do c) Tu no viste no cu um negrume?
matagal, incendiavam-nos; ouviam-se l dentro, de envolta com o d) Um casal meu amigo, convidou-me certa vez...
crepitar de queimadas sem labaredas, extintas nos brilhos da manh
clarssima, brados de clera e de dor; (...). O sujeito de ouviam-se : 14. (PUC) Nesse momento comearam a feri-lo nas mos, a
a) brados de clera e de dor. pau. Nessa frase o sujeito do verbo :
b) indeterminado. a) nas mos
c) o crepitar de queimadas. b) indeterminado
d) brilhos da manh clarssima. c) eles (determinado)
e) os sertanejos, oculto. d) inexistente ou eles: dependendo do contexto
e) n.d.a
06. (ENCE) Marque a opo que no apresenta sujeito indeter-
minado.
a) Precisa-se de funcionrios competentes.
b) Come-se bem neste restaurante.
c) Morre-se de tuberculose ainda hoje. PREDiCADO
d) Deixaram a luz do ptio acesa.
e) Vendem-se pianos reformados. Leia:
Meu cavalo minhas pernas
07. (ENCE) Assinale a alternativa em que o sujeito inexistente. Meu arreio meu assento
a) Nesta terra, faz muito calor. Meu capote minha cama
b) Divulgaram-se notcias assustadoras. Meu perigo meu sustento.
c) Necessita-se de roupas e mantimentos. (Joo Guimares Rosa)
d) Caminhamos sob um sol ardente.
e) Algum responsvel por tamanha desordem. No verso: Meu cavalo minhas pernas, qual a informao
declarada sobre meu cavalo? minhas pernas.
08. Assinale a alternativa em que o termo grifado no funciona Chamamos de predicado tudo aquilo que se informa sobre o
como sujeito. sujeito e estruturado em torno de um verbo. Ele sempre con-
a) O vento soprava forte. corda em nmero e pessoa com o sujeito. Quando um caso de
b) Algum esqueceu um chapeu na sala.. orao sem sujeito, o verbo do predicado fica na forma impessoal,
c) Vossa Excelncia governa o maior pas deste continente. 3 pessoa do singular. O ncleo do predicado pode ser um verbo
d) Fazem-se unhas francesinha. significativo, um nome ou ambos.
e) Chegaram s mos do ministro da Defesa os projetos de lei Ex.: Seu trabalho tem uma ligao muito forte com a psicanlise.
que mudam o Cdigo Brasileiro de Aeronutica. (Revista
(Revista Nova Escola, 11/00)
poca, 06/12/99)

UECEVEST 9

01 GRAMTICA.indd 9 06/02/2011 20:25:19


G R A M TICA

Tipos de predicado e) ( )Podemos considerar ( partculas expletivas / conectivos


Verbal e agentes da passiva ) exercendo a funo de objeto indireto
Nominal do verbo transitivo indireto declinava.
Verbo-nominal
02. (EFOMM) Assinale o perodo cujo predicado nominal.
Predicado verbal a) Vi-o doente.
Aquele que tem como ncleo (palavra mais importante) um b) Encontrei-o muito doente.
verbo significativo. c) Vi o doente.
Ex.: Ministro anuncia reajuste de impostos. d) O aluno foi chamado ao quadro.
Ncleo: anuncia (verbo significativo) e) O jogador, aps a falta, virou bicho.

diCA 03. (Escola Naval) Ocorre predicado verbo-nominal em:


O verbo significativo pode ser: transitivo direto (VTD), a) A tua resposta no verdadeira.
transitivo indireto (VTI), transitivo direto e indireto (VTDI) ou b) O co vadio virou a lata de lixo.
intransitivo (VI). c) Viraram moda os jogos eletrnicos.
Ex.: O tcnico comprou vrias bolas. (VTD) d) Todos permaneam em seus lugares.
O tcnico gosta de bolas novas. (VTI) e) Pensativo e triste vinha o rapaz.
O tcnico prefere melhores condies de trabalho a aumento de
salrio. (VTDI) 04. Em Sacou da arma, a funo sinttica do termo sublinhado :
O tcnico viajou. (VI) a) Objeto direto preposicionado.
b) Objeto indireto.
Predicado nominal c) Adjunto adverbial de meio.
Aquele cujo ncleo um nome (predicativo). Nesse o verbo d) Objeto direto
de ligao. Serve de elo entre o sujeito e o predicativo. e) Complemento nominal.
Ex.: Todos estavam apressados.
Estavam: verbo de ligao (VL) 05. (UNIMEP)
Ncleo: apressados (predicativo) I. Paulo est adoentado.
II. Paulo est no hospital.
Predicado verbo-nominal
Aquele que possui dois ncleos: um verbo significativo e um a) O predicado verbal em I e II
predicativo do sujeito ou do objeto. b) O predicado nominal em I e I
Ex.: O juiz julgou o ru culpado. c) O predicado verbo-nominal em I e II
Ncleos: julgou- verbo significativo d) O predicado verbal em I e nominal em II
culpado- predicativo do objeto (o ru) e) O predicado nominal em I e verbal em II

06. (FCMPA-MG) Assinale a alternativa em que aparea predi-


cado verbo- nominal:
E x E R C C i O a) A chuva permanecia calma.
b) A tempestade assustou os habitantes da vila.
01. O assassino era o escriba
c) Paulo ficou satisfeito.
Meu professor de anlise sinttica era o tipo de sujeito inexis-
d) Os meninos saram do cinema calados.
tente. Um pleonasmo, o principal predicado da sua vida, regula
e) Os alunos estavam preocupados.
como um paradigma da 1 conjugao. Entre uma orao subor-
dinada e um adjunto adverbial, ele no tem dvidas: sempre acha- 07. (TTN) Observe as duas oraes abaixo:
va um sujeito assindtico de nos torturar com um aposto. Casou I. Os fiscais ficaram preocupados com o alto ndice de
com uma regncia. Foi infeliz. Era possessivo como um pronome. sonegao fiscal.
E ela era bitransitiva. Tentou ir para os EUA. No deu. Acharam II. Houve uma sensvel queda na arrecadao do ICM em
um artigo indefinido em sua bagagem. A interjeio do bigode alguns Estados. Quanto ao predicado, elas classificam-se,
declinava partculas expletivas, conectivos e agente da passiva, o respectivamente, como:
tempo todo. Um dia, matei-o com um objeto direto na cabea.
(Paulo Leminski) a) nominal e verbo-nominal
b) verbo-nominal e verbal
Observe os itens abaixo e assinale V ou F c) nominal e verbal
a) ( ) Na primeira orao temos um verbo de ligao, um d) verbal e verbo-nominal
predicativo do objeto e um predicado verbo-nominal. e) verbal e nominal
b) ( ) Acharam um artigo indefinido em sua bagagem.,
classificamos o sujeito como indeterminado, pois no h 08. (EEAR) Assinalar a alternativa em que h predicado nominal.
indcios de um sujeito claro e explcito nessa orao. H a) A tropa continuava parada na trincheira.
tambm um verbo transitivo direto e um objeto direto b) Todo homem tem grandes sonhos na vida.
seguido de um adjunto adverbial de lugar. c) A ambio tornou-o avarento.
c) ( ) Em ... Era possessivo como um pronome. E ela d) Ouvi e conservei-me calado.
era bitransitiva., temos dois predicados nominais
respectivamente, pois existem dois verbos de ligao e dois Termos ligados ao nome
predicativos do sujeito respectivamente. Existem alguns termos que se ligam aos nomes. So eles:
d) ( ) Analisando o contexto, percebemos que ( tentou ir... / Adjunto adnominal
no deu. ) possuem verbos considerados quanto predicao Complemento nominal
verbal como intransitivos porque no necessitam de um Predicativo
complemento verbal. Aposto

10 UECEVEST

01 GRAMTICA.indd 10 06/02/2011 20:25:19


GR A M T I C A

Adjunto adnominal Depois de refletir, uma menina ergueu a mo.


o termo que se liga a um nome ou palavra substantivada (circunstncia de tempo)
para qualific-lo ou determin-lo. expresso geralmente por um Revista Nova Escola, 11/00
adjetivo, locuo adjetiva, artigo, pronome ou numeral.
Ex.:Neste Natal, estimule a criatividade de seus alunos. preciso ter em mente que o adjunto adverbial representa
(Revista Nova Escola, 11/00) uma ideia acessria mensagem. Ele no deve ser confundido
com o objeto indireto nem com o complemento nominal, que
Complemento nominal so termos integrantes da frase, indispensveis compreenso da
o termo da orao exigido como complementao de al- mensagem.
guns nomes (substantivos, adjetivos ou advrbios). Geralmente Os principais adjuntos adverbiais so:
regido de preposio. tempo: Agora, o asfalto anda em Tabatinga.
Ex.: A criana tinha necessidade de brincadeiras. Lugar: Aqui no tem ningum com esse nome.
Os turistas tinham disposio para a caminhada. modo: Acidentalmente, derrubou a bandeja de doces.
negao: Devemos amar os animais, e no maltrat-los de jei-
Predicativo to nenhum.
o termo da orao que qualifica, classifica ou expressa um Afirmao: Sim eu poderia abrir as portas.
estado do ncleo do sujeito ou do ncleo do objeto. dvida: Talvez seja um maluco fingindo ser mdico.
Ex.: Os torcedores saram do estdio alegres. intensidade: Eu j chorei bastante.
(predicativo do sujeito) meio: Iremos de avio Salvador.
Causa: O pas ficar em runas com a inflao.
AtenO Companhia: Fomos ao cinema com papai.
O verbo aqui intransitivo. Temos predicativo do sujeito por- instrumento: A criana estragou a parede com o martelo.
que temos um verbo de ligao de fcil identificao pelo contex- Finalidade: Haviam escrito um artigo novo para a edio da
to, o verbo estar. Assim temos: ... e estavam... tarde.
Ex.: Os torcedores consideraram o jogo fraco.
(predicativo do objeto) ObS.:
Os advrbios (classe de palavra invarivel) j foram estudados
Aposto no mdulo 2 de gramtica.
o termo da orao que resume, explica ou especifica um nome.
Ex.:Graas ao pai da psicanlise, Sigmund Freud, a masturbao Agente da passiva
comeou a ser entendida como um hbito saudvel em qualquer o termo da orao que se liga ao verbo para indicar o agente
idade, da infncia velhice. (Revista Nova Escola, 11/00) da ao verbal. Sempre vem precedido de preposio.
Ex.: O abaixo-assinado foi feito pelos alunos.
diCA Ateno para a identificao do agente da passiva.
O aposto geralmente vem marcado por algum tipo de pontu-
ao: vrgula, travesso, parnteses ou dois-pontos. diCA
Ex.: Algumas frutas- duas ou trs- foram escolhidas para a exposio. O agente da passiva s existe quando a orao estiver na voz
passiva analtica.
Termo independente
Objeto direto
Vocativo o termo da orao que completa o verbo transitivo direto
o nico termo isolado dentro da orao, pois no se liga ao (VTD) sem mediao de uma preposio.
verbo nem ao nome. No faz parte do sujeito nem do predicado. Ex.:A prtica estimula a reflexo filosfica independentemente
A funo do vocativo chamar ou interpelar o elemento a que se da leitura.
est dirigindo. marcado por sinal de pontuao e admite ante- Revista Nova Escola, 11/00
posio de interjeio de chamamento.
Ex.: Pai, perdoai nossos pecados. diCA
Querida, obrigado pela surpresa. Voc sabe o que um objeto direto preposiciona-
do? Como no confundi-lo com um objeto indireto?
Termos ligados ao verbo O objeto direto preposicionado completa um verbo transitivo
Existem alguns termos que se ligam aos verbos. So eles: direto (VTD) enquanto um objeto indireto completa um verbo
Adjunto adverbial transitivo indireto (VTI). Geralmente usado para solucionar
Agente da passiva casos de ambiguidade de orao ou por uma questo de estilo.
Objeto direto Ex.:Amou a seu pai com a mais plena grandeza da alma. (FEFASP)
Objeto indireto amar (VTD); a seu pai (objeto direto preposicionado)
Adjunto adverbial O objeto direto tambm pode ser pleonstico, ou seja, quan-
o termo da orao que se liga ao verbo, adjetivo ou advrbio do h a repetio do objeto direto e vem sempre representado por
para indicar uma circunstncia ( tempo, lugar, modo, intensida- um pronome.
de, negao, finalidade...). Ex: O menino, no o vi.
AtenO Objeto indireto
Circunstncia uma particularidade que modifica um fato. o termo que completa um verbo transitivo indireto (VTI)
Ex.: Na escola, fala-se muito pouco sobre o que as crianas pen- com mediao de uma preposio.
sam espontaneamente. (circunstncia de lugar) Ex.: Na formatura, ele lembrou-se da faculdade.
Revista Nova Escola, 11/00

UECEVEST 11

01 GRAMTICA.indd 11 06/02/2011 20:25:20


G R A M TICA

d) predicativo do objeto direto


E x E R C C i O e) objeto direto
01. (FMU) Em Eu era, enfim, senhores, uma graa de aliena-
do., os termos da orao grifados so respectivamente, do ponto 07. (FGV) Em Motoristas, mantenham direita!, h um erro
de vista sinttico: no uso da crase, pois o termo direita :
a) adjunto adnominal, vocativo, predicativo do sujeito a) objeto direto
b) adjunto adverbial, aposto, predicativo do objeto b) adjunto adverbial de lugar
c) adjunto adverbial, vocativo, predicativo do sujeito c) objeto indireto
d) adjunto adverbial, vocativo, objeto direto d) aposto do sujeito
e) adjunto adnominal, aposto, predicativo do sujeito e) adjunto adnominal

02. (MACK) No serei o poeta de um mundo caduco.; entre 08. (FGV) Leia atentamente: O vigilante guarda-noturno e o
eles considero a enorme realidade.; No serei o cantor de uma seu valente auxiliar, nunca esmoreceram no cumprimento do de-
mulher; O tempo a minha matria. As expresses sublinha- ver. No perodo acima, a vrgula est mal colocada, pois separa:
das nos versos do texto exercem, respectivamente, as funes de: a) o sujeito e o objeto direto
a) adjunto adnominal - adjunto adverbial - complemento b) o sujeito e o predicado
nominal - predicativo do sujeito c) a orao principal e a orao subordinada
b) complemento nominal - adjunto adverbial - complemento d) o sujeito e o seu adjunto adnominal
nominal - predicativo do sujeito e) o predicado e o objeto direto
c) predicativo do sujeito - ncleo do predicado - adjunto
adnominal - ncleo do predicativo do sujeito 09. (FGV) Leia atentamente: A maior parte dos funcionrios
d) predicativo do sujeito - ncleo do predicado - complemento classificados no ltimo concurso, optou pelo regime de tempo
nominal - predicativo do sujeito integral. Na frase acima, h um erro de pontuao, pois a vrgula
e) complemento nominal - adjunto adverbial - adjunto est separando de modo incorreto:
adnominal - ncleo do predicativo do sujeito a) o sujeito e o predicado
b) o aposto e o objeto direto
03. (PUC) No sintagma verbal: ... foi espantar as moscas1 do c) o adjunto adnominal e o predicativo do sujeito
rosto2 do anjinho3., temos trs sintagmas nominais que funcio- d) o sujeito e o predicativo do objeto direto
nam respectivamente como: e) o objeto indireto e o complemento da agente da passiva.
a) objeto direto, objeto indireto, adjunto adnominal do objeto
indireto 10. (FGV) Leia atentamente: O funcionrio referiu o incidente
b) objeto direto, adjunto adverbial de lugar, complemento nominal a Diretoria. Na frase acima, o termo a deve levar crase, pois
c) objeto indireto, complemento nominal, adjunto adnominal diretoria tem funo de:
do complemento nominal a) adjunto adverbial de finalidade
d) objeto indireto, objeto indireto, complemento nominal b) objeto direto
e) objeto direto, adjunto adverbial de lugar, adjunto adnominal c) sujeito
do adjunto adverbial d) adjunto adnominal preposicionado
e) objeto indireto
04. (FUVEST) No texto: Acho-me tranquilo - sem desejos, sem
esperanas. No me preocupa o futuro, os termos destacados 11. (FGV) Leia com ateno: Infelizmente, vocs enviaram uma
so, respectivamente: carta ao diretor sem assinatura. Na frase acima, h ambiguidade,
a) predicativo, objeto direto, sujeito pois a expresso sublinhada pode ser entendida como adjunto
b) predicativo, sujeito, objeto direto adnominal:
c) adjunto adnominal, objeto direto, objeto indireto a) do sujeito ou do objeto direto
d) predicativo, objeto direto, objeto indireto b) do adjunto adverbial de modo ou do objeto direto
e) adjunto adnominal, objeto indireto, objeto direto c) do objeto direto ou do aposto
d) do objeto direto ou do objeto indireto
05. (FUVEST) No mar, tanta tormenta e tanto dano, / Tantas e) do sujeito ou do predicativo do objeto direto
vezes a morte apercebida; / Na terra, tanta guerra, tanto engano,
/ Tanta necessidade aborrecida! / Onde pode acolher-se um fraco 12. (PUCC) No revelou o que descobrira a ningum. Assinale
humano, / Onde ter segura a curta vida, / Que no se arme e se a alternativa em que se analisa a classe gramatical e a funo sin-
indigne o Cu sereno / Contra um bicho da terra to pequeno? ttica das palavras destacadas, respeitando a ordem em que elas
Na orao Onde ter segura a curta vida...: ocorrem:
a) o adjetivo segura predicativo do objeto vida a) artigo, adjunto adnominal, conjuno integrante, conectivo
b) o adjetivo curta adjunto adnominal do sujeito vida b) pronome demonstrativo, sujeito, conjuno integrante,
c) os dois adjetivos - segura e curta - so adjuntos do conectivo
substantivo vida c) artigo, adjunto adnominal, pronome relativo, sujeito
d) o adjetivo segura est empregado com valor de adjunto d) pronome demonstrativo, objeto direto, pronome relativo,
adverbial objeto direto
e) os adjetivos - segura e vida - so predicativos do sujeito vida e) artigo, adjunto adnominal, pronome relativo, objeto direto

06. (FGV) Aponte a correta anlise do termo destacado: Ao fun- 13. (UF-PR) Na orao O alvo foi atingido por uma bomba for-
do, as pedrinhas claras pareciam tesouros abandonados. midvel, a locuo por uma bomba formidvel tem a funo de:
a) predicativo do sujeito a) objeto direto d) complemento nominal
b) complemento nominal b) agente da passiva e) adjunto adnominal
c) adjunto adnominal c) adjunto adverbial

12 UECEVEST

01 GRAMTICA.indd 12 06/02/2011 20:25:21


GR A M T I C A

14. (UNIMEP) Em ... as empregadas das casas saem apressa- PERODO COmPOSTO
das, de latas e garrafas na mo, para a pequena fila do leite, os
termos destacados so, respectivamente: aquele formado por mais de uma orao.
a) adjunto adverbial de modo e adjunto adverbial de matria Orao absoluta: aquela que constitui um perodo simples.
b) predicativo do sujeito e adjunto adnominal Ex.: Os filhos so um subproduto do amor.
c) adjunto adnominal e complemento nominal
d) adjunto adverbial de modo e adjunto adnominal Orao coordenada: aquela que se junta a outra, mantendo
e) predicativo do sujeito e complemento nominal independncia do ponto de vista sinttico.
Ex.: O dono do armazm comprou a mercadoria /e a vendeu na
15. (UNESP) De resto no bem uma greve, um lock-out, gre- mesma semana.
ve dos patres, que suspenderam o trabalho noturno.; Muitas
vezes lhe acontecera bater campainha de uma casa e ser aten- Orao subordinada: aquela que se liga a outra, mantendo
dido por uma empregada ou por outra pessoa qualquer; E, s uma dependncia sinttica, ou seja, ela exerce uma funo
vezes, me julgava importante. Assinalar a alternativa em que os sinttica com relao orao principal.
termos em destaque aparecem corretamente analisados quanto Ex.: O dono da imobiliria achava / que daria escndalo.
funo sinttica:
a) predicativo, sujeito, objeto direto Orao principal: aquela da qual depende a orao
b) aposto, agente da passiva, predicativo subordinada.
c) objeto direto, objeto indireto, adjunto adverbial Ex.: A areia do cho no permitia que me segurasse em nada.
d) complemento nominal, adjunto adverbial, aposto
e) vocativo, adjunto adnominal, predicativo O perodo composto pode ser:

16. (UM-SP) Apesar de vistosa, a construo acelerada daquele Perodo composto por coordenao
edifcio deixou-nos insatisfeitos novamente. Os termos em des-
taque no perodo so, respectivamente: Leia:
a) adjunto adnominal, objeto indireto, adjunto adverbial Ou isto ou aquilo?
b) complemento nominal, objeto direto, adjunto adverbial Ou se tem chuva e no se tem sol,
c) adjunto adnominal, objeto direto, predicativo do objeto Ou se tem sol e no se tem chuva!
d) complemento nominal, objeto direto, predicativo do objeto
e) adjunto adnominal, objeto indireto, adjunto adnominal Ou se cala a luva e no se pe o anel,
Ou se pe o anel e no se cala a luva!
17. (F. TIBIRIA-SP) Na orao Jos de Alencar, romancista Ou guarda o dinheiro e no se compra doce
brasileiro, nasceu no Cear, o termo destacado exerce a funo Ou compro o doce e gasto o dinheiro,
sinttica de:
a) aposto Ou isto ou aquilo, ou isto ou aquilo...
b) vocativo E vivo escolhendo o dia inteiro!
c) predicativo do objeto (Ceclia Meireles)
d) complemento nominal
e) n.d.a No perodo: ou se tem chuva e no se tem sol, temos duas
oraes coordenadas que se ligam pelo sentido, mas no existe
dependncia sinttica entre elas.
As oraes coordenadas de subdividem em:
Assindticas No so introduzidas por conjuno.
18. (2003) No ano passado, o governo promoveu uma campanha Ex.: Trabalhou, sempre ir trabalhar.
a fim de reduzir os ndices de violncia. Noticiando o fato, um
jornal publicou a seguinte manchete: Sindticas So introduzidas por conjuno. Esse tipo de
CAMPANHA CONTRA A VIOLNCIA DO GOVERNO orao se subdivide em:
DO ESTADO ENTRA EM NOVA FASE
Aditiva: ideia de adio, acrscimo. Principais conjunes
A manchete tem um duplo sentido, e isso dificulta o entendi- usadas: e, nem, (no somente) ...como tambm.
mento. Considerando o objetivo da notcia, esse problema pode- Ex.: O professor no somente elaborou exerccios como tambm
ria ter sido evitado com a seguinte redao: uma extensa prova.
a) Campanha contra o governo do Estado e a violncia entram
em nova fase. Adversativa: ideia de contraste, oposio. Principais conjun-
b) A violncia do governo do Estado entra em nova fase de es usadas: mas, contudo, entretanto, porm...
Campanha. Ex.: O professor elaborou um exerccio simples, mas a prova foi
c) Campanha contra o governo do Estado entra em nova fase bastante complexa.
de violncia.
d) A violncia da campanha do governo do Estado entra em Alternativa: ideia de alternativa, excluso. Principais conjun-
nova fase. es usadas: quer...quer, ora...ora, ou...ou.
e) Campanha do governo do Estado contra a violncia entra Ex.: Ou o professor elabora o exerccio/ ou desiste de aplicar a prova.
em nova fase.
Conclusiva: ideia de deduo, concluso. Principais conjun-
es usadas: portanto, pois, logo...
Ex.: O professor no elaborou a prova, logo no poder aplic-la
na data planejada.

UECEVEST 13

01 GRAMTICA.indd 13 06/02/2011 20:25:21


G R A M TICA

Explicativa: ideia de explicao, motivo. Principais conjun- As oraes subordinadas substantivas podem ser:
es usadas: pois, porque.
Ex.: O professor no elaborou a prova, porque ficou doente. Oraes subordinadas substantivas objetivas diretas
Exercem a funo de objeto direto do verbo da orao principal.
ATENO Ex.: Paulo Jos observa que o anti-herosmo uma caracterstica
As conjunes coordenativas foram estudadas no mdulo 2 forte dos personagens da cultura latino-americana. (em. 01.10.00)
de gramtica.
Oraes subordinadas substantivas objetivas indiretas
DICA Exercem a funo de objeto indireto do verbo da orao
A conjuno pois pode introduzir oraes conclusivas (pois, principal.
aps o verbo) ou explicativas(pois, antes do verbo). Ex.: A nova mquina necessitava de que os funcionrios supervi-
sionassem mais o trabalho.
Perodo composto por subordinao
Oraes subordinadas substantivas predicativas
Leia: Exercem a funo de predicativo do sujeito da orao principal.
Ex.: Meu consolo era que o trabalho estava no fim.
Chega de saudade
Vai, minha tristeza Oraes subordinadas substantivas subjetivas
E diz a ela Exercem a funo de sujeito da orao principal.
Que sem ela no pode ser Ex.: difcil que ele venha.
Diz-lhe uma prece
Que ela regresse DICA
Porque eu no posso mais sofrer O verbo da orao principal sempre estar na 3 pessoa do
singular quando a orao subordinada for subjetiva.
Chega de saudade
A realidade que sem ela Oraes subordinadas substantivas completivas nominais
No h paz, no h beleza Exercem a funo de complemento nominal da orao
s tristeza e a melancolia principal.
Que no sai de mim, no sai de mim Ex.: Sua falha trgica a dificuldade de ser malevel em relao
No sai (...) realidade.
(Antnio Carlos Jobim e Vincius de Moraes)
Oraes subordinadas substantivas apositivas
No perodo: E diz a ela / que sem ela no pode ser, temos Exercem a funo de aposto de algum nome da orao
duas oraes. A 1: E diz a ela a principal. A 2: que sem principal.
ela no pode ser, a subordinada, pois completa o sentido da Ex.:H nas escolas uma norma: que os alunos so respeitados.
principal.
No perodo subordinado, existem pelo menos uma orao DICA
principal e uma subordinada.A orao principal sempre in- A orao apositiva sempre estar pontuada, ou entre vrgulas
completa, ou seja, alguma funo sinttica est faltando. As ora- ou depois de dois pontos.
es subordinadas desempenham a funo sinttica que falta na
principal: objeto direto, indireto, sujeito, predicativo, comple- Oraes subordinadas adjetivas
mento nominal...
Ex.: O rapaz gostava / de que todos olhassem para ele. Podem ser:

Orao principal: O rapaz gostava Restritivas


Orao subordinada: de que todos olhassem para ele. Exercem a funo de adjunto adnominal da orao princi-
pal, restringem o nome ao qual se referem, no so separadas por
A orao principal est incompleta, falta objeto indireto para vrgulas.
o verbo gostar, o orao subordinada desempenha a funo de Ex.: O trabalho que realizei ontem foi produtivo.
objeto indireto da principal.
As oraes subordinadas se subdividem em: Substantivas, Explicativas
Adjetivas e Adverbiais Exercem a funo de aposto da orao principal, explicam
o nome ao qual se referem, so sempre separadas por vrgulas.
Oraes subordinadas substantivas Ex.: O computador, que um meio rpido de comunicao, est
As oraes subordinadas substantivas exercem funes espec- conquistando todas as famlias.
ficas do substantivo: sujeito, objeto, predicativo...
DICA
DICA As oraes subordinadas adjetivas sempre sero introduzidas
As oraes subordinadas substantivas desenvolvidas so intro- por pronomes relativos: que, o qual, a qual, as quais, quem, onde,
duzidas pelas conjunes integrantes se ou que e possuem verbos cujo(s), cuja(s), quanto etc.
conjugados. As oraes subordinadas substantivas reduzidas no
so introduzidas por conjunes e possuem verbos nas formas Perodo composto subordinado adverbial
nominais (particpio, gerndio ou infinitivo). Oraes subordinadas adverbiais:
Ex.: possvel que eu fracasse. ( orao desenvolvida)
possvel fracassar. ( orao reduzida de infinitivo) Causais
Expressam a causa da consequncia expressa na orao principal.

14 UECEVEST

01 GRAMTICA.indd 14 06/02/2011 20:25:22


GR A M T I C A

Principais conjunes: porque, pois que, j que, porquanto... Adjetiva


Ex.: Chegou atrasado ao encontro, porque estava em uma reunio. Encontrei as meninas danando no meio da rua.

Consecutivas Adverbial
Expressam a consequncia, o resultado da causa expressa na Causal
orao principal. no vendo o posto, colidiu com ele.
Principais conjunes: (to)...que, (tanto)...que, (tal)...que, estando com medo do diretor, pediu demisso do cargo.
de modo que...
Ex.: A reunio atrasou tanto que ele se atrasou para o encontro. Concessiva
Sendo rico, mentiu que era pobre. (D. Trevisan)
Proporcionais
Expressam proporo. Condicional
Principais conjunes: media que, a proporo que, ao pas- havendo demanda, haver produo maior. (Viso)
so que...
Ex.: medida que a reunio avanava, ele se atrasava para o en- modal
contro. Por aqui passou Garrincha, inventando dribles e alegrias.
(A. Nogueira)
Temporais
Expressam tempo.
Oraes subordinadas reduzidas de particpio
Principais conjunes: quando, enquanto, logo que, at que...
Podem ser substantivas, adjetivas ou adverbiais:
Ex.: Logo que ele chegou, arrumou os trabalhos.
Adjetiva
Finais
O bichinho subia pela roupa estendida no varal
Expressam finalidade, objetivo.
Principais conjunes: para que, a fim de que, porque (=para
Adverbial
que)...
temporal
Ex.: Professores, tenham mais argumentos para pedir aumento
acabada a aula, fomos ao clube.
salarial.
Causal
Condicionais
amargurado, queria suicidar-se.
Expressam condio, obstculo.
Principais conjunes: se, caso, salvo, desde que, a menos que...
Concessiva
Ex.: Se ele partir, o projeto ser cancelado.
Advertido do perigo, continuava lutando.
Comparativas
Condicional
Expressam comparao.
aceitas as condies do contrato, estaramos fracassados.
Principais conjunes: que/ do que, como, assim como...
Ex.: Sua famlia to importante quanto seu trabalho.
Oraes subordinadas reduzidas de infinitivo
Concessivas
Podem ser substantivas, adjetivas ou adverbiais:
Expressam uma concesso.
Subjetiva
Principais conjunes: embora, ainda que, posto que, (por
Era difcil andar.
mais) que...
Era-lhe to enfadonho escrever cartas compridas. (M. Assis)
Ex.: Mesmo que trabalhe muito, no ser recompensada.

Conformativas
Objetiva direta
Expressa um acordo, uma conformidade. Resolveu no mostrar o convite a ningum. (R. Queiroz)
Principais conjunes: conforme, como, consoante, segundo.
Ex.: Segundo havamos combinado, o viagem ser cancelada. Objetiva indireta
Ningum pensa em cavalgar numa guia. (Idem)
AtenO
As conjunes subordinativas foram estudadas no mdulo 2 Completiva nominal
de gramtica. Sentiu vontade de vomitar e de morrer. (A. Prado)

SAibA mAiS Predicativa


As oraes desenvolvidas so aquelas nas quais o verbo Vai, teu ofcio alegrar o homem. (X. Marques)
estconjugado em algum tempo: presente, pretrito e futuro.
Ex.: Esperamos que passe de ano. Apositiva
As oraes reduzidas so aquelas nas quais uma orao subor- Prometi-lhes apenas isto: esper-los at s dez horas.
dinada se apresenta sem conjuno ou pronome relativo e com
Adjetiva
o verbo no infinitivo, no particpio ou no gerndio, dizemos
que ela uma orao reduzida, acrescentando-lhe o nome de
Adverbial
infinitivo, de particpio ou de gerndio.
Causal
Morreu de tanto esperar.
Oraes subordinadas reduzidas de gerndio por serem apressados, fizeram um pssimo trabalho.
Podem ser adjetivas ou adverbiais:

UECEVEST 15

01 GRAMTICA.indd 15 06/02/2011 20:25:23


G R A M TICA

Concessiva III. O fiscal deu o sinal. Os candidatos entregaram a prova.


apesar de sentir medo, no fugiu. Acabara o exame.

Condicional Nota-se que existe coordenao assindtica em:


No saia sem pedir licena. a) I apenas. d) I, II, III.
Consecutiva b) II apenas. e) Nenhum deles.
O exame foi difcil a ponto de provocar revolta nos alunos. c) III apenas.

5) Final 07. (FES- SP) No perodo: Paredes ficaram tortas, animais en-
Maria Clara acordou de seu sonho para encarar a realidade. louqueceram e as plantas caram, temos:
(B. Rocha)
a) duas oraes coordenadas assindticas e uma orao
subordinada substantiva.
6) temporal b) trs subordinadas substantivas.
ao comear o sculo, ainda ramos um satlite da Frana. c) trs oraes coordenadas.
d) quatro oraes coordenadas.
(Nosso Sculo) e) uma orao principal e duas oraes subordinadas.

08. (Mack- SP) Embora todas as conjunes sejam aditivas, uma


E x E R C C i O orao apresenta ideia adversria, identifique-a:
a) No achou os documentos e nem as fotocpias.
01. Apressa-te, que tempo pouco / Leve-lhe flores, que ela b) Queria estar atento palestra e o sono chegou.
aniversaria amanh. A conjuno que presente nas duas ora- c) No s aprecio a Medicina como tambm a Odontologia.
es estabelece, respectivamente, o sentido de: d) Escutei o ru e lhe dei razo.
a) explicao e adio.
b) explicao e explicao. 09. (UFMS-RS) Identifique a alternativa que expressa a ideia
c) concluso e explicao. correta da segunda orao, considerando a conjuno que a in-
d) concluso e concluso. troduz: A torcida incentivou os jogadores; esses, contudo, no
e) explicao e concluso. conseguiram vencer.
a) proporo. d) oposio.
02. Conquanto os salrios dos brasileiros so baixos, ainda b) concluso. e) concesso.
conseguem sentir-se felizes. A conjuno em destaque pode ser c) explicao.
classificada como:
a) consecutiva. d) comparativa. 10. (Fuvest- SP) Podem acusar-me: estou com a conscincia
b) causal. e) conformativa. tranquila.
c) concessiva. Os dois pontos(:) do perodo acima poderiam ser substitu-
dos por vrgula, explicando-se o nexo entre as duas oraes pela
03. H uma ideia comparativa no item: conjuno:
a) Clia,embora fosse rica, no tinha luxo. a) portanto. d) embora.
b) Como j lhes contei aqui, de vez em quando digo que no b) e. e) pois.
vou ler mais jornal algum. c) como.
c) Os soldados respondiam medida que eram chamados.
d) E pia tal qual a caa procurada. 11. (FEI- SP) A orao destacada No viole a lei. Portanto, no
e) No podem ver brinquedo que no queiram comprar. ser julgado :
a) explicativa. d) aditiva.
04. No perodo: b) alternativa. e) adversativa.
Minha me hesitou um pouco, mas acabou cedendo, depois c) conclusiva.
que o padre Cabral, tendo consultado o bispo, voltou a dizer-lhe
que sim, que poderia ser. 12. (UFC) Existe orao coordenada sindtica explicativa em:
A expresso depois que, morfologicamente, : a) No s deves regar as plantas, mas tambm aduba-las.
a) locuo prepositiva. b) Acordamos tarde, mas ainda nos sentamos cansados.
b) advrbio de tempo. c) Calvin chorava, ora se conformava com a perda do seu
c) locuo conjuntiva. ursinho.
d) advrbio de modo. d) Mandou parar o nibus que estava se sentindo mal.
e) explicativo. e) Ele vive mentindo; logo, no nenhum crdito de nossa
parte.
05. Em: Orai porque no entreis em tentao, o sentido da
conjuno do perodo de : 13. (UECE-CE) Aponte afirmao correta quanto s coordena-
a) causa. d) explicao. das explicativas:
b) condio. e) finalidade. a) Jamais a conjuno e pode representar valor explicativo.
c) conformidade. b) Em O mdico disse que ele morreria a orao destacada
coordenada explicativa.
06. (FMSCSP) Por definio, orao coordenada que seja des- c) comum encontrarmos um verbo no imperativo na orao
provida de conectivo denominada assindtica. Observando-se coordenada assindtica relacionada coordenada sindtica
os perodos seguintes: explicativa.
I. No caa um galho, no balanava uma folha. d) Toda orao coordenada explicativa apresenta apenas o
II. O filho chegou, a filha saiu, mas a me nem notou. conectivo pois.

16 UECEVEST

01 GRAMTICA.indd 16 06/02/2011 20:25:23


GR A M T I C A

14. (UFRJ) Leia as informaes e marque o que for correto: 22. (Fac. Md Pouso Alegre MG) Assinale o item em que h
I. As oraes coordenadas sindticas so dependentes das orao subordinada adverbial condicional reduzida de particpio.
coordenadas assindticas. a) Feita a partilha, o leo tomou a palavra.
II. Uma orao coordenada sindtica pode ter valor semntico. b) Armado com tais provas, at eu o enfrentaria.
III. Uma orao coordenada pode representar o papel de orao c) A tropa, acampada s margens do Iguau, foi surpreendida.
principal num perodo composto por subordinao. d) Ernestina estava certa de ser a escolhida.
e) Transpondo o rio, seguimos viagem.
a) Apenas o item I. d) I, II, III.
b) Apenas o item II. e) I e III.
c) II e III.

15. (Univ. Est. Ponta Grossa PR) Em: possvel que comuni-
23. Leia os textos abaixo:
cassem sobre poltica, a segunda orao :
i - A situao de um trabalhador
a) Subordinada substantiva subjetiva.
Paulo Henrique de Jesus est h quatro meses desempregado.
b) Subordinada substantiva predicativa.
Com o Ensino Mdio completo, ou seja, 11 anos de estudo, ele
c) Subordinada substantiva apositiva.
perdeu a vaga que preenchia h oito anos de encarregado numa
d) Principal.
transportadora de valores, ganhando R$800,00. Desde ento, e
e) Subordinada substantiva objetiva direta.
com 50 currculos j distribudos, s encontra oferta para ganhar
16. (UECE) Em: No sei onde pegou meu p, na barriga tal- R$300,00, um salrio mnimo. Ele aceitou trabalhar por esse va-
vez..., a orao destacada classifica-se como subordinada: lor, sem carteira assinada, como garom numa casa de festas para
a) Substantiva objetiva direta. fazer frente s despesas.
b) Adjetiva restritiva. (O Globo, 20/07/2005.)
c) Substantiva predicativa.
d) Substantiva adjetiva. ii - Uma interpretao sobre o acesso ao mercado de trabalho
Atualmente, a baixa qualificao da mo-de-obra um dos
17. (Univ. Fed. Acre) Assinale a alternativa cuja orao predicativa: responsveis pelo desemprego no Brasil.
a) claro que eles no viro. A relao que se estabelece entre a situao (I) e a interpretao
b) Acontece que ela mentiu. (II) e a razo para essa relao aparece em:
c) Sabe-seque a notcia no verdadeira. a) ii explica i - Nos nveis de escolaridade mais baixos h
d) Parece que tudo mudou. dificuldade de acesso ao mercado de trabalho.
e) O certo foi que mudou. b) i refora ii - Os avanos tecnolgicos da Terceira Revoluo
Industrial garantem somente o acesso ao trabalho para
18. (UEBA) Meu pai, que havia arrancado trs dentes, no pde
aqueles de formao em nvel superior.
viajar naquele dia.
c) i desmente ii - O mundo globalizado promoveu desemprego
A orao destacada classifica-se como:
especialmente para pessoas entre 10 e 15 anos de estudo.
a) Adverbial temporal.
d) ii justifica i - O desemprego estrutural leva a excluso de
b) Substantiva predicativa.
trabalhadores com escolaridade de nvel mdio incompleto.
c) Adjetiva restritiva.
e) ii complementa i - O longo perodo de baixo crescimento
d) Substantiva apositiva.
econmico acirrou a competio, e pessoas de maior
e) Adjetiva explicativa.
escolaridade passam a aceitar funes que no correspondem
19. (Univ. Fed. Santa Maria RS) Leia, com ateno, os perodos a sua formao.
abaixo:
I. Caso haja justia social, haver paz. 24. A gentileza algo difcil de ser ensinado e vai muito alm da
II. Embora a televiso oferea imagens concretas, ela no palavra educao. Ela difcil de ser encontrada, mas fcil de ser
fornece uma reproduo fiel da realidade. identificada, e acompanha pessoas generosas e desprendidas, que
III. Como todas aquelas pessoas estavam concentradas, no se se interessam em contribuir para o bem do outro e da sociedade.
escutou um nico rudo. uma atitude desobrigada, que se manifesta nas situaes coti-
dianas e das maneiras mais prosaicas.
Assinale a alternativa que apresenta, respectivamente, as circuns- SIMURRO, S. A. B. Ser gentil ser saudvel. Disponvel em: http://www.abqv.
tncias indicadas pelas oraes destacadas: org.br. Acesso em: 22 jun. 2006 (adaptado).
a) tempo, concesso, comparao.
b) tempo, causa, concesso. No texto, menciona-se que a gentileza extrapola as regras de boa
c) condio, consequncia, comparao. educao. A argumentao construda:
d) condio, concesso, causa. a) apresenta fatos que estabelecem entre si relaes de causa e
e) concesso, causa, conformidade. de consequncia.
b) descreve condies para a ocorrncia de atitudes educadas.
20. (UFMA) A orao adjetiva na opo: c) indica a finalidade pela qual a gentileza pode ser praticada.
a) Co que late no morde. d) enumera fatos sucessivos em uma relao temporal.
b) Espere, que j estou cansado. e) mostra oposio e acrescenta ideias.
c) O pescador disse que voltaria logo.
d) bom que saibas essas coisas. 25. Os filhos de Ana eram bons, uma coisa verdadeira e suma-
renta. Cresciam, tomavam banho, exigiam para si, malcriados,
21. (UECE) Em: Ao me deitar, eu tinha posto uma caixa de instantes cada vez mais completos. A cozinha era enfim espaosa,
fsforos num tamborete... a orao destacada reduzida: o fogo enguiado dava estouros. O calor era forte no aparta-
a) causal c) temporal mento que estavam aos poucos pagando. mas o vento batendo
b) final d) concessiva. nas cortinas que ela mesma cortara lembrava-lhe que se quisesse

UECEVEST 17

01 GRAMTICA.indd 17 06/02/2011 20:25:24


G R A M TICA

podia parar e enxugar a testa, olhando o calmo horizonte. Como 02. Leia o perodo e indique a alternativa que classifica correta-
um lavrador. Ela plantara as sementes que tinha na mo, no mente suas oraes.
outras, mas essas apenas. Era assim que falava, a princpio, para excitar o entusiasmo,
LISPECTOR, C. Laos de famlia. Rio de Janeiro: Rocco, 1998. espertar os indiferentes, congregar, em suma as multides ao p
de si. (Machado de Assis)
A autora emprega por duas vezes o conectivo mas no fragmento apre- a) orao principal, orao subordinada substantiva objetiva
sentado. Observando aspectos da organizao, estruturao e funcio- direta, orao subordinada adverbial consecutiva, orao
nalidade dos elementos que articulam o texto, o conectivo mas: coordenada assindtica, orao coordenada sindtica
a) expressa o mesmo contedo nas duas situaes em que conclusiva.
aparece no texto. b) orao principal, orao subordinada substantiva subjetiva,
b) quebra a fluidez do texto e prejudica a compreenso, se orao subordinada adverbial final, orao coordenada
usado no incio da frase. assindtica, orao subordinada sindtica conclusiva.
c) ocupa posio fixa, sendo inadequado seu uso na abertura da c) orao subordinada substantiva objetiva direta, orao
frase. principal, orao subordinada adverbial final, orao
d) contm uma ideia de sequncia temporal que direciona a subordinada adverbial final, orao coordenada sindtica
concluso do leitor. conclusiva.
e) assume funes discursivas distintas nos dois contextos de d) orao principal, orao subordinada substantiva subjetiva,
uso. orao subordinada adverbial final, orao subordinada
adverbial final, orao subordinada adverbial final.
e) orao principal, orao subordinada adverbial final, orao
subordinada substantiva objetiva, orao coordenada
Exerccios Complementares sindtica conclusiva, orao coordenada sindtica conclusiva.

Texto 03. Leia o texto abaixo e depois responda as questes referentes


O homem velho a ele.
O homem velho deixa vida e morte para trs Necrolgio dos desiludidos do amor
Cabea a prumo, segue rumo e nunca, nunca mais Os desiludidos do amor
O grande espelho que o mundo ousaria refletir os sinais. esto desfechando tiros no peito.
O homem velho o rei dos animais Do meu quarto ouo a fuzilaria.
A solido agora slida, uma pedra ao sol As amadas torcem-se de gozo.
As linhas do destino nas mos a mo apagou Oh quanta matria para os jornais.
Ele j tem a alma saturada de poesia, soul e rockn roll (.....)
As coisas migram e ele serve de farol. (Carlos Drummond de Andrade, Brejo das Almas)
A carne, a arte arde, a tarde cai
No abismo das esquinas a) Quantas frases h no texto?
A brisa leve traz o olor fugaz
Do sexo das meninas.
Luz fria, seus cabelos tm tristeza de non b) Aponte uma frase sem verbo.
Belezas, dores e alegrias passam sem um som
Eu vejo o homem velho rindo numa curva do caminho de
Hebron c) Aponte uma orao com locuo verbal.
E a seu olhar tudo que cor muda de tom.
Os filhos, filmes, livros, ditos como um vendaval
Espalham-no alm da iluso do seu ser pessoal
Mas ele di e brilha nico, indivduo, maravilha sem igual d) Na orao: Os desiludidos do amor esto desfechando tiros
J tem coragem de saber que imortal. no peito,aponte o sujeito e diga qual o seu ncleo.
(Caetano Veloso)
e) Na orao: Do meu quarto ouo a fuzilaria , aponte e classi-
01. Observe os itens abaixo e d a soma das afirmativas corretas. fique o sujeito e diga que tipo de predicado ocorre.
01. Em O homem velho deixa vida e morte para trs, as partes
grifadas exercem a funo sinttica de predicativo do sujeito.
04. Em As linhas do destino nas mos a mo apagou, temos
como sujeito a palavra linhas que, se a frase estivesse em 04. Nos perodos a seguir, as oraes em negrito so todas subor-
ordem direta seria: A linha do destino apagou a mo nas dinadas adverbiais. Classifique-as
mos. de acordo com a ideia que transmitem.
08. Nas passagens Ele j tem a alma saturada de poesia... e a) Como estava com as mos ocupadas, o rapaz no pde
O homem o rei dos animais. , temos um predicado verbal ajud-la.
com verbo transitivo direto e um predicado nominal com b) medida que o tempo passava, mais ficvamos impacientes.
verbo de ligao. c) O elevador enguiou quando estvamos no terceiro andar.
16. Migram, arde, cai, passam, rindo, di e brilha podem ser d) Se cobrarem gio, no trocaremos o carro.
classificados como verbos intransitivos dentro do contexto. e) O documento foi entregue ao presidente do jri para que
32. No trecho Os filhos, filmes, livros, ditos como um vendaval todos comprovassem a sua autenticidade.
/ Espalham-no alm da iluso do seu ser pessoal, temos o pro- f ) medida que o tempo passa, as nossas iluses desaparecem.
nome oblquo no exercendo a funo sinttica de objeto direto g) Trabalho como um escravo.
que se refere passagem ao homem velho. h) Foi to enfadonha a palestra que muitos cochilavam.

18 UECEVEST

01 GRAMTICA.indd 18 06/02/2011 20:25:24


GR A M T I C A

05. Classifique as oraes destacadas em negrito dos perodos REFERnCiAS BiBliOGRFiCAS


abaixo:
a) Deus, que nosso pai, onipotente. BECHARA, Evanildo Gramtica escolar da Lngua
b) No sa de casa; logo, no fui ao jogo. Portuguesa 1 ed. 4 reimp. Rio de Janeiro: Lucena, 2004.
c) No vi o carro que bateu no poste.
d) Saia daqui antes que o Rui chegue. CEREJA, Willian Roberto & MAGALHES, Thereza Cochar.
e) Tirei o palet e pendurei-o no cabide. Gramtica Reflexiva: texto semntica e interao So Paulo:
Atual, 1999.

COMISSO COORDENADORA DO VESTIBULAR(CCV)


G A B A R i T O www.ufc.br.
Anlise sinttica ou sintaxe COMISSO EXECUTIVA DO VESTIBULAR(CEV) www.
01. * 02. e 03. * 04. * 05. a uece.br.
06. e 07. a 08. e 09. c 10. c
CUNHA, Celso & SINTRA, Lus. F. Lindley. nova Gramtica
11. a 12. a 13. a 14. c
do Portugus Contemporneo 3 ed. Rio de Janeiro:
01. Um esparso tilintar de chocalhos e guisos. Sujeito Simples Lexikon Informtica, 2007.
03. a) v, b) f, c) f, d) f, e) v
04. a) sujeito oculto (ns) d) sujeito oculto (tu) DIONSIO, A. P. et alii. O livro didtico de portugus. Rio
b) sujeito simples e) sujeito simples de Janeiro: Ed. Lucerna, 2001.
c) sujeito composto
FARACO & MOURA. Gramtica 18 ed. tica. So Paulo,
Predicado 1999.
01. * 02. e 03. e 04. a 05. e
06. d 07. c 08. a 01. a)f, b)v, c)v, d)f, e)f FERREIRA, Mauro. Aprender e praticar gramtica: teoria,
sntese das unidades, atividades prticas, exerccios de
Termos ligados ao nome e ao verbo vestibulares: 2 grau So Paulo, FTD, 1992.
01. c 02. b 03. e 04. a 05. a 06. a MARCUSCHI, Luiz Antonio. Concepo de lngua falada
07. a 08. b 09. a 10. e 11. d 12. d nos manuais de portugus de 1 e 2 graus: uma viso
13. b 14. b 15. b 16. d 17. a crtica. Trabalhos em Lingstica Aplicada, 1997, 30: 39-79
Perodo composto _________. A gramtica e o ensino de lngua no contexto da
01. b 02. c 03. d 04. c 05. e investigao lingstica. In:
06. d 07. c 08. b 09. d 10. e
MEC - Ministrio da Educao e Cultura INEP Instituto
11. c 12. d 13. c 14. c 15. a
Nacional de Estudos e Pesquisas Educacionais Ansio Teixeira
16. a 17. e 18. e 19. d 20. a Matrizes de Referncia para o ENEM
21. c 22. b 23. e 24. e 25. e
MATTOS E SILVA, Rosa V. Contradies no ensino de
Exerccios Complementares portugus. So Paulo: Contexto, 1995.
01. Soma: 01+08+32 = 41
02. d MIRA MATEUS, M.H. et alii. Gramtica da lngua
portuguesa. 5a ed. Rev. Aum. Lisboa: Caminho, 2003
03.a) quatro
b) Oh quanta matria para os jornais. MONTEIRO, Jos Lemos morfologia portuguesa. 4 ed.
c) Os desiludidos do amor esto desfechando tiros no peito. Pontes, 2002
d) Sujeito: Os desiludidos do amor. Ncleo: desiludidos.
e) Sujeito Oculto(Eu). Predicado: Verbal PERINI, Mrio. A Gramtica descritiva do portugus. 4 ed.
04. a) Orao S. Adv. Causal 6 reimp. tica, 2003.
b) Orao S. Proporcional
c) Orao S. Adv. Temporal SARMENTO, Leila Lauar. Gramtica em textos 2 ed. So
d) Orao S. Adv. Condicional Paulo: Moderna, 2005.
e) Orao S. Adv. Final
f ) Orao S. Adv. Proporcional SILVA, Thas Cristfaro Fontica e fonologia do portugus.
g) Orao S. Adv. Comparativa 7 ed. Contexto, So Paulo, 2003. www.brasilescola.com/
h) Orao S. Adv. Consecutiva novoacordoortografico.
05. a) Orao S. Adj. Explicativa
b) Orao C. S. Conclusiva TRAVAGLIA, Luiz Carlos. Gramtica e interao: uma
c) Orao S. Adj. Restritiva proposta para o ensino de gramtica no 1 e 2 graus. So
d) Orao S. Adv. Temporal Paulo: Cortez, 1996.
e) Orao C. S. Aditiva
www.brasilescola.com/novaortografia

UECEVEST 19

01 GRAMTICA.indd 19 06/02/2011 20:25:25


01 GRAMTICA.indd 20 06/02/2011 20:25:25
P R - V E S T I B U l A R

LITERATURA

Apostilas UECEVEST mod3.indb 21 06/02/2011 09:57:18


Caro(a) Aluno(a),

O mdulo que voc tem em mos possui contedos relacio- Objeto de conhecimento
nados s Matrizes de Referncia para a rea de Linguagem, C-
digos e suas Tecnologias, do Exame Nacional do Ensino Mdio Produo e recepo de textos artsticos: interpre-
(ENEM). Abaixo, h a indicao das competncias da rea e de tao e representao do mundo para o fortalecimento
suas habilidades que, neste mdulo, so contempladas, e, em se- dos processos de identidade e cidadania - Artes Visuais:
guida, dos objetos de conhecimento associados s Matrizes. estrutura morfolgica, sinttica, o contexto da obra arts-
tica, o contexto da comunidade. Teatro: estrutura morfo-
Competncia de rea 3 Compreender e usar a linguagem lgica, sinttica, o contexto da obra artstica, o contexto
corporal como relevante para a prpria vida, integradora so- da comunidade, as fontes de criao. Msica: estrutura
cial e formadora da identidade. morfolgica, sinttica, o contexto da obra artstica, o con-
H9 Reconhecer as manifestaes corporais de movimento texto da comunidade, as fontes de criao. Dana: estru-
como originrias de necessidades cotidianas de um grupo social. tura morfolgica, sinttica, o contexto da obra artstica,
H11 Reconhecer a linguagem corporal como meio de interao o contexto da comunidade, as fontes de criao. Conte-
social, considerando os limites de desempenho e as alternativas dos estruturantes das linguagens artsticas (Artes Visuais,
de adaptao para diferentes indivduos. Dana, Msica, Teatro), elaborados a partir de suas es-
truturas morfolgicas e sintticas; incluso, diversidade e
Competncia de rea 4 Compreender a arte como saber cul- multiculturalidade: a valorizao da pluralidade expressa-
tural e esttico gerador de significao e integrador da organi- da nas produes estticas e artsticas das minorias sociais
zao do mundo e da prpria identidade. e dos portadores de necessidades especiais educacionais.
H12 Reconhecer diferentes funes da arte, do trabalho da pro-
duo dos artistas em seus meios culturais. Estudo do texto literrio: relaes entre produo
H13 Analisar as diversas produes artsticas como meio de literria e processo social, concepes artsticas, pro-
explicar diferentes culturas, padres de beleza e preconceitos. cedimentos de construo e recepo de textos - pro-
duo literria e processo social; processos de formao
Competncia de rea 5 Analisar, interpretar e aplicar recur- literria e de formao nacional; produo de textos lite-
sos expressivos das linguagens, relacionando textos com seus rrios, sua recepo e a constituio do patrimnio lite-
contextos, mediante a natureza, funo, organizao, estrutu- rrio nacional; relaes entre a dialtica cosmopolitismo/
ra das manifestaes, de acordo com as condies de produ- localismo e a produo literria nacional; elementos de
o e recepo. continuidade e ruptura entre os diversos momentos da
H16 Relacionar informaes sobre concepes artsticas e pro- literatura brasileira; associaes entre concepes artsti-
cedimentos de construo do texto literrio. cas e procedimentos de construo do texto literrio em
seus gneros (pico/narrativo, lrico e dramtico) e formas
Competncia de rea 6 Compreender e usar os sistemas sim- diversas.; articulaes entre os recursos expressivos e es-
blicos das diferentes linguagens como meios de organizao truturais do texto literrio e o processo social relacionado
cognitiva da realidade pela constituio de significados, ex- ao momento de sua produo; representao literria: na-
presso, comunicao e informao. tureza, funo, organizao e estrutura do texto literrio;
H18 Identificar os elementos que concorrem para a progresso relaes entre literatura, outras artes e outros saberes.
temtica e para a organizao e estruturao de textos de diferen-
tes gneros e tipos.

Apostilas UECEVEST mod3.indb 22 06/02/2011 09:57:18


LIT E R AT U R A

PR-mODERnISmO ironiza tanto os escritores importantes que utilizavam uma


linguagem pomposa, quanto os leitores que se deixavam im-
pressionar: Quanto mais incompreensvel ela [a linguagem],
mais admirado o escritor que a escreve, por todos que no lhe
entenderam o escrito (Os bruzundangas).
Denncia da realidade brasileira - Nega-se o Brasil literrio
herdado do Romantismo e do Parnasianismo, o Brasil no-ofi-
cial do serto nordestino, dos caboclos interioranos, dos subr-
bios, o grande tema do Pr-Modernismo.
Regionalismo - Monta-se um vasto painel brasileiro: o Norte
e o Nordeste com Euclides da Cunha; o Vale do Paraba e o
interior paulista com Monteiro Lobato; o Esprito Santo com
Graa Aranha; o subrbio carioca com Lima Barreto.
Tipos humanos marginalizados - O sertanejo nordestino, o
caipira, os funcionrios pblicos, os mulatos.
Ligao com fatos polticos, econmicos e sociais contem-
O que se convencionou chamar de Pr-Modernismo, no porneos. - Diminui a distncia entre a realidade e a fico. So
Brasil, no constitui uma escola literria, ou seja, no temos exemplos: Triste fim de Policarpo Quaresma, de Lima Barreto
um grupo de autores afinados em torno de um mesmo iderio, (retrata o governo de Floriano e a Revolta da Armada), Os ser-
seguindo determinadas caractersticas. Na realidade, Pr-Moder- tes, de Euclides da Cunha (um relato da Guerra de Canudos),
nismo um termo genrico que designa a produo literria de Cidades mortas, de Monteiro Lobato (mostra a passagem do
alguns autores que, no sendo ainda modernos, j promovem caf pelo Vale do Paraba Paulista), e Cana, de Graa Aranha
rupturas com o passado. (um documento sobre a imigrao alem no Esprito Santo).
Esse perodo tem incio com a publicao de Os Sertes em 1902
e se estende at 1922, com a realizao da Semana de Arte Moderna. Euclides da Cunha
Euclides da Cunha nasceu no
momento Histrico Rio de Janeiro, em 20 de janeiro de
Enquanto a Europa se preparava para a I Guerra Mundial, 1866 e morreu neste mesmo esta-
o Brasil comeava a viver, a partir de 1894, um novo perodo do, em 1909, aos 43 anos de idade.
de sua histria republicana: com a posse do paulista Prudente Era engenheiro, porm seu talento
de Morais, primeiro presidente civil, iniciou-se a Repblica do literrio no passava desapercebi-
caf-com-leite dos grandes proprietrios rurais, que substitua a do, e, logo recebeu um convite para
Repblica da espada (governos dos marechais Deodoro e Flo- ser correspondente do jornal O
riano). Essa poca foi marcada pelo auge da economia cafeeira Estado de S.Paulo, este fato ocor-
no sudeste, pela entrada de grandes levas de imigrantes no pas, reu durante o perodo da Guerra de
notadamente de italianos, pelo esplendor da Amaznia, com o Canudos.
ciclo da borracha, e pelo surto de urbanizao de So Paulo. Posteriormente, escreveu sobre esta revolta no livro: Os Ser-
Mas toda essa prosperidade veio acentuar cada vez mais os fortes tes, obra que atingiu um grande sucesso. Nesta obra ele faz ainda
contrastes da realidade brasileira. Por isso, nesse perodo tambm uma anlise brilhante da psicologia do sertanejo e de seus costu-
ocorreram vrias agitaes sociais, muitas delas no abandonado mes.Escreveu tambm: Peru versus Bolvia, Contrastes e Confron-
Nordeste. No final do sculo XIX, na Bahia, ocorreu a Revolta de tos e A Margem da Histria.
Canudos, tema de Os Sertes, de Euclides da Cunha; nos primeiros Sua vida privada foi repleta de contrariedades e grandes difi-
anos do sculo XX, o Cear foi palco de conflitos que tiveram como culdades financeiras. No ano de 1909 ele prestou concurso para
figura central o Padre Ccero, o famoso Padim Cio; o serto vi- o magistrio. Apesar de ter sido aprovado ele no teve tempo de
veu o tempo do cangao, com a figura lendria de Lampio. assumir o cargo, pois, foi assassinado pouco tempo depois. Sua
Em 1904, o Rio de Janeiro assistiu a uma rpida mas intensa obra rconhecida at os dias de hoje e seus livros so lidos pelos
rebelio popular, a pretexto de lutar contra a vacinao obriga- apreciadores da literatura brasileira.
tria idealizada por Oswaldo Cruz; na realidade, tratava-se de
uma revolta contra o alto custo de vida, o desemprego e os rumos lima Barreto
da Repblica. Em 1910, houve uma outra importante rebelio, Lima Barreto nasceu a 13 de maio de 1881, no Rio de Ja-
dessa vez dos marinheiros, liderados por Joo Cndido, o Almi- neiro. Filho de uma escrava com um
rante Negro, contra o castigo corporal - a Revolta da Chibata. portugus, cursou as primeiras letras
Ao mesmo tempo, em So Paulo, as classes trabalhadoras, sob em Niteri e depois transferiu-se para
orientao anarquista, iniciavam os movimentos grevistas por o Colgio Pedro II. No ano de 1909
melhores condies de trabalho. fez sua estria como escritor com o
Essas agitaes eram sintomas da crise na Repblica do caf- lanamento da obra Recordaes
com-leite, que se tornaria mais evidente na dcada de 1920, servindo do Escrivo Isaas Caminha publi-
de cenrio para os questionamentos da Semana da Arte Moderna. cada em Portugal. Em 1911 escreveu
o romance Triste fim de Policarpo
Principais caractersticas Quaresma, publicado em folhetins
Ruptura com o passado, com o academicismo - Apesar de no Jornal do Comrcio. Lima Barreto
algumas posturas que podem ser consideradas conservadoras, considerado um autor Pr-moder-
h esse carter inovador em determinadas obras. A linguagem nista por causa da forma com que en-
de Augusto dos Anjos, por exemplo, ponteada de palavras cara os verdadeiros problemas do Bra-
no-poticas (como cuspe, vmito, escarro, vermes), era sil. Dessa forma, critica o nacionalismo ufanista surgido no final
uma afronta poesia parnasiana ainda em vigor. Lima Barreto do sc. XIX e incio do XX. Apesar de Lima Barreto no ter sido

UECEVEST 23

Apostilas UECEVEST mod3.indb 23 06/02/2011 09:57:19


L I T E R ATURA

reconhecido, em seu tempo, como um grande escritor, inegvel te esttico, Lobato assumiu posies antimodernistas, como bem
que pelo menos o romance Triste Fim de Policarpo Quaresma atesta seu artigo sobre a exposio de Anita Malfatti em 1917,
figure entre as obras primas da nossa literatura. intitulado Parania ou mistificao?, por meio do qual critica a
pintura caricatural da artista. Esse artigo desempenhou impor-
Principais Obras: tante papel na histria do Modernismo brasileiro, ao reunir, na
Romances: Triste fim de Policarpo Quaresma (1915); Clara dos defesa de Anita, alguns nomes novos, como Oswald de Andrade,
Anjos (1948). Stira : Os Bruzundangas (1923). Contos: Histrias Mrio de Andrade e Di Cavalcanti, revoltados com o conservado-
e sonhos (1920); Outras histrias e Contos argelinos (1952). rismo de Lobato, avesso verdadeira revoluo artstica que se ini-
ciava em terras paulistas e resultaria na Semana de Arte Moderna.
Veja o Texto: Lobato confessa que durante a Semana isolou-se jogando xadrez
Captulo III nas praias do Guaruj.
Quaresma chegou ao seu quarto, despiu-se, enfiou a camisa A partir de 1915, seus artigos na imprensa aumentam-lhe a
de dormir e, deitado, ps-se a ler um velho elogio das riquezas e popularidade e o prestgio, que se solidificam entre 1918 e 1921
opulncias do Brasil. com a publicao dos livros de contos Urups, Ideias de Jeca Tatu,
A casa estava em silncio; do lado de fora, no havia a mnima Cidades mortas e Negrinha. Ao lado da chamada literatura adul-
bulha. Os sapos tinham suspendido um instante a sua orquestra ta, Monteiro Lobato deixou extensa obra voltada para o pbli-
noturna. Quaresma lia; e lembrava-se que Darwin escutava com co infantil, justamente um campo at ento mal explorado em
prazer esse concerto dos charcos. Tudo na nossa terra extraor- nossas letras. Seu primeiro livro para crianas foi publicado em
dinrio! pensou. Da despensa, que ficava junto a seu aposento, 1921, com o ttulo A menina do narizinho arrebitado, mais tarde
vinha um rudo estranho. Apurou o ouvido e prestou ateno. rebatizado como Reinaes de Narizinho. A literatura infantil lo-
Os sapos recomearam o seu hino. Havia vozes baixas, outras batiana, alm do carter moralista e pedaggico, no abandona a
mais altas e estridentes; uma se seguia outra, num dado instan- luta pelos interesses nacionais empreendida pelo autor, com per-
te todas se juntaram um unssono sustentado. Suspenderam um sonagens representativos das vrias facetas de nosso povo e o Stio
instante a msica. O major apurou o ouvido; o rudo continuava. do Pica-pau Amarelo, que a imagem do prprio Brasil. Leia-se,
Que era? Eram uns estalos tnues; parecia que quebravam grave- por exemplo, O poo do visconde, em que a fico e a realidade se
tos, que deixavam outros cair ao cho... Os sapos recomearam; o misturam em torno do problema do petrleo.
regente deu uma martelada e logo vieram os baixos e os tenores.
Demoraram muito; Quaresma pde ler umas cinco pginas. Os Principais obras:
batrquios pararam; a bulha continuava. O major levantou-se, Contos: Urups (1919); Negrinha (1920);O Choque das Ra-
agarrou o castial e foi dependncia da casa donde partia o ru- as ou O Presidente Negro (1926). Literatura Infantil: Reinaes
do, assim mesmo como estava, em camisa de dormir. de Narizinho; Emlia no Pas da Gramtica; D. Quixote para as
Abriu a porta; nada viu. Ia procurar nos cantos, quando sen- Crianas; O Pica-pau Amarelo; O Marqus de Rabic.
tiu uma ferroada no p. Quase gritou. Abaixou a vela para ver
melhor e deu com uma enorme sava agarrada com toda a fria Augusto dos Anjos
sua pele magra. Descobriu a origem da bulha. Eram formigas Formou-se em direito, mas
que, por um buraco no assoalho, lhe tinham invadido a despensa foi sempre professor de Literatu-
e carregavam as suas reservas de milho e feijo, cujos recipientes ra. Nervoso, misantropo e solit-
tinham sido deixados abertos por inadvertncia. O cho estava rio, este possvel ateu morreu de
negro, e carregadas com os gro, elas, em pelotes cerrados, mer- forte gripe antes de assumir um
gulhavam no solo em busca da sua cidade subterrnea. cargo que lhe daria mais recursos.
Quis afugent-las. Matou uma, duas, dez, vinte, cem; mas Publicou apenas um nico
eram milhares e cada vez mais o exrcito aumentava. Veio uma, livro de poesias, Eu, mais tar-
mordeu-o, depois outra, e o foram mordendo pelas pernas, pelos de reeditado como Eu e outras
ps, subindo pelo seu corpo. No pde aguentar, gritou, sapateou poesias. Sua obra cientificista,
e deixou a vela cair. profundamente pessimista. Sua
Estava no escuro. Debatia-se para encontrar a porta; achou viso da morte como o fim, o
e correu daquele nfimo inimigo que, talvez, nem mesmo luz linguajar e os temas usados por
radiante do sol, o visse distintamente... muitos so considerados como sendo de mau gosto, mas caracte-
(Barreto, Lima. Triste Fim de Policarpo Quaresma. So Paulo, Ed. tica, 1983, p.92) rizam sua poesia como nica na literatura brasileira.
J o verme este operrio das runas / Que o sangue
Monteiro Lobato podre das carnificinas / Come e vida em geral declara guerra,
Jos Bento Monteiro Lobato Trabalhou, assim, como parnasiano e como simbolista, com
nasceu em Taubat, Estado de So sonetos e verso decasslabo. Sua viso de mundo e a interrogao
Paulo, em 18 de abril de 1882. do mistrio da existncia e do estar-no-mundo marcam esta nova
Aps os primeiros estudos em vertente potica. H uma aflio pessoal demonstrada com in-
sua cidade natal, matricula-se, em tensidade dramtica, alm do pessimismo. Constncia da morte,
1900, na Faculdade de Direito do desintegrao e os vermes.
Largo So Francisco, tornando-
se um dos integrantes do grupo Principal obra:
literrio do Minarete. Monteiro Poesias : Eu (1912).
Lobato estudado aqui como
um pr-modernista por duas ca- Versos ntimos
ractersticas fundamentais de sua Vs! Ningum assistiu ao formidvel
obra de fico: o regionalismo e a Enterro de tua ltima quimera.
denncia da realidade brasileira. Somente a Ingratido - esta pantera -
No entanto, no plano puramen- Foi tua companheira inseparvel!

24 UECEVEST

Apostilas UECEVEST mod3.indb 24 06/02/2011 09:57:20


LIT E R AT U R A

Acostuma-te lama que te espera! a) Inocncia Narra a histria de Cirino, um garimpeiro que se
O Homem, que, nesta terra miservel, apaixona por uma moa chamada Inocncia e, aps conseguir
Mora, entre feras, sente inevitvel enriquecer e eliminar o rival, se casa com a mulher amada.
Necessidade de tambm ser fera. b) O Cortio Descreve a habitao coletiva dirigida por um
portugus chamado Jernimo, que se apaixona por uma
Toma um fsforo. Acende teu cigarro! mulata chamada Rita Baiana e lhe compra a carta de alforria.
O beijo, amigo, a vspera do escarro, c) Os Sertes Divide-se em trs partes (a Terra, o Homem, a
A mo que afaga a mesma que apedreja. Luta), sendo que, na primeira, seguindo a moda cientfica da
poca, o autor faz um apanhado geral da regio das secas e
Se a algum causa inda pena a tua chaga, busca as suas possveis causas.
Apedreja essa mo vil que te afaga, d) A Escrava Isaura Nessa obra, l-se sobre a bela e sedutora
Escarra nessa boca que te beija! Isaura, que no conseguiu casar com lvaro, j que seu
apaixonado foi assassinado por Lencio, o dono da fazenda
O morcego onde vivia e que era, portanto, seu proprietrio.
Meia-noite. Ao meu quarto me recolho.
Meu Deus! E este morcego! E, agora, vede: 04. A obra de Lima Barreto:
Na bruta ardncia orgnica da sede, a) Reflete a sociedade rural do sculo XIX, podendo ser
Morde-me a goela gneo e escaldante molho. considerada precursora do romance regionalistamoderno.
b) Tem cunho social, embora esteja presa aos cnones estticos
Vou mandar levantar outra parede... e ideolgicos romnticos e influencioufortemente os
Digo. Ergo-me a tremer. Fecho o ferrolho romancistas da primeira gerao modernista.
E olho o teto. E vejo-o ainda, igual a um olho, c) considerada pr-modernista, uma vez que reflete a vida
Circularmente sobre a minha rede! urbana paulista antes da dcada de 20.
d) pr-modernista, refletindo forte sentimento nacional e
Pego de um pau. Esforos fao. Chego grande conscincia critica de problemasbrasileiros.
A toc-lo. Minhalma se concentra.
Que ventre produziu to feio parto?! 05. A obra rene uma srie de artigos, iniciados com Velha Pra-
ga, publicados em O Estado de So Paulo a 14-11-1914. Nes-
A Conscincia Humana este morcego! ses artigos, o autor insurge-se contra o extermnio das matas da
Por mais que a gente faa, noite, ele entra Mantiqueira pela ao nefasta das queimadas, retrgrada prtica
Imperceptivelmente em nosso quarto! agrcola perpetrada pela ignorncia dos caboclos, analisa o primi-
tivismo da vida dos caipiras do Vale da Paraba e critica a literatura
romntica que cantou liricamente esses marginais da civilizao:
a) Urups (Monteiro Lobato).
E X E R C C I O b) Margem da Histria (Euclides da Cunha)
c) Contrastes e Confrontos (Euclides da Cunha)
01. (UEL/PR) Nas duas primeiras dcadas de nosso sculo, as d) Ideias de Jeca Tatu (Monteiro Lobato)
obras de Euclides da Cunha e as de Lima Barreto, to diferentes
entre si, tm como elemento comum: 06. Em Os Sertes, de Euclides da Cunha, a natureza:
a) A inteno de retratar o Brasil de modo otimista e idealizante. a) funciona como contraponto narrao, ressaltando o
b) A adoo da linguagem coloquial das camadas populares do contaste entre o meio inerte e o homemagressivo.
serto. b) cenrio desolador, dentro do qual vivem e lutam os homens que
c) A expresso de aspectos at ento negligenciados da podem transform-la, sem que sejam por ela transformados.
realidade brasileira. c) condiciona o comportamento do homem, de acordo com as
d) A prtica de um experimentalismo lingustico radical. concepes do determinismo cientifico de fins do sculo XIX.
d) objeto de uma descrio romntica impregnada dos
02. (UFRS/RS) Considere as seguintes afirmaes sobre Os Ser- sentimentos humanos do autor.
tes, de Euclides da Cunha.
I. Nas duas primeiras partes do livro, o autor descreve, 07. (FUVEST) No romance Triste Fim de Policarpo Quaresma, o
respectivamente, o homem e o meio ambiente que nacionalismo exaltado e delirante da personagem principal mo-
constituam o serto baiano, valendo-se, para tanto, das tiva seu engajamento em trs diferentes projetos, que objetivam
teorias cientficas desenvolvidas na poca. reformar o pas. Esses projetos visam, sucessivamente, aos se-
II. Ao descrever os sertanejos, o autor idealiza suas qualidades guintes setores da vida nacional:
morais e fsicas e conclui que seu herosmo era resultado da a) cultural, agrcola e poltico
f nos ensinamentos religiosos do lder Antnio Conselheiro. b) lingustico, poltico e militar
III. O autor descreve, na terceira parte do livro, as vrias etapas c) lingustico, industrial e militar
da guerra de Canudos e denuncia o massacre dos sertanejos d) escolar, agrcola e militar
pelas tropas do exrcito brasileiro, revelando a misria e
08. Augusto dos Anjos autor de um nico livro, Eu, editado
subdesenvolvimento da regio.
pela primeira vez em 1912. Outras Poesias acrescentaram-se s
Marque o item correto: edies posteriores. Considerando a produo literria desse po-
a) Apenas I. c) Apenas III. eta, pode-se dizerque:
b) Apenas II. d) Apenas I e III. a) Foi elogiada poeticamente pela crtica de sua poca,
entretanto no representou um sucesso de pblico.
03. (Ufam) Assinale a alternativa que relaciona corretamente b) Traduz a sua subjetividade pessimista em reao ao homem
obra e assunto: e ao cosmos, por meio de um vocabulrio em reao ao

UECEVEST 25

Apostilas UECEVEST mod3.indb 25 06/02/2011 09:57:20


L I T E R ATURA

homem e ao cosmos, por meio de um vocabulrio tcnico- A mquina tornou-se parte integrante de todos os setores so-
cientfico-potico. ciais e o conforto passou a ser a preocupao fundamental do
c) Anuncia o Parnasianismo, em virtude das suas inovaes homem burgus da poca. Esse momento de euforia vivido pela
tcnico-cientficas e de sua temticapsicanaltica. burguesia ficou conhecido como belle poque. Vale lembrar que
d) Revela uma militncia poltico-ideolgica que o coloca entre essa vida agradvel e fcil abrangia somente a classes dominantes,
principais poetas brasileiros de veiosocialista. o proletariado continuou a ser marginalizado.
Todo esse progresso material provocou uma disputa acirrada
09. Sofreu influncias das ideias deterministas de Taine; nacio- pelo domnio do mercado consumidor entre as principais potn-
nalista ferrenho, deu grande valor mestiagem;foi o primeiro cias mundiais, que culminou com a ecloso da Primeira Guerra
intrprete da evoluo cultural e espiritual brasileira; ignorando Mundial, em 1914. Esse conflito, que durou at 1918 e envolveu
Hegel, Engels e Marx faltou- lhe uma concepo totalizante e praticamente quase todos os pases do mundo, aliado Revoluo
dialtica da cultura.: Russa de 1917, que levou o proletariado ao poder pela primeira
a) Domingos Olmpio c) Slvio Romero vez, provocando pavor nas classes dominantes, gerou um forte
b) Raul Pompia d) Adolfo Caminha sentimento de derrota, perda, destruio e total desconfiana nos
sistemas polticos vigentes.
10.Assinale a alternativa falsa, sobre Monteiro Lobato: Esse clima foi propcio para que surgissem, no decorrer do
a) vale-se das tradies orais do caipira, personificado pelo Jeca sculo, novas correntes ideolgicas baseadas no nacionalismo,
Tatu, valendo-se do coloquialismo docontador de casos. como o Nazismo e o Fascismo.
b) nos romance Urups e Cidades Mortas aborda a decadncia Onze anos depois do trmino da Primeira Guerra, o mundo
da agricultura no Vale do Paraba, aps o ciclo do caf. enfrentou a terrvel crise econmica de 1929, provocada pela es-
c) traz a paisagem do Vale do Paraba paulista, denunciando a peculao nas bolsas de valores e que culminou com a Segunda
devastao da natureza pela pratica agrcola da queimada. Grande Guerra Mundial (1939-1945). Esse curto perodo entre
d) explora os aspectos visveis do ser humano; seus contos tm as duas Grandes Guerras ficou conhecido como entre-guerras
quase sempre finais trgicos e deprimentes. ou anos loucos e caracterizado pela nsia de viver frenetica-
mente, uma vez que a guerra despertou no homem a incerteza de
uma paz duradoura.
G A B A R I T O
Expressionismo
01. c 02. d 03. c 04. d 05. a
06. c 07. a 08. b 09. c 10. d

VAnGUARDAS EUROPIAS

O grito, Edvard Munch

O Expressionismo a arte do instinto, trata-se de uma pin-


tura dramtica, subjetiva, expressando sentimentos humanos.
Utilizando cores irreais, d forma plstica ao amor, ao cime, ao
medo, solido, misria humana, prostituio. Deforma-se a
figura, para ressaltar o sentimento.
Predominncia dos valores emocionais sobre os intelectuais.
Corrente artstica concentrada especialmente na Alemanha entre
O termo vanguarda vem do francs avant-garde, termo 1905 e 1930.
militar que usado para caracterizar o grupo de soldados que,
durante as batalhas, vo frente das tropas. A partir do incio
Futurismo
do sc. XX esse termo passou a ser usado para designar um gru-
po de indivduos que, devido a seus conhecimentos ou por uma
tendncia natural, exercem o papel de precursor ou pioneiro em
um determinado movimento cultural, artstico, cientfico etc. No
campo das artes esse termo est sempre associado ruptura, uma
vez que, invariavelmente, se ope ao estilo vigente em uma poca.
Nos primeiros 20 anos do sc. XX as espetaculares invenes
automvel, cinema; mquinas voadoras etc. provocaram uma eu-
foria e crena no progresso exageradas. Dessa forma, o modo de viver
e de encarar a realidade sofreu radicais transformaes e a arte voltada
para o sonho e para as ideias vigentes no sculo anterior (positivismo,
cientificismo etc.) passaram a ser consideradas ultrapassadas.

26 UECEVEST

Apostilas UECEVEST mod3.indb 26 06/02/2011 09:57:21


LIT E R AT U R A

Futurismo um movimento artstico e literrio surgido Formado em 1916 em Zurique por jovens franceses e alemes
oficialmente em 20 de fevereiro de 1909 com a publicao do que, se tivessem permanecido em seus respectivos pases, teriam
Manifesto Futurista, do poeta italiano Filippo Marinetti, no jor- sido convocados para o servio militar, o Dada foi um movimen-
nal francs Le Figaro. A obra rejeitava o moralismo e o passado. to de negao. Durante a Primeira Guerra Mundial, artistas de
Apresentava um novo tipo de beleza, baseado na velocidade e na vrias nacionalidades, exilados na Sua, eram contrrios ao en-
elevao da violncia. volvimento dos seus prprios pases na guerra.
O novo uma caracterstica to forte do movimento, que este Fundaram um movimento literrio para expressar suas decep-
chegou a defender a destruio de museus e de cidades antigas. es em relao incapacidade da cincias, da religio, da filosofia
Considerava a guerra como forma de higienizar o mundo. que se revelaram pouco eficazes em evitar a destruio da Europa.
O futurismo desenvolveu-se em todas as artes, influenciando A palavra Dada foi descoberta acidentalmente por Hugo Ball e
vrios artistas que posteriormente instituram outros movimentos por Tzara Tristan num dicionrio alemo-francs. Dada uma
modernistas. Repercutiu principalmente na Frana e na Itlia, onde palavra francesa que significa na linguagem infantil cavalo de
vrios artistas, entre eles Marinetti, se identificaram com o fascismo. pau. Esse nome escolhido no fazia sentido, assim como a arte
O futurismo enfraqueceu aps a Primeira Guerra Mundial, que perdera todo o sentido diante da irracionalidade da guerra.
mas seu esprito rumoroso e inquieto refletiu no dadasmo, no con- Sua proposta que a arte ficasse solta das amarras raciona-
cretismo, na tipografia moderna e no design grfico ps-moderno. listas e fosse apenas o resultado do automatismo psquico, sele-
A pintura futurista recebeu influncia do cubismo e do abstra- cionado e combinando elementos por acaso. Sendo a negao
cionismo, mas utilizava-se de cores vivas e contrastes e a sobrepo- total da cultura, o Dadasmo defende o absurdo, a incoerncia,
sio das imagens com a pretenso de dar a ideia de dinamismo. a desordem, o caos. Politicamente , firma-se como um protesto
Na literatura, as principais manifestaes ocorreram na poesia contra uma civilizao que no conseguiria evitar a guerra.
italiana, que se dedicava s causas polticas. A linguagem espont- Veja como se faz um poema dadasta segundo Tristan Tzara:
nea e as frases so fragmentadas para exprimir a ideia de velocidade. Pegue um jornal
Pegue a tesoura.
Cubismo Escolha no jornal um artigo do tamanho que voc deseja dar
a seu poema.
Recorte o artigo.
Recorte em seguida com ateno algumas palavras que for-
mam esse artigo e meta-as num saco.
Agite suavemente.
Tire em seguida cada pedao um aps o outro.
Copie conscienciosamente na ordem em que elas so tiradas
do saco.
O poema se parecer com voc.
E ei-lo um escritor infinitamente original e de uma sensibili-
Guernica, Pablo Picasso dade graciosa, ainda que incompreendido do pblico.

Historicamente o Cubismo originou-se na obra de Czanne, Surrealismo


pois para ele a pintura deveria tratar as formas da natureza como
se fossem cones, esferas e cilindros. Entretanto, os cubistas foram
mais longe do que Czanne. Passaram a representar os objetos
com todas as suas partes num mesmo plano. como se eles es-
tivessem abertos e apresentassem todos os seus lados no plano
frontal em relao ao espectador. Na verdade, essa atitude de de-
compor os objetos no tinha nenhum compromisso de fidelidade
com a aparncia real das coisas.
O pintor cubista tenta representar os objetos em trs dimen-
ses, numa superfcie plana, sob formas geomtricas, com o pre-
domnio de linhas retas. No representa, mas sugere a estrutura
dos corpos ou objetos. Representa-os como se movimentassem
em torno deles, vendo-os sob todos os ngulos visuais, por cima e
por baixo, percebendo todos os planos e volumes.

Dadasmo Young Virgin - Salvador Dal

O surrealismo foi, por excelncia, a corrente artstica moder-


na da representao do irracional e do subconsciente. Suas ori-
gens devem ser buscadas no Dadasmo e na pintura metafsica
de Giorgio De Chirico.Este movimento artstico surge todas as
vezes que a imaginao se manifesta livremente, sem o freio do
esprito crtico. O que vale o impulso psquico. Os surrealistas
deixam o mundo real para penetrarem no irreal, pois a emoo
mais profunda do ser tem todas as possibilidades de se expressar
apenas com a aproximao do fantstico, no ponto onde a razo
humana perde o controle.
Marcel Duchamp. A publicao do Manifesto do Surrealismo, assinado por
Fonte. 1917/1964. Readymade: Mictrio. Andr Breton, em outubro de 1924, marcou historicamente o

UECEVEST 27

Apostilas UECEVEST mod3.indb 27 06/02/2011 09:57:22


L I T E R ATURA

nascimento do movimento. Nele se propunha a restaurao dos Poesia ortnima


sentimentos humanos e doinstinto como ponto de partida para Quando se estuda a obra potica de Pessoa, necessrio fazer
uma nova linguagem artstica. Para isso, era preciso que o homem uma distino entre todos os poemas que assinou com o seu nome
tivesse uma viso totalmente introspectiva de si mesmo e encon- verdadeiro, portanto considerados poesia ortonmia, e todos os
trasse esse ponto do esprito no qual a realidade interna e externa outros, atribudos a diferentes heternimos (dentre os quais desta-
so percebidas totalmente isentas de contradies. cam-se Alberto Caeiro, lvaro de Campos e Ricardo Reis).
A livre associao e a anlise dos sonhos, ambos mtodos da O fenmeno da heteronmia resolve, de modo interessante,
psicanlise freudiana, transformaram-se nos procedimentos b- uma questo que persegue Fernando Pessoa durante toda a sua
sicos do surrealismo, embora aplicados a seu modo. Por meio vida: o desdobramento do eu, a multiplicao de identidades.
do automatismo, ou seja,qualquer forma de expresso em que Outra questo que o ocupou foi a da sinceridade do fingimen-
a mente no exercesse nenhum tipo de controle, os surrealistas to, condio da criao literria, e que deu origem ao bastante
tentavamplasmar, seja por meio de formas abstratas ou figura- conhecido poema Autopsicografia (leia na Antologia).
tivas simblicas, as imagens da realidade mais profunda do ser Em inmeros poemas, o tema do questionamento do eu
humano: o subconsciente. trabalhado, s vezes de modo mais sofrido e denotando uma
O Surrealismo apresenta relaes com o Futurismo e o Da- grande angstia, s vezes de modo mais casual, mas sempre in-
dasmo. No entanto, se os dadastas propunham apenas a des- vestigando o que pode ser chamado de eu, buscando entender
truio, os surrealistas pregavam a destruio da sociedade em como se constitui a individualidade humana.
que viviam e a criao de uma nova, a ser organizada em outras
bases. Os surrealistas pretendiam, dessa forma, atingir uma outra Mensagem: a releitura mtica do destino portugus
realidade, situada no plano do subconsciente e do inconsciente. A Dentre as grandes obras da poesia ortnima de Fernando Pes-
fantasia, os estados de tristeza e de melancolia exerceram grande soa, destacam-se os belssimos poemas de Mensagem, obra com a
atrao sobre os surrealistas, e, nesse aspecto, eles se aproximam qual obteve o segundo lugar em um concurso do Secretariado de
dos romnticos, embora sejam muito mais radicais. Propaganda Nacional. Nesta obra, acompanha-se uma releitura
do destino de Portugal, tendo como base o fenmeno das nave-
gaes, a ligao entre os portugueses e o mar e o mito do enco-
berto, associado ao desaparecimento misterioso de D. Sebastio
(veja o poema transcrito na Antologia).
O MODERNISMO EM PORTUGAL
Poesia heternima
Fernando Pessoa Foi numa carta, dirigida ao amigo Adolfo Casais Monteiro,
que Fernando Pessoa explicou, pela primeira vez, o fenmeno do
seu desdobramento em vrios heternimos, ou seja, em outros
autores por ele criados, com qualidades e tendncias literrias
prprias, individuais, diferentes das do criador:
Desde criana tive a tendncia para criar em meu torno um
mundo fictcio, de me cercar de amigos e conhecidos que nunca
existiram. (...) Desde que me conheo como sendo aquilo a que
chamo eu, me lembro de precisar mentalmente, em figura, mo-
vimentos, carcter e histria, vrias figuras irreais que eram para
mim to visveis e minhas como as cousas daquilo a que chama-
mos, porventura abusivamente, a vida real. (...)

Gerao da Presena
no contexto do incio do Estado Novo que surge a revista
Presena (1927) com a proposta de uma literatura mais introspec-
tiva e intimista. Como trao caracterstico da literatura produzida
pelos jovens presencistas, observamos a opo por uma certa alie-
nao social. Atuando em um momento de srias crises polticas
O maior fenmeno da poesia portuguesa em Portugal, esses escritores preferiam interrogar o sentido da
Fernando Pessoa nasceu no dia 13 de junho de 1888, na ci- existncia humana, criando uma literatura introspectiva.
dade de Lisboa. Criana isolada e no desejando seno assim Seu maior mrito foi, na verdade, o de divulgar as conquis-
estar, o menino Fernando comea a povoar o seu universo com tas da primeira gerao modernista, consolidando a nova viso
figuras inventadas: um capito Thibeaut, um Chevalier de Pas, esttica em Portugal. Alm disso, suas preocupaes existenciais
em nome dos quais escreve cartas a si prprio. Na inveno des- aproximaram a literatura das teorias de Freud sobre o inconscien-
ses amigos comea j a manifestar-se, embora de uma maneira te humano. Foram, como seria de se esperar, influenciados pelas
embrionria e apenas ldica, aquilo que ele quis que fosse uma narrativas psicologizantes de Proust e Dostoievski.
dilatao do mundo real, a concretizao do possvel, a materia- Destacam-se, como autores do perodo, Jos Rgio (o prin-
lizao do fictcio. cipal terico do grupo), Miguel Torga, Joo Gaspar Simes,
Educado na frica do Sul, para onde seguiu em 1895, com o Adolfo Casais Monteiro e Branquinho da Fonseca. Em 1940,
segundo casamento de sua me, Pessoa regressa a Lisboa no ano com a circulao do ltimo nmero de Presena, essa gerao
de 1905. Em 1915, participa da fundao da revista Orpheu, que chegou ao fim.
dar incio ao Modernismo portugus.
Alm do escndalo da publicao de Orpheu, que lhe rendeu
alguma celebridade, a vida de Fernando Pessoa foi basicamente
Ultimatum
Mandado de despejo aos mandarins do mundo
annima e solitria. Morreu em 1935, de cirrose heptica, aos 47
Fora tu reles esnobe plebeu
anos de idade.
E fora tu, imperialista das sucatas

28 UECEVEST

Apostilas UECEVEST mod3.indb 28 06/02/2011 09:57:23


LIT E R AT U R A

Charlato da sinceridade e tu, da juba socialista, me aponta todas as cores que h nas flores/ e me mostra como as
e tu qualquer outro. pedras so engraadas/ quando a gente as tem na mo/ e olha deva-
gar para elas/. Damo-nos to bem um com o outro/ na companhia
Ultimatum a todos eles e a todos que sejam como eles todos. de tudo/ que nunca pensamos um no outro./ Vivemos juntos os
Monte de tijolos com pretenses a casa dois/ com um acordo ntimo,/ como a mo direita e a esquerda./
Intil luxo, megalomania triunfante Ao anoitecer ns brincamos as cinco pedrinhas/ no degrau da porta
E tu Brasil, blague de Pedro lvares Cabral de casa./ Graves, como convm a um Deus e a um poeta./ Como
que nem te queria descobrir. se cada pedra /fosse todo o Universo/ e fosse por isso um perigo
Ultimatum a vs que confundis o humano com o popular muito grande deix-la cair no cho./ Depois eu lhe conto histrias
Que confundis tudo! das coisas s dos homens./ E Ele sorri, porque tudo incrvel./ Ele
Vs anarquistas deveras sinceros ri dos reis e dos que no so reis./E tem pena de ouvir falar das
Socialistas a invocar a sua qualidade de trabalhadores para guerras e dos comrcios./Depois Ele adormece e eu o levo no colo
quererem deixar de trabalhar. para dentro da minha casa,/ deito-o na minha cama, despindo-o
lentamente,/ como seguindo um ritual/ todo humano e todo ma-
Sim, todos vs que representais o mundo, terno at Ele estar nu./ Ele dorme dentro da minha alma./ s vezes
homens altos, passai por baixo do meu desprezo Ele acorda de noite,/ brinca com meus sonhos./ Vira uns de perna
Passai, aristocratas de tanga de ouro, pro ar,/ pe uns por cima dos outros,/ e bate palmas, sozinho,/
Passai, frouxos sorrindo para o meu sono./ Quando eu morrer, Filhinho,/ seja eu a
Passai, radicais do pouco! criana, o mais pequeno,/ pega-me Tu ao colo,/ leva-me para den-
tro a Tua casa./ Deita-me na tua cama./ Despe o meu ser, cansado
Quem acredita neles? e humano./Conta-me histrias caso eu acorde/ para eu tornar a
Mandem tudo isso para casa, descascar batatas simblicas adormecer,/ e d-me sonhos Teus para eu brincar.
Fechem-me isso a chave e deitem a chave fora.
Sufoco de ter s isso a minha volta. Autopsicografia
O poeta um fingidor.
Deixem-me respirar! Finge to completamente
Abram todas as janelas Que chega a fingir que dor
Abram mais janelas do que todas as A dor que deveras sente.
janelas que h no mundo.
E os que leem o que escreve,
Nenhuma ideia grande, nenhuma corrente
Na dor lida sentem bem,
poltica que soe a uma ideia gro!
No as duas que ele teve,
Mas s a que eles no tm.
E o mundo quer a inteligncia nova
O mundo tem sede de que se crie E assim nas calhas de roda
O que a est a apodrecer a vida, quando muito, Gira a entreter a razo,
estrume para o futuro. Esse comboio de corda
O que a est no pode durar porque no nada. que se chama o corao.
Fernando Pessoa
Eu, da raa dos navegadores, afirmo que no pode durar!
Eu, da raa dos descobridores, desprezo
o que seja menos que descobrir o mundo novo.
Proclamo isso bem alto, braos erguidos, fitando o Atlntico
E X E R C C I O
e saudando abstratamente o infinito. 01. (F. M. Tringulo Mineiro-MG) Leia os versos a seguir para
lvaro de Campos - 1917 responder questo.
Multipliquei-me, para me sentir,
Poema do menino Jesus Alberto Caeiro Para me sentir, precisei sentir tudo,
Num meio-dia de fim de primavera eu tive um sonho como Transbordei-me, no fiz seno extravasar-me.
uma fotografia/ eu vi Jesus Cristo descer Terra./ Ele veio pela en-
costa de um monte,/ mas era outra vez menino,/ a correr e a rolar- Nos versos acima, o poeta expressa a sua necessidade de conhecer
se pela erva/ A arrancar flores para deitar fora,/ e a rir de modo a o universo alm da contingncia individual. Para isso, ele recorreu
ouvir-se de longe./ Ele tinha fugido do cu./ Era nosso demais pra aos heternimos e desdobrou-se em personalidades. Trata-se de:
fingir/ de Segunda pessoa da Trindade./Um dia que Deus estava a) Fernando Pessoa. d) Alexandre Herculano.
dormindo/ e o Esprito Santo andava a voar,/ Ele foi at a caixa dos b) Lus Vaz de Cames. e) Ea de Queirs.
milagres e roubou trs./ Com o primeiro Ele fez com que ningum c) Camilo Castelo Branco.
soubesse que Ele tinha fugido;/com o segundo Ele se criou eterna-
mente humano e menino;/ e com o terceiro Ele criou um Cristo 02. (UFPI) A potica de Fernando Pessoa compreende o artifcio
eternamente na cruz/ e deixou-o pregado na cruz que h no cu /e de multiplicar-se, de criar vrios sujeitos. Em relao s obras de
serve de modelo s outras. /Depois Ele fugiu para o Sol/ e desceu Pessoa ortnimo e do heternimo Caeiro, pode-se afirmar:
pelo primeiro raio que apanhou./ Hoje Ele vive na minha aldeia, I. Enquanto o ortnimo complicado, angustiado, Caeiro
comigo./ uma criana bonita, de riso natural./ Limpa o nariz simples, sereno.
com o brao direito, /chapinha nas poas dgua, /colhe as flores, II. O ortnimo o mestre buclico, antimetafsico.
gosta delas, esquece./Atira pedras aos burros,/ colhe as frutas nos III. Caeiro o conciliado consigo prprio e com o mundo.
pomares,/e foge a chorar e a gritar dos ces./ S porque sabe que IV. Os poemas de Caeiro so marcados por muita emoo e
elas no gostam,/ e toda gente acha graa, /Ele corre atrs das rapa- musicalidade.
rigas/ que levam as bilhas na cabea e levanta-lhes a saia./A mim, V. Os poemas do ortnimo apresentam a naturalidade de um
Ele me ensinou tudo./ Ele me ensinou a olhar para as coisas./ Ele discurso oral.

UECEVEST 29

Apostilas UECEVEST mod3.indb 29 06/02/2011 09:57:23


L I T E R ATURA

Esto corretas as afirmativas: MODERNISMO BRASILEIRO


a) I e II; c) II e III;
b) I e III; d) I e IV.

03. (UMC-SP) Assinale a alternativa correta a respeito das trs


afirmaes abaixo:
I. Os heternimos de Fernando Pessoa nascem de um mltiplo
desdobramento de sua personalidade.
II. Alberto Caeiro o poeta que se volta para o campo,
procurando viver em simplicidade.
III. Ricardo Reis um poeta moderno, que do desespero extrai a
prpria razo de ser.
a) Apenas I e II esto corretas;
b) Todas esto corretas;
c) Apenas I e III esto corretas;
d) Nenhuma est correta;

04. UFBA Abaporu, de Tarsila do Amaral


Quem me dera que eu fosse o p da estrada
E que os ps dos pobres me estivessem pisando... Todo este sangue de mil raas / corre em minhas veias / sou
Quem me dera que eu fosse os rios que correm brasileiro / mas do Brasil sem colarinho / do Brasil negro / do
E que as lavadeiras estivessem minha beira... Brasil ndio. Srgio Milliet
Quem me dera que eu fosse os choupos margem do rio
E tivesse s o cu por cima e a gua por baixo...
Iniciou-se no Brasil com a SAM (Semana de Arte Moderna)
Quem me dera que eu fosse o burro do moleiro
de 1922. Mas nem todos os participantes da Semana eram mo-
E que ele me batesse e me estimasse...
dernistas: o pr-modernista Graa Aranha foi um dos oradores.
Antes isso que ser o que atravessa a vida
Apesar de no ter sido dominante no comeo, como atestam as
Olhando para trs de si e tendo pena...
vaias da plateia da poca, este estilo, com o tempo, suplantou os
PESSOA, Fernando. Obra potica. Poemas completos de Alberto Caeiro. Rio de anteriores. Era marcado por uma liberdade de estilo e aproxima-
Janeiro: Jos Aguilar, 1992. p. 215.
o da linguagem com a linguagem falada; os de primeira fase
Constitui caracterstica da poesia de Caeiro comprovvel no texto: eram especialmente radicais quanto a isso.
01. Repdio ao misticismo. Didaticamente, divide-se o Modernismo em trs fases: a pri-
02. Desejo de integrao plena com a natureza. meira fase, mais radical e fortemente oposta a tudo que foi ante-
04. Valorizao das sensaes visuais. rior, cheia de irreverncia e escndalo; uma segunda mais amena,
08. Percepo de fragmentos da realidade. que formou grandes romancistas e poetas; e uma terceira, tam-
16. Descompromisso com o futuro. bm chamada Ps-Modernismo por vrios autores, que se opu-
32. Viso de mundo marcada pela simplicidade do existir to- nha, de certo modo, primeira e era por isso ridicularizada com
somente. o apelido de neoparnasianismo.
64. Negao da interioridade das coisas.
D, como resposta, a soma das alternativas corretas. Momento Histrico
Incio do sculo XX: apogeu da Belle poque. O burgus com-
05. U. F. Uberlndia-MG portado, tranquilo, contando seu lucro. Capitalismo monet-
Ah, o mundo quanto ns trazemos. rio. Industrializao e Neocolonialismo.
Existe tudo porque existo. Reivindicaes de massa. Greves e turbulncias sociais. Socia-
Fernando Pessoa. lismo ameaa.
Da minha pessoa de dentro no tenho noo de realidade. Progresso cientfico: eletricidade. Motor a combusto: auto-
Sei que o mundo existe, mas no sei se existo. mvel e avio.
Alberto Caeiro. Concreto armado: arranha-cu. Telefone, telgrafo. Mundo
Lendo comparativamente os dois fragmentos, e considerando a da mquina, da informao, da velocidade.
proposta potica pessoana, pode-se afirmar que: Primeira Guerra Mundial e Revoluo Russa.
a) Tanto em Alberto Caeiro como em Fernando Pessoa ele- Abolir todas as regras. O passado responsvel. O passado,
mesmo, o eu sempre uma identidade fingida. sem perfil, impessoal. Eliminar o passado.
b) H uma espcie de neoromantismo em Fernando Pessoa, Arte Moderna. Inquietao. Nada de modelos a seguir. Reco-
devido ao centramento no eu. mear. Rever. Reeducar. Chocar. Buscar o novo: multiplicida-
c) Observa-se uma permanncia do naturalismo do sculo de e velocidade, originalidade e incompreenso, autenticidade
XIX, devido ao naturismo de Caeiro. e novidade.
d) Em ambos, observa-se uma mesma relao entre o eu e o mundo. Vanguarda - estar frente, repudiar o passado e sua arte. Abai-
xo o padro cultural vigente.

Primeira fase Modernista no Brasil (1922-1930)


G A B A R I T O Caracteriza-se por ser uma tentativa de definir e marcar posi-
es. Perodo rico em manifestos e revistas de vida efmera.
01. a 02. d 03. b 04. 42 05. a 06. d Um ms depois da SAM, a poltica vive dois momentos im-
portantes: eleies para Presidncia da Repblica e congresso
(RJ) para fundao do Partido Comunista do Brasil. Ainda no

30 UECEVEST

02 LITERATURA.indd 30 07/02/2011 19:26:30


LIT E R AT U R A

campo da poltica, surge em 1926 o Partido Democrtico que Revista Antropofagia (1928-1929)
teve entre seus fundadores Mrio de Andrade. Contou com duas fases (denties): a primeira com 10 nme-
a fase mais radical justamente em consequncia da necessi- ros (1928 e 1929) direo Antnio Alcntara Machado e gerncia
dade de definies e do rompimento com todas as estruturas do de Raul Bopp; a segunda foi publicada semanalmente em 16 n-
passado. Carter anrquico e forte sentido destruidor. meros no jornal Dirio de So Paulo (1929) e seu aougueiro
Principais autores desta fase: Mrio de Andrade, Oswald (secretrio) era Geraldo Ferraz. uma nova etapa do nacionalis-
de Andrade, Manuel Bandeira, Antnio de Alcntara Machado, mo Pau-Brasil e resposta ao grupo Verde-amarelismo. A origem
Menotti del Picchia, Cassiano Ricardo, Guilherme de Almeida e do nome movimento esta na tela Abapuru de Tarsila do Amaral.
Plnio Salgado. 1 fase inicia-se com o polmico manifesto de Oswald e con-
ta com Alcntara Machado, Mrio de Andrade (2 nmero
publicou um captulo de Macunama), Carlos Drummond (3
nmero publicou a poesia No meio do caminho); alm de
desenhos de Tarsila, artigos em favor da lngua tupi de Plnio
Salgado e poesias de Guilherme de Almeida.
2 fase mais definida ideologicamente, com ruptura de
Oswald e Mrio de Andrade. Esto nessa segunda fase Oswald,
Bopp, Geraldo Ferraz, Oswaldo Costa, Tarsila, Patrcia Galvo
(Pagu). Os alvos das crticas (mordidas) so Mrio de Andrade,
Alcntara Machado, Graa Aranha, Guilherme de Almeida,
Menotti del Picchia e Plnio Salgado.
Organizadores da semana de arte moderna
S A ANTROPOFAGIA nos une, Socialmente. Econo-
Caractersticas micamente. Filosoficamente. / nica lei do mundo. Expresso
busca do moderno, original e polmico; mascarada de todos os individualismos, de todos os coletivismos.
nacionalismo em suas mltiplas facetas; De todas as religies. / De todos os tratados de paz. / Tupi or not
volta s origens e valorizao do ndio verdadeiramente brasileiro; tupi, that is the question. (Manifesto Antropfago)
lngua brasileira - falada pelo povo nas ruas; A nossa independncia ainda no foi proclamada. Frase t-
pardias - tentativa de repensar a histria e a literatura brasileiras. pica de D. Joo VI: Meu filho, pe essa coroa na tua cabea,
antes que algum aventureiro o faa! Expulsamos a dinastia.
A postura nacionalista apresenta-se em duas vertentes preciso expulsar o esprito bragantino, as ordenaes e o rap de
nacionalismo crtico, consciente, de denncia da realidade, Maria da Fonte. (Revista de Antropofagia, n. 1)
identificado politicamente com as esquerdas.
nacionalismo ufanista, utpico,exagerado, identificado com as Oswald de Andrade
correntes de extrema direita.

manifestos e Revistas
Revista Klaxon mensrio de Arte moderna (1922-1923)
Recebe este nome, pois klaxon era o termo usado para desig-
nar a buzina externa dos automveis. Primeiro peridico moder-
nista, consequncia das agitaes em torno da SAM. Inovadora
em todos os sentidos: grfico, existncia de publicidade, oposio
entre o velho e o novo.
Klaxon sabe que o progresso existe. Por isso, sem renegar
o passado, caminha para diante, sempre, sempre. Oswald de Andrade, de Tarsila do Amaral.

manifesto da Poesia Pau-Brasil (1924-1925) Foi poeta, romancista, ensasta e teatrlogo. Figura de muito
Escrito por Oswald e publicado inicialmente no Correio da destaque no Modernismo Brasileiro, ele trouxe de sua viagem
Manh. Em 1925, publicado como abertura do livro de poesias Europa o Futurismo. Foi um dos principais artistas da Semana de
Pau-Brasil de Oswald. Apresenta uma proposta de literatura vincu- Arte Moderna e lanou o Movimento Pau-Brasil e a Antropofagia,
lada realidade brasileira, a partir de uma redescoberta do Brasil. corrente que pretendia devorar as culturas europias e brasileira
A poesia existe nos fatos. Os casebres de aafro e de ocre da poca e criar uma verdadeira cultura brasileira. Fazendeiro de
nos verdes da Favela sob o azul cabralino, so fatos estticos. caf, perdeu tudo e foi falncia em 1929 com o crash da Bolsa
A lngua sem arcasmos, sem erudio. Natural e neol- de Valores. Militante esquerdista, passou a divulgar o Comunismo
gica. A contribuio milionria de todos os erros. Como falamos. junto com Pagu em 1931, mas desligou-se do Partido em 1945.
Como somos. Sua obra marcada por irreverncia, coloquialismo, naciona-
lismo, exerccio de demolio e crtica. Incomodar os acomoda-
Verde-Amarelismo (1926-1929) dos, estimular o leitor atravs de palavras de coragem eram cons-
uma resposta ao nacionalismo do Pau-Brasil. Grupo for- tantes preocupaes desse autor.
mado por Plnio Salgado, Menotti del Picchia, Guilherme de Depois de participar da SAM, viaja Europa e o dirio de
Almeida e Cassiano Ricardo. Criticavam o nacionalismo afran- bordo destas viagens o romance cubista Memrias Sentimentais
cesado de Oswald. Sua proposta era de um nacionalismo primi- de Joo Miramar, que os crticos chamaram de prosa telegrfica.
tivista, ufanista, identificado com o fascismo, evoluindo para o Este romance-caleidoscpio inaugura, no nvel da prosa, a ten-
Integralismo de Plnio Salgado (dcada de 30). Idolatria do tupi dncia antinormativa da literatura contempornea, rompendo os
e a anta eleita smbolo nacional. Em maio de 1929, o grupo modelos realistas. Seus 163 fragmentos registram a trajetria do
verde-amarelista publica o manifesto Nhengau Verde-Amarelo brasileiro rico de todos os tempos: Europa - casamento - amante
Manifesto do Verde-Amarelismo ou da Escola da Anta. - desquite - vida literria - apertos financeiros - ...

UECEVEST 31

Apostilas UECEVEST mod3.indb 31 06/02/2011 09:57:25


L I T E R ATURA

O momento esteticamente mais radical do Modernismo foi a regies do Brasil, constri-se um heri que encarna o homem
Antropofagia. Invocando a cultura e os costumes primitivos do Bra- latino-americano. Macunama uma figura totalmente fora dos
sil, este movimento afirma a necessidade de sermos um povo antro- esquemas tradicionais da prosa de fico, uma aglutinao de al-
pfago, para no nos atrofiarmos culturalmente. Devem-se filtrar as guns possveis tipos brasileiros. Sempre na defesa, Macunama
contribuies estrangeiras para alcanar uma sntese transformadora. comea comendo terra e acaba sendo comido pela terra.
Com a crise econmica de 1929, Oswald passa por dif- Na obra Lira Paulistana, Mrio faz uma interpretao po-
ceis condies financeiras e v-se obrigado a conjugar o verbo tica de seu destino e integrao com a cidade de So Paulo. O
crakar. Falido economicamente, Oswald vai-se pendurar nos reflexo do eu na transparncia do rio Tiet mostra as guas do rio
reis da vela, os agiotas do beco do escarro (zona bancria de SP). como se fosse um espelho mgico. O Banquete um explosivo
Com isso, o autor vai recolhendo material para sua pea O Rei depoimento sobre as linhas mestras do pensamento esttico de
da Vela. Morreu sofrendo dificuldades de sade e financeiras, Mrio de Andrade, alm de constituir uma stira sobre certos
mas sem perder o contato com os artistas da poca. comportamentos tpicos no tempo da ditadura estadonovista.

Principais obras: Principais obras


Poesia: Pau-Brasil (1925); Romance: Memrias Sentimentais Poesia: Paulicia Desvairada (1922); Cl do Jabuti (1927);
de Joo Miramar (1924); Serafim Ponte Grande (1933); Manifes- Lira Paulistana (1946). Romance: Amar, Verbo Intransitivo
tos, teses e ensaios: Manifesto Pau-Brasil (1925); Manifesto Antro- (1927); Macunama (1928).
pfago (1928); Teatro: O Rei da Vela (1937).
Macunama
As meninas da gare
Eram trs ou quatro moas bem moas e bem gentis
Com cabelos mui pretos pelas espduas
E suas vergonhas to altas e to saradinhas
Que de ns as muito bem olharmos
No tnhamos nenhuma vergonha

Erro de Portugus
Quando o portugus chegou
Debaixo de uma bruta chuva
Vestiu o ndio
Que pena!
Fosse uma manh de sol
O ndio tinha despido
O portugus

Mrio de Andrade

Rapsdia escrita em 1926 e publicada em 1928, traz uma


variedade de motivos populares que Mrio de Andrade juntou
de acordo com as afinidades existentes entre eles. Trata-se de uma
espcie de coquetel do folclrico e do popular do Brasil. Mrio
Um dos organizadores do Modernismo e da SAM e foi o de Andrade mistura o maravilhoso e o sobre-humano ao retratar
que apresentou projeto mais consistente de renovao. Come- as faanhas de um heri que no apresenta rigorosos referenciais
ou escrevendo crticas de arte e poesia (ainda parnasiana) com o espao-temporais Macunama o representante de todas as
pseudnimo de Mrio Sobral. Rompeu com o Parnasianismo e o pocas e de todos os espaos brasileiros. Macunama, que leva o
passado com Paulicia Desvairada e a Semana, da qual participou subttulo de heri sem nenhum carter, tambm o nome do
ativamente. personagem central, um heri amerndio que trai e trado, que
Injetou em tudo que fez um senso de problemtico brasilei- preguioso, indolente, mas esperto e matreiro, individualista
rismo, da sua investida no folclore. De jeito simples, sua colo- e dbio. Destitudo da aurola idealizada dos romnticos, Ma-
quialidade desarticulou o esprito nacional de uma montanha de cunama o ndio moderno, mltiplo e contraditrio. Nasce na
preconceitos arcaicos. Lutou sempre por uma literatura brasileira selva, filho de uma ndia tapanhumas, fala tardiamente e s anda
e com temas brasileiros. quando ouve o som do dinheiro. Vira prncipe e trai o irmo
Seu primeiro romance Amar, Verbo Intransitivo que pene- Jigu ao brincar com as cunhadas, primeiro Sofar e depois Iri-
tra na estrutura familiar da burguesia paulistana, sua moral e seus qui. Vira homem e mata a me, enganado por Anhang. Casa-se
preconceitos. Aborda, ao mesmo tempo, os sonhos e a adapta- com Ci, a me do mato, guerreira amazonas da tribo das Icamia-
o dos imigrantes na agitada Paulicia.J em Macunama, Heri bas. Macunama torna-se o Imperador do Mato Virgem. Aps
sem nenhum carter, cria um anti-heri com um perfil indolente, seis meses, tem um filho. A criana morre, transformando-se em
brigo, covarde, sincero, mentiroso, trabalhador, preguioso, ma- planta do guaran. Ci, cansada e desiludida, vira a estrela Beta
landro, otrio - multifacetado. Inspirando-se no folclore indgena da Constelao Centauro. Antes de morrer, porm, Ci deixa ao
da Amaznia, mesclando a lendas e tradies das mais variadas esposo a muiraquit, uma pedra talism que lhe daria a garantia

32 UECEVEST

Apostilas UECEVEST mod3.indb 32 06/02/2011 09:57:26


LIT E R AT U R A

de felicidade. Mas o heri perde a pedra que acaba nas mos do ciais. Assim haveis de compreender de que alvaras falamos; por-
rico comerciante peruano Venceslau Pietro Pietra, colecionador que lagostas so carssimas, carssimas, subditas, e algums hemos
de pedras em So Paulo. Em companhia de seus dois irmos ns adquiridas por sessenta contos e mais; o que, convertido em
Maanape e Jigu vem para So Paulo a fim de reconquistar a pe- nossa moeda tradicional, alcana a vultosa soma de oitenta mi-
dra, que simboliza seu prprio ideal. Porm, Venceslau, que est lhes de bagos de cacau... Bem podereis conceber, pois, quanto
disfarado de comerciante, na verdade o gigante Piaim, come- hemos gasto; e que j estamos carecidos do vil metal, para brincar
dor de gente; por isso, as investidas de Macunama contra ele no com tais difceis donas. Bem quisramos impormos nossa ardi-
do resultado. S depois de apelar para a macumba Macunama da chama uma abstinncia, penosa senhora, para vos pouparmos
consegue derrotar o gigante. Reconquistada a pedra, Macunama despesas; porm, que nimo forte no cedera ante os encantos e
retorna ao Amazonas e se deixa atrair pela Iara, perdendo definiti- galanteios de to agradveis pastoras.
vamente a pedra. Como j no v mais graa no mundo, vai para
o cu, onde se transforma em estrela da Constelao Ursa Maior, manuel Bandeira
ficando relegado ao brilho intil das estrelas. uma das figuras mais importantes da poesia brasileira e um
dos iniciadores do Modernismo. Do penumbrismo ps-simbolis-
Trechos: ta de A Cinza das Horas s experincias concretas da dcada de
60 de Composies e Ponteios, a poesia de Bandeira destaca-se
I - macunama pela conscincia tcnica com que manipulou o verso livre. Parti-
No fundo do mato-virgem nasceu Macunama, heri da nos- cipa indiretamente da SAM, quando Ronald de Carvalho decla-
sa gente. Era preto retinto e filho do medo da noite. Houve um ma seu poema Os Sapos.
momento em que silncio foi to grande escutando o murmurejo
do Uraricoera, que a ndia tapanhumas pariu uma criana feia.
Essa criana que chamaram de Macunama.
J na meninice fez coisas de sarapantar. De primeiro passou
mais de seis anos no falando. Si o incitavam a falar exclamava:
Ai que preguia!...
e no dizia mais nada. Ficava no canto da maloca, trepado
no jirau de paxiba, espiando o trabalho dos outros e principal-
mente os dois manos que tinha, Maanape j velhinho e Jigu
na fora do homem. O divertimento dele era decepar cabea de
sava. Vivia deitado mas si punha os olhos em dinheiro, Macu-
nama dandava pra ganhar vintm. E tambm espertava quando
a famlia ia tomar banho no rio, todos juntos e nus. Passava o
tempo do banho dando mergulho, e as mulheres soltavam gritos
gozados por causa dos guaiamuns diz-que habitando a gua-doce Manuel Bandeira, por Portinari.
por l. No mucambo si alguns cunhat se aproximava dele pra
fazer festinha, Macunama punha a mo nas graas dela, cunhat Sempre pensando que morreria cedo (tuberculoso), acabou vi-
se afstava. Nos machos guspia na cara. Porm respeitava os velhos vendo muito e marcando a literatura brasileira. Morte e infncia
e frequentava com aplicao a murua a porac o tor o bacoroc so as molas propulsoras de sua obra. Ironizava o desnimo provo-
a cucuicogue, todas essas danas religiosas da tribo. cado pela doena, mas em Cinza das Horas apresenta melancolia
e sofrimento por causa da dama branca. Alm de ser um poeta
V Piaim fabuloso, tambm foi ensasta, cronista e tradutor. O prprio autor
(...) define sua poesia como a do gosto humilde da tristeza.
Venceslau Pietro Pietra morava num tejupar maravilhoso ro- Em Estrela da Manh, atinge a plenitude de seu lirismo
deado de mato no fim da rua Maranho olhando pra Noruega libertrio, mostrando que tudo pode ser matria potica: um cls-
do Pacaembu. Macunama falou pra Maanape que ia dar uma sico esquecido, uma frase de criana, uma notcia de jornal, a casa
chegadinha at l por amor de conhecer Venceslau Pietro Pietra. em que morava e at mesmo uma propaganda de trs sabonetes
Maanape fez um discurso mostrando as inconvenincias de ir l (Baladas das trs mulheres do sabonete Arax).
porque o regato andava com o calcanhar pra frente e si Deus o
assinalou alguma coisa lhe achou. De certo um manuari malvo... Principais obras
Quem sabe si o gigante Piaim comedor de gente!... Macunama Poesia: A Cinza das Horas (1917); Carnaval (1919); Liberti-
no quis saber. nagem (1930);Estrela da Manh (1936); Lira dos CinquentAnos
Pois vou assim mesmo. Onde me conhecem honras me (1940); Estrela da Tarde (1963); Estrela da Vida Inteira (1966).
do, onde no me conhecem me daro ou no!
Ento Maanape acompanhou o mano. Debussy
(...) Para c, para l...
Para c, para l...
IX Carta Pras Icamiabas Um novelozinho de linha...
(...) Para c, para l...
Ponde tento na acentuao deste vocbulo, senhoras Amazo- Para c, para l...
nas, pois muito nos pesara no prefersseis conosco, essa pronncia, Oscila no ar pela mo de uma criana...
condizente com a lio dos clssicos, pronncia Cleopatra, dico (Vem e vai...)
mais moderna; e que alguns vocabulistas levianamente subscrevem, Que delicadamente e quase a adormecer o balana
sem que se apercebam de que ganga desprezvel, que nos trazem, Psiu... -
com o enxurro de Frana, os galiparlas de m morte. Para c, para l...
Pois com esse delicado monstro, vencedor dos mais delica- Para c e...
dos vus paladinos, que as donas de c tombam nos leitos nup- O novelozinho caiu.

UECEVEST 33

Apostilas UECEVEST mod3.indb 33 06/02/2011 09:57:26


L I T E R ATURA

Vou-me embora pra Pasrgada E X E R C C I O


Vou-meemborapraPasrgada
Lsouamigodorei 01. (PUC-RS) Assinale com V (verdadeiro) ou com F (falso) as
Ltenhoamulherqueeuquero afirmativas sobre a produo literria brasileira das primeiras d-
nacamaqueescolherei cadas do sculo XX.
Vou-meemborapraPasrgada ( )Os movimentos de vanguarda europeus influenciaram
significativamente os escritores brasileiros.
Vou-meemborapraPasrgada
( )Houve maior preocupao em romper com os padres
Aquieunosoufeliz
temticos do que com os formais.
Laexistnciaumaaventura
( )Oswald de Andrade e Mario de Andrade protagonizaram as
Detalmodoinconsequente
principais mudanas na primeira fase modernista.
QueJoanaaLoucadeEspanha
( )A recorrncia pardia em relao poesia antecedente
Rainhaefalsademente
evidenciou a opo ideolgica dos modernistas.
Vemasercontraparente
( )As diferenas entre poesia e prosa foram enfatizadas neste
Danoraquenuncative
perodo.
Ecomofareiginstica
A sequncia correta, resultante do preenchimento dos parnteses,
Andareidebicicleta
de cima para baixo,
Montareiemburrobrabo
a) V F V F V c) F V F F V
Subireinopau-de-sebo
b) V F V V F d) V V F V F
Tomareibanhosdemar!
Equandoestivercansado
02. (PUC-RS) Para responder questo, ler o texto abaixo.
Deitonabeiradorio
Irene no cu
Mandochamarame-dgua
Irene preta
Pramecontarashistrias
Irene boa
Quenotempodeeumenino
Irene sempre de bom humor.
Rosavinhamecontar
Vou-meemborapraPasrgada. Imagino Irene entrando no cu:
EmPasrgadatemtudo Licena, meu branco!
outracivilizao E So Pedro, bonacho:
Temumprocessoseguro Entra, Irene. Voc no precisa pedir licena.
Deimpediraconcepo
Temtelefoneautomtico Manuel Bandeira, poeta modernista, revela no texto em questo
Temalcalidevontade uma das suas fortes caractersticas, qual seja, a tendncia a
Temprostitutasbonitas a) Tematizar o cotidiano em linguagem cifrada e metafrica.
paraagentenamorar b) Excluir personagens associadas s minorias marginalizadas.
c) Recorrer ao mundo real para abordar questes metafsicas.
Equandoeuestivermaistriste d) Associar subjetividade e objetividade.
Mastristedenoterjeito
Quandodenoitemeder 03. (PUC-PR) Mrio de Andrade considerado o mestre dos
Vontadedemematar escritores modernistas por sua atuao crtica e pela variedade de
Lsouamigodorei gneros textuais que escreveu e que se transformaram em exem-
Tereiamulherqueeuquero plos da escrita literria desse perodo esttico. Assinale a alternati-
Nacamaqueescolherei va que contm a afirmao correta sobre sua obra:
Vou-meemborapraPasrgada. a) Macunama uma colagem de mitos amaznicos e
narrativas urbanas, e estilos narrativos diversificados.
Tragdia Brasileira b) Amar verbo intransitivo o mais completo exemplo da
Misael, funcionrio da fazenda, com 63 anos de idade. prosa sentimental modernista, exaltada e idealizadora.
Conheceu Maria Elvira na Lapa prostituta, com sfilis, der- c) Paulicia desvairada obra patritica, de forte cunho
mite nos dedos, uma aliana empenhada e os dentes em petio nacionalista e uma concepo irracionalista da existncia.
de misria. d) Contos novos uma coletnea de narrativas que combinam
Misael tirou Maria Elvira da vida, instalou-a num sobrado no a narrativa enraizadamente colonial e a vanguarda europia e
Estcio, pagou mdico, dentista, manicura... experimental.
Dava tudo quanto ela queria.
Quando Maria Elvira se apanhou de boca bonita, arranjou (Unifesp/SP) Leia o poema de Oswald de Andrade e responda as
logo um namorado. Misael no queria escndalo. Podia dar uma questes de nmeros 04 e 05.
surra, um tiro, uma facada. No fez nada disso: mudou de casa. Senhor feudal
Viveram trs anos assim. Se Pedro Segundo
Toda vez que Maria Elvira arranjava namorado, Misael mu- Vier aqui
dava de casa. Com histria
Os amantes moraram no Estcio, Rocha, Catete, Rua Gene- Eu boto ele na cadeia.
ral Pedra, Olaria, Ramos, Bonsucesso, Vila Isabel,Rua Marqus
de Sapuca, Niteri, Encantado, Rua Clapp, outra vez no Estcio, 04. (Unifesp/SP) Considere as seguintes caractersticas do Mo-
Todos os Santos, Catumbi, Lavradio, Boca do Mato, Invlidos... dernismo brasileiro:
Por fim na Rua da Constituio, onde Misael, privado de I. Busca de uma lngua brasileira;
sentidos e de inteligncia, matou-a com seis tiros, e a polcia foi II. Versos livres;
encontr-la cada em decbito dorsal, vestida de organdi azul. III. Ironia e humor.

34 UECEVEST

Apostilas UECEVEST mod3.indb 34 06/02/2011 09:57:27


LIT E R AT U R A

Nos versos de Oswald de Andrade: 10. (FATEC)E o olhar estaria ansioso esperando e a cabea ao
a) Apenas I est presente. sabor da mgoa balanado e o corao fugindo e o corao vol-
b) Apenas III est presente. tando e os minutos passando e os minutos passando...
c) Apenas I e II esto presentes. (Vincius de Moraes, O olhar para trs)
d) I, II e III esto presentes.
A figura de linguagem que predomina nestes versos :
05. (Unifesp/SP) Considerando os pressupostos do Modernismo a) O pleonasmo, marcado pela repetio desnecessria da
e da potica oswaldiana, correto afirmar que a aluso a D. Pe- conjuno coordenada sindtica aditiva e.
dro II, figura da corte portuguesa, sugere b) O paradoxo, expresso pela contradio das aes
a) A reafirmao da base literria brasileira, decalque dos manifestadas pelos verbos no gerndio.
valores europeus. c) O polissndeto, caracterizado pela repetio da conjuno
b) A negao do valor da literatura portuguesa e apresenta a coordenada aditiva e, para conotar j intensidade da crescente
brasileira como insupervel. sensao de ansiedade contraditria do ato de esperar.
c) A stira ao referencial artstico portugus e, por extenso, d) A sinestesia, manifestada pela referncia interao dos
critica a importao de valores literrios europeus. sentidos: viso e corao no momento de espera.
d) O confronto entre a arte literria brasileira e a portuguesa, 11. (FUVEST) correto afirmar que, em Morte e Vida Severina:
elucidando a inevitvel influncia desta para a formao a) O carter de afirmao da vida, apesar de toda a misria,
daquela. comprova-se pela ausncia da ideia desuicdio.
b) A viagem do retirante, que atravessa ambientes menos e mais
06. (UFC) A respeito do livro Macunama, correto afirmar que:
hostis, mostra-lhe que a misria amesma, apesar dessas
a) A obra faz uma leitura do Brasil sob a tica do colonizador.
variaes do meio fsico.
b) A histria se passa predominantemente na capital paulista,
c) As falas finais do retirante, aps o nascimento de seu filho,
da porque o livro pode ser considerado uma crnica do
configuram o momento afirmativo, porexcelncia, do poema.
cotidiano paulistano.
d) A viso do mar aberto, quando Severino finalmente chega ao
c) O episdio de base da narrativa consiste na perda e
Recife, representa para o retirante a primeira afirmao da
reconquista da muiraquit.
vida contra a morte.
d) O processo de criao do livro no mantm nenhum vinculo
com qualquer obra anteriormente escrita. 12. (PUCCAMP) O alpinista de alpenstock desceu nos Alpes.
O texto acima, captulo do romance Memrias Sentimentais de
07. (FUVEST) Indique a alternativa em que a proximidade esta- Joo Miramar, exemplifica uma tendncia doautor de:
belecida est correta: a) Procurar as barreiras entre poesia e prosa, utilizando estilo
a) A terra paradisaca, em Gonalves Dias, projeo nacionalista; alusivo e elptico.
a Pasrgada, de Manuel Bandeira, anseiointimista. b) Procurar ser regional e puro em sua poca, negando
b) A figura do malandro, positiva em Manuel Antonio de Almeida, influencias das vanguardas europias.
alvo de Mrio de Andrade em sua stiraMacunama. c) Buscar uma interpretao lrica de seu pas, explorando a
c) Jos de Alencar buscou expressar nossa diversidade cultural- forca sugestiva das palavras.
projeto que s a obra de Machado de Assis viria a realizar d) Explorar o poema em forma de prosa, satirizando as
d) O lirismo de Gregrio de Matos conflitivo e confessional; manifestaes literrias do Pr-modernismo.
o de Cludio Manuel da Costa sereno eimpessoal.
13. (MACKENZIE) Voc, que s faz usufruir e tem mulher para usar
08. (UFSM-RS-adaptada) Assinale a alternativa incorreta a res- ou para exibir, voc vai ver um dia em que toca voc foi bulir. A mulher
peito da poesia de Carlos Drummond de Andrade: foi feita pro amor e pro perdo. Cai nessa, no. Cai nessa, no.
a) O sujeito potico, constantemente, transmite sensaes de (Vincius de Moraes e Toquinho)
dvida e de negao.
b) Importantes poemas publicados na dcada de 1940 tratam Assinale a alternativa correta, de acordo com o trecho acima:
de temas de carter social. a) Usufruir, no texto, significa esbanjar dinheiro.
c) O sujeito potico, vrias vezes, reveste suas expresses de um b) Aproxima tcnicas romnticas das modernas na estruturao
fino trao de humor. do romance como um todo.
d) Os versos que contm uma nfase mstica podem ser vistos c) O importante, na relao amorosa, so as aparncias.
como produtos do fervor catlico do poeta. d) No se deve crer que a mulher sabe apenas amar e perdoar.
14.(PUC-MG) Leia o texto atentamente.
09. (PUCCAMP) Assinale a alternativa em que se encontram
Na feira-livre do arrebaldezinho
preocupaes estticas da Primeira Gerao Modernista:
um homem loquaz apregoa balezinhos de cor:
a) Na exausto causada pelo sentimentalismo, a alma ainda
O melhor divertimento para as crianas!
tremula e ressoante da febre do sangue, a alma que ama
Em redor dele h um ajuntamento de menininhos pobres...
e canta porque sua vida amor e canto, o que pode seno
fazer o poema dos amores da vida real? No caracterstica presente na estrofe acima:
b) Vestir a Ideia de uma forma sensvel que, entretanto, no a) Preocupao social
ter seu fim em si mesma, mas que, servindo para exprimir b) Valorizao de fatos e elementos do cotidiano
a Ideia, dela se tornaria submissa. c) Metalinguagem
c) O poeta deve ter duas qualidades: engenho e juzo; aquele, d) Utilizao do verso livre
subordinado imaginao, este, seu guia, muito mais
importante, decorrente da reflexo. Da no haver beleza sem 15. (MACKENZIE)
obedincia razo, que aponta oobjetivo da arte: a verdade. Cidadezinha qualquer
d) Minhas reivindicaes? Liberdade. Uso dela; no abuso. Casas entre bananeiras
E no quero discpulos. Em arte: escola = imbecilidade de Mulheres entre laranjeiras
muitos para vaidade dum s. Pomar amor cantar

UECEVEST 35

Apostilas UECEVEST mod3.indb 35 06/02/2011 09:57:27


L I T E R ATURA

Um homem vai devagar. H duas espcies de artistas. Uma composta dos que veem as
Um cachorro vai devagar. coisas e em consequncia fazem arte pura, guardados os eternos
Um burro vai devagar. ritmos da vida, e adotados, para a concretizao das emoes estti-
Devagar... as janelas olham. cas, os processos clssicos dos grandes mestres. (...) A outra espcie
Eta vida besta, meu Deus! formada dos que veem anormalmente a natureza e a interpretam
luz das teorias efmeras, sob a sugesto estrbica das escolas re-
Assinale a alternativa incorreta sobre o autor desse poema: beldes, surgidas c e l como furnculos da cultura excessiva. (...).
a) Em A Rosa do Povo expressa a esperana num mundo mais Estas consideraes so provocadas pela exposio da sra. Malfatti,
justo. onde se notam acentuadssimas tendncias para uma atitude estti-
b) Antilirismo e ironia so traos estilsticos de sua poesia. ca forada no sentido das extravagncias de Picasso & cia.
c) O humor, como recurso crtico, uma das caractersticas de
O Dirio de So Paulo, dez./1917.
sua poesia.
d) Destacou-se como poeta da fase herica do Modernismo. Em qual das obras abaixo identifica-se o estilo de Anita Malfatti
criticado por Monteiro Lobato no artigo?

16. Namorados
O rapaz chegou-se para junto da moa e disse:
Antonia, ainda no me acostumei com o seu
[corpo, com a sua cara.
A moa olhou de lado e esperou.
Voc no sabe quando a gente e criana e de
[repente v uma lagarta listrada?
A moa se lembrava: a)
A gente fica olhando... Acesso a Monte Serrat - Santos
A meninice brincou de novo nos olhos dela.
O rapaz prosseguiu com muita doura:
Antonia, voc parece uma lagarta listrada.
A moa arregalou os olhos, fez exclamaes.
O rapaz concluiu:
Antonia, voc e engraada! Voc parece louca.
Manuel Bandeira. Poesia completa & prosa. Rio de Janeiro: Nova Aguilar, 1985.
b)
No poema de Bandeira, importante representante da poesia moder-
Vaso de flores
nista, destaca-se como caracterstica da escola literria dessa poca:
a) a reiterao de palavras como recurso de construo de rimas
ricas.
b) a utilizao expressiva da linguagem falada em situaes do
cotidiano.
c) a criativa simetria de versos para reproduzir o ritmo do tema
abordado.
d) a escolha do tema do amor romntico, caracterizador do
estilo literrio dessa poca.
c)
e) o recurso ao dialogo, gnero discursivo tpico do Realismo.
A Santa Ceia
17. Erro de Portugues
Quando o portugus chegou
Debaixo de uma bruta chuva
Vestiu o ndio
Que pena!
Fosse uma manha de Sol
O ndio tinha despido
O portugus.
Oswald de Andrade. Poesias reunidas. Rio de Janeiro: Civilizao Brasileira, 1978. d)
O primitivismo observvel no poema acima, de Oswald de An- Nossa Senhora Auxiliadora e Dom Bosco
drade, caracteriza de forma marcante:
a) o regionalismo do Nordeste.
b) o concretismo paulista.
c) a poesia Pau-Brasil.
d) o simbolismo pr-modernista.
e) o tropicalismo baiano.

18. Sobre a exposio de Anita Malfatti, em 1917, que muito


influenciaria a Semana de Arte Moderna, Monteiro Lobato escre-
veu, em artigo intitulado Paranoia ou Mistificao: e)
A Boba

36 UECEVEST

02 LITERATURA.indd 36 06/02/2011 10:10:10


LIT E R AT U R A

19. e) usar as sombras em tons de cinza e preto e com efeitos


esfumaados, tal como eram realizadas no Renascimento.

21. Todas as manhs quando acordo, experimento um prazer


supremo: o de ser Salvador Dal.
NRET, G. Salvador Dal. Taschen, 1996.

Assim escreveu o pintor dos relgios moles e das girafas em


chamas em 1931. Esse artista excntrico deu apoio ao general
Franco durante a Guerra Civil Espanhola e, por esse motivo, foi
afastado do movimento surrealista por seu lder, Andr Breton.
Antonio Rocco. Os imigrantes, 1910, Pinacoteca do Estado de So Paulo.
Dessa forma, Dal criou seu prprio estilo, baseado na interpre-
Um dia, os imigrantes aglomerados na amurada da proa che- tao dos sonhos e nos estudos de Sigmund Freud, denominado
gavam fedentina quente de um porto, num silncio de mato mtodo de interpretao paranoico. Esse mtodo era constitu-
e de febre amarela. Santos. aqui! Buenos Aires aqui! do por textos visuais que demonstram imagens:
Tinham trocado o rtulo das bagagens, desciam em fila. Faziam a) do fantstico, impregnado de civismo pelo governo
suas necessidades nos trens dos animais onde iam. Jogavam-nos espanhol, em que a busca pela emoo e pela dramaticidade
num pavilho comum em So Paulo. Buenos Aires aqui! desenvolveram um estilo incomparvel.
Amontoados com trouxas, sanfonas e bas, num carro de bois, b) do onrico, que misturava sonho com realidade e interagia
que pretos guiavam atravs do mato por estradas esburacadas, refletindo a unidade entre o consciente e o inconsciente
chegavam uma tarde nas senzalas donde acabava de sair o brao como um universo nico ou pessoal.
escravo. Formavam militarmente nas madrugadas do terreiro ho- c) da linha inflexvel da razo, dando vazo a uma forma de
mens e mulheres, ante feitores de espingarda ao ombro. produo despojada no trao, na temtica e nas formas
Oswald de Andrade. Marco Zero II Cho. Rio de Janeiro: Globo, 1991. vinculadas ao real.
d) do reflexo que, apesar do termo paranoico, possui
Levando-se em considerao o texto de Oswald de Andrade e a sobriedade e elegncia advindas de uma tcnica de cores
pintura de Antonio Rocco reproduzida acima, relativos imigra- discretas e desenhos precisos.
o europeia para o Brasil, correto afirmar que: e) da expresso e intensidade entre o consciente e a liberdade,
a) a viso da imigrao presente na pintura trgica e, no declarando o amor pela forma de conduzir o enredo
texto, otimista. histrico dos personagens retratados.
b) a pintura confirma a viso do texto quanto imigrao de
argentinos para o Brasil. 22. Negrinha
c) os dois autores retratam dificuldades dos imigrantes na Negrinha era uma pobre rf de sete anos. Preta? No; fusca,
chegada ao Brasil. mulatinha escura, de cabelos ruos e olhos assustados.
d) Antonio Rocco retrata de forma otimista a imigrao, Nascera na senzala, de me escrava, e seus primeiros anos vive-
destacando o pioneirismo do imigrante. ra-os pelos cantos escuros da cozinha, sobre velha esteira e trapos
e) Oswald de Andrade mostra que a condio de vida do imundos. Sempre escondida, que a patroa no gostava de crianas.
imigrante era melhor que a dos ex-escravos. Excelente senhora, a patroa. Gorda, rica, dona do mundo,
amimada dos padres, com lugar certo na igreja e camarote de
20. luxo reservado no cu. Entaladas as banhas no trono (uma cadei-
ra de balano na sala de jantar), ali bordava, recebia as amigas e
o vigrio, dando audincias, discutindo o tempo. Uma virtuosa
senhora em suma dama de grandes virtudes apostlicas, esteio
da religio e da moral, dizia o reverendo.
tima, a dona Incia.
Mas no admitia choro de criana. Ai! Punha-lhe os nervos
em carne viva.
[...]
A excelente dona Incia era mestra na arte de judiar de crian-
as. Vinha da escravido, fora senhora de escravos e daquelas
ferozes, amigas de ouvir cantar o bolo e estalar o bacalhau. Nunca
se afizera ao regime novo essa indecncia de negro igual.
MONET,C. Mulher com sombrinha. 1875, 100x81cm. In: BECKETT, W. LOBATO, M. Negrinha. In: MORICONE, I. Os cem melhores contos
Histria da Pintura. So Paulo: tica, 1997. brasileiros do sculo. Rio de Janeiro: Objetiva, 2000 (fragmento).
Em busca de maior naturalismo em suas obras e fundamentando- A narrativa focaliza um momento histrico-social de valores con-
se em novo conceito esttico, Monet, Degas, Renoir e outros ar- traditrios. Essa contradio infere-se, no contexto, pela:
tistas passaram a explorar novas formas de composio artstica, a) falta de aproximao entre a menina e a senhora, preocupada
que resultaram no estilo denominado Impressionismo. Observa- com as amigas.
dores atentos da natureza, esses artistas passaram a: b) receptividade da senhora para com os padres, mas
a) retratar, em suas obras, as cores que idealizavam de acordo deselegante para com as beatas.
com o reflexo da luz solar nos objetos. c) ironia do padre a respeito da senhora, que era perversa com
b) usar mais a cor preta, fazendo contornos ntidos, que melhor as crianas.
definiam as imagens e as cores do objeto representado. d) resistncia da senhora em aceitar a liberdade dos negros
c) retratar paisagens em diferentes horas do dia, recriando, em evidenciada no texto.
suas telas, as imagens por eles idealizadas. e) rejeio aos criados por parte da senhora, que preferia trat-
d) usar pinceladas rpidas de cores puras e dissociadas los com castigos.
diretamente na tela, sem mistur-las antes na paleta.

UECEVEST 37

Apostilas UECEVEST mod3.indb 37 06/02/2011 09:57:28


L I T E R ATURA

23. Aps estudar na Europa, Anita Malfatti retornou ao Brasil com REFERnCIAS BIBlIOGRFICAS
uma mostra que abalou a cultura nacional do incio do sculo XX.
Elogiada por seus mestres na Europa, Anita se considerava pronta BOSI, Alfredo. Histria Concisa da Literatura Brasileira
para mostrar seu trabalho no Brasil, mas enfrentou as duras crticas 46. ed. So Paulo: Cultriz, 2006.
de Monteiro Lobato. Com a inteno de criar uma arte que valori-
zasse a cultura brasileira, Anita Malfatti e outros artistas modernistas: CAMPEDELLI, Samira Yousse & SOUZA, Jsus Barbosa.
a) buscaram libertar a arte brasileira das normas acadmicas Literaturas: brasileira e portuguesa: teoria e texto: volume
europeias, valorizando as cores, a originalidade e os temas nico So Paulo: Saraiva, 2003.
nacionais.
b) defenderam a liberdade limitada de uso da cor, at ento CEREJA, Willian Roberto & MAGALHES, Thereza Cochar.
utilizada de forma irrestrita, afetando a criao artstica nacional. Literatura brasileira: ensino mdio 2. ed. reform. So
c) representaram a ideia de que a arte deveria copiar fielmente Paulo: Atual, 2000.
a natureza, tendo como finalidade a prtica educativa.
d) mantiveram de forma fiel a realidade nas figuras retratadas, CHIAPPINI, Lgia. Invaso da catedral. Literatura e ensino
defendendo uma liberdade artstica ligada tradio em debate. Porto Alegre: Mercado Aberto,1983.
acadmica.
e) buscaram a liberdade na composio de suas figuras,
respeitando limites de temas abordados. COMISSO COORDENADORA DO VESTIBULAR(CCV)
www.ufc.br
24. Texto I
Eu amo a rua. Esse sentimento de natureza toda ntima no vos COMISSO EXECUTIVA DO VESTIBULAR(CEV) www.
seria revelado por mim se no julgasse, e razes no tivesse para julgar, uece.br
que este amor assim absoluto e assim exagerado partilhado por to-
dos vs. Ns somos irmos, ns nos sentimos parecidos e iguais; nas COUTINHO, Afrnio A literatura no Brasil, vol. 2. 3 e.d
cidades, nas aldeias, nos povoados, no porque soframos, com a dor e So Paulo, 1986.
os desprazeres, a lei e a polcia, mas porque nos une, nivela e agremia
o amor da rua. este mesmo o sentimento imperturbvel e indisso- GARCIA, O. M. Comunicao em prosa moderna. Rio de
lvel, o nico que, como a prpria vida, resiste s idades e s pocas. Janeiro: Fundao Getlio Vargas, 1973.
RIO, J. A rua. In: A alma encantadora das ruas. So Paulo: Companhia das
Letras, 2008 (fragmento). LAJOLO, Marisa. Leitura-literatura: mais do que uma rima,
menos do que uma soluo. In:
Texto II MACHADO, Ana Maria. Entrevista. Revista Nova Escola. So
A rua dava-lhe uma fora de fisionomia, mais conscincia Paulo. Editora Abril, setembro de 2001.
dela. Como se sentia estar no seu reino, na regio em que era
rainha e imperatriz. O olhar cobioso dos homens e o de inveja MEC - Ministrio da Educao e Cultura INEP Instituto
das mulheres acabavam o sentimento de sua personalidade, exal- Nacional de Estudos e Pesquisas Educacionais Ansio Teixeira
tavam-no at. Dirigiu-se para a rua do Catete com o seu passo Matrizes de Referncia para o ENEM
mido e slido. [...] No caminho trocou cumprimento com as
raparigas pobres de uma casa de cmodos da vizinhana. MOISS, Massaud. A Anlise literria - 1. ed .16 reimpr.
[...] E debaixo dos olhares maravilhados das pobres raparigas, So Paulo: Cultrix, 2007
ela continuou o seu caminho, arrepanhando a saia, satisfeita que
nem uma duquesa atravessando os seus domnios.
MOISS, Massaud. Literatura brasileira atravs dos textos -
BARRETO, L. Um e outro. In: Clara dos anjos. Rio de Janeiro: Editora Mrito
6. ed . So Paulo: Cultrix, 1978.
(fragmento).

A experincia urbana um tema recorrente em crnicas, contos NICOLA, Jos de. Literatura brasileira: das origens aos
e romances do final do sculo XIX e incio do XX, muitos dos nossos dias. So Paulo: Scipione,1998.
quais elegem a rua para explorar essa experincia. Nos fragmentos
I e II, a rua vista, respectivamente, como lugar que: ZILBERMAN & SILVA (org.) Leitura: perspectivas
a) desperta sensaes contraditrias e desejo de reconhecimento. interdisciplinares. So Paulo: tica, 1991.
b) favorece o cultivo da intimidade e a exposio dos dotes fsicos.
c) possibilita vnculos pessoais duradouros e encontros casuais. http://www.moderna.com.br/moderna/didaticos/em/literatura/
d) propicia o sentido de comunidade e a exibio pessoal. litbrasil/vestibular/index_html
e) promove o anonimato e a segregao social.

G A B A R I T O
01. d 02. d 03. a 04. d 05. c
06. c 07. a 08. d 09. d 10. c
11. b 12. a 13. d 14. c 15. d
16. b 17. c 18. e 19. c 20. d
21. b 22. d 23. a 24. d

38 UECEVEST

Apostilas UECEVEST mod3.indb 38 06/02/2011 09:57:29


P R - V E S T I B U l A R

REDAO

Apostilas UECEVEST mod3.indb 39 06/02/2011 09:57:35


Apostilas UECEVEST mod3.indb 40 06/02/2011 09:57:35
R E DA O

O TEXTO DESCRITIVO Olfativos: Nas barcas, os armazns tresandavam a lixo e peixe


podre, as latas vazias de leo, com o cheiro de homens esfarra-
Costuma-se dizer que a descrio uma pintura com pala- pados. (Autran Dourado, A barca dos homens.)
vras. Portanto, deve ressaltar as formas materiais ou geomtricas, Tcteis: O pai comprou o sapato dos dois nmeros maiores.
os aspectos cromticos a cor, a luz, a sombra as dimenses do Enfiou no p o sapato branco de tnis. Ao pentear-lhe o loiro
espao e suas perspectivas, o relativo. Com a descrio, faz-se um cabelo, a cabea ainda em fogo. (Dalton Trevisan, Pedrinho).
desenho dos objetos, pessoas, lugares, em um determinado estado Gustativos: Deitado , ele beliscou dois ou trs gros. Chupou
e durante um tempo considerado. o sumo azedo, deixou cair a casca no prato. Apanhou outro
Na descrio, deve-se recorrer a adjetivos, locues adjetivas, bago, mais doce. (Dalton Trevisan, As uvas).
verbos de estado (no presente e no imperfeito do indicativo) e
oraes adjetivas, s vezes para fazer comparaes. A partir das impresses acima, apresente os principais ele-
Apresenta caractersticas de linguagem denotativa (tcnica/ mentos a serem utilizados nas propostas de descries abaixo:
objetiva) ou conotativa(potica). 01. O seu primeiro dia na universidade:

E X E R C C I O
Veja o texto abaixo e, em seguida, marque os elementos coesivos,
alm de ordenar as ideias de forma coerente:
1. Ela cercada por vrios galhos secos e uma grande
mangueira
2. O porto da casa de madeira do Par e, devido ao tempo,
j est corrodo
3. O banheiro no grande, tem um sanitrio e um pequeno 02. Um acontecimento inesquecvel:
tanque que serve como depsito de gua potvel
4. A cozinha espaosa: tem fogo, vrias cadeiras, uma
geladeira que foi comprada em uma loja de mveis
5. A frente amarela e a porta tem um tom cinza, bem neutro
6. A minha casa, como vimos, tem uma sala, um quarto, uma
cozinha, um banheiro e um quintal
7. Entre o porto e a porta que d acesso a casa, existem alguns
bancos de cimento, colocados em pontos estratgicos do jardim
8. Na sala, as paredes so pintadas com uma textura azul, tem
um sof macio, um aqurio bastante colorido, um ambiente
03. O produto capaz de acabar com os altos ndices de obesidade:
gostoso, bom para receber visitas
9. A minha casa, apesar de pequena, um ambiente gostoso
10. O quarto bem arejado, fica do lado da sombra, d para o
nascente. Nele h cama, armrio , uma estante com livros
diversificados e muitos cds
11. O quintal grande, tem vrias rvores frutferas, um antigo
catavento dos tempos do meu av e um pilo que ainda
muito til
R. A sequncia certa das frases em ordem coerente :
( )( )( )( )( )( )( )( )( )( )( )
Veja se as caractersticas dos textos abaixo facilitam a percep-
ANOTAES o do que est sendo descrito:

Texto 01
Sou quadrado, s vezes retangular, outras vezes me pareo
com um grampeador.
Sou um sdito de meus reis.
Tomo conta do trono, que no de madeira nem de pedra.
Um trono de vrios reis que me usam para afogar seus inimi-
Observe os elementos sensitivos presentes nos textos abaixo: gos parasitas em leito contaminado.
Visuais: A dona era uma velha balofa, de peruca mais negra So jogados da mais alta nuvem at as guas, onde eu tomo
do que a asa da grana. Vestia um pijama desbotado e de seda minha posio de provocar a enchente e a vazo das guas.
japonesa e tinha unhas recobertas por uma crosta de esmalte Sou orgulhoso de meu servio, pois, sem mim, o que seria do
vermelho-escuro, descascado nas pontas encardidas. (Ligya Fa- aroma do meu reino?
gundes Telles, As formigas) Visto minha armadura niquelada, s vezes de bronze, e, com
Auditivos: De uma mesa distante, a nica ocupada ainda, vi- um simples toque de meus superiores, cumpro minha tarefa de
nha o rudo de vozes de homens. Uma gargalhada rebentou carrasco.
sonora em meio de vozes exaltadas. E a palavra cabrito saltou
dentre as outras que se arrastavam pastosas. Num rdio da vi- O que est sendo descrito? ____________________________
zinhana, ligado ao volume mximo, havia uma cano que
contava uma histria de uma violeteira vendendo violetas na Texto 02
porta de um teatro. A voz da cantora era plana e um pouco Chato, no real sentido da palavra, este objeto pode ter vrios
fanhosa.. (Lygia Fagundes Telles, A ceia). cheiros, cheiros artificiais, qumicos, novos, ou ento o que de

UECEVEST 41

Apostilas UECEVEST mod3.indb 41 06/02/2011 09:57:35


R E DA O

que mais gosto, aromas mgicos, assim como bolor, poeira, uma COMENTRIO
poeira de ideias, poeira fina que gostosa de se tirar com os dedos Repare que, neste poema, a autora compe um retrato de si
vendo a cor aparecer aos poucos, ntida, convidativa. mesma. Vamos examinar esse retrato mais de perto, para ver-
Para se apreciar esse objeto, preciso tempo, abri-lo devagar, mos os elementos que ela selecionou na descrio e como eles
ir consumindo lenta e preguiosamente, at que o apetite se torne foram considerados.
voraz e seja impossvel larg-lo. Observe na primeira estrofe que os elementos destacados so:
Quando consumido at o fim, ele se renova e ganha uma rosto, olhos e lbio. Mas dos adjetivos que modificam esses subs-
conotao saudosa, de tesouro bem guardado, do qual agente as tantivos apenas um indica caracterstica fsica: magro. Todos os
vezes se lembra e corre pra ver se est no mesmo lugar, se nos diz outros referem-se ao estado interior da autora: calmo, triste, vazios
a mesma coisa. e amargo. Assim, vemos que o retrato que est sendo composto
O que est sendo descrito? ____________________________ representa principalmente o estado de esprito da autora e no
seu estado fsico.
Texto 03 Na Segunda estrofe, ela comea a descrever as mos, quali-
Na maioria das vezes , tenho a forma de tijolo. Claro que no ficando-se de sem fora, paradas, frias e mortas. Percebe-se nova-
sou to grande quanto ele. s vezes pareo um envelope, mas mente que a autora, com esse processo de adjetivao, vai alm
quando tenho esta forma no posso ser comprado. das aparncias, revelando por meio das mos o seu estado de es-
Sou uma casa, com muitos habitantes bastantes magros e de prito. Dessa maneira, revendo os elementos descritos nas duas
cabelos negros. primeiras estrofes (rosto, olhos, lbios e mos), percebemos que eles
Geralmente, sou escura e em uma de minhas paredes externas funcionam como smbolo do mundo interior da autora, que se
possuo um painel, colorido, cobrindo-me inteirinha. Em outras revela totalmente desesperanada. E, no fim da segunda estrofe, o
duas paredes, possuo algo parecido com lixas, que so verdadeiras ltimo elemento escolhido para descrio o corao acaba por
armas contra meus habitantes. Se esbarram algo com fora nelas, mostrar o verdadeiro objetivo da autora: compor um retrato de
eles, morrem, exalando um cheiro caracterstico. seu [intimo, do seu eu interior e no de seus traos fsicos. Ao
No custo caro. Alias, acho quase impossvel viver sem minha falar que tem um corao/que nem se mostra, a autora refora a
presena. Sou muito til em qualquer lugar, em qualquer parte ideia de desesperana que marca os versos anteriores, pois sugere
do mundo, mas ningum me d o devido valor: cada habitante que est fechada para a vida.
meu que morre jogado fora, displicentemente. Na ltima estrofe, a descrio interrompida para dar lugar
O objeto descrito : _________________________________ a um momento de reflexo da autora: diante desse retrato, que
mostra a mudana ocorrida em seu intimo, s lhe resta perguntar
Texto 04 ansiosamente pela sua verdadeira face, pois no se reconhece mais.
quadrado, bem pesado, parece frio e quando encosto minha
mo nele, percebo que gelado. ANOTAES
Na sua frente h uma roda que d a impresso de ser um
soldado guardando um castelo. No emite sons, no tem cheiro.
Parece morto, ou melhor, parece um caixo de defunto. No
tem habitat fixo, mas pode ficar em qualquer lugar onde a ambi-
o pode chegar. Alguns tm ps prprios, outros esto suporta-
dos atrs de figuras, as mais diversas possveis.
No se movimenta, totalmente esttico. Apenas seus braos
em roda se movimentam e, quando o fazem, para abrir seu
prprio corpo. Dentro dele h um sangue verde coagulado, mis-
turado com pedras e segredos.
O que est sendo descrito? ____________________________
Descrio de Nair
Quando a vi pela primeira vez, em um fim de tarde do ms de
junho, ela estava em uma sala prxima s dependncias do depar-
tamento de ginstica olmpica de um clube de So Paulo. Sentada
DESCRIO DE PESSOAS E AMBIENTES em seu colo havia uma menina loira, de aproximadamente dez
anos, que ela criara no desempenho das suas funes de emprega-
Leia este texto de Ceclia Meireles: da domstica daquela famlia por quase vinte anos.
Retrato De baixa estatura, mais gorda do que magra, aparentava Ter
Eu no tinha este rosto de hoje trinta e quarenta anos. A cor negra de sua pele falava-me muito
assim calmo, assim triste, assim magro, de perto da herana involuntariamente legada plos seus antepas-
nem estes olhos to vazios, sados herana de trabalho rduo na condio servil que lhes foi
nem o lbio amargo. imposta, atravs das geraes, pelas condies histricas e socio-
econmicas.
Eu no tinha estas mos sem fora, Os olhos de Nair, negros e arredondados, refletiam amor e
to paradas e frias e mortas; preocupao por aquela criana sentada em seu colo. Os outros
eu no tinha este corao traos do rosto configuravam uma fisionomia bastante simptica.
que nem se mostra. Seus lbios grossos proferiam frases inteligentes, que demonstra-
vam vivacidade e esperteza, atributos que preconceituosamente
Eu no dei por esta mudana, no costumam ser associados a algum de sua profisso.
to simples, to certa, to fcil; Nair vestia roupas simples, assim como eram simples as suas
Em que espelho ficou perdida aspiraes: continuar cuidando de uma casa que no era sua e
a minha face? de uma filha que tambm no era sua, mas de quem se julgava
(in Poesia, Rio de Janeiro, Agir, 1974, p. 19). amada. Parecia ser um exemplo claro daqueles que abrem mo

42 UECEVEST

Apostilas UECEVEST mod3.indb 42 06/02/2011 09:57:36


R E DA O

do direito de seguir seu prprio destino para viver em funo Comente o texto abaixo:
de outras pessoas, canalizando para elas toda a sua sensibilidade. Um ar buclico envolvia a cena que se passava por trs da ja-
Leal, sincera, de personalidade forte, no sei se, no ntimo, nela do meu quarto. Aquela praa sempre esteve l, mas eu nunca
desejava Ter seus prprios filhos, um outro modo de viver. Muito havia parado para observ-la.
provavelmente, sim. A copa das rvores balanava ao menor toque do vento fazen-
do com que o lugar tivesse um som harmonioso, uma melodia que
ANOTAES transmitia s pessoas dali uma sensao de vitalidade renovada.
Era possvel ver a simplicidade mpar de um casal de andorinhas a
voar suavemente pelo cu e, de forma graciosa, pousar nas folhas
altas dos benjamins. Os bancos eram olhos e mostravam, atravs
de pequenos rabiscos, quantas histrias de amor nasceram e cres-
ceram ali, pois os casais que l se encontravam faziam questo
de ver suas iniciais com pequenas dedicatrias ao lado de tantas
outras. A fonte, que chamava a ateno por ser esculpida numa
pedra brilhante, dava um toque potico arcadista ao lugar, como
no bastassem as flores brancas e cor-de-rosa que coloriam sin-
Descrio de ambientes gelamente os cantos ao redor dos bancos. O piso era desenhado
Nem sempre, porm, o objetivo de um escritor registrar com figuras abstratas no cinza de pedra refletiam fracamente o sol
tudo como se fosse uma mquina fotogrfica. Ele pode descrever tmido, que aparecia no cu azul, pincelado do branco das nuvens
determinado ambiente de maneira bem pessoal, permitindo que que se moviam devagar, como se quisessem deixar aquela cena.
seus sentimentos e experincias penetrem no texto. Voc enten- Aps tal contemplao, ainda com os braos apoiados no pa-
der melhor a diferena desse tipo de descrio lendo este texto rapeito da janela, fechei os olhos e imaginei um mundo maravi-
de Rubem Braga. lhoso, no qual a praa era meu modelo.
(UECEVEST)
Veja o texto cujo objetivo descrever um quarto:
Algum me fala do apartamento em que voc morou em Paris, COMENTRIOS
em uma pequena praa cheia de rvores; outra pessoa esteve em
sua casa de Npoles; eu me calo. Mas, eu conheci seu quarto de
solteira. Era pequeno, gracioso e azul; ou a distancia que o azula
na minha lembrana? Junto janela, havia uma grande amen-
doeira antiga; s vezes, o vento levava para dentro uma grande
folha cor de cobre gentileza da amendoeira. Que tinha outras:
pssaros, quase sempre pardais, s vezes um tico-tico, ou uma
rolinha, ou um casal de sanhaos azulados. E no vero como as
cigarras ziniam! Lembro o armrio escuro e simples, onde cabiam
os vestidos de solteira, que no eram muitos; e lembro alguns
deles, um roxinho singelo, um estampado alegre, de flores, um
outro de linho grosso, cor de areia. Havia uma pequena estante e,
entre os livros, o meu primeiro livro com uma dedicatria tmida.
TEXTO PUBlICITRIO
OBSERVAAO:
Voc deve ter observado que o autor descreve os aspectos fsi- O objetivo principal desse gnero promover um produto e
cos do quarto: era pequeno, tinha uma janela, um armrio escuro conquistar o leitor a consumi-lo. Para esse fim, o publicitrio se
e simples, onde ficavam os vestidos, uma pequena estante etc. No utiliza de argumentos. Por isso que o texto publicitrio faz parte
entanto, esse quarto descrito de um ngulo bem pessoal, pois do grupo de textos que utilizam argumentos de persuaso.
fruto de lembranas afetivas do autor, que projeta no texto seus Esses argumentos esto diretamente relacionados s supostas van-
sentimentos. Observe, por exemplo, esta exclamao: E, no vero, tagens que o leitor vai adquirir ao comprar um produto. Essas van-
como as cigarras ziniam! Ela mostra muito bem o envolvimen- tagens podem ser de ordem quantitativa ( mais barato que a con-
to do autor com o ambiente que descreve, pois sugere alegria e corrncia, rende mais), qualitativa ( mais bonito, delicioso)
saudade ao mesmo tempo. A referncia gentileza da amendo- e ideolgica (100% reciclvel, no prejudica o meio-ambiente).
eira que levava para dentro do quarto folhas e pssaros, outro Quanto estrutura do texto publicitrio, a linguagem con-
exemplo da interferncia das recordaes saudosas do autor no cisa, direta e, principalmente, persuasiva, a fim de atingir rapida-
ambiente que est descrevendo. Na verdade, esses detalhes nada mente o interlocutor. Frequentemente apresenta verbos no modo
acrescentam representao puramente material do quarto, mas Imperativo (corra, adquira j o seu!, no perca tempo) .
o modo pelo qual o autor comunica ao leitor algo que est alm O texto que segue como exemplo do tipo publicitrio.
das aparncias o clima potico que havia naquele ambiente.
Estrura do texto publicitrio
ANOTAES Introduo chamada inicial(necessidade do produto,
curiosiade sobre ele)
Desenvolvimento corpo do texto (vantagens/funes, uso)
Concluso convite compra ou aceitao

Veja os textos:
Texto 01
Se voc no aguenta mais as comuns faltas de energia em sua
casa, se o Governo todo dia fala sobre o programa de raciona-

UECEVEST 43

Apostilas UECEVEST mod3.indb 43 06/02/2011 09:57:37


R E DA O

mento residencial de energia eltrica. Chegou a soluo para seus Com Bandeira da Silva nosso estado ser livre. Mas voc deve
problemas: Velax. pensar: a liberdade to importante assim para mim? Voc j
A Velax produz velas com funes e qualidades inigualveis. parou para pensar se realmente livre? Como considerar-se livre
Alm de garantir uma maior luminosidade em sua casa, afastam se sua casa j uma priso, e voc vive cercado de muros altos e
qualquer tipo de insetos como baratas, besouros, inclusive, o que grades nas janelas? Na rua, voc tambm no livre, caminha
transmite o dengue. So usadas tambm contra germes nocivos com medo de ser assaltado ou violentado, ou dirige com os vidros
sade. Nos dias de frio, elas podem aquecer qualquer ambiente e, fechados, aprisionado.
alm disso, podem ser utilizadas nas terapias relaxantes contra o Cabe a voc mudar essa realidade, com seu voto, com sua
estresse. A Velax tambm est presente na ornamentao do seu confiana. Vote em Bandeira da Silva pela liberdade do cidado.
espao, pois temos velas de vrias cores e tamanhos, no estilo de Se voc no se lembra do que Bandeira da Silva fez no passado
sua preferncia. Oferecemos tambm as aromatizadas, com vrios em nosso Estado, como educador(a favor do aumento de ver-
tipos de fragrncia: ctricas, doces e afrodisacas. Produzimos in- bas para a merenda escolar; como vereador(aprovou a lei da meia
clusive para serem utilizadas em cerimnias religiosas e espirituais. passagem para estudantes), como deputado(criou leis a favor do
Adquira j o seu produto, Velax pode ser encontrada nas me- agricultor), deixe pelo menos ele se lembrar de voc.
lhores lojas de sua cidade e na quantidade que voc desejar. Ele O dia seis de outubro ser o dia da liberdade, da sua liberda-
a soluo. de. Portanto, faa a diferena, vote consciente, Bandeira da Silva
(UECEVEST) para o governo do Estado. Conhea mais as propostas do seu
candidato no comit da Av. Paranjana, 1700-Itaperi, e engaje-se
ANOTAES nessa luta, que do povo em busca da realizao dos seus sonhos.

E X E R C C I O
A respeito do texto ACIMA, responda:
01. Qual o gnero de texto lido?

Texto 02 02. Qual o objetivo proposto pelo texto?


Voc se sente frio e indiferente com a desgraa alheia? A mis-
ria estampada nos olhos baos de um pobre menino no sinal no
o comove mais? Voc acha piegas um casal apaixonado namoran-
do na pracinha? Voc precisa da mquina dos sentimentos, o 03. A que tipo textual pertence o gnero? Justifique.
mais moderno equipamento sentimental de ltima gerao, per- a) ( ) Dissertativo b) ( ) Descritivo c) ( ) Narrativo
feito para os dias de hoje, em que o materialismo fala mais alto.
Essa engenhoca ligada ao corao e voc carrega para onde
vai. A partir da, ela passa a control-lo sempre que necessrio.
Voc nunca mais vai reclamar de solido, pois viver apaixonado. 04. A linguagem utilizada:
Quando um mendigo lhe pedir uma esmola, por exemplo, auto- a) culta formal
maticamente a mquina entra em funcionamento, e voc acabar b) denotativa
dando uma gorda ajuda. Com a mquina voc viver cercado de c) culta informal
amigos; vai sorrir sempre que vir uma criana e chorar com o nas- d) conotativa
cer do sol Voc se emocionar ouvindo a Nona de Beethowen ou
assistindo a um casamento no ltimo captulo da novela das oito. 05. Que outras caractersticas complementam a estrutura desse
Enfim, meu amigo de corao petrificado pela vida, voc se gnero textual?
tornar mais humano, mais vivo e depois de experimentar o nos-
so equipamento voc entender que ele a soluo para todos os
problemas da humanidade, que s precisa de mais amor.
(UECEVEST)

ANOTAES

RECEITA

Para se fazer uma boa administrao em qualquer grande ca-


pital brasileira h a necessidade de se usar responsabilidade com o
dinheiro pblico, seriedade nos processos de licitao para a reali-
zao de obras e transparncia na prestao de servios sociedade.
Texto 03
Para isso junte a responsabilidade de administrar o dinheiro
ano de eleies, tempo de mudar nosso Estado, nosso pas,
da populao, empregando em atividades essenciais como educa-
nossas vidas, mas ano tambm de muitas promessas, de muitos
o e sade, com a construo de escolas de qualidade e postos de
comcios, de muita enganao. Por isso procure um candidato
sade na periferia, com atendimento 24 horas; alm disso, utilize
honesto, capaz de realizar, no apenas de falar que faz. Afinal,
uma boa dose de seriedade nos processos de licitao, fiscalizando
atravs do voto que conquistamos a liberdade. Para o governo
a apresentao das propostas de empresas prestadoras de servios e
do Estado, busque um candidato que abrace essa causa, vote em
o resultado desses processos com o acompanhamento da realizao
Bandeira da Silva, o defensor da liberdade para o cidado.
das atividades, a partir da qualidade dos servios prestados; some-se

44 UECEVEST

Apostilas UECEVEST mod3.indb 44 06/02/2011 09:57:37


R E DA O

a esses ingredientes, a prestao de contas pelos servios realizados do prefeito, 90 comprimidos; deputado tomar 120; governador,
e o emprego dos impostos pagos e quais os benefcios que sero re- 150; ministro, 180; senador, 210; juzes, 240; empresrios, 270;
tornados a essa populao em melhoria da qualidade de vida como presidentes de tribunais, 290; presidente da nao, 365. Em to-
pavimentao, saneamento e qualificao da mo de obra. dos os casos, persistindo a doena, triplicar a dosagem.
Com isso, constataremos uma administrao de qualidade,
fiel credibilidade depositada pela sociedade e receberemos os ATENO
benefcios produzidos nas principais reas sociais. Isso implicar Tanto a Receita como a Bula apresentam-se na nossa sociedade
em grandes capitais produzindo a seriedade que tanto faz falta no com um formato de texto corrido (texto em prosa), mas sim,
nas prefeituras de todo o pas, em sua maioria envoltas em situa- na maioria dos casos, organizadas em tpicos. A proposta agora
es de corrupo e falcatruas. produzir textos desses gneros considerando as caractersticas de
um texto em prosa padro: respeitando os pargrafos, a linguagem
Gnero textual Receita adequada ao contexto, a pontuao apropriada, entre outros.
Texto com predominncia descritiva
Caracteriza-se como um instrumento de composio, Texto-sugesto
prescrio, preparao e indicao na utilizao de um Uma doena muito comum tem afetado pessoas de diferentes
determinado produto para atingir um fim proposto. classes sociais como polticos, juzes, advogados, dentre outros: a
Utiliza, frequentemente, linguagem de imposio do produto corrupo. Para combate-la tome Corruptol, uma caixa com 365
(verbos no modo imperativo) comprimidos em que cada um deles contm honestidade, sinceri-
Os itens utilizados nesse gnero textual criam espao para que o dade, personalidade e carter, o que vai acabar de vez com esse mal.
produtor possa apresentar conhecimento de mundo e posio Esse medicamento dever ser conservado em lugar fresco e ao
crtica a respeito do tema. abrigo da luz e no poder ser utilizado com prazo de validade
A prescrio um recurso necessrio produo do texto que vencido. O Corruptol dever ser dissolvido em 200ml de gua
prima pela apresentao de indicativos(ingredientes) para e no poder ser mastigado ou ingerido com outros lquidos. Ele
atingir um fim. ser utilizado para combater os efeitos de doenas como o desvio
de verba, oferecimento de propinas ou subornos(como os veri-
Comente as caractersticas no texto lido: ficados nos rgos de proteo ambiental superfaturamento em
compras e servios(como exemplo a merenda escolar), lavagem
de dinheiro e outros.
indicado para pacientes com corrupo, sendo eles eleitores,
vereadores, prefeitos e at mesmo presidentes. No um remdio
apenas para polticos corruptos, mas para pessoas corrompidas
em todos os nveis e setores. Em alguns casos de estgio avanado
da doena, os efeitos podem ser catastrficos, levando o paciente
loucura por no haver como inverter o quadro, como os casos
de fiscais de rgos pblicos j investigados e presos.
O uso deve ser oral, com unidade diria, e as dosagens variam
BUlA DE REmDIO de acordo com o cargo e o estado do paciente. Para eleitor, 30
comprimidos; para vereador, 60 cpsulas; prefeito: 90 compri-
Veja os elementos que compem a Bula e, a partir dessa ob- midos; deputado: 120 drgeas; governador: 150 comprimidos;
servao, construa a sua prpria Bula: ministro: 180 cpsulas; presidente da nao: 365 comprimidos.
Nome: Corruptol. Em todos os casos, persistindo a doena, triplicar a dosagem. Esse
Apresentao: caixa com 365 comprimidos. medicamento dever ser mantido fora do alcance das crianas.
Composio: cada comprimido contm 500mg de honesti-
dade, 200mg de sinceridade, 100mg de vergonha na cara, 200mg
de personalidade e 100mg de carter. E X E R C C I O
Informaes ao paciente: conservar em lugar fresco e ao
abrigo da lua. No use remdio com prazo de validade vencido. 01. Explique, com suas palavras, por que os textos que voc aca-
O Corruptol deve ser dissolvido em 200 ml de gua e no pode bou de ler so considerados TEXTOS INSTRUCIONAIS.
ser mastigado ou ingerido com outros lquidos.
Indicaes: indicado para pacientes com corrupo, sendo
eles eleitores, vereadores, prefeitos e at mesmo Presidente da Re-
pblica. No um remdio apenas para polticos corruptos, mas
para pessoas corrompidas desde as pequenas representaes aos
cargos majoritrios em todos os nveis e setores. 02. Qual a inteno comum Receita e Bula de Remdio?
Informaes tcnicas: para combater os efeitos da doena
como desvio de verba, oferecimento de propinas ou subornos,
superfaturamento em compras e servios, lavagem de dinheiro
e outros.
Reaes adversas: em alguns casos, com o estagio da doena 03. O que necessrio para que um texto instrucional seja eficaz
j bastante avanado, os efeitos podem ser catastrficos, levando e atinja com eficincia seus objetivos principais?
o paciente loucura por no haver como inverter o quadro. Isto
comum em casos que envolvem pacientes como Presidentes,
Governadores e Juzes Federais.
Posologia: uso oral. As dosagens variam de acordo com o
cargo e o estado do paciente. Para eleitor, 30 comprimidos, um
por dia; para vereador, 60 durante dois meses, um por dia; no caso

UECEVEST 45

Apostilas UECEVEST mod3.indb 45 06/02/2011 09:57:38


R E DA O

DESCRIO - RESUmO E X E R C C I O
Objetiva caracterizar, por meio de palavras, seres, lugares, ob- 01. Use V (verdadeiro) ou F (falso) para as afirmativas abaixo:
jetos e sentimentos; 1) ( ) Uma boa descrio tem que ser necessariamente objetiva.
Emprega adjetivos, locues adjetivas, estabelece comparaes 2) ( ) A descrio permite reconhecer um objeto ou imagin-lo.
e faz referncia s impresses sensitivas(tato, viso, olfato, pa- 3) ( ) A descrio caracteriza-se por apresentar uma sequncia
ladar e audio); de aes que vo gerar um conflito e um desfecho.
Utiliza linguagem objetiva e tcnica para criar uma imagem 4) ( ) Uma descrio eficaz usa a linguagem para tornar o
do ausente; ausente visvel.
Tem a funo de dar elementos a um determinado referente 5) ( ) Dependendo dos objetivos da escrita, a descrio deve
(animado ou inanimado/local, cena) a fim de localiz-lo, iden- ser geral ou minuciosa.
tific-lo ou qualific-lo. 6) ( ) As palavras numa descrio cientfica (manual de
instrues) devem permitir mais de uma interpretao.
Descrio de seres humanos 7) ( ) A descrio objetiva representa uma viso individual,
fsica prpria e exclusiva de quem descreve.
psquica 8) ( ) Os elementos fundamentais do objeto, e no os detalhes
fsico-psquica sem importncia, devem ser destacados.

Descrio de ambientes 02. Veja o texto abaixo e complete os espaos a partir das carac-
Geral p/ particular (especfico) tersticas do gnero presente:
Particular p/ geral A aprovao no vestibular o sonho de qualquer jovem, uma
conquista que a superao de grandes concorrncias. Para essa
Descrio de objetos realizao, necessrio muita dedicao aos estudos, escolha de
Esttica (visa a identificao) um curso de preparao adequado e __________________.

ATENO Com o objetivo de atingir esse fim, junte a dedicao aos estu-
Os dados utilizados numa descrio so frutos de uma sele- dos atravs_______________________, alm disso, adicione a
o interessada do observador. escolha de um curso De preparao_______________________,
some-se a esses ingredientes__________________________.
Estruturas
Dessa forma, constataremos ___________________________
Panfleto ____________________________________________.
Apresentao do produto (relao entre a necessidade e o
produto). 03. Em cada srie a seguir, complete duas palavras com ss e duas
As vantagens do produto (argumentos) Por que usar o com :
produto? a) ace___vel -conce___o -exce___o -descri___o
Concluso (convite aceitao do produto). b) discu__o -associa__o -cassa__o -opre__o
c) arreme__o -extin___o -posse__o -asser___o
Receita d) se___o -precau__o -exce___ivo -obse__o
Apresentar o objetivo para uso de determinado produto
(ingredientes) 04. Em cada srie de palavras a seguir, uma apresenta palavras
Como e para que usar o produto(ingredientes) com a ortografia correta:
Quais os resultados a serem obtidos (concluso) a) discaso, pedrestres, analizar, reivindicar, concientizar.
b) atravez, ancioso, tilidade, pespectiva, analisar.
Bula c) insentivo, perspectiva, discursso, consciente, estrupo.
Apresentao do produto (soluo para o problema indicado) d) atravs, reivindicar, requisitos, discusso, incentivo.
Como deve ser utilizado (indicaes e contra-indicaes)
Quais os resultados / Convite aceitao 05. Marque com um X onde h uso correto dos termos abaixo:
a) 1- ( ) entupir 2- ( ) entopir
IMPORTANTE b) 1- ( ) atrs 2- ( ) atrz
A linguagem deve ser concisa e direta; c) 1- ( ) espectativa 2- ( ) expectativa
Produo da persuaso como forma de impor o produto; d) 1- ( ) obsesso 2- ( ) obseo
Frequentemente usa verbos no imperativo; e) 1- ( ) pespectiva 2- ( ) perspectiva
Pode utilizar slogan como estratgia de propaganda;
Produo de itens argumentativos como forma de apresentao 06. Das seguintes redaes abaixo, assinale onde o uso da crase
das vantagens da utilizao do produto. est correto:
a) ( ) Ns encaminhamos s autoridades a reivindicao dos
moradores.
b) ( ) Devido ansiedade, o vestibulando precisa relaxar indo
festas.
c) ( ) A vinda da Copa 2014 trar prosperidade a populao
fortalezense.
d) ( ) O governo do estado assistiu os desabrigados atento s
possibilidades de saques.
e) ( ) O Papa posionou-se contra aprovao do aborto.

46 UECEVEST

Apostilas UECEVEST mod3.indb 46 06/02/2011 09:57:38


R E DA O

07. Marque os itens em que a acentuao est correta nas expres- ( )Estacione o veculo convenientemente.
ses grifadas: ( )Puxe o freio de estacionamento e calce a roda oposta, a fim
a) ( ) As produes escritas tm melhorado e vm de evitar qualquer deslocamento.
estimulando os alunos. ( )Retire a calota, comprimindo-a junto ao aro, em um ponto
b) ( ) O governo Lula mantm o mesmo discurso de de seu dimetro.
estabilidade economica. ( )Sinalize o local com o tringulo de segurana.
c) ( ) Os bons redatores desenvolvem a sua crticidade e ( )Introduza o macaco no respectivo encaixe quadrado,
obtem fluncia na escrita debaixo do estribo, perto do pra-lama traseiro. Em seguida,
d) ( ) A utilizao da tecnolgia tem sido, diriamente, alvo acione-o at que o veculo comece a levantar.
de polmicas. ( )Continue a levantar o carro, at que os furos dos parafusos
da roda sobressalente coincidam aproximadamente com os
08. Assinale a alternativa correta nos itens abaixo (Marque a ou b). do cubo e aperte os parafusos.
1) a) ( ) Fui no shopping comprar uma cala. ( )Em seguida, aps reapertar os parafusos, abaixe o veculo.
b) ( ) Fui ao shopping comprar uma cala.

2) a) ( ) Foi feita uma missa para homenagear os policiais mortos.


b) ( ) Foi celebrada uma missa para homenagear os policiais
mortos. POnTUAO

3) a) ( ) Ficou sob a mira do assaltante. Introduo


b) ( ) Ficou sobre a mira do assaltante. Para reproduzirmos, na linguagem escrita, os inumerveis re-
cursos da fala, contamos com uma srie de sinais grficos deno-
4) a) ( ) A notcia chegou derrepente, porisso nos assustamos. minados sinais de pontuao. So eles:
b) ( ) A notcia chegou de repente, por isso nos assustamos. o ponto ( . )
os dois pontos ( : )
5) a) ( ) Ao Excelentssimo Senhor Patativa do Assar, o ponto de exclamao ( ! )
b) ( ) Ao Senhor Patativa do Assar, a vrgula ( , )
o ponto e vrgula ( ; )
6) a) ( ) Para a aprovao no vestibular necessrio dedicao as aspas ( )
s aulas. o travesso ()
b) ( ) Para a aprovao no vestibular necessrio dedicao as reticncias (...)
as aulas. os parnteses ( ( ) )
7) a) ( ) Atualmente o dia dos adolescentes comeam e Alguns sinais de pontuao servem, fundamentalmente, para
terminam influenciados na mdia. marcar pausa (o ponto, a vrgula, o ponto e vrgula); outros tm
b) ( ) Atualmente o dia dos adolescentes comea e termina a funo de marcar a melodia, a entonao da fala (ponto de
influenciado pela mdia. exclamao, ponto de interrogao etc.)
No fcil fixar para o emprego correto dos sinais de pontua-
8) a) ( ) A prtica do aborto no se caracteriza pela negligncia. o, uma vez que, alm dos casos em que o uso de determinados
b) ( ) A prtica do aborto no caracteriza-se pela negligncia. sinais obrigatrio, existem razes de ordem subjetiva par sua
utilizao. A seguir, passaremos a expor algumas orientaes so-
9) a) ( ) Ns reclamamos com relao a carncia de professores
bre o assunto.
nas escolas pblicas.
b) ( ) Ns reclamamos com relao carncia de
Ponto ( . )
professores nas escolas pblicas.
utilizado para encerrar qualquer tipo de perodo, exceto os
10) a) ( ) O combate venda de bebidas lcoolicas deve ser terminados por oraes interrogativas ou exclamativas. um dos
intensificado. sinais que indica pausa.
b ( ) O combate venda de bebidas alcolicas deve ser Ex.: Anoitecia.
intensificado. Eu sou estudante.
Refiz as contas e no descobri onde errei.
11) a) ( ) A partir das 22hrs, o ingresso mais barato.
b) ( ) A partir das 22h, o ingresso mais barato. OBS.: O ponto tambm usado para indicar abreviao de
palavras. Exceto de hora, peso e medidas.
12) a) ( ) Estrada carroal Sr. Sr. Srt. V.Ex. Obs. Ex. etc.
b) ( ) Estrada carrovel
Ponto de interrogao ( ? )
13) a) ( ) O aborto um crime que tem tudo haver com o usado no fim de oraes interrogativas diretas. Nunca
comportamento da famlia. colocado no fim de uma orao interrogativa indireta.
b) ( ) O aborto um crime que tem tudo a ver com o Ex.: Entendeu?
comportamento da famlia. Ser que vai chover?
Se eu terminar os exerccios, posso ir com voc?
09. Organize a sequncia coerente do manual de instrues abaixo:
( )Solte os parafusos da roda com a chave sextavada, enquanto Ponto de exclamao ( ! )
o pneu estiver ainda no solo. colocado aps determinadas palavras, como interjeies e
( )Levante o veculo. oraes enunciados com entoao exclamativa. Denota entusias-
( )Acabe de afrouxar os parafusos e retire a roda. mo, alegria, dor surpresa, espanto, ordem etc.

UECEVEST 47

Apostilas UECEVEST mod3.indb 47 06/02/2011 09:57:38


R E DA O

Ex.: Ol! Que susto! vez que no houve quebra da sequncia lgica do enunciado.
Ah! Mos ao alto! Ex.: Os candidatos / sempre / receberam a imprensa.
Entendi! No toque em nada! advrbio
timo!
2. Marcar termos deslocados: Normalmente, quando um
Vrgula ( , ) termo deslocado de seu lugar original na frase, deve vir separado
A virgula o sinal de pontuao que indica uma pausa de por vrgula. Nesse sentido, separam-se:
curta durao, sem marcar o fim do enunciado. A vrgula pode o adjunto adverbial anteposto
ser empregada para separar termos de uma orao (vrgula no Ex.: Naquele dia,/os candidatos receberam a imprensa.
interior da orao), ou para separar as oraes de um perodo adj. adv. anteposto
(vrgula entre oraes).
Vrgula no interior da orao Em Portugus, a ordem OBS.:
normal dos termos na frase a seguinte: sujeito, verbo, comple- Se o adjunto adverbial anteposto for um simples advrbio, a
mentos do verbo, adjuntos adverbiais. Quando os termos da ora- vrgula dispensvel.
o se dispem nessa ordem, dizemos que ocorreu ordem direta Ex.: Hoje/os candidatos devero receber os jornalistas credenciados.
(ou ordem lgica). advrbio
Ex.: Muitos alunos/estudaram/a matria da prova/com afinco.
Sujeito verbo obj. direto adj. adverb o complemento pleonstico antecipado:
Ex.: Este assunto, / j o li em algum lugar.
Quando ocorre qualquer alterao na sequncia lgica dos compl. Pleonstico anteposto
termos, temos a ordem indireta.
Ex.: Com afinco, /muitos alunos estudaram a matria da prova O nome do lugar na indicao de datas:
termo deslocado Ex.: So Paulo, 28 de agosto de 1966.
Roma, 14 de fevereiro de 1981.
Quando a orao se dispe, em ordem direta, no se separam
por vrgulas seus termos imediatos. Assim, no se usa vrgula en- 3. Marcar a omisso de uma palavra (geralmente o verbo)
tre o sujeito e o predicado, entre o verbo e seu complemento, e Ex.: Ela prefere cinema e eu, teatro.
entre o nome e seu complemento ou adjunto.
Ex.: Muitos imigrantes europeus/chegaram ao Brasil naquele ano. 4. Marcar o vocativo
sujeito predicado Ex.: Meus amigos,/a ordem a base do governo.
vocativo (Machado de Assis)
Todos os alunos apresentaram/a redao ao professor.
verbo complemento OBS.:
Pode-se, em vez de vrgula, marcar o vocativo com um ponto
A spera/resposta/ao candidato/deixou-o magoado. de exclamao a fim de dar nfase.
adj. adnom. nome compl. nominal Ex.: Deus! Deus! Onde ests que no respondes?
(Castro Alves)
Usa-se vrgula no interior da orao para:
1. Marcar intercalaes: Os termos que se intercalam na or- 5. Separar termos coordenados assindticos:
dem direta, quebrando a sequncia natural da frase, devem vir Ex.: Aquela paisagem nos despertava confiana, tranquilidade,
isolados por vrgulas. Assim, separam-se: calma.
o aposto intercalado: Quaresma convalesce longamente, demoradamente, melan-
Ex.: Kak,/ex-jogador do So Paulo/atualmente joga no Milan colicamente. (Lima Barreto)
aposto
OBS.:
expresses de carter explicativo ou corretivo: Se os termos coordenados estiverem ligados pelas conjunes
Ex.: A sua atitude,/isto ,/o seu comportamento na aula merece e, ou, nem, no se usa a vrgula.
elogios. Ex.: Aquela paisagem nos despertava confiana, tranquilidade e
expresso explicativa calma.
Pedra ou Paulo casar com Helosa.
No haver aula amanh,/ou melhor,/depois de amanh. No necessitavam de dinheiro nem de auxlio.
expresso corretiva
Se essas conjunes vierem repetidas para dar ideia de nfase,
conjunes coordenativas intercaladas: usa-se vrgula.
Ex.: A sua atitude,/no entanto,/causou srios desentendimentos. Ex.: E os pais, e os amigos, e os vizinhos, magoaram-no.
conj. intercalada No caminhava por montanhas, ou florestas, ou cavernas.
Havia, / porm, / um inconveniente srio. No estudava Fsica, nem Qumica, nem Matemtica, nem
conj. intercalada Histria.

adjuntos adverbiais intercalados: Vrgula entre oraes


Ex.: Os candidatos,/naquele dia,/receberam a imprensa. 1. Oraes subordinadas adjetivas explicativas: As ora-
adj. adv. intercalado es subordinadas adjetivas explicativas sempre so separadas
por vrgula.
OBS.: Ex.: O homem, / que um ser racional, / vive pouco.
Se o adjunto adverbial intercalado for de pequena extenso o.p. or. subo adj. Explicativa o.p.
(um simples advrbio, por exemplo), no se usa a vrgula, uma

48 UECEVEST

Apostilas UECEVEST mod3.indb 48 06/02/2011 09:57:39


R E DA O

OBS.: Os indignados rus protestaram; os severos juzes, no


As oraes subordinadas adjetivas restritivas normalmente entanto, no cederam.
no se separam por vrgulas. Podem terminar por vrgula (mas
nunca comear por ela!) separar oraes coordenadas que se contrabalanam em fora
Quando tiverem uma certa extenso: expressiva (formando anttese, por exemplo)
Ex.: O homem que encontramos ontem noite, perto do lago, Ex.: Muitos se esforam; poucos conseguem. Uns trabalham;
parecia aborrecido. outros descansam.

Quando os verbos se seguirem: separar oraes coordenadas que tenham certa extenso.
Ex.: O homem que fuma, vive pouco. Ex.: Os excelentes jogadores de futebol olmpico reclamaram
Quem estuda, aprende. com razo das constantes crticas do tcnico; porm o
teimoso tcnico ficou completamente indiferente aos
2. Oraes subordinadas adverbiais: Oraes dessa modali- constantes pedidos dos jogadores.
dade (sobretudo quando estiverem antecipadas) separam-se por
vrgulas. separar os diversos itens de um considerando ou de uma enu-
Ex.: Quando o cantor entrou no palco, todos o aplaudiram. merao.
or. subo adverbial o.p. Ex.: Considerando:
a) a alta taxa de desemprego no pas;
Entrando o cantor, / todos devem aplaudi-lo. b) a excessiva inflao;
or. subo adverbial o.p. c) a recesso econmica; solicitamos especial ateno ao
nosso pedido.
A questo,/conforme se esperava,/era complicadssima.
o.p. or.sub. adverbial o.p. Art. 112. O Poder Judicirio exercido pelos seguintes rgos:
I. Supremo Tribunal Federal;
3. Oraes subordinadas substantivas: Oraes desse tipo II. Conselho Nacional de Magistratura;
(com exceo das apositivas) no se separam da principal por vrgula. III. Tribunal Federal de Recursos e juzes federais;
Ex.: Espero / que voc me telefone. IV. Tribunais e juizes militares;
o.p. or. subo substantiva obj. direta V. Tribunais e juizes eleitorais;
VI. Tribunais e juzes do trabalho;
O remdio era / ficar em casa. VII. Tribunais e juzes estaduais.
o.p. or. subo substantiva (Constituio Federal)

4. Oraes coordenadas: As oraes coordenadas (exceto as Dois pontos ( : )


iniciadas pela conjuno aditiva) separam-se por vrgula. Os dois pontos marcam uma sensvel suspenso da melodia
Ex.: Cheguei,/pedi silncio,/aguardei alguns minutos/e comecei de uma frase para introduzir algo bastante importante. Nesse sen-
a palestra. tido, utilizam-se os dois pontos para:
or. Coord or. coord. or. coord. or. coord.
dar incio a fala ou citao textual de outrem.
5. Oraes intercaladas: So sempre separadas por vrgulas Ex.: J dizia o poeta: A vida a arte do encontro.
(ou duplo travesso, equivalente a elas). A porta abriu-se, um brado ressoou:
Exs.: Eu, / disse o orador, / no concordo. At que enfim, meu rapaz!
or. intercalada (Ea de Queirs - Os Maias)

O problema das enchentes,/disse o candidato,/ser prioritrio. dar incio a uma sequncia que explica, esclarece, identifica,
or. intercalada desenvolve ou d&crimina uma ideia anterior.
Ex.: Descobri a grande razo da minha vida: voc. J lhe dei
Eu - / disse o orador / - no concordo. tudo: amor, carinho, compreenso, apoio.
or. intercalada
Tivemos uma tima ideia: abandonar a sala. O resultado no
O problema das enchentes-/disse o candidato-/ser prioritrio. se fez esperar: fomos chamados diretoria.
or. intercalada
Aspas ( )
Ponto e vrgula ( ; ) Empregam-se as aspas para:
O ponto e virgula marca uma pausa mais longa que a vrgula; isolar citao textual colhida a outrem.
no entanto menor que a do ponto. Justamente por ser um sinal Ex.: Como afirma Caio Prado Jr. em Histria Econmica do
intermedirio entre a vrgula e o ponto, fica difcil sistematizar Brasil: A questo da imaginao europia do sculo passado
seu emprego. Entretanto, h algumas normas para sua utilizao. est intimamente ligada da escravido.
Emprega-se o ponto e vrgula para: Diz Thomas Mann em A Montanha Mgica: Todo caminho
separar oraes coordenadas que j venham quebradas no seu que trilhamos pela primeira vez muito mais longo e difcil do
interior por vrgula. que o mesmo caminho, quando j o conhecemos.
Ex.: Os indignados rus mostravam suas razes para as
autoridades de forma firme; alguns, no entanto, por receio isolar palavras ou expresses estranhas lngua culta, tais como:
de punies, escondiam detalhes aos policiais. grias e expresses populares, estrangeirismos, neologismos,
arcasmos etc.
Ela prefere cinema; eu, teatro. No esperava outra coisa; Ex.: Ele era um gentleman.
afinal, eu j havia sido avisado. Ele estava numa boa.

UECEVEST 49

Apostilas UECEVEST mod3.indb 49 06/02/2011 09:57:39


R E DA O

O rapaz ficou grilado com o resultado. Ela veio Parnteses ( ( ) )


desentristecer meu corao. Os parnteses servem para isolar explicaes, indicaes ou
comentrios acessrios.
mostrar que uma palavra est em sentido diverso do usual Ex.: Aborrecido, aporrinhado, recorri a um bacharel (trezentos
(geralmente, em sentido irnic) mil-ris, fora despesas midas com automveis, gorjetas etc)
Ex. Fizeste excelente servio. e embarquei vinte e quatro horas depois (...)
Sua ideia foi mesmo fantstica. (Graciliano Ramos - So Bemardo).

OBS.: Fui hoje casa deste ltimo, apresentar desculpas (deve


Podem-se ainda utilizar as aspas para dar destaque a uma pa- ter ficado aborrecido com a minha ausncia no local
lavra ou expresso. determinado para o encontro) e repetir o convite para a
Ex.: J entendi o porqu do seu projeto; s no percebo pretendida visita.
como execut-lo. (Cyro dos Anjos - O Amanuense Be/miro).

Travesso () Ela (a rainha) a representao viva da mgoa (...)


O travesso simples () serve para indicar que algum est (Lima Barreto - Recordaes do Escrivo Isaas Caminha)
falando de viva voz (discurso direto). Emprega-se, pois, o traves-
so para marcar a mudana de interlocutor nos dilogos.
Ex.: De quem so as pernas?
Da Madalena, respondeu Gondim. E X E R C C I O
Quem? 01. Dadas as sentenas, marque o item correto:
Uma professora. No conhece? Bonita. I. Quase todos os habitantes daquela regio pantanosa e longe
Educada, atalhou Joo Nogueira. da civilizao, morrem de malria.
Bonita, disse outra vez 6ondim. Uma lourinha a, de uns II. Homem que trabalha, constri um futuro promissor.
trinta anos. III. Muitas pessoas observavam com interesse o eclipse solar.
Quantos? perguntou Joo Nogueira.
Uns trinta, pouco mais ou menos. Deduzimos que:
(Graciliano Ramos - So Bemardo) a) apenas a sentena I est correta.
b) apenas a sentena II est correta.
Pode-se usar o duplo travesso ( ) para substituir dupla vr- c) Todas esto corretas
gula, sobretudo quando se quer dar nfase ou destaque ao termo d) n.d.a
intercalado.
Ex.: O ministro profundo conhecedor do mercado 02. Marque a alternativa pontuada corretamente:
intemacional est consciente das dificuldades. a) Com as graas de Deus vou indo mestre Jos Amaro!
Machado de Assis grande romancista brasileiro b) Com as graas de Deus, vou indo mestre Jos Amaro!
tambm escreveu contos. c) Com as graas de Deus, vou indo, mestre Jos Amaro!
d) Com as graas, de Deus, vou indo, mestre Jos Amaro!
Reticncias (...)
As reticncias marcam uma interrupo da sequncia lgi- 03. Das seguintes redaes abaixo, assinale a que no est pon-
ca da frase, antes que ela tenha chegado ao seu fim. Podem ser tuada corretamente:
usadas com valor estilstico, isto , com a inteno deliberada de a) os meninos, inquietos, esperavam o resultado do pedido.
permitir que o leitor complete o pensamento que foi suspenso, b) Inquietos, os meninos esperavam o resultado do pedido.
ou para marcar fala quebrada e desconexa, prpria de quem est c) Os meninos esperavam, inquietos, o resultado do pedido.
nervoso ou inseguro. d) Os meninos inquietos esperavam o resultado do pedido.
Ex.: Eu no vou dizer mais nada. Voc j deve ter percebido que ele... e) Os meninos, esperavam inquietos, o resultado do pedido.
Depois de um instante, Carlos lanou de l entre um
rumor de gua que caa: 04. Em: A menina, conforme as orientaes recebidas, estudou:
No sei... Talvez... Logo te digo..: a) h erro na colocao das vrgulas
(Ea de Queirs - Os Maias) b) a primeira vrgula deve ser omitida.
c) a segunda vrgula deve ser omitida
Bem... Sabe... Pois ... Quer dizer... No era bem assim... d) a forma de colocao das vrgulas est correta.
Sei l...
05. Marque a alternativa incorreta quanto ao uso da vrgula.
Usam-se tambm as reticncias (de preferncia, entre parn- Emprega-se a vrgula para...
teses) para indicar que parte de uma citao foi omitida. a) intercalar expresses explicativas (isto , a saber)
Ex.: O sertanejo (...) um forte b) separar o verbo do seu objeto direto.
(Euclides da Cunha - Os Sertes)
c) separar o local e a data na correspondncia.
d) separar oraes adjetivas explicativas.
(...) nenhuma tinha os olhos de ressaca, nem os de cigana
oblqua e dissimulada. 06. Pontue corretamente:
a) A enorme expanso demogrfica, que comeou no sculo
(Machado de Assis - Dom Casmurro)
XVIII, surge a mim etnlogo como um fenmeno capital.
b) A enorme expanso demogrfica que comeou no sculo
No alcancei a celebridade do emplasto, no fui ministro, XVIII, surge a mim etnlogo como um fenmeno capital.
no fui califa (...) c) A enorme expanso demogrfica, que comeou no sculo
(Machado de Assis - Memrias Pstumas de Brs Cubas) XVIII, surge a mim, etnlogo, como um fenmeno capital.

50 UECEVEST

Apostilas UECEVEST mod3.indb 50 06/02/2011 09:57:40


R E DA O

d) A enorme expanso demogrfica, que comeou no sculo Produza um texto em prosa, para ser publicado nessa coletnea,
XVIII surge a mim etnlogo como um fenmeno capital. no qual voc descreve a rotina de um dos seguintes profissionais:
ascensorista, carteiro, balconista, entregador, pedreiro, vigilante.
07. Pontue o texto abaixo e faa as alteraes necessrias.
S faltava essa a batata delcia que causou uma revoluo nos 09. UFC 2006
hbitos alimentares da Europa no sculo XVII e mais recente- Proposta 1
mente ganhou o mundo com a difuso do fast food foi colocada Texto 1
no banco dos rus segundo uma pesquisa da Universidade de Es- Japo lana chiclete para turbinar seios
tocolmo a batata frita principalmente contm uma molcula can- Seios Pamela Anderson sem precisar de cirurgia um
cergena a acrilamida ela aparece quando o alimento submetido sonho impossvel nos dias atuais. Mas uma empresa no Japo
s altas temperaturas necessrias ao preparo culinrio divulgado garante que mascar um chiclete entre trs e quatro vezes diaria-
h algumas semanas o estudo virou tema de acaloradas discusses mente o suficiente para dar uma turbinada na comisso de frente.
na Inglaterra e nos Estados Unidos os pases que mais consomem O produto, batizado de Bust-Up (busto para cima, em traduo
esse tipo de prato no mundo. livre), foi lanado na semana passada e os pedidos de venda cho-
veram o pote com duzentos chicletes custa cerca de R$ 52,00.
De sabor morango, o chiclete contm a erva puerria mi-
rifica. O fabricante, a indstria cosmtica B2Up, garante que
a goma libera na mucosa da boca fitoestrgenos, substncias
PROPOSTAS DE REDAO extradas de plantas que imitam a ao do estrgeno (hormnio fe-
minino). Eles seriam importantes para a circulao sangunea e es-
01. UECE 2002.2 tariam envolvidos no processo de acmulo de lquido e gordura no
Escolha uma das propostas abaixo para desenvolver seu texto: tecido mamrio. Assim, o busto ganharia centmetros a mais. (...)
a) Imagine-se no papel de um publicitrio encarregado da CASTELLN, Lena. Medicina & Bem-estar. Isto, 30/03/2005. p.56.
campanha de um candidato a um dos cargos nas prximas
eleies. Texto 2
Crie um nome fictcio para seu candidato e escreva um texto,
apresentando esse candidato como um defensor da liberdade. Passa fcil
Seu texto ser distribudo ao pblico em forma de panfleto. Era o que faltava para facilitar a vida do consumidor mo-
derno. At o final do ano deve chegar ao mercado a mquina
02. UECE 2004/2 de passar roupa. Do tamanho de uma geladeira, a traquitana
Escreva um texto mostrando como se deu o seu processo de projetada por Clia Jaber, no Centro Incubador de Empresas de
aquisio da leitura e da escrita. Tecnologia da Universidade de So Paulo, passa at doze peas
por hora. base de vapor, a Agillisa consome 50% menos energia
03. UECE 2005/2 do que o ferro eltrico e ainda tem a vantagem de passar roupas
Escreva um texto mostrando o que voc faria se descobrisse secas ou sadas da mquina de lavar. Ela faz prega de calas e
que tinha apenas mais um ano de vida. vem com acessrios para passar colarinhos e punhos de ca-
misas. Seu preo estimado de R$ 2,5 mil.
04. UECE 2006/1 PINHO, Cludia. Cincia, Tecnologia e Meio Ambiente. Isto, 30/03/2005. p.90.
O Brasil um pas complexo e contraditrio, com manifes-
taes culturais diversificadas, paisagens distintas e contrastes so- Texto 3
ciais bastante acentuados.
Redija um texto em que pode se encaixar a ideia: O QUE Cortina bloqueia a radiao solar
D PRA RIR D PRA CHORAR. Uma cortina capaz de bloquear a passagem da radiao
solar em 94% e reduzir os gastos com ar-condicionado em 60%
05. UECE 2008/2 no vero. Essa a novidade da Vacuoflex, uma pequena empre-
Voc candidato a vereador de seu municpio e vai se apre- sa instalada na Incubadora de Empresas de Base Tecnolgica da
sentar em um comcio. Valendo-se do gnero textual Discurso, Universidade Estadual de Campinas (Unicamp). A inovao da
escreva um texto em que apresentar propostas que contribuiro empresa foi aplicar um filme plstico metalizado sobre uma cor-
para melhorar a qualidade de vida de sua comunidade. tina de lona plstica ou tecido.
Revista Pesquisa FAPESP www.revistapesquisa.fapesp.br.
06. UECE 2009.1
Escreva um texto em forma de Receita, apresentando os ingre- Assim como fizeram a empresa japonesa, Clia Jaber e
dientes necessrios concretizao de um Feliz Ano Novo. No a Vacuoflex, voc tambm pode ter suas criaes. Imagine
esquea de dar um ttulo ao seu texto, usar a linguagem adequada que projetou A MQUINA DO SENTIMENTO uma
e apresentar os itens necessrios a esse gnero textual(ingredientes, mquina que, quando acoplada ao corao, este s obedecer
modo de preparo e rendimento). (adaptado UECE) aos comandos dela.
Produza um texto publicitrio, para ser publicado em uma
07. UFC 2004 revista de circulao nacional, no qual voc descreve sua
02) A sua escola est promovendo um concurso de Receitas Cria- engenhoca, esclarecendo, aos possveis usurios, suas caracte-
tivas. Para participar, produza um texto em prosa no qual voc rsticas, funes e usos.
apresenta uma receita para enfrentar o medo.
Veja as seguintes propostas:
08. UFC 2005 01) Voc foi convidado para representar a sua cidade numa reu-
4) Imagine que o Governo esteja organizando uma obra com a nio com autoridades da rea de educao do governo. Para
qual tornaria pblica a tarefa de tantos profissionais que, apesar tal, foi incumbido de produzir um texto descritivo, mostrando a
de lidarem com o pblico, vivem praticamente no anonimato. realidade das escolas pblicas.

UECEVEST 51

Apostilas UECEVEST mod3.indb 51 06/02/2011 09:57:40


R E DA O

02) Produza um texto descritivo, em forma de receita, com os COMISSO COORDENADORA DO VESTIBULAR(CCV)
indicativos necessrios sua aprovao no vestibular. www.ufc.br.
03) Imagine-se no papel de um publicitrio e faa uma campanha
para um produto que venha solucionar a crise de falta de compa- COMISSO EXECUTIVA DO VESTIBULAR(CEV)
nheirismo entre as pessoas. www.uece.br.

GARCIA, Othon Moacyr Comunicao em Prosa Moderna:


Aprenda a escrever, aprendendo a pensar 26. ed. Rio de
G A B A R I T O Janeiro: Ed, FGV, 2006.
Descrio
O seu primeiro dia na Universidade KLEIMAN, ngela. Texto e leitor: aspectos cognitivos da
R. Discusso com os alunos. leitura. 5 edio. Ed. Pontes. Campinas, 1997.
Um acontecimento inesquecivel
R. Discusso com os alunos. MARCUSCHI, Luiz Antonio. Ligustica de texto: o que e
O produto que vai acabar com os altos ndices de obesidade como se faz. Recife. Ed.UFPE, 1983.
R. Discusso com os alunos.
PLATO & FIORIN, Para entender o texto: leitura e
Texto 01- descarga de privada redao. 16 e.d. 6 reimp., tica, 2003.
Texto 02- livro
Texto 03-caixa de fsforos REGIS, Herman & MEDEIROS, Graa Produo textual
Texto 04-cofre no ensino mdio: gerao de idias Fortaleza: Primus, 2.ed.
2002. www.revistaescola.abril.com.br.
CARACTERSTICAS DO TEXTO PUBLICITRIO
R. Discusso com os alunos.

CARACTERSTICAS DO PANFLETO
R. Discusso com os alunos.

CARACTERSTICAS DA RECEITA
R. Discusso com os alunos.

CARACTERSTICAS DA BULA DE REMDIO


R. Discusso com os alunos.

Exerccios
01. V, V, F, V, V, F, V, V
02. Discusso com os alunos.
03. a) ss / ss / / b) ss / / / ss
c) ss / / ss / d) / / ss / ss
04. d 05. a) 2 b) 1 c) 2 d) 1 e) 2
06. a) v b) f c) f d) v e) f
07. a) v b) f c) f d) f
08. 1/b 2/b 3/a 4/b 5/a 6/a 7/b
8/a 9/b 10/b 11/b 12/b 13/b
09. Discusso com os alunos.

REFERnCIAS BIBlIOGRFICAS

ANTUNES, Irand Costa Lutar com palavras: coeso e


coerncia So Paulo: Ed. Parbola Editorial, 2005.

ARAJO, Olmpio & ARAJO, Murilo. Desenvolvendo a


habilidade de escrever. Fortaleza, Primus, 2007

CARNEIRO, Augustinho Dias. Redao em construo: a


escritura do texto. 2 edio. Ed. Moderna. So Paulo, 2002.

CEREJA, Willian Roberto & MAGALHES, Thereza Cochar


Texto e interao: uma proposta de produo textual a partir
de gneros e projetos So Paulo: Atual, 2000.

52 UECEVEST

Apostilas UECEVEST mod3.indb 52 06/02/2011 09:57:41


P R - V E S T I B U l A R

INGLS

Apostilas UECEVEST mod3.indb 53 06/02/2011 09:57:46


Caro(a) Aluno(a),

Competncia de rea 2 - Conhecer e usar lnguas(s) estrangeiras(s) moderna(s)


como instrumento de acesso a informaes e a outras culturas e grupos sociais*.
H5 - Associar vocbulos e expresses de um texto em LEM ao seu tema.
H6 Utilizar os conhecimentos da LEM e de seus mecanismos como meio de
ampliar as possibilidades de acesso a informaes, tecnologias e culturais.
H7 Relacionar um texto em LEM, as estruturas lingsticas, sua funo e seu
uso social.
H8 Reconhecer a importncia da produo cultural em LEM como representa-
o da diversidade cultural e lingustica.

Apostilas UECEVEST mod3.indb 54 06/02/2011 09:57:46


INGLS

mODAl VERBS (VERBOS mODAIS) 3. Must (precisar, dever, ter de) obrigao;
necessidade; deduo.
Os Modal verbs so verbos que possuem algumas anomalias, Armative Interrogative Negative
por exemplo, o fato de seu infinitivo no possuir to. Descrevemos os I I I
principais que so may, can, must, should e ought to. You you You
So usados antes de outros verbos no infinitivo e sem to (com He he He
exceo de ought to) e no recebem s na terceira pessoa do singular. She must Must she ? She mustnt.
Uma outra caracterstica que no possuem gerndio nem particpio. It it It (must not)
So tambm verbos auxiliares, portanto, no precisam dos auxiliares We we We
do/does (dont/doesnt) ou did (didnt) para formar as frases interrogativas They they they
e negativas.
4. should/ought to (deveria) conselho; recomendao;
O primeiro verbo a ser estudado may (poder), que indica
obrigao moral.
probabilidade e permisso. Nesse caso, pode ser substitudo por to
be allowed to. O verbo can (poder) usado para indicar capacidade, Armative Interrogative Negative
habilidade, habilidade cognitiva e tambm possibilidade e probabilidade. I I I
O passado de may e can so respectivamente might e could. You you You
He he He
She should / Should she She shouldnt
May Can It (ought to) it It (oughtnt)
Sue may/is allowed We we We
Roger Federer can/(is
1. Presente to come with us every They they they
able to) play tennis.
weekend.
Catherine might/was Roger Federer could/
2. Passado allowed to stay on our (was able to) play tennis Q U E S T I O n S
farm last year. better six years ago.
Roger Federers will 01. ______________ I help you? Im looking for a book about
She may/will be ecology.
be able to) play tennis
3. futuro allowed to come with a) Should c) Must
much better than his
us tomorrow. b) Can d) Ought to
father when he grows up.

O verbo must no possui passado nem futuro. No entanto, 02. The cheetah, one of the endangered species, _____________
utiliza-se have to (sinnimo) de must para formar o passado e o run 100 km an hour.
futuro de must. Assim, para indicar uma obrigao no passado, a) must c) is allowed to
a conjugao ser had to (precisou, teve de) e, para indicar uma b) should d) can
obrigao no futuro ser will have to (precisar, ter de). A
forma must not e a mustnt indicam proibio no sentido de 03. Rapahel _________ see outside; its too foggy.
no dever ou no poder. J dont (doesnt) have to utili- a) cant c) is not allowed to
zado quando se quer dar o sentido de no haver necessidade b) can d) should
(no precisa).
Os verbos modais should e ought to (deveria) so usados em 04. Alice cant go to the movies with us because she _________
caso de conselho, recomendao ou obrigao moral. Embora study for the test.
sejam sinnimos, nas interrogativas e negativas recomendvel a) mustnt c) must
usar o verbo should. b) cant d) have to

05. Dad is still taking a rest; he ________ be very tired.


1. May (poder) permisso (formal); probabilidade;
a) should c) may not
possibilidade.
b) cannot d) must
Armative Interrogative Negative
I I I 06. Parents ________ support their children.
You you You a) have c) should not
He he He b) must d) must not
She may May she ? She may not .
It it It 07. Look at that animal over there on your left. It ________ be
We we We a tiger.
They they they a) must c) should
2. Can (poder) habilidade (fsica, cognitiva); b) must not d) is able to
capacidade; permisso (informal).
Armative Interrogative Negative 08. In you should start each day with a song in your soul,
I I I should expresses an idea of:
You you You a) certainty. c) impossibility.
He he He b) advice. d) obligation.
She can try. Can she ? She cannot
It it It (cant) 09. Assinale a alternativa que preenche corretamente a lacuna da
We we We frase a seguir. We ________ hurry. The bus leaves in 10 minutes.
They they They a) can c) did
b) must d) would

UECEVEST 55

Apostilas UECEVEST mod3.indb 55 06/02/2011 09:57:47


INGLS

10. Qual das expresses corresponde a ele no deveria ter feito isso: O presente simples usado para:
a) He mustnt have made it; a) falar de aes que acontecem habitualmente;
b) He shouldnt have done that; We have lunch every day.
c) He could not have made it; Nicole Kidman walks in Central Park on Sundays.
d) He might not have done that.
b) dizer verdades universais.
The sun shines.
Birds fly.
SImPlE PRESEnT TEnSE Nesse tempo verbal, usamos os advrbios de frequncia:
always, never, usually, generally, frequently, often, sometimes, seldom,
Sleep rarely antes do verbo principal ou depois do verbo to be e de ou-
Armative Interrogative Negative tros auxiliares e modais.
I/You/We/They Do I/you/we/they I/You/We/They Mr. Cruise always wakes up early.
sleep well. sleep well? dont sleep well. Cameron Diaz reads a book once a month.
Jim Carrey is always in love.
He/She/It sleeps Does he/she/it sleep He/She/it doesnt Jennifer Lopes can never go out at night.
well. well? sleep well.
Utlilizamos, ainda, no simple present, expresses como every
Verbos terminados em ss, -ch, -sh, -o, -x, e z recebem es
..., once a..., twice a... (month, week, year etc.) no final das oraes.
na Terceira pessoa do singular em oraes afirmativas.
Nas oraes interrogativas e negativas, usa-se o verbo auxiliar
do/dont para I, you, we e they e does/doesnt para he, she e it. Nes-
ses casos, o verbo principal aparece no infinitivo. Q U E S T I O n S
01. George Lucas and Steven Spielberg usually __________ in
Wash Central Park.
Armative Interrogative Negative a) walks c) walk
I/You/We/They Do I/you/we/they I/You/We/They b) doesnt walk d) walking
wash every day. wash every day? dont wash every day
He/She/It washes Does he/she/it He/She/it doesnt 02. ____ Rene Zellweger _______ when she ___ to Las Vegas?
every day. wash every day? wash every day. a) Does/go shopping/goes c) Do/goes shopping/goes
b) Does/goes shopping/go d) Do/going shop/go
Verbos terminados em y aps consoante em oraes afir-
mativas perdem o y e recebem ies mas apenas na 3 pessoa do 03. Mel Gibson ______ about terrorism.
singular (He, she, it). a) worry c) worries
Em oraes interrogativas e negativas usa-se do/dont ou b) dont worry d) worrying
doesnt e o verbo principal no infinitivo.
04. The sun _________ rises in the west.
a) always c) never
Fly b) often d) usually
Armative Interrogative Negative
I/You/We/They Do I/you/we/they I/You/We/They 05. Indique a resposta certa para Are you fond of Susan? Yes, ...
fly high. fly high? dont fly high. a) Yes, you are fond of Susan; c) Yes, you are;
He/ She/It flies Does he/she/it fly He/She/it doesnt b) Yes,I am; d) Yes, very fond of Susan.
high. high? fly high.
06. In the sentence Everyone lies, the present tense is being
O verbo have na Terceira pessoa do singular em oraes afir- used to express a fact that will never change in time (historical
mativas muda para has. present). In which of the alternatives below is the present tense
being used to express a similar idea?
Have a) It is hot and sunny today.
b) Water freezes at 0 Celsius.
Armative Interrogative Negative c) My plane leaves at 5 pm tomorrow.
I/You/We/They Do I/you/we/they I/You/We/They d) Joe is late for work today.
have friends. have friends? dont have friends.
He/ She/It has Does he/she/it He/She/it doesnt 07. This is a ghost which _________ in fact appear. Instead, it
friends. have friends? have friends. makes noises and throws objects around.
a) does c) are
Be b) doesnt d) arent
Armative Interrogative Negative
08. Choose the alternative which is in the simple present tense:
I am a star. Am I a star? I am not a star. a) The children are watching the film.
He/ She/It is Is he/she/it famous? He/She/It isnt b) The film was a great success.
famous. famous. c) You will never forget the film.
You/We/They Are you/we/they You/We/They d) E.T. learns about life on Earth.
are actors. actors? arent actors.
O verbo irregular to be, por j ser auxiliar, no precisa de do 09. The theater is expensive, and I dont have much money.
ou does. So, I _________ go there.

56 UECEVEST

Apostilas UECEVEST mod3.indb 56 06/02/2011 09:57:48


INGLS

a) often; c) seldom; 04. Another positive aspect of this recently devised plastic is that it
b) always; d) frequently. a) recycles grocery bags faster.
b) does not use an organic compound.
10. Many people ________ coee. They think coee_________ c) has an unchangeable cross-linking reaction.
them stay awake. d) retains its mechanical characteristics.
a) likes/make c) like/makes
b) doesnt like/dont make d) like/makes 05. The Dutch chemists discovery is relevant because it
a) makes plastic recycling more aordable.
b) contributes to scientific understanding of self-healing plastics.
c) enables plastic factories to work faster.
d) increases the greenhouse eect.
UECE 2009.2
06. One aspect that characterizes thermoset resins is that they
From: www.nytimes.com. a) do not reverse the cross-linking reaction.
April 27, 2009 b) are used to make water bottles.
c) create a soft polymer.
Most of the plastics that are recyclable today water d) often decompose with heat.
bottles and grocery bags, for instance are what are called
thermoplastics. They are polymers that can be melted down and
molded into something else.
But there is another category of plastics, thermoset resins,
that cant be easily recycled. These polymers the stu of circuit SImPlE PAST TEnSE
boards, electrical insulation and epoxy glue, among other things
have strong cross-links and when heated tend to decompose. O passado simples (past tense) usado para descrever aes
Most products made from these plastics end up as waste. que aconteceram num passado geralmente definido, como in
But chemists at the University of Groningen in the 1997, yesterday, ten years ago etc.
Netherlands have devised a thermoset plastic that, rather than Diferente da lngua portuguesa, a conjugao verbal do passa-
decomposing, heals itself when heated. Writing in the journal do dos verbos regulares e irregulares em ingls igual para todas
Macromolecules, the researchers, Youchun Zhang, Antonius as pessoas, exceto a do verbo to be . compare:
A. Broekhuis and Francesco Picchioni, say the material has the to walk to break to be
potential to be recycled and reused many times. (verbo regular) (verbo irregular)
The building blocks of the polymer are polyketones, and they I I I
are cross-linked using another organic compound, bismaleimide. You You He
One secret to the materials success is that the cross-linking He He She was
reaction is reversible: when heated to about 300 degrees She walked She broke It (wasnt)
Fahrenheit, the material becomes unlinked, but as it cools the It didnt walk It didnt break
links re-form, creating a rigid polymer once again. We We We
The researchers demonstrated that the material can be You You You were
shredded, melted and remolded at least seven times with no loss of They They They (werent)
mechanical properties. Their discovery, they say, adds to scientific
understanding of the nature of self-healing materials, and with
to walk / to break to be
more research may eventually lead to the full development of
recyclable thermoset plastics. I I
you he
he Was she ?
Did she walk / break? it
Q U E S T I O n S it we
01. A tough kind of plastics to recycle is we were you ?
a) the one containing an acid in its molecules. you they
b) the type called thermoset resins. they
c) a kind that only decomposes at 100 degrees Fahrenheit.
d) water bottles thrown in the ocean. Regular verbs (affirmative)
1. Nas frases afirmativas, acrescenta-se ed aos verbos regulares.
02. Researchers in the Netherlands have found that a specific play played
type of plastic can walk walked
a) replace its micromolecules. They played tennis yesterday.
b) improve weak cross-links.
2. Se o verbo terminar em e, acrescenta-se o d.
c) heal itself instead of decomposing.
Arrive arrived
d) be reused up to fifty times.
She arrived from Recife last week.
03. The thermoset plastic that has been discovered at the 3. Se o verbo terminar na sequncia consoante-vogal-consoante
University of Gronigen has the unique capability of (CVC) e em slaba tnica (ou for um verbo monossilbico)
a) being recycled lots of times. dobra-se a ltima consoante antes do ed.
b) being stored in cold temperatures. Prefer preferred
c) recreating polyketones. Stop stopped
d) remodeling temperature patterns. I stopped crying an hour ago.

UECEVEST 57

Apostilas UECEVEST mod3.indb 57 06/02/2011 09:57:49


INGLS

4. No entanto, a consoante de verbos polissilbicos terminados a) Did he push or pull the door when he got there?
em CVC no ser dobrada se a ltima no for a tnica. b) Did he pushed or pulled the door when he arrived there?
open opened c) Did he pull or push the door when he got there?
visit visited d) Pushed or pulled he the door when he gots there?
We opened the gate an hour ago. e) Pushed or pulled him the door when he arrived there
They visited their cousins last weekend.
10. (UECE/20009.2)The sentences At the same time, vernacular
5. Se o verbo terminar em consoante + y, substitui-se o y por prose was developing, The printer William Caxton shared this
i e acrescenta-se ed. concern. and The vitality of Elizabethan writing owed much to
carry carried both free borrowings from ancient and modern tongues. Are res-
cry cried pectively in the
They cried two hours ago. a) present continuous, simple present, simple past.
b) past continuous, simple past, simple past.
Irregular verbs c) simple past, simple past, past perfect.
Os verbos irregulares so assim chamados justamente porque d) past perfect, simple past, past continuous.
cada um apresenta uma forma diferente no passado. Portanto,
no h outra sada devemos memoriz-los!
Para todas as pessoas, o auxiliar o did para a forma interro-
gativa e o didnt para a negativa. O verbo principal tambm volta
para o infinitivo. Exceo: to be. UECE 2009.1
Did you feel good in the park last Sunday?
No, I didnt. I didnt feel good in the park last Sunday. Instead of Eating to Diet, Theyre Eating to Enjoy
By TARA PARKER-POPE
Porm: Were you sad in the Park last Sunday? www.nytimes.com, Sep. 16, 2008
No,I wasnt. I wasnt sad in the park last Sunday.
AFTER decades of obsessing about fat, calories and carbs,
many dieters have made the unorthodox decision to simply enjoy
food again.
Q U E S T I O n S That doesnt mean theyre giving up on health or even weight
01. In the sentence Thousands of visitors () were surprised loss. Instead, consumers and nutritionists say they are seeing a
when they SAW and HEARD this invention, the capital verbs shift toward positive eating shunning deprivation diets and
are the past tenses of: instead focusing on adding seasonal vegetables, nuts, berries and
a) save and hide. c) say and hate. other healthful foods to their plates.
b) sit and have. d) see and hear. For 32-year-old Rina Gonzalez-Echandi of Los Angeles, gi-
ving up calorie counting and packaged foods and adding real
02. When I asked if Jim liked his job he replied that he ________. food back into her diet has helped her maintain her weight and
a) did c) do even be happier. She used to watch fat and calories so obsessively
b) does d) doing she would sometimes avoid socializing.
You forget how wonderful it is to have a meal with friends
03. He __________ to return to his home. and family, said Ms. Gonzalez-Echandi, a special-education aide
a) not wanted c) wanted and mother of a 10-year-old daughter. I realize I had taken that
b) did wanted d) does want joy away from myself.
Now she focuses on the pleasure of eating fresh, home-cooked
04. Brazil _________ last years world soccer championship. food. She has started cooking with olive oil and occasionally but-
a) win c) won ter, and has increased her consumption of nuts and peanut but-
b) wins d) to win ter. She even got to know her grocer to find out which fruits and
vegetables are in season and grown locally.
05. His body __________ in the cemetery. The market research firm NPD Group gets a glimpse of na-
a) lies c) lays tional eating habits through the food diaries it has collected from
b) laid d) lain 5,000 consumers since 1980. The percentage of those consumers
who are on a diet is lower than at any time since information
06. Assinale a alternativa que completa a frase corretamente: on dieting was first collected in 1985. At the peak in 1990, 39
I didnt come to class yesterday because ______________. percent of the women and 29 percent of the men were dieting.
a) I am feeling sick. c) I have felt sick. Today, that number has dropped to 26 percent of women and 16
b) I felt sick. d) I feel sick. percent of men.
The diarists also report eating more organic foods and whole
07. When did she _________? She left yesterday. grains, said Harry Balzer, an NPD vice president.
a) left c) leave Instead of trying to avoid things, theyve started adding
b) lived d) leaves. things, Mr. Balzer said.
Even the Calorie Control Council, which represents makers
08. In 1960, women _________ 35 percent of the new jobs of commercial diet foods, notes the percentage of people who are
created in American economy. dieting has declined to 29 percent in 2007 from 33 percent
a) take c) takes in 2004.
b) has taken d) took And there are other indicators of a shift in eating habits. In
May, the market research firm Information Resources reported
09. A alternativa que corretamente traduz (mantendo a ordem das
that 53 percent of consumers say they are cooking from scratch
palavras) Ele puxou ou empurrou a porta quando chegou l? :

58 UECEVEST

Apostilas UECEVEST mod3.indb 58 06/02/2011 09:57:49


INGLS

more than they did just six months ago, in part, no doubt, be- 06. The idea of positive eating includes
cause of the rising cost of prepared foods. a) eating whatever you want without worrying about the
Sales of organic foods have surged, and the number of farm- consequences.
ers markets has more than doubled since the mid-1990s. b) focusing on seasonal vegetables and increasing the
Nutrition experts and consumers say positive eating trends consumption of packaged foods.
are being fueled in part by the failures of the past. A national c) exercising 30 minutes a day and eating peanut butter.
epidemic of obesity suggests that the spread of diet foods, sugar- d) keeping away from deprivation diets and consuming
free soft drinks and low-fat snacks hasnt helped people manage healthful foods.
their weight.
Cynthia Sass, a New York dietitian and author who was a
spokeswoman for the American Dietetic Association from 2001
to 2007, said many clients embrace positive eating after years of
failed dieting. They would much rather focus on what to eat 01. Pearl Carlson was shaken awake at 3:30 a.m. by an urgent
instead of what not to eat, Ms. Sass said. tugging. King, the family dog, was trying to pull her out of the
Most people I have encountered have a track record of try- bed. Then she smelled smoke and heard the crackle of flames
ing dierent things that didnt work for them. engulfing her parents Granite Falls, Wash., home. Pearls screams
Meanwhile, books like Gary Taubess Good Calories, Bad roused her mother, Fern, and father, Howard, who had recently
Calories (Alfred A. Knopf, 2007) and Michael Pollans In De- been hospitalized for lung disease. Helping Howard to a first-floor
fense of Food (Penguin, 2008) have prompted a rethinking of window, Fern told him to climb out, then ran to her daughter.
Americans eating habits and dependence on processed and re- The two escaped through Pearls bedroom window.
fined foods. Still inside, King appeared at Pearls window, making
Martha McClintock, 46, of Riverdale, in the Bronx, said she squeaking sounds as he tried in vain to bark. When the German
was more focused these days on adding healthful foods like avo- shepherd-husky mix ran toward the master bedroom, Fern
cados, blueberries and walnuts to her plate. She said she tries to realized her husband hadnt escaped yet. She made her way back
improve the quality of food she eats, such as switching to blue through the smoke and flames, following Kings whimpering to
corn chips as a snack rather than potato chips. where Howard lay semiconscious on the floor. Fern helped him
If something is high in calories, I try to look at the big pic- get outside. King emerged only after both were safe.
ture, said Ms. McClintock, a photo service account executive.
De acordo com o texto, o pai de Pearl Carlson
If youre going to indulge in something, just try and walk it o
a) trabalhava como mdico no hospital local.
or limit it to once a week.
b) seria, em breve, hospitalizado por um longo perodo.
c) estivera internado por causa de uma doena pulmonar.
d) havia muito tempo sofria de grave enfermidade.
Q U E S T I O n S e) conseguiu salvar-se rapidamente.
01. The NDP Group has found that consumers nowadays tend to
a) avoid sugar free soft drinks. 02. Accountant
b) reduce sugar intake and low-fat snacks. for downtown firm. Applicant must be good with figures
c) consume more organic foods and whole grains. and have __________ five years experience. Permanent position
d) cook with olive oil and margarine. with excellent salary benefits. Write for an interview. Personnel
Manager, P. O. Box 28, Los Angeles.
02. For many people, positive eating habits come after (Practice, Plan and Write in English)
a) months of starvation. O anncio em questo procura que tipo de profissional?
b) a long time of failed dieting. a) banqueiro d) instrutor
c) years of consuming potato chips. b) bancrio e) vendedor
d) previous successful dieting. c) contador
03. Martha McClintock says that if some food is very caloric, 03. Steve is filling up his pencil case before school. He wants to
you can bring one pen, one pencil, one highlighter, and one extra item.
a) limit it to once a week or exercise to burn the calories. He has black, green, or red pens, and he has pencils decorated
b) eat it once a month and walk two miles per week. with pumpkins, dinosaurs, dogs, or dots. The highlighters come
c) indulge in it for a week and then forget about it. in pink, green, and light blue. He has room left for a small ruler,
d) mix it with some slow food. an eraser, or a pack of tissues. Given these choices, how many di-
erent combinations of items could Steve put in his pencil case?
04. Rina Gonzalez-Echandi no longer counts calories nor eats a) 105 d) 111
packaged food, and this has aided her in b) 108 e) 112
a) losing 5 pounds a month. c) 109
b) avoiding socializing.
c) enjoying organic foods. 04. Mud Chic
d) maintaining her weight. Do you drive a 4x4 SUV (Sports Utility Vehicle) and never
travel _____________ than the supermarket? Now you can
05. Some of the healthy foods mentioned in the text are: give your SUV that extreme look with spray-on mud. Buy the
a) mangoes, avocados, and peanut butter. spray online at www.sprayonmud.com for just $10. The spray
b) vegetables, potato chips, and berries. contains mud from Shropshire, England. Spray it onto your car
c) walnuts, avocados, and seasonal vegetables. and it will give the impression you have just returned from a wild
d) sugar-free soft drinks, walnuts, and gluten. adventure. Colin Dowse, the owner of the company, had the idea
one night at his local pub. Some drivers have another use for the

UECEVEST 59

Apostilas UECEVEST mod3.indb 59 06/02/2011 09:57:50


INGLS

mud: they spray it over their number plates to confuse speed


cameras. Dowse warns his clients against this illegal use and the
possible fines of up to $1000 for oenders. In an case, many
speed cameras use infra-red technology and the mud is useless.
(Speak up)

A palavra que melhor completa o espao em branco na linha 2 e:


a) farthest d) far
b) more far e) farer
c) further

G A B A R I T O
modal Verbs
01. b 02 .d 03. a 04. c 05. d
06. b 07. a 08. b 09. b 10.b

Simple Present Tense


01.c 02. a 03. c 04. c 05. b
06. b 07. b 08. e 09. c 10. d

UECE 2009.2
01. b 02. c 03. a 04. d 05. b 06. d

Simple Past Regular and Irregular Verbs


01. d 02. a 03. c 04. c 05. a
06. b 07. b 08. d 09. c 10. b

UECE 2009.1
01. c 02. b 03. a 04. d 05. c 06. d

Especial EnEm
01. c 02. c 03. b 04. c

REFERnCIAS BIBlIOGRFICAS
GUERIOS, Floriano et al. Keys: volume nico. 1. Ed. - So
Paulo: Saraiva , 2006.

http://gemeos2.uece.br/cev/vest/vest20092/v92doc/v92_f1-pro-
vaing-gab1.pdf, acesso em: 06/11/2010.

http://gemeos2.uece.br/cev/vest/vest20091/v91doc/v91_f1-pro-
vaing-gab1.pdf, acesso em: 06/11/2010.

http://portal.mec.gov.br/dmdocuments/matriz_referencia_no-
voenem.pdf, acesso em: 11/11/2010.

http://www.curso objetivo.br/concursos/arquivos/lmp97njs23_
objetivo_simaberto_1508_PROVA2.pdf, 11/11/2010.

http://www.curso-objetivo.br/concursos/arquivos/0808_objeti-
vo_simaberto_2808_PROVA2.pdf, acesso em 11/11/2010.

60 UECEVEST

Apostilas UECEVEST mod3.indb 60 06/02/2011 09:57:51


P R - V E S T I B U l A R

ESPANHOL

Apostilas UECEVEST mod3.indb 61 06/02/2011 09:57:58


Caro(a) Aluno(a),

Competncia de rea 2 - Conhecer e usar lnguas(s) estrangeiras(s) moderna(s)


como instrumento de acesso a informaes e a outras culturas e grupos sociais*.
H5 - Associar vocbulos e expresses de um texto em LEM ao seu tema.
H6 Utilizar os conhecimentos da LEM e de seus mecanismos como meio de
ampliar as possibilidades de acesso a informaes, tecnologias e culturais.
H7 Relacionar um texto em LEM, as estruturas lingsticas, sua funo e seu
uso social.
H8 Reconhecer a importncia da produo cultural em LEM como representa-
o da diversidade cultural e lingustica.

Apostilas UECEVEST mod3.indb 62 06/02/2011 09:57:58


E S PA N H O L

dueo en cualquier compaa. AVISO ESPECIAL: en Espaa


los animales deben haber sido vacunados contra la rabia antes de
Bilingismo en la Educacin media su dueo solicitar la documentacin. Consultar a un veterinario.
Continuidad, no continuismo Disponvel em: http://www.agencedelattre.com. Acesso em: 2 maio 2009 (adaptado).
Aun sin escuela e incluso a pesar de la escuela, paraguayos y pa-
raguayas se estn comunicando en guaran. La comunidad para- De acordo com as informaes sobre aeroportos e estaes fer-
guaya ha encontrado en la lengua guaran una funcionalidad real rovirias na Europa, uma pessoa que more na Espanha e queira
que asegura su reproduccin y continuidad. Esto, sin embargo, viajar para a Alemanha com o seu cachorro deve
no basta. La inclusin de la lengua guaran en el proceso de edu- a) consultar as autoridades para verificar a possibilidade de
cacin escolar fue sin duda un avance de la Reforma Educativa. viagem.
Gracias precisamente a los programas escolares, aun en contex- b) ter um certificado especial tirado em outubro de 2004.
tos urbanos, el bilingismo ha sido potenciado. Los guaranha- c) tirar o passaporte do animal e logo vacin-lo.
blantes se han acercado con mayor fuerza a la adquisicin del d) vacinar o animal contra todas as doenas.
castellano, y algunos castellanohablantes perdieron el miedo al e) vacinar o animal e depois solicitar o passaporte dele.
guaran y superaron los prejuicios en contra de l. Dejar fuera
de la Educacin Media al guaran sera echar por la borda tanto
trabajo realizado, tanta esperanza acumulada.
Cualquier intento de marginacin del guaran en la educacin
paraguaya merece la ms viva y decidida protesta, pero esta pos- VERBOS
tura tica no puede encubrir el continuismo de una forma de
enseanza del guaran que ya ha causado demasiados estragos Los verbos espaoles se clasifican en trs conjugaciones:
contra la lengua, contra la cultura y aun contra la lealtad que 1 CONJUGACIN: Infinitivo terminado en -AR: Hablar
las paraguayas y paraguayos sienten por su querida lengua. El 2 CONJUGACIN: Infinitivo terminado en -ER: Comer
guaran, lengua de comunicacin s y mil veces s; lengua de im- 3 CONJUGACIN: Infinitivo terminado en -IR: Partir
posicin, no.
MELI, B. Disponvel em: http://www.sta.uni-mainz.de.Acesso em: 27 abr. 2010 Forma verbal
(adaptado.) En ella se puede distinguir: la raz o radical; a veces
caractersticas que marcan el tiempo y el modo; cuando se trata
de una forma personal, la terminacin o desinencia.
Las formas verbales pueden ser personales si indican la
E J E R C I C I O S persona y no personales si no la indican. (ver apartado de los
01. No ltimo pargrafo do fragmento sobre o bilinguismo no pronombres personales).
Paraguai, o autor afirma que a lngua guarani, nas escolas, deve Las formas no personales que no indican persona son
ser tratada como lngua de comunicao e no de imposio. tres:Infinitivo, Gerundio y Participio.
Qual dos argumentos abaixo foi usado pelo autor para defender Las formas verbales se clasifican en simples y compuestas. Las
essa ideia? formas compuestas se forman con el verbo auxiliar HABER.
a) O guarani continua sendo usado pelos paraguaios, mesmo
sem a escola e apesar dela. modos
b) O ensino mdio no Paraguai, sem o guarani, desmereceria Actualmente se consideran tres modos: 1. Indicativo. 2.
todo o trabalho realizado e as esperanas acumuladas. Subjuntivo. 3. Imperativo.
c) A lngua guarani encontrou uma funcionalidade real que El condicional (simple y compuesto), que se clasificaba antes
assegura sua reproduo e continuidade, mas s isso no como modo, ahora se incluye como tiempo en el modo
basta. indicativo.
d) A introduo do guarani nas escolas potencializou a difuso Dentro de cada modo hay tiempos.
da lngua, mas necessrio que haja uma postura tica em
seu ensino. los tiempos verbales en espaol: Verbos regulares
e) O bilinguismo na maneira de ensinar o guarani tem Los tiempos son, esencialmente, presente, pasado y futuro,
causado estragos contra a lngua, a cultura e a lealdade dos pero por ejemplo, para referirnos al pasado tenemos varios
paraguaios ao guarani. tiempos con nombres diferentes. Los tiempos pueden ser
formas verbales simples o compuestas si forman con el verbo
02. Em alguns pases bilngues, o uso de uma lngua pode se auxiliar HABER.
sobrepor outra, gerando uma mobilizao social em prol da
valorizao da menos proeminente. De acordo com o texto, no los tiempos del indicativo
caso do Paraguai, esse processo se deu pelo(a) Presente de indicativo
a) falta de continuidade do ensino do guarani nos programas
Uds./
escolares. Ud./ nosotros, vosotros,
yo t ellos/
b) preconceito existente contra o guarani, principalmente, nas l/ella -as -as
ellas
escolas.
c) esperana acumulada na reforma educativa da educao
hablar: hablo hablas habla hablamos hablis hablan
mdia.
d) incluso e permanncia do ensino do guarani nas escolas.
e) continusmo do ensino do castelhano nos centros urbanos. comer: como comes come comemos comis comen

03. Los animales vivir: vivo vives vive vivimos vivs viven
En la Unin Europea desde el 1 de octubre de 2004 el uso de
un pasaporte es obligatorio para los animales que viajan con su

UECEVEST 63

Apostilas UECEVEST mod3.indb 63 06/02/2011 09:57:59


E S PA N HOL

Futuro Imperfecto comer: comiera comieras comiera comiramos comierais comieran


Uds./ vivir: viviera vivieras viviera viviramos vivierais vivieran
Ud./l/ nosotros, voso-
yo t ellos/
ella -as tros, -as
ellas Presente perfectode subjuntivo
hablar: hablar hablars hablar hablaremos hablaris hablarn Ud./l/ nosotros, vosotros, Uds./
comer: comer comers comer comeremos comeris comern yo t
ella -as -as ellos/ellas
vivir: vivir vivirs vivir viviremos viviris vivirn haya hayas haya hayamos hayis hayan
hablar:
hablado hablado hablado hablado hablado hablado
Pretrito Imperfecto haya hayas haya hayamos hayis hayan
comer:
Uds./ comido comido comido comido comido comido
Ud./l/ vosotros,
yo t nosotros, -as ellos/ haya hayas haya hayamos hayis hayan
ella -as vivir:
ellas vivido vivido vivido vivido vivido vivido
hablar: hablaba hablabas hablaba hablbamos hablabais hablaban
comer: coma comas coma comamos comais coman Pluscuamperfectode subjuntivo
vivir: viva vivas viva vivamos vivais vivan Ud./l/ nosotros, vosotros, Uds./
yo t
ella -as -as ellos/ellas
Pretrito de indefinido hubiera hubieras hubiera hubiramos hubierais hubieran
hablar:
hablado hablado hablado hablado hablado hablado
Ud./ nosotros, vosotros, Uds./
yo t hubiera hubieras hubiera hubiramos hubierais hubieran
l/ella -as -as ellos/ellas comer:
comido comido comido comido comido comido
hablar: habl hablaste habl hablamos hablasteis hablaron
hubiera hubieras hubiera hubiramos hubierais hubieran
comer: com comiste comi comimos comisteis comieron vivir:
vivido vivido vivido vivido vivido vivido
vivir: viv viviste vivi vivimos vivisteis vivieron
Imperativo
Presente perfecto de indicativo
t vosotros, -as usted ustedes
Ud./l/ noso- voso- Uds./ellos/
yo t Habla! Hablad! Hable! Hablen!
ella tros, -as tros, -as ellas hablar:
No hables! No hablis! No hable! No hablen!
he has ha hemos habis han
hablar: Come! Comed! Coma! Coman!
hablado hablado hablado hablado hablado hablado comer:
No comas! No comis! No coma! No coman!
he has ha hemos habis han
comer: Vive Vivid! Viva! Vivan!
comido comido comido comido comido comido vivir:
No vivas! No vivis! No viva! No vivan!
he has ha hemos habis han
vivir:
vivido vivido vivido vivido vivido vivido

Condicional Simple
E J E R C I C I O S
Uds./ 01. (UECE/2010.2) La forma propuso pertenece al verbo
Ud./l/ vosotros,
yo t nosotros, -as ellos/ PROPONER y est en el Pretrito Indefinido. EL verbo AN-
ella -as
ellas DAR en el mismo tiempo y persona es
hablar: hablara hablaras hablara hablaramos hablarais hablaran a) ando. c) anduso.
comer: comera comeras comera comeramos comerais comeran b) anduvo. d) andu.
vivir: vivira viviras vivira viviramos vivirais viviran
02. (UECE/2010.1) En la frase Mara ha llegado retrasada la
Condicional perfecto forma verbal est en el siguiente tiempo
Ud./l/ nosotros, vosotros, Uds./ a) pretrito pluscuamperfecto.
yo t b) pretrito perfecto de indicativo.
ella -as -as ellos/ellas
habra habras habra habramos habrais habran c) pretrito indefinido.
hablar: d) pretrito imperfecto de subjuntivo.
hablado hablado hablado hablado hablado hablado
habra habras habra habramos habrais habran
comer: 03. (UECE/2009.2) La forma verbal apretarse lleva el pronom-
comido comido comido comido comido comido
habra habras habra habramos habrais habran bre pospuesto por estar en el infinitivo. Qu otros dos tiempos
vivir: verbales tambin obligan ese uso?
vivido vivido vivido vivido vivido vivido
a) Gerundio y participio.
los tiempos del subjuntivo b) Participio e imperativo.
Presente de subjuntivo c) Gerundio y potencial.
d) Imperativo y gerundio.
Ud./l/ nosotros, vosotros, Uds./ellos/
yo t
ella -as -as ellas
04. (UECE/2009.2) La forma verbal ha decidido est en el
hablar: hable hables hable hablemos hablis hablen
a) pretrito pluscuamperfecto.
comer: coma comas coma comamos comis coman b) pretrito perfecto de indicativo.
vivir: viva vivas viva vivamos vivis vivan c) pretrito anterior.
d) pretrito indefinido.
Pretrito Imperfecto
Ud./l/ nosotros, vosotros, Uds./ 05. (UECE/2008.2) Enseguida empez a llover . El verbo em-
yo t
ella -as -as ellos/ellas pezar, que en la oracin arriba trascrita se emplea en el pretrito,
hablar: hablara hablaras hablara hablramos hablarais hablaran conjugado en el potencial imperfecto se escribe

64 UECEVEST

Apostilas UECEVEST mod3.indb 64 06/02/2011 09:58:01


E S PA N H O L

a) empezara. c) empezaba. Interposio de Consoantes


b) empezara. d) empezar. C zc nos verbos terminados em acer / -ecer / -ocer / -ucir
(Exemplo: permanecer permanezco, nacer nazco) H
06. (UECE/2007.2) El verbo conocer (Vase: ninguna de las algumas excees: hacer, satisfacer, etc
personas que conoca , en la primera persona singular del presente G A/O. Normalmente ocorre na primeira pessoa do singular
de subjuntivo, es: (Ejs: hacer hago, salir salgo, poner pongo)
a) Coneza. c) Conosca. Y+ A / E / O (Ejemplos: huir huyamos, huyendo)
b) Conozca. d) Conezca. Cambio de vogal a consoante i y (Ej: caer cayendo)

07. (UECE/2007.2)El verbo saber, conjugado en el potencial Dupla irregularidade


imperfecto, tercera persona plural: Acontecem as duas irregularidades (Ejs: Tener tengo, tie-
a) Sabran. c) Supieron. nes; venir vengo, vienes; decir digo, dices; or oigo,
b) Sabrn. d) Sepan. oyes).

08. (UECE/2007.1) En tuve como siempre un sueo , el Irregulares Totalmente


verbo tener est conjugado en el pretrito indefinido. Seale, Ser soy, eres, es; Estar estoy, ests, est; Ir: voy, vas, va.
abajo, la frase en que el mismo verbo se conjuga en el potencial
simple (imperfecto). Em espanhol h dois tipos de irregularidade:
a) Tengo dudas respecto a su aprecio por el arte. Irregularidad comn (so os verbos que alteram seu radical,
b) Si tuviese tiempo viajara a menudo al oriente. mas podem serem colocados em grupos de acordo com sua
c) Ten paciencia con las personas mayores. irregularidade);
d) Tendra muchas horas para reflexionar sobre la invitacin. Irregularidad propia (so os verbos que no tem modificaes
comuns a outros verbos ou apresentam mais de uma
09. Frase en la que el verbo venir se conjuga en el potencial im- irregularidade).
perfecto:
a) en cambio vino una fragancia compuesta y oscura como la Verbos de irregularidad Comn
noche.
b) viene una taza de maravilloso caldo de oro. Primera Clase
c) las luces vendran a reflejarse en los torsos sudados. Muchos verbos de la primera y de la segunda conjugacin que
d) vendr a cortar el aire una o dos veces. tiene una E en la penltima slaba, y los verbos CONCERNIR y
DISCERNIR, diptongan la E en IE siempre que es tnica.
Esta irregularidad ocurre en todas las personas de singular
y tercera del plural de los tiempos del primer grupo. Ejemplos:
Confesar, entender, discernir, acertar, empezar, apacentar, apre-
VERBOS IRREGUlARES tar, cerrar, sembrar.
Presente de Indicativo
Dizemos que um verbo irregular quando em alguma(s) de Confieso Entiendo Discierno
suas formas ocorrem alteraes no radical ou na terminao ou Confiesas Entiendes Disciernes
em ambos. Confiesa Entiende Discierne
Confesamos Entiendemos Disciernimos
Verbos que sofrem mudanas ortogrficas Confesis Entiendis Discierns
Terminao Mudana Antes Exemplo Confiesan Entienden Disciernen
Ortogrfica de
-CAR C por QU E pecar: peque peque Presente de Subjuntivo
-GAR G por GU E entregar: entregue - entregue Confiese Entienda Discierna
Confiesas Entiendas Disciernas
-ZAR Z por C E utilizar: utilic - utilice Confiesa Entienda Discierna
-CER Confesemos Entiendamos Disciernamos
C por Z A/O vencer: venzo venza
-CIR Confesis Entiendis Disciernis
-GER Confiesen Entiendan Disciernan
G por J A/O recoger: recojo recoja
-GIR
-GUIR perda do U A/O conseguir: consigo consiga Imperativo
-QUIR QU por C A/O delinquir: delinco delinca Confiesa Entiende Discierne
Confiese Entienda Discierna
Irregularidades Voclicas Confesemos Entiendamos Disciernamos
Ditongao: 1) e ie, 2) o eu, sempre nas pessoas em que Confesad Entiended Disciernid
a slaba a qual pertencem as vogais e/o tnica. (Ejs: querer, Confiesen Entiendan Disciernan
poder, acertar). So exceo os verbos depender e pretender
(regulares). Segunda Clase
Fechamento de vogal: e i. (Ejs.: seguir siga, medir Muchos verbos de la primera y segunda conjugacin que tie-
midamos). nen una O en la penltima slaba, diptongan sta en UE en los
Ambos: e ie + e i; o eu + o u (Ejs: mentir mismos casos de la primera clase. Ejemplos: Contar, mover, acor-
mientes, mintiendo; dormir duermo, durmiendo; morirse dar, recordar, amoblar, acostar, aprobar, mostar.
me muero, se muri) Presente de Indicativo
Verbos que terminam por IAR ou UAR (Exemplo: copio, Cuento Muevo
averiguo, desvio, acto) Cuentas Mueves

UECEVEST 65

Apostilas UECEVEST mod3.indb 65 06/02/2011 09:58:02


E S PA N HOL

Cuenta Mueve PODER pude, pudiste, pudo, pudimos, pudisteis, pudieron;


Contamos Muevemos PONER puse, pusiste, puso, pusimos, pusisteis, pusieron;
Contis Mueveis QUERER quise, quisiste, quiso, quisimos, quisisteis, quisieron;
Cuentan Mueven SABER supe, supiste, supo, supimos, supisteis, supieron;
TENER tuve, tuviste, tuvo, tuvimos, tuvisteis, tuvieron;
Presente de Subjuntivo TRAER traje, trajiste, trajo, trajimos, trajisteis, trajeron;
Cuente Mueva VENIR vine, viniste, vino, vinimos, vinisteis, vinieron.
Cuentes Muevas
Cuente Mueva Participios Regulares y Participios Irregulares
Contemos Muevamos Forma no personal del verbo, susceptible de recibir marcas
Contis Muevis de gnero y nmero, que se asimila frecuentemente al adjetivo en
Cuenten Muevan su funcionamiento gramatical. En espaol, puede formar tiempos
compuestos y perfrasis verbales. Hay verbos que tienen dos par-
Imperativo ticipios, uno regular y otro irregular. En algunos casos, el uso ha
Cuenta Mueve impuesto una de las dos formas (por lo general, la regular) que es la
Cuente Mueva que se nos ha hecho ms familiar en la conversacin, mientras que
Contemos Muevamos la otra nos resulta extraa y aparentemente incorrecta. Pero no es
Contad Mueved as, ambas formas son vlidas. Por ejemplo en este listado:
Cuenten Muevan Infinitivo Participio Regular Participio Irregular
Tercera Clase
Abstraer abstrado Abstracto
Los verbos terminados en HACER (excepto HACER y sus Bendecir bendecido Bendito
compuestos, PLACER y YACER), -ECER) excepto MECER y Compeler compelido Compulso
REMECER), -OCER (menos COCER, ESCOCER y RECO- Despertar despertado Despierto
CER) y UCIR (excepto los terminados en DUCIR) toman una Elegir elegido Electo
Z antes de la C del radical cuando esta viene seguida de O o A.
Fijar fijado Fijo
Esta irregularidad ocurre en la primera persona singular del
presente de indicativo, todo el presente de subjuntivo y en tres Hartar hartado Harto
personas de imperativo. Ejemplos: Nacer, crecer,conocer, lucir, pa- Incluir incluido Incluso
recer, renacer, reconocer, desconocer, relucir, ofrecer, desmerecer. Maldecir maldecido Maldito
Presente de Indicativo Oprimir oprimido Opreso
Nazco Crezco Conozco Luzco Poseer posedo Poseso
Naces Creces Conoces Luces
Recluir recluido Recluso
Nace Crece Conoce Luce
Nacemos Crecemos Conocemos Lucemos Salvar salvado Salvo
Nacis Crecis Conocis Lucis Teir teido Tinto
Nacen Crecen Conocen Lucen
Se aconseja no usar las formas irregulares en los tiempos com-
Presente de Subjuntivo puestos, para evitar la construccin de oraciones como yo he con-
Nazca Crezca Conozca Luzca cluso mi trabajo; la polica ha preso a un delincuente; nosotros
Nazcas Crezcas Conozcas Luzcas habamos expreso nuestro punto de vista; ellos hubieran suspenso
Nazca Crezca Conozca Luzca la reunin, en lugar de yo he concluido mi trabajo; la polica
Nazcamos Crezcamos Conozcamos Luzcamos ha aprendido a un delincuente; nosotros habamos expresado
Nazcis Crezcis Conozcis Luzcis nuestro punto de vista; ellos hubieran suspendido la reunin.
Nazcan Crezcan Conozcan Luzcan Las formas irregulares slo se emplean como sustantivos o adje-
tivos, en expresiones como persona atenta, sustantivo abstrac-
Imperativo to, los tuertos, un convicto. Se exceptan de esta disposicin
Nace Crece Conoce Luce los participios irregulares de los verbos romper, rer, proveer y
Nazca Crezca Conozca Luzca prender, con los que se pueden formar las construcciones hemos
Nazcamos Crezcamos Conozcamos Luzcamos roto, han fredo, hayas provisto y haban prendido.
Naced Creced Conoced Lucid
Nazcan Crezcan Conozcan Luzcan
E J E R C I C I O S
Verbos Irregulares (Irregularidad Propia)
01. (UECE/2009.2) La forma verbal empieza pertenece a un
ANDAR anduve, anduviste, anduvo, anduvimos, verbo irregular comn de primera clase. Apunta otro verbo con
anduvisteis, anduvieron; la misma irregularidad.
CABER cupe, cupiste, cupo, cupimos, cupisteis, cupieron; a) Ofrecer c) Mecer.
DAR di, diste, dio, dimos, disteis, dieron; b) Gobernar d) Vender.
DECIR dije, dijiste, dijo, dijimos, dijisteis, dijeron;
ERGUIR ergu, erguiste, irgui, erguimos, erguisteis, irguieron; 02. (UECE/2009.1 ) Las formas tuvo, estuvo y dijo son voces
ESTAR estuve, estuviste, estuvo, estuvimos, estuvisteis, de verbos
estuvieron; a) de irregularidad comn de la primera clase.
HACER hice, hiciste, hizo, hicimos, hicisteis, hicieron; b) de irregularidad propia o especial.
IR / SER fui, fuiste, fue, fuimos, fuisteis, fueron; c) regulares, de la segunda, primera y tercera conjugaciones,
PLACER plac, placiste, placi (o plugo), placimos, placisteis, respectivamente.
placieron (o pluguieron); d) de irregularidad (slo) aparente.

66 UECEVEST

Apostilas UECEVEST mod3.indb 66 06/02/2011 09:58:03


E S PA N H O L

03. (UECE/2006.2) Seale el verbo cuya irregularidad consiste InTERPRETACIn DE TEXTO


en la adicin de una consonante en el presente de indicativo:
a) decir c) conocer
UECE 2008.1
b) beber d) empezar
01 Kassim era un hombre enfermizo, joyero de profesin,
04. (UECE/2005.2)Es cierto que encontrar en (El organismo
02 bien que no tuviera tienda establecida. Trabajaba para las
indic que 36 pases enfrentan una escasez grave de alimentos; 23
03 grandes casas, siendo su especialidad el montaje de piedras
de ellos se encuentran en frica, la regin ms afectada, seguida
04 preciosas. Pocas manos como las suyas para los engarces
por Asia y Amrica Latina) es un verbo:
05 delicados. Con ms arranque y habilidad comercial hubiera
a) regular
06 sido rico. Pero a los treinta y cinco aos prosegua en su
b) de irregularidad propia
07 pequeo taller en el stano de su vivienda.
c) de irregularidad aparente
08 Kassim, de cuerpo mezquino, rostro exange
d) de irregularidad comn
09 sombreado por rala barba negra, tena una mujer exquisita
10 y fuertemente apasionada. La joven, de origen callejero,
05. (UECE/2004.2)El verbo nacer en (la vieja casa de los abue-
11 haba aspirado a un ms alto enlace. Esper hasta los veinte
los de Aracataca, donde tuve la buena suerte de nacer y donde no
12 aos, provocando a los hombres y a sus vecinas con su
volv a vivir despus de los ocho aos), de irregularidad comn de
13 cuerpo. Temerosa al fin, acept nerviosamente a Kassim.
la tercera clase, conjugado en la primera persona singular del pre-
14 Sin embargo, no ms sueos de lujo. Su marido
sente de indicativo:
15 hbil artista an careca completamente de carcter para
a) nazo c) nazco
16 hacer una fortuna. Por lo cual, mientras el joyero trabajaba
b) nao d) nasco
17 doblado sobre sus pinzas, ella, de codos, sostena sobre
18 su marido una lenta y pesada mirada, para arrancarse
06. (UECE/2004.2)Las formas tuve, pudo , hizo y dijo
19 algo bruscamente y seguir con la vista tras los vidrios al
son voces de los verbos siguientes:
20 transente de posicin que poda haber sido su marido.
a) traer, poder, hacer, decir c) traer, podar, haber, dar
21 Cuanto ganaba Kassim, no obstante, era para ella.
b) tener, podar, haber, dar d) tener, poder, hacer, decir
22 Los domingos trabajaba tambin a fin de poderle ofrecer
23 un suplemento. Cuando Mara deseaba una joya y con
07. (UECE/2008.1) Las voces quiso, hizo y tuvo pertenecen a
24 cunta pasin deseaba ella! trabajaba de noche. Despus
verbos que son:
25 haba tos y puntadas al costado; pero Mara tena sus
a) regulares, de la segunda conjugacin
26 chispas de brillante. Poco a poco el trato diario con las
b) de irregularidad aparente
27 gemas lleg a hacerle amar la tarea del artfice, y segua con
c) de irregularidad comn
28 ardor las ntimas delicadezas del engarce. Pero cuando la
d) de irregularidad propia
29 joya estaba concluida deba partir, no era para ella caa
30 ms hondamente en la decepcin de su matrimonio.
08. Supo sabe sabr ha sabido
31 Se probaba la alhaja, detenindose ante el espejo.
A sequncia correta dos tempos verbais do verbo saber no
32 Al fin la dejaba por ah, y se iba a su cuarto. Kassim se
Modo Indicativo, dados acima, ?
33 levantaba a or sus sollozos, y la hallaba en la cama, sin
a) Pretrito Indefinido/presente/Futuro/Pretrito Perfecto.
34 querer escucharlo.
b) Pretrito Perfecto/Presente/Futuro/Pretrito Indefinido.
QUIROGA, Horacio. Cuentos de Amor, Locura y Muerte, Editores
c) Pretrito Pluscuamperfecto/presente/Futuro/Pretrito
Uruguayos, Montevideo, 2004, p. 35.
Imperfecto.
d) Pretrito Imperfecto/Presente/Futuro/Pretrito
Pluscuamperfecto.
E J E R C I C I O S
09. (UECE 2000.2) El verbo saber, que es un verbo de irregulari-
dad propia, se emplea en el potencial imperfecto en la alternativa: 01. El texto trascrito se refiere a un hombre que:
a) Creo que sabra llegar a tu casa a) tena su propia tienda pero trabajaba para otras casas en una
b) No saba que te ibas a ir tan pronto vivienda contigua al tejado
c) ayer supe que llegar maana b) haba sido rico en una poca en la que sola montar piedras
d) No s nada de ellos preciosas
c) sus manos estaban siempre sucias cuando laboraba en los
10. (UECE/99.1) En cuanto al verbo establecer es EXACTO engarces delicados
referir que: d) trabajaba modestamente en el piso situado por debajo del
a) se incluye entre los de irregularidad propia, como hacer. nivel del suelo
b) gana una Z delante de la C radical en el presente de indicativo.
c) se conjuga como mecer y remecer. 02. De conformidad con el texto, el personaje descrito, llamado
d) es de la cuarta clase de irregularidad comn. Kassim:
a) era una persona robusta y tena una barba espesa y abundante.
b) tena una mujer cuyo origen era modesto y que so ser rica.
c) era casado con una seora cuyos padres haban sido dueos
de una cadena de tiendas.
d) posea una esposa joven, enclenque, extraa, que lo amaba
demasiado.

03. Por lo expuesto en la narracin se extrae que:


a) el joyero era un hombre manifiestamente malo, ambicioso,

UECEVEST 67

Apostilas UECEVEST mod3.indb 67 06/02/2011 09:58:04


E S PA N HOL

sin carcter. http://www.guiapraticodeespanhol.com.br/2010/05/o-passado-


b) aunque l fuera un hbil artista, era su cnyuge quien em-espanhol-ii-preterito.html, acesso em: 21/11/2010.
sostena a la familia.
c) su esposa deploraba el hecho de que no era rica. http://aprenderespanholesfacil.wordpress.com/2008/08/25/
d) apenas la joya era concluida, el joyero se dispona a usarla en participios-regulares-y-participios-irregulares/, acesso em:
la calle. 21/11/2010.

04. Tal y como los vocablos montaje (lnea 03) y origen (lnea
10), son heterogenricos, mejor dicho, divergen del portugus en
el gnero:
a) fraude, lumbre, sangre
b) especie, luz, monte
c) drama, corriente, humor
d) ave, guila, arma

05. Ubique el sustantivo cuyo significado cambia segn el gnero


en que se emplee:
a) piedras (lnea 03)
b) barba (lnea 09)
c) ardor (lnea 28)
d) cama (lnea 33)

06. Las locuciones sin embargo (Sin embargo, no ms sueos de


lujo. / lnea 14) y an (Su marido hbil artista an careca
/lneas 14/15) exhiben como sinnimos, respectivamente:
a) sin dificultad, adems
b) entretanto, tambin
c) en realidad, pero
d) con todo, todava

G A B A R I T O
Especial EnEm
01. d 02. d 03. e

Verbos
01. b 02. b 03. d 04. b 05. b
06. b 07. d 08. d

Verbos irregulares
01. b 02. b 03. c 04. d 05. c
06. d 07. b 08. a 09. a 10. b

UECE 2008.1
01. d 02. b 03. c 04. a 05. b 06. d

REFERnCIAS BIBlIOGRFICAS
http://www.c7s.com.br/vs4/index.php/gabaritos-enem-2010,
acesso em: 18/11/2010.

http://www.infoescola.com/espanhol/verbos-2/, acesso em:


18/11/2010.

http://users.ipfw.edu/jehle/courses/vtenses.htm#fut, acesso em:


18/11/2010.

http://www.apostilasgratuitas.info/component/content/
article/72-idiomas/224-dicas-de-espanhol, acesso em:
20/11/2010.

68 UECEVEST

Apostilas UECEVEST mod3.indb 68 06/02/2011 09:58:05


P R - V E S T I B U l A R

GEOGRAFIA

Apostilas UECEVEST mod3.indb 69 06/02/2011 09:58:11


Caro(a) Aluno(a),

Para facilitar o acompanhamento de tais contedos, abaixo esto indicadas as nomenclaturas utilizadas pela
UECE e pelo ENEM:

Contedo UECE ENEM


Geografia da Populao Crescimento demogrfico e teorias Estrutura da Populao;
demogrficas Movimentos Populacionais;
Populao Brasileira
Geografia do Nordeste Estudo das sub-regies do Nordeste A Diviso Regional Brasileira-
Regio Nordeste
Geografia do Cear Estudos dos componentes do espao Regio Nordeste: Estado do Cear;
fsico e humano do Estado do Cear.

Apostilas UECEVEST mod3.indb 70 06/02/2011 09:58:12


GEOGRAFIA

CRESCImEnTO DEmOGRFICO mUnDIAl CV positivo = TN > TM


CV negativo = TN < TM
Evoluo do crescimento da populao mundial
As estatsticas demonstram que a populao mundial tem Pais Crescimento vegetativo
apresentado um crescimento contnuo ao longo do tempo, po-
rm com intensidades e propores diferenciadas. Alemanha -0/4
Argentina 1/19
Evoluo do crescimento da Populao mundial Austrlia 0/99
Ano Populao (milhes de hab.) Brasil 1,22
8.000a.c. 5
Rssia -0/64
1d.C. 250
1650 500 India 1/52
1800 900 Uganda 3/19
1850 1.200 Nigria 2/61
1900 1.600
1950 2.500 A Taxa de Natalidade a relao entre o nmero de nasci-
mentos ocorridos em 1 ano e o nmero de habitantes. Uma taxa
2000 6.100 de natalidade de 300/00 (por mil) significa que nasceram trinta
2025 8.200* crianas (vivas) para cada grupo de mil habitantes em um ano.
2050 9.500*
* Estimativa N de nascidos X 1 000
TN =
Populao Absoluta
Observa-se, de acordo com os dados da tabela, que at o in-
cio do sculo XIX o ritmo de crescimento da populao era mui-
to lento. Em compensao, nos 160 anos seguintes (1800-1960) Pas Taxa de natalidade (em %)
ela triplicou para, finalmente, nos ltimos 52 anos, novamente Frana 12
dobrar, atingindo 6 bilhes. Portanto em 2 sculos a populao
mundial passou de 1 bilho para 6 bilhes de habitantes. Atual- Uganda 51
mente, a cada ano que passa, a populao mundial acrescida de Alemanha 8,2
aproximadamente 130 milhes de pessoas. Argentina 19
O crescimento demogrfico China 14
O crescimento da populao est basicamente relacionado Japo 9,2
com dois processos: o crescimento natural ou vegetativo que ndia 24
medido pela diferena entre nascimentos e mortes no interior de
uma populao, e o de migrao, que consiste na diferena entre EUA 13
entrada e sada de pessoas de um territrio. Onde, por qualquer Brasil 19,2
motivo, o nmero de mortes supera o de nascimentos e houve
sada de populaes, o crescimento negativo; ocorrendo o con- Austrlia 12,7
trrio, ele positivo e, quando o nmero da populao estvel, Sucia 8,2
ele nulo. Mxico 22,2
O crescimento demogrfico calculado a partir da seguinte
frmula: Importante ressaltar que a taxa de natalidade se diferencia da
taxa de fecundidade, sendo esta ltima a mdia de filhos por mu-
CD = CV + (Imigrao Emigrao lher em idade reprodutiva (15 a 45 anos), sendo que em muitos
pases subdesenvolvidos os ndices de mortalidade infantil so
Onde: muito altos se comparados com os pases do primeiro mundo
CD: Crescimento Demogrfico ou mesmo alguns pases que, apesar de serem subdesenvolvidos,
CV: Crescimento Vegetativo conseguiram reduzir a taxa de fecundidade.
Imigrantes: Nmero de pessoas que entram em uma deter-
minada rea. Pais Taxa de fecundidade
Emigrantes: Nmero de pessoas que saem de uma determi-
nada rea. Espanha 1,1
Alemanha 1,3
O crescimento demogrfico pode variar de uma regio para
Canad 1,5
outra em funo do nvel de desenvolvimento econmico/ ndice
de escolaridade, poltica demogrfica etc. Uganda 7,1
Brasil 2,3
O crescimento vegetativo
O crescimento vegetativo ou natural resultante das diferen- Paquisto 5,3
as entre as taxas de Natalidade e Mortalidade Estnia 1,2

CV = TN TM A taxa de mortalidade a relao entre o nmero de bitos


ocorridos em um ano e o nmero de habitantes. Assim, se falar-

UECEVEST 71

Apostilas UECEVEST mod3.indb 71 06/02/2011 09:58:12


G E O G R AFIA

mos que a taxa de mortalidade de 20%, isto quer dizer que, para Maiores gastos com filhos principalmente nas reas urbanas,
cada mil habitantes, morrem 15 pessoas em um ano. as famlias sabem que os custos com alimentao, vesturio,
transporte, educao e sade so cada vez mais altos, o que
Pas Taxa de mortalidade (em %) impossibilita famlias grandes.
O aborto. Embora ilegais e considerados criminosos, na maio-
Frana 9
ria dos pases, os abortos so cada vez mais praticados em razo
Uganda 21 tambm das alteraes do comportamento sexual e do temor
Alemanha 10,8 de uma gravidez indesejada.
Argentina 7,8
China 7 Reduo das taxas de mortalidade
Japo 8,3 A queda dos ndices de mortalidade decorre dos avanos m-
ndia 8,4 dicos, aumento da estrutura hospitalar, programas de vacinao e
melhor saneamento bsico.
EUA 8,4
Brasil 7
Austrlia 7,4 E X E R C C I O
Sucia 10,6
01. (FGV) No incio deste novo milnio, a Diviso de Populao
Mxico 5,1 da ONU havia prognosticado uma populao de 9,3 bilhes de
pessoas para 2050, mas a nova reviso reduziu a cifra para 8,9
A mortalidade infantil corresponde ao nmero de bitos
bilhes, indicando que a populao mundial ser menor que o
at o primeiro ano de vida para cada grupo de mil habitantes,
esperado. Fonte: O Estado de So Paulo, 26/02/2003. Dentre os
sendo que estes ndices se constituem em um bom indicador da
fatores que contriburam para essa diferena, podem-se destacar:
qualidade de vida. O grupo com menores taxas de mortalidade
a) na Amrica Latina, a mudana de orientao da Igreja
infantil formado pelos pases do primeiro mundo e algumas na-
Catlica, com maior tolerncia para com o uso de mtodos
es do terceiro mundo. Exemplos: Sucia (3%), Japo e Frana
anticoncepcionais e o apoio ao planejamento familiar.
(4%), Austrlia e Canad (5%). Afeganisto (163%), Serra Leoa
b) no continente africano, o efeito da expanso da AIDS sobre
(154%), Iraque (93%) esto entre os pases que apresentam os
as populaes jovens, diminuindo a expectativa de vida e
maiores ndices.
aumentando os ndices de mortalidade.
c) o sucesso das campanhas para a reduo das taxas de
natalidade na ndia, atravs de leis punitivas sobre os casais
que ultrapassam a mdia de um filho por casal.
d) a intensa urbanizao nos pases pobres, nas ltimas dcadas,
o que fez melhorar as condies de vida de suas populaes,
permitindo melhor planejamento familiar.
e) a melhoria dos nveis educacionais em todos os pases, pela
ao da UNESCO, o que fez maior nmero de jovens tomar
conhecimento dos mtodos anticoncepcionais.

02. (UFPI) O conceito de crescimento natural, ou vegetativo,


a diferena entre:
a) os ndices de natalidade e os de vida mdia.
b) as taxas de emigrao e de imigrao.
c) as taxas de natalidade e as de mortalidade.
d) as taxas de mortalidade geral e de mortalidade infantil.
Tendncias do ritmo de e) o nmero de nascimento e o nmero de migrantes.
crescimento da populao mundial
Crescimento demogrfico da populao mundial apresenta 03. (UECE) Analise com ateno os dados do quadro:
uma tendncia estabilizao do ritmo de seu crescimento, resul-
tante do declnio das taxas de natalidade e mortalidade. Este fato Paises de renda media (entre 1993 e o ano 2000)
decorre da chamada transio demogrfica, teoria elaborada para Natalidade Mortalidade Crescimento
Pas
explicar a desacelerao do ritmo de crescimento. (%) (%) anual (%)
Brasil 24 7 1,7
Reduo das Taxas de natalidade Mxico 27 5 2,2
Chile 22 6 1,6
Causas Coreia do Sul 16 6 1,0
Maior participao da Mulher no mercado de trabalho. cada Fonte: Banco Mundial. Relatrio sobre o desenvolvimento mundial 1995; o
vez mais comum mulheres que buscam empregos fora do lar. trabalhador e o processo de integrao mundial.
Para elas, a maternidade seria um obstculo para a vida profis-
sional. A partir de suas observaes, marque o correto:
Mtodos anticoncepcionais as mudanas dos hbitos sexuais a) Todos os pases apresentam altos ndices de mortalidade e
e a diminuio de restries do tipo moral fazem com que, nos de crescimento vegetativo, refletindo o modo tradicional de
grandes centros, seja bastante comum o uso de mtodos para suas sociedades rurais.
evitar a gravidez, tais como plulas anticoncepcionais, tcnicas b) Esses dados nos oferecem uma viso da distribuio espacial
de esterilizao e preservativos, estes ltimos so cada vez mais dos idosos e dos jovens.
utilizados em razo de doenas de origem sexual, notadamente c) Temos um grupo de pases com uma estrutura econmica
a AIDS (Sndrome da Imunodeficincia Adquirida). muito frgil, apoiada em atividades agrrias.

72 UECEVEST

Apostilas UECEVEST mod3.indb 72 06/02/2011 09:58:13


GEOGRAFIA

d) Os pases relacionados so de industrializao recente, Pode-se concluir que os dados revelam:


nos quais a urbanizao se encontra em estgio adiantado, 1) que o pas E assemelha-se a pas desenvolvido;
promovendo a reduo do crescimento populacional. 2) caractersticas de todos os pases da frica e da sia
Meridional.
04. (URCA/2000) Tratando-se da dinmica populacional, cor- 3) uma gradao diferenciada de nvel de desenvolvimento de
reto afirmar que: diferentes pases;
a) A ideia de exploso demogrfica, como exploso 4) que os pases A, B e C apresentam baixas condies de vida;
incontrolvel e malfica, acita pelos que defendem medidas 5) caractersticas de todos os pases do continente europeu.
de controle da natalidade.
Esto corretos:
b) O rtmo de crescimento da populao mundial se acelerou
a) 1, 2 e 3. d) 2, 4 e S.
enormemente a partir do incio do sculo XX.
b) 1, 2 e 4. e) 3, 4 e S.
c) As taxas de natalidade declinaram antes e mais rapidamente
c) 1, 3 e 4.
que as de mortalidade.
d) Nos pases desenvolvidos ainda existem um elevado
crescimento demogrfico.
e) Ainda h nos pases do hemisfrio sul um descompasso entre
natalidade muito baixa e a mortalidade que se manteve em AS TEORIAS DEmOGRFICAS
nveis altos.
Introduo
05. (UFC) Em realao ao estudo da populao falso afirmar que: O problema do crescimento demogrfico acelerado tem sido
a) As polticas de populao empregadas no Brasil tm uma das temticas mais discutidas no campo das cincias huma-
interferido na queda da taxa de fecundidade. nas. Entre os aspectos que determinaram o acirramento dessa
b) Segundo os neomalthusianos o crescimento demogrfico controvrsia, particularmente no sculo XIX, est a publicao da
acelerado do terceiro mundo obstculo ao obra malthusiana, que chamava a ateno para o acelerado cres-
desenvolvimento econmico. cimento da populao europeia no perodo que se seguiu a Revo-
c) A ingerncia dos pases ricos nas questes demogrficas dos luo Industrial, crescimento acelerado que iria afetar tambm os
pases do terceiro mundo tem sido uma constante. pases do terceiro mundo logo aps a II Grande Guerra Mundial,
d) A situao populacional que o terceiro mundo apresenta fato que provocou o surgimento de uma nova teoria alarmista -
mais causa da situao econmica e social do que o Neomalthusianismo. Essas duas teorias foram refutadas pelos
consequncia. seguidores da Teoria Reformista que creditavam no rpido cresci-
e) Os elevados ndices de mortalidade infantil do terceiro mento demogrfico dos pases subdesenvolvidos ante a situao de
mundo decorrem das desigualdades e das contradies da misria e explorao aos quais estes esto condenados.
estrutura econmica desses pases.
A teoria malthusiana
06. (URCA) Sobre o mundo tropical, correto afirmar que: A relao entre o crescimento populacional e a fome j era
a) a populao sempre teve um elevado ndice de crescimento. preocupao de alguns pensadores no sculo XVIII, e foi a partir
b) nas ltimas dezenas de anos, a mortalidade elevou-se da publicao da teoria de Thomas Robert Malthus (1798) que
sensivelmente. a questo passou a ter maiores repercusses, advindas do prprio
c) a natalidade muito baixa. momento histrico em que vivia a Inglaterra o desenvolvimento
d) depois da ltima guerra mundial, houve uma exploso do capitalismo industrial, a intensa urbanizao e os receios do
demogrfica. nascente socialismo.
e) o elevado crescimento demogrfico provocou uma melhoria A teoria Malthusiana, exposta no livro um ensaio sobre os
das condies de vida. princpios da populao, defende dois princpios:
Podemos estar certos de que quando a populao no detida
07. (UVA) As taxas de fecundidade esto em queda em todo o por algum obstculo duplica em cada vinte e cinco anos e cres-
mundo desde os anos 60. Nos pases mais ricos, caram tanto que ce de perodo em perodo segundo uma progresso geomtrica.
ficaram abaixo dos nveis de reposio populacional. Entende- Os meios de subsistncia, nas circunstncias mais favorveis,
mos por taxa de fecundidade: nunca podem aumentar mais rapidamente que segundo uma
a) o saldo entre o nmero de nascimentos e o nmero de mortes. progresso aritmtica.
b) a relao entre o nmero de crianas com menos de cinco Os obstculos ao aumento da populao so classificados por
anos e o nmero de mulheres em idade reprodutiva. Malthus em trs ordens de fatores:
c) o nmero de crianas que morrem para cada mil nascidos as epidemias, as doenas, as guerras e outras calamidades e a
vivos. fome;
d) a quantidade de anos que um recm-nascido pode esperar as prticas de anticoncepo (condenadas por Malthus por
viver, levando em conta os recursos de seu pas. causa de sua formao religiosa pastor da Igreja Anglicana);
a sujeio moral (o homem no devia casar-se enquanto no
08. (UFC) Considere os indicadores sociais de cada pas, confor- tivesse recursos suficientes para educar a famlia).
me a tabela abaixo:
Para elaborar sua teoria, Malthus serviu-se da lei dos Ren-
dimentos Decrescentes, utilizada pelos economistas clssicos
desde o sculo XVIII. Segundo essa lei, medida que se adi-
cionam acrscimos iguais de um fator varivel a um fator fixo
de produo ou a uma combinao fixa de fatores, a partir de
um determinado ponto, os consequentes aumentos do produto
tornar-se-o menores, ou seja, os produtos marginais diminuiro.
Malthus apoiou-se no fato de que as reas de terras cultivveis so

UECEVEST 73

Apostilas UECEVEST mod3.indb 73 06/02/2011 09:58:14


G E O G R AFIA

limitadas, sendo, portanto, um fator fixo de produo. Assim, A populao jovem numerosa exige grandes investimentos em
ocorrendo o aumento da populao (fator varivel de produo), educao, sade, gerao de empregos, habitao, o que desvia
em consequncia do crescimento demogrfico chegar-se-ia a um recursos que poderiam ser investidos no setor produtivo;
ponto limite, onde o acrscimo do produto deixaria de ser pro- O grande crescimento demogrfico acarreta a reduo da renda
porcional adio de fatores variveis, para depois o produto to- per capita.
tal obtido se reduzir a zero.
A soluo defendida pelos neomalthusianos o controle cog-
Demonstrativo da lei dos rendimentos decrescentes nitivo da natalidade, atravs da instituio de normas que obri-
guem a populao a limitar o nmero de filhos.
Fator fixo de Fator varivel Produo Acrscimo da
A crtica que pode ser feita Teoria Neomalthusiana que
produo de produo obtida produo
o comportamento das taxas demogrficas no pode ser apontado
(rea de terra (n de homens em km adio de um
como causa dos problemas socioeconmicos de uma regio.
= 10ha) trabalhando a (milho) fator varivel
terra) de 10ha (n de homens)
A Teoria Reformista
10ha 0 0 Os Reformistas contrapem-se Teoria Neomalthusiana.
10ha 1 2.000 2.000 No acreditam que o grande crescimento populacional seja causa
10ha 2 5.000 3.000 da situao de misria em que vive a maioria da populao do
10ha 3 7.000 2.000 mundo subdesenvolvido.
10ha 4 8.000 1.000 Os Reformistas pregam a necessidade de implementao nos
pases subdesenvolvidos, de polticas sociais que possibilitem a
10ha 5 8.200 200 redistribuio da renda e a adequao dos fatores de produo
10ha 6 8.300 100 realidade demogrfica.
10ha 7 8.300 zero
Controle populacional na China
Crticas Teoria malthusiana Na China, onde desde 1979 se pratica a poltica do filho ni-
Malthus errou ao considerar que o crescimento populacional co para controlar o crescimento demogrfico, as famlias rurais
uma varivel independente. Esse crescimento est estreitamente continuam optando pelo nascimento de homens, como demons-
relacionado a fatores polticos, sociais, culturais e econmicos. O tra a persistncia do infanticdio feminino. Muitas famlias re-
comportamento demogrfico mundial depende das conjunturas solvem no registrar o nascimento das meninas, o que causa um
econmicas de cada nao. Pode-se perceber, por exemplo, que desequilbrio nas estatsticas oficiais sobre a porcentagem de nas-
os pases do primeiro mundo j completaram a chamada transi- cimentos de homens contra o de mulheres, que de 117 a 100,
o demogrfica, isto , completaram um processo em que pri- enquanto que a mdia mundial de 106 a 100.
meiro ocorre uma diminuio da mortalidade e, em seguida, tem Em outros casos se recorre ao aborto quando detecta e na gravi-
incio a queda da natalidade. A maioria dos pases Subdesenvol- dez que o beb do sexo feminino, apesar de o governo ter ilegali-
vidos encontra-se, ainda na primeira fase. zado a verificao do sexo nas ecografias a fim de evitar o problema.
A Teoria Malthusiana foi concebida em um momento his-
trico caracterizado pela passagem, na Inglaterra, do capitalismo
comercial para o industrial. A agricultura inglesa, nesse perodo,
passou por transformaes. Grandes reas de cultivo foram subs-
E X E R C C I O
titudas pela pecuria de ovinos, visando-se produzir a matria- 01. Durante a Conferncia Internacional sobre a Populao em
prima para a indstria txtil, havendo, em consequncia disso, Desenvolvimento, realizada na primeira quinzena de setembro de
intenso xodo rural, grande urbanizao e mudanas nos hbitos 1994, no Egito, foi amplamente discutida a mais famosa teoria
e costumes da populao. Assim, essas mudanas no poderiam sobre populao, elaborada no final do sculo XVIII, de autoria de:
servir de parmetros para se fazer projees para o resto do mun- a) F. Ratzel. d) La Blache.
do. b) T. Malthus. e) F. Engels.
Malthus no considerou uma varivel importante: o desen- c) T. Morus.
volvimento da cincia e da tecnologia aplicadas agricultura re-
sultou em um aumento considervel da produtividade agrcola, 02. Qual dos slogans a seguir poderia ser utilizado para defender
sendo que a insuficincia alimentar que atinge boa parte da po- o ponto de vista neomalthusiano?
pulao mundial no resultado do rpido crescimento demo- a) Controle populacional nosso passaporte para o
grfico e sim da concentrao da riqueza nas mos de poucos. desenvolvimento.
Malthus no levou em considerao que na medida em que b) Sem reformas sociais o pas se reproduz e no produz.
a populao melhora as suas condies de vida, a taxa de na- c) Populao abundante, pas forte!
talidade declina, sendo isto perceptvel at mesmo nos pases d) O crescimento gera fraternidade e riqueza para todos.
mais pobres. e) Justia social, sinnimo de desenvolvimento.

O neomalthusianismo 03. A teoria reformista uma resposta aos neomalthusianos. De


Teoria Neomalthusiana, a exemplo da de Malthus, tambm acordo com essa teoria, correto afirmar que.
considerada antinatalista. Teve origem aps a Segunda Guer- a) as precrias condies econmicas e sociais acarretam uma
ra Mundial, como decorrncia da necessidade de uma resposta reduo espontnea das taxas de natalidade.
s inquetaes de um mundo dividido em pases ricos e pases b) uma populao jovem numerosa, devido s elevadas taxas de
pobres. natalidade, a causa principal do subdesenvolvimento.
c) o controle da natalidade s ser possvel mediante rgidas
Os principais fundamentos desta Teoria so: polticas demogrficas desenvolvidas pelo Estado.
O crescimento populacional acelerado dificulta o desenvolvi- d) o equilbrio da dinmica populacional se d pelo
mento econmico; enfrentamento das questes sociais e econmicas.

74 UECEVEST

Apostilas UECEVEST mod3.indb 74 06/02/2011 09:58:14


GEOGRAFIA

04. A teoria neomalthusiana difere um pouco dos conceitos ori- c) o nico argumento de consenso entre as vrias teorias
ginais de Malthus na medida em que relaciona populao e de- populacionais refere-se necessidade de diminuir o
senvolvimento econmico, em lugar de populao e produo contingente populacional para que a renda per capita
de alimentos. Assim, o conceito terico diretamente relacionado aumente e, consequentemente, haja uma elevao qualitativa
com a teoria neomalthusiana do nvel de vida no pas.
a) exrcito de reserva. d) os altos ndices de crescimento demogrfico dos pases
b) mais-valia. latino-americanos (considerados os mais altos do mundo)
c) encargo econmico. tornam-se um obstculo ao desenvolvimento econmico,
d) leis dos rendimentos decrescentes. pois cria-se uma estrutura etria formada essencialmente por
e) economia de mercado. jovens e crianas, exigindo investimentos no produtivos
hospitais, escolas etc., desviando recursos que poderiam ser
05. A soluo malthusiana para o crescimento populacional re- diretamente produtivos como a construo de fbricas.
sidia na expresso moral restraint. A conteno moral significava e) um dos grandes desafios atuais da humanidade encontrar a
adiar a data do casamento e adotar uma estrita abstinncia sexual. melhor soluo para deter o crescimento populacional que se
Pessimista, o autor no achava, contudo tal fosse possvel processa de forma acelerada em todos os pases do mundo.
(Moacyr Scliar. Folha de So Paulo, 15/11/98, em A Bomba demogrftca.)
Sobre a questo populacional, incorreto afirmar que:
a) o autor a que se refere Scliar, o reverendo Malthus,
sustentava a tese segundo a qual os meios de subsistncia DISTRIBUIO DA POPUlAO mUnDIAl
crescem em proporo aritmtica, enquanto a populao
cresce em proporo geomtrica. Fatores de distribuio da populao mundial
b) em seu ensaio, sobre o Princpio da Populao, Malthus A populao da Terra no est distribuda igualmente em
defendia a ideia de que, abandonada a si prpria, a todas as partes do globo. Ao contrrio, h excesso de gente em
humanidade acabaria consumindo os recursos do planeta. algumas regies e falta em outras. Os aspectos naturais, como o
c) os problemas relacionados fome no planeta esto relevo, o clima, a vegetao e os rios exercem influncia sobre a
diretamente relacionados confirmao das teses levantadas distribuio dos grupos humanos. As regies facilmente ocupa-
pelo reverendo Malthus. das pelo homem so denominadas ecmenas. As regies adver-
d) o envelhecimento da populao mexe com o sistema sas ocupao humana so chamadas de regies anecmenas.
pblico da sade, j que as doenas da velhice so crnicas e Os fatores histricos tambm podem influenciar na distribui-
degenerativas, de tratamento mais caro. o da populao mundial, reas de ocupao antiga (NE dos
e) depois dos anos 60, a emancipao da mulher e a plula USA, Sudeste Asitico) podem apresentar grandes aglomeraes
anticoncepcional facilitaram o planejamento familiar, demogrficas. Os fatores econmicos so decisivos na anlise da
alterando-se padres de comportamento populacional. distribuio da populao, pois as principais concentraes de-
06. Qual dos slogans a seguir poderia ser utilizado para defender mogrficas esto relacionas presena da atividades industriais,
o ponto de vista dos reformistas? de prestao de servios ou agropecurias.
a) Controle populacional j, ou o pas no resistir.
Densidade demogrfica ou populao relativa
b) Com sade e educao, o planejamento familiar vir por opo!
o nmero (a mdia) de habitantes por km2. Para obt-la
c) Populao controlada, pas rico!
basta dividir a populao absoluta pela rea da regio analisada
d) Basta mais gente, que o pas vai pra frente!
(pas, cidade etc).
e) Populao menor, educao melhor!
Nmero de Habitantes
07. ndia, Bangladesh e Japo so pases considerados formiguei Dd =
Extenso Territorial
ros humanos, mas apresentam diferentes perfis socioeconmi-
cos. Isto significa que: Distribuio da populao por continentes
a) a adoo da poltica neomalthusiana tem resultado numa Pela distribuio da populao, nos continentes, notamos que:
visvel melhoria de vida da populao daqueles pases. A sia o continente mais populoso, com quase 60% do total
b) as grandes concentraes populacionais no determinam mundial;
necessariamente a ocorrncia de graves problemas sociais. A sia , tambm, o continente mais povoado, com quase 80
c) a superpopulao um fator determinante para tornar esses hab/km2;
pases reas de repulso de investimentos. A Oceania o continente menos populoso e menos povoado;
d) o endividamento externo e as grandes desigualdades sociais A Antrtida o continente no habitado (despovoado).
so causas diretas da superpopulao.
e) so pases superpopulosos, que possuem problemas de
pobreza e misria, resultados do grande nmero de habitantes.
08. Sobre o crescimento populacional e as suas implicaes na
vida social e econmica, correto afirmar que
a) a teoria malthusiana baseia-se no princpio de que o
crescimento populacional ocorre num ritmo geomtrico
enquanto o crescimento dos produtos de subsistncia ocorre
num ritmo aritmtico, logo a produo de alimentos no
acompanha o crescimento populacional.
b) os governos da Frana e da Inglaterra impuseram as mais
conhecidas e difundidas polticas antinatalistas do ps- Populao relativa e superpovoamento
Segunda Guerra Mundial, reduzindo drasticamente as taxas Uma regio densamente povoada no necessariamente su-
de natalidade de seus pases. perpovoada isso porque o conceito de superpovoamento no diz

UECEVEST 75

Apostilas UECEVEST mod3.indb 75 06/02/2011 09:58:15


G E O G R AFIA

respeito apenas ao nmero de habitantes por km2, mas tambm 03. Considere as afirmativas sobre a distribuio da populao
se refere ao nvel de desenvolvimento socioeconmico e tecnol- dos Estados Unidos em seu territrio:
gico da populao em relao rea ocupada. Nesse caso, ocorre I. A populao se distribui de forma muito irregular, sendo
superpovoamento quando h descompasso do ponto de vista das registrados os maiores adensamentos nas pradarias centrais;
condies socioeconmicas da populao em relao rea ocu- II. A taxa de urbanizao muito elevada, ultrapassando os 80%;
pada. A Holanda, por exemplo, um pas densamente povoado III. O nordeste do litoral atlntico e o sul dos Grandes Lagos
(434 hab/km2) mas no superpovoado (a populao desfruta de constituem zonas de alta densidade demogrfica;
alto padro de vida em um espao muito pequeno), ao passo que IV. A industrializao, sobretudo a petroqumica, tem
pases como a ndia (247 hab/km2) e Bangladesh (740 hab/km2) provocado um rpido crescimento da populao do
so superpovoados. O superpovoamento portanto relativo. Sul, que foi, durante uns dois sculos, uma zona quase
exclusivamente agrria;
Pases mais populosos e povoados V. O litoral sul do Pacfico a Califrnia perdeu, nas ltimas
Pases mais populosos Pases mais povoados dcadas, a fora do crescimento populacional.
(em milhes) (hab/km2) Esto corretas apenas as afirmativas:
China - 1.285,00 Bangladesh - 975 a) I, lIe 111 d) II, III e IV
ndia - 1.025,10 Formosa - 610 b) I, III e IV e) II, IV e V
EUA - 285,9 Coreia do Sul- 474 c) I, IV e V
Indonsia - 214,8 Holanda - 382
04. Assinale a alternativa INCORRETA.
Brasil- 170,00 Japo - 370
a) A distribuio da populao brasileira tem como
Paquisto -145,00 Blgica - 332 componentes, alm dos fatores naturais, fatores econmicos
Rssia - 144,7 Lbano - 290 e histricos, tais como os movimentos migratrios internos.
Bangladesh -140,00 ndia - 275 b) Apesar de ser um dos pases mais populosos do mundo, o
Japo -127,00 Gr-Bretanha - 230 Brasil continua a ser um pas de baixa densidade demogrfica.
Nigria - 116,9 Alemanha - 225 c) Na atualidade, a maior concentrao populacional brasileira
encontra-se na regio Sudeste.
d) Desde a dcada de 1990, a regio Centro-Oeste tem
consolidado sua importncia como plo de atrao
E X E R C C I O populacional do pas.
01. Refere-se densidade demogrfica das regies A e B: e) Com exceo da regio Nordeste, nas demais regies
brasileiras a populao rural menor que a populao urbana.

05. O Brasil um pas populoso e despovoado. Tal contradio


aparente pode ser explicada da seguinte maneira:
a) Tem um nmero relativo de habitantes acima das mdias
normais.
b) Tem um nmero absoluto de populao correspondente ao
tamanho de sua rea.
c) Tem uma taxa de crescimento demogrfico muito baixa.
d) Tem densidade demogrfica pequena em relao ao total de
sua rea.
e) Tem populao relativa alta e populao absoluta baixa.
I. As regies A e B tm a mesma dimenso real de territrio.
II. A regio A tem maior concentrao de habitantes. 06. Observe as seguintes proposies:
III. A regio B tem menor nmero de habitantes por rea. I. Os pases que possuem uma elevada populao absoluta so
considerados povoados;
a) se todas as afirmativas estiverem corretas. II. O Brasil um pas de dimenses continentais e muito populoso;
b) se apenas as afirmativas I e II estiverem corretas. III. A Antrtida um continente que no possui populao em
c) se apenas as afirmativas I e III estiverem corretas. carter permanente;
d) se apenas as afirmativas II e III estiverem corretas. IV. O Japo um dos pases do mundo que apresenta uma das
e) se nenhuma afirmativa estiver correta. mais baixas densidades demogrficas;
V. O Instituto Brasileiro de Geografia e Estatstica IBGE o
02. Considerando-se os aspectos gerais da populao europeia rgo responsvel pelo recenseamento e divulgao oficial
INCORRETO afirmar que: dos dados de populao do pas.
a) o vale do rio Reno, devido grande concentrao-
industrial na regio do Ruhr, apresenta alta concentrao Esto corretos:
populacional. a) I, IV e V d) II, III e V;
b) atualmente a populao da Europa apresenta crescimento b) I, III e II; e) I, III, V.
lento e equilibrado devido diminuio da natalidade. c) II, I e V;
c) as terras baixas da Blgica e da Holanda, com srios
obstculos naturais, abrigam uma das menores densidades 07. Entre os pases mais populosos esto Indonsia, Brasil, Japo
demogrficas europeias. e Bangladesh. Entre os mais povoados se encontram Holanda,
d) as reas interiores da pennsula escandinava, onde os invernos Blgica e Japo. O critrio para a classificao de um pas como
so longos e rigorosos, so regies fracamente povoadas. populoso e povoado refere-se respectivamente a:
e) muito irregular a distribuio da populao europeia, havendo a) taxas de natalidade e mortalidade.
muita diferena entre pases e regies da mesma nao. b) taxas de natalidade e expectativa de vida.

76 UECEVEST

Apostilas UECEVEST mod3.indb 76 06/02/2011 09:58:16


GEOGRAFIA

c) populao absoluta e relativa. Populao Absoluta Brasileira


d) ndices de sade e morbidez. Resultado dos Censos
e) densidade demogrfica e movimentos migratrios. 1872 9.930.478
1890 14.333.915
1900 17.438.434
POPUlAO DO BRASIl 1920 30.635.605
1940 41.236.315
Dinmica da populao brasileira 1950 51.944.397
Durante meio sculo, de 1890 a 1940, a populao brasileira 1960 70.191.370
apresentou taxas de crescimento prximas a 1,8% ao ano. Nes- 1970 93.139.037
sa poca, predominava um padro de crescimento populacional
1980 119.002.706
resultante da convivncia de altas taxas de natalidade com taxas
elevadas de mortalidade. Entre 1920 e 1940, por exemplo, a taxa 1991 146.825.475
de natalidade girou em torno de 44% e a taxa de mortalidade ul- 1996 157.079.573
trapassou os 2500%. Em consequncia, o incremento vegetativo 2000 169.590.693*
manteve-se em patamares inferiores a 2%. 2025 246.000.000*
Nas primeiras dcadas do sculo XX, a maior parte da po- Projees Fonte: IBGE
pulao brasileira vivia na zona rural. As crianas participavam
desde muito cedo dos trabalhos na lavoura: uma famlia nume- Esses nmeros fundamentaram o alarmismo demogrfico que
rosa dispunha de mais trabalhadores e, portanto, de maior renda tomou conta do pas nas dcadas de 1950 e 1960. Para muitos
familiar. Isso ajuda a entender as altas taxas de natalidade ento especialista, a alta taxa de natalidade era responsvel pela pobreza
vigentes. Por outro lado os servios de saneamento bsico (abas- do pas e constitua um entrave ao desenvolvimento. Eles acre-
tecimento de gua e tratamento de esgotos) e o acesso ao sistema ditavam que controlar o crescimento vegetativo da populao
de sade eram privilgios de uma parcela minoritria da popula- era uma das tarefas mais urgentes do governo. Outros pensavam
o. As doenas epidmicas e endmicas espalhavam-se descon- justamento o inverso: a pobreza que constitua a causa da alta
troladamente. Da, as altas taxas de mortalidade. natalidade. Para eles, a melhor forma de diminuir as taxas de na-
Esse padro de crescimento populacional comeou a ser rom- talidade era elevar o nvel de vida da populao, atravs de uma
pido nos anos 1940, quando as taxas de mortalidade tenderam a melhor distribuio da renda nacional. Demografia e desenvolvi-
diminuir, lentamente a princpio. Nessa poca, iniciaram-se as mento eram temas gmeos na discusso nacional.
campanhas nacionais de erradicao de doenas epidmicas atra- No final da dcada de 1960, porm, a natalidade brasileira
vs da pulverizao de drogas que eliminam os agentes transmis- comeou a cair de forma generalizada. Essa tendncia prosseguiu
sores e da vacinao em massa. Sistemas de comunicaes e trans- nas dcadas seguintes, puxando para baixo as taxas mdias de
portes mais eficientes ajudavam a disseminar as novas prticas incremento anual da populao do pas.
mdicas nas regies mais populosas do pas. O nmero de casos Essa rpida alterao do comportamento reprodutivo da po-
de malria, tuberculose, ttano, paralisia infantil e muitas outras pulao se relaciona com as transformaes estruturais na eco-
doenas diminuiu sensivelmente a partir de ento. O sistema de nomia brasileira, nas ltimas dcadas. O Brasil se transformou
saneamento bsico era e continua sendo precrio na maioria das em um pas urbano-industrial: a mudana do foco econmico da
cidades brasileiras, mas as poucas melhorias introduzidas nesse produo e a concentrao da populao nas cidades alteraram
campo foram suficientes para fazer declinar as taxas de mortali- profundamente os comportamentos reprodutivos.
dade das populaes urbanas. Nas cidades, ter muitos filhos significa acumular despesas
Nessa etapa, o declnio da mortalidade no foi acompanhado com alimentao, sade e educao at que eles atinjam a idade
pelo declnio da natalidade. O resultado foi o aumento das taxas produtiva. Esse elevado custo de formao tem funcionado como
de crescimento vegetativo da populao brasileira: 2,39% de in- um poderoso freio natalidade, mesmo entre as famlias mais po-
cremento anual mdio entre 1940 e 1950, 2,99% entre 1950 e bres. Na dcada de 1960, cada brasileira entre 15 e 44 anos tinha,
1960, 2,89% entre 1960 e 1970. Em 1940, a populao total do em mdia, seis filhos; atualmente, o nmero de filhos por mulher
pas era de 41,2 milhes; em 1970, de 93,1 milhes, um cresci- gira em torno de 2,5. De acordo com dados oficiais recentes,
mento de cerca de 130% em apenas trinta anos. cerca de 70% das mulheres brasileiras casadas, com idades entre
15 e 44 anos, utilizam mtodos anticoncepcionais. Cerca de 45%
Brasil, de censo a censo fizeram laqueadura de trompas e no tero mais nenhum filho.
A natalidade caiu, mas a renda permaneceu fortemente con-
centrada. A misria continua existindo, s que ela mudou de
endereo e de comportamento. E ela que explica os elevados
ndices de esterilizao cirrgica vigentes no Brasil. Sem acesso
a um acompanhamento mdico de qualidade e desinformadas
sobre o seu prprio corpo e sua fisiologia reprodutiva, as mulhe-
res brasileiras recorrem cada vez mais a esse mtodo definitivo. A
demografia responde ao crescimento econmico, no ao desen-
volvimento social.
Projees recentes indicam que o Brasil atingir a estabilidade
demogrfica em 2050, quando ter populao inferior a 250 mi-
lhes de habitantes (equivalente dos Estados Unidos em 1990).
Entretanto, essa nova realidade demogrfica, marcada por taxas
de crescimento demogrfico decrescente, ainda bastante des-
conhecida no pas.

UECEVEST 77

Apostilas UECEVEST mod3.indb 77 06/02/2011 09:58:16


G E O G R AFIA

Texto complementar n 1
Transio demogrfica
E X E R C C I O
Mesmo com todas essas mudanas, a populao do pas con- 01. (UFPB) Nos ltimos anos, o Brasil passou por um processo
tinua a crescer, s que mais lentamente. Nos anos de 1990, ela de transio demogrfica que provo-cou, dentre outras mudan-
aumentou em 23 milhes de pessoas, num ritmo de crescimen- as, uma nova estrutura da populao, conforme demonstra o
to de 1,64% ao ano. A Regio Nordeste foi a que apresentou a grfico abaixo. Com base nessas informaes, correto afirmar:
mais baixa taxa de crescimento na ltima dcada. Isso ocorreu
por causa de uma combinao de fatores, como a, migrao e a
diminuio do ritmo de crescimento da populao.
Se por um lado a populao aumenta mais devagar, por outro
o brasileiro est vivendo mais. A expectativa de vida aumentou
bastante nos ltimos 50 anos. As mulheres, na mdia, vivem 7,8
anos a mais que os homens e tinham em 2000 uma expectativa
de vida na faixa dos 72 anos. J a expectativa de vida dos homens,
embora tambm cresa, est em torno de 65 anos. Em todvo
o mundo a expectativa de vida masculina inferior feminina,
mas, no Brasil, ela sofre um grande impacto da violncia. So
eles as principais vtimas de assassinato e de acidente de trnsito.
A queda na taxa de fecundidade, aliada a uma maior expectativa Adaptado de MAGNOU, D. e ARJO, R. Projeto de Ensino de Geografia.
de vida, provoca dois efeitos imediatos: a reduo proporcional do Geografia do Brasil, p. 191.
nmero de crianas e de adolescentes no conjunto da populao e o a) Os dados referentes s taxas de natalidade, em relao aos
aumento de idosos. Esse processo chamado de transio demogrfi- perodos 1940/50 e 1990/95, apontam para uma tendncia
ca. Para ter uma ideia, a participao de menores de 17 anos, que em de aumento crescente, mantendo-se o contingente de
1940 era de 55,4% no total da populao, atualmente de 46,3%. J populao jovem no pas.
as pessoas com 60 anos ou mais representam 8,6% da populao, ou b) O envelhecimento da populao brasileira resulta,
14,5 milhes de indivduos em 1991 eram 10,7 milhes. exclusivamente, das taxas de mortalidade, cuja tendncia
As alteraes na estrutura etria tm grande impacto no pla- predominante a de reduo constante.
nejamento das polticas pblicas. O aumento de jovens, que for- c) O crescimento vegetativo, no atual ritmo de desacelerao
mam a populao economicamente ativa, em idade de trabalhar, demogrfica, levar a um gradativo envelhecimento da
sinaliza a importncia de criar mais postos de trabalho. J o cres- populao brasileira.
cimento do nmero de idosos mostra a necessidade de maiores d) A relao natalidade versus mortalidade aponta para uma
investimentos no sistema de sade para atender essa populao. tendncia de aumento contnuo e progressivo da populao
Fonte: Almanaque Abril/2004 pg. 146. brasileira.
e) A tendncia de aumento constante das taxas de natalidade
Texto complementar n 2 da populao brasileira, nos ltimos anos, aponta para um
Populao mais velha equilbrio entre a populao idosa e a jovem.
O envelhecimento da populao tambm aumenta a presso so-
bre a Previdncia Social. Na medida em que os idosos vivem mais, 02. (UFF) O Censo 2000 do IBGE registrou, conforme ilustra o
eles passam mais tempo recebendo benefcios previdencirios. No grfico a seguir, significativa reduo do nmero mdio de pesso-
longo prazo, o problema tende a se agravar, pois haver um au- as na famlia em todo o pas.
mento do nmero de pessoas recebendo aposentadorias e penses,
ao mesmo tempo que se reduz, em termos relativos, a quantidade
de trabalhadores contribuindo para a manuteno da Previdncia.
Uma boa notcia revelada pelo Censo de 2000 foi a redu-
o da mortalidade infantil de 48 bitos por mil nascimentos em
1990 para 29,6 por mil em 2000, uma expressiva queda de 38%.
Foi a primeira vez na histria do pas que a taxa ficou abaixo de
30 por mil. Esse ndice se refere ao nmero de crianas entre mil
nascidas, que morrem antes de completar 1 ano de vida.
A Organizao Mundial da Sade (OMS) classifica a taxa de
mortalidade infantil dos pases em trs nveis: Primeiro - nme- Fonte: Censo 2000 IBGE
ros acima de 40 por mil so considerados muito preocupantes. Assinale a opo que apresenta consideraes adequadas acerca
Segundo - entre 20 por mil e 40 por mil, o ndice classificado dessa reduo quantitativa de componentes da famlia brasileira.
como mdio e no ideal. Terceiro - at 20 mortes por mil crian- a) A reduo do nmero mdio de membros das famlias
as nascidas vivas, a taxa considerada baixa e aceitvel. nesse no pas est associada, sobretudo nas reas de fronteiras
patamar que se encontram os pases desenvolvidos e alguns dos agrcola, s pssimas condies sanitrias e concentrao de
vizinhos do Brasil na Amrica Latina, como Chile e Argentina. terras que impedem o pleno desenvolvimento das famlias.
Outra modificao no perfil da populao apontada pelo IBGE b) As polticas demogrficas natalistas nas duas ltimas dcadas
diz respeito ao tamanho da famlia brasileira. Ela est cada vez me- do sculo XX, implementadas pelo governo federal, foram
nor. O nmero mdio de indivduos que compem uma famlia mal sucedidas, uma vez que o Brasil apresenta queda no
tpica de 3,5 pessoas. Na dcada de 1980, a formao mdia do nmero mdio de pessoas nas famlias em todo o pas.
grupo familiar era de 4,5 integrantes. Do total de famlias, 59,4% c) A grande migrao da populao do campo para as cidades,
tm filhos. H um aumento no nmero de casais sem filhos e entre fenmeno caracterstico da segunda metade do sculo
os indivduos que vivem sozinhos (8,6% das famlias). passado, a principal responsvel pela reduo das famlias
Fonte: Almanaque Abril/2004 pg. 147 em grande parte do pas, sobretudo nas periferias e nas
favelas das grandes metrpoles.

78 UECEVEST

Apostilas UECEVEST mod3.indb 78 06/02/2011 09:58:17


GEOGRAFIA

d) A grande diferena do nmero mdio de membros das II. O aumento das taxas de natalidade da populao brasileira
famlias rurais e urbanas resultou do baixo nvel cultural da vem ocorrendo paralelamente com a intensidade do
populao camponesa, incapaz de adotar um planejamento crescimento da populao urbana.
familiar mais eficaz. III. Os ndices mdios de mortalidade infantil, apesar de
e) A adoo do modo de vida urbano, pelo campo, implicando terem diminudo nas ltimas dcadas, ainda esto longe
o estmulo ao consumo de bens, utilizao de servios e de se igualar aos ndices dos pases do chamado Primeiro
s prticas de lazer, bem como as mudanas culturais nos Mundo.
relacionamen-tos interpessoais, contriburam para a reduo
do nmero mdio de pessoas nas famlias em todo o pas. De acordo com as afirmativas, pode-se dizer corretamente:
a) Apenas I verdadeira.
03. (UNIFOR) Em alguns pases emergentes tem-se observado b) Apenas II verdadeira.
alteraes na composio demogrfica, tais como a diminuio c) Apenas I e III so verdadeiras.
do percentual de jovens e o aumento dos percentuais de adultos d) Apenas I e III so verdadeiras.
e velhos. O processo descrito denomina-se: e) I, II e so verdadeiras.
a) malthusianismo d) imploso demogrfica
b) neomalthusianismo e) reforma demogrfica
c) transio demogrfica
CRESCImEnTO mIGRATRIO DO BRASIl
04. (UFPI) Com relao dinmica da populao brasileira, nos
ltimos vinte anos, correto afirmar que: Imigraes
a) a imigrao o principal fator do incremento demogrfico A imigrao para o Brasil intensificou-se a partir de 1850,
do pas. quando cessou o trfico de escravos. A maior parte dessa vinda
b) a transformao de sociedade agrria em urbano-industrial de imigrantes para o pas esteve ligada necessidade de mo-de-
resultou em um declnio da natalidade. obra para a lavoura cafeeira e foi uma iniciativa do Estado ou de
c) os ndices de vida mdia aumentaram e so equivalentes a particulares (notadamente fazendeiros).
ndices de pases europeus como Inglaterra e Sucia. Antes de 1850 j ocorrera, mas em pequeno nmero, a vinda
d) a mortalidade infantil apresentou um crescimento acentuado de imigrantes para o Brasil. Pode-se afirmar que a imigrao co-
apesar das campanhas de vacinao e melhorias sanitrias. meou em 1808, com a vinda da famlia real e a abertura dos por-
e) o crescimento natural, ou vegetativo, o nico existente nas tos s naes amigas. O Brasil tornou-se, ento, devido fuga dos
principais reas urbanas. governantes portugueses frente ao exrcito de Napoleo, a sede
do Reino. Aqui instalado, algo passou a preocupar D. Joo VI:
05. (UFPI) Com relao dinmica da populao brasileira, im- a numerosa e ostensiva populao negra. Achando que isso no
portantes transformaes vm ocorrendo nos ltimos 20 anos. A ficava bem para uma rea que, na poca, era sede da monarquia
esse respeito, assinale a alternativa correta. (e no se sabia por quanto tempo, pois os franceses poderiam
a) As taxas de natalidade vm diminuindo, porm as taxas de ficar ocupando Portugal indefinidamente), D. Joo VI tratou de
mortalidade vm apresentando aumento substancial. incentivar a vinda de colonos aorianos, em 1808, e de suos,
b) O crescimento vegetativo deixou de ser o elemento principal em 1818. Aps a independncia, em 1822, houve a vinda de
do incremento demogrfico devido ao incentivo imigrao. alemes (em 1824, 27, 29 e 30), promovida por D. Pedro I. Mas
c) As taxas de natalidade e mortalidade vm diminuindo, o volume total dessa imigrao at 1850 insignificante e sensi-
proporcionando o aumento da esperana de vida ou velmente inferior vinda de africanos como escravos.
expectativa de vida dos brasileiros. Com a intensificao das presses inglesas para o fim do trfico
d) Existe pequena relao entre a diminuio dos ndices de negreiro e com a Lei Eusbio de Queiroz, de 1850, os proprietrios
mortalidade e o crescimento das taxas de urbanizao. de terras, especialmente de fazendas de caf (atividade predomi-
e) Os ndices de expectativa de vida da populao so iguais nante no pas na segunda metade do sculo passado e primrdios
para todas as regies brasileiras e em todas as classes sociais. deste), comearam a promover a vinda de imigrantes como subs-
titutos para a mo-de-obra escrava. O perodo ureo da imigrao
06. (UFPI) Com relao dinmica da populao brasileira, ana- para o Brasil deu-se de 1850 at 1934, ocasio em que diminuiu
lise as afirmativas abaixo. sensivelmente devido, principalmente, Constituio desse ano,
I. As taxas de natalidade vm diminuindo a partir dos anos 60 que estabeleceu certas medidas restritivas vinda de estrangeiros.
como decorrncia dos novos padres de comportamento da O total de imigrantes que entrou no Brasil de 1850 at 1980
populao, vinculados urbanizao. foi cerca de 5 milhes, dos quais uns 3 milhes se fixaram defi-
II. As taxas de mortalidade vm diminuindo nos ltimos 40 nitivamente aqui, enquanto que os 2 milhes restantes acabaram
anos, em funo do alargamento do uso de antibiticos, sulfas deixando o pas. Os momentos de maior entrada de imigrantes
e inseticidas e de certas melhorias nas condies ambientais. ocorreram com a abolio da escravido (1888), que compeliu o
III. Os ndices de mortalidade infantil tm diminudo governo a buscar nova fora de trabalho na Europa ou no Japo.
substancialmente nas ltimas dcadas, sendo hoje Assim, no perodo de 1888 at 1914-18 (anos da Primeira Guer-
equivalentes aos ndices dos pases desenvolvidos. ra Mundial), ocorreram as maiores entradas anuais de imigrantes.
Sobre as afirmativas acima, pode-se dizer corretamente que: Aps a guerra, com o avano da indstria paulista, que j se
a) I e II so verdadeiras. d) Apenas I verdadeira. tornava a mais importante do pas e expandia seu mercado consu-
b) II e III so verdadeiras. e) Apenas II verdadeira. midor at o Nordeste, o deslocamento de migrantes nordestinos
c) I, II e III so verdadeiras. para So Paulo e outras reas cafeeiras comeou a tornar-se mais
importante que a entrada de imigrantes. Isso porque a expanso
07. (UFC) Sobre a dinmica populacional no Brasil, nos ltimos da indstria paulista provocou, algumas vezes, pela competio,
40 (quarenta) anos, considere as seguintes afirmativas: a falncia de empresas txteis nordestinas. A par disso, ocorreu
I. O crescimento natural ou vegetativo o principal elemento nessa poca o declnio de atividades tradicionais (como o acar
do incremento demogrfico no pas. e o algodo), que dependiam do mercado externo.

UECEVEST 79

Apostilas UECEVEST mod3.indb 79 06/02/2011 09:58:18


G E O G R AFIA

Em 1934 quando o sistema de cotas restringiu drasticamente a cimento econmico do Sudeste no incio, como caf; depois, com
imigrao para o Brasil, esta j no era to importante para suprir a indstria, mas tambm devido ao declnio econmico do Nor-
as necessidades de mo-de-obra da lavoura cafeeira, pois o deslo- deste face menor procura internacional de seus produtos agrcolas
camento de migrantes nordestinos a ultrapassava numericamente. tradicionais de exportao e estagnao de seu setor industrial.
Dessa forma, desde o final do sculo passado at princpios
deste, o Brasil foi um pas de imigrao bastante intensa, embora
sem nunca se comparar a pases como os Estados Unidos (onde E X E R C C I O
entraram cerca de 40 milhes de imigrantes), Canad ou mes-
mo Argentina. E o perodo de maior imigrao, como j vimos, 01. (UFPE) Sobre os movimentos migratrios ocorridos no Brasil,
coincidiu com o final da escravido e o esforo do governo e at a dcada de 70 do sculo passado, analise as afirmativas a seguir.
particulares em trazer fora de trabalho para ocupar o lugar do es- 1. A partir dos anos 30, do sculo XX, as migraes internas,
cravo. Essa imigrao, portanto, foi provocada e no espontnea, no Pas, comearam a ganhar fora, refletindo, de certa
ou seja, foi fruto de propaganda brasileira no exterior. E essa pro- maneira, a diferenciao socioeconmica regional.
paganda muitas vezes era ilusria, oferecendo vantagens irreais. 2. O perodo ps 80 do sculo passado foi marcado por duas
A viagem para o Brasil normalmente era paga pelo governo ou tendncias redistributivas da populao: uma centrfuga,
proprietrios de terras, e o imigrante e famlia assinavam em seu apoiada na ocupao e abertura de fronteiras agrcolas, e
pas e origem um contrato de trabalho. Esse contrato, todavia, era outra, centrpeta, que implicou a ocupao das cidades.
feito em proveito exclusivo do empregador e nunca dos empre- 3. Aps a dcada de 50 do sculo XX, h um notvel freio
gados, mas os imigrantes o assinavam iludidos pela propaganda e nos processos migratrios internos no Pas; isso foi uma
sem conhecer o meio onde iriam trabalhar. Isso deu origem a con- decorrncia do processo de industrializao.
flitos, fugas de famlias das fazendas (pois no podiam sair antes 4. As migraes pendulares, nas reas metropolitanas do Pas,
do trmino do contrato) e a emigrao daqueles que tinham re- s passaram a ocorrer a partir da dcada de 70 do sculo XX,
cursos para custear a viagem de volta ou at outro pas da Amrica. em face do Milagre Brasileiro.
O sistema de cotas, imposto pela Constituio de 1934 e pela 5. A crise do caf marca o incio do processo de
de 1937/ restringiu sensilvemente a imigrao, pois estabeleceu industrializao, no perodo ps dcada de 30, que
que, para cada ano, no poderiam entrar no pas mais de 2% estimulou consideravelmente a migrao interna.
sobre o total de entradas de cada nacionalidade nos ltimos 50
So corretas as afirmativas
anos. Alm disso, adotaram-se medidas restritivas, como a seleo
a) 1, 2 e 3 d) 2, 3 e 5
doutrinria (ideias polticas) e a exigncia de 80% de agricultores
b) 1, 2 e 4 e) 3, 4 e 5
para cada nacionalidade.
c) 1, 2 e 5
As migraes internas ou inter-regionais 02. (UFPI) O xodo rural, no Brasil, foi mais acentuado entre as
As migraes internas, entre reas ou regies do Brasil, vm dcadas de 50 e 80 do sculo XX. Esse foi um perodo em que:
ocorrendo desde a poca colonial, embora se tenham intensifica- a) as migraes pendulares diminuram, sobretudo nos grandes
do a partir de incios deste sculo, notadamente aps a Primeira centros urbanos da Regio Sudeste.
Guerra Mundial. b) as maiores metrpoles brasileiras atuaram como plos de
Durante toda a sua histria, a economia brasileira caracte- atrao populacional.
rizou-se pela existncia de fases ou ciclos, nos quais um de- c) as atividades agrcolas no Centro-Oeste e no Nordeste do
terminado produto despontava como o mais importante. Assim, pas foram intensificadas, fixando o homem ao campo.
tivemos a fase da cana-de-acar nos sculos XVI e XVII, a mine- d) diminuiu o processo de favelizao das reas urbanas
rao no sculo XVIII, o caf, no final do sculo XIX e incio do brasileiras.
XX e o surto da borracha, de 1870 a 1910. e) as migraes, no pas, passaram a ser predominantemente do
O perodo ureo de cada produto, que era determinado pela sua tipo sazonais.
valorizao no mercado internacional, sempre necessitou de mo-
de-obra, ocasionando para a regio que o produzia deslocamentos 03. (UPE) As migraes internas, de suma importncia no es-
de grandes contingentes humanos, oriundos de outras regies do tudo da Geografia Humana e Econmica, so, sem dvida, um
pas. Mas essas migraes interregionais eram relativamente fracas, fenmeno social de grande significado na vida socioeconmica
tendo se intensificado somente na segunda dcada do sculo XX. das naes. Sobre esse tema, analise as afirmativas.
Com a abolio da escravatura, a mobilidade espacial da po- 1. Os fatores de expulso, que levam s migraes, podem ser
pulao aumentou, uma vez que o trabalhador livre ou assala- agrupados em duas categorias distintas: fatores de mudana,
riado pode deslocar-se vontade pelo territrio, ao contrrio do que decorrem da introduo de relaes de produo
escravo, que objeto de compra e venda. E a precariedade das es- capitalistas nas reas de produo para subsistncia; fatores
tradas que ligavam as diversas pores do pas era to grande que de estagnao, que se manifestam sob a forma de uma
muitas vezes saa mais barato comprar escravos na frica que em crescente presso populacional sobre uma disponibilidade de
uma outra regio onde eles existissem em disponibilidade. S na reas cultivveis.
fase do caf, especialmente a partir do final do sculo XIX, que 2. O processo de industrializao, quando atinge a agricultura,
tem incio a construo de uma rede de transportes mais extensa traz consigo mudanas de tcnica e, consequentemente,
- no comeo, ferrovias, depois, j no sculo XX, predominam as aumento de produtividade do trabalho, agindo, assim, como
rodovias. Essas novas estradas facilitaram bastante as migraes um fator de estmulo migrao.
inter-regionais no Brasil. 3. Os setores da economia, que mais empregam recm-
As mais numerosas migraes inter-regionais de nossa histria chegados a diversas grandes cidades brasileiras, so, dentre
foram as de populaes nordestina e mineira para os grandes cen- outros, a construo civil e o emprego em casas de famlia.
tros do Sudeste. Essas migraes comearam j no final do scu- 4. As migraes internas, no Brasil, passaram a ocorrer, com
lo passado, aceleraram-se no incio deste e prosseguem at nossos
maior intensidade, a partir da dcada de 30, do sculo
dias, embora j no to intensamente como h algumas dcadas.
XX, quando houve uma srie de mudanas estruturais na
Esse movimento populacional, como j vimos, originou-se do cres-
economia e na poltica nacionais.

80 UECEVEST

Apostilas UECEVEST mod3.indb 80 06/02/2011 09:58:18


GEOGRAFIA

5. As migraes internas constituem um dos principais fatores a) II e III so corretas e I e IV so erradas.


responsveis pelo considervel aumento da populao b) I e III so corretas e II e IV so erradas.
urbana no Brasil. c) I, III e IV so corretas e II errada.
Assinale a alternativa que contempla as afirmativas corretas acima d) I, II e III so corretas e IV errada.
a) 1 e 3, apenas. d) 2, 3 e 5, apenas.
b) 2 e 3, apenas. e) 1,2,3,4 e 5.
c) 1, 4 e 5, apenas.
DISTRIBUIO POPUlAO DO BRASIl
04. (UFPI) Leia o texto abaixo sobre as migraes no Brasil, nos
ltimos vinte anos.
Desde o final da dcada de 80 a intensidade dos fluxos migratrios
interregionais vem diminuindo sistematicamente. O esgotamento
da oferta de emprego nos centros industriais e a concentrao fun-
diria nas antigas fronteiras agrcolas produziram um assentamen-
to indito no pas: cada vez mais, a busca de terra e de trabalho
realiza-se dentro dos limites da regio de origem da populao.
(MAGNOLI, D. e ARAJO, R. In A Nova Geografia - Estudos de Geografia do
Brasil, 2 Ed. So Paulo: Ed. Moderna, 1997, p. 180).

Com base no texto, assinale a alternativa correta.


a) Os fluxos migratrios so, principalmente, intra-regionais
em funo de mudanas econmicas e sociais no pas.
b) O crescimento do emprego industrial no Sudeste vem
incrementar os fluxos migratrios para essa regio. Distribuio Geogrfica da Populao Brasileira
c) No perodo assinalado, inexistem fluxos migratrios Apesar de ser um dos pases mais populosos do mundo, a
interregionais para todo o conjunto do pas. populao do Brasil se distribui muito irregular. como se ti-
d) A concentrao fundiria vem provocando intensos fluxos vssemos dois Brasis: o Brasil Atlntico e o Brasil Interior, onde
migratrios entre as regies brasileiras. aparecem grandes espaos quase vazios da populao.
e) A expanso da fronteira agrcola intensificou os fluxos De fato, as grandes concentraes populacionais do Brasil es-
migratrios interregionais no pas. to situadas na poro oriental ou fachada atlntica, enquanto a
poro interior do pas caracterizada pelos vazios populacionais.
05. (UFPE) As afirmativas a seguir referem-se a alguns aspectos Isso reflete um grande desequilbrio na distribuio geogrfica da
relacionados populao brasileira. Quais as que esto correta- populao, ou seja, cerca de 90% da populao brasileira est con-
mente formuladas? centrada numa faixa de aproximadamente 400km de largura, que
1) O exame da dinmica populacional brasileira permite se estende desde o Nordeste at o Sul e que corresponde a cerca de
que se chegue concluso de que o ritmo de crescimento 36% da rea total do pas, enquanto a poro interior, com cerca
populacional no tem sido regular. de 64% da rea, s concentra 10% da populao total.
2) O grande aumento da populao brasileira, na ltima Essa irregularidade possui uma herana histrica. Veja: Du-
metade do sculo XIX e primeiras dcadas do sculo XX, rante o sculo XVI, o povoamento do Brasil restringiu-se apenas
correspondeu poca de grande afluncia de imigrantes ao poro litornea, estando sujeito aos estmulos gerados pelo ci-
Pas. clo do pau-brasil e, posteriormente, ao ciclo da cana-de-acar,
3) O principal fator de crescimento da populao brasileira, no cuja produo voltava-se para a Europa.
sculo XX, foi a imigrao internacional, em face do avano No sculo XVIII, o povoamento continuou com o seu carter
da indstria automobilstica. litorneo, mas j se notam os efeitos da expanso para o interior
4) A regio Sudeste, desde a sua ocupao agrcola, caracteriza- do pas, em consequncia das bandeiras. procura de metais pre-
se pelo elevado crescimento demogrfico, decorrente, em ciosos, os bandeirantes se embrenharam em direo ao corao
parte, da sua condio de centro de atrao s migraes do continente, rompendo a linha do tratado de Tordesilhas e
internas. propiciando o incio do ciclo da minerao, responsvel pelo po-
5) No Sul do Brasil, uma rea bem povoada corresponde s voamento de vasta rea em Minas Gerais e Gois.
antigas terras florestais do Planalto Meridional ocupadas por Na Amaznia, o povoamento ligou-se s misses religiosas,
colonos europeus. ocorridas por iniciativa do governo, que permitiram a explorao
Esto corretas apenas: das chamadas drogas do serto, comercializadas na Europa da
a) 1 e 3 d) 1 e 4 mesma maneira que as especiarias orientais.
b) 3 e 4 e) 1, 2, 4 e 5 No Nordeste, a expanso do povoamento decorreu da criao
c) 2 e 5 de gado no serto, sobretudo ao longo do vale do rio So Francis-
co, melhor servido de gua e pastos naturais.
06. (UECE) O Brasil apresentou intenso fluxo migratrio du-
rante as dcadas de 1960 e 1970. Entre as principais causas deste
processo teramos:
I. a elevada concentrao da terra.
II. o desejo de mudana permanente da maior parte da populao.
III. a migrao do campo para a cidade e para a fronteira agrcola.
IV. a seletividade da distribuio das variveis sociais e
econmicas pelo territrio.
Marque a opo verdadeira.

UECEVEST 81

Apostilas UECEVEST mod3.indb 81 06/02/2011 09:58:19


G E O G R AFIA

Roraima 197.919 225.000 0,9

populosas
Acre 355.597 153.000 2,3

Menos
Amap 398.747 143.000 2,7
Rondnia 836.023 238.000 3,5
Tocantins 1.141.233 278.000 4,1
Distrito Federal 1.980.740 5.800 341,5

densidades
Rio de Janeiro 13.836.818 44.000 314,4

Maiores
So Paulo 35.941.332 249.000 144,3
Alagoas 2.719.073 27.000 100,7
Pernambuco 7.594.177 98.000 77,4
Roraima 217.000 225.000 0,9

densidades
Amazonas 1.952.288 1.577.000 1,2

Menores
Acre 355.597 153.000 2,3
Amap 398.747 143.000 2,7
Restrita ao litoral no sculo XVI, a marcha do povoamento Brasil 147.000.000 8.547.000 17,2
atingiu vastas pores interiores no sculo XVIII. Fonte: IBGE. Sntese de indicadores sociais 2000, p. 25.

A distribuio por regies


Brasil: Populao Absoluta e Densidade E X E R C C I O
Demogrfica por Regies (1999)
Densidade 01. (UVA) O Estado do Cear, segundo o novo censo do IBGE
Populao (2000), tem apenas duas cidades, alm de Fortaleza, com mais
Regio rea (Km2) Demogrfica
Absoluta de 200 mil habitantes. Jornal Dirio do Nordeste - 24/12/2000)
(HAB./Km2)
Centro-Oeste 11.616.145 1.612.077,2 7,2 So elas:
Norte 12.893.561 3.869.637,9 3,3 a) Juazeiro e Sobral c) Crato e Caucaia
b) Sobral e Crato d) Juazeiro e Caucaia
Sul 25.089.783 577.214,0 43,4
Nordeste 47.693.253 1.561.177,8 30,5 02. (UNIFOR) Em 1991, o Estado do Cear contava com apro-
Sudeste 72.297.351 927.286,2 77,9 ximadamente 6.300.000 habitantes e o Estado do Rio Grande
Brasil 169.590.693 8.547.403,5 19,8 do Norte com 2.400.000 e as densidades demogrficas dos dois
Fontes: IBGE. Anurios estatsticos e Censo Demogrfi-co 2000; Sinopse preliminar. Estados estavam em torno de pouco mais de 40 hab/km2. Dos
dados apresentados, pode-se concluir que:
Alm das grandes regies e dos estados, existem certas reas a) ambos os Estados apresentam reas territoriais semelhantes.
ou sub-regies que apresentam elevadas concentraes popula- b) a rea territorial do Estado do Rio Grande do Norte maior.
cionais (reas densamente povoadas) e outras que so verdadeiros c) a rea territorial do Estado do Cear maior.
vazios populacionais (reas fracamente povoadas). d) a rea sertaneja do Cear maior e, portanto, a populao
Exemplos: encontra-se mal distribuda pelo territrio.
reas densamente povoadas (acima de 100 hab./km2) - Regi- e) o litoral mais extenso do Estado do Rio Grande do Norte
es de Teresina (PI) e Fortaleza (CE); litoral do Nordeste (Joo possibilita uma melhor distribuio da populao.
Pessoa, Grande Recife, Macei e Aracaju); Grande Salvador e
regio de IIhus-ltabuna (BA); regio da Grande Belo Hori- 03. (UECE) Observando um mapa de densidade demogrfica do
zonte - Juiz de Fora (MG); regio do Grande Rio -Volta Re- Brasil, nota-se que maior adensamento se encontra na faixa litor-
donda (RJ); regio da Grande So Paulo - Santos - Campinas nea. Esse carter perifrico e litorneo do povoamento explicado:
- So Jos dos Campos (SP); regio da Grande Curitiba, regio a) pelos obstculos fsicos - relevo montanhoso e densas
de Blumenau - Florianpolis - Laguna (SC); regio da Grande florestas - entre o litoral e o interior.
Porto Alegre e regio de Braslia - Anpolis - Goinia. b) pela forte tropicalidade - altas temperaturas e chuvas
reas fracamente povoadas (abaixo de 2 hab./km2) - Regio copiosas - no interior do pas.
Sul e Sudeste do Piau e Sudoeste do Cear; regio Oeste da c) pela belicosidade das naes indgenas que dominavam o
Bahia; extremo Norte de Minas Gerais; regio Norte de Gois; interior.
Leste do Tocantins; extremo Sul do Maranho; a maior parte d) pelo sentido da colonizao, voltada para o
da poro central e ocidental das regies Norte e Centro-Oeste. enriquecimento da metrpole e com base numa poltica
mercantilista.
Estados de maior e menor populao absoluta e densidade
demogrfica (1999). 04. (UESB) O Brasil, o maior pas da Amrica Latina, com di-
Unidades polticas de maior e menor populao absoluta e densi- menses continentais, apresenta, na ocupao do seu territrio,
dade demogrfica (1999) caractersticas peculiares em cada uma das suas regies.
Densidade Com base nos, conhecimentos sobre o assunto, pode-se afirmar:
Unidades Populao rea (km2)
demogrfica a) O cacau, o petrleo e o fumo so produtos responsveis
Polticas absoluta
(hab./km2) pela ocupao e pela elevada densidade demogrfica do
So Paulo 35.941.332 249.000 144,3 Recncavo Baiano.
b) A baixa densidade demogrfica da regio Norte deve-se
populosas

Minas Gerais 17.341.721 588.000 29,4


Mais

Rio de Janeiro 13.836.818 44.000 314,4 escassez de recursos naturais existentes na rea.
Bahia 13.026.171 567.000 22,9 c) O clima ameno e a suave topografia explicam a elevada
densidade demogrfica da Campanha Gacha, na regio Sul.
Rio G. do Sul 9.996.461 282.000 35,4

82 UECEVEST

Apostilas UECEVEST mod3.indb 82 06/02/2011 09:58:20


GEOGRAFIA

d) As mais altas taxas de densidade demogrfica do pas Pirmide Etria


encontram-se na regio Sudeste, devido diversificao da
economia.

05. (UFAC) Podem-se identificar trs grandes correntes migrat-


rias no Brasil, nos ltimos quarenta anos:
I. do Nordeste para o Centro-Sul;
II. do Nordeste para a Amaznia;
III. uma terceira, mais recente e ainda no esgotada, do Sul para
o Centro-Oeste e Norte.
Relacionam-se com a expanso da fronteira agrcola:
a) I, II e III. d) apenas I.
b) apenas I e II. e) apenas III.
c) apenas II e III.

A ESTRUTURA ETRIA
Introduco
A distribuio da estrutura etria da populao feita em trs
faixas: Aos jovens, do nascimento at 19 anos;
Aos adultos, de 20 at 59 anos;
Aos idosos, ou a terceira idade, de 60 anos em diante.

Pirmide Etria
A construo de uma pirmide de idades simples. Dos dois
Chamamos de pases jovens, os pases que apresentam mais lados de um eixo vertical, sobre o qual so indicadas as idades, so
de 50% da sua populao na faixa etria jovem. Estas naes representadas as diversas faixas por retngulos de tamanho propor-
apresentam como caractersticas os altos ndices de natalidade e cional ao nmero de habitantes (representados no eixo horizontal),
de mortalidade. Pases como o Brasil, Mxico e Egito apresentam os dos sexos masculino esquerda e o feminino direita.
respectivamente 48,5%, 56,8% e 50,8% das suas populaes for- evidente que a estrutura etria de uma populao no se
madas pelo nmero de jovens. divide apenas naquelas trs faixas mencionadas. Pode-se tambm
Os pases maduros so aqueles que apresentam mais de dividir a populao em faixas menores, por exemplo, de cinco anos
50% da sua populao na fase adulta. Estes pases apresentam, h ou de dez anos. Isso costuma ser feito atravs de um grfico, que se
vrias dcadas, baixos ndices de natalidade e mortalidade e uma denomina pirmide etria. Ao observar a pirmide de uma popu-
expectativa de vida elevada. A se encontram pases capitalistas lao, notamos que a base da pirmide representa a populao jo-
desenvolvidos e tambm pases socialistas. vem; j o corpo, parte intermediria, representa a populao velha.
Pela anlise das pirmides etrias, percebemos que nos pases
Estrutura etria e expectativa de vida de alguns pases velhos
desenvolvidos a base menos larga, o que significa uma menor
Pas Jovens (%) Adultos (%) Velhos (%) Expectativa proporo de jovens, e tambm o cume mais largo, fato indi-
de vida (anos) cador da grande proporo de idosos na populao total. E nos
Sucia 18 59 23 77 pases subdesenvolvidos ocorre o inverso, com uma base bem larga
Reino Unido 19 60 21 75 e um cume ou topo bem estreito, o que significa uma elevada taxa
Frana 20 66 14 76 de natalidade, com grande nmero de jovens, e uma elevada taxa
Fonte: Relatrio sobre o Desenvolvimento Mundial, 1990; e Almanaque Abril, 1991. de mortalidade, com baixa proporo de idosos. Mas h inmeros

UECEVEST 83

Apostilas UECEVEST mod3.indb 83 06/02/2011 09:58:21


G E O G R AFIA

casos intermedirios (China, Brasil, Coreia do Sul e outros) de pa- ainda demonstrava o pas com o domnio da populao jovem,
ses que j foram jovens nas ltimas dcadas e atualmente esto em o que hoje j no mais verdadeiro, embora seja verdade que a
transio para a fase madura, com sensvel diminuio nos ndices proporo de velhos ainda se encontra baixa.
de mortalidade e natalidade e, consequentemente, aumento na
proporo dos idosos e diminuio relativa do nmero de jovens O Brasil Fica Mais Velho e Estvel
na populao total, que j no constituem mais a maioria absoluta.
Alguns estudiosos consideram negativa para a economia de
um pas a predominncia da faixa etria jovem em sua populao
total. Segundo eles, a elevada proporo de jovens constituiria
um peso para os adultos, que teriam de sustent-los. Essa tarefa,
porm, no poderia ser bem cumprida em virtude da baixa pro-
dutividade dos adultos, j que nesses pases a industrializao e a
mecanizao do campo so escassas.
Em consequncia, haveria necessidade de uma boa parte dos jo-
vens trabalharem, prejudicando assim seus estudos. Isso costuma ser
considerado um crculo vicioso, pois esses jovens, ao se tornarem
adultos, tero baixa especializao, no podendo, portanto, sustentar
as demais faixas etrias (jovens e velhos). Repete-se ento o passado,
ou seja, cria-se a necessidade de seus filhos trabalharem desde cedo.
Esse argumento basicamente correto, mas h outras ma-
neiras de encarar o fato. Uma elevada proporo da populao
jovem no total da populao de um pas pode vir a ser benfica
no futuro, desde que esses jovens sejam bem preparados, isto ,
desde que os pases jovens invistam mais em sade e educa-
o, criando boas escolas em grande quantidade, que preparem
futuros trabalhadores especializados com boas condies fsicas As pirmides etrias dos pases subdesenvolvidos apresentam
e mentais; e desde que procurem modernizar a economia, utili- normalmente uma base larga e um topo estreito, que indicam
zando mtodos de cultivo mais racionais no campo, incentivan- o predomnio de jovens e o reduzido nmero de idosos. J nos
do a multiplicao de indstrias nas cidades e, dessa maneira, pases desenvolvidos, observa-se geralmente uma base mais estrei-
oferecendo um nmero crescente de empregos que exijam um ta, sinal de menor proporo de jovens, e um cume mais largo,
mnimo de especializao, mas que sejam bem remunerados. devido ao grande nmero de idosos.

A estrutura etria do Brasil


Podemos dividir a populao em grupos de idades e classific-
la da maneira que se seguem:
Populao jovem: de O a 19 anos
Populao adulta: de 20 a 59 anos
Populao velha: + de 60 anos
Analisando os dados de 1940 a 2000, na tabela que se segue,
pode-se ter uma avaliao da evoluo da populao brasileira:
Brasil: Dados demogrficos de 1940 a 2000
Dados 1940 1950 1960 1970 1980 1991 1999
% de jovens (0 a 53,5 / 52,5 / 53 / 53 / 42,5 / 45 / 40 /
19/ 0 a 14 anos 42,5 42 42,5 42 38 34,5 29,4 O Brasil um pas de transio para a maturidade. Sua po-
% de adultos pulao aos poucos est envelhecendo, resultado do declnio da
42,5 / 43,5 / 42,5 / 42 / 45 / 48 / 50,8 /
(20 a 59/ 15 a 53,5 54 52,5 53 55,5 58 61,4 mortalidade e da natalidade, com um aumento da faixa etria su-
59 anos perior a 60 anos e uma diminuio relativa da faixa etria jovem
% de velhos (60 4 4 4,5 5 6,5 7 9 de pessoas com menos de 20 anos.
anos ou mais)
Expectativa de 43 46 52 53 60 65,5 68,4
A estrutura por sexo
vida em anos Quanto distribuio por sexos, verifica-se na pirmide bra-
Taxa de sileira uma grande aproximao quantitativa entre a populao
natalidade por 44 43,5 44 37,7 33 26 21,2 masculina e feminina.
1.000 Existe quase uma igualdade entre o contingente dos dois se-
Taxa de xos, sendo que a quantidade da populao feminina um pouco
fecundidade em
n de filhos por 6,2 6,2 6,2 5,7 4,3 2,7 2,7 superior masculina, conforme atesta a tabela.
mulher Brasil: Distribuio regional da populao por sexos (1999)
Fonte: IBGE. Anurios estatsticos do Brasil e sntese de indicadores sociais Regio Mulheres (%) Homens (%)
Norte 51 49
Analisando a estrutura etria do Brasil percebemos um au-
mento da terceira idade e dos adultos e uma diminuio na por- Nordeste 51,1 48,9
centagem de jovens. Da concluirmos um maior envelhecimento Sudeste 51,3 48,7
de sua populao, pois em 1950 a distribuio era a seguinte: Centro-Oeste 50,5 49,5
idosos = 40%, adultos = 43,7% e jovens = 52,3%. Isso foi uma Sul 50,7 49,3
decorrncia da diminuio das taxas de mortalidade e natalidade Brasil 50,8 49,2
e do aumento da expectativa de vida. O censo realizado em 1991 Fonte: IBGE. Sntese de indicadores sociais 2000, p. 24.

84 UECEVEST

Apostilas UECEVEST mod3.indb 84 06/02/2011 09:58:22


GEOGRAFIA

Em todas as grandes regies brasileiras predominam as mu-


lheres. As maiores diferenas a favor das mulheres so encontradas
nas regies Sudeste (elevada emigrao e mortalidade masculina),
Nordeste (maior sada de homens) e Norte (elevada mortalidade
masculina).
O fato da populao feminina no Brasil ser numericamente
superior populao masculina tambm caracterstica da maior
parte dos pases. Mas, mesmo assim, pode-se falar num certo
equilbrio biolgico, pois a diferena pequena entre os sexos.
No nascimento, a proporo entre os sexos de 105 do sexo
masculino para 100 do sexo feminino. Mas a mortalidade do sexo
masculino superior a do feminino, fazendo com que por volta
dos 4 anos de idade se estabelea um equilbrio entre ambos.

Tabela Sntese Populao masculina e feminina no Brasil,


segundo as grandes regies e as unidades da federao
Grandes regies
e unidades da Total Homens Mulheres
federao
Brasil 169.799.170 83.576.015 86.223.155
Regio Norte 12.900.704 6.533.555 6.367.149
(49,3% do total)
Rondnia 1.379.787 708.140 671.647
Acre 557.526 280.983 276.543
Amazonas 2.812.557 1.414.367 1.398.190
Rorama 324.397 166.037 158.360
Par 6.192.307 3.132.768 3.059.539
Amap 477.032 239.453 237.579
Tocatins 1.157.098 594.807 565.291
Regio Nordeste 47.741.711 23.413.914 24.321.797
(50,9% do total
Maranho 5.651.475 2.812.681 2.838.794
Piau 2.843.278 1.398.290 1.444.988 a) A pirmide etria da ndia apresenta um topo mais largo que
Cear 7.430.661 3.628.474 3.802.187 a do Mxico, indicando que a expectativa de vida maior no
Rio Grande do Norte 2.776.782 1.359.953 1.416.829 primeiro pas citado.
Paraba 3.443.825 1.671.978 1.771.847 b) A expectativa de vida muito alta para o Mxico que um
Pernambuco 7.918.344 3.826.657 4.091.687 pas em desenvolvimento e baixa para a populao indiana,
Alagoas 2.822.621 1.378.942 1.443.679 distribuda numa sociedade de castas.
Sergipe 1.784.475 874.906 909.569 c) Na ndia a taxa de natalidade muito baixa, devido ao controle
do governo enquanto que no Mxico alta, pela presena de
Bahia 13.070.250 6.462.033 6.608.217
uma poltica que incentiva o crescimento demogrfico.
Regio Sudeste 72.412.411 35.426.091 36.986.320
(50,9% do total) d) A base larga da pirmide do Mxico indica que a populao
desse pas composta por jovens, ao contrrio da ndia que
Minas Gerais 17.891.494 8.851.587 9.039.907
tem uma populao predominantemente idosa.
Esprito Santo 3.097.232 1.534.806 1.562.426 e) A taxa de natalidade alta e a expectativa de vida baixa
Rio de Janeiro 14.391.282 6.900.335 7.490.947 para os dois pases.
So Paulo 37.032.406 18.139.363 18.893.040
Regio Sul 25.107.616 12.401.450 12.706.166 02. (UFRN) As pirmides etrias do Brasil, segundo os grupos
(50,5% do total) de idades, relativos aos anos de 1970 e 2000, demonstram que
Paran 9.563.458 4.737.420 4.826.038 ocorreram mudanas em seus perfis. Essas mudanas refletem al-
Santa Catarina 5.356.360 2.669.311 2.687.049 teraes na dinmica demogrfica brasileira.
Rio Grande do Sul 10.187.798 4.994.719 5.193.079
Regio Centro-Oeste 10.187.798 4.994.719 5.193.079
Mato Grosso do Sul 2.078.001 1.040.024 1.037.977
Mato Grosso 2.504.353 1.287.187 1.217.166
Gois 5.003.228 2.492.438 2.510.790
Distrito Federal 2.051.146 981.356 1.069.790
Fonte: IBGE. Censo Demogrfico 2000

E X E R C C I O
01. (UEPB) Ao se comparar as duas pirmides etrias, pode-se
concluir que as mesmas demonstram que:

UECEVEST 85

Apostilas UECEVEST mod3.indb 85 06/02/2011 09:58:23


G E O G R AFIA

A anlise das pirmides etrias permite afirmar que:


a) a pirmide II, ao apresentar um percentual maior na
faixa dos 70 anos, sugere que o Brasil seja um pas de
populao idosa, havendo, assim, uma necessidade maior de
investimentos que beneficiem essa faixa etria.
b) a pirmide I revela a realidade populacional brasileira
de 1970, que se expressa por meio de baixas taxas de
natalidade, bem como de fecundidade, associadas elevao
da expectativa de vida dos brasileiros. a) a populao adulta teve um significativo aumento durante as
c) a pirmide II reflete um declnio nas taxas de natalidade, de trs ltimas dcadas, devido ao xodo rural.
fecundidade, de mortalidade e o aumento da expectativa de b) a populao idosa se encontrava em menor nmero em
vida da populao idosa, sugerindo o incio do processo de 2000, em funo do aumento do nmero de mortes,
transio demogrfica. causadas principalmente pela violncia urbana.
d) a pirmide I, ao apresentar uma base larga que expressa c) ocorreu um declnio das taxas de natalidade e de
altas taxas de natalidade, acompanhada de baixas taxas mortalidade, porm observa-se, comparando os dois
de mortalidade, indica um perodo de diminuio do grficos, um sensvel aumento das taxas de fecundidade.
crescimento populacional brasileiro. d) o grfico de 2000 apresenta uma maior expectativa de vida e
uma maior mdia de idade, se comparado ao grfico de 1970.
03. (UECE) O Brasil passa por importantes mudanas sociais nas e) a evoluo verificada na relao entre os grficos evidencia
ltimas dcadas. A tabela a seguir trata de uma delas. que o Brasil se encontra no incio da transio demogrfica,
Percentual de pessoas pelos domiclios particulares a chamada exploso da populao.
permanentes por situao do domiclio e sexo
Total Urbano Rural
Regio
Homem Mulher Homem Mulher Homem Mulher
Brasil 75,1 24,9 72,7 27,3 87,2 12,8 A ESTRUTURA OCUPACIONAL
Norte 77,1 22,9 71,9 28,1 90,8 9,2
Nordeste 74,1 25,9 69,8 30,2 84,8 15,2 A parcela da populao de um pas que trabalha ou que est
Sudeste 74,4 25,6 73,2 26,8 87,4 12,6 procurando servio recebe o nome de populao economicamen-
Sul 77,4 22,6 74,8 25,2 89,2 10,8 te ativa. E a parcela que no trabalha e no est empenhada na
Centro-Oeste 75,8 24,2 73,4 26,6 92,5 7,5 busca de emprego denomina-se inativa. Assim, entre os inativos
Fonte: IBGE, Censo Demogrfico 2000 incluem-se as crianas, os aposentados, os estudantes (desde que
no trabalhem) e as mulheres que exercem funes domsticas.
Analise-as e assinale a alternativa verdadeira. E, entre a populao economicamente ativa, podemos fazer uma
a) As mulheres tm participao inferior a 20% no comando diviso entre aqueles que esto efetivamente trabalhando, que se-
dos domiclios em todo o pas, especialmente nas reas ria a populao ocupada, e os que esto temporariamente sem
urbanas. emprego, os desempregados. Assim, a populao ativa aque-
b) O Nordeste apresenta o menor ndice de participao das la voltada para o mercado de trabalho, o que inclui tambm os
mulheres enquanto chefes de famlia no meio urbano, entre desempregados. Costuma-se considerar como economicamente
outros fatores, pela forte emigrao dos homens. ativos aqueles com mais de 10 anos de idade, fato esse que induz
c) O crescimento da participao das mulheres no comando a alguns pequenos erros nas estatsticas, pois nos pases subdesen-
da famlia uma das mais importantes mudanas sociais do volvidos (inclusive no Brasil) existem pessoas com menos de 10
Brasil contemporneo. anos que j trabalham, mesmo que no sejam registrados.
d) H homogeneidade regional no percentual de participao
das mulheres enquanto responsveis pelos domiclios rurais. Os setores de atividades econmicas
Setor Atividades
04. (PUCRS/Porto Alegre) Instruo: Responder questo, com
base nos grficos que relacionam a quantidade de habitantes do Primrio Agricultaura, pecuria, silvicultura, caa, pesca.
Brasil segundo idade e sexo. Secundrio Indstria de transformao e construocivil.
Tercirio Servio: comrcio, bancos, transporte, comuni-
caes, educao, administrao pblica etc.

86 UECEVEST

Apostilas UECEVEST mod3.indb 86 06/02/2011 09:58:24


GEOGRAFIA

O setor primrio abrange uma grande porcentagem da popu- Distribuio (em%) da populao ativa
lao ativa nos pases pouco industrializados como a Nigria ou a por setores da atividades no Brasil - 2000
Tailndia. medida que o pas vai se industrializando e urbani- Setor Setor Setor Tercirio
zando, diminui a porcentagem do setor primrio e aumenta a dos Primrio Secundrio
setores secundrio e tercirio.
O setor secundrio pode ser considerado como o mais im- comrcio
portante dos trs, pois o que indica o peso na indstria e na servios
economia do pas. 24,2% 29,3% 53,5% administrao pblica
O setor tercirio o mais problemtico dos trs, pois muito atividade sociais
amplo e engloba atividades muito diversas e s vezes at discre- transportes e comunicaes
pantes, como por exemplo um tcnico de computador e um ven- outros(*)
dedor de pipoca na rua. * inclui os que esto procurando emprego
Nos pases do Terceiro Mundo, coexistem um setor tercirio Fonte: TeAbela elaborada a parir de dados do IBGE. Anurio estatstico do Brasil 2000
moderno e um setor tercirio arcaico, fruto do intenso cresci-
mento urbano, sem que ocorresse, ao mesmo tempo, uma gera-
o de emprego no mesmo ritmo. E X E R C C I O
A terciarizao 01. (UECE) O desemprego, situao de ociosidade involuntria
O fenmeno da terciarizao (aumento da importncia das em que se encontram pessoas que compem a fora de trabalho
atividades tercirias no processo econmico, com a consequente de uma nao, um dos maiores problemas contemporneos em
elevao de pessoas ocupadas no setor) ocorre intensamente nas todo o mundo. O desemprego classificado em vrias categorias
economias centrais e nas perifricas, embora com caractersticas conforme sus causas.
diferentes. O tercirio muito heterogneo, e apesar de no ser Leia atentamente os itens abaixo.
difcil a compreenso dos seus dois grupos, mais complexo es- I. O desemprego tecnolgico origina-se em mudanas na
tabelecer os exatos limites entre um mais sofisticado e outro mais tecnologia de produo (aumento da mecanizao e
simples. automao ou nos padres de demanda dos consumidores,
A nomenclatura dos dois grupos varia conforme os autores: tornando obsoletas certas indstrias e profisses e fazendo
superior e inferior, desenvolvido, primitivo, novo (ou renovado) surgir outras novas.
e tradicional, tercirio de metrpole ou tercirio banal. II. Nas grandes recesses econmicas, quando a produo
A ampliao das funes dos estados contemporneos, a che- declina drasticamente, manifesta-se o chamado desemprego
gada cena das atividades de marketing gerenciamento, enge- cclico, ligado a uma fase de queda do ciclo econmico.
nharia (engineering), propaganda e assessorias (o que alguns III. O desemprego disfarado ou subemprego consiste na
chamam de quaternrio), bem como a ascenso da informtica remunerao muito abaixo de padres aceitveis, que afeta
tornaram ainda mais variadas e complexas as atividades do cha- trabalhadores no registrados, mas que, nem por isso,
mado tercirio superior. deixam de compor a fora de trabalho de uma nao.
IV. Em certas atividades, como agricultura e hotelaria, ocorre o
A populao do Brasil e a estrutura ocupacional desemprego sazonal, ligado a certas pocas do ano por no
Distribuio da populao economicamente haver oferta homognea de emprego durante o ano inteiro.
ativa do Brasil segundo os setores da economia
Assinale a alternativa verdadeira.
Setores 1940 1950 1960 1970 1980 1990 2000 a) I e IV so certas e II e III so erradas.
Primrio 70,2% 60,7% 54% 44,3% 30% 23,2% 24,2% b) II e III so certas e I e IV so erradas.
Secundrio 10% 13,1% 12,7% 17,8% 25% 23,8% 19,3% c) I, II, III e IV so certas.
Tercirio 19,8% 26,2% 33% 38% 45% 53% 56,5% d) I, II e IV so certas e 111 errada.
Analisando o comportamento da distribuio da populao 02. (UEPB) Analise a charge e as proposies que seguem:
economicamente ativa do Brasil, percebe-se a reduo do setor
primrio, que pode ser justificada pelo intenso processo de ur-
banizao do pas, o que resultou no crescimento da populao
urbana em funo do atrativo: a cidade como a maior oferta de
emprego, salrio e uma melhor assistncia mdica, hospitalar. A
mecanizao da agricultura j um fator que interfere na menor
absoro de mo-de-obra no setor primrio da economia.
Como resultante tambm da urbanizao, ocorreu um cres-
cimento da populao economicamente ativa na indstria, no
comrcio e nas diferentes atividades prestadoras de servio.
Quando comparamos a distribuio da populao ativa do
Brasil com a de pases subdesenvolvidos agrrios entendemos o
quanto o pas se urbanizou. J comparando com os pases de-
senvolvidos, percebemos que ainda somos um pas muito rural e
que o nosso setor tercirio se encontra muito inchado, ou seja,
marcado por baixos salrios, alta taxa de desemprego e mo-de-
obra ativa pouco qualificada. I. As novas formas de gerenciamento de produo e as novas
tecnologias que so inseridas no processo produtivo,
eliminam muitos empregos.
II. Uma economia desaquecida provoca ondas gigantescas de
desemprego, gerando revoltas e crises institucionais. Essa

UECEVEST 87

Apostilas UECEVEST mod3.indb 87 06/02/2011 09:58:25


G E O G R AFIA

realidade vivida por grande parte da populao em idade e) as estatsticas demonstram que no h qualquer relao
de trabalhar no Brasil. entre a dinmica econmica do pas e a qualidade de vida da
III. O desemprego tambm est associado ao enxugamento dos populao.
custos das empresas, que contribuem para demisses em
larga escala. 05. (UNIFOR) No Brasil, nestas ltimas dcadas, houve uma
significativa incorporao da mo-de-obra feminina ao mercado
Assinale a alternativa correta.
de trabalho. Este fato:
a) Apenas as proposies I e III esto corretas.
a) reduziu, consideravelmente, a proporo do chamado
b) Apenas as proposies II e III esto corretas.
desemprego estrutural.
c) As proposies I, II e III esto corretas.
b) promoveu mudanas no comportamento demogrfico,
d) Apenas as proposies I e II esto corretas.
inclusive na taxa de fecundidade.
e) Apenas a proposio I est correta.
c) possibilitou a retomada do crescimento econmico,
estagnada desde a dcada de 1970.
03. (UEPB) Esse texto consta na declarao dos Direitos da
d) impediu que o processo de privatizao atingisse setores
Criana e do Adolescente. A criana deve ser protegida de todas
estratgicos da economia.
as formas: de abandono, do trfico, da prostituio e no deve
e) permitiu que o aumento da populao economicamente
trabalhar, antes da idade adequada. Analise as proposies que
ativa freasse o emprego no setor informal.
contradizem essa declarao e que viraram manchetes nos princi-
pais jornais que circulam no pas. 06. (UNIFOR) A globalizao e a revoluo tecnocientfica tm
I. O Brasil, come, veste e dorme em produtos feitos por mos
promovido grandes transformaes no mundo do trabalho. Den-
de crianas pobres. Elas colhem laranjas nas fazendas de So
tre elas citam-se:
Paulo, que depois viram suco para crianas ricas e, em forma
a) o desaparecimento de trabalhos autnomos e temporrios e
de concentrado, rendem milhes de dlares em exportao
o aumento de empregos no setor primrio.
para o pas.
b) o crescimento de empregos nos setores de baixa qualificao
II. Crianas pobres colhem algodo que usado na fabricao
e o aumento do nmero de horas dirias de trabalho.
de roupas, enquanto seus corpos esto nus, ajudam a
c) o fortalecimento dos sindicatos e a expanso do setor de
fabricar o carvo para produzir o ao que faz o automvel,
atividades industriais.
enquanto a maioria vai a p para trabalhar. difcil no
d) a maior concentrao de desempregados nos pases
encontrar na cadeia produtiva produtos que, por trs, no
subdesenvolvidos e o crescimento do setor tercirio.
tenham a mo de criana.
e) a sensvel diminuio dos setores informais da economia e a
III. Nas principais cidades brasileiras e nas reas de atrao
reduo do desemprego.
turstica normal a presena de crianas pobres na
distribuio de drogas como tambm na prostituio
07. (UNIFOR) Observe o grfico para responder questo.
infantil. Essa realidade ressalta a violncia, o desamparo e
a desesperana dessa gente.
Assinale a resposta correta.
a) As proposies I, II e III esto corretas.
b) Apenas a proposio I est correta.
c) Apenas a proposio II est correta.
d) Apenas a proposio III est correta.
e) Todas as proposies so falsas.
A leitura do grfico e seus conhecimentos sobre a dinmica de-
04. (UNIFOR) Considere o grfico para responder questo. mogrfica brasileira permitem afirmar que o setor tercirio
a) Poderia ter crescido mais, porm as sucessivas crises
econmicas frearam esse setor que o mais dinmico e
qualificado.
b) Ultrapassou os outros setores, e por abranger variadas
atividades, ainda no pode ser equiparado ao tercirio dos
pases desenvolvidos.
c) Apresentou um aumento pouco expressivo por que depende
da existncia de mo-de-obra altamente qualificada.
d) Teve um crescimento lento mas contnuo e reflete a
expanso do capital externo nos setores produtivos da
economia.
A leitura do grfico e seus conhecimentos sobre a realidade eco- e) Cresceu muito rapidamente e, atualmente, um dos mais
nmica mundial permitem afirmar que elevados do mundo, o que destaca o Brasil como pas
a) a concentrao de riquezas nos pases ricos deve, emergente.
gradativamente, se reduzir pela expanso do neoliberalismo
por todo o Globo. 08. (UEPB) A questo de gnero vem ganhando espao nas dis-
b) nos pases ricos possvel encontrar populaes economicamente cusses geogrficas. Analise as proposies e marque a resposta
homogneas graas a concentrao de riquezas. correta.
c) embora o perfil demogrfico dos pases ficos e pobres seja I. A mudana cultural e econmica neste final de sculo
semelhante, nestes ltimos h forte dependncia econmica. incorporou uma massa importante de mulheres estrutura
d) as desigualdades socioeconmicas entre pases ricos e pobres produtiva. S que, a reforma econmica no tem diminudo
so aprofundadas devido forte concentrao de riquezas a tradicional diviso sexual do trabalho, persistindo a
nos pases ricos. discriminao da mulher.

88 UECEVEST

Apostilas UECEVEST mod3.indb 88 06/02/2011 09:58:26


GEOGRAFIA

II. A globalizao da economia vem gerando novos pobres. mOVImEnTOS POPUlACIOnAIS


As mudanas profundas advindas da estrutura produtiva e
do Estado no afetaram as mulheres trabalhadoras.
III. O elevado nmero de mulheres como chefes de famlia
indica uma elevao da misria social, ampliando o
fenmeno da feminizao da pobreza.
IV. A expresso boa aparncia presente nos anncios de oferta
de emprego, na prtica, significa discriminar mulheres
negras, pobres ou consideradas fora de padro.

Assinale a alternativa correta:


a) Apenas as proposies I e II esto corretas.
b) Apenas as proposies I, III e IV esto corretas.
c) Apenas as proposies I e IV esto corretas.
d) Apenas as proposies II e III esto corretas.
e) Apenas as proposies I e III esto corretas.
Aspectos gerais
09. (UFC) Significativa parcela da populao trabalhadora do As migraes so deslocamentos demogrficos de uma regio
mundo globalizado discriminada por diferentes razes: sexo, para outra. So internas, quando ocorrem dentro de um mes-
etnia, idade e renda. Partindo dessa assertiva, analise as afirma- mo pas, e externas, quando se do de um pas para outro. As
es abaixo. migraes so ocasionadas por motivaes econmicas a mais
I. As mulheres tm menor participao no controle dos importante ao longo da histria, poltico, ideolgicas, naturais,
recursos familiares, reduzido poder administrativo e salrios conflitos militares e religiosas.
mais baixos. Emigrao: a sada da populao de um lugar;
II. As comunidades indgenas, afroamericanas, negroafricanas Imigrao: a entrada ou chegada de estrangeiros num lugar.
e latino-americanas tm menor participao no mercado de
trabalho global. As migraes humanas so feitas de regies de repulso popu-
III. Os idosos tm baixa participao no mercado de trabalho, lacional, onde h baixos salrios, desemprego e subemprego, para
principalmente nos pases de Terceiro Mundo. reas de atrao populacional, onde h melhores condies de
vida, condies de emprego e salrios mais elevados.
Da anlise das afirmaes, correto afirmar que:
a) apenas I e II so verdadeiras. migraes externas
b) I, II e III so verdadeiras. No incio do sculo XXI, um em cada 35 humanos, era um
c) apenas II e III so verdadeiras. migrante internacional. Isso significa 175 milhes de pessoas, ou
d) apenas II verdadeira. 2,9% da populao mundial. Alm disso, esses nmeros excluem
e) apenas I verdadeira. a corrente de migrantes ilegais. Todos os cerca de 190 Estados so-
beranos so agora, nas palavras da Organizao Internacional para
10. (UEPB) Observe a tabela e, com base nos indicadores apre- as Migraes (IOM, na sigla em ingls), pases de origem, trnsito
sentados, escolha a alternativa que melhor interpreta os dados. ou destino de migrantes, e cada vez mais as trs coisas simultanea-
Renda per capira US$ 16.400 mente. Os dados so do World Migration 2003: Managing Mi-
gration Challenges and Responses for People on the Move (Ge-
Vida mdia 70 anos nebra: International Organization for Migration, 2003). A maior
Populao urbana 86% fonte de migrantes foi o Mxico, com um fluxo de sada lquido de
6 milhes entre 1970 e 1995. Em seguida est Bangladesh com 4,1
Populao ativa empregada na indtria 26%
milhes, Afeganisto com 4,1 milhes e Filipinas com 2,9 milhes.
Analfabetos 4,7% Estados Unidos o pas que mais recebe imigrantes do mundo.
Entre 1970 e 1995, seu fluxo de entrada lquido foi de 16,7 mi-
a) A alta renda per capita o melhor indicador para definir que lhes. O segundo lugar pertence Rssia com 4,1 milhes, Arbia
a tabela refere-se aos dados de um pas desenvolvido. Saudita com 3,4 milhes, ndia e Canad com 3,3 milhes cada,
b) A baixa taxa de analfabetos e a alta mdia de vida so tpicos Alemanha com 2,7 milhes e Frana com 1,4 milho.
dos pases ex-socialistas do leste europeu que investiram em
sade e educao. Pases de maior emigrao declarada* (1980 1995)
c) A taxa de analfabetos nos pases desenvolvidos de 0%,
portanto, a tabela mostra dados de um pas subdesenvolvido. Pas Nmeros de emigrados
d) As mdias estatsticas, embora sejam um importante Mxico 4.446.000
elemento de anlise, podem mascarar a realidade da Bangladesh 4.048.000
populao de um pas, alm do mas, os indicadores Filipinas 2.962.000
apresentados na tabela so insuficientes para um diagnstico China 2.400.000
criterioso das condies socioeconmicas de um pas. Indonsia 2.400.000
e) A renda per capita s um indicador preciso do nvel de
desenvolvimento, se acompanhado da distribuio da renda, Vietn 1.013.000
porm, no resta dvidas de que a tabela refere-se a um pas Sri Lanka 900.000
urbano e industrial, portanto, desenvolvido. Ex-Iugoslvia 897.000
Zmbia, Tailndia
Colmbia 750.000

UECEVEST 89

Apostilas UECEVEST mod3.indb 89 06/02/2011 09:58:26


G E O G R AFIA

Emigrantes principais destinos * (milhares) Desemprego e subemprego, quando o mercado de trabalho


EUA 798,4 pequeno para a quantidade de mo-de-obra disponvel;
Carncia habitacional;
Alemanha 615,3 Intensificao do processo de favelizao;
Japo 274,8 Aumento nas reas urbanas da populao marginalizada;
Reino Unido 236,9 Aumento da medicncia.
Canad 216
Frana 102,4 migrao Urbano-Urbana
Austrlia 85,8 Deslocamento de uma cidade para outra, nos pases mais po-
bres a continuao do xodo rural.
Holanda 76,7
*Pases selecionados.
Fonte: OCDE, in Almanaque Abril 2000. So Paulo, Abril, 2000. p. 74. nomadismo
O nomadismo o movimento constante praticado pelos po-
Transumncia vos sem residncia fixa. o caso dos ciganos.
A transumncia um movimento peridico e reversivo, cau-
sado por fatores climticos, com a mudana das estaes ou se- migrao Diria ou Pendular
cas temporrias.O pastor nmade das regies montanhosas um Constitui deslocamento dirio de milhes de trabalhadores
transumante. Ele vive com seu rebanho: que moram na periferia e nos subrbios, pela manh, em direo
nas montanhas, durante o vero e o outono; ao centro, retornando aos seus lares no final da jornada de tra-
na plancie, durante o inverno e a primavera. balho. Nos pases subdesenvolvidos esta migrao intensificada
pela precariedade do sistema de transporte.
xodo Rural

E X E R C C I O
01. Desde os primrdios da histria do homem, ocorrem mo-
vimentos migratrios, promovidos por diversas motivaes (re-
ligiosa, poltica, econmica, guerras e conflitos civis, principal-
mente). A partir da dcada de 1980, o fluxo de migrantes no
mundo aumentou consideravelmente.
Leia atentamente os itens abaixo.
I. A xenofobia, a discriminao e a violncia contra os
imigrantes so realidades presentes nas principais reas de
atrao do globo.
II. Os Estados Unidos, os pases europeus e o Japo aparecem
como reas de atrao para os migrantes.
III. Na maioria dos pases ricos, as medidas de restrio
entrada de migrantes se tornam menos severas a cada ano.
IV. Os refugiados representam uma parcela bern superior
dos fluxos migratrios associados chamada migrao
econmica.
V. Os fluxos migratrios internacionais se originam do enorme
abismo que separa os pases pobres dos pases ricos.
VI. Apesar das medidas de restrio imigrao adotadas
nos Estados Unidos, o pas atrai grandes contingentes de
O xodo rural o abandono do campo em busca dos centros imigrantes.
urbanos. Tem sido muito comum nos pases subdesenvolvidos
em processo de industrializao. Cerca de 150 milhes de chine- Assinale a alternativa verdadeira.
ses esto migrando do campo para as cidades. As cidades em fase a) I, II, III so certas e IV, V e VI so erradas.
de crescimento e de industrializao oferecem melhores condi- b) I, II, IV, V e VI so certas e III errada.
es de trabalho e de vida. Nos pases desenvolvidos esta migra- c) I, II, V e VI so certas e III e IV so erradas.
o foi provocada pelo processo de mecanizao das atividades d) III, IV e V so certas e I, II e VI so erradas.
rurais e praticamente j cessou. Nos pases subdesenvolvidos, a
desestruturao das atividades rurais, a concentrao fundiria e a 02.
incipiente mecanizao podem ser apontadas como os fatores que
tm forado milhares de migrantes a abandonarem as reas rurais
e se aventurarem em direo dos centros urbanos.
O xodo rural tem muitas consequncias e todas elas so bas-
tante negativas.
Para o campo, as consequncias do xodo rural so:
Reduo da populao rural;
Diminuio da mo-de-obra rural;
Comprometimento dos cultivos de subsistncia.

As consequncias do xodo rural mais desastrosas ocorrem


nas cidades. So elas:

90 UECEVEST

Apostilas UECEVEST mod3.indb 90 06/02/2011 09:58:27


GEOGRAFIA

Considerando o esquema acima, I, II, III, IV e V correspondem, as migraes rural-urbanas C..). Este processo traz como conse-
respectivamente, a: quncia a urbanizao. C..)
a) bias-frias / pendular / sul da Bahia / zona metropolitana de VESENTINI, J. William. Sociedade e Espao Geografia Gera! e do Brasil. So
Salvador / pr-capitalista. Paulo: tica, 1998.
b) pees / pendular / Serto ou Agreste / Zona da Mata
aucareira / pr-capitalista. Seria uma interpretao CORRETA da proposio anterior, a
c) bias-frias / sazonal/sul da Bahia / zona metropolitana de afirmativa:
Salvador / assalariado temporrio. a) Ambos os fenmenos citados na proposio so decorrentes
d) pees / pendular / reserva extrativista (AC) / Serto / das melhorias de condio de vida nas cidades.
escravo. b) As migraes rural-urbanas tambm so conhecidas como
e) bias-frias /sazonal / Serto ou Agreste / Zona da Mata xodo rural.
aucareira / assalariado temporrio. c) Entende-se por urbanizao a transferncia de pessoas do
campo para as cidades.
03. O movimento pendular da populao que se verifica, dia- d) As migraes rural-urbanas e a urbanizao so fenmenos
riamente, com bastante intensidade, em quase todas as grandes idnticos.
cidades brasileiras, est associado a: e) Na proposio, destaca-se o sculo XIX como elemento
a) movimentos rtmicos sazonais, resultantes da causador dos fenmenos.
homogeneidade do espao urbano.
b) uma modalidade transumnica para aproveitar trabalhadores 07. Migrao peridica dos rebanhos da plancie, os quais vo
temporrios nas reas centrais. habitar durante o calor as altas montanhas, delas descendo ao
c) expanso horizontal urbana e periferizao de mo-de-obra. proximar-se o inverno, ou vice-versa.
d) um intenso nomadismo gerado pela especulao imobiliria
com verticalizao da marcha urbana.
e) movimentos rtmicos sazonais ligados s atividades do setor
tercirio.

04. No processo de migraes inter e intraregionais no Brasil,


podemos identificar alguns perodos de maior relevncia. Entre
esses perodos, podemos destacar o que vai de 1970 a 1990, ca-
racterizado por:
a) incio da marcha para o Centro-Oeste e ocupao do norte
do Paran. Considerando-se a representao de um pas do hemisfrio Norte
b) grande fluxo migratrio para o Sudeste e particularmente e a situao descrita anteriormente, no inverno, assinale a alter-
para So Paulo. nativa correta:
c) migraes em direo Amaznia, com projetos de a) Conceito: transumncia, meses predominantes: maio a
colonizao ao longo de rodovias. agosto, deslocamentos: C para B.
d) intenso movimento migratrio das periferias das reas b) Conceito: aclimao, meses predominantes: novembro a
metropolitanas para o ncleo. maro, deslocamentos: A para B.
e) paralisao do xodo rural em direo s cidades mdias e c) Conceito: aclimatao, meses predominantes: novembro a
regies metropolitanas. maro, deslocamentos: C para B.
d) Conceito: transumncia, meses predominantes: novembro a
05. As migraes internas no Brasil na dcada de 1990 mantiveram maro, deslocamentos: B para A.
algumas tendncias observadas durante a dcada de 1980, ou seja: e) Conceito: aclimatao, meses predominantes: maio aagosto,
I. queda do movimento interno migratrio em direo Legi. deslocamentos: A para B.
o Sudeste e, particularmente, s metrpoles.
II. diminuio do crescimento populacional do municpio de 08. ...e qualquer um que ceder intolerncia deve ter em conta
So Paulo, que nas dcadas anteriores destacou-se como o que sem O TRABALHO DESSES ESTRANGEIROS a riqueza
principal plo de atrao da populao. de nosso pas no seria possvel. O texto retrata a advertncia,
III. permanncia do vetor migratrio em direo Amaznia. provavelmente, de um dirigente
(Adas Melhem. Panorama Geogrfico do Brasil. So Paulo: Moderna, 1998. p. 535.) a) turco ou srio, contra o movimento separatista dos curdos.
b) espanhol, contra o movimento separatista dos bascos.
Dentre os fatores que explicam essas tendncias pode-se citar a c) paquistans ou indiano, contra a onda de perseguio aos
a) crescente concentrao fundiria e o incentivo explorao muulmanos.
madeireira na regio Norte. d) alemo ou francs, contra a onda de xenofobia crescente em
b) desconcentrao industrial e a busca de novas fronteiras seu pas.
agropecurias. e) africano, contra a possibilidade de perseguio aos brancos
c) crise econmica da dcada de 1980 e o agravamento da seca europeus.
no serto nordestino.
d) mecanizao da agricultura e programas governamentais de 09. Percorrendo o sculo XIX e, principalmente, o sculo XX, o
assentamentos rurais. fenmeno das migraes internacionais foi a energia forjadora e
e) decadncia industrial no Estado de So Paulo e abertura de constituidora do novo quadro social dos Estados-nao moder-
novas estradas na Amaznia. nos. Tendo em conta esse fenmeno, leia atentamente as afirma-
es a seguir:
06. Observe a seguinte proposio: 1. As sociedades receptoras de imigrantes so mais ou menos
Aps o sculo XIX, vem ocorrendo um contnuo processo de assimiladoras. H sociedades que se constituem em formas
crescimento do meio urbano custa do meio rural, .isto , grande pluriculturalistas, o que dar origem (pensando-se em
quantidade de pessoas transfere-se do campo para as cidades: so cidades) a bairros de comunidades especficas (guetos),em

UECEVEST 91

Apostilas UECEVEST mod3.indb 91 06/02/2011 09:58:27


G E O G R AFIA

que membros de uma comunidade se casam, em geral, no 11. As migraes internacionais so fluxos de populaes que
interior dela e normalmente s encontram os de fora no atravessam fronteiras polticas, deslocando-se dos pases de ori-
trabalho. Esse o modelo que est mais disseminado nos gem para fixar residncia em outros pases. O golfo da Guin,
pases anglo-saxes (EUA, principalmente). na costa ocidental da frica, palco de um duplo movimento
2. Os movimentos migratrios europeus so vigorosos migratrio. Considere esses fluxos migratrios na figura a seguir.
entre 1880 e 1913. So majoritariamente dirigidos s Migraes no golfo da guin
Amricas, embora haja correntes que vo povoar a Austrlia
e a Nova Zelndia. A assimilao desses europeus nas
sociedades americanas foi facilitada, pois eram sociedades
em constituio nas quais muitos imigrantes europeus se
posicionaram razoavelmente bem e alguns acabaram por
pertencer aos segmentos sociais dominantes.
3. O fluxo migratrio internacional no sculoXX, praticamente
se deu em todas as direes. A sia - em especial China
e Japo - recebeu enormes fluxos de ocidentais atrados
por seu desenvolvimento tecnolgico, tambm, forneceu
grandes contingentes populacionais para a Amrica.
Nesse momento, a caminho do sculo XXI, h uma certa
estabilidade no fluxo migratrio, e a nica regio do planeta
que ainda recebe grandes fluxos a Amrica Latina. Fonte: Adaptado de Magnoli, 1997
Qual a alternativa que contm as afirmaes corretas? Os conjuntos de setas, identificados na figura acima pelos nme-
a) Todas so corretas. ros 1 e 2, representam, respectivamente, o
b) Somente a 2 e a 3 so corretas. a) fluxo sazonal para as reas de plantations e o fluxo das
c) Nenhuma delas correta. plantations para as reas de minerao.
d) Somente a 1 e a 2 so corretas. b) fluxo do Sahel para as plantations e o fluxo das plantations
e) Somente a 1 e a 3 so corretas. para as reas petrolferas.
c) fluxo de povos animistas para os pases catlicos e o fluxo de
10. Observe com ateno o mapa apresentado a seguir: catlicos para ex-colnias francesas.
A imigrao de europeus na Europa d) fluxo de Sahel para as reas petrolferas e o fluxo das reas
petrolferas para as plantations.
e) fluxo sazonal para as reas de minerao e o fluxo de reas de
minerao para as plantations.

G A B A R I T O
Crescimento demogrfico mundial
01. d 02. c 03. d 04. a 05. d
06. d 07. b 08. c 09. b 10. b

As teorias demogrficas
Fonte: Traduzido de SCIENCE PO Cartographic 01. b 02. a 03. c 04. a 05. d
06. c 07. c 08. b 09. b 10. a
correto afirmar que
a) o crescimento da emigrao da regio da ex-Iugoslvia, aps Distribuio da populao mundial
1989, deve-se imensa oferta de postos de trabalho de alta 01. d 02. c 03. d 04. e 05. d 06. d 07 c
qualificao nos principais pases da Unio Europeia.
b) a imensa emigrao da ex-Iugoslvia, aps 1989, devese Populao do Brasil
configurao da Unio Europeia em 1992 que criou a 01. c 02. e 03. c 04. b 05. c 06. a 07 d
figura do cidado europeu, que poder morar e trabalhar em
qualquer pas-membro. Crescimento migratrio do Brasil
c) antes de 1989, Portugal foi um grande fornecedor de 01. c 02. b 03. e 04. a 05. e 06. c
mo-de-obra para a Frana e um dos pases mais pobres da
Europa. No entanto, com sua incluso na Unio Europeia, Distribuio populao do Brasil
essa situao foi alterada.
d) os pases nrdicos, que ingressaram atualmente na Unio 01. d 02. c 03. d 04. d 05. c
Europeia como a Sucia, transformaram-se, aps 1989, em
A estrutura etria
plos de atrao de imigrantes dos pases europeus mais
pobres. 01. a 02. e 03. e 04. c 05. c 06. d
e) as imigraes de europeus em direo aos principais pases
desse continente, tanto antes como aps 1989, esto A estrutura ocupacional
diretamente relacionados queda dos pases socialistas do 01. c 02. c 03. a 04. d 05. b
Leste Europeu. 06. d 07. e 08. c 09. b 10. d

92 UECEVEST

Apostilas UECEVEST mod3.indb 92 06/02/2011 09:58:28


GEOGRAFIA

movimentos populacionais O Nordeste a segunda regio brasileira em porcentagem de


01. c 02. e 03. c 04. c 05. b 06. b mestio (64,3%) superada apenas pelo Norte (68,1%) . A ex-
07. d 08. d 09. d 10. c 11. a pectativa de vida do nordestino a menor do pas (menos 65
anos). Embora concentre 28,1 % da populao nacional, o
Nordeste possui 53% dos analfabetos 50% dos indigentes e
48% das famlias pobres.
A regio Nordeste a mais dividida politicamente, ou seja, a
ESTUDO DAS SUB-REGIES DO nORDESTE mesma apresenta 09 estados. (ver tabela)

Introduo O territrio de Fernando de Noronha, por votao da as-


O Nordeste foi de fato a primeira regio do pas ocupada sembleia nacional constituinte (1988) foi extinto e sua rea foi
efetivamente pelos portugueses. O cultivo da cana, primeira ati- reincorporada ao estado de Pernambuco.
vidade econmica estvel da colonial, garantiu a Portugal a posse
do territrio. Densidade
Populao rea /
A regio Nordeste foi tambm a primeira a possuir uma ati- Estados Capitais em 2000 Km Demogrfica
vidade industrial em nosso pas, pois foi aqui que no sculo XVI hab / Km
instalou se a Agroindstria da cana. Desde ento, ao longo de Alagoas Macei 2.822.621 29.106 96,9
mais de quatro sculos, a regio passou por grandes transforma- Bahia Salvador 13.070.250 566.978 23,0
es socioeconmicas. Cear Fortaleza 7.430.661 145.693 51,0
Maranho So Lus 5.651.475 329.555 17,0
Paraba Joo Pessoa 3.443.825 53.958 63,8
Pernambuco Recife 7.918.344 101.023 78,3
Piau Teresina 2.843.278 251.273 11,3
R. G. Norte Natal 2.776.782 53.166 52,2
Sergipe Aracaj 1.784.475 21.862 81,6
(Fonte: IBGE. Disponive/ em: www.ibge.gov.br. Acesso em 2003).

A Diferenciao do Espao
Como produto do modo de ocupao, influenciado, por sua
vez, pelas variadas condies naturais do territrio, o Nordeste
apresenta quatro sub-regies bem diferentes, so elas: Meio-Nor-
te, Serto, Agreste e Zona da Mata.

Entre as cinco grandes regies brasileiras o Nordeste a que


apresenta os maiores contrastes fsicos de formas diversas, ocasio-
nando grandes diferenas econmicas e sociais, tornando a regio
muito complexa, sendo at denominada de Regio Problema.

Indicadores Socioespaciais
O Nordeste uma regio de 1.552.614 km2. A mesma repre-
senta cerca de 18,2% do territrio nacional, sendo considerada
a 3 regio em extenso territorial, ficando logo atrs do Norte
com 3.851.557 km2 (45,3%) e o Centro Oeste com uma rea
de 1.604.850 km2 (18,9%). 1 . Meio-Norte
Dentro do contexto nacional, a regio Nordeste a 2 regio Sub-Regies Econmicas 2 . Serto
mais populosa, sendo superada apenas pelo Sudeste. Com re- 3 . Agreste
lao densidade demogrfica, o Nordeste apresenta-se como 4 . Zona da Mata
sendo a 3, ficando ultrapassada pelas regies Sudeste e Sul.
As densidades mais baixas so encontradas em alguns trechos do meio-norte
serto, que apresentam menos de 5 habitantes por km2. J as
maiores densidades ficam localizadas na Zona da Mata e Agreste.
Apesar de ter apresentado o maior aumento da taxa de urbani-
zao de todo o pas, no incio dos anos 1990 (7,5%), o Nor-
deste a regio brasileira com maior ndice de populao rural
(31 %). (Vertabela:)
Regio ndice de Urbanizao
Sudeste 90,52%
Centro-Oeste 86,73%
Sul 80,94%
Norte 69,83%
Nordeste 69,04%

UECEVEST 93

Apostilas UECEVEST mod3.indb 93 06/02/2011 09:58:29


G E O G R AFIA

O Meio-Norte abrange o estado, do Maranho e a maior parte Principais destaques do serto


do estado do Piau. , portanto, uma rea de transio entre a regio Predomnio da prtica da pecuria extensiva (bovinos - ca-
Norte extremamente mida, e o serto nordestino, muito seco. prinos - equinos e asininos) destaque para o estado da Bahia.
Essa sub-regio classificada como sendo a menos desenvol- Lembre-se que a ocupao do interior do Nordeste deve-se
vida do Nordeste, merecendo destaque: prtica dessa atividade.
A pecuria extensiva do gado bovino, que representa para a Neste setor nordestino ainda podemos citar a prticado extra-
populao local uma atividade complementar de renda. tivismo: carnaba (rvore da providncia), licuri e a oiticica.
Nessa zona de transio, a paisagem mais caracterstica a Em certos trechos do serto h maior grau de umidade, o que
chamada mata de cocais, onde se destaca a carnaba e o babau, resulta em paisagens menos ridas, so as reas de vrzeas (frios) e
explorada para diversas finalidades, especialmente produo de os brejos. Lugares chamados de brejos ou p de serras permitem
leos e ceras (extrativismo). cultivar diversos produtos, tanto para subsistncia como comer-
No tocante agricultura comercial temos como destaque: cial tais; como: milho, feijo, cana, banana, mandioca etc.
produo de algodo nas reas mais secas, o arroz, plantado nos Na produo agrcola ainda merece destaque o cultivo do algo-
vales fluviais do Maranho, e mais recentemente a cultura da do arbreo, de frutas e a produo de soja no oeste baiano. O
soja, cultivada no sul. do Maranho e no Piau. Cear o maior produtor de algodo arbreo.
Associada modernizao implantada pela cultura de soja, des- A metrpole regional de maior influncia sobre o serto Forta-
tacamos ainda as atividades como a avicultura e a suinocultura. leza; destaque: plo txtil, indstria alimentar, bebidas, couro etc.
So Lus, capital do Maranho, a principal cidade do Meio-
Norte, e teve sua economia incrementada pelas atividades portu- O Agreste
ria e de servios implantados a partir da construo do Porto de
Itaqui e da estrada de ferro Carajs - Itaqui.
A capital do Maranho tambm foi favorecida com o Projeto
Alumar, pelo qual vrias empresas se associaram para aproveitar a
bauxita do Par e dela produzir alumnio.

Serto

Por situar-se entre duas outras sub-regies uma seca e outra mi-
da, o Agreste uma zona de transio entre o serto e zona da mata.
Grande parte do agreste constituda pelo Planalto da Borbo-
rema. Esse planalto responsvel, em parte, pela secura do serto,
pois funciona como uma barreira que impede a penetrao para o
interior dos ventos midos que vm do atlntico e que se transfor-
O serto a sub-regio que ocupa a maior parte da regio Nor-
mam em chuvas na Zona da Mata e na parte oriental do Agreste.
deste, com uma rea de cerca de 75% do espao total e que abrange
terras de todos os estados, com exceo do. Maranho. Estende-se
Suas principais caractersticas
do litoral setentrional do Nordeste, em terras do Cear e do Rio
Enquanto na zona da mata prevalecem os latifndios mono-
Grande do Norte, at a fronteira com Minas Gerais, rea sob o
cultores, no agreste temos o predomnio de minifndios poli-
domnio do clima semirido e de vegetao de caantiga, se localiza
cultores (comerciais e subsistncia).
especialmente no trecho conhecido como Polgono das Secas.
Destaca-se tambm a prtica da pecuria intensiva (gado de leite)
Polgono das secas Produtos cultivados em destaque: (feijo, milho, batata, horta-
lias etc), alm do algodo e do sisal ou gave.
Principais cidades comerciais: Campina Grande (PB); Feira
de Santana (BA), Vitria da Conquista (BA), Caruaru e Ga-
ranhuns (PE).

A Zona da Mata

Delimitada pelo governo federal, a rea do polgono das


secas abrange os municpios que sofrem os efeitos das secas.
Observe que esta rea abrange tambm reas do Agreste e do
MeioNorte, alm do Norte de Minas ultrapassando os limites
do serto e da prpria regio Nordeste.
(Fonte: IBGE; geografia do Brasil, regio nordeste, vol. 2.)

94 UECEVEST

Apostilas UECEVEST mod3.indb 94 06/02/2011 09:58:30


GEOGRAFIA

Essa faixa de terra foi assim denominada por ter sido, no A transposio suscita questo quanto sua viabilidade: pre-
passado, ocupada por uma floresta tropical, chamada de Mata ciso conhecer a real vazo do rio e saber se seu volume de gua
Atlntica. A mesma ocupa uma extensa faixa de terra prxima ao ser capaz de suprir o potencial necessrio para o abastecimento,
litoral, estende-se desde a poro mais oriental do Rio Grande do a irrigao, a navegao, a produo de energia e, ainda, para a
Norte at a extremidade Sul da Bahia. transposio. Afinal, o desmatamento da rea de suas nascentes,
a eroso das margens e o assoreamento do leito vm contribuin-
Caracterizando a sub-regio do, h tempos, para a reduo do potencial hdrico,do rio So
A Zona da Mata apresenta no seu quadro natural um domnio Francisco.
de clima tropical mido, e a presena de um solo frtil conhe- Alm disso, o projeto dever envolver soma considervel de
cido como Massap. recursos financeiros e grande detalhamento sobre os impactos
Essa faixa de terra foi ocupada desde o incio da colonizao ambientais causados durante e aps a implantao. Para reflexo:
portuguesa no Brasil e se destacou com a extrao do pau-brasil apesar do alto custo, a transposio das guas do So Francisco
e posteriormente com a lavoura de cana-de-acar. pode amenizar as consequncias das secas e ajudar muitas pesso-
A zona da mata a parte do Nordeste mais industrializada, as; mas tambm pode prejudicar o ambiente, alm de duvidosos
mais desenvolvida, mais povoada e urbanizada. melhorias socioeconmica para, a populao mais carente, pense
Essa sub-regio se caracteriza por apresentar uma estrutura sobre isso...
agrria concentrada, onde se verifica a predominncia das gran- (Adaptao: A Transposio das guas do So Francisco. Pg. 93 Projeto Arariba.
des propriedades voltada na maioria das vezes para exportao. Geografia Modema).
Produtos: cana-de-acar destaque para Pernambuco e Alago-
as; cacau no litoral do Sul da Bahia, fumo no Recncavo Baia-
no; o arroz no Baixo Vale do So Francisco, o coco-da-bahia E X E R C C I O
na Bahia etc.
A maior concentrao industrial da regio Nordeste est situada 01. (UFV-MG) o quadro a seguir apresenta as quatro sub-regies
no centro industrial de Aratu (BA). Outros destaques so: refi- do Nordeste brasileiro, com algumas de suas caractersticas.
naria de petrleo Landulfo Alves (Recncavo Baiano) e o plo Sub-Regies Caractersticas
petroqumico de Camaari (regio metropolitana de Salvador).
A atividade pesqueira estar presente em toda a extenso do A policultura praticada em pequenas proprie-
litoral nordestino. Os pescadores, alm de pescarem para o dades a principal atividade econmica dessa
consumo prprio, comercializam, sendo que os de maior valor sub-regio. As reas mais midas e aproveitadas
comercial so a lagosta e o camaro. I para a agricultura so reconhecidas com brejos.
Abriga algumas das cidades mais importantes do
A transposio das guas do So Francisco Nordeste, como Feira de Santana, Caruaru, e
Campina Grande.
Compreende o Maranho e quase todo o Piau.
Sua principal atividade econmica o extrativismo
vegetal, destacando-se a carnaba e o babau, que
II empregam grande quantidade de mo-de-obra
em sua coleta. Seus produtos so empregados no
artesanato local e como matria-prima para as in-
dstrias.
Estreita faixa de terra que se estende do litoral do
Rio Grande do Norte at o sul da Bahia. Apre-
senta clima tropical mido. Possui belas praias
III
e dunas. Tem grande destaque na produo de
cana-de-acar, fumo e cacau e na explorao
mineral de petrleo e sal marinho.
Corresponde a uma vasta subregio castigada pela
aridez de seu clima. Submetida a secas frequentes,
sua vegetao constituda por rvores e arbus-
IV
tos recobertos de espinhos. Desde o incio de sua
ocupao, a pecuria a atividade econmica
mais importante.
Fonte: Adaptatadl de I. IBGE, Brasil em nmeros 1992. 2. IBGE, Atlas Nacional
do Brasil, 1992.
Identifique a opo que nomeia de forma correta as regies I, II,
III e IV, respectivamente:
Sabemos que grande parte dos rios do Nordeste so intermi- a) Agreste, Zona da Mata, Meio-Norte e Serto.
tentes, principalmente aqueles localizados no serto. Por causa b) Zona da Mata, Agreste, Serto e Meio-Norte.
disso, o abastecimento hdrico da regio prejudicado e causa c) Agreste, Meio-Norte, Zona da Mata e Serto.
vrios problemas sociais e econmicos aos habitantes. d) Meio-Norte, Serto, Agreste e Zona da Mata.
Para amenizar os efeitos da secas e normalizar os nveis das e) Serto, Agreste, Zona da Mata e Meio-Norte.
guas da regio, o governo brasileiro props a transposio das
guas do So Francisco. Isso quer dizer que parte das guas do rio
02. (UFC-ADAPTADA) Esto delimitadas, no mapa a seguir,
seria levada para abastecer os rios secos das bacias hidrogrficas.
quatro reas do Nordeste Brasileiro. Tradicionalmente, essas re-
O projeto gerou polmica entre os estados que recebero a
as assim se destacaram. Observe o mapa abaixo e a seguir analise
gua e aqueles que o disponibilizaro, e entre polticos, tcnicos
as afirmaes propostas abaixo.
e ambientalistas.

UECEVEST 95

Apostilas UECEVEST mod3.indb 95 06/02/2011 09:58:31


G E O G R AFIA

I. A rea A foi um espao exclusivamente utilizado pela


pecuria extensiva, nas dcadas de 1950 e 1960. ( ) As capitais nordestinas so litorneas, exceto a cidade de
II. A rea, situada numa faixa quente e mida, corresponde a Teresina.
um espao de grandes lavouras comerciais. ( ) As trs principais capitais, dentre as capitais estaduais, so as
III. A rea que corresponde a uma regio agroextrativa. metrpoles: Fortaleza (3), Recife (6) e Salvador (9).
IV. A rea D, que abrange trechos do Agreste, apresenta amplos ( ) As capitais tm uma posio litornea por ser sua origem
espaos utilizados para a criao de gado e para a policultura. ligada a uma atividade exportadora (atividade aucareiro).
V. A rea A, por se assemelhar climatologicamente rea C, ( ) O territrio de Fernando de Noronha foi criado na Segunda
sempre foi empregada para o plantio de arroz. Guerra Mundial (1942), desmembrando do Estado de
Pernambuco, mas a Constituio de 1988 devolveu o
Aps anlise da mesmas podemos concluir que so verdadeiras arquiplago ao Estado.
apenas as afirmaes: ( )As capitais dos estados principais produtores de sal, babau e
a) I, II, III d) III, IV, V petrleo do Brasil so, respectivamente, as cidades de Natal
b) II, III, IV, V e) I, lI, V (4); So Lus e Aracaju (8).
c) II, III, IV
As opo correta :
03. (UFC-ADAPTADA) Considerando as relaes entre as con- a) FVVVV d) VVVVF
dies ambientais e as atividades econmicas do Nordeste, colo- b) VFVVF e) FVFVV
que (V) ou (F) nas alternativas abaixo: c) FFFFV
I. Na poro oriental da Zona da Mata h primazia de climas
tropicais midos com solos frteis onde se desenvolve a 06. Observe o mapa Nordeste Econmico e associe os concei-
monocultura canavieira. tos abaixo com as reas geoeconmicas numeradas:
II. A maior parte do Nordeste, inserido no polgono das
secas, apresenta clima semirido, rios perenes, vegetao de
cerrado e atividades agrcolas muito diversificadas.
III. O serto deprimido, submetido s condies de semiaridez,
rea de domnio da pecuria extensiva.
IV. As principais reas de extrativismo vegetal correspondem aos
nveis elevados midos onde predomina a caatinga arbustiva.
V. Os brejos constituem paisagem de exceo no contexto
espacial nordestino e a atividade agrcola marcada pela
policultura.
VI. O agreste representa uma faixa de transio entre a Zona da ( ) Zona da Mata, a rea mata tradicional na cultura da cana-
Mata e o Serto e a economia rural est ligada policultura de-acar, no sistema Plantation
de subsistncia. ( ) Serto, rea de clima semirido e coincide com o chamado
Polgono das Secas, onde a atividade econmica ,
A opo com os itens corretos : predominantemente a pecuria extensiva.
a) II, II, V e IV d) III, IV e V ( ) Meio-Norte Maranho, zona dos cocais de babau,
b) I, III, V e VI e) Todos principal produto de extrativismo vegetal brasileiro,
c) I, III, IV e V infelizmente explorado com mtodos rudimentares, gerando
baixos rendimentos.
04. (UFC-2002) Com relao s caractersticas geogrficas das ( ) Sul da Bahia, rea de plantation de cacau, com destaque
sub-regies do Nordeste brasileiro, assinale a opo correta. para as cidades: Itabuna e Ilhus, estando nesta ltima o
a) O Meio Norte, caracterizado por clima mido, foi ocupado porto exportador: Malhados.
em funo de uma pecuria intensiva. ( ) Litoral semirido, rea de explorao de sal, com destaque
b) O Serto, marcado pela semiaridez, foi inicialmente para as cidades: Macau e Mossor.
ocupado graas agricultura de subsistncia. ( ) Vale dos rios, matas ciliares de carnaubeira, a rvore da vida.
c) A Zona da Mata foi a zona de ocupao inicial do Nordeste ( ) Serto, importante rea de cultura de algodo de boa
brasileiro. qualidade, destinado exportao.
d) O Agreste corresponde zona de transio entre o Serto e o ( ) Recncavo Baiano, importante rea de explorao e
Meio Norte. industrializao do petrleo. localiza-se o plo petroqumico
e) O Cerrado tem como atividade predominante o de Camaari.
extrativismo vegetal de babau e carnaba.
07. Entre os sculos XVI e XVIII, praticava-se no Agreste a criao
05. (UFC-ADAPTADA) Assinale as alternativas verdadeiros exa- de gado. Por que essa atividade se desenvolveu no Agreste? Atu-
minando o mapa do Nordeste, no qual as capitais esto indicadas almente, qual a atividade econmica predominante nessa regio?
por nmeros.

96 UECEVEST

Apostilas UECEVEST mod3.indb 96 06/02/2011 09:58:32


GEOGRAFIA

08. (FUVEST-SP) Mencione o produto dominante em cada IV. A probreza da populao residente faz com que a maior
uma das reas do Nordeste assinaladas no mapa. parte das atividades de comrcio e servios se concentre nas
capitais dos estados, no existindo centros urbanos regionais
importantes no serto nordestino.
So corretas somente:
a) II e IV d) I e IV
b) I e II e) II e III
c) I e III

13. A heterogeneidade econmica intra-regional do Nordeste


09. (UNICAMP) brasileiro histrica. Assinale a alternativa FALSA por no apre-
Os audes nordestinos esto geralmente instalados nos latifn- sentar um subespao dinmico.
dios. a) O complexo petroqumico de Camaari.
Os audes servem aos latifndios apenas para que seu gado b) O plo txtil e de confeces de Fortaleza.
beba a gua. c) Plo de moderna agricultura de produo de frutas no Baixo
A gua dos audes serve principalmente para irrigar a cultura Jaguaribe (CE).
de alimentos, praticada pelos colonos dos latifundirios e pelos d) As zonas cacaureias e canavieiras.
pequenos lavradores.
Essas culturas no interessam aos latifundirios, que geralmen- 14. (UECE) Sobre a produo do espao nordestino correto
te produzem culturas industriais como algodo e mamona, no afirmar que o(a):
necessitando, portanto, de terras irrigadas. a) binmio gado-algodo representa as atividades econmicas
Na poca das secas, o latifundirio vende a gua. responsveis pela ocupao humana do serto nordestino.
b) latifndio encontrado no Agreste foi de fundamental
Levando em conta as informaes acima, responda: importncia para a fixao do homem.
Qual a relao entre a construo dos audes e a indstria da seca? c) Zona da Mata teve sua ocupao associada policultura
comercial que necessitava de muita mo-de-obra na rea.
10. (FUVEST) d) expanso da pecuria na faixa litornea e a chegada do
escravo africano foram fatores destacados para a fixao do
homem nessa sub-regio.

15. (UECE) No Brasil, como um todo, e no Nordeste, em espe-


cial, nas ltimas dcadas, elevou-se o processo de deslocamento
da populao rural para as grandes cidades. No conjunto das for-
as poltico-econmicas, este fato est relacionado:
a) expanso das grandes propriedades agrcolas e
mecanizao da lavoura.
Extrado de: 13 de Maio Ncleo de Educao Popular b) aos projetos de irrigao instalados pelo governo, auxiliando
no desenvolvimento agrrio.
gua... e terra... explique por que, na problemtica regional c) ao maior equilbrio econmico entre a cidade e o campo.
nordestina, estes dois elementos possuem grande importncia. d) ao estatuto do trabalhador rural que facilitou a permanncia
do pequeno agricultor, registrando seus benefcios em carteira.
11. (FMU/FIAM-SP) Assinale a alternativa incorreta.
a) A cana-de-acar, cultivada na Zona da Mata do Nordeste, 16. (UECE) A insero da Regio Nordeste no processo de glo-
concentra-se principalmente nos vales fluviais, originando a balizao da economia reflete-se num dinamismo econmico.
expresso regional: rios-de-acar. Poderamos afirmar que:
b) O Recncavo Baiano rea produtora de tabaco. I. O baixo curso do rio Jaguaribe est entre as reas mais
c) A refinaria de Mataripe (Landulfo Alves) localiza se no recentemente incorporadas realizao de uma agricultura
Recncavo Baiano, bacia sedimentar do Cretceo, onde intensiva.
existem reservas petrolferas. II. A modernizao da economia e do territrio tende a acabar
d) O babau produto explorado principalmente na Mata dos com as diferenciaes socioeconmicas intra-regionais.
Cocais, formao vegetal de transio do Meio-Norte ou III. A agropecuria passou por um processo de modernizao
Nordeste ocidental. que afetou todo o campo.
e) A agroindstria aucareiro do Nordeste tem rentabilidade IV. Dentre os plos industriais mais dinmicos destacam-se
mais elevada que a do Brasil Sudeste. o complexo petroqumico na Bahia, o plo txtil e de
confeces no Cear e o complexo minero-metalrgico no
12. (UNIFOR/2002.1) Considere as afirmaes abaixo, sobre as Maranho.
caractersticas naturais e humanas do serto semirido nordestino:
I. A associao da unidade com temperatura mais amenas levou o Assinale a alternativa verdadeira.
sertanejo a designar do inverno o perodo das chuvas de vero; a) I e II so corretas e III e IV so erradas.
II. Os brejos constituem rea de exceo no serto semirido, b) II e IV so corretas e I e III so erradas.
apresentando condies de umidade e de solos capazes de c) II e III so corretas e I e IV so erradas.
suportar a atividade agricola sem a necessidade de irrigao; d) I e IV so corretas e II e III so erradas.
III. A estrutura agrria do serto semirido relativamente
equilibrada, com o predomnio de pequenas e mdias 17. (UNIFOR) Considere o texto abaixo.
proprieades, baseadas no trabalho familiar e voltado para a A e o so produtos predominantemente nordestinos, contri-
policultura; buindo a regio com mais de 80% da produo nacional, que

UECEVEST 97

Apostilas UECEVEST mod3.indb 97 06/02/2011 09:58:32


G E O G R AFIA

se destina sobretudo exportao. So culturas adaptadas ao Agricultura


semirido. b) Pecuria Primitiva comercial Pecuria melhorada
Para complet-Io corretamente as lacunas devem ser preenchidas, tradicional
respectivamente, por: Agricultura
a) cana-de-acar fumo c) mamona - sisal Pecuria
c) comercial Extrativismo vegetal
b) mandioca - feijo d) soja - algodo primitiva
modernizada
Agricultura Agricultura Pequena agricultura
18. (UECE) Com relao rede urbana nordestina, assinale a d) comercial comercial comercial e de
alternativa verdadeira: tradicional modernizada subsistncia
a) densa na faixa litornea e mais ainda medida que avana Pequena agricultura Extrativismo Agricultura comercial
para o interior. e) comercial e de
b) So trs as metrpoles e duas se localizam regio costeira. vegetal modernizada
subsitncia
c) E desarticulada e fragmentada.
d) A macrocefalia no uma das principais caractersticas da 21. (UNICAMP-2002) Leia o trecho a seguir e responda:
rede urbana cearense. A transposio do rio So Francisco discutida desde o tempo
do Imprio. Um dos registros mais antigos da ideia remonta a
19. (UNIFOR) Considere o esquema com caractersticas natu- 1847, quando o intendente do Crato (CE), deputado Marcos
rais de uma sub-regio do Nordeste brasileiro. Antonio de Macedo, props o mesmo que se debate hoje: lanar
as guas do Velho Chico no rio Jaguaribe. Na obra Contrastes e
Extensa Clima de Vegetao de Confrontos, Euclides da Cunha ressuscitou a ideia do intendente
Nmerosos
plancie, transio floresta. Mata cearense e a incluiu entre as grandes intervenes civilizadoras de
e volumosos
planaltos e entre mido dos Cocais e que carecia a regio, como audes, barragens, arborizao, estra-
rios
chapadas e semirido caatinga das de ferro e poos artesianos.
(Adaptado de Marcelo Leite, Folha de S. Paulo. 09/10/2005.)

a) Por que o rio So Francisco chamado de o rio da unidade


nacional?
b) Aponte e explique um argumento contra e um a favor da
A lacuna deve ser preenchida por: transposio do rio So Francisco.
a) Meio Norte d) Agreste c) A precipitao pluviomtrica anual mdia no semirido
b) Recncavo Baiano e) Sul da Bahia nordestino de cerca de 700 milmetros/anos, superior a
c) Zona da Mata algumas regies agrcolas da Europa. Quais so os principais
problemas de ordem natural que expem grande parte do
20. (UNIFOR) Observe o mapa do uso da terras no Nordeste territrio, em especial o chamado Polgono das Secas, a uma
para responder questo. situao de vulnerabilidade?

22. (UECE/2000) No Nordeste oriental as paisagens naturais


exibem mudanas bruscas do litoral para o interior, ou seja, de
leste para oeste. Tal fato decorre, basicamente, de influncias de-
terminadas por:
a) disposio da rede hidrogrfica.
b) distribuio das rochas cristalinas e sedimentares.
c) disposio e compartimentao do relevo.
d) distribuio dos solos e da cobertura vegetal.

23. (UNIFORl98) No Nordeste, esta vegetao ocupa o maior


parte dos estados do Maranho, Piau e Bahia e est relaciona-
da com a predominncia de um clima tropical com uma estao
chuvosa e outra seca, fato que propicia o aparecimento de vege-
tais de razes profundas e folhas pequenas recobertas de cera.
O texto refere-se vegetao de:
a) cerrado. d) campos sujos.
b) cocais. e) florestas residuais.
c) caatinga.

24. (UFC/97) No serto nordestino, de clima semirido, fre-


quente a ocorrncia de perodos de secas e de enchentes. Com
relao a estes fenmenos, diz-se que:
Adaptado de Graa Maria L. Ferreira. Geografia em mapas. So Paulo: Moderna,
I. a distribuio pluviomtrica irregular ao longo dos meses
2000. p.69 um dos principais fatores climticos para a ocorrncia de
secas e enchentes;
Assinale a alternativa que identifica corretamente nas legendas II. as grandes variaes de temperaturas mensais e anuais no
do mapa. semirido nordestino tambm contribuem para a ocorrncia
1 2 3 de secas; .
Pequena agricultura III. devido a um planejamento regional adequado, as cidades
Pecuria de mdio porte no serto nordestino no so afetadas por
a) Pecuria melhorada comercial e de
primitiva problemas decorrentes das enchentes, mas sim pelos das secas.
subsistncia

98 UECEVEST

Apostilas UECEVEST mod3.indb 98 06/02/2011 09:58:33


GEOGRAFIA

De acordo com o exposto anteriormente, marque a opo correta: a) Os solos aluviais so medianamente profundos, mal drenados,
a) as afirmativas I, II e III so corretas. tm textura areno-argilosa e fertilidade natural de alta a mdia.
b) apenas a afirmao I correta. b) Os solos litlicos so profundos, mal drenados, tm textura
c) apenas a afirmao III correta. argilosa e fertilidade natural alta.
d) as afirmaes I e III so corretas. c) Os latossolos so profundos, bem drenados, tm textura
e) as afirmaes I e II so corretas. areno-argilosa e fertilidade natural de baixa a mdia.
d) Os vertissolos so medianamente profundos, tm textura
25. (ADAPTADA/UFC) Tratando-se do ambiente nordestino e argilosa, fendilham na estiagem e tm fertilidade natural de
das condies do uso e ocupao do solo, verdadeiro afirmar que: mdia a alta.
I. a escassez de gua subterrnea s se verifica nas reas de
chapadas sedimentares, onde se pratica uma agricultura 30. (ADAPTADA UFC) Quanto ao quadro climtico de semia-
adequada ao perfil do relevo; ridez do Nordeste brasileiro, correto afirmar que:
II. as serras midas cristalinas tm relevos acidentados I. a seca, enquanto fenmeno climtico, pode abranger tanto
que configuram problemas para a ocupao agrcola e as reas estritamente semiridas como os ambientes midos
degradao dos solos; e submidos regionais;
III. as depresses sertanejas se desenvolvem, de modo II. a rea do semirido coincide, geograficamente, com todo o
preponderante, em terrenos do embasamento cristalino, territrio nordestino;
com destaque para a cobertura vegetal xerfila e para a III. sendo um fenmeno dependente da circulao atmosfrica, a
presena de aquferos. seca tem, contudo, profundas repercusses de natureza social;
IV. o planejamento dos recursos hdricos visando ao
correto afirmar que: abastecimento urbano independe do regime pluviomtrico;
a) somente a afirmativa I correta. V. dos rios nordestinos, apenas o So Francisco no apresenta
b) somente as afirmativas II e III so corretas. regime intermitente, ou seja, este rio nunca seca sendo uma
c) somente a afirmativa III correta. ddiva para as populaes do serto seco.
d) somente as afirmativas I e III so corretas.
e) somente a afirmativa 11 correta. correto afirmar que:
a) somente I e III so corretas.
26. (ADAPTADA UFC) Considerando as condies ambientais b) somente III e IV so corretas.
e a organizao socioespacial da regio Nordeste, correto afir- c) todas so corretas.
mar que: d) apenas I e II so corretas.
a) os processos de degradao ambiental se manifestam mais e) apenas III, IV e V so corretas.
intensamente nas chapadas devido predominncia da
31. (UECE-2006) Em relao ao Nordeste brasileiro, pode-se
eroso elica;
afirmar verdadeiramente que:
b) as condies climticas e o relevo aplainado das depresses
a) O fenmeno da desertificao afeta, indistintamente, toda a
sertanejas favorecem a existncia de uma agricultura
regio.
intensiva;
b) Historicamente essa regio vem sendo utilizada desde o
c) na Zona da Mata, rea tradicional de produo de cana-
sculo XVIII pela agricultura comercial e pela pecuria
de-acar, dominam as LEIS DE PRESERVAO
intensiva, intensificando os efeitos da degradao ambiental.
AMBIENTAL e o clima semirido;
c) Sob o ponto de vista geoambiental e ecolgico, o Nordeste
d) as principais sub-regies: Zona da Mata, Agreste e Serto
a mais diversificada dentre as Grandes Regies brasileiras.
se diferenciam apenas nos aspectos do meio natural;
d) Nas serras midas e no Agreste com melhores
e) as melhores condies de clima e solo encontram-se na
potencialidades de recursos naturais, a estrutura fundiria
faixa litornea, regio ocupada desde a colonizao e que
tem predominncia de latifndios.
apresenta grande degradao geoecolgica.
32. (UECE-2006) Os enclaves midos das serras cristalinas do
27. Quanto s caractersticas ambientais de cada sub-regio nor- Nordeste brasileiro tm como caractersticas ambientais relevan-
destina, marque o que for correto: tes, as seguintes:
a) Zona da Mata - clima tropical tpico, seco e mido. a) Solos com baixa fertilidade natural e relevos planos.
b) Agreste - clima supermido devido aos elevados ndices de b) Vegetao de cerrado recobrindo solos rasos e afloramentos
chuvas orogrficas. rochosos.
c) Serto - depresso sertaneja com relevos residuais c) Condies hidroclimticas e edficas mais favorveis s
desgastados pela pediplanao. atividades agrcolas.
d) Meio-Norte - faixa de transio que se encontra entre a zona d) Abundncia de guas subterrneas e deficincia de recursos
da mata e o serto. hdricos superficiais.
e) Litoral - rea de prtica da monocultura da cana-de-acar e
do cumprimento das leis ambientais no desenvolvimento de 33. (UECE-2006) Tratando-se do Nordeste brasileiro e das relaes
atividades econmicas. entre a sociedade e a natureza, considere as seguintes afirmaes:
I. O bioma das caatingas no compem a matriz energtica
28. Quanto s unidades de relevo do Nordeste, o possvel afir- regional.
mar verdadeiramente que: II. As maiores potencialidades de utilizao das guas
a) predominam as grandes altitudes. subterrneas ocorrem nas bacias e coberturas sedimentares.
b) as depresses ocupam o espao dos sertes. III. Indistintamente, toda a regio submetida s influncias
c) apresenta apenas unidades cristalinas de antigas eras. do clima semirido, limitando a capacidade produtiva dos
d) apresenta apenas unidades sedimentares planlticas. recursos naturais.
IV. A fragilidade ambiental das plancies fluviomarinhas
29. (UECE/2002.1) Em relao s caractersticas dos solos do revestidas por manguezais, justifica o baixo potencial de uso
Nordeste brasileiro, marque a opo FALSA. da ecossistema aliada a uma biodiversidade muito pobre.

UECEVEST 99

Apostilas UECEVEST mod3.indb 99 06/02/2011 09:58:33


G E O G R AFIA

So corretas: Com uma rea territorial de 148.826km2 o estado do Cear


a) apenas I e III. c) apenas II. juntamente com outros oito (8) estados MA- PI - RN - PB - PE
b) I, lI, III e IV. d) apenas II e IV. - AL - SE - BA) formam a chamada regio Nordeste do Brasil.
O Cear est inserido predominantemente no semirido nordes-
tino, tendo como posio geogrfica as seguintes, coordenadas
2 46 e 7 52 de latitude meridional (Sul) e 3714 e 41 24 de
G A B A R I T O longitude ocidental (Oeste).
01. c 02. c 03. b 04. c 05. d 06. *
limites do Cear
07. 08. 09. 10. 11. e 12. b
Norte - Oceano Atlntico
13. d 14. a 15. a 16. d 17. c 18. c Sul - Pernambuco
19. c 20. d 21. 22. c 23. b 24. e Leste - Rio Grande do Norte e Paraba.
25. b 26. e 27. c 28. b 29. b 30. a Oeste ~ Estado do Piau
31. c 32. c 33. d * 7, 4, 2, 10, 6, 3, 5, 9
Fortaleza a capital do estado do Cear, a mesma tem uma
extenso territorial de 336km e est localizada prxima ao lito-
ral, de onde se desenvolve uma ampla rede rodoviria, interligan-
O ESPAO CEAREnSE do toda a regio metropolitana, municpios do estado e demais
regies do Brasil.
Aspectos espaciais
Eu sou de uma terra em que o povo padece mas no esmo-
rece e procura vencer da terra querida, que a linda cabocla d riso
na boca zomba no sofrer no nego meu sangue, no nego meu
nome olho para a fome, pergunto: o que h? Eu sou brasileiro,
filho do Nordeste sou cabra da peste, sou do Cear
Patativo do Assar . (Cabra da Peste).

Introduo

limites e dimenses do Cear


rea Geogrfica: Brasil, Nordeste e Cear
Relao (%)
Descriminao Total (km)
CE/NE CE/BR
Brasil 8.547.403
Nordeste 1.561.177
Cear 148.826 9,4 1,7

Fonte: Atlas do Cear/2000. Jose B. da Silva e Trcio Cavalcante

Aspectos demogrficos do Cear


Pelos dados do IBGE 2000, o estado do Cear conta com
uma populao total de 7.430.661 habitantes, isso corresponde a
4,3% da populao geral do Brasil e 15,6% da populao nordes-
tina. O mesmo considerado um estado populoso, porm com
grandes vazios demogrficos.
Populao Densidade
Unidades rea/km
total(hab) demogrfica
Brasil 8.547.403 169.799.170 19,9 hab/km
Nordeste 1.561.177 47.357.709 30,5 hab/km
CEar 148.826 7.430.661 51 hab/km
Fortaleza 336 2.141.402 6.833 hab/km
Fonte: IBGE 2000. Adaptao f.f 2005.

O Cear o terceiro estado mais populoso do Nordeste, sen-


do superado pelos estados da Bahia e Pernambuco.
Fortaleza a quinta maior cidade do Brasil em populao e a
segunda do Nordeste, sendo superada apenas por Salvador (BA).
A regio Nordeste, como vimos, a segunda mais populosa
do pas, com uma populao total de 47.357.709 habitantes, se-
gundo o recenseamento do IBGE em 2000. Entretanto, sua po-
pulao relativa (densidade demogrfica) no se destaca na mes-
ma proporo, sua densidade considerada mdia para o Brasil,

100 UECEVEST

Apostilas UECEVEST mod3.indb 100 06/02/2011 09:58:35


GEOGRAFIA

ou seja, a mesma fica em torno de 30 hab/km2. (menor que as Pacajs 44.070 241 0,678 1999
densidades das regies Sudeste e Sul). Pacatuba 51.696 137 0,710 1974
A populao nordestina est distribuda de modo irregular,
h reas de grande concentrao e outras praticamente desrticas. So Gonalo 35.608 842 0,639 1999
As mais elevadas densidades, acima de 50 hab/km, so encontra- SE LIGA
das na Zona da Mata. As mais baixas densidades menos de 5 hab/ Segundo o IBGE (1987), a hierarquia urbana do espao cea-
km2, so encontradas no serto. rense assim constituda:
A distribuio populacional do Cear segue as mesmas ca- Metrpole Nacional e Regional Fortaleza.
ractersticas da distribuio populacional nordestina, ou seja, a Capital Regional Crato, Juazeiro do Norte, Sobral e Iguatu.
mesma tambm se encontra m distribuda pelo territrio, em Centro Sub-Regional Crates.
decorrncia de vrios aspectos socioeconmicos que foram se de- Centro de zona Cidades menores (influncia em sua zona).
finindo ao longo dos anos. Centros Locais Influncia municipal.
De uma maneira geral, podemos afirmar que as grandes con-
centraes demogrficas se encontram em Fortaleza e sua rea Aspectos econmicos
metropolitana, destaque para Maracana e Eusbio. No interior O estado do Cear atualmente apresenta uma taxa de urbani-
do estado destaque para Juazeiro do Norte, Sobral e Crato. zao de 71,5%, ou seja, a maior parte de sua populao ocupa
As menores concentraes populacionais do nosso estado as cidades.
(at 15 hab/km) compreende os municpios do Serto Central
(Inhamuns) e no mdio Jaguaribe. Participao da populao economicamente
ativa por setores em % (P.E.a)
municpios mais povoados 2000 Agropecuria 39,7%.
Densidade Servios 45,9%
Municpios Pop. Total rea/Km Indstria 14,3%
Demogrfica
Fortaleza 2.141.402 313,1 6.838,5 (Fonte: Implance e IBGE - 1999)
Maracana 179.732 105,7 1.700,5
Agropecuria cearense
Juazaeiro do Norte 212.133 248,5 853,4 O estado do Cear ocupa uma posio perifrica, consideran-
do a diviso do trabalho agropecuarista brasileiro, seja em termos
Euzbio 31.500 76,6 411,3 de nveis de difuso de inovaes, seja na participao do P.I.B
Pacatuba 51 .696 132,4 390,4 agrcola. Porm, os produtos de origem agropecurio, os pesca-
Horizonte 33.790 159,9 211,2 dos, alm do extrativismo tm peso determinante na economia
Caucaia 250.479 1.227,9 204,0 estadual.
(Fonte: IBGE - 2000) Considerando a agropecuria cearense segundo sua distribui-
o espacial, poderamos distinguir quatro grandes sub-unidades:
municpios mais extensos e pouco povoados O litoral, com destaque para o caju e algumas outras frutas.
Pop. Densidade A regio metropolitana, que se destaca com a avicultura.
Municpio rea/Km As serras midas, com a horticultura.
Total Demogrfica
O Serto semirido, com a pecuria extensiva e a produo de
Santa Quitria 42.375 4.260,68 9,95
gros.
Tau 51.948 4.018,19 12,93
Quixeramobim 59.235 3.275,89 18,08 OBS.:
Canind 69.601 3.218,42 21,63 Principais lavouras do Cear 2000: milho, mandioca, algo-
(Fonte: IBGE - 2000) do, arroz, cana, banana, caju etc.

A Regio metropolitana de Fortaleza (R.m.F) A pecuria cearense


A Regio Metropolitana de Fortaleza foi criada oficialmente A criao de gado foi responsvel pela expanso, ocupao e
em 1974, a mesma era composta por 5 municpios e atualmente organizao inicial do espao cearense. A pecuria se desenvolveu
conta com 13. A R.M.F abrange uma rea de quase 5 mil km2 no serto, notadamente de forma extensiva, seguindo os cursos
e conta com uma populao de aproximadamente 3 milhes de dos principais rios e riachos. At hoje a pecuria permanece como
pessoas, distribudas da seguinte maneira. (ver tabela). a maior riqueza da economia agrria cearense. O rebanho que me-
rece destaque o de bovinos, estando o mesmo localizado nas re-
(R.m.F.) Planefor: 2001 as do serto central e no serto dos Inhamuns, pois essa atividade
rea/ exige grandes extenses de terra (latifndios) para sua prtica.
Municpios Pop. Total IDH Ano
Km Como j foi citado, o gado mais numeroso do estado o bovino
Fortaleza 2.141 402 312 0,786 1974 (2.205.954 cabeas) seguido pelos ovinos (1.606.914 cabeas), su-
Aquiraz 60.469 481 0,6 1974 nos com (1.025.109 cabeas) e os caprinos com (789.894 cabeas).
Caucaia 250.489 1. 1 90 0,721 1974
SE LIGA
Chorozinho 18.707 307 0,633 1999 O grande destaque da atividade agropecuria cearense fica
Eusbio 31 .500 78 0,684 1987 para a avicultura, especialmente quando o importante destacar
Guiaba 19.884 270 0,652 1987 os processos de competitividade e produtividade. Muito embora
Horizonte 33.790 191 0,679 1999 praticado em todos os municpios cearenses o maior efetivo de
Itaitinga 29.217 155 0,68 1992 galinhas (56,21 %) fica por conta de dez (10) municpios locali-
zados na regio metropolitana que ainda se destacam na criao
Maracana 171.732 98 0,736 1983
de galos, frangos e pintos, assim como na produo de ovos.
Maranguape 88.135 652 0,691 1974

UECEVEST 101

Apostilas UECEVEST mod3.indb 101 06/02/2011 09:58:36


G E O G R AFIA

O extrativismo vegetal I. Os municpios de baixa densidade demogrfica concentram-


O extrativismo vegetal, praticamente inexistente nas reas de se predominantemente no interior do estado.
agropecuria mais modernizadas, porm ainda uma atividade II. As reas com densidades demogrficas mais elevadas situam-
importante no estado do Cear. A carnaba, chamada rvore da se na Regio Metropolitana de Fortaleza (RMF) e em
vida o produto de destaque, sendo a mesma encontrada nos Juazeiro do Norte.
vales (Varzeas) dos rios Acara e Jaguaribe. III. Com exceo da RMF, os municpios com a mais baixa
densidade demogrfica situam-se no litoral.
A pesca cearense
A pesca uma atividade responsvel pela ocupao e povoa- A esse respeito, correto afirmar que:
mento do litoral cearense, a mesma um meio de sobrevivncia do a) I, II e III so verdadeiras.
homem litorneo, destacando-se as espcies: cavala, serra, ariac, b) apenas I e II so verdadeiras.
cioba, camarupim, pargo etc. outros destaques so os crustceos c) apenas II e III so verdadeiras.
dentre os mais procurados temos: camaro, caranguejo e a lagosta. d) apenas I e III so verdadeiras.
A produo de lagosta e de camaro de cativeiro merecem e) apenas I verdadeira.
destaque na pauta das exportaes cearense.
02. (UFC-200S) Sobre a posio do Brasil, da regio Nordeste e
SE LIGA do estado do Cear na economia internacional, assinale a alter-
A pesca, que era uma atividade quase que exclusivamente do nativa correta.
mar, hoje se destaca tambm em audes (pesca continental). a) O Brasil manteve-se excludo da economia
internacionalizada at o final do sculo XX, quando ento
O Setor Industrial do Cear passou a participar de associaes internacionais como a
O estado do Cear apresentou nos ltimos anos o maior cres- ALCA e o MERCOSUL.
cimento industrial do Nordeste, os setores caladistas, txtil e de b) A dvida externa do Brasil, assim como o dficit da balana
vesturio, metalmecnico e alimentos, destacam-se nesta nova comercial, manteve-se crescente nas ltimas dcadas, em
fase de industrializao do estado. virtude do elevado predomnio da importao de produtos
Com uma poltica de expanso industrial o governo do esta- primrios.
do incentivou a chamada interiorizao industrial, criando para c) A regio Nordeste atrelou-se economia internacional,
isso uma ampla infra-estrutura no campo da energia, linhas de na condio de exportadora e importadora, o que reduziu
transmisso, rodovias, subestaes de gua etc. Merece destaque o desemprego e a economia informal nas suas principais
nesse processo expansionista o complexo industrial e porturio cidades.
do Pecm que foi construdo com a finalidade d) Entre as regies brasileiras, a Nordeste destaca-se na
de atrair vrias outras indstrias tais como uma termoeltrica, si- economia internacional pela produo industrial cujo
derrgica e uma refinaria de petrleo, promovendo e gerando empre- escoamento dificultado pela ausncia de infra-estrutura de
go e renda, alm de permitir uma maior projeo industrial nacional. meios a de transporte.
reas de concentrao industrial do Cear: Fortaleza e sua e) A insero do Cear na economia globalizada tornou
regio metropolitana, destaque para Maracana e Caucaia vindo necessria a melhoria dos meios de transporte, de
depois Euzbio e Maranguape; no interior do estado, destaque comunicao e ampliao das redes de informao.
para Juazeiro do Norte, Sobral, Iguatu e Crato.
O Turismo Cearense 03. (UFC-2004) Com relao ao espao geogrfico cearense, as-
O principal centro de atrao do Estado a prpria capital, sinale a alternativa correta.
que possui praias famosas como as de Iracema, as do Futuro, a) As condies climticas e o relevo aplainado das depresses
Barra do Cear, a Volta da Jurema e o Mucuripe. A cidade possui sertanejas favorecem a presena de elevadas densidades
ainda o Museu Histrico do Cear e o Museu de Arte da Uni- demogrficas.
versidade Federal do Cear, alm do Teatro Jos de Alencar e do b) Os modernos projetos de irrigao na Chapada do Apodi
Centro Cultural Drago do Mar. A 28 km de Fortaleza encontra vm empregando muita mo-de-obra na regio, sobretudo
-se a antiga cidade de Aquiraz, com o Museu Sacro de So Jos de trabalhadores permanentes.
de Ribamar. Nas proximidades da divisa com o Piau situa-se o c) Na regio do Cariri, a maioria das propriedades rurais do
Parque Nacional de Ubajara, com uma gruta calcria. Finalmen- tipo latifndio e l se encontram as mais baixas densidades
te a cidade de Juazeiro do Norte constitui importante centro de demogrficas.
artesanato e de romarias ligadas memria do Padre Ccero. d) A plancie litornea, as serras midas, a depresso sertaneja e
as chapadas se diferenciam apenas nos seus aspectos naturais.
e) O Planalto da Ibiapaba, de clima mido e sub-mido,
uma regio essencialmente agrcola, que apresenta elevadas
E X E R C C I O densidades demogrficas.
01. (UFC-2002) Observando o mapa de densidade demogrfica
do estado do Cear, analise as seguintes afirmaes. 04. (UFC) Assinale a alternativa em que todos os municpios
citados fazem parte da Regio Metropolitana de Fortaleza.
a) Maranguape, Maracana, Baturit e Fortaleza.
b) Caucaia, Pacatuba, Eusbio e So Gonalo do Amarante.
c) Aquirz, Eusbio, Canind e Horizonte.
d) Pacajus, Guaiba, Ouixad e Maracana.
e) Cascavel, Chorozinho, Itaitinga e Pacatuba.

05. (UECE-ADAPTADA) As afirmaes a seguir tm relao


com a Regio Metropolitana de Fortaleza. Assinale a verdadeira:
a) O crescimento urbano ordenado e tem obedecido
Fonte: IBGE 1996
preservao das reas ambientalmente vulnerveis.

102 UECEVEST

Apostilas UECEVEST mod3.indb 102 06/02/2011 09:58:37


GEOGRAFIA

b) O clima local de Fortaleza no tem sido afetado pela c) os planaltos sedimentares como as serras de Baturit,
verticalizao registrada na orla martima e nos bairros da Meruoca e Uruburetama renem condies favorveis para a
rea oriental da cidade. prtica da policultura de subsistncia.
c) A ocupao das dunas, fundos de vales e das plancies d) o ecossistema da caatinga no utilizado como fonte
fluviomarinhas revestidas por manguezais, desobedece produtora de energia.
legislao ambiemtal vigente.
d) reas de Proteo Ambiental (APAs) foram criadas para proteger 10. (UNIFOR) A poltica de atrao de investimentos industriais
ecossistemas como os manguezais dos rios Coc e Cear, para o Cear concedeu incentivos adicionais s empresas que de-
campos de dunas e reas serranas de Maranguape e Pacatuba. sejassem se localizar fora da Regio Metropolitana de Fortaleza.
Como exemplo dessa poltica desenvolvimentista do governo im-
06. (UFC) No que se refere ao espao geogrfico cearense, assi- plantam-se no Cear o porto do Pecm e a nova zona industrial
nale a alternativa correta. de Pacajs-Horizonte. Esses dois plos industriais determinaram,
a) O algodo, tradicionalmente cultivado nas superfcies dentre outros, os seguintes reflexos no espao metropolitano:
sertanejas, ainda hoje o principal produto agrcola do Cear. a) perda da polarizao de Fortaleza.
b) No Cariri cearense, regio de produo de cana-de-acar b) deslocamento do setor tercirio para municpios vizinhos.
e lavouras de subsistncia, esto as mais baixas densidades c) fechamento do porto do Mucuripe e do Distrito Industrial
demogrficas. de Maracana.
c) Na regio da Chapada do Apodi desenvolvem-se modernos d) ampliao dos fluxos de pessoas e de bens da grande Fortaleza.
projetos agroindustriais com base na irrigao. e) sucateamento e desvalorizao da rede ferroviria e rodoviria
d) Entre os macios residuais destacam-se o Planalto da entre o municpio de So Gonalo do Amarante e Fortaleza.
Ibiapaba, onde predominam as atividades pecurias e as
baixas densidades demogrficas. 11. (UECE) E, 25 anos, de 1974 a 1999, a Regio Metropoli-
e) No macio de Baturit predominam as grandes propriedades tana de Fortaleza (RMF) passou de 5 para 13 municpios e neIa
rurais ocupadas pela pecuria extensiva. ocorreram as seguintes mudanas:
a) Maranguape atinge a segunda posio de municpio mais
07. (UFC) Sobre a organizao do espao do Cear correto populoso do estado e Aquirz o de maior arrecadao.
afirmar: b) em 1999, a RMF agregou novos territrios de 4 municpios
a) A recente ocupao de largas faixas do litoral a de pores e triplicou os distritos industriais.
das serras midas vm sendo redefinidas, principalmente, c) a rede viria foi ampliada, embora haja, ainda, municpios
por pequenas e mdias propriedades voltadas para a sem rodovia
produo agrcola de subsistncia. d) foram implantadas linhas de metr, interligando
b) A ocorrncia de conflitos de terras, desapropriaes, os municpios litorneos.
assentamentos e assassinatos, revela as contradies da
estrutura econmica e social presente na organizao do
espao agrrio cearense.
c) O semirido cearense sempre foi um obstculo para o G A B A R I T O
capital, uma vez que este territrio nunca foi capaz de 01. e 02. b 03. e 04. b 05. c 06. c
proporcionar riqueza e acumulao.
07. b 08. c 09. a 10. d 11. b
d) A concentrao de terras e as relaes de produo no
assalariadas so caractersticas irrelevantes da organizao do
semirido cearense.
e) A expanso industrial de Fortaleza se deu atravs da
produo de bens de consumo durveis. A GEOlOGIA E A GEOmORFOlOGIA
DO ESPAO CEAREnSE
08. (UFC) As comunidades indgenas que habitaram o territrio
cearense sofreram constantes ameaas de perda de suas terras. Com
relao s comunidades ainda existentes, correto afirmar que:
a) existem 22 comunidades indgenas distribudas nas regies
da serra e do serto.
b) entre as principais indgenas, destacam-se os Ik, os Juka e
os Paiaku.
c) continuam a resistir, no territrio cearense, as comunidades
indgenas Tapeba, Trememb, Pitaguary e Jenipapo-Kanind.
d) todas as comunidades indgenas possuem suas terras
demarcadas e ocupadas em reservas federais.
e) os Trememb e os Tapeba so os grupos que, conseguiram a
retirada dos posseiros e a demarcao de suas reas.

09. (UECE) Sobre a Geografia do Cear, assinale a alternativa


correta:
a) a melhoria das condies naturais das serras midas favorece
o fracionamento da terra e o consequente predomnio de
pequenas propriedades e minifndios.
b) o serto semirido o domnio das grandes propriedades
onde se pratica a pecuria intensiva e com altos ndices de
produtividade.

UECEVEST 103

Apostilas UECEVEST mod3.indb 103 06/02/2011 09:58:38


G E O G R AFIA

O Estado do Cear compreende uma rea de 146.348,3 km2,


essa dimenso lhe confere a quarta posio na regio nordeste
frente aos demais Estados. Vale ressaltar tambm que existem
2.614 km em litgio com o Piau.
Sua capital, Fortaleza, a terceira metrpole regional do Nor-
deste e integra o grupo das regies metropolitanas do Brasil.
Desde que foi oficialmente criada, em 1974, a regio metro-
politana de Fortaleza passou a agregar novos municpios, hoje
j so 13: FORTALEZA, Aquiraz, Pacatuba, Caucaia, Maran-
guape, Eusbio, Maracana, Guaiba, Itaitinga, e os mais novos
municpios agregados em dezembro de 1999, Horizonte, Paca-
jus, Chorozinho e So Gonalo do Amarante.

Relevo
O relevo do Estado do Cear resultado da conjugao de
uma srie de fatores, onde podemos destacar como sendo mar-
cantes, a morfologia estrutural, a natureza geolgica das estrutu-
ras, a morfodinmica e os aspectos paleoclimticos.
Com base nesses elementos podemos identificar as seguintes
unidades:
Atlas escolar do Cear: Espao geo-histrico e cultural/ Jos B. da Silva e Trcia Plancies: litornea (costeira), fluvial (aluvial) e fluviomarinha.
CAvalcante, Joo Pessoa: grafset, 2000. p. 04 Glacis pr-litorneos (tabuleiros).

104 UECEVEST

Apostilas UECEVEST mod3.indb 104 06/02/2011 09:58:39


GEOGRAFIA

Planaltos sedimentares. Plancie fluvial


Macios residuais. Situada no interior do continente, esta plancie formada
Depresses sertanejas. pela ao de sedimentao do rio no seu curso mdio.

Plancies Plancie fluviomarinha


Localiza-se jusante do curso do rio, na regio de sua foz, e
Plancie Iitornea (costeira) influenciada simultaneamente pelo rio e pelo mar.
Situada aps os tabuleiros (glacis), a plancie litornea acom-
panha a orla martima. Essa plancie constituda de ecossistemas Glacis pr-litorneos (tabuleiros)
dinmicos e sensveis interferncia humana, como as praias (de- Superfcie de gradao que apresenta-se inclinada decaindo
psitos constitudos predominantemente de gros de quartzos, suavemente em direo ao litoral percorrendo quilmetros de
isto , as areias, agrupados pela ao do mar), as dunas (morros de extenso e de altimetria inferior a 100 m. Os tabuleiros so cons-
areia que sofrem deflao e adio constantes pela ao do vento, titudos por sedimentos tercirios (Era cenozica) sobrepostos ao
podem ser classificadas como dunas fixas, mveis e poleodunas), embasamento cristalino.
e os mangues (ecossistema litorneo sujeito a inundaes peri- comum o aparecimento dessas estruturas no litoral cearense em
dicas pela variao das mars, com solo, flora e fauna tpicos do formas de falsias (resultado da abraso marinha em costas litorneas
seu ambiente). elevadas), como nas praias de Morro Branco, Canoa Quebrada etc.

Atlas escolar do Cear: Espao geo-histrico e cultural / Jos B. da Silva e Trcia Cavalcante, Joo Pessoa: grafset, 2000. p-04

UECEVEST 105

Apostilas UECEVEST mod3.indb 105 06/02/2011 09:58:41


G E O G R AFIA

Planaltos sedimentares

Ibiapaba
Localizado no limite do Cear com o Piau, esse planalto constitui um exemplo tpico de cuesta com um frontngreme para o Cear
(barlavento) e um reverso suave para o Piau (sotavento).

Araripe
Localizada no extremo sul do Estado, esse planalto de topo plano e vertentes que caem bruscam.ente um clssico exemplo de
chapada.

106 UECEVEST

Apostilas UECEVEST mod3.indb 106 06/02/2011 09:58:43


GEOGRAFIA

Apodi
Tambm classificado como chapada, este planalto a nica unidade de relevo que apresenta a vertente de sotavento para o. Cear,
poro do relevo de clima seco e rido e de solos finos e pedregosos.

macios residuais Depresses sertanejas


Os macios residuais compreendem as serras cristalinas que Recobrem extensas regies do interior do Cear situadas en-
apresentam extenses e altitudes variadas. Algumas dessas serras, tre os macios antigos e os planaltos sedimentares, so marcados
normalmente as de maiores extenses e altitudes, chegam a cons- pela presena tpica da vegetao da caatinga e dos campos de
tituir verdadeiras ilhas de umidade no contexto geral semirido inselbergs. As depresses possuem solos finos e pedregosos e
do interior cearense. As serras de Baturit, Maranguape, Meruoca, dificultam a formao de lenis freticos pela pouca permeabili-
Machado, dentre outras, destacam-se entre as mais importantes. dade de suas rochas (cristalino).

UECEVEST 107

Apostilas UECEVEST mod3.indb 107 06/02/2011 09:58:44


G E O G R AFIA

E X E R C C I O 02. Quanto caracterizao geogrfica e ambiental do espao


cearense, destacam-se as seguintes unidades: plancie litornea,
01. (UFC!2001) As partes tracejadas do mapa abaixo destacam macios residuais, planaltos sedimentares e superfcies sertanejas.
as reas correspondentes a trs das grandes unidades de relevo do Analise as afirmaes a seguir e assinale a alternativa correta:
Cear, modeladas em rochas sedimentares. a) A plancie litornea a principal rea de desenvolvimento de
Marque a alternativa que indica correta- projetos tursticos, sendo que esses atendem aos cuidados da
mente essas unidades. preservao ambiental.
a) Depresso Sertaneia de Tau, Cariri e b) As superfcies sertanejas e os planaltos sedimentares so as
Jaguaribe. reas de maiores altitudes, sendo utilizadas principalmente
b) Planalto da Ibiapaba, Chapadas do para a exportao agrcola.
Apodi e Araripe. c) Os macios residuais compreendem as serras cristalinas, onde
c) Macios e Serras da Meruoca, Pereiro esto as maiores altitudes, as mais baixas temperaturas mdias
e Uruburetama. anuais e os maiores ndices pluviomtricos do Estado.
d) Plancies e Vales do Acara, Salgado e d) Nas superfcies sertanejas, tambm denominadas de serto
Jaguaribe. semirido, esto as reas de maior explorao do petrleo
e) Tabuleiro de Russas, Iguatu e Crates. no Cear.

108 UECEVEST

Apostilas UECEVEST mod3.indb 108 06/02/2011 09:58:46


GEOGRAFIA

e) Entre os macios residuais destaca-se a Chapada do Araripe, d) as condies de semiaridez vigoram, indistintamente, nos
onde predominam as atividades pecurias. diferentes ambientes fsicos do Estado.
e) nos setores relativos letra B, os solos tendem a ser mais
03. (URCA) Representam a unidade geomorfolgica mais exten- desenvolvidos que nas reas relativas letra C.
sa do Estado do Cear, abrangendo a maioria das reas dos muni-
cpios cearenses. Elas compem preferencialmente reas planas e 06. Sobre a questo ambienta I no Estado do Cear correto
suaves onduladas com altitudes inferiores a 500 metros, posicio- afirmar que:
nadas por entre os macios residuais e os planaltos sedimentares. a) o problema de degradao ambiental tem afetado somente a
Estamos nos referindo: rea litornea por ser a mais ocupada.
a) aos planaltos sedimentares; d) s plancies fluviais; b) a expanso agrcola nas serras midas, como na serra de
b) s depresses sertenejas; e) aos planaltos cristalinos. Baturit, tem seguido rigorosamente a preservao da
c) aos tabuleiros costeiros; natureza e a legislao de proteo ambiental.
c) o desmatamento indiscriminado tem sido a causa primeira
04. (UECE/2006) Tratando-se das condies geoambientais do da descaracterizao e degradao ambiental no Cear.
Estado do Cear correto afirmar que d) as plantaes obedecem os declives das serras e tm
a) h evidente preponderncia de rochas sedimentares, resolvido as perdas dos melhores solos do Cear, devido ao
condies hidroclimticas semi-ridas e submidas com processo de eroso pluvial.
baixas taxas de evaporao e evapotranspirao. e) incndios e queimadas so frequentes apenas no Serto, no
b) predominam as rochas cristalinas com solos rasos revestidos ocorrendo nas chapadas do Araripe e Apodi, uma vez que as
por plantas xerfitas e rios intermitentes sazonais. mesmas so reas de preservao ambiental.
c) h maior ocorrncia de cobertura sedimentares quaternrias,
solos rasos com baixas fertilidade natural e rede de drenagem 07. Tratando-se de Geografia do Estado do Cear, INCORRE-
sub perene. TO afirmar que:
d) Os sertes tm predominncia de arenito e calcrios, a) a bacia do Jaguaribe se destaca na parte noroeste do Estado.
condies hidroclimticas semiridas, baixa frequncia de b) os solos aluviais so potencialmente favorveis para um
cursos dgua e bom potencial hidrogeolgico. melhor desempenho das lavouras de irrigao.
c) a maior parte do territrio apresenta pequenas
05. A figura a seguir apresenta um esboo geomorfolgico do reservas de recursos hdricos subterrneos em funo de
Cear. Com base na figura e nas condies naturais do Estado, predominncia de terrenos cristalinos.
possvel afirmar verdadeiramente que: d) as lavouras xerfilas encontram condies mais
propcias de desenvolvimento nas depresses sertanejas.
e) a vegetao dos esturios constituda princi
palmente pejos mangues.

08. Explique a disposio dos aquferos em nosso Estado, utili-


zados muitas vezes como alternativas para o desenvolvimento de
projetoslocais de irrigao e abastecimento a partir da construo
de poos artesianos profundos.

09. O que deve ser mencionado quando nos propomos a classifi-


car o Cear do ponto de vista geolgico e geomorfolgico?

10. (ADAPTADA/UECE/2006) Considerando as condies


geoambientais do Cear, analise as seguintes afirmativas:
I. O desequilbrio no balano sedimentolgico do litoral tem
contribudo para intensificara eroso costeira na Regio
Metropolitana de Fortaleza.
II. As serras midas constituem enclaves de florestas e tm sido
fortemente impactadas por desmatamentos desordenados.
III. As plancies fluviais tm predominncia de solos aluviais que
so revestidos por matas ciliares.
(so) correta (s):
a) apenas I e II. c) apenas II e III.
b) apenas III. d) I, II e III so corretas

11. (UFC/2006) Sobre as diferenciaes altimtricas cearense e sua


relao com os riscos ambientais, indique a alternativa verdadeira.
a) As altitudes abaixo dos 100 metros so comuns nas
a) os setores relativos letra A apresentam condies de reas de domnio das depresses sertanejas, onde se eleva
impermeabilidade superiores s reas compreendidas pela consideravelmente o risco de deslizamento de terra.
letra B. b) A cidade de Fortaleza, localizada na rea de domnio dos
b) toda a rea correspondente letra C vulnervel tabuleiros pr-litorneos, apresenta cotas altimtricas
degradao ambienta I, pelo predomnio de topografia elevadas, acima dos 100 metros, o que elimina riscos
movimentada e fortes declives. relativos variao do nvel do mar.
c) nas formas erosivas conservadas, extremamente comum a c) A altitude em torno dos 500 metros registrada nas reas
ocorrncia de reas ribeirinhas de manguezais. mais elevadas do Estado, como na serra de Baturit e no

UECEVEST 109

Apostilas UECEVEST mod3.indb 109 06/02/2011 09:58:46


G E O G R AFIA

Planalto da Ibiapina, e estas constituem as principais reas medida que se penetra para o interior do estado, as pre-
de deslizamento de terras. cipitaes declinam, excetuando-se algumas manchas de isoietas
d) As cotas altimtricas diminuem nas plancies fluviais, o que mais elevadas, decorrentes naturalmente, de fatores locais. Influ-
as torna propensas as inundaes, portanto, estas plancies, ncias orogrficas, como o caso da Regio do Cariri, Serra do
quando ocupadas, vm a constituir as principais reas de Pereiro, Ibiapaba etc.
risco em Fortaleza. Os municpios que mais se destacaram (1953 a 1983), com
e) Os pontos mais elevados do Estado so o Pico Alto na Serra maior ndice de chuvas foram: So Benedito, Ibiapina, Aratuba,
das Matas, o Pico da Serra do Olho Dgua, na Serra de Ubajara. J os com menor ndice de chuvas foram: Parambu, In-
Baturit, reas propensas aos desmoronamentos. dependncia, Campos Sales, Aiuaba, Tau e lrauuba.

12. (UECE/2006) Sobre os sistemas ambientais do Estado do Vegetao


Cear, pode-se afirmar que estas reas so geologicamente muito A semiaridez do clima cearense visvel na paisagem vegetal
recentes, ecologicamente diversificados e ambientalmente muito das caatingas, que recobrem quase todo o territrio do estado e
vulnerveis. Tais caractersticas so constatadas principalmente so caractersticas dos sertes, porm, avanam pelo litoral, com
nos sistemas: aspectos variados, dependendo das condies climticas.
a) Litorneos Alm das caatingas, destacam-se na vegetao do Estado do
b) Serranos Cear, as formaes de matas. Matas como as de vrzeas de alu-
c) Sertanejos vio e as matas de serras. Embora de pequenas extenses, essas
d) dos Planaltos sedimentares formaes assumem importncia como reas mais dotadas para o
extrativismo e para a vida agrcola.
As matas de vrzea esto localizadas nas plancies aluviais dos
rios Jaguaribe e Acara. Tambm nas depresses e vrzeas dos
G A B A R I T O pequenos rios que possuem solos hidromrficos.
01. b 02. c 03. b 04. b 05. e 06. c As matas de carnabas ou carnaubais ocupam vastas reas do
litoral, na periferia de Fortaleza sobre os solos hidromrficos das
07. a 08. 09. 10. d 11. d 12.a depresses, lagoas e vrzeas de aluvies. Nas vrzeas sertanejas
do alto e mdio curso dos rios Jagubribe e Acara, a carnaubeira
torna-se menos frequente, desenvolvendo-se, porm, a oiticica.
Outra formao vegetal do Cear a floresta subperenif-
A HIDROGRAFIA E A VEGETAO nO lia (matas midas, localizadas sobre os setores mais elevados das
COnTEXTO ClImTICO CEAREnSE serras cristalinas e nas vertentes superiores, no norte e leste do
Planalto da Ibiapaba e da Chapada do Araripe).
Clima As chuvas orogrficas so as condicionantes principais da
As condies climticas do estado esto influenciadas pela ocorrncia das florestas serranas, aliadas a outra forma de preci-
conjugao de diferentes sistemas de circulao atmosfrica, que, pitao o orvalho, determinado pelo nevoeiro sobre os nveis
associados aos fatores geogrficos, tornam bastante complexa a mais elevados.
climatologia.
De um modo geral, a circulao atmosfrica, encontra-se regido Outras formaes vegetais
pelos deslocamentos das massas de ar. Destaca-se no estado a Massa
Equatorial Norte (mEn), a Massa Equatorial Atlntica (mEa) e, a
Massa Equatorial Continental (mEc). So tambm de represen-
tativas influncias as penetraes das massas frias provenientes do
Sul, bem como a penetrao da Convergncia Intertropical (CIT).
A combinao desses elementos, com os fatores geogrficos, tais
como, latitude, orientao do litoral, orientao das serras e a altitude,
influenciam as condies climticas vigentes no espao em estudo.
Esta caracterizam-se por apresentar elevadas temperaturas,
baixa amplitude trmica anual e, principalmente pela marcante
irregularidade das chuvas no tempo e no espao.
Mangues - vegetao halfita, localizados em reas que so-
Distribuio das precipitaes frem a influncia das mars, (Plancies Fluviomarinhas).
Carrasco - vegetao xerfita, ocorre no reverso do Planalto
da Ibiapaba, sul da Chapada do Araripe.
Cerrado - em manchas esparsas, tem sido registrada para o
Cear, a ocorrncia de vegetao de cerrado. Inicialmente sobre
os tabuleiros litorneos, e, posteriormente em pleno serto cea-
rense, os cerrados ocorrem ilhados pela caatinga.
Cerrado - est sobre a Chapada do Araripe, no nvel entre
800 e 900 metros. O porte de suas espcies e a densidade maior
de seus indivduos tem particularizado a denominao cerrado.

Hidrogafia do Cear

As precipitaes mais significativas e melhor distribudas Recursos hdricos


ocorrem no litoral e nas serras, enquanto que a maior irregulari- O regime hidrolgico dos cursos dgua no estado condicio-
dade e escassez, verifica-se nos sertes. nado, e principalmente afetado, dentre outros fatores, pela irre-

110 UECEVEST

Apostilas UECEVEST mod3.indb 110 06/02/2011 09:58:47


GEOGRAFIA

gularidade das chuvas e pelas condies geolgicas das reas onde rios com pequenas extenses, por se situarem suas nascentes em
situam-se as diversas bacias hidrogrficas. Os cursos naturais so serras relativamente prximas ao litoral. As de importncia mais
portanto intermitentes, embora que o Jaguaribe, Acara e Curu, significativas so: Pacoti, Chor e Pirangi (Serras de Baturit e
atualmente estejam perenizados a partir dos audes Ors, Araras Estevo). Vamos salientar ainda os rios Aracatiau, (Serra da Ca-
e Pentecoste, respectivamente. minhadeira) e Corea (nordeste e norte da Ibiapaba).
O estado conta com as seguintes bacias fluviais: Ressalta-se que, no Rio Pacoti, existem as represas (Aude do
Bacia do Jaguaribe Pacoti, Aude do Riacho, Aude Gavio e Acarape) responsveis
Bacia do Acara pelo abastecimento d gua, no s de Fortaleza, mas tambm de
Bacia do Curu grande parte da regio metropolitana.
Sub-bacia do Poti
Bacia dos Rios Litorneos: gua Subterrnea
A figura a seguir, mostra a distribuio das reas cristalinas e
Coc sedimentares do Estado do Cear. Pode-se comprovar a incidn-
Bacia do Pacoti Litoral Leste cia quase que total das rochas cristalinas. As reservas subsuperfi-
Chor - Pirangi ciais so alimentadas principalmente, pela infiltrao no solo de
guas de precipitao, de guas superficiais dos cursos naturais,
Bacia do Aracatiau ou ainda de lagos. Este fato vem demonstraras dificuldades de se
Bacia do Corea estimar a verdadeira potencialidade dos recursos interiores, uma
Bacia do Cear Litoral Norte vez que o armazenamento do cristalino, que representa quase que
Bacia do S. Gonalo toda a geologia do estado limitado devido sua alta resistncia
Bacia do Munda infiltrao, se restringindo, basicamente ao preenchimento das
suas reas abertas ou fraturas.
Bacia do Jaguaribe As regies onde os recursos hdricos no substrato so not-
Dentre todas as bacias do Cear, a do Rio Jaguaribe destaca-se veis, esto representados pelas reas sedimentares. O que ple-
como a mais extensa e mais importante. O Rio Jaguaribe tem suas namente justificvel, devido boa porosidade e permeabilidade
nascentes na Serra do Calogi/Pipocas/Joaninha, no Centro-Oeste dessas rochas.
do Estado. O aproveitamento de suas guas realizado principal-
mente atravs do barramento do rio principal, bem como de seus
afluentes. Podem ser citados como principais audes dessa bacia o
Ors (2.100.000.000m), alm do Aude Castanho.
Os principais afluentes so: Banabui, Palhano e Salgado. O
cristalino representa quase que o total dessa bacia. A rea restante
ento representada pelos sedimentos. Em vista do predomnio
do cristalino, a percentagem de precipitao transformada em de-
flvio alta, devido baixa taxa de infiltrao.

Bacia do Acara
A Bacia do Acara ocupa uma rea que perfaz 10% da rea to-
tal do estado. Seu principal rio, o Acara, tem suas nascentes nas
Serras das Matas, Matinha, Branca e Cupira, tendo como prin-
cipais afluentes os rios Groaras, Jaibaras e Riacho dos Macacos.
So audes importantes dessa bacia o Araras (1.000.000.000m
- Reriutaba) e Ayres de Souza (1 04.000.000m - Sobral). E o
Araras o maior reservatrio dessa bacia, tendo como finalidades
principais a perenizao e controle de cheias do Rio Acara, a
irrigao das vrzeas e a piscicultura. Texto complementar

Bacia do Curu - Perene - Pentecoste A Histria do Aude Castanho: um alvio para os


o Curu, o principal rio dessa bacia, nascendo na Serra do problemas da seca
Machado e apresentando como principais afluentes os rios Ca- O rio Jaguaribe o maior do estado e j foi considerado o
nind e Caxitor. Os principais audes so: maior rio seco do mundo. Com a instalao de vlvulas disper-
Caxitor (202.000.000m - Itapaj); soras no aude de Ors, conseguiu-se sua perenizao. Barrando
General Sampaio (322.200.000m - General Sampaio) o Jaguaribe e o Salgado, aproveitando adequadamente as guas
Pentecoste (390.638.000m - Pentecoste), Matas, Matinha, represados no seu reservatrio, o Castanho ir contribuir poro o
Branca e Curira (1.000.000.000m) desenvolvimento sustentvel de todo, o Vale do Jaguaribe.
Jaibaras (104.000.000m). O Castanho ir receber os guas do rio So Francisco, que se-
ro distribudos atravs do transposio interno de bacias, um pro-
Sub-bacia do Poti jeto que o Governo do Cear denominou de Caminho dos Aguas.
A sub-bacia do Poti tem sua origem no Estado do Cear, nas A obra ir garantir o abastecimento do Regio Metropolitano
confluncias dos riachos Correntes e do Meio, e, segue para o de Fortaleza, que em 1993 sofreu um srio problema de falto
Estado do Piau onde desgua no Rio Parnaba. dgua, resolvido emergencialmente pelo Canal do Trabalhador;
A sub-baciado Poti apresenta apenas um aude importante, que posteriormente servir para a transposio de bacias. As guas
o Realejo. do Castanho iro para o aude Banabui, depois para o Pedras
Brancas, Chor e passaro pelo Canal do Trabalhador com des-
Bacia dos rios litorneos tino o Fortaleza. Sem essa transposio, num futuro prximo o
As bacias formadas pelos pequenos rios do litoral apresentam cidade sofreria um colapso de gua.

UECEVEST 111

Apostilas UECEVEST mod3.indb 111 06/02/2011 09:58:47


G E O G R AFIA

Registro Cartogrfico Controle das enchentes do Baixo Vale do Jaguaribe, levando


tranquilidade e progresso aos proprietrios e moradores das
reas ribeirinhas;
Incremento da piscicultura local, viabilizando a produo de
3.800 toneladas de pescado por ano, o que proporcionar ab-
soro de mo-de-obra de 2.300 famlias e produo de ali-
mentos pora outras 21 mil pessoas por dia;
Gerao de energia eltrica, com a instalao de uma usina hi-
dreltrica com capacidade para produzir 22, 5 Megawatts de
potncia;
Instalao de um plo turstico, que proporcionar lazer e me-
lhor qualidade de vida aos moradores dos municpios circun-
vizinhos.
(Revista Abastece. Ano 1 - Julho/Agosto/Setembro de 1998. Adaptao: Marclio
Maurcio, Nmero zero - pg. 16 - Fortaleza-CE.)

O rio Jaguaribe
uma artria aberta
por onde escorre
e se perde
o sangue do Cear.
O mar no se tinge de vermelho
porque o sangue do Cear azul...
Todo o plasma
Toda essa hemoglobina
Na sstole dos invernos
Vai perder-se no mar.
Conhecendo mais ... e o teu crebro ainda pensa
e o teu corao ainda pulsa e
nmeros do Castanho o teu pulmo ainda respira
O aude Castanho poder acumular trs vezes o volume do e o teu brao ainda constri.
Ors e foi dimensionado para suportar uma cheio decamilenar (que Demcrito Rocha
ocorre o cada 10 mil anos), evitando o rompimento da barragem.
A vazo regularizado ser de 35m3/s, adicionado ao Ors, que
de 22m3/s, passar o ter 57m3/s. Vai gerar em torno de 90 mil E X E R C C I O
empregos permanentes, 30 mil diretos e 60 mil indiretos, benefi-
ciando uma populao de cerco de 2,5 milhes de habitantes. 01. (UFC/2002) Do ponto de vista climtico, o estado do Cear
caracteriza-se pela prevalncia do clima tropical semirido. Entre-
O Canteiro de Obras tanto, algumas reas apresentam clima tropical quente e mido.
O canteiro de obras do Castanho imenso e muitas das edifi- Marque a alternativa que indica corretamente exemplos de reas
caes construdas so permanentes, como o caso do show room com esse tipo de clima.
onde hoje so recebidos os inmeros visitantes. No futuro ser usa- a) Serto Central e dos Inhamuns.
do para divulgao e preservao da memria desta obra secular b) Litoral Norte e Mdio Jaguaribe.
e marcante para uma regio como o Nordeste brasileiro. O inte- c) Serto do Salgado e Tabuleiros do Litoral Oeste.
resse da populao em relao ao aude tem sido grande. Mais de d) Topos das serras de Baturit e Aratanha.
100 pessoas visitam a obra semanalmente. Desde outubro de 96, e) Serto Central e Chapada do Apodi.
quando foi inaugurado, o show roam j recebeu a visita de mais de
8.500 pessoas (entre pesquisadores, estudantes e profissionais de 02. (UFC/97) Quanto caracterizao geogrfica e ambiental do
todas as reas) que vm de diversos estados brasileiros e at do exte- espao cearense, destacam-se as seguintes unidades: plancie litor-
rior. O sistema de trabalho funcionou em dois turnos de 12 horas. nea, macios residuais, planaltos sedimentares e superfcies serta-
Dos quase 1.200 trabalhadores, 92% eram da prprio regio (Os nejas. Analise as afirmativas abaixo e assinale a alternativa correta:
nibus dirios que transportavam as pessoas para as cidades de Alto a) A plancie litornea a principal rea de desenvolvimento de
Santo, Jaguaribe, So Joo do Jaguaribe, Tabuleiro do Norte, Li- projetos tursticos, sendo que estes atendem aos cuidados da
moeiro do Norte e Jaguaribara, funcionaram durante toda a obra). preservao ambiental.
A obra do Castanho foi considerada de risco I mximo, por b) As superfcies sertanejas e os planaltos sedimentares so as
isso foi construdo no local um ambulatrio mdico equipado reas de maiores altitudes sendo utilizadas principalmente
para atender casos de urgncia, onde ficavam 2 mdicos e 2 enfer- para a explorao agrcola.
meiras de planto. c) Os macios residuais compreendem as serras cristalinas,
onde esto as maiores altitudes, as mais baixas temperaturas
Finalidades do Castanho mdias anuais e os maiores ndices pluviomtricos do
Desenvolvimento hidroagrcola, com irrigao de 43 mil hec- Estado.
tares de terras frteis no chapado do Castanho e derivao d) Nas superfcies sertanejas, tambm denominadas de serto
para a chapada do Apodi, via Sistema Jaguaribe-Apodi; semirido, esto as reas de maior explorao de petrleo
Reforo no abastecimento de gua da Grande Fortaleza, me- no Cear.
diante derivao, compondo o sistema Castanho - Banabui e) Entre os macios residuais destaca-se a chapada do Araripe,
- Pedras Brancas Chor - Fortaleza; onde predominam as atividades pecurias.

112 UECEVEST

Apostilas UECEVEST mod3.indb 112 06/02/2011 09:58:48


GEOGRAFIA

03. Por que a bacia do Jaguaribe considerada a mais importante d) mata seca.
bacia hidrogrfica do Cear? e) floresta de araucria (pinhais).

04. Analise as afirmativas a seguir que tratam das caractersitcas


geoambientais do Estado do Cear.
I. Os planaltos do Araripe e da Ibiapaba possuem melhores G A B A R I T O
condies litolgicas para obteno de reservas hdricas 01. b 02. c 03. 04. a 05. e
subterrneas do que os: enclaves midos Baturit e Meruoca.
06. c 07. 08. 09. e
II. Os maicios, como Baturit, so planaltos cristalinos que
constituem ambientes de exceo no contexto climtico em
que esto inseridos.
III. Nosso:Estado indistintamente marcado por solos
rasos, vegetao de caatinga, rios alimentados por chuvas REFERnCIAS BIBlIOGRFICAS
irregulares e clima semirido.
AYOADE, J. O; SANTOS, Maria Juraci Zani dos. Introduo
Diante do exposto, correto afirmar que: climatologia para os trpicos. 10. ed. Rio de Janeiro: Bertrand
a) Somente I e II so corretas. Brasil, 2004. 332 p.
b) Somente II correta.
c) Todas so corretas. CASTRO,Ina Elias de; GOMES, Paulo Cesar da Costa; COR-
d) Somente III correta. REA , Roberto Lobato. Geografia: conceitos e temas. Rio de
e) Somente I e III so corretas. Janeiro: Bertrand Brasil, 1995. 353p.

05. O Estado do Cear possui uma geografia muito particular CORRA, Roberto Lobato. O espaco urbano. Sao Paulo: tica,
dentro do contexto nacional. Por isso quando estudiosos neces- 1989. 94p.
sitam tratar de algumas questes ecolgicas do espao cearense, a GOMES, Paulo Cesar da Costa. Geografia e modernidade. Rio
melhor proposta a de se estudar o fenmeno em loco. de Janeiro: Bertrand Brasil, c 1996. 366p.
Sobre o ambiente do Cear correto afirmar que:
a) todos os planaltos midos cearenses so cristalinos. MONIZ, Antonio C.; MEDINA, H. Penna. Elementos de pe-
b) a maior parte do Estado constituda de rocha sedimentar. dologia. So Paulo: Polgono: Editora da Universidade de So
c) o Cear encontra-se inserido em uma depresso formada por Paulo, 1972. 459 p.
rocha magmtica vulcnica.
d) a inviabilidade da poltica de audagem deve se a MORAES, Antonio Carlos Robert. Geografia: pequena histo-
abundncia dos aquferos e lenis freticos no interior da ria critica. 10a ed. Sao Paulo: Hucitec, 1991. 138p.
rocha.
e) as chapadas e as serras midas so agentes determinantes ROSS, Jurandyr L. Saches. Geografia do Brasil. Ed. Ver. e
da diversificao ambiental no Estado, criando sempre ampl., 1. reimpr. So Paulo: Editora da Universidade de So
microdimas onde quer que estejam. Paulo, 2008.

06. Sobre as condies ambientais que reinam no Estado do Ce- SANTOS, Milton. A urbanizao brasileira. 5. ed. So Paulo,
ar, possvel afirmar verdadeiramente que: SP: EDUSP, 2005. 174 p.
a) no serto semirido, predomina a policultura irrigado com
rios perenes. SANTOS, Milton. O Brasil: territrio e sociedade no incio
b) na serra de Uruburetama, a prtica agrcola tem seguido as de sculo XXI. 9. ed. Rio de Janeiro, RJ; Record, 2006. 473 p.
orientaes do IBAMA quanto preservao do solo e da
flora. ____. Pensando o espao do homem. 5. ed. So Paulo: EDUSP,
c) a existncia de bacias hidrogrficas como a do Jaguaribe 2004. 90 p.
e como a do Corea, foram decisivas para a formao de
ncleos de povoamentos no serto. VILLAA, Flvio. Espaco intra-urbano no Brasil. Rio de Janei-
d) o turismo que vem sendo fortificado no litoral e nas serras ro: Studio Nobel; c1998. FAPESP, 373p.
midas tem seguido religiosamente a legislao ambiental
estabelecido pelo IBAMA.
e) a caatinga a vegetao marcante do Cear e ocupa extensas
reas do Estado, sendo encontrada principalmente no litoral,
na serra de Baturit e na chapada do Araripe.

07. Estabelea a relao entre o clima que reina em nosso Estado


e a composio de nossa bacia hidrogrfica por rios intermitentes.

08. Por que a verticalizao de Fortaleza provocou profundas al-


teraes climtico-ambientais em nossa cidade?

09. (CEFET 12006) No se relaciona cobertura vegetal do Es-


tado do Cear:
a) complexo vegetacional da zona costeira.
b) caatinga.
c) manguesais.

UECEVEST 113

Apostilas UECEVEST mod3.indb 113 06/02/2011 09:58:49


Apostilas UECEVEST mod3.indb 114 06/02/2011 09:58:49
P R - V E S T I B U l A R

HISTRIA GERAL I

Apostilas UECEVEST mod3.indb 115 06/02/2011 09:58:54


Caro(a) Aluno(a),

Para facilitar o acompanhamento de tais contedos, abaixo esto indicadas as nomenclaturas utilizadas
pela UECE e pelo ENEM:

UECE ENEM
Bizantinos e Muulmanos Sociedades medievais mediterrneas:
Muulmanos e Bizantinos
Alta Idade Mdia A Alta Idade Mdia: economia, poltica Diferentes formas de organizao
e sociedade; feudalismo europeu; da produo: feudalismo.
Baixa Idade Mdia A crise do feudalismo; Cultura Diferentes formas de organizao
medieval; processo de urbanizao; da produo: feudalismo.

Apostilas UECEVEST mod3.indb 116 06/02/2011 09:58:54


HISTRI A G E R A L I

BIZAnTInOS E mUUlmAnOS (UECE) insatisfaes internas violentas, como a Revolta de Nika (532), ini-
ciada no Hipdromo de Constantinopla, violentamente reprimida
Imprio Bizantino e enfraqueceu as fronteiras, debilitando o Imprio Bizantino.
O Imprio Bizantino tem esse nome devido cidade de Bi- Justiniano queria uma Igreja unificada, para us-la como
zncio, ex-colnia grega fundada pela desagregao do sistema apoio de seu governo. Isto explica seu cesaropapismo, isto , sua
gentlico grego. Bizncio sempre foi uma cidade muito ligada ao constante interveno na Igreja. Para no desagradar ao papa,
comrcio, em grande parte devido a sua localizao geogrfica, na tentou conciliar a heresia com a ortodoxia. Mas acabou pondo
interseo entre sia e Europa, onde hoje fica a cidade de Istam- sob sua influncia papa e a Igreja do Ocidente, que passou a as-
bul, na Turquia. Em 330 dC, o Imperador romano Constantino sumir traos da Igreja do Oriente. As divergncias entre a Igreja
autoriza a realizao de uma reforma estrutural em Bizncio e Romana e o culto bizantino foram se aprofundando com o passar
apartir da a cidade passa a ser chamada Constantinopla, cidade dos sculos, principalmente a partir do desenvolvimento das he-
de Constantino. Em 395 dC, o Imperador Teodsio procede a resias. Outra divergncia foi sobre a Iconoclastia, isto , a destrui-
uma diviso do Imprio romano em Imprio Romano do Oci- o de imagens, tambm chamadas cones. O imperador queria
dente, sediado em Roma, e Imprio Romano do Oriente, com obrigar o povo a adorar s a Deus, de imagem irrepresentvel.
capital em Constantinopla. Sendo Constantinopla anteriormen- Finalmente, em 1054 dC ocorreu o Cisma, ou diviso da
te chamada Bizncio, o Imprio passou a ser chamado informal- Igreja Crist, aps a qual foi criada a Igreja Catlica Apostlica
mente de Bizantino. Em contraste com o Imprio Romano do Romana, ocidental, e a Igreja Ortodoxa Bizantina, no Oriente.
Ocidente, que neste perodo estava decadente, sofrendo o im- A morte de Justiniano reforou o processo de lenta decadn-
pacto das invases brbaras germnicas. o Imprio do Oriente cia do Imprio Bizantino, que foi perdendo seus territrios e sua
mantinha as instituies do Imprio Romano. fora poltica e comercial.
O Imprio Bizantino um mosaico formado pela soma da No sculo XIII, Constantinopla acabou invadida pela Quarta
cultura latina, da grega, da crist e da oriental, tendo a sua econo- Cruzada e saqueada pelos venezianos. O fim do Imprio ocorreu
mia baseada principalmente no comrcio. com a invaso da Cidade, em 1453, pelos turcos otomanos.

Apogeu e decadncia do imprio Imprio muulmano


O apogeu do Imprio Bizantino se deu no governo de Justi- A Pennsula Arbica, localizada no Oriente Mdio, a terra
niano (527 a 565). Este controlava a diplomacia, as finanas, as natal dos muulmanos. Essa regio, em sua maior parte desrtica,
leis e os negcios militares, cercando-se de autoridade absoluta, possua numerosos osis e postos de caravaneiros alm de algu-
dando ao seu poder um carter quase sagrado, sendo chamado mas cidades situadas na proximidade da costa, principalmente
por isso de Csar ou at Basileus. na regio sudoeste, onde surgiram as cidades de Meca e Yatreb
(depois chamada Medina).
Os rabes, apesar de ter a mesma origem tnica (semitas),
dividiam-se em dois grupos bastante distintos entre si. Os bedu-
nos eram os habitantes do interior da pennsula, seminmades
que se deslocavam com seus rebanhos caprinos em busca de osis,
fontes de gua provisrias, onde nascia vegetao bsica. Como
havia bem menos osis que tribos, as disputas entre elas pela pos-
se dos mesmos era violentssima. O outro grupo era chamado de
rabes Felizes, habitantes do litoral, eram sedentrios, habitavam
em cidades e praticavam um intenso comrcio.

Justiniano foi importante tambm no campo jurdico, pois


preservou e organizou o Direito Romano, mandando compilar
as leis romanas no Corpus Juris Civilis (Cdigo de Direito Civil),
mais conhecido como Cdigo Justiniano, composto de quatro
trechos: o Digesto, manual de Direito, coletnea de leis redigidas
por grandes juristas; as Institutas, que reuniam os princpios fun-
damentais do Direito Romano; o Cdigo Justiniano, compilao
de todas as constituies imperiais; as Novelas, constituies pro-
mulgadas por Justiniano depois de 534.
A poltica externa defendida por Justiniano consistia em res-
taurar as antigas fronteiras do Imprio Romano, atravs de guer-
ras ofensivas. Com este objetivo, estabeleceu uma paz perptua A religio era o nico vnculo unificador dos rabes na poca
com os persas, seus antigos inimigos do lado oriental. Esse expan- pr-islmica. Meca servia como uma espcie de capital religiosa,
sionismo teve incio com a reconquista da frica, sob domnio porque nesta cidade estavam os tempos dos mais de trezentos e
dos vndalos. Depois, Justiniano investiu contra os ostrogodos na sessenta deuses adorados pelos rabes, bem como a Caaba, em cujo
Itlia e contra os visigodos na Espanha meridional. interior era guardada a Pedra Negra, provavelmente um meteoro,
O Imprio Bizantino chegava, dessa forma, ao limite mximo reverenciada por todos os rabes que para l se dirigiam em pe-
de sua expanso geogrfica e militar. regrinao. Todos os anos as tribos se deslocavam para Meca em
Para sustentar administrativa e militarmente suas novas fron- peregrinao e eram recebidas pelos Coraixitas, tribo dos guardies
teiras, Justiniano foi obrigado a aumentar os impostos. Isso gerou dos lugares sagrados e controladores do comercio religioso.

UECEVEST 117

Apostilas UECEVEST mod3.indb 117 06/02/2011 09:58:55


H I S T RIA GERAL I

maom (570-632) A arte muulmana no tem muita originalidade. A proibio


A Histria da Arbia dividida em antes e depois do Islamis- religiosa de reproduzir formas vivas prejudicou o desenvolvimen-
mo pregado por Maom. Este rabe nasceu pobre em Meca, no to da pintura e da escultura. A contribuio mais significativa
570, e ficou rfo de pai e me com pouco mais de cinco anos de est na arquitetura das mesquitas, nos arabescos para ilustrao
idade. Sendo criado por um tio, Dedicou-se desde muito cedo ao e decorao.
trabalho em caravanas, conheceu outros povos do Oriente M- No campo das cincias, os rabes cultivaram a Astronomia, a
dio e entrou em contato com o cristianismo e o judasmo. Por Matemtica, a Qumica e a Fsica. Inventaram o cido sulfrico
volta de 595, Maom casou-se com a rica viva Khadidja e pode e o lcool. Introduziram os algarismos hindus, depois chamados
dedicar-se contemplao e meditao religiosa. arbicos. A medicina outra rea do conhecimento de destaque,
Em torno do ano 610, recebeu a revelao do Anjo Gabriel atravs de Avicena, maior mdico da idade mdia.
e iniciou suas pregaes em Meca. Maom converteu primeiro Na literatura, destacam-se As Mil e Uma Noite, o Livro dos
seus familiares e em segui da tentou converter os Coraixitas, ao Reis e Rubayat, de Omar Khayan. Na filosofia, destaca-se a figura
seu novo credo: S Al Deus e Maom seu nico Profeta, de Averris (1126-1198), de Crdova, que traduziu Aristteles
sem obter xito. para o rabe e o apresentou ao Ocidente.
Perseguido, fugiu de Meca para Yatreb, no episdio conhecido
como Hgira (622), que marca o incio do calendrio muulmano.
Chegando a segunda maior cidade rabe, Maom foi bem recebi- E X E R C C I O
do, tornando-se em pouco tempo administrador da cidade. Para
homenage-lo, o nome da cidade foi modificado para Medina (ci- 01.(UFPB 2005) Bizncio, tambm chamada de Constantino-
dade do profeta). Maom, ento, organizou um grande exrcito e pla, e, depois, de Istambul, capital da atual Turquia, era o centro
dominou Meca, em 630, adotando o principio do Jihad. de poder do Imprio Romano do Oriente, constituindo-se numa
Em 632, morreu em Medina, onde construra a primeira experincia histrica relevante e distinta, sob muitos aspectos, em
mesquita do Isl. Ao morrer, deixou o mundo rabe politicamen- relao s sociedades medievais do Ocidente europeu. A civiliza-
te unificado e reunido em torno de uma religio, o islamismo. o bizantina no tem como caracterstica:
A unidade do mundo muulmano foi quebrada aps a mor- a) Cristianismo ortodoxo, diferenciado do apostlico romano.
te do Profeta com o surgimento de vrios movimento, entre os b) Poder poltico fragmentado e feudal.
quais se destaca o Xiita e o Sunita, que ainda hoje apresentam c) Desenvolvimento do comrcio e da vida urbana.
antagonismos. d) Poder poltico centralizado e teocrtico.
e) Diversidade cultural de base grega, romana e asitica.
Expanso muulmana (Sculos VII a XI)
Com a morte de Maom, surgiram os califas (sucessores do 02. (UFPI 2004) Leia o trecho do Coro, livro sagrado da reli-
Profeta). Os dois primeiros califas, Abu Bekr e Omar, souberam gio islmica: Quando vos dispuserdes orao, purificai antes o
assumir a sucesso e a herana de Maom, exercendo autoridade rosto, as mos at os cotovelos, a face at as orelhas, e os ps at os
civil, militar e religiosa. Essa autoridade consolidou a unificao tornozelos. (...) Socorrei vossos filhos, vossos parentes, os rfos,
da Pennsula Arbica e possibilitou a expanso islmica. os pobres, os peregrinos; o bem que fizerdes ser reconhecido do
Mas foi na dinastia dos Omadas que os maometanos con- Onipotente. (...) Aquele que d por ostentao semelhante a
quistaram a frica do Norte, a Pennsula Ibrica at o sul da G- um rochedo coberto de p; vem a chuva e no lhe resta seno
lia e as ilhas da Crsega, Sardenha e Siclia. Em 750, em Damas- sua dureza. A partir do texto acima e de seus conhecimentos,
co, um golpe poltico afastou os califas Omadas e colocou no assinale a alternativa correta sobre o Coro.
poder os Abssidas, parentes do Profeta, que instalaram a capital a) Exerce, na atualidade, profunda influncia na vida cotidiana
em Bagd (Mesopotmia) e orientaram as conquistas rumo ao dos islmicos, como no que diz respeito alimentao e a
Oriente, em direo ndia e China. higiene.
b) Contm as biografias dos califas e constitui-se o livro
sagrado dos muulmanos.
c) Sugere comportamento para os fiis, evitando uma
linguagem que expresse ordem ou advertncia.
d) Oferece ensinamentos para a vida espiritual, omitindo-se das
questes relativas ao convvio social.
e) Influenciou profundamente a Bblia, livro sagrado dos
cristos, quanto doao de esmola.

03. (FEI 2004) No Sculo XI, o cristianismo sofreu o Grande


Cisma do Oriente. Este fato levou perda do controle da Igreja
de Roma sobre grande parte da Cristandade. A consequncia ad-
vinda do Cisma foi:
a) O surgimento do Islamismo
b) O surgimento da Igreja Anglicana
c) A proliferao de heresias pela Europa Oriental
d) A separao entre a Igreja Catlica Apostlica Romana e a
Cultura muulmana Igreja Ortodoxa
Os rabes criaram um imprio mercantil. Produtos iam e vi- e) A revogao de uma srie de dogmas por parte da Igreja
nham da sia central, frica, Europa central e ocidental. Surgiram Catlica, como a Santidade de Cristo.
grandes centros comerciais, como Bagd, Crdova, Cairo, Baora,
Damasco, Fez. Novas culturas chegaram ao Ocidente, como c- 04. (UFRN 2007) Desde a poca dos apstolos, a Igreja crist
nhamo, amoreira, algodoeiro, cana-de-acar, laranjeira, limoeiro afirmava-se una, mas isso no a impedia de assumir caractersticas
e arroz. Novas indstrias, como mosaico, cermica e vidro. peculiares em diversos territrios. Em 1054, tais diferenas no

118 UECEVEST

Apostilas UECEVEST mod3.indb 118 06/02/2011 09:58:55


HISTRI A G E R A L I

seio da Cristandade provocaram o Cisma do Oriente, que cul- d) As duas opinies representam verses dogmticas, que
minou com desrespeitam os direitos humanos.
a) o fracionamento do Imprio Bizantino em Imprio Romano
do Ocidente, dominado pelo Papa, e Imprio Romano do 08. (UEL 2000) Muito se tem falado sobre a origem do islamis-
Oriente, controlado pelo Patriarcado de Constantinopla. mo. Os contatos entre o mundo muulmano e o Ocidente marca-
b) o desmembramento do Tribunal da Inquisio, com uma ram profundamente a histria de ambos, seja pelos conflitos, to
seo liderada pelo Papa na Igreja Catlica Romana, e constantes no percurso da humanidade, seja pelos intercmbios
outra chefiada pelo Patriarcado de Constantinopla na Igreja culturais, to necessrios ao desenvolvimento de todos. Sobre a
Ortodoxa. cultura islmica durante o perodo medieval, correto afirmar:
c) a separao entre o poder espiritual, comandado pelo Papa a) O islamismo nasceu com a tribo dos sunitas, povo de origem
no Ocidente, e o poder temporal, exercido pelo Imperador indo-europeia.
bizantino no Oriente. b) Os europeus da Pennsula Ibrica foram responsveis pela
d) a diviso entre a Igreja Catlica Romana, dirigida pelo introduo da filosofia grega na cultura dos muulmanos.
Papa, e a Igreja Ortodoxa, subordinada ao Patriarcado de c) A expanso muulmana, liderada pelos rabes, foi marcada
Constantinopla. pelo respeito aos costumes e crenas dos povos conquistados.
d) A lenta difuso do Isl explica-se pela complexidade de seus
05. (UECE 2008/1) Sobre os fundamentos do Isl ou Islame, preceitos, que dificultaram o aumento do contingente de
assinale o correto. crentes no Mediterrneo.
a) uma religio politesta que surgiu no final do sculo e) A supremacia econmica da Europa feudal incentivou o
IV d.C. e tem em Maom seu principal mrtir. Seu livro desenvolvimento do comrcio no Imprio rabe.
sagrado o Talmude.
b) uma religio monotesta que surgiu no sculo X d.C.. Sua 09. (UECE 2010/2) Os rabes deixaram traos permanentes na
sede religiosa a cidade de Medina e seu livro sagrado a cultura europeia, seja nas artes, seja no campo lingustico. H
Kaaba. inmeras palavras, em vrios idiomas indo-europeus, que so
c) uma religio politesta que surgiu no sculo I d.C.. Sua de origem rabe. Por exemplo: lgebra, alquimia, lcool, cifra,
sede Jerusalm, Maom seu fundador e no tem um livro zero etc. Sobre a presena dos povos rabes na Europa, assinale
sagrado. o correto.
d) uma religio monotesta que surgiu no sculo VII d.C. a) Os rabes no conseguiram consolidar seu poder poltico,
Seu profeta Maom e seu livro sagrado o Alcoro. criando apenas um relacionamento comercial e de trocas
com alguns lugares da Europa.
06. (ESPM 2007) Quando vos dispuserdes orao, purificai b) Os rabes fundaram um imenso imprio islmico,
ante o rosto, as mos at os cotovelos, a face at as orelhas, e foram tolerantes em relao s populaes submetidas e
os ps at os tornozelos. O asseio a chave da orao. Socorrei contriburam para o florescimento das artes.
vossos filhos, vossos parentes, os rfos, os pobres, os peregrinos; c) As trocas lingusticas entre rabes e europeus foram muito
o bem que fizerdes ser conhecido do Onipotente. Dai esmola superficiais, dado o curto tempo de contato entre estes
de dia, noite, em pblico ou em segredo; sereis recompensados povos.
pelas mos do Eterno e ficareis isentos dos terrores e tormentos. d) Em nenhum momento da histria humana houve o
Aquele que d por ostentao semelhante a um rochedo coberto predomnio dos povos rabes em solo ocidental.
de p; vem a chuva e no lhe resta seno sua dureza. No ms de
Ramad, comer e beber s vos permitido at o momento em
que a claridade vos deixar distinguir um fio branco de um fio
preto: jejuai ento do comeo do dia at a noite e passai o dia em
orao. (Leonel Itaussu e Lus Csar. Histria Antiga e Medieval) 10. (ENEM 2008) Existe uma regra religiosa, aceita pelos pra-
Quanto ao trecho apresentado no enunciado devemos relacion- ticantes do judasmo e do islamismo, que probe o consumo de
lo com: carne de porco. Estabelecida na Antiguidade, quando os judeus
a) O Alcoro e algumas das obrigaes dos muulmanos. viviam em regies ridas, foi adotada, sculos depois, por rabes
b) O Suna e algumas das obrigaes dos muulmanos. islamizados, que tambm eram povos do deserto. Essa regra pode
c) O Talmud e algumas das obrigaes dos muulmanos. ser entendida como
d) O Tora e algumas das obrigaes dos muulmanos. a) uma demonstrao de que o islamismo um ramo do
e) O Zend-Avesta e algumas das obrigaes dos muulmanos. judasmo tradicional.
b) um indcio de que a carne de porco era rejeitada em toda a
07. (UECE 2005/1) Para o Oriente, Maom o Apstolo de sia.
Deus, todos os atos de sua vida foram divinamente inspirados e o c) uma certeza de que do judasmo surgiu o islamismo.
Coro que ele proclamou a Palavra de Deus... Para seus adver- d) uma prova de que a carne do porco era largamente
srios, a partir dos de Meca, idlatras, at os sectrios de outras consumida fora das regies ridas.
religies, Maom um impostor que se investiu de Profeta para e) uma crena antiga de que o porco um animal impuro.
satisfazer sua sede de poder e assentar sua dominao sobre Meca
e as tribos rabes. (PELLEGRIN, Arhur. LIslam des le monde.
Paris: Ed. Payot, 1950, p. 26) Ante as duas verses apresentadas,
correto afirmar: G A B A R I T O
a) A primeira expressa a viso dos seguidores de Maom, 01. b 02. a 03. d 04. d 05. d
defensores do ecumenismo.
06. a 07. d 08. c 09. b 10. c
b) A Segunda revela a opinio da Igreja Catlica ante a
expanso do islamismo.
c) As duas explicaes atestam a fora do monotesmo como
opo religiosa, favorvel a conciliao.

UECEVEST 119

Apostilas UECEVEST mod3.indb 119 06/02/2011 09:58:56


H I S T RIA GERAL I

IDADE MDIA OCIDENTAL (UECE/ENEM) e a ustria. Dessa maneira, os territrios voltaram a ser divididos
entre seus filhos e netos, de tal forma que cerca de sessenta anos
A Idade Mdia um perodo compreendido entre o ano de aps a morte de Carlos Magno, o Imprio estava todo fraciona-
476 e 1453 sendo caracterizada pela desagregao da sociedade do, beneficiando a nobreza rural (condes, duques e marqueses).
romana antiga e pela formao do sistema feudal. Enquanto a O poder dos nobres passou a ser quase absoluto em suas pro-
parte oriental do Imprio Romano mantinha-se com o nome de priedades, tornando-os independentes do poder real. Assim, o
Imprio Bizantino, a parte ocidental foi invadida por diversos poder local tornou-se mais importante do que o poder central,
povos brbaros, dentre os quais se destacam os francos. que acabou sendo descentralizado. Essa descentralizao foi fun-
damental para a estruturao do sistema feudal.
Francos
Os francos eram um povo de origem germnica que comeou Feudalismo
a se fixar na regio que hoje compreende a Frana, por volta do S- Feudalismo ou sistema feudal o sistema econmico, poltico
culo IV dC, aproveitando-se da fragilidade do Imprio Romano, e social, vigente na Europa Ocidental, durante a Idade Medieval.
mergulhado em sua crise estrutural. Assim como todos os outros O Feudalismo deve seu nome ao feudo, grande propriedade rural
germnicos, os francos ainda eram descentralizados politicamente pertencente a um nobre, que busca a autossuficincia. O feudo
em tribos. Clvis foi o lder que os unificou, em meados do Sculo subdividido em trs partes:
V, dando origem Dinastia Merovngia. Com a morte de Clvis Manso senhorial local onde se localizava o castelo; a posse da
houve um enfraquecimento do poder central. Fracos e quase sem produo pertencia ao senhor;
nenhuma autoridade, os prncipes herdeiros foram denominados Manso servil dividido em lotes; local onde os camponeses
de reis indolentes. Administrao passou a ser exercida pelos pre- (servos) produziam seu sustento;
feitos do palcio. Estes exerciam funes de administradores do Manso comunal composto por pastagens, bosques e flores-
palcio, comandantes do exrcito, coletores de impostos, organi- tas, era usado pelo senhor e pelos servos.
zadores das divises da terra etc. Em 732, o prefeito do palcio
Carlos Martel deteve o avano muulmano sobre o reino franco
na Batalha de Poitiers. Aps a morte de Martel, seu filho Pepino,
o Breve, assumiu o cargo. Apoiado pelo papa, Pepino afastou o
ltimo rei merovngio e foi coroado rei dos francos, em 751, ini-
ciando a dinastia carolngia, em troca foi cedido Igreja Catlica
o Territrio de So Pedro, localizado no Centro da Itlia, que deu
origem ao que hoje o Estado do Vaticano.

Modo de vida feudal


Carlos Magno, Rei dos Francos
Origens
Carlos Magno foi o mais importante Rei dos Francos, tanto O sistema feudal pode ser compreendido como uma fuso de
assim que cedeu seu nome Dinastia. Carlos lembrado pela s- duas outras culturas: a romana e a germnica. Pelo lado roma-
rie de conquistas que fez. Por exemplo territrios como a Saxnia no, contriburam para a formao do feudalismo a ruralizao da
(atual Alemanha), a Frsia (parte da atual Holanda) e a Catalunha economia provocada pela desintegrao do mundo romano devi-
(parte da atual Espanha)regies conquistadas em seu mandato. do s invases brbaras, bem como o colonato, ou seja, o trabalho
Entretanto, o mais importante de seu longo governo (771-814) de camponeses livres que eram remunerados pela produo. Os
foi a aliana com a Igreja, tanto assim que em 800 foi coroado germnicos trouxeram para o feudalismo duas contribuies b-
imperador do Sacro Imprio Romano do Ocidente. Para facilitar sicas: comitatus, o pacto de lealdade celebrado entre os guerreiros
a administrao de seu territrio, Magno distribua terras entre e seu lder pela conquista de novas terras. Aps a conquista dessas
seus guerreiros, dividindo-o em condados, ducados e marcos. novas terras, o comandante dividia entre seus comandados a pos-
Havia ainda os missi dominici, funcionrios encarregados da fis- se das mesmas, gerando o chamado beneficium.
calizao do imenso territrio.
Caractersticas
Em 814, Carlos Magno morreu, sendo sucedido por seu fi-
O feudalismo pode ser compreendido a partir de suas cinco
lho Lus, o Piedoso, que governou at 840. Aps algumas lutas
caractersticas bsicas:
internas, os netos de Carlos Magno assinaram o Tratado de La-
Descentralizao poltica: Cada feudo era uma entidade po-
tro (843) que dividiram o Imprio da seguinte forma: Luis,o
ltica autnoma, sendo a relao do senhor feudal com o seu
Germnico, com o territrio que hoje corresponde Alemanha,
soberano, o rei, era meramente formal e de carter militar.
Carlos, o Calvo, ficou com aquilo que hoje a Frana e Lotrio,
Economia agrria autossuficiente: A unidade bsica de riqueza
com o trecho central que compreende o Norte da Itlia, a Sua

120 UECEVEST

Apostilas UECEVEST mod3.indb 120 06/02/2011 09:58:57


HISTRI A G E R A L I

no mundo feudal era a posse da terra. Dessa maneira, o feudo preparao para a vida aps a morte.
buscava a autossuficincia como meio de assegurar tanto a au- importante perceber que na Idade Mdia contrapunham-
tonomia poltica, quanto a estabilidade econmica. se dois elementos importantes: de um lado, a fora da Igreja no
Comrcio incipiente: Devido a autosuficincia, almejada pelos campo espiritual; de outro, a. fraqueza dos reis. Assim, a Igreja
feudos, o comrcio virtualmente deixou de existir. Sendo uma Catlica monopolizou o saber e o poder na Europa Ocidental.
atividade econmica meramente secundria.
Economia Amonetria: Sendo o comrcio prejudicado pela
autossuficincia, o dinheiro praticamente deixou de circular E X E R C C I O
durante o apogeu do feudalismo.
Sociedade Estamental: A lgica da sociedade feudal pode ser 01. (UECE 2010/2) Entre os sculos VI e VIII, passaram a figu-
muito bem resumida na frase: Uns rezam, outros combatem e rar no contexto histrico da Europa povos que at ento estavam
muitos trabalham. Na verdade, o mundo feudal praticamente s margens, que eram conhecidos como brbaros: germanos,
desconhecia a mobilidade social, dado que a riqueza medida francos, godos, lombardos, hunos etc. Estes povos consolidaram
pela posse da terra, que no comercializada, ou seja, a sociedade seu poder e formaram reinos independentes, dando incio a uma
estaticamente dividida em trs nveis: Clrigos, Nobres e Servos. nova fase da histria do mundo ocidental. Assinale a afirmao
verdadeira.
Sociedade a) O termo brbaro era utilizado para reconhecer e
A base da sociedade feudal eram os Servos. O servo era um glorificar aqueles povos que negociaram com os romanos e
trabalhador braal responsvel pela produo agrcola. No en- alternaram, com estes, a posio de poder.
tanto, por no ser proprietrio da terra, que pertencia ao senhor b) A palavra brbaro era empregada por gregos e romanos
feudal, o servo devia a este uma srie de obrigaes, como as que para delimitar o que para eles significava povos no
se seguem: civilizados.
Talha Obrigao paga pelo servo para trabalhar na proprie- c) Os diferentes povos, oriundos de locais geograficamente
dade feudal, funcionando como uma espcie de arrendamento. distantes, se autodenominavam brbaros.
Captao Taxa paga por cada habitante do feudo para ter o d) A denominao brbaros foi empregada especificamente
direito de residir na propriedade feudal. pelos historiadores para contrapor a romanos.
Banalidade Imposto cobrado sobre o uso de ferramentas
(ps, foices, etc) e benefcios da propriedade (fornos, silos, de- 02. (UECE 2009/2) A sociedade feudal foi estratificada e imobi-
psitos) que pertencem ao senhor feudal, pelo servo. lista, caracterizada por relaes sociais verticais. A posse da terra
Corveia Trabalho gratuito obrigatrio realizado pelo servo constitua o critrio diferenciador entre os grupos sociais. Sobre a
no trecho da propriedade chamada de manso senhorial, de usu- sociedade feudal, correto afirmar que
fruto exclusivo do senhor feudal. a) Dividiu-se em inmeros grupos sociais com facilidade para
Mo-Morta Indenizao paga pela famlia do servo falecido a mobilidade social graas s conquistas das cruzadas e das
para ter o direito de continuar a habitar o feudo aps a sua guerras.
morte. b) Foi estamental e rgida, apresentou dificuldade para
Tosto de So Pedro Pagamento feito pelo servo Igreja, mudana de status social, sendo os senhores feudais e os
equivalente a uma espcie de dzimo. servos seus principais representantes.
c) Permitiu a mudana de status, pois combateu a hierarquia
bom que fique claro: quando se fala de pagamento este rgida e as guerras permitiram que houvesse ascenso social.
realizado atravs de percentuais da produo retidos pelos senho- d) Manteve a centralizao do poder nas mos da realeza
res feudais e pela Igreja, ficando o servo, ao final dos descontos, e possibilitou a desestruturao da hierarquia social
com apenas um oitavo do que houvera colhido. predominante.
A nobreza feudal a ordem detentora do poder poltico e
tambm do poder militar. O poder poltico no sistema feudal era 03. (UFC 2007) Acerca dos aspectos socioeconmicos do sistema
exercido pelos senhores feudais (nobres), da seu carter local e feudal, correto afirmar que:
descentralizado. O rei mantinha apenas um poder nominal, no a) era um sistema que tinha como base a existncia da pequena
exercendo autoridade efetiva. propriedade fundiria, com produo de subsistncia.
A defesa militar era feita pelos senhores feudais, que, para se b) foi uma forma de organizao do Estado e da sociedade cuja
protegerem, procuravam relacionar-se diretamente por meio de essncia residia nos vnculos de subordinao pessoal.
um compromisso: O juramento de fidelidade. Esse juramento era c) impediu o desenvolvimento das cidades e do comrcio nos
realizado em uma cerimnia na qual um nobre prestava homena- vrios sculos em que caracterizou o conjunto do mundo
gem a outro, recebendo terras em troca de algumas obrigaes. europeu.
O nobre que prestasse o juramento tornava-se vassalo e aque- d) significou a completa substituio da cultura e das
le que o recebesse tornava-se suserano. Na hierarquia feudal, su- instituies romanas pelas organizaes germnicas,
seranos e vassalos tinham obrigaes recprocas, como servios difundidas pelos invasores brbaros.
polticos (garantir a hereditariedade de transmisso do feudo) e e) caracterizou-se pelo trabalho escravo e pelo desenvolvimento
militares (proteo contra invases ou revoltas servis). acelerado das tcnicas agrcolas, que garantiu incremento
Durante a Idade Mdia, a Igreja Catlica adquiriu uma im- demogrfico durante toda a Idade Mdia.
portncia fundamental, uma vez que foi a nica instituio que
se manteve intacta aps a desagregao do Imprio Romano do 03. (IFCE 2006/2) Refere-se s caracterstica bsicas do sistema
Oriente. O poder da Igreja advinha do fato de ela exercer o mo- feudal:
noplio cultural (controle do conhecimento) que exercia. A Igre- a) Produo destinada s trocas comerciais
ja era a responsvel pela produo dos livros e fazia um controle b) Poder poltico centralizado em relao ao rei
muito rgido sobre o que deveria ser lido pelos catlicos. c) Sociedade estamental que favorecia a ascenso social
A influncia da Igreja era incontestvel e dominava a menta- d) As relaes entre servos e senhores feudais eram
lidade do homem medieval, para quem a vida na terra era uma determinadas por obrigaes recprocas: os servos

UECEVEST 121

Apostilas UECEVEST mod3.indb 121 06/02/2011 09:58:57


H I S T RIA GERAL I

trabalhavam nas terras do senhor e pagavam com produtos b) A mentalidade era profundamente marcada pelo iderio
a utilizao da terra, e o senhor feudal oferecia proteo catlico, que preconizava, inclusive, o papel que homens e
militar mulheres deveriam desempenhar na sociedade.
e) O sistema de produo feudal apresentava alta produtividade c) A submisso feminina autoridade masculina caracterizou
e relaes de trabalho baseadas no pagamento de salrios a sociedade daquele tempo como uma organizao
tipicamente matriarcal.
05. (URCA 2006/1) Com uma base agrria muito forte, o siste- d) A mulher, ainda que posta em uma condio submissa em
ma conhecido como feudalismo, desenvolveu formas especficas relao ao homem, tinha grande poder e influncia sobre a
de explorao da propriedade rural chamada senhorio ou dom- Igreja Catlica.
nio. O dono da terra era senhor de tudo: declarava a guerra, fazia e) A condio feminina era fruto da grande influncia que o
a paz, aplicava as leis, concedia privilgios. As relaes sociais do racionalismo cientfico exercia sobre a cultura daquele perodo.
feudalismo foram caracterizadas:
a) Pela ausncia da mobilidade social. Quem tinha uma origem 09. (UFRN 2007) O feudalismo substituiu o escravismo antigo
pobre, morria pobre trabalhando nas terras do seu senhor, e estabeleceu novas relaes de trabalho, baseadas na explorao
quem nascia nobre iria viver para defender e ampliar suas da mo-de-obra servil. Nessa condio, os servos
riquezas; a) seriam tratados como mercadorias e vendidos nas feiras
b) O servo poderia deslocar-se livremente para ofertar sua mo- realizadas nos burgos, quando aprisionados nas guerras feudais.
de-obra ao senhor que estivesse disposta a pagar mais pelos b) eram trabalhadores que deviam obedincia e obrigaes ao
seus servios; seu senhor e estavam ligados terra em que viviam, no
c) Apesar de deter o poder da terra, o senhor feudal no tinha podendo ser vendidos.
obrigaes para com o servo, que estava sendo assegurado c) eram considerados propriedade dos senhores feudais e
pelo Rei Absolutista; poderiam ser trocados ou vendidos nos mercados locais.
d) Por uma rpida transio entre os valores medieval-cristos d) seriam transformados em trabalhadores assalariados caso no
para moderno-cientficos, onde as relaes sociais e polticas pagassem regularmente os tributos devidos ao senhor.
foram objeto de formulaes cientficas baseadas em
princpios como cincia e racionalismo; 10. (UFRN 2002) No ano de 786, Carlos Magno afirmou: A
e) O servo estava preso - terra, devendo obrigaes ao seu nossa funo , segundo o auxlio da divina piedade, (...) defen-
senhor. Essas obrigaes eram renovadas periodicamente der com as armas e em todas as partes a Santa Igreja de Cristo dos
atravs de processos de eleies, o que permitia uma ataques dos pagos e da devastao dos infiis. (PINSKY, Jaime
constante renovao dos nobres que ocupavam o poder (Org.). O modo de produo feudal. 2. ed. So Paulo: Global,
feudal. 1982. p. 101.) O fragmento acima expressa a orientao poltica
do Imprio Carolngio no governo de Carlos Magno. O objetivo
06. (IFCE 2005/2) Caracteriza corretamente o Feudalismo: dessa poltica pode ser definido como um(a)
a) Economia essencialmente agrria, trabalho regulado pelas a) Esforo para estabelecer uma aliana entre os carolngios e a
obrigaes servis e cultura teocntrica Igreja bizantina para fazer frente ao crescente poderio papal.
b) Economia sob a interveno do Estado, trabalho assalariado b) Inteno de anexar a Pennsula Ibrica aos domnios do
e cultura teocntrica papado, com a finalidade de impedir o avano rabe.
c) Sociedade rural e estamental, trabalho assalariado c) Desejo de subordinar os domnios bizantinos dinastia
d) Economia agrria, trabalho assalariado e cultura teocntrica carolngia, no intuito de implantar uma teocracia
e) Economia estatal e trabalho regulado pelas obrigaes servis centralizada no Imperador.
d) Tentativa de restaurar o Imprio Romano, com vistas a
07. (UECE 2007/2) A morte do estado, como era concebida promover a unio da cristandade da Europa Ocidental.
no mundo antigo, coincide com a afirmao do feudalismo. Isso
pode ser explicado, pois:
a) A autoridade central delegou cada vez mais o exerccio dos G A B A R I T O
poderes pblicos aos senhores feudais, que se tornaram
praticamente soberanos em seus feudos. 01. b 02. b 03. b 04. d 05. a
b) A figura do soberano se reduz quela de um fantoche nas 06. a 07. c 08. b 9. b 10. d
mos da Igreja e dos grandes senhores feudais.
c) A nobreza aproveitou-se para conseguir, do poder do soberano,
concesses cada vez maiores, como a hereditariedade dos feudos.
d) O controle das frequentes insurreies camponesas contra o
abuso de poder dos senhores e do mal governo torna-se difcil. BAIXA IDADE mDIA (UECE/EnEm)
08. (UFC 2008) Na sociedade medieval, vigorava uma ideologia
que considerava as mulheres inferiores aos homens, resultando
em um cotidiano marcado pela hegemonia da autoridade mas-
culina. Ainda que a Igreja pregasse que homens e mulheres eram
objetos do amor de Deus, no eram poucos os religiosos que per-
cebiam as mulheres como agentes do demnio. Com base nas
informaes acima e em seus conhecimentos, assinale a alterna-
tiva correta sobre a cultura e a sociedade europeias, no perodo
classicamente conhecido como Idade Mdia.
a) As mulheres eram consideradas inferiores aos homens por
serem incapazes de trabalhar com as tcnicas tradicionais de
cura por meio do uso de plantas medicinais. Episdio da Fuga do Egito, de Fra Anglico (1387-1455)

122 UECEVEST

Apostilas UECEVEST mod3.indb 122 06/02/2011 09:58:58


HISTRI A G E R A L I

Durante a Baixa Idade Mdia, na Europa Ocidental, ocorre- Renascimento comercial e urbano
ram vrias transformaes que provocaram a decadncia do siste- As Cruzadas dinamizaram as relaes comerciais na Europa
ma feudal. A partir do sculo XI, houve um perodo de paz e de Ocidental. Novos produtos entraram em circulao dentre eles,
segurana, resultante do fim das invases brbaras e da diminui- as especiarias (produtos de luxo vindos do Oriente) e as moedas,
o das epidemias. Como consequncia, houve a diminuio do que voltaram a ser usadas no intercmbio comercial.
ndice de mortalidade. A populao crescia e, ao mesmo tempo, a Os comerciantes dirigiam-se das cidades italianas (Gnova e
economia se dinamizava. O crescimento populacional gerou mais Veneza) para os entrepostos comerciais do Mediterrneo Oriental:
mo-de-obra e maior mercado consumidor, fatores que estimula- Constantinopla, Alexandria, Cairo, Antioquia (Rota do Mediter-
ram a produo de excedentes comercializveis. rneo). De Veneza e Gnova, atravs da Frana, atingiam Flandres,
Uma parcela da populao deslocou-se para atividades no vin- centro manufatureiro de l, promovendo o desenvolvimento das
culadas terra, como o artesanato e o comrcio, que impulsionou atividades comerciais nas feiras de Champagne (Rota de Cham-
o renascimento de antigas cidades e a fundao de outras novas. Es- pagne). De Flandres, seguindo o roteiro dos antigos vikings atravs
sas transformaes geraram novas relaes entre os senhores feudais da Rssia, chegavam a Constantinopla (Rota do Mar do Norte).
e seus servos. Os senhores passaram a ter novas necessidades, prin- No cruzamento das principais rotas comerciais surgiram fei-
cipalmente produtos de luxo. Para consumi-los, tinham que elevar ras, isto , pontos de encontro temporrio de comerciantes. Aos
suas rendas, aumentando as obrigaes dos servos. Essa presso poucos, com a dinamizao da vida mercantil, esses centros se
causou a fuga dos servos, em busca das cidades e de novas oportu- foram transformando em cidades.
nidades de vida. Entretanto, as cidades no eram suficientes para As vilas e as cidades se multiplicaram to rapidamente, que,
absorv-los. Surgiu ento uma camada de marginalizados que bus- em algumas regies, por volta do sculo XIV, metade da popula-
cava alternativas para a sobrevivncia. O banditismo foi uma delas. o tinha sido deslocada das atividades agrcolas para as comer-
Assim, devemos entender a desintegrao do sistema feudal a ciais e artesanais. As cidades se tornaram to populosas, que, mui-
partir da sua inadequao ao crescimento populacional e suas con- tas delas, conseguiram sua autonomia em relao aos senhores
sequncias. Uma das solues para essa crise foram as Cruzadas. feudais, por meio das Cartas de Franquia. A produo artesanal
dentro das cidades foi organizada em torno das corporaes de
As Cruzadas (1096-1270) ofcio(associaes de artesos) ou guildas (associaes de comer-
ciantes). Estas regulamentavam a produo e o comrcio, fixando
a quantidade, a qualidade e o preo.

A Peste negra (1347-1350)


A partir do incio do sculo XIV, uma profunda crise anun-
ciou o final da poca medieval. Fome, pestes, guerras e rebelies
de servos atingiram a essncia do sistema feudal.
A chamada Peste Negra foi, ao que parece, um surto de peste bu-
bnica, transmitida por ratos. Acredita-se que tenha sido trazida do
Oriente, por um navio veneziano e dali tenha se propagado por toda
a Europa. De 1347 a 1350, um tero da populao europeia desapa-
receu vitimada pela epidemia. Essa queda demogrfica variou de re-
gio para regio, sendo de um tero a sua mdia. Segundo Boccacio,
escritor do sculo XIV, na cidade de Florena, entre o ms de maro
e o de julho... mais de 100 mil criaturas morreram dentro das mu-
ralhas, onde, antes disso, no se supunha que tanta gente houvesse.
Impotentes diante da desconhecida doena, os mdicos da
As Cruzadas foram expedies militares dos catlicos contra Faculdade de Medicina de Paris emitiram o seguinte parecer:
os inimigos de Deus, fossem eles muulmanos, hereges, pagos Digamos antes de mais nada, que a causa longnqua e primeira
ou cristos ortodoxos. Seus integrantes usavam uma cruz como desta peste foi e ainda alguma constelao celeste a qual... cau-
smbolo. Essas expedies resultaram de uma convergncia de sa real da corrupo mortfera do ar que nos rodeia, pressagia a
interesses da populao europeia. Por um lado, a Igreja Cat- mortalidade e a fome....
lica buscava se afirmar como de fato uma igreja universal, posto
tivesse sua posio desestabilizada pelo Cisma do Oriente (1054 A Guerra dos Cem Anos (1337-1453)
dC) que deu origem Igreja Ortodoxa do Oriente. Alm disso, Muitas catstrofes atingiram a Europa Centro-Ocidental no
o excedente populacional tambm influenciou a quantidade de sculo XIV. Provavelmente a de maiores consequncias polticas
nobres que aumentou sem ter acesso posse da terra que era tenha sido a Guerra dos Cem Anos. Envolvendo a Frana e a
obtido apenas atravs da herana ou da guerra. Visando a manter Inglaterra, terminou por diminuir o poder dos senhores feudais
uma certa estabilidade em seus domnios a Igreja atraiu nobres dos dois pases, aumentando a autoridade real.
em troca da posse de feudos na Terra Santa. O primeiro importante fator que originou a Guerra dos Cem
Todos esses interesses foram justificados e legitimados pela Igre- Anos foi a disputa anglo-francesa pela posse da Flandres. Esta
ja Catlica, tendo como causa imediata o bloqueio peregrinao regio (onde hoje se estendem as fronteiras da Blgica e Holanda)
ao Santo Sepulcro (tmulo de Cristo, em Jerusalm). Este havia era possuidora da mais numerosa indstria de tecidos da Europa.
sido tomado pelos turcos seldjcidas, que impediram as peregri- Utilizava como matria-prima l importada da Inglaterra. A ex-
naes s Terras Santas, costume bastante difundido e valorizado. portao de l para a Flandres constitua a atividade econmica
O resultado das Cruzadas foi o mais desastroso possvel, por- fundamental de uma razovel parcela dos senhores feudais ingle-
que, no fim das contas, os muulmanos foram vencedores de sete ses. Ameaadoramente, os reis franceses, posteriores a Filipe, o
das oito cruzadas e acabaram por confirmar o seu poderio no Belo, implementaram uma poltica que visava a controlar a re-
mundo oriental. No entanto, do ponto de vista econmico, as gio flamenga. Se a Frana efetivasse esse controle, certamente
cruzadas trouxeram uma grande contribuio para a reconstitui- obrigaria a Flandres a comprar l francesa. Isso inviabilizaria as
o econmica da Europa. exportaes inglesas dessa matria-prima.

UECEVEST 123

Apostilas UECEVEST mod3.indb 123 06/02/2011 09:58:58


H I S T RIA GERAL I

Outro fator importante para a ecloso do conflito foi a disputa definitivamente expulsos. Depois de 116 anos, terminava a guer-
por territrios na Frana. Os reis da Inglaterra eram senhores de ra. O poder do rei francs foi fortalecido, pois as fronteiras foram
grandes feudos na Frana, sendo, consequentemente, vassalos do consolidadas. Criou-se um forte sentimento nacional e muitos
rei francs. Essa situao teve sua origem no sculo XI, quando nobres feudais haviam morrido. Apoiados pela burguesia, os reis
um senhor feudal francs, Guilherme, o Conquistador, realizou a da Dinastia Valois centralizaram ainda mais o poder poltico,
conquista da Inglaterra e se tornou seu rei. No sculo XIV, o rei da consolidando a monarquia. Para as estruturas feudais, a guerra
Frana no podia ter em seu territrio um vassalo to poderoso. teve consequncias irreversveis. A nobreza francesa emergiu da
Por seu lado, o monarca ingls no podia suportar a humilhao guerra numericamente dizimada, economicamente arruinada e
de ser vassalo de outro rei. Alm disso, os principais feudos ingle- politicamente enfraquecida.
ses na Frana (Guiana e Aquitnia) tinham grande importncia
econmica. Dessas regies saam, a cada ano, a produo de vinho As rebelies dos servos
que lotava entre 800 a 1.400 navios ingleses. A Inglaterra recebia A crise agrcola, a estagnao do comrcio, a consequente
dali cerca de 90% do vinho que o pas consumia. fome, peste e as guerras foram fatores que geraram uma abrupta
O pretexto ou fator que serviu de justificativa para a declarao diminuio da populao europeia. A Inglaterra teve reduzida sua
de guerra foi o problema da sucesso da Coroa francesa. Com a populao de 3.750.000 habitantes, em 1348, para 2.250.000,
morte do rei Carlos IV, em 1328, sem deixar descendentes, extin- em 1374, e 2.100.000, em 1430. Grosso modo, essas propores
guiu-se a Dinastia dos Capetos. Apresentaram-se, ento, dois pre- da queda demogrfica ocorreram em toda Europa Centro-Oci-
tensos sucessores ao trono: Filipe de Valois, senhor feudal francs, dental. Isso significava menos servos trabalhando em cada feudo.
sobrinho de Filipe, o Belo, e Eduardo III, rei da Inglaterra e neto Nessas condies, a nobreza feudal, j acostumada a novos hbi-
de Filipe, o Belo, por parte de me. Uma assembleia dos grandes tos de consumo criados pelo comrcio, intensificou suas exign-
senhores feudais franceses, reunida em Paris, escolheu Filipe de Va- cias sobre os servos. Para conservar seu padro de vida e financiar
lois para o trono. Assim, ficavam rejeitadas as possibilidades de um as guerras, passaram a impor uma verdadeira super explorao
rei ingls reinar sobre a Frana. A assembleia baseou sua deciso aos trabalhadores do campo. Estes reagiram atravs de revoltas ou
da Lei Slica, pela qual, desde o incio da Idade Mdia, as mulheres fugas para as cidades.
no podiam ocupar ou transmitir o trono francs. Os camponeses da Frana eram chamados, pejorativamente,
de Jacques Bonhomme. Essa expresso era utilizada pelos no-
bres, caracterizando a situao do homem do campo, explorado,
subnutrido e sempre submisso.
No sculo XIV, os numerosos levantes camponeses ficaram
conhecidos como jacqueries. A mais importante delas ocorreu em
1358. Conforme descreveu um cronista da poca, algumas pesso-
as das populaes rurais, sem chefe, juntaram-se em Beauvais e no
foram mais de 100 os primeiros; e disseram que todos os nobres do
reino da Frana... atraioavam o reino, e seria grande bem que todos
os destrussem... Logo se juntaram e foram... armados de paus ferra-
dos e facas, casa dum cavaleiro que morava perto. E a destruram a
casa... e mataram o cavaleiro, a dama e os filhos, pequenos e grandes
Joana DArc e puseram fogo casa... Assim fizeram em vrios castelos e paos.
E multiplicaram-se tanto que foram bem seis mil; e onde quer que
A Guerra dos Cem Anos dividiu-se em duas fases: a primeira chegavam seu nmero crescia... E tinham feito um rei entre eles...
favorvel Inglaterra e a segunda que terminaria com a vitria e esse rei chamava-se Jacques Bonhomme. Essa jacquerie durou
da Frana. Todo o conflito se desenvolveu em territrio francs. apenas cinco semanas, de 21 de maio a 24 de junho de 1358. Na
A difcil situao e o sentimento nacionalista eram propcios ao terceira semana de junho, recebendo auxlio da Coroa, os nobres
aparecimento de heris nacionais. Entre eles, a figura mais c- passaram contra-ofensiva. Travaram duas grandes batalhas com
lebre foi Joana DArc, a Santa Joana, que possua duas virtudes os camponeses, uma em Meaux e outra em Beauvais. Sem armas
fundamentais para a sociedade feudal: era considerada santa e apropriadas, os camponeses foram massacrados. Guilherme Cal-
revelou-se excelente guerreira. le, campons que assumiu a liderana do levante, foi chamado a
Nascida no meio campons, Joana DArc, ainda adolescente, Paris para negociar a paz com o rei. L acabou sendo preso, tor-
afirmava ter recebido de Deus a misso de libertar a Frana dos turado e morto. Todas as aldeias camponesas entre os rios Oise,
ingleses. Conseguindo engajar-se no exrcito real, liderou in- Sena e Marne foram destrudas e seus habitantes liquidados.
meros combates que resultaram em vitrias francesas. Com isso, Nas dcadas seguintes, ocorreram outras inmeras jacqueries
tornou-se importante figura junto ao rei e seu crescente prest- de menor porte. Tiveram todas o mesmo fim. Embora derrota-
gio fortalecia o monarca. Em 1429, conseguiu apoio da maioria dos, os levantes camponeses foraram a progressiva abolio de
dos senhores feudais franceses ao rei Carlos VII. Convenceu-o vrias obrigaes feudais.
a ser coroado em Reims, de acordo com as antigas tradies da
monarquia francesa. Dessa forma, fortaleceram-se ainda mais a
monarquia e a luta nacional dos franceses. E X E R C C I O
Aprisionada em 1430 pelos borguinhes, franceses aliados
dos ingleses, foi julgada por um tribunal eclesistico. Acusada de 01. (UECE 2010/1) As Ordens Religiosas Militares formadas
ter parte com o demnio, e ser uma bruxa, recebeu a condenao por monges cavaleiros nasceram durante as Cruzadas. Acrescen-
morte na fogueira. A sentena foi cumprida na cidade de Rou- taram ao ideal cavalheiresco de proteo dos oprimidos os votos
en, em 20 de maro de 1432. Mais tarde, a Igreja decidiu pela monsticos de pobreza, castidade e obedincia. Sobre as Ordens
sua canonizao. Religiosas Militares correto afirmar que
Morta, Joana DArc transformou-se em smbolo do patrio- a) atuaram em um curto perodo, protegendo Jerusalm contra
tismo popular francs. Imbudos de novo esprito, os franceses os ataques turcos e foram extintas por falta de voluntrios e
continuaram a conquistar vitrias. Em 1453, os ingleses foram recursos financeiros.

124 UECEVEST

Apostilas UECEVEST mod3.indb 124 06/02/2011 09:58:59


HISTRI A G E R A L I

b) foram ricas e influentes e constituram-se em quatro ordens: e) Uma relao econmica pois a Guerra Santa foi
os Hospitalrios, os Cavaleiros Teutnicos, os Cavaleiros de sistematicamente financiada por grupos judeus dispostos a
So Joo e os Templrios. contribuir com a expanso do Cristianismo.
c) constituram-se em criaes fictcias e literrias,
configurando a formao de um imaginrio medieval 05. (UFC 2006) No ano de 1348, a peste negra devastou
conhecido como Cavalarias. a Europa e ceifou um tero de sua populao. Analise as
d) impediram o desenvolvimento do comrcio ao afirmaes abaixo sobre essa catstrofe.
desenvolverem um sistema de bloqueio das estradas e vias. I. Veio da sia pela rota da seda, em virtude do
comrcio estabelecido por negociantes genoveses e
02. (UECE 2010/1) Francisco Bernardone nasceu (1181 ou venezianos.
1182) em Assis na Itlia e ficou conhecido como Francisco de II. Ocorreu num sculo de retrao da economia europeia,
Assis. Filho de comerciantes mudou o conceito de santidade e marcado por vrias revoltas camponesas, e contribuiu
devoo e, do mesmo modo, a atitude da Igreja e dos leigos em para o enfraquecimento do feudalismo.
relao ao sagrado, na virada do sculo XII para o sculo XIII. III. Atingiu indiscriminadamente as vrias categorias sociais,
Sobre o perodo em que Francisco de Assis viveu, e correto afir- tanto das cidades como das reas rurais, como ocorria
mar que se trata com uma outra doena comum na poca, a lepra.
a) da Idade Moderna, fase de ascenso do Capitalismo e
reorganizao da vida urbana na Europa. Com base nas trs assertivas, correto afirmar que somente:
b) da Idade Media que, sob o sistema feudal, foi uma fase de a) I correta d) I e II so corretas
renovao da sociedade e suas tradies. b) II correta e) II e III so corretas
c) da virada do sculo XII para o sculo XIII que indica o fim c) III correta
do mundo antigo com a vitoria do Cristianismo sobre o
06. (IFCE 2008/1) Entre os principais filsofos medievais, consi-
paganismo.
derados os mais renomados telogos catlicos com obras de gran-
d) do Renascimento com o retorno aos valores e costumes
de influncias na Idade Mdia, destacam-se:
clssicos, porm com a perspectiva crist.
a) Santo Incio de Loyola e So Francisco de Assis
b) Santo Agostinho e Santo Anselmo
03. (UECE 2009/1) Numa sociedade em que toda conscincia
uma conscincia religiosa, os obstculos so antes de tudo ou c) Santo Toms de Aquino e So Francisco de Assis
finalmente religiosos. A esperana de escapar ao Inferno, graas d) Santo Toms de Aquino e Santo Agostinho
ao Purgatrio, permite ao usurrio fazer avanar a economia e a e) So Pedro e So Paulo
sociedade do sculo XIII em direo ao capitalismo. (LE GOFF,
J. A bolsa e a vida a usura na Idade Mdia. Trad. Rogrio Sil- 07. (UECE 2007/2) O fenmeno das cruzadas se estende ao longo
veira Muoio. So Paulo: Brasiliense, 2004. p. 90). Sobre a usura, de dois sculos de histria europeia. Desde a conquista de Jerusa-
assinale o correto. lm pelos turcos (sc. XI) at a queda de So Joo DAcre, ltima
a) Foi uma prtica utilizada e incentivada pela Igreja Catlica fortaleza crist (sc.XIII) dominada pelos turcos muulmanos. Das
no decorrer da Idade Mdia. inmeras causas da difuso do esprito da cruzada, destaca-se:
b) Foi considerada um pecado grave para a doutrina e a a) A necessidade dos abastados nobres europeus de encontrar
mentalidade eclesistica da Idade Mdia. aventura, glria e de obter o principal prmio: o casamento
c) Foi utilizada como inspirao para o modelo econmico com uma jovem nobre, bela e rica.
predominante na Idade Mdia. b) O cumprimento de uma obrigao religiosa: combater
d) Foi o impulso econmico para o incentivo das grandes o infiel muulmano era uma ao santa e representava a
navegaes ps-medievais. possibilidade de salvao eterna.
c) A presena de um profundo zelo religioso, caracterstico
04. (UFC 2004) (...) Por volta de 1010 comearam a circular da medievalidade, acompanhada de motivos econmicos e
rumores no Ocidente de que, sob a instigao dos judeus, os sar- sociais da Europa.
racenos tinham causado a destruio do Santo Sepulcro e deca- d) A predominncia de interesses econmicos das cidades
pitado o patriarca de Jerusalm (...) Ento, na esteira da Cruzada comerciais que desejavam expandir suas atividades
proclamada pelo Papa Urbano II, no Concilio de Clermont em mercantis.
1095, foi engendrada uma atmosfera de histeria religiosa ... (RI-
08. (UFRN 2009) Em 1095, atendendo ao apelo do papa Ur-
CHARD, Jefrei Richards. Sexo, desvio e danao: as minorias da
bano II para que iniciassem uma guerra contra os muulmanos,
Idade Mdia. Rio de Janeiro: Jorge Zahar, 1993, p. 97)
os nobres cristos, motivados por ideais religiosos e econmicos,
A partir do texto e considerando os objetivos da Cruzadas, assi-
nale a alternativa que corresponde relao entre a Igreja Cat- organizaram as Cruzadas. Considerando-se o conjunto dessas ex-
lica e os judeus na Idade Mdia. pedies, que se prolongaram at 1270, pode-se destacar como
a) Uma colaborao recproca, pois os judeus eram uma de suas consequncias:
considerados fiis observadores da f e dos ritos cristos. a) o enfraquecimento do comrcio italiano no Mar
b) Uma ao conjunta em defesa da Terra Santa, uma vez Mediterrneo, em razo da insegurana e dos perigos
gerados pelos conflitos militares.
que os judeus participaram como bravos combatentes nas
b) o fortalecimento da autoridade dos senhores feudais, cujas
Primeiras Cruzadas.
c) Uma aproximao entre judeus e cristos em virtude finanas foram consolidadas com a explorao dos territrios
da prtica da usura, defendida arduamente pela Igreja do Oriente.
Medieval. c) a difuso e a assimilao da cultura germnica pelo imprio
d) Uma grande hostilidade, pois a Igreja, no Sculo XI, buscou bizantino, alterando significativamente o modo de viver dos
cristianizar o mundo e muitas comunidades judaicas, sob povos orientais.
a acusao de adoradores do Diabo, foram perseguidas e d) a ampliao do universo cultural dos povos europeus,
exterminadas. possibilitada pelo contato com a rica cultura dos povos
orientais.

UECEVEST 125

Apostilas UECEVEST mod3.indb 125 06/02/2011 09:58:59


H I S T RIA GERAL I

G A B A R I T O
09. (ENEM 2006) Os cruzados avanavam em silencio, encon- 01. b 02. d 03. b 04. d 05. d
trando por todas as partes ossadas humanas, trapos e bandeiras. 06. d 07. c 08. d 09. d 10. a
No meio desse quadro sinistro, no puderam ver, sem estremecer
de dor, o acampamento onde Gauthier havia deixado as mulheres
e crianas. L, os cristos tinham sido surpreendidos pelos mu-
ulmanos, mesmo no momento em que os sacerdotes celebravam
o sacrifcio da Missa. As mulheres, as crianas, os velhos, todos os REFERnCIAS BIBlIOGRFICA
que a fraqueza ou a doena conservava sob as tendas, perseguidos
ate os altares, tinham sido levados para a escravido ou imola- AQUINO, Rubim Santos Leo de Aquino; FRANCO, Denize
dos por um inimigo cruel. A multido dos cristos, massacrada de Azevedo; LOPES, Oscar Guilherme P.C. Histria das Socie-
naquele lugar, tinha ficado sem sepultura. (J. F. Michaud. Hist- dades. Das Comunidades primitivas s sociedades modernas.
ria das cruzadas. So Paulo: Editora das Amricas, 1956 com 36 Ed. Rio de Janeiro: Editora ao Livro Tcnico, 1997.
adaptaes). Foi, de fato, na sexta-feira 22 do tempo de Chaaban,
do ano de 492 da Hegira, que os franj* se apossaram da Cidade ARRUDA, Jos Jobson de A. Toda a Histria: Histria Geral e
Santa, apos um sitio de 40 dias. Os exilados ainda tremem cada Histria do Brasil. So Paulo: tica, 2007.
vez que falam nisso, seu olhar se esfria como se eles ainda tivessem
diante dos olhos aqueles guerreiros louros, protegidos de arma- AZEVEDO, Gislaine; SERIACOPI, Reinaldo. Histria Srie
duras, que espelham pelas ruas o sabre cortante, desembainha- Brasil. So Paulo: Editora tica, 2005.
do, degolando homens, mulheres e crianas, pilhando as casas,
saqueando as mesquitas. *franj = cruzados. (Amin Maalouf. As BRAICK, Patrcia R.; MOTA, Myriam B. Histria das cavernas
Cruzadas vistas pelos rabes. 2. ed. So Paulo: Brasiliense, 1989 ao terceiro milnio. 3 Ed., So Paulo: Moderna, 2007
com adaptaes). Avalie as seguintes afirmaes a respeito dos
textos acima, que tratam das Cruzadas. BURNS, Edward McNall; LERNER, Robert; MEACHAM,
I. Os textos referem-se ao mesmo assunto as Cruzadas, Standish. Histria da Civilizao Ocidental. Do Homem das
ocorridas no perodo medieval , mas apresentam vises Cavernas s naves espaciais. Vol . I 40 ed. So Paulo: Globo,
distintas sobre a realidade dos conflitos religiosos desse 2000
perodo histrico.
II. Ambos os textos narram partes de conflitos ocorridos entre CAMPOS, Flvio de; MIRANDA, Renan Garcia. A Escrita da
cristos e muulmanos durante a Idade Mdia e revelam Histria. So Paulo:Escala Educacional, 2005.
como a violncia contra mulheres e crianas era prtica
comum entre adversrios. COTRIM, Gilberto. Histria Global Brasil e Geral. 7 Ed.,
III. Ambos narram conflitos ocorridos durante as Cruzadas So Paulo: Saraiva, 2003.
medievais e revelam como as disputas dessa poca, apesar de
ter havido alguns confrontos militares, foram resolvidas com ORDOES, Marlene; QUEVEDO, Jlio. Horizonte da His-
base na ideia do respeito e da tolerncia cultural e religiosa. tria. Histria para o ensino mdio. So Paulo: IBEP, 2005.
correto apenas o que se afirma em SCHMIDT, Mario. Nova Histria Crtica. So Paulo: Nova
a) I. d) I e II. Gerao, 2005.
b) II. e) II e III.
c) III. VICENTINO, Claudio; DORIGO, Gianpaolo. Histria para o
ensino mdio. So Paulo: Scipione, 2006.l
10. (ENEM 2008) A Peste Negra dizimou boa parte da popu-
lao europeia, com efeitos sobre o crescimento das cidades. O
conhecimento mdico da poca no foi suficiente para conter a
epidemia. Na cidade de Siena, Agnolo di Tura escreveu: As pes-
soas morriam s centenas, de dia e de noite, e todas eram jogadas
em fossas cobertas com terra e, assim que essas fossas ficavam
cheias, cavavam-se mais. E eu enterrei meus cinco filhos com mi-
nhas prprias mos (...) E morreram tantos que todos achavam
que era o fim do mundo. (Agnolo di Tura. The Plague in Siena:
An Italian Chronicle. In: William M. Bowsky. The Black De-
ath: a turning point in history? New York: HRW, 1971 com
adaptaes).O testemunho de Agnolo di Tura, um sobrevivente
da Peste Negra, que assolou a Europa durante parte do sculo
XIV, sugere que
a) o flagelo da Peste Negra foi associado ao fim dos tempos.
b) a Igreja buscou conter o medo da morte, disseminando o
saber mdico.
c) a impresso causada pelo nmero de mortos no foi to
forte, porque as vtimas eram poucas e identificveis.
d) houve substancial queda demogrfica na Europa no perodo
anterior Peste.
e) o drama vivido pelos sobreviventes era causado pelo fato de
os cadveres no serem enterrados.

126 UECEVEST

Apostilas UECEVEST mod3.indb 126 06/02/2011 09:59:00


P R - V E S T I B U l A R

HISTRIA GERAL II

Apostilas UECEVEST mod3.indb 127 06/02/2011 09:59:04


Caro(a) Aluno(a),

Para facilitar o acompanhamento de tais contedos, abaixo esto indicadas as nomenclaturas utilizadas
pela UECE e pelo ENEM:

UECE ENEM
Movimento Operrio Doutrinas socialistas no sculo XIX: Diferentes formas de organizao
marxismo e anarco-ssindicalismo da produo: escravismo antigo,
feudalismo, capitalismo, socialismo e
suas diferentes experincias.
Liberalismo O desenvolvimento do pensamento
liberal na sociedade capitalista e seus
crticos nos sculos XIX e XX.
Imperialismo Imperialismo europeu e partilha afro- Geopoltica e conflitos entre os
asitica sculos XIX e XX: Imperialismo,
a ocupao da sia e da frica, as
Guerras Mundiais e a Guerra Fria.
Primeira Guerra Primeira Guerra Mundial Geopoltica e conflitos entre os
Mundial sculos XIX e XX: Imperialismo,
a ocupao da sia e da frica, as
Guerras Mundiais e a Guerra Fria.
Revoluo Russa Revoluo Russa de 1917 e formao A atuao dos grupos sociais e os
da URSS grandes processos revolucionrios do
sculo XX: Revoluo Bolchevique

Apostilas UECEVEST mod3.indb 128 06/02/2011 09:59:04


HISTRIA G E R A L I I

O mOVImEnTO OPERRIO E AS sentantes da classe trabalhadora no Parlamento Ingls. Essa ma-


DOUTRIAnS SOCIAIS (UECE/EnEm) nifestao operria ganhou milhares de adeptos em poucos meses
e deu origem ao chamado movimento cartista. O governo in-
gls negou-se a atender as reivindicaes do movimento cartista.
Alm disso, deu ordens para que a polcia reprimisse duramente
as greves e os tumultos que explodiram logo aps a recusa.

Doutrinas econmicas e sociais do sculo xix


Inmeros pensadores europeus procuraram respostas para
os graves problemas sociais decorrentes do rpido processo de
industrializao e urbanizao que revolucionou a Inglaterra a
partir de 1760. Parte desses pensadores, identificados com os
interesses da burguesia, elaborou doutrinas que justificavam a
explorao do trabalho humano e a pobreza existentes na nova
sociedade industrial capitalista. Outros, olhando a realidade do
Fonte: SCOTTI, A. Il Quarto Stato di Giuseppe Pellizza da Volpedo. Milano: TEA ponto de vista dos trabalhadores, formularam teorias que critica-
Arte, 1998. vam frontalmente o capitalismo e propunham, inclusive, novas
formas de organizao da sociedade.
A Revoluo Industrial acarretou grandes mudanas no cen-
rio poltico, econmico e social da poca. O capitalismo (sistema liberalismo econmico
de produo que se baseia no trabalho assalariado) foi consoli- Entre os pensadores identificados com os interesses das cama-
dado e, com isso, formaram-se dois grupos sociais antagnicos: das burguesas, que merecem especial destaque esto os defensores
a burguesia industrial e o operariado. Ao mesmo tempo, como do liberalismo econmico: Adam Smith (o pai do liberalismo,
resultado do processo de industrializao, verificou-se um cresci- j apresentado no capitulo sobre iluminismo); Thomas Malthus
mento extraordinrio da populao mundial. De 600 milhes de e David Ricardo. Tais pensadores so chamados de liberais por-
habitantes, em 1750, a populao aumentou para 1 bilho e 200 que defendiam basicamente o livre comrcio entre pases, a no
mil, em 1850. A Inglaterra, pioneira da Revoluo Industrial, interveno do Estado na economia e a livre concorrncia que,
quase triplicou a populao nesse perodo. Em consequncia des- na opinio deles, serviria para eliminar os incompetentes e forar
ta industrializao, ocorreu ainda um crescimento vertiginoso das a queda de preos.
cidades, que passaram a abrigar um nmero cada vez maior de Thomas Malthus Escreveu O Ensaio sobre o Principio da Po-
fbricas e operrios. pulao. Nessa obra, ele diz que a populao sempre tende a
Todavia, nestas cidades, a vida dos operrios era muito di- crescer muito mais do que a produo de alimentos.E a conse-
ferente da que levavam os industriais. Enquanto a burguesia in- quncia inevitvel dessa desapropriao a pobreza crescente.
dustrial investia, consumia e lucrava cada vez mais, colhendo os Assim, a nica forma de evitar catstrofes seria negar a assis-
melhores frutos dessa modernizao, o operariado era submetido tncia social aos pobres e orient-los no sentido de controlar
a longas jornadas de trabalho, salrios baixos, disciplina e hor- a natalidade.
rios rigorosssimos. Alm disso, as famlias operrias viviam, em David Ricardo Autor de Princpios de Economia Poltica e
sua maioria, em cmodos minsculos, midos e mal ventilados, Tributao, tambm justificou a explorao do trabalho, afir-
o que contribua para que fossem as principais vtimas da clera e mando que os salrios sempre tendem a ser equivalentes ao
do tifo, doenas que castigavam a Europa com frequncia naque- mnimo necessrio sobrevivncia do trabalhador. Tomando
la poca. Essa realidade, tpica das maiores cidades da Inglaterra, por base esse pensamento, os salrios baixos so consequncia
na primeira metade do sculo XIX, reproduziu-se em todos os de uma lei natural a lei da oferta e da procura e, portanto,
centros urbanos dos pases para os quais a Revoluo Industrial no h como se opor a essa situao. Tais ideias, como se v,
foi-se propagando. atendiam claramente aos interesses da burguesia industrial.

O movimento operrio Socialismo


A explorao, a rotina diria massacrante e as pssimas con- Entre os pensadores que formularam duras crticas nova or-
dies de vida a que estavam submetidos os operrios, estimu- dem industrial da sociedade capitalista, destacam-se os adeptos
laram alguns deles a entregarem-se ao crime, prostituio e ao do socialismo. Alm de ser uma doutrina social, o socialismo
alcoolismo. S na cidade industrial inglesa de Manchester havia, tambm um sistema socioeconmico no qual, a produo e a dis-
em 1850, cerca de 1200 tavernas. Muitos operrios, contudo, tribuio dos produtos planejada com vistas a atender as neces-
decidiram reagir e se unirem para lutar por condies melhores. sidade bsicas da populao. Essa doutrina propunha a realizao
Uma de suas primeiras reaes foi o ataque e a destruio das de profundas mudanas sociais a fim de eliminar a pobreza e me-
mquinas.Esse movimento espontneo do operariado ocorreu na lhorar a pobreza e melhorar a vida dos trabalhadores. Os socia-
segunda dcada do sculo XIX e ficou conhecido como Ludis- listas quase sempre concordavam quanto aos problemas causados
mo, pois um de seus lderes chamava-se Ned Ludd. pelo capitalismo, mas discordavam quanto s solues para esses
Posteriormente, refletindo melhor sobre a situao em que problemas. Em vista disso, existiam no Sculo XIX, quatro im-
se encontravam, os operrios passaram a reagir com greves e portantes correntes no interior do socialismo: o socialismo ut-
passeatas organizadas por suas associaes, as trade-unions. Em- pico, o socialismo cientfico, o anarquismo e o socialismo cristo.
bora tenham sido reprimidas violentamente pelos governos, tais
associaes reuniram um nmero cada vez maior de operrios, Socialismo utpico
fortaleceram-se e evoluram com o nome de sindicatos. Os primeiros socialistas foram chamados de utpicos porque
Alm disso, o operariado passou a lutar tambm para ampliar a maioria de seus planos eram irrealizveis ou utpicos. Tais pen-
sua participao poltica.Com esse objetivo, em 1838, trabalha- sadores idealizaram solues criativas para os problemas sociais,
dores ingleses redigiram a Carta do Povo, um documento por mas no levaram em conta as inmeras dificuldades de coloc-
meio do qual exigiam o voto secreto e a participao de repre- las em prtica. Entre os socialistas utpicos, merecem destaque

UECEVEST 129

Apostilas UECEVEST mod3.indb 129 06/02/2011 09:59:04


H I S T RIA GERAL II

especial: condies de vida do operariado de seu tempo para elaborarem


Saint-Simon: socialista francs, era na verdade um liberal o que eles prprios chamaram socialismo cientifico. Considera-
avanado para a poca. Escrevendo no incio do sculo XIX vam a sua doutrina cientfica por terem chegado a ela atravs da
quando a industrializao dava seus primeiros passos e a ex- anlise crtica do desenvolvimento da sociedades humanas. Tra-
plorao capitalista era mais intensa , preocupou-se com a balhando em conjunto, Marx e Engels publicaram, em 1848, o
questo operria. Suas idealizaes visavam a solues atravs Manifesto Comunista, obra que contm a ideia central do socia-
de reformas dentro do prprio sistema capitalista. Saint-Simon lismo cientifico e que pode ser resumida na frase: O que move
dividia a sociedade em dois grupos: os ociosos e os trabalhado- a histria so as lutas de classes. Num outro trecho da mesmo
res. Falava em um governo dos trabalhadores, que organizaria obra, afirmavam: de todas as classes que ora enfrentam a burgue-
a economia para evitar a m distribuio das riquezas. Como sia, somente o proletariado uma classe verdadeiramente revolu-
trabalhadores, colocava os operrios, os banqueiros, os indus- cionria. Assim, a classe trabalhadora devia organizar-se, fazer a
triais e os comerciantes. Defensor da livre-iniciativa, para ele o revoluo, tomar o poder poltico das mos da burguesia, pr fim
lucro deveria ser mantido desde que os patres tivessem pre- ao capitalismo e implantar a ditadura do proletariado.
ocupaes sociais com seus empregados. Na verdade, era um
humanizador do capitalismo.
Charles Fourier: outro socialista utpico francs do incio do
sculo XIX, Fourier idealizava a criao de uma falange co-
munidade de at 2 mil homens , onde se trabalharia para um
fundo comum. A diviso das riquezas produzidas seria feita de
acordo com a quantidade e a qualidade do trabalho de cada in-
divduo. Cada falange possuiria um edifcio comum, o falans-
trio, que abrigaria todos os membros e onde seriam instalados
os bens coletivos da comunidade, como a cozinha, a biblioteca
e os servios mdicos.
Robert Owen: homem empreendedor, inteligente, criativo e
capaz, de empregado comercirio chegou a ser scio e diretor
de uma grande indstria inglesa. Solidrio e humanista, Owen
condoeu-se da situao dos operrios e passou a ser estudioso
da Revoluo Industrial e um crtico dos rumos que ela to-
mava. Entre 1820 e 1829, alterou o funcionamento de sua
fbrica, em New Lamarck, na Esccia. Essa indstria tornou-
se um modelo de legislao social: a jornada de trabalho foi
diminuda e os operrios dispunham de moradia e boas condi-
es sanitrias, alm de escolas para os filhos. Com todas essas
melhorias na condio de vida dos operrios e com a racio- Manifestao de operrios novaiorquinos
nalizao do trabalho, os lucros da empresa no cessaram de durante uma greve em 1913.
crescer. Satisfeito com o sucesso do empreendimento, Owen
radicalizou sua viso de mundo: passou a pregar o fim da pro- Para Marx e Engels, a ditadura do proletariado seria necess-
priedade privada e a criao de uma sociedade comunista, isto ria s at a eliminarem-se as desigualdades sociais. Para tanto, era
, igualitria. Tambm passou a criticar a religio, pelo menos preciso que os meios de produo (terras, fbricas e mquinas)
a dominante na Inglaterra, que, em sua hipocrisia, santificava o fossem socializados, isto , deixassem de ser propriedade de al-
trabalho dos pobres e omitia-se em relao misria dos traba- guns poucos e passassem pertencer a sociedade como um todo.
lhadores. Esse radicalismo valeu-lhe o dio dos capitalistas e do Eliminadas as desigualdades sociais, as lutas de classe cessariam.
governo ingls, que o baniram do pas. Foi com alguns compa- Com isso, o Estado deixaria de ser necessrio. A partir da, dizia
nheiros para Indiana, nos Estados Unidos, onde fundou a col- Marx, a ditadura do proletariado chegaria ao fim e o socialismo
nia Nova Harmonia, baseada em suas ideias de cooperativas de cederia lugar ao comunismo. A sociedade comunista seria uma
consumo e produo operrias, a qual durou apenas dois anos. sociedade sem classes, na qual no haveria explorao econmica
Louis Blanc: socialista francs, participou ativamente dos mo- nem outras formas de injustia social. Nessa sociedade haveria
vimentos liberais de 1848. As propostas de Blanc, apresentadas grande abundncia e cada um de seus membros trabalharia no
no livro A organizao do trabalho, defendiam a criao de as- que lhe desse prazer.
sociaes profissionais de trabalhadores de um mesmo ramo Outra obra importante para compreender o socialismo cien-
de produo as Oficinas Nacionais financiadas pelo Esta- tfico o livro de Marx, O Capital. Nele, o autor faz uma longa
do, para controlar a produo industrial. O lucro resultante anlise crtica do capitalismo, prevendo, inclusive, a ocorrncia
seria dividido em trs partes; a primeira pagaria o Estado: a de crises peridicas no interior desse sistema scio-econmico.
segunda seria distribuda entre os associados; e a terceira seria
usada para fins assistenciais. Essa ideia foi posta em prtica na Anarquismo
Frana, 1848, quando houve uma coligao de republicanos Na linguagem cotidiana, anarquia tem o sentido de bagun-
e socialistas para derrubar a monarquia. Com o dficit criado a, desordem. Mas, em sua lngua de origem, o grego, anarquia
pelas Oficinas Nacionais e o rompimento da coligao, o Esta- significa ausncia ou falta de necessidade de governo. Foi nes-
do as fechou, passou a perseguir os socialistas e anulou todas as se sentido que os tericos anarquistas definiram suas doutrinas.
reformas feitas em benefcio dos operrios. Usamos a palavra doutrinas, no plural, porque existem variantes
do anarquismo, que tambm se filiam s concepes socialistas.
Socialismo cientfico O inimigo mortal de todos os anarquistas o Estado, que,
Dois filsofos alemes, Karl Marx (1818-1883) e Frederich segundo os tericos anarquistas, acumulou muito poder duran-
Engels (1820-1895) apoiaram-se em estudos aprofundados de te a histria e tolhe a liberdade do homem. Embora houvesse
filosofia, histria, e economia, na observao e no registro das anarquistas cristos, que negavam a necessidade de violncia na

130 UECEVEST

Apostilas UECEVEST mod3.indb 130 06/02/2011 09:59:06


HISTRIA G E R A L I I

transformao histrica, a maior parte dos anarquistas apontava trabalho menos extenuante, descanso semanal e frias, alm da
Deus e o Estado como os maiores inimigos da liberdade humana. regulamentao do trabalho de mulheres e crianas.
Os anarquistas eram amantes extremados da liberdade das comu-
nidades e dos indivduos, que tm todo o direito de decidir suas
vidas, no delegando a ningum a autoridade de geri-las. E X E R C C I O
Michail Bakunin (1814-1876) O primeiro grande terico
e fundador do anarquismo. Era contrrio ao comunismo, que, 01. (UFC 2009/1) Leia o texto a seguir. A cada 1 de maio, lem-
segundo ele, concentrava no Estado todas as foras da socieda- bramos de Parsons, Spies e seus companheiros de patbulo. Mas
de, bem como toda a propriedade, em benefcio desse monstro poucos se lembram do nome de James Towle, que foi, em 1816,
que sempre tolheu a liberdade e escravizou os homens. Era fa- o ltimo destruidor de mquinas enforcado. Caiu pelo poo da
vorvel extino do Estado e da organizao da sociedade de forca gritando um hino luddita [sic] at que suas cordas vocais se
baixo para cima, defendendo a livre associao dos indivduos, fecharam num s n. (FERRER, Christian. Os destruidores de
declarando-se um coletivista. Para os seguidores de Bakunin, mquinas. In Libertrias, n. 4, dez/1998, So Paulo, p. 5.) Sobre
a revoluo seria feita pela ao espontnea e contnua das os destruidores de mquinas, de que trata o texto acima, assinale
massas, por meio de grandes organizaes de trabalhadores, a alternativa correta.
guiadas por anarquistas conscientes e convictos reunidos em a) Foram trabalhadores ingleses que combateram com aes
sociedades secretas. Esses anarquistas no seriam os lderes ou diretas a mecanizao dos teares durante a Revoluo
a vanguarda do povo, mas apenas ajudariam as massas em sua Industrial.
autolibertao. O Estado seria substitudo por federaes de b) Eram grupos de rebeldes irlandeses liderados pelos radicais
comunidades autnomas, que poderiam separar-se da federa- jacobinos insatisfeitos com a restaurao da monarquia dos
o e deveriam garantir a liberdade individual. Cada membro Bourbon na Frana.
das comunidades autnomas seria retribudo de acordo com o c) Eram integrantes das vanguardas das trade unions, os
trabalho que nelas houvesse aplicado. primeiros sindicatos de trabalhadores da Inglaterra, que
Joseph Proudhon (1804-1865) Foi um dos mais originais elaboraram a Carta do Povo.
pensadores socialistas, amante da liberdade, inimigo do buro- d) Foram trabalhadores anarquistas que morreram enforcados
cratismo estatal e das verdades absolutas da religio. Era um por terem lutado pela jornada de oito horas durante a greve
anarquista, de esprito independente, e procurava captar cien- geral de Haymarket Riot, em Chicago.
tificamente uma sociedade viva e mostrar seus conflitos a seus e) Eram grupos de indgenas do meio oeste dos EUA, entre
ouvintes e leitores. Era um pensador e no um homem de ao, eles os sioux, que atacavam os trens (cavalos de ao) que
dedicando-se mais aos estudos do que ao poltica. Mas, dividiam as manadas de bfalos dentro de seus territrios.
mesmo assim, teve grande influncia entre os socialistas fran-
ceses e europeus at o final do sculo XIX. Proudhon defendia 02. (UEL 2005) Analise a figura a seguir.
a diminuio sistemtica e gradual da ao governamental e
da religio sobre a sociedade. Avesso violncia, era contr-
rio luta de classes e seu reformismo consistia em pedir aos
governantes que liquidassem progressivamente as engrenagens
do Estado. Seu ideal econmico era uma repblica de peque-
nos produtores livres e iguais. Os trabalhadores comprariam os
meios de produo atravs de financiamento dos bancos de
trocas, sem juros e a fundo perdido, ou seja, sem pagamento
dos emprstimos.
Piotr Kropotkin (1842-1921) O grande mentor do co-
munismo anrquico acreditava na igualdade como base para
a liberdade. Desaconselhava o uso da violncia e achava que a
evoluo para a anarquia viria do amadurecimento da opinio
pblica, sem muita agitao e desordem. Para os adeptos do co-
munismo anrquico, o trabalho deveria ser agradvel e jamais
imposto. Todos contribuiriam para a coletividade com o tra- (Edvard Munch. Operrios na sada da fbrica, 1913-1915.)
balho que pudessem doar e retirariam dos armazns coletivos
tudo de que necessitassem, independentemente de sua contri- A partir da segunda metade do sculo XIX o capitalismo in-
buio. Acreditavam que, eliminados o Estado e a explorao dustrial desenvolveu-se de forma acelerada em diversos pases da
do homem pelo homem, todos trabalhariam voluntariamente, Europa ocidental. Esse desenvolvimento teve profunda influncia
retirando apenas o necessrio para uma existncia digna. na constituio das cincias humanas, enquanto campo de co-
nhecimento fundado em bases cientficas, mas tambm se refletiu
Socialismo cristo no campo das artes, com diversos pintores abordando a temti-
Essa doutrina ganhou impulso e consistncia em 1891 com ca do trabalho e das condies de vida das classes trabalhadoras.
a publicao da Encclica Rerum Novarum, do Papa Leo XIII. Dentre esses artistas encontra-se Edvard Munch (1863-1944),
Em tal documento, o Sumo Pontfice colocou-se a favor de uma que procurou traduzir os efeitos desse cotidiano sobre os traba-
redistribuio de terras (reforma agrria) e de associaes ope- lhadores no quadro Operrios na sada da fbrica. Com base na
rrias autnticas. Afirmou tambm que as classes no so por imagem e nos conhecimentos sobre o tema, assinale a alternativa
natureza hostis entre si e que deveria haver colaborao entre que apresenta a interpretao que remete condio geral que
patres e empregados. No entanto, o socialismo cristo alm de estavam submetidos os operrios fabris na transio do sculo
atacar a explorao capitalista quanto o marxismo e prope a ur- XIX para o sculo XX, na Europa.
gente humanizao do capitalismo. Leo XIII defendeu, ainda, a) Meu corao encontrava sua felicidade em meu trabalho;
a necessidade de o Estado impedir a explorao dos trabalhador, o fruto que dele eu retirava. (Antigo Testamento,
promulgando leis que assegurassem salrios dignos, jornada de Eclesiastes)

UECEVEST 131

Apostilas UECEVEST mod3.indb 131 06/02/2011 09:59:07


H I S T RIA GERAL II

b) O trabalho afasta de ns trs grandes males: a Qual das afirmativas abaixo corresponde s condies sociais da-
contrariedade, o vcio e a necessidade. (Voltaire, Candido) quele perodo?
c) necessrio trabalhar [...], uma vez que, tudo bem a) A rgida estratificao social impedia que os camponeses
verificado, trabalhar menos fastidioso do que se divertir. procurassem trabalho fora dos limites feudais.
(Charles Baudelaire, Jornais ntimos) b) A estagnao do setor econmico-produtivo, centralizado
d) Escravo do trabalho, no somente em ns homens/ Que num mundo agrrio incapaz de atender s necessidades
verdadeiros filhos do sofrimento e de misrias somos. humanas de subsistncia.
(Ronsart, Hino Morte) c) Leis trabalhistas que reconheciam os direitos dos homens,
e) Nenhum homem, segundo a ordem da Natureza, nasceu mulheres e crianas.
para o trabalho infrutfero/ Nenhum [homem] para a d) As pssimas condies de vida dos mais pobres, com longas
felicidade sem trabalho. (Mirabeau, Os Economistas) jornadas de trabalho e precrias condies de habitao.
e) A expanso dos governos democrticos, abertos participao
03. (UFMG 2006) Em 1891, o Papa Leo XIII editou um do- popular e incluso dos mais pobres na poltica.
cumento, a encclica Rerum Novarum, que deixou marcas pro-
fundas na Igreja Catlica. A importncia desse documento 06. (UFC 2002) A respeito do anarquismo, correto afirmar que:
transcende os muros da Igreja, haja vista que ele redefiniu o pen- a) Como doutrina, defendia a necessidade de eliminar qualquer
samento catlico e o modo como essa Instituio se relacionava forma de interveno estatal.
com as sociedades em que atuava. Considerando-se a influncia b) Seus tericos defendiam a interveno do Estado na
da Rerum Novarum, CORRETO afirmar que essa encclica economia com o apoio do operariado.
a) significou uma condenao vigorosa da guerra e c) Condenava a violncia como meio de ao, angariando
do colonialismo, pela manifestao do pacifismo e do assim o apoio da Igreja Catlica.
humanismo inerentes aos valores cristos. d) A sua difuso representou a primeira ruptura surgida no
b) deu origem ao pensamento social catlico, a partir do partido comunista da Rssia.
impacto da expanso do capitalismo e do crescimento do e) O movimento restringiu-se aos pases da Amrica do Sul.
iderio socialista.
c) transformou a Igreja em aliada do movimento fascista, abrindo 07. (URCA 2005/2) Com o Manifesto Comunista, publicado
caminho para a Concordata entre o Papa e o Estado italiano. em 1848, Karl Marx e Friedrich Engels tratam dos principais
d) representou uma tomada de posio do Vaticano contra a postulados do chamado socialismo cientifico, depois sistema-
religio muulmana, que crescia em ritmo acelerado e tizadas na obra O capital. Essas obras propuseram uma an-
ameaava a posio hegemnica do catolicismo. lise da condio e luta do operariado, procurando compreender
a realidade e transform-la. Dentre os princpios que marcam o
04. (IFCE 2007/1) Proletrios de todos os pases, uni-vos.. socialismo cientfico destacam-se
Esse trecho, do Manifesto Comunista de Marx e Engels, foi es- a) Interpretao socioeconmica da Histria, o conceito de luta
crito inspirado num contexto histrico marcado: de classes, de mais-valia e a revoluo socialista;
a) pelos conflitos entre trabalhadores e patres nos pases b) A criao de cooperativas e de crdito gratuito aos operrios,
capitalistas que comearam a partir da Revoluo Russa a luta de classes e o conceito de transformao pela reforma
b) pelas contradies polticas, econmicas e sociais decorrentes jurdica;
da Revoluo Industrial c) A ideia de modo de produo, o conceito de luta de classes e
c) pelo imperialismo norte-americano que firmava os Estados a interpretao da Histria como dimenso imutvel;
Unidos como maior potncia mundial d) A defesa da Revoluo Burguesa, a constituio do Estado
d) pelo conflito de interesses entre vassalos e suseranos durante Nacionalista e a conscincia de classe tratada pela religio;
a crise do Feudalismo e) A ideia de modo de produo, a supresso de toda forma de
e) pelos movimentos de contestaes aos regimes autoritrios governo e o respeito pequena propriedade.
da Europa no incio do sculo XX
08. (UFRN 2000) Leia o fragmento seguinte, cuja referencia biblio-
05. (UEL 2007) O quadro abaixo, criado pelo italiano Giuse- grfica foi intencionalmente omitida. A burguesia no forjou apenas
ppe Pellizza, uma expressiva representao da emergncia dos as armas que lhe traro a morte, produziu tambm os homens que
movimentos sociais no final do sculo XIX, ao mostrar uma empunharo essas armas: os operrios modernos, os proletrios. A
multido de trabalhadores que, determinadamente, avana para queda da burguesia e a vitria do proletariado so igualmente ine-
reivindicar seus direitos. Esse fenmeno de desenvolvimento das vitveis. Os proletrios nada tm a perder, a no ser as prprias ca-
organizaes coletivas, como o movimento sindical e os partidos deias. E tm um mundo a ganhar. Proletrios de todos os pases,
polticos, teve incio na Europa e Estados Unidos do sculo XIX, uni-vos.As ideias contidas nesse fragmento so representativas do(a)
espalhando-se por todo o mundo ocidental. a) Tratado de Versalhes, que criou uma srie de determinaes,
visando enfraquecer o poder da burguesia na Europa.
b) Declarao dos Direitos do Homem e do Cidado, que se
colocou contra a sociedade, a qual mantinha privilgios
exclusivos da burguesia.
c) Doutrina Monroe, que consolidou a autonomia latino-
americana, propondo a unio dos povos americanos.
d) Manifesto Comunista, que esboou as proposies que se
tornaram o alicerce do movimento comunista internacional.

09. (FATEC 2000) A queda da burguesia e a vitria do prole-


tariado so igualmente invitveis. (...) Os proletrios nada tm a
(Fonte: SCOTTI, A. Il Quarto Stato di Giuseppe Pellizza da Volpedo. Milano: TEA perder com ela, a no ser as prprias cadeias. E tm um mundo
Arte, 1998.) a ganhar. Proletrios de todos os paises, uni-vos. Esse trecho,

132 UECEVEST

Apostilas UECEVEST mod3.indb 132 06/02/2011 09:59:07


HISTRIA G E R A L I I

extrado do Manifesto Comunista de Marx e Engels, foi escrito Thomas Malthus Escreveu O Ensaio sobre o Principio da
no contexto histrico marcado Populao. Nessa obra, ele diz que a populao sempre tende
a) Pelo acirramento das contradies polticas, econmicas e a crescer muito mais do que a produo de alimentos.E a
sociais decorrentes do processo conhecido como Revoluo consequncia inevitvel dessa desapropriao a pobreza
Industrial. crescente. Assim, a nica forma de evitar catstrofes seria
b) Pelos conflitos entre trabalhadores e patres que comearam negar a assistncia social aos pobres e orienta-los no sentido de
a pontuar os pases capitalistas a partir da ocorrncia da controlar a natalidade.
Revoluo Russa. David Ricardo Autor de Princpios de Economia Poltica
c) Pela afirmao dos Estados Unidos como potncia e Tributao, tambm justificou a explorao do trabalho,
imperialista com interesses econmicos e polticos em vrias afirmando que os salrios sempre tendem a ser equivalentes ao
regies do planeta. mnimo necessrio sobrevivncia do trabalhador. Tomando
d) Pelo confronto entre vassalos e suseranos, no momento por base esse pensamento, os salrios baixos so consequncia
de pice da crise do modo de produo feudal e de de uma lei natural a lei da oferta e da procura e, portanto,
enfraquecimento da autoridade religiosa. no h como se opor a essa situao. Tais ideias como se v,
e) Pelo incremento das contestaes populares s diretrizes atendiam claramente aos interesses da burguesia industrial.
polticas implantadas pelos regimes autoritrios que
floresceram na Europa, na primeira metade do sculo XX. Os Estados Unidos no sculo XIX
Alm de terem sido os precursores dos movimentos de indepen-
dncia da Amrica, os Estados Unidos foram os primeiros a formar
um slido parque industrial no continente. Por essa razo, foram
tambm os primeriros a necessitar de um mercado consumidor
10. (ENEM 2010) a Poluio e outras ofensas ambientais ainda grande o bastante para dar conta de suas necessidades capitalistas.
no tinham esse nome, mas j eram largamente notadas no Sculo
XIX, nas grandes cidades inglesas e continentais. E a prpria che- A marcha para o oeste
gada ao campo das estradas de ferro suscitou protestos. A reao
antimomaquinista, protagonizada pelos diversos luddismos, ante-
cipa a a batalha dos atuais ambientalistas. Esse era ento, o com-
bate social contra os miasmas urbanos. (SANTOS M. A natureza
do espao: Tcnica e tempo, razo e emoo. So Paulo: EDUSP,
2002, adaptado).O crescente desenvolvimento tcnico-produtivo
impe modificaes na paisagem e nos objetos culturais vivencia-
dos pelas sociedades. De acordo com o texto, pode-se dizer que
tais movimentos sociais emergiram e se expressaram por meio:
a) Das ideologias conservacionistas, com milhares de adeptos
no meio urbano.
b) Das polticas governamentais de preservao dos objetos
naturais e culturais.
c) Das teorias sobre a necessidade de harmonizao entre
tcnica e natureza.
d) Dos boicotes aos produtos das empresas exploradoras e A marcha para o Oeste pode ser entendida como um movimen-
poluentes. to de expanso territorial rumo ao Pacfico motivado pela crescente
e) Da contestao degradao do trabalho, das tradies e da necessidade de consolidar o mercado consumidor de produtos in-
natureza. dustrializados produzidos pelos Estados Unidos. Esse movimento
ganhou um forte impulso a partir do incio do Sculo XIX, quando
o governo norte-americano passou a oferecer terras a preos muito
G A B A R I T O baixos, alm de vrias outras facilidades a todos os que concor-
dassem em desbravar o interior do continente. Atendendo a este
01. a 02. d 03. b 04. b 05. d apelo, uma verdadeira multido formada por imigrantes irlande-
06. a 07. a 08. d 09. a 10. e ses, escandinavos, alemes, poloneses, russos, dentre outros, partiu
em carroas ou em pequenos navios (pelo Missisipi-Missouri) em
direo ao Oeste. Como parmetro: em 1801, a populao norte-
americana era de 4 milhes; em 1860 j chegava a 32 milhes.
Porm, grande parte das terras situadas alm dos Apalaches
lIBERAlISmO DO SCUlO XIX era habitada por centenas de povos indgenas que reagiram
GUERRA CIVIl AmERICAnA (UECE/EnEm) invaso dos seus territrios. Houve, ento, uma longa srie de
conflitos ente indgenas e desbravadores. S no perodo de 1800
liberalismo econmico a 1860, esses conflitos resultaram no extermnio de cerca de
Entre os pensadores identificados com os interesses das cama- 700mil ndios. Milhares de outros foram confinados nas in-
das burguesas, que merecem especial destaque esto os defensores meras reservas indgenas criadas pelo governo. As terras at ento
do liberalismo econmico: Adam Smith (o pai do liberalismo, habitadas pelos nativos passaram para os colonizadores.
j apresentado no capitulo sobre iluminismo); Thomas Malthus A expanso territorial norte-americana fez-se tambm atravs
e David Ricardo. Tais pensadores so chamados de liberais por- da compra de territrios, de acordos diplomticos e da guerra con-
que defendiam basicamente o livre comrcio entre pases, a no tra o Mxico. Em 1803, o governo dos Estados Unidos comprou a
interveno do Estado na economia e a livre concorrncia que, Louisiana do governo francs por 5 milhes de dlares (86 bilhes
na opinio deles, serviria para eliminar os incompetentes e forar de dlares em valores atuais). Em 1819, a Flrida foi obtida da
a queda de preos. Espanha por 15 milhes de dlares (257 bilhes de dlares em

UECEVEST 133

Apostilas UECEVEST mod3.indb 133 06/02/2011 09:59:08


H I S T RIA GERAL II

valores atualizados). O Maine, na Costa do Atlntico, e Oregon, Em 1860, o resultado das eleies presidenciais indicou a
no Pacfico, foram cedidos pela Inglaterra por via diplomtica em nomeao de Abraham Lincoln para a Presidncia da Repbli-
1846. O Alasca foi comprado da Rssia por 7 milhes de dlares ca. Lincoln era um advogado republicano, nortista, abolicionista
da poca (120 bilhes de dlares em valores corrigidos) em 1867. moderado e defensor intransigente das indstrias e da unidade
Uma terceira maneira de expandir o territrio foi atravs da norte-americana. Reagindo vitria de Lincoln, onze estados es-
guerra. A maior vtima deste procedimetno foi o vizinho pobre cravistas do sul decidiram separar-se da Unio. Em fevereiro de
do sul, o Mxico. Faz lembrar o velho ditado: Pobre Mxico, to 1861, os estados separatistas fundaram a Repblica dos Estados
perto dos Estados Unidos, to longe de Deus. A guerra contra o Confederados da Amrica, que era presidida por Jeerson Davis
Mxico comeou em 1846 e teve como causa principal a anexao e tinha como capital Richmond, na Virgnia. Trs meses depois,
do Texas que havia sido incorporado arbitrariamente ao territrio comeava a Guerra Civil, com a tentativa dos estados do norte de
norte-americano no ano anterior. Ao trmino desta guerra, ven- reintegrar unio os sulistas.
cida pelos Estados Unidos, o Mxico foi obrigado a reconhecer a No decorrer desse conflito, que levou morte 620 mil pes-
perda de metade de seu antigo territrio: alm do Texas, teve de soas, os nortistas impuseram sua superioridade econmica e mi-
entregar para os Estados Unidos a rea onde hoje a Califrnia, litar, que estava fundamentada nos seguintes pontos: populao
Nevada, Utah, Arizona, Colorado e Novo Mxico. Logo em segui- de cerca de 20 milhes de pessoas (todos livres); parque indus-
da, descobriu-se ouro na Califrnia, fato que provocou uma nova trial capaz de produzir grandes quantidades de armas, munies
onda migratria em direo ao Oeste. Na poca desta corrida do e roupas; eficiente rede de transportes (ferrovias e portos) e uma
ouro, os Estados Unidos j eram donos de um territrio imenso, gil e poderosa marinha. O sul tinha uma populao de aproxi-
banhado por dois oceanos: o Atlntico e o Pacfico. madamente 10 milhes de habitantes (3,5 milhes eram escra-
Na dcada de 1860, atendendo s necessidades geradas pela r- vos) e poucas indstrias. Em fins de 1862, no auge da guerra,
pida industrializao do Nordeste, iniciou-se a construo das pri- o Presidente Lincoln decretou a abolio da escravatura, conse-
meiras ferrovias norte-americanas. Sua funo principal era garantir guindo, com isso, enfraquecer ainda mais os sulistas e aumentar
o transporte de matrias-primas e alimentos do Oeste para o Leste. o seu prestgio poltico. Depois de sucessivas derrotas militares e
da ocupao de sua capital pelas foras nortistas, os confederados
A guerra de secesso (1861-1865) finalmente se renderam em 9 de Abril de 1865.
Profundos ressentimentos, contudo, continuaram envene-
nando e opondo sulistas e nortistas. Abraham Lincoln, reeleito
presidente, foi assassinado por um sulista fantico; o racismo
contra os negros explodiu com toda a fora. Elementos da classe
dirigente sulista, de mentalidade escravista, inconformados com
a possibilidade de ascenso dos negros, fundaram associaes ra-
cistas (como a Ku-Klux-Klan), conhecidas por perseguir, intimi-
dar e matar negros.

E X E R C C I O
Abraham Lincoln
01. (UECE 2007/1) O que ope o Norte industrial ao Sul agr-
Enquanto os Estados Unidos expandiam-se at o Pac- cola uma divergncia mais de ordem econmica: primeiro
fico, os Estados do Norte e do Sul divergiam e se desentendiam protecionista, o segundo quer a liberdade de comrcio. No ,
cada vez mais. Essas divergncias entre o Norte (que se industria- portanto, a questo do escravismo que pode explicar a origem
lizava cada vez mais) e o Sul (que continuava aferrado agricul- das hostilidades e de um conflito que causar a morte de mais
tura escravista e exportadora) explicam-se principalmente pelas de 600 mil americanos. (Fonte: KERSAUDY, Franois. Estados
diferenas de interesses entre as camadas dominantes dessas duas Unidos: o nascimento de uma nao. Trad. Ana Montoia. In:
grandes regies. O quadro a seguir resume as diferenas socioe- Revista Histria Viva. So Paulo: Duetto, nov. 2003, p. 28. No.
conmicas entre os estados nortistas e os sulistas, bem como as 1.) De acordo com o texto, podemos reconhecer como fator que
posies defendidas pelas suas camadas dominantes. desencadeou a Guerra de Secesso americana:
Sul Norte a) a pretenso dos nortistas de impedir a expanso do
escravismo nos territrios do Oeste, ainda no constitudos
Tipo de economia Economia de base Economia de base em estados.
agrcola, dependen- industrial, apoia- b) o radicalismo anti-escravista de Abrao Lincoln, eleito
te das exportaes da nas indstrias presidente da Repblica, ameaava os direitos dos
de anil, fumo e de ferro, couro e proprietrios de escravos.
algodo tecidos. c) a ao da sociedade secreta Ku Klux Klan, que acabou com
Mo-de-obra Uso intensivo de Predomnio a segregao racial ao conceder o igual direito de voto aos
trabalho escravo. de trabalho negros.
assalariado. d) a manuteno do escravismo nos Estados do Sul propiciava
Camada Grandes Burguesia industrial a industrializao nos Estados do Norte, devido mo-de-
dominante fazendeiros obra barata.
O que defendiam A continuidade da A abolio da
escravido, porque escravido, a fim de 02. (UECE 2008/1) Foi um dos conflitos mais sangrentos ocor-
suas fazendas ampliar o mercado ridos no Continente Americano (1861-1865). Estima-se a morte
dependiam do consumidor de de 970 mil pessoas entre civis e soldados. Dentre as causas do
trabalho escravo de seus produtos conflito destacam-se a questo da escravido e a rivalidade eco-
origem africana. industrializados. nmica entre o Norte industrializado e o Sul agrcola. Estamos
nos referindo

134 UECEVEST

Apostilas UECEVEST mod3.indb 134 06/02/2011 09:59:09


HISTRIA G E R A L I I

a) Guerra Civil Americana, mais conhecida como Guerra de 06. (FUVEST 2010) No Ocidente, o perodo entre 1848 e 1875
Secesso. primariamente o do macio avano da economia do capitalismo
b) Guerra das Rosas, travada entre o Canad e os Estados industrial, em escala mundial, da ordem social que o representa, das
Unidos da Amrica pela disputa de mercado consumidor e ideias e credos que pareciam legitim-lo e ratific-lo. (E. J. Hobsba-
exportador. wm. A era do capital 1848-1875.) A ordem social e as ideias e
c) Guerra do Paraguai, maior conflito armado internacional credos a que se refere o autor caracterizam-se, respectivamente, como
ocorrido no continente americano no sc. XIX, tambm a) aristocrtica e conservadoras.
chamada de Guerra da Trplice Aliana. b) socialista e anarquistas.
d) a um dos conflitos pela independncia do Mxico c) popular e democrticas.
desencadeando uma Guerra Civil travada entre diferentes d) tradicional e positivistas.
provncias que divergiam entre si. e) burguesa e liberais.

03. (FUVEST 2009) Uma casa dividida contra si mesma no 07. (IFCE 2009/1) Os liberais, no sculo XIX, diante dos pro-
subsistir. Acredito que esse governo, meio escravista e meio li- blemas causados pela industrializao, eram contrrios ao pensa-
vre, no poder durar para sempre. No espero que a Unio se mento socialista e defendiam
dissolva; no espero que a casa caia. Mas espero que deixe de a) a propriedade privada e a livre concorrncia.
ser dividida. Ela se transformar s numa coisa ou s na outra. b) o fim da livre iniciativa e da propriedade coletiva dos bens
(Abraham Lincoln, em 1858.) Esse texto expressa a de produo.
a) Posio poltica autoritria do presidente Lincoln. c) uma revoluo social, para derrubar a sociedade dividida em
b) Perspectiva dos representantes do sul dos EUA. classes sociais.
c) Proposta de Lincoln para abolir a escravido. d) a organizao do Partido Comunista para a luta dos
d) Proposio nortista para impedir a expanso para o Oeste. trabalhadores.
e) Preocupao de Lincoln com uma possvel guerra civil. e) a abolio do Estado.

04. (PUCMG 2007) A Guerra Civil norte-americana, tambm


chamada de Secesso, por se tratar do conflito entre o Sul e o
Norte dos EUA, trouxe desdobramentos polticos ideolgicos G A B A R I T O
graves para a formao da identidade nacional do pas. Assinale 01. a 02. a 03. e 04. d 05. c 06. e 07. a
o fato que mais aprofundou a radicalizao dos conflitos tnicos
em territrio americano ao longo do sculo XX.
a) a criao da Ku-Klux-Klan em territrio sulista ao final da
guerra.
b) a formao dos Estados Confederados liderados Abraham O ImPERIAlISmO nO SCUlO XIX
Lincoln. (UECE/EnEm)
c) o estabelecimento da doutrina Monroe, que garantia as
liberdades civis a todas as raas nos EUA. A Revoluo Industrial, em meados do Sculo XVIII, criou
d) as leis a favor do apartheid, promulgadas aps o desfecho do condies para a consolidao do capitalismo, que evoluiu, mo-
conflito. dificou-se e partiu para a conquista poltica, econmica e cultural
na frica, sia e Amrica Latina. Na segunda metade do sculo
05. (UFMG 2005) Leia este trecho de documento: Odeio-a XIX, o capitalismo foi impulsionado por um crescimento da in-
porque impede a nossa Repblica de influenciar o mundo pelo dustrializao fenmeno que, por sua repercusso, ficou conheci-
exemplo da liberdade; oferece possibilidade aos inimigos das ins- do como Segunda Revoluo Industrial. O principal fator dessa
tituies livres de taxar-nos, com razo, de hipocrisia e faz com grande industrializao foi a acumulao de capital resultante
que os verdadeiros amigos da liberdade nos olhem com descon- dos altos lucros obtidos pelos capitalistas nos primeiros cem anos
fiana. Mas, sobretudo, porque obriga tantos entre ns, realmen- de industrializao. Outros fatores responsveis pelo vertiginoso
te bons, a uma guerra aberta contra os princpios da liberdade crescimento da indstria foram as inovaes tcnicas, na produ-
civil. (Discurso de Abraham Lincoln, em 1859.) Nesse trecho de o, nos meios de transporte e de comunicao.
discurso, Abraham Lincoln, que seria eleito Presidente dos Esta- Uma dessas inovaes foi a descoberta, em 1865, por Hen-
dos Unidos no ano seguinte, faz referncia ri Bressemer, de um processo relativamente simples e barato de
a) poltica de segregao racial existente nos estados do sul fabricao de ao. Muito mais resistente do que o ferro, o ao
dos Estados Unidos, que gerou a formao de organismos passou a ser amplamente utilizado na indstria de armas, na
voltados ao extermnio dos negros, destruio de construo civil e na fabricao de mquinas e equipamentos.
suas propriedades e a atentados constantes contra suas Outra importante inovao tcnica foi o aproveitamento de no-
comunidades. vas fontes de energia, como eletricidade, o petrleo e seus deri-
b) posio dos estados do sul de defesa intransigente de vados. Substituindo o carvo com grande vantagem, essas novas
tarifas protecionistas, o que levava os Estados Unidos a fontes possibilitaram a construo de mquinas mais potentes, de
comprometer a crena na liberdade de mercado, numa novas indstrias, estimulando enormemente o desenvolvimento
conjuntura de predomnio do capitalismo liberal. dos meios de transporte (locomotivas, navios, automveis) e dos
c) questo da escravido, que levou a uma guerra civil, nos meios de comunicao (telgrafo eltrico, telgrafo sem fio, rdio,
Estados Unidos, entre o Norte, industrializado, e o Sul, telefone etc.) Essas inovaes tcnicas e o progresso dos meios de
que lutava para preservar a mo-de-obra escrava nas suas transporte e comunicao foram, ao mesmo tempo, efeito e causa
plantaes de produtos para a exportao. do desenvolvimento do capitalismo. Ao mesmo tempo em que
d) defesa, pelos imigrantes, do extermnio dos ndios nas resultaram da aplicao de capitais em pesquisa cientifica, provo-
terras conquistadas a oeste, especialmente aps a edio do caram um fabuloso aumento no acmulo de capitais nos pases
Homestead Act, visando ao desenvolvimento da agricultura onde se localizavam como Inglaterra, Frana, Blgica, Holanda,
e da pecuria naquelas reas. Itlia, Alemanha, Estados Unidos e Japo.

UECEVEST 135

Apostilas UECEVEST mod3.indb 135 06/02/2011 09:59:10


H I S T RIA GERAL II

O capitalismo financeiro de nativos na frica e na sia, enquanto dividiam entre si as ri-


Com o desenvolvimento da industrializao, os gastos para se quezas desses continentes. clebre a frase do magnata ingls
montar e manter uma empresa aumentaram de modo exorbitante. Cecil Rhodes: Eu anexaria os planetas, se pudesse.
As mquinas e as instalaes tornaram-se muito caras. Os servios No entanto, h inmeras diferenas entre o neocolonialismo
de venda, transporte e propaganda, a obteno de matria-prima do sculo XIX e o colonialismo do sculo XVI. O quadro a seguir
e o pagamento de mo-de-obra exigiam investimentos cada vez apresenta algumas dessas diferenas:
mais volumosos. Ao mesmo tempo, para enfrentar a concorrn-
cia, que tambm aumentava, era preciso manter os preos baixos. Colonialismo Neocolonialismo
Assim, somente as empresas com grandes reservas de capital pu- poca de atuao Sculos XVI Sculo XIX
deram resistir. Nessa poca, houve uma crescente concentrao XVIII
de produo e do capital em poucas empresas gigantescas. Isso
aconteceu graas formao de trustes, holdings e cartis. rea de atuao Amrica frica e sia
Truste Origina-se da fuso de vrias empresas numa nica, Fase do capitalismo Capitalismo Capitalismo financeiro
que passa a dominar todas as fases da produo: da obteno da comercial
matria-prima at a comercializao do produto. Objetivos Busca de metais Busca de metais
Holding Nasce da associao de diversas empresas sob a di- preciosos; preciosos; mercados
reo de uma empresa central, que possui o controle a maioria mercados consumidores
das aes de seus associados. consumidores de produtos
Cartel Resulta de acordos entre empresas livres e indepen- de produtos industrializados;
dentes que, para evitarem os desgastes da concorrncia, divi- manufaturados; matrias primas como
dem o mercado entre si. Cada uma delas atua numa determi- mercados carvo, ferro, petrleo,
nada rea geogrfica. produtores cobre e borracha; reas
de gneros para investimento de
Por causa dessa disputa cada vez mais acirrada, as grandes em- tropicais. capitais disponveis.
presas se viram obrigadas a pedir aos bancos cada vez mais emprs- Justificativas Converter Civilizar os povos
timos. Aproveitando-se disso, os bancos foram se associando a elas, populaes atrasados, difundindo
com o tempo, passaram a dirigi-las. medida que esse processo nativas ao entre eles o progresso
foi se desenrolando, o capitalismo industrial foi cedendo lugar ao cristianismo. tcnico-cientifco;
capitalismo financeiro: etapa do capitalismo, na qual a indstria, o estimular a evoluo
comrcio e a agricultura passaram a depender dos bancos. biolgica dos negros
e dos amarelos (povos
Imperialismo e neocolonialismo considerados inferiores).
Com o capitalismo financeiro e a produo em massa, as na-
es capitalistas passaram a ter uma grande necessidade de mer- Partilha da frica
cados consumidores, matrias-primas e reas nas quais pudessem Desde o sculo XV, quando o litoral africano comeou a ser
investir e obter mais lucros. O que explica essa necessidade o ocupado pelos portugueses at a segunda metade do Sculo XIX,
fato de que, ao decolarem rumo industrializao, todas as naes a frica foi vista pelos europeus como uma grande mercearia de
capitalistas adotaram um conjunto de medidas protecionistas, re- escravos e especiarias (pimenta, plantas, animais raros, marfim,
servando seus mercados internos para as suas prprias indstrias. etc). A partir da segunda metade do Sculo XIX, porm, com o
Em consequncia, comearam a sobrar produtos industrializa- extraordinrio avano do capitalismo na Europa Ocidental, es-
dos. Diante disso, as naes capitalistas decidiram expandir-se tadistas e magnatas europeus passaram a encarar a frica como
em direo frica, sia e Amrica Latina. O expansionismo vastssimo mercado consumidor de produtos industrializados e
dessas naes teve um carter imperialista e neocolonialista. fornecedor de matrias-primas. Nessa mesma poca, ainda, des-
O imperialismo consiste na dominao econmica (com re- cobriu-se que a frica era rica em pedras preciosas, principalmen-
flexos polticos e culturais) de um pas sobre outro. O imperia- te em diamantes. Essa notcia aumentou a cobia europeia em
lismo ingls, por exemplo, penetrou com intensidade no Brasil relao s terras africanas, que passaram a ser vistas como reas
principalmente aps a vinda de famlia real. O neocolonialismo, onde investimentos em minerao, portos e estradas dariam, com
por sua vez, significa a dominao total de um pas sobre o outro. certeza, lucros extraordinrios. Assim, munidos de ideias racistas
A maioria dos pases africanos e asiticos foram vtimas do neo- e movidos por fortes interesses econmicos, os europeus do Scu-
colonialismo: tiveram seus territrios conquistados e submetidos lo XIX, lanaram-se sobre o continente negro.
econmica, poltica, administrativa, cultural e militarmente.

Justificativas
Para justificar a dominao sobre os povos estrangeiros, os
europeus do sculo XIX usaram trs argumentos principais:
As naes crists usavam o argumento de cristianizar todos os
povos, para com o seu esprito missionrio, livr-las da super-
tio e da barbrie.
O homem branco tinha uma misso civilizadora. Ou seja, esta-
va destinado a levar a civilizao (o progresso tcnico-cientfico
e os bons costumes) aos povos no-brancos, o que caracteriza
o darwinismo social.
A superioridade da raa branca.

Valendo-se desses argumentos elitistas e racistas, as grandes


potncias industriais exploraram, violentaram e mataram milhes

136 UECEVEST

Apostilas UECEVEST mod3.indb 136 06/02/2011 09:59:11


HISTRIA G E R A L I I

Em 1885, quando a penetrao imperialista no territrio Japo Era meiji


africano j se encontrava num estagio avanado, as potncias eu- O Japo conseguiu manter-se fechado ao Ocidente por mais
ropeias reuniram-se na Conferncia de Berlim, convocada pelo tempo que a China. Vivendo em uma espcie de feudalismo onde
chanceler alemo Bismarck, a fim de oficializar a partilha da o poder de fato estava nas mos dos senhores de terra, os daims,
frica. Essa conferncia determinou que, para tornar-se dono de sendo o Imperador, com o ttulo de Mikado, apenas uma pea
um territrio africano, o pas europeu deveria ocup-lo efetiva- decorativa, submissa at mesmo ao poder do primeiro-ministro, o
mente e, em seguida, comunicar o ocorrido s demais potncias. Xgum. Os samurais, guerreiros profissionais, sustentavam o regi-
Os pases que lideraram a partilha da frica foram justamente me com base no poder das armas. Em 1853, entretanto, ameaados
os que primeiro se industrializaram: Inglaterra, Frana e Blgica. pelos canhes de uma esquadra americana, comandada pelo Almi-
Posteriormente, depois da unificao, Alemanha e Itlia tambm rante Perry, os japoneses foram forados a assinar um tratado que
entraram na corrida imperialista. Em fins do sculo XIX, s dois abria seus portos ao comrcio com os Estados Unidos. Nos anos
pases africanos permaneciam livres: Libria e Etipia. seguintes, apesar dos fortes protestos populares, o governo do Japo
assinou tratados semelhantes com outras potncias estrangeiras.
A dominao da sia Muito antes que os demais povos asiticos, os japoneses com-
Cobiado pelos europeus desde longa data, o vasto continen- preenderam que, para evitar invases e humilhaes, precisavam
te asitico foi mais uma vtima do imperialismo. modernizar-se. Com esse propsito, a partir de 1868, foi feita
uma verdadeira revoluo cultural, que ps fim ao feudalismo
ndia no pas, fortaleceu o poder imperial e deu incio chamada Era
A dominao inglesa na ndia teve inicio quando os navios da Meiji, ou Era das Luzes. O governo japons incentivou um
Companhia Inglesa das ndias Orientais aportaram no litoral hin- programa de assimilao da tecnologia ocidental, o que levou o
du, ainda no Sculo XVI. Pouco a pouco, os ingleses desalojaram pas a industrializar-se rapidamente.
os comerciantes de outras naes e venceram a resistncia hindu. Em fins do Sculo XIX, o Japo que nessa poca j havia se
No final do Sculo XVIII, j estendiam a sua dominao por quase tornado uma grande potncia, comeou tambm a praticar o
todo o interior do pas. Nessa poca, a Inglaterra comprava da n- imperialismo: Em 1895, venceu a China e conquistou Formosa;
dia sedas e tecidos de algodo. No entanto, com o desenvolvimento em 1905, surpreendeu o mundo vencendo uma guerra contra a
da Revoluo Industrial, os ingleses comearam a vender seus pr- Rssia e conquistando a Coreia.
prios tecidos e, ao mesmo tempo, foraram a alta dos preos dos
tecidos hindus, taxando-os de modo abusivo. Em pouco tempo, a Estados Unidos Big Stick
produo txtil hindu foi falncia. Com isso, a ndia, que antes
exportava tecidos, passou a compr-los, da Inglaterra.
Reagindo dominao inglesa, soldados nacionalistas hindus
promoveram a Revolta dos Cipaios (1857-1858). Valendo-se de
sua superioridade militar, os ingleses esmagaram a revolta e es-
magaram seus principais lderes. A seguir, a Rainha Vitria, da
Gr-Bretanha, foi coroada Imperatriz da ndia e oficializou-se o
controle absoluto da Inglaterra sobre a regio. Aps isso, foi cons-
trudo um sistema ferrovirio e de comunicaes que lhes possi-
bilitou uma melhor explorao econmica do territrio indiano.

China
A presena dos ingleses na China comeou no Sculo XVII,
quando foi lhes concedido o controle do Porto de Canto, onde
compravam ch, seda e objetos de porcelana e vendiam pio,
uma droga entorpecente originria da ndia. Essa droga aluci-
ngena e que causava dependncia qumica era proibida na In- Apoiados na ideologia do Destino Manifesto, que pregava a
glaterra. Apesar disso, o criminoso e lucrativo comrcio de pio, misso civilizadora dos Estados Unidos, e na Doutrina Monroe,
representava em 1833, cerca de 50% do total de exportaes da Amrica para os americanos, o Presidente Theodore Roosevelt
Inglaterra na China. Decidido a acabar com essa situao, o go- criou o Corolrio Roosevelt, segundo o qual o pas e a Amrica
verno chins ordenou a queima de 20 mil caixas de pio que iam Latina tinham os mesmos interesses e, por isso, deviam cooperar
ser distribudas pelos comerciantes ingleses. para o desenvolvimento comum. A aplicao desse corolrio foi a
A Inglaterra reagiu declarando guerra China e, depois de poltica do Big Stick (Grande Porrete), que se expressou na pol-
vencer o conflito, conhecido como Guerra do pio (1840- tica imperialista desenvolvida pelos Estados Unidos, em relao
1842), obrigou os chineses a assinar o Tratado de Nanquim. Por Amrica Latina: intervenes militares para defender os interesses
esse tratado, a Inglaterra obteve o direito de controlar a Ilha de do governo e dos grupos econmicos norte-americanos.
Hong-Kong, a abertura de seu porto ao comrcio e uma pesada No final do Sculo XIX, os Estados Unidos voltaram-se
indenizao de guerra. Dois anos depois, os norte-americanos e para o domnio da Amrica Central, realizando intervenes em
os franceses tambm conseguiram, atravs da presso militar, que Cuba, Panam e Nicargua. Nessa mesma poca, o enfraquecido
a China lhes concedesse inmeros benefcios comerciais. imprio espanhol cedeu as ilhas de Porto Rico, na Amrica Cen-
Em 1894, a China reagiu ao expansionismo japons na sia, tral, e Filipinas e Guam, na Oceania. Em 1914, os americanos
mas foi facilmente derrotado na Guerra Sino-Japonesa. Saindo en- ocuparam o Haiti.
fraquecida dessa guerra, a China perdeu o domnio que tinha sobre
vrios territrios e foi forada a entregar seus principais portos ao
controle estrangeiro. Assim, no final do Sculo XIX, embora tivesse E X E R C C I O
conseguido manter a sua unidade territorial, a China encontrava-
se submetida aos interesses de sete naes imperialistas: Inglaterra, 01. (UFC 2007) A partir das ltimas dcadas do sculo XIX,
Frana, Alemanha, Itlia, Rssia, Estados Unidos e Japo. uma nova onda colonialista levou partilha quase total da frica

UECEVEST 137

Apostilas UECEVEST mod3.indb 137 06/02/2011 09:59:12


H I S T RIA GERAL II

e da sia entre pases industrializados. Sobre esta fase imperialis- a nica coisa que a Gr-Bretanha podia vender China). Houve
ta, correto afirmar que foi motivada fundamentalmente: ento fome em massa em Bengala. [...] A partir do sculo XVIII,
a) pelo interesse de importar bens manufaturados da ndia, a GrBretanha imps duras leis tarifrias para impedir que os
China e frica islmica e foi estimulada pelos pases produtos industrializados indianos competissem com a produ-
industriais emergentes: Blgica, Alemanha e Japo. o txtil dos ingleses. Eles tiveram de enfraquecer e destruir as
b) pela poltica religiosa e missionria de difundir o indstrias txteis indianas, pois a ndia tinha uma relativa vanta-
cristianismo no mundo e foi liderada pelos pases catlicos gem utilizava um algodo de melhor qualidade, e um sistema
europeus, como a Frana e a Blgica. industrial, em muitos aspectos, comparvel ou superior ao brit-
c) pela exigncia do conhecimento cientfico positivista de nico. (Fonte: CHOMSKY, N. A minoria prspera e a multido
ocupar os territrios a serem estudados e foi impulsionada inquieta. Traduo de Mary Grace Figheira Perptuo. Braslia:
pela Gr-Bretanha. UNB, 1999, p. 84-85. Com base no texto e nos conhecimentos
d) pela necessidade de adquirir facilmente matria-prima a sobre o tema, assinale a afirmativa correta:
baixo custo e foi facilitada pela poltica imperialista dos a) A poltica comercial britnica refletia as ideias do liberalismo
Estados Unidos. neo-colonial. Postulavam e realizavam, assim, a total liberdade
e) pelo interesse de continuar a expandir o capitalismo num de produo e comercializao entre metrpoles e colnias.
perodo de crise e teve sua frente a Frana e a Gr- b) A Gr-Bretanha no abriu os mercados da China com
Bretanha. o pio das papoulas, pois a corte inglesa pressupunha o
exerccio da cincia e da preservao da conduta humana
02. (UECE 2008/2) A frica do inicio do sculo XX encontra- na constituio bsica da civilizao e dos direitos frente aos
se partilhada entre diferentes pases, configurando uma face do povos de outros continentes.
imperialismo europeu. Assinale o correto. c) A acumulao primitiva de capital advinda da abertura das
a) A poltica imperialista impulsionou Inglaterra, Frana, terras pblicas, como exemplo de liberdade de competio entre
Alemanha, EUA, Portugal, Espanha, Itlia a estabelecerem, os camponeses, para o desenvolvimento da economia agrria,
entre si, inmeros acordos e alianas que dividiram, entre foi um dos fatores principais para a Revoluo Industrial.
eles, diferentes territrios africanos. d) A indstria txtil inglesa e a Companhia de Navegao das
b) A ocupao imperialista na frica foi resultado de inmeras ndias Orientais estabeleceram um ramo comercial e fabril
negociaes e acordos entre territrios africanos e as naes articulado ao poder monrquico, impondo uma legislao
europeias que queriam ampliar o seu mercado consumidor. de tarifas aduaneiras s outras metrpoles, impedindo-as de
c) A poltica imperialista francesa reuniu seus domnios por terem uma economia agrcola.
meio de um bloco contnuo unindo entre si, e pelo interior, e) O processo histrico de constituio do capitalismo
suas colnias litorneas (frica Ocidental Francesa e frica britnico representou, para as suas colnias na sia, uma
Equatorial Francesa). destruio dos sistemas econmicos autctones.
d) Os territrios da frica oriental, especialmente o sul do
continente, foram ocupados e, posteriormente, negociados 05. (UEMG 2006) Leia os fragmentos abaixo.
e divididos em postos estratgicos entre a Inglaterra e seu I. No vamos deixar a frica para os pigmeus, quando uma
aliado, os EUA. raa superior se est multiplicando ... Esses indgenas esto
destinados a serem dominados por ns ... O indgena deve
03. (UECE 2009/2) Durante o sculo XIX, as naes imperia- ser tratado como uma criana, e o direito eleitoral lhe
listas europeias justificaram seu poderio sobre grande parte do proibido pelas mesmas razes do lcool. (Cecil Rhodes,
continente africano afirmando: Age-se, assim, para o bem de ingls, fundador da Rodsia)
todos. A Europa no abandonar, absolutamente, sua autoridade II. Escutei tuas palavras, mas no vi qualquer motivo para
colonial. Ela est no comando e no comando deve permanecer. obedecer-te, antes preferiria morrer. Se o que queres
(SARRALT, Alberte. Grandeza y servidumbres coloniales, 1931. amizade, estou pronto a oferecer-te, hoje e sempre; mas
Apud BRUIT, Hector. O imperialismo. So Paulo/Campinas: quanto a ser teu sdito, isso nunca! Se o que queres guerra,
Atual/Editora da Unicamp, 1987, p. 11.) Sobre o neocolonialis- estou pronto para ela, mas ser teu sdito, nunca! No cairei
mo europeu, correto afirmar que a teus ps, porque s uma criatura de Deus, assim como
a) foi aceito e defendido sem restries, especialmente pelos eu. Sou sulto aqui na minha terra. Tu s sulto l na tua!
pases do continente africano. (Machemba, rei dos yaos de Tanganica, ao comandante
b) foi um discurso ideolgico e preconceituoso que em alemo, em 1890.)
nenhum momento existiu na prtica.
c) foi amplamente justificado por meio de um discurso A contextualizao histrica dos dois fragmentos apresentados
autoritrio, civilizatrio e eurocntrico. PERMITE afirmar que
d) foi justificado pelas aes expansionistas e imperialistas das a) o domnio da frica expressou interesses econmicos e
naes europeias e aceito por todos os pases. culturais dos europeus e teve que enfrentar vozes dissonantes
de poderosos africanos.
04. (UEL 2007) Quando os britnicos chegaram pela primeira b) a colonizao da frica no sculo XIX significou, para
vez, Bengala era um dos lugares mais ricos do mundo. Os primei- europeus e africanos, um fardo a ser carregado em nome do
ros mercadores britnicos descreveram-na como um paraso. [...] desenvolvimento industrial.
L havia ricas reas agrcolas, que produziram um algodo de rara c) a superioridade racial europeia, visvel nos equipamentos
qualidade, e tambm uma indstria avanada para os padres da de guerra, foi o aspecto determinante da partilha da frica
poca. Para se ter uma ideia, uma firma indiana construiu, du- quando da corrida imperialista.
rante as guerras napolenicas, uma das naus para um almirante d) por natureza submissos, os africanos acabaram por se curvar
ingls. [...] Segundo [Adam] Smith, os ingleses destruram pri- aos europeus no processo conhecido como neocolonialismo.
meiro a economia agrcola, depois transformaram a carncia em
fome coletiva. Uma maneira de fazer isso foi transformar terras 06. (IFCE 2009/2) Leia atentamente os versos abaixo.
agrcolas em reas para a produo de papoulas (j que o pio era Aceitai o fardo do homem branco,
Enviai os melhores de vossos filhos.

138 UECEVEST

Apostilas UECEVEST mod3.indb 138 06/02/2011 09:59:12


HISTRIA G E R A L I I

Condenai vossos filhos ao exlio, a) a interveno dos EUA na Amrica Central foi rejeitada
Para que sejam os servidores de seus cativos. pelos movimentos populares, como as revolues sandinista
(RUDYARD KIPLING, poeta ingls de origem hindu. Fonte: PAZZINATO, A. L e e mexicana.
SENISE, M. H. V. Histria moderna e contempornea. So Paulo: tica, 2002. p. 135). b) a poltica do Big Stick foi amplamente rechaada pelo
Os versos acima retratam a expanso neocolonialista e esto rela- governo brasileiro graas ao apoio poltico e financeiro da
cionados diretamente s opes abaixo, com exceo da Inglaterra e da Frana.
a) Ideologia do Imperialismo que, utilizando-se da Biologia, c) o governo estadunidense favoreceu o Paraguai na guerra
passou a justificar a superioridade dos conhecimentos contra a Argentina pelo controle da regio petrolfera do
cientficos alcanados pelos europeus, como sendo da Chaco, onde atuava a Standard Oil Co.
natureza da raa branca. d) os movimentos populares apoiados na luta e no pensamento
b) Crena, poca, da superioridade da raa branca sobre as poltico de Jos Mart evitaram que Cuba, logo aps a
demais, merecendo com isso o direito de dominao do planeta. independncia, se tornasse um protetorado dos EUA.
c) A Geopoltica que procurava mostrar que a natureza e) a Repblica do Panam proclamou sua independncia da
geogrfica determinava a necessidade de certos povos de Colmbia em 1903, tornando-se um protetorado dos EUA,
conquistar um espao vital para sua sobrevivncia. e, em 1914, foi inaugurado um canal ligando o Atlntico ao
d) Darwinismo social, uma teoria baseada no evolucionismo de Pacfico.
Charles Darwin, que admitia a superioridade social de umas
classes ou povos sobre outros. 10. (ENEM 2005) Um professor apresentou os mapas ao lado
e) Esprito filantrpico das naes europeias que lanavam o numa aula sobre as implicaes da formao das fronteiras no
melhor de sua formao intelectual e cientfica aos mais continente africano. Com base na aula e na observao dos ma-
diversos recantos do planeta, a fim de promover o progresso pas, os alunos fizeram trs afirmativas:
e a harmonia entre os povos.

07. (UFMG 2004) Entre, aproximadamente, 1880 e 1914, ocor-


reu a corrida para a frica, ou seja, uma acelerao no processo
de conquista desse Continente por parte das potncias europeias.
Nesse curto perodo cerca de trs dcadas , o Continente Afri-
cano foi quase inteiramente retalhado por alguns Estados euro-
peus, que disputavam a primazia na formao de imprios colo-
niais. Considerando-se a conquista imperialista e a subsequente
colonizao da frica, correto afirmar que
a) Os missionrios religiosos e cientistas que atuavam
nesse Continente denunciaram as aes praticadas pelos
conquistadores, tentando deter a colonizao.
b) A instalao efetiva de colonos europeus se deu em maior
proporo nas atuais regies da frica do Sul e Arglia.
c) Os Estados dominantes reservaram para si as conquistas,
impedindo a participao das potncias europeias de menor
expresso na diviso das terras.
d) Os europeus encontraram facilidade para estabelecer o
domnio militar, dada a ausncia de instituies polticas e
de lderes locais capazes de organizar a resistncia.
(Atualidades/Vestibular 2005, 1 sem., ed. Abril, p. 68)
08. (UECE 2004/1) Do sculo XV at o XIX, a violncia dos
colonialistas e imperialistas europeus contra os povos da frica, I. A brutal diferena entre as fronteiras polticas e as fronteiras
sia e Amrica era justificada de variadas formas, especialmente tnicas no continente africano aponta para a artificialidade em
como uma tarefa civilizadora a ser cumprida pelos povos mais uma diviso com objetivo de atender apenas aos interesses da
desenvolvidos. (Fonte:VICENTINO, Cludio e DORIGO, maior potncia capitalista na poca da descolonizao.
Gianpaolo.Histria Para o Ensino Mdio: histria geral e do Bra- II. As fronteiras polticas jogaram a frica em uma situao de
sil. So Paulo: Scipione, 2001, p.436.) A respeito do chamado constante tenso ao desprezar a diversidade tnica e cultural,
eurocentrismo, correto afirmar: acirrando conflitos entre tribos rivais.
a) As atividades de melhoria e bem estar dos povos colonizados III. As fronteiras artificiais criadas no contexto do colonialismo,
superava as condies de explorao, que lhes eram impostas. aps os processos de independncia, fizeram da frica um
b) Os sacrifcios impostos aos colonizados, pelos europeus, continente marcado por guerras civis, golpes de estado e
foram atividades oriundas de exigncias da Igreja Catlica. conflitos tnicos e religiosos.
c) A ao dos missionrios catlicos e protestantes contribuiu
para respeitar o direito de livre expresso dos povos colonizados. verdadeiro apenas o que se afirma em
d) A posio ocupada pela Europa, como centro poltico e a) I. d) I e II.
econmico do mundo, entrou em declnio a partir das duas b) II. e) II e III.
guerras mundiais. c) III.

09. (UFC 2010) Com a adoo da poltica do Big Stick, os EUA,


no governo de Theodore Roosevelt, inauguraram uma prtica de G A B A R I T O
interveno, inclusive armada, em especial nos pases latino-ame-
01. e 02. c 03. c 04. e 05. a
ricanos, onde o capital estadunidense tornou-se hegemnico. Em
decorrncia desta poltica, correto afirmar que: 06. e 07. b 08. d 09. a 10. e

UECEVEST 139

Apostilas UECEVEST mod3.indb 139 06/02/2011 09:59:13


H I S T RIA GERAL II

A PRIMEIRA GUERRA MUNDIAL (1914-1918) libertar e unificar os territrios habitados pelos povos eslavos
(UECE/ENEM) dessa regio. Seu propsito era formar a Grande-Srvia. Alguns
desses territrios estavam sob o domnio do Imprio Austro-
Hngaro; outros, como a Bsnia, por exemplo, estavam sob o
O Sculo XX comeou na prtica com a Primeira Guerra
domnio turco. Opondo-se aos austracos e aos turcos, a Srvia
Mundial. Esta foi a primeira guerra na histria a envolver, ao
aproximou-se cada vez mais da Rssia, que comprometeu-se a
mesmo tempo, pases dos cinco continentes, teve como objetivos
apoi-la e a proteg-la militarmente. Em 1908, entretanto, a
a dominao de territrios nos quatro cantos da terra e foi tam-
ustria ocupou a Bsnia-Herzegovina. Considerando-se pre-
bm um conflito altamente equilibrado, porque os principais en-
judicada por este ato, a Srvia passou a conspirar abertamente
volvidos eram possuidores dos melhores armamentos disponveis
contra a ustria. Tudo indicava que o desfecho dessa histria
na poca, tais como, o avio e o submarino.
de intensas rivalidades seria muito violento.

Causas
A formao das alianas a paz armada
O motivo fundamental da Primeira Guerra Mundial foi a dis-
Nesse quadro de interesses conflitantes, as grandes potncias
puta por mercados consumidores entre as naes imperialistas. A
firmaram entre si vrios tratados de aliana, com o objetivo de
necessidade crescente por mercados consumidores, fornecedores de
se protegerem contra um inimigo comum. As primeiras alianas
matrias-primas e reas para investimentos levou as grandes potn-
foram arquitetadas pelo chanceler alemo Bismarck. Ciente de
cias a dominarem inmeros povos e territrios em diversas partes do
que a qualquer momento a Frana poderia partir para a revanche
mundo. Assim, em algumas dcadas, atravs da prtica do imperia-
contra o seu pas, Bismarck decidiu isol-la poltica e diplomati-
lismo, essas grandes potncias acumularam riquezas e aumentaram
camente. Num primeiro momento, a Alemanha aliou-se ao Im-
muito sua capacidade de produzir mercadorias. Por isso, o alvorecer
prio Austro-Hngaro, com o qual tinha estreitos laos culturais.
do sculo XX foi marcado por vrios conflitos entre as naes im-
Posteriormente, cortejou e conseguiu aliar-se Itlia, nao que,
perialistas. Entretanto, existiram tambm outras razes, tais como:
por pretender a Tunsia, no se conformou quando esse pas afri-
Rivalidade Franco-Alem Na Frana, havia um sentimen-
cano foi anexado pela Frana.
to de antigermanismo muito presente em toda a sociedade.
A Frana, por sua vez, reagiu ao isolamento em que se encon-
Principalmente devido ao revanchismo, provocado pela perda
trava fazendo um acordo militar secreto com a Rssia, pas que
da Alscia-Lorena (regio rica em minrios) para a Alemanha
temia o avano alemo para o leste. Depois, foi a vez da Inglaterra
em 1870, durante a Guerra Franco-Prussiana. Outro ponto
assustada com o crescente o poderio alemo assinar um acordo
de conflito entre franceses e alemes era sobre o controle do
com a Frana e outro com a Rssia.
Marrocos, pas ao norte da frica importantssimo do ponto
Assim, em 1907, a Europa j se encontrava dividida em dois
de vista estratgico. Essa disputa foi provocada pelos interesses
blocos poltico-militares: a Trplice Aliana, com a Alemanha,
germnicos em se apoderar de uma regio que ficara designada
Itlia e a ustria-Hungria e a Trplice Entente, com a Inglater-
para a Frana na Conferncia de Berlim, em 1884.
ra, Frana e Rssia. Enquanto se organizavam em blocos rivais,
Rivalidade Anglo-Germnica A origem da rivalidade entre
as potncias europeias lanaram-se numa desenfreada corrida
Inglaterra e Alemanha era a competio industrial e comercial.
armamentista: adotaram o servio militar obrigatrio, criaram
A industrializao alem foi muito rpida. Em apenas trinta
novas armas e passaram a produzir armamentos e munio em
anos, a contar da sua unificao, a Alemanha deixou de ser um
quantidades cada vez maiores. Faltava apenas um incidente para
pas fundamentalmente agrcola para converter-se numa po-
a guerra comear.
tncia industrial. Paralelamente indstria de bens de consu-
mo imediato, a Alemanha desenvolveu tambm gigantescas in-
dstrias de ao, armas e navios. Tornado-se na virada do sculo O incio do conflito
o maior produtor mundial de ao. Fortalecida desta maneira, a
Alemanha pressionava por uma nova partilha do mundo colo-
nial entre as potncias. A Inglaterra, por sua vez, na condio
de dona da maior poro do mundo colonial, mostrava-se dis-
posta a manter suas conquistas a qualquer custo.
Rivalidade Austro-Russa A Rssia foi um dos ltimos pases
europeus a ingressar na era industrial, apenas no ltimo quar-
tel do sculo XIX. Diante disso, a pretenso imperialista russa
foi muito prejudicada pelo avano dos demais pases europeus.
Para resolver esse impasse, o grupo dirigente russo almejava
dominar o decadente imprio Turco-Otomano, apossando-se
dos estreitos de Bsforo e Dardanelos, pois isso lhe daria uma
sada para o Mar Mediterrneo. Alm disso, aps eliminar o
Imprio Turco, pretendia colocar a Pennsula Balcnica sob seu
controle. Para justificar esse expansionismo, a Rssia, maior
nao eslava do mundo, criou o pan-eslavismo, um movimen-
to poltico de carter fortemente nacionalista. O pan-eslavismo
defendia a ideia de que a Rssia tinha o direito de defender e
proteger as pequenas naes eslavas do mundo. No interior do O incidente que deu inicio Primeira Guerra Mundial ocor-
Imprio Austro-Hngaro havia vrias naes eslavas que de- reu num domingo, 28 de Junho de 1914, em Sarajevo, Bsnia.
sejavam a independncia. Para consegu-la, esperavam receber Nesse dia, o herdeiro do trono austraco, Francisco Ferdinando,
ajuda da me-Rssia. Por isso, a Rssia e o Imprio Austro- e sua esposa, Sophia, foram assassinados a tiros por um estudante
Hngaro tornaram-se arquiinimigos. pertencente a uma organizao terrorista srvia que tinha como
Nacionalismo da Srvia A Srvia era uma pequena nao objetivo libertar a regio do domnio austraco.
eslava independente, situada na regio dos Blcs, que almejava A polcia austraca afirmou que o crime foi arquitetado na
Srvia. Com base nessa afirmao, em 28 de Julho de 1914, a

140 UECEVEST

Apostilas UECEVEST mod3.indb 140 06/02/2011 09:59:14


HISTRIA G E R A L I I

ustria declarou guerra Srvia, dando incio Primeira Guerra derrotados e obrigados a recuar. Com uma sucesso de derrotas
Mundial. A partir da, o que se viu foi uma reao em cadeia: a impostas pelo poder de fogo do exrcito norte-americano, o
Rssia, em apoio Srvia, mobilizou seu exrcito contra a us- governo imperial alemo caiu, e a repblica proclamada logo
tria; a Alemanha reagiu declarando guerra Rssia; a Inglaterra, em seguida se rendeu, fato que ps fim ao conflito.
por sua vez, declarou guerra Alemanha... Essas declaraes to-
das ocorreram em apenas sete dias. Consequncias da 1 guerra

As fases da guerra

Woodrow Wilson
Trincheira da 1 Guerra Mundial
Terminada a guerra, os governos da Frana e da Inglaterra fo-
Considerando os movimentos dos exrcitos, essa guerra pode ram implacveis, impondo vrios acordos de paz aos vencidos.
ser dividida em trs fases: O mais humilhante deles foi o Tratado de Versalhes, imposto
Guerra de Deslocamento (1914) Esse perodo caracterizou- Alemanha em 28 de Junho de 1919. O Tratado de Versalhes esta-
se por movimentos rpidos envolvendo grandes exrcitos. Certo beleceu que a Alemanha era obrigada a:
de que venceria a guerra em pouco tempo, o exrcito alemo Restituir a Alscia-Lorena Frana.
invadiu a Blgica. Armado de artilharia pesada venceu a resis- Ceder as minas de carvo do Sarre Frana por um prazo de
tncia belga e penetrou no territrio francs at as proximidades 15 anos.
de Paris. Os franceses reagiram e contra-atacaram conseguindo Ceder suas colnias, submarinos e navios mercantes Frana,
deter o avano alemo na Batalha do Marne. Aps isso, a guerra Inglaterra e Blgica.
estacionou, devido o equilbrio entre as partes envolvidas. Pagar aos vencedores, a ttulo de indenizao, a quantia de 33
Guerra de Trincheiras (1915-1916) Nesta fase, os exrcitos bilhes de dlares.
ficaram praticamente parados, pois defendiam suas posies Reduzir seu poderio blico, ficando proibido de possuir fora
originais utilizando-se de uma vasta rede de trincheiras que eles area, de fabricar armas e de ter um exrcito superior a 100 mil
prprios cavaram. Enquanto isso, na frente oriental, o exercito homens.
alemo impunha sucessivas vitrias sobre o mal-treinado e mal-
armado exrcito russo. Apesar disso, entretanto, no teve fle- Os pases vencedores tambm trataram de desmantelar os
go para conquistar a Rssia. Em 1915, a Itlia, que at ento imprios adversrios (turco-otomano e austro-hngaro) da re-
se mantivera neutra, traiu a aliana que fizera com a Alemanha sultando o surgimento de diversos novos pases, como a Iugos-
e entrou na guerra do lado da Entente, porque esperava ob- lvia, a Hungria, a Tchecoslovquia, a Turquia, Lbano, Sria,
ter territrios austracos ainda encravados na Itlia. Ao mesmo Jordnia, etc.
tempo em que foi se alastrando, o conflito tornou-se cada vez No entanto, as maiores consequncias da primeira Guerra fo-
mais trgico devido o uso de novas aramas, como por exemplo, ram o declnio da Europa (30 milhes de vtimas) e a ascenso
o canho de tiro rpido, o lana-chamas e o submarino. dos Estados Unidos, como a maior potncia econmica mun-
Guerra de Desfecho (1917-1918) Em 1917, primeiro ano dial. Alm disso, o aparecimento e regimes polticos autoritrios,
dessa nova fase, ocorreram dois fatos decisivos para o desfecho como o nazismo e o fascismo tambm podem ser relacionados
da guerra: a entrada dos Estados Unidos e a sada da Rssia. Os Primeira Guerra Mundial.
Estados Unidos entraram na guerra do lado da Entente. A ex- Atendendo sugesto do Presidente norte-americano Woo-
plicao simples: os americanos tinham feito grandes investi- drom Wilson, as naes vencedoras fundaram a Liga das Naes,
mentos nesses pases e queriam assegurar o seu retorno. Porm, em 28 de Abril de 1919. Com sede em Genebra, na Sua, essa
o que precipitou a deciso dos Estados Unidos foi o torpede- organizao internacional tinha como principal objetivo a pre-
amento de um navio da marinha mercante pelos submarinos servao da paz do mundo. Na prtica, porm, a Liga das Naes
alemes. Com suas indstrias e recursos humanos, os Estados se mostrou tendenciosa e incapaz de arbitrar os graves conflitos
Unidos contriburam de modo decisivo em favor dos pases surgidos aps a Primeira Guerra Mundial.
da Entente. Outras naes tambm se envolveram na guerra.
Turquia e Bulgria juntaram-se Trplice Aliana, enquanto,
Japo, Portugal, Romnia, Argentina, Canad, Grcia e Brasil E X E R C C I O
colocaram-se ao lado da Entente. A sada da Rssia do conflito
est relacionada revoluo socialista ocorrida em seu territ- 01. (IFCE 2004) Foi um acontecimento histrico que colaborou
rio no final de 1917. O novo governo alegou que a guerra era para o surgimento da Primeira Grande Guerra Mundial:
imperialista e que o seu pas tinha muitos problemas internos a) A Guerra do Chaco
para resolver. Com este cenrio, a Alemanha concentrou seu b) A Guerra do Pacfico entre Chile, Bolvia e Peru
exrcito em uma nica frente ocidental para um esforo final, c) A Crise de 1929
antes da chegada dos norte-americanos Europa, mas foram d) O Nazismo na Alemanha

UECEVEST 141

Apostilas UECEVEST mod3.indb 141 06/02/2011 09:59:15


H I S T RIA GERAL II

e) A Paz Armada e a Poltica de Alianas coletividade, condenava o conflito armado na poltica externa
02. (IFCE 2007/1) Em relao Primeira Grande Guerra Mun- 06. (UECE 2003/2) Sobre a Primeira Guerra Mundial correto
dial, correto afirmar-se que: afirmar:
a) foi provocada pela difuso das ideias iluministas; a) Os movimentos nacionalistas, como o Pan-Germanismo e o
b) resultou da primeira tentativa de implantao de uma Pan-Eslavismo buscavam uma soluo pacfica para o conflito.
sociedade comunista na Alemanha; b) A competio, entre as potncias, por mercados
c) foi fruto do imperialismo, onde as grandes potncias consumidores de produtos industriais foi um dos fatores que
disputavam a hegemonia econmica, colocando Alemanha e levaram Primeira Guerra Mundial.
Inglaterra numa tremenda rivalidade; c) Um dos efeitos mais duradouros da Primeira Guerra
d) constituiu-se num processo de afirmao das potncias Mundial foi o fortalecimento dos chamados Imprios: o
emergentes, como China e Estados Unidos; Austro-Hngaro, o Russo e o Imprio Otomano.
e) a definio da Guerra se deu com a entrada da Unio d) Ao sarem vencedoras da Guerra, Frana e Inglaterra tiveram
Sovitica (URSS) no confronto, tendo em vista a ruptura do suas economias fortalecidas e inauguraram um novo ciclo de
acordo de paz entre Alemanha e URSS. hegemonia e prosperidade para a Europa.

03. (UECE 2005/2) O Tratado de Versalhes encerrava oficial- 07. (UFC 2006) Analise as afirmaes abaixo sobre a Grande
mente os trabalhos da Conferncia de Paz de Paris (1919-1920), Guerra de 1914-1918 e suas consequncias.
convocada para deliberar sobre o fim da Primeira Guerra Mun- I. Embora sua ao militar tenha ocorrido sobretudo na
dial (1914-1918). O Brasil esteve presente a essa Conferncia e Europa, ela envolveu, direta ou indiretamente, grande
Epitcio Pessoa foi o chefe da Delegao Brasileira. (GARCIA, parte do mundo, e americanos, canadenses, australianos,
Eugnio Vargas. Paz para acabar com a Paz. Revista de Histria neozelandeses, indianos e africanos lutaram tambm na
da Biblioteca Nacional. pp. 35-39.) Acerca da participao do guerra, na Trplice Entente.
Brasil na citada Conferncia, correto afirmar que II. As grandes perdas humanas e a desestruturao
a) o Brasil centralizou as atenes da Conferncia, em virtude da produo atingiram todos os pases do mundo,
da participao fundamental que o pas ocupou no conflito provocando um enfraquecimento generalizado das
de 1914-1918. economias e um vazio de poder, com o fim da
b) os assuntos referentes Amrica do Sul foram amplamente hegemonia europeia.
debatidos nessa Conferncia de Paz de Versalhes. Em virtude III. Aps a guerra, uma onda revolucionria atravessou a Europa
desse fato, o Brasil foi um dos atores exponenciais do citado que, nas dcadas seguintes, vivenciou o colapso dos valores e
evento. instituies liberais, com a instalao de regimes autoritrios.
c) o Brasil criticou o fato de os assuntos europeus ocuparem
as pautas da Conferncia, deixando registrado seu protesto Com base nas trs assertivas, correto afirmar que somente:
em defesa dos pases alijados das principais deliberaes da a) I verdadeira. d) I e II so verdadeiras.
Conferncia. b) II verdadeira. e) I e III so verdadeiras.
d) Brasil e Estados Unidos adotaram posies divergentes c) III verdadeira.
nessa Conferncia o que enfraqueceu a defesa dos interesses
brasileiros e da Amrica como um todo. 08. (UECE 2006/1) Muito foi escrito sobre a Primeira Guerra
Mundial, particularmente sobre suas causas.... o historiador bri-
04. (UECE 2009/2) Estima-se que na Primeira Guerra Mundial tnico Anthony Taylor... chegou a afirmar a inutilidade... de bus-
(1914- 1918) cerca de 10 milhes de pessoas morreram no cam- car a origem dos fatos em vez de entender o que eles significam
po de batalha. Um aumento significativo em comparao com as e, por fim, o que representam. (Fonte: MESQUITA, Julio. A
demais guerras no sculo Guerra (1914-1918). So Paulo: O Estado de So Paulo/Editora
anterior. Sobre a Primeira Guerra Mundial correto afirmar que Terceiro Mundo, 2002, p.41. V.1.) Considerando o comentrio
a) foi um conflito restrito entre algumas naes europeias e que apresentado, correto afirmar:
no trouxe nenhuma inovao em armas e tcnicas de guerra. a) A consolidao do nacionalismo, no incio do sculo
b) esta guerra marcou as aes de bastidores, com batalhas e XX, quebrou o pacto de unio entre as principais naes
confrontos ocasionais. europeias, firmado na primeira metade do sculo XIX, aps
c) marcou uma nova fase blica com a introduo das a derrota de Napoleo Bonaparte
trincheiras, tcnicas e tticas para avanar nas linhas inimigas. b) Vrios foram os fatores que contriburam para o incio
d) teve curta durao e no representou mudana em relao do conflito, mas nenhum fator pode ser considerado,
aos conflitos anteriores. isoladamente, como a explicao certa, sem uma anlise do
processo histrico
05. (UECE 2004/1) O nacionalismo... ataca a democracia, de- c) A origem da guerra pode ser compreendida atravs da
mole o anticlericalismo, luta contra o socialismo e solapa o paci- tradicional disputa dos povos de cultura latina, como os
fismo, o humanitarismo e o internacionalismo... Declara termi- franceses e italianos, que rivalizavam com os eslavos e anglo-
nado o programa do liberalismo. (Fonte: Alfredo Rocco, 1914 saxes, ocupantes do sul da Europa
apud HOBSBAWM, Eric J. A Era dos Imprios: 1875-1914.Rio d) O litgio entre os pases catlicos e os protestantes, iniciado
de Janeiro: Paz e Terra, 1988, p.203.) Baseando-se no comentrio no sculo XVI, explica o processo de competio, no campo
apresentado, correto afirmar: poltico e econmico, que atingiu seu pice em 1914
a) O nacionalismo constitua uma manifestao poltica
organizada pela ideologia socialista contra a expanso capitalista 09. (UECE 2007/2) A Primeira Guerra Mundial foi uma das mais
b) O avano do nacionalismo constituiu um prejuzo sangrentas e dispendiosas guerras do mundo contemporneo.
concretizao dos valores democrticos Sabe-se que no foram, apenas, dois tiros de pistola, um nico
c) O surgimento do nacionalismo foi registrado em diversos pases, ato o assassinato do arquiduque Francisco Ferdinando e de sua
que se sentiam prejudicadas com a unificao da Alemanha mulher Sofia , o que assinalou o conflito. Inmeros outros fato-
d) O nacionalismo, por defender os interesses de uma res contriburam para essa guerra. Como fatores que contriburam
para a Primeira Guerra Mundial foram listados os seguintes:

142 UECEVEST

Apostilas UECEVEST mod3.indb 142 06/02/2011 09:59:15


HISTRIA G E R A L I I

franceses, sob a forma de emprstimos ou investimentos diretos.


I. Desde o sculo XIX, os povos dominados por outros Em consequncia disso, o nmero de fbricas multiplicou-se,
pases desenvolveram sentimentos nacionalistas. Alguns se acelerou-se a urbanizao e surgiram grandes ncleos industriais
agruparam em alianas militares e disputaram a posse das em cidades como Moscou e So Petersburgo. Esses centros passa-
colnias e de outras terras. ram a abrigar uma prspera burguesia industrial e um numeroso
II. A intensa rivalidade entre a Alemanha e a ustria-Hungria, operariado concentrado em torno de grandes fbricas. Vivendo
na disputa por mercados consumidores para a venda de em condies precrias, estes operrios, desalojados do campo
seus produtos industriais e a aquisio de matrias-primas, devido ao fim da servido, enfrentavam jornadas de at 12 horas
acirrou-se, tomando propores mundiais. de trabalho, sob disciplina rigorosa, em troca de salrios baixs-
III. Uma combinao de interesses geopolticos e uma dose simos. Alm disso, suas moradias e alimentao eram precrias.
de anarquia internacional resultaram na combinao Reagindo a esta situao opressiva, o operariado russo come-
de competies econmicas, chauvinismos nacionais e ou a promover greves, motins e passeatas para exigir melhores
rivalidades imperialistas. condies de vida e de trabalho. Surgiram partidos populares
fortemente influenciados pelas ideias socialistas trazidas da Eu-
Entretanto, correto afirmar que: ropa pelos intelectuais russos. O mais consistente dentre eles foi
a) Apenas o I contribuiu. o Partido Operrio Social Democrata Russo, fundado em 1898
b) Apenas o I e o III contriburam. por intelectuais marxistas. Duramente perseguido pela Okrana
c) Apenas o II e o III contriburam. (Polcia Poltica do Czar) o Partido Operrio foi desmantelado e
d) Apenas o I e o II contriburam. muitos de seus lderes foram obrigados a se exilar e comandar o
grupo de fora do pas. Em 1903, o partido foi dividido devido
10. (UECE 2008/2) Em relao participao do Brasil na Pri- a divergncias internas. Surgiram dois grupos: os bolcheviques
meira Guerra Mundial (1914-1918), pode-se afirmar, (maioria) e mencheviques (minoria). O quadro abaixo apresenta
corretamente, que algumas dessas diferenas entre os dois grupos:
a) o Brasil foi o nico pas da Amrica do Sul a participar
ativamente, do incio ao final do conflito, na qualidade de Mencheviques Bolcheviques
pas beligerante. Lder Martov Lnin
b) quando o conflito eclodiu, em 1914, o governo brasileiro Mtodos a obteno por meio A obteno do
adotou uma postura de neutralidade, mantendo-se assim at de uma aliana com a poder atravs da luta
o final do mesmo, em 1918. nascente burguesia. revolucionria.
c) os ataques de submarinos franceses contra navios brasileiros, Objetivo promover Promover uma
principalmente aps o torpedeamento do paquete Paran, prioritariamente uma revoluo socialista.
obrigou o Brasil a romper relaes diplomticas com os aliados. revoluo burguesa Esta devia ser
d) O Brasil, na prtica, teve uma participao simblica, para depois lutar para conduzida por um
porm, enviou matrias primas e suprimentos aos aliados conduzir a Rssia ao partido centralizado
bem como realizou operaes de patrulhamento no socialismo. que unisse soldados,
Atlntico Sul. operrios e camponeses.

O domingo sangrento
G A B A R I T O A situao da Rssia tornou-se ainda mais tensa aps a guerra
contra o Japo, entre os anos de 1904 e 1905. As sucessivas derro-
01. e 02. c 03. c 04. c 05. b tas militares exibiram a ineficincia do regime czarista e contribu-
06. b 07. e 08. b 09. b 10. d ram para aprofundar a crise interna, aumentando a insatisfao
popular. Para conter a onda de protestos, que vinham se avolu-
mando dia-a-dia, o governo simplesmente redobrava a violncia.
A 9 de Janeiro de 1905, em So Petersburgo, os soldados abriram
A REVOlUO RUSSA (UECE/EnEm) fogo contra os operrios que apenas pretendiam entregar uma
petio ao czar pedindo-lhe melhorias nas condies de vida.
A Revoluo Russa foi o movimento responsvel pela implan- Em decorrncia desse episdio, que ficou conhecido como o
tao da primeira experincia de um Estado Socialista de moldes Domingo Sangrento, explodiram greves, motins e rebelies so-
marxistas na histria. Contrrio expectativa de seus idealiza- ciais por todo o pas. Os militares tambm se sublevaram, dessa
dores, o primeiro governo de trabalhadores no foi implantado poca a clebre revolta dos marinheiros do encouraado Pote-
em um pas prioritariamente industrial, como a Inglaterra, por mkim, no Mar Negro, imortalizada pelo filme de Sergei Einsens-
exemplo, mas em uma monarquia absolutista de base rural. tein, de 1926. Todos esses movimentos rebeldes eram organiza-
dos e liderados por comits de soldados, operrios e camponeses,
Causas chamados Soviets, que nasceram e se multiplicaram rapidamente
Por volta de 1880, a Rssia era um pas com 22 milhes aps o domingo sangrento.
de km2 essencialmente agrrio. A maioria de sua populao era Disposto a diminuir a presso popular, o Czar Nicolau II
formada por camponeses recm-sados da servido (abolida em prometeu convocar a Duma, ou seja, uma assembleia legislativa
1861), que viviam em extrema penria, tendo de pagar altos im- que poria fim ao absolutismo dos czares, elaborando uma consti-
postos ao governo pelo uso da terra. O imperador era chamado tuio para o pas. A promessa conseguiu diminuir e enfraquecer
Czar e governava de modo absolutista, com apoio da nobreza as oposies. Meses depois, porm, com a situao sob controle,
rural e da Igreja Ortodoxa. o czar mandou reprimir violentamente os rebeldes e seus sobre-
A partir dessa poca, a realidade scio-econmica do pas viventes. A Duma foi dominada pelo czar. As rebelies populares
comeou a mudar: com mais de um sculo de atraso em rela- de 1905 no conseguiram derrubar o czarismo. Entretanto, se-
o Inglaterra, a Rssia comeou a se industrializar. Tal fato gundo a avaliao do prprio Lnin, Vladimir Ilitich Ulianov,
aconteceu prioritariamente devido entrada macia de capitais lder da Revoluo Russa, os eventos daquele ano serviram de

UECEVEST 143

Apostilas UECEVEST mod3.indb 143 06/02/2011 09:59:16


H I S T RIA GERAL II

ensaio geral para a revoluo.


A revoluo Russa de 1917
A participao desastrosa da Rssia na Primeira Guerra
Mundial multiplicou as tenses no interior da sociedade e pos-
sibilitou o surgimento de uma nova e forte onda revolucionria.
Iniciada a guerra, o governo do Czar Nicolau II enviou para
os campos de batalha um exrcito que, embora possusse 15
milhes de homens, era extremamente mal-treinado e no ti-
nha sequer fuzis, botas e cobertores suficientes para todos. Esse
exrcito sofreu contnuas derrotas frente aos alemes. Decorri-
dos dois anos e meio de guerras, j tinham morrido mais de trs
milhes de soldados russos e outros tantos haviam desertado. A
populao civil russa tambm padecia com a guerra: o sistema
de transportes entrara em colapso, a populao agrcola cara,
o desemprego aumentara, o preo dos alimentos disparara e a
fome atingira a maioria.
Diante dessa situao desesperadora, uma gigantesca onda de Em outubro, tendo frente Lnin e Trotsky, os bolcheviques
protestos sacudiu o pas. Uma passeata organizada por mulheres e ocuparam os principais edifcios pblicos de So Petersburgo e
operrios em Maro de 1917, na cidade de So Petersburgo, deu entregaram a chefia do governo Lnin, que passou a exercer o
incio ao movimento revolucionrio. A cavalaria czarista repri- cargo de Presidente do Conselho de Comissrios do Povo.
miu os manifestantes, mas unidades do exrcito os apoiaram. A Assim que assumiu o poder, o governo leninista adotou um
rebeldia alastrou-se rapidamente pelos grandes centros industriais conjunto de medidas de grande repercusso social:
e pelo interior da Rssia. A populao exigia o fim do czarismo. Enviou imediatamente um pedido de paz aos alemes. Em 3
Sete dias depois de iniciado o movimento, o czar abdicou. de Maro de 1918, pelo Tratado de Brest-Litovsky, assinou a
paz com a Alemanha.
Revoluo burguesa Revoluo branca Efetuou a reforma agrria. Confiscou milhes de hectares de
Logo aps a abdicao de Nicolau II, formou-se um governo terras dos grandes proprietrios e os distribuiu entre milhares
provisrio, presidido pelo Prncipe Lvov, mas comandado por de camponeses.
Alexander Kerensky, um poltico, que na prtica, mostrou-se Nacionalizou bancos, indstrias e estradas de ferro. Vrias des-
conservador e de carter burgus. J no incio de sua gesto, o sas empresas passaram a ser controladas por representantes dos
novo governo frustrou as expectativas populares, declarando que operrios.
a Rssia no sairia da guerra e nem aprovaria a reforma agrria
pretendida por milhes de camponeses. Por causa disso, recome- Guerra civil
aram as lutas populares, mais uma vez lideradas pelos Soviets, Em poucas semanas, os chamados russos brancos, inimigos
clulas revolucionrias bolcheviques. Vendo-se ameaado, o go- da revoluo socialista, montaram um numeroso exrcito sob
verno burgus fez algumas concesses: instituiu o direito de livre a liderana de antigos oficiais czaristas a fim de investir con-
expresso e associao, libertou os presos polticos e deu consen- tra o governo bolchevique. Para auxiliar os russos brancos, as
timento para que os exilados voltassem ao pas. grandes naes capitalistas, como a Inglaterra, Frana e Estados
Beneficiado por essa anistia, Lnin retornou do exlio em Unidos, desembarcaram tropas em diversos pontos do territrio
abril de 1917 e, no dia seguinte, exps as Teses de Abril, sinte- russo. Dispostos a resistir, os bolcheviques criaram o Exrcito
tizadas no slogan Paz, Terra e Po. Nesse documento, o mais Vermelho, cujo comando coube a Lev Davidovitch Braustein,
importante lder bolchevique afirmava que o governo provisrio conhecido como Trotsky. Em 1918, iniciou-se ento, uma san-
era imperialista e que, por isso, os Soviets deviam lutar para con- grenta guerra civil, que se prolongou por trs anos e foi vencida
quistar o poder imediatamente. pelo exrcito vermelho. Com a vitria sobre seus inimigos, os
Ao mesmo tempo em que o exrcito russo sofria novos reve- bolcheviques, organizados pelo Partido Comunista, firmaram-
ses no exterior, internamente o governo provisrio no tomava se no poder.
providncias para resolver os grandes problemas sociais. Conse-
quncia disso que os bolcheviques foram ganhando fora junto A NEP Nova Poltica Econmica
aos operrios, camponeses e soldados. Depois de duas guerras consecutivas uma externa e outra
interna a economia russa encontrava-se fortemente abalada,
Revoluo bolchevique registrando uma queda acentuada da produo agrcola e indus-
trial. Visando a reerguer o pas, Lnin adotou a Nova Poltica
Econmica (NEP), um programa econmico que apresentava ao
mesmo tempo medidas socialistas e capitalistas. A NEP permitia,
por exemplo, que as indstrias com menos de vinte funcion-
rios funcionassem em regime de empresa privada; os camponeses
comercializassem livremente as sobras das colheitas; os capitais
estrangeiros entrassem no pas sob a forma de emprstimos e fi-
nanciamentos diretos. O estado socialista, por sua vez, reservava-
se o direito de controlar o comrcio exterior, a rede bancria, o
sistema de transportes, comunicaes e a indstria de base. A
Nova Poltica Econmica surtiu o efeito esperado e o pas come-
ou a se reerguer economicamente.
Em Dezembro de 1922, um grande congresso, que reuniu di-
ferentes povos do extinto imprio russo, decidiu formar a Unio
das Repblicas Socialistas Soviticas (URSS), federao cujo

144 UECEVEST

Apostilas UECEVEST mod3.indb 144 06/02/2011 09:59:17


HISTRIA G E R A L I I

principal estado era a Rssia. tentar colocar os lderes polticos acima das instituies, como
aconteceu com a figura de Josif Stalin, consagrado pela hist-
Stalinismo ria oficial como um dos grandes agentes da Revoluo de 1917.
Hoje, graas a divulgao de novas verses, correto afirmar:
a) Stalin foi condenado devido ascenso do trotskismo ao
poder, atravs da vitria de Nikita Krushev na Unio das
Repblicas Socialistas Soviticas.
b) Stalin subiu ao poder em virtude de uma barreira criada
pelas lideranas russas, que consideravam Trotsky um
elemento prepotente e ardiloso.
c) A indicao de Stalin, para assumir o poder aps a morte de
Lnin, resultou de uma eleio direta realizada, em virtude
de uma exigncia das velhas lideranas partidrias.
d) A estimativa de milhes de pessoas mortas durante o
Vladimir Lenin Leon Trotsky Joseph Stalin processo de coletivizao do campo constitui uma verso
deturpada da histria da Rssia.
Em 1924, com a morte de Lnin, assistiu-se na URSS a uma 03. (UECE 2006/1) Petersburgo est rodeada por um anel de
terrvel luta pelo poder entre dois lderes bolcheviques: Trotsky e fbricas de muitas chamins. Como um enxame, muitos milha-
Stalin. Trotsky, o organizador do Exrcito Vermelho, defendia a res de pessoas a elas se dirigem de manh, e todos os subrbios
ideia de que era necessrio expandir o socialismo para outros pa- esto vazios. Todas as fbricas experimentavam agora (outubro
ses. Segundo ele, se a Rssia ficasse isolada, as grandes naes ca- de 1905) um estado de terrvel agitao. Os trabalhadores se
pitalistas liquidariam a revoluo. J Stalin afirmava que a URSS transformavam em sombras tagarelas. Entre elas circulavam re-
deveria primeiro se consolidar como nao socialista para, depois, vlveres Browning. E alguma coisa mais. (FONTE: BERMAN,
influenciar outros pases a fazerem sua revoluo. Marshall. Tudo que slido desmancha no ar: a aventura da
Stalin, cujo nome verdadeiro era Jossip Vissiarovitch Douga- modernidade. So Paulo: Companhia das Letras, 1986, p. 243.)
chivili, venceu a disputa e instalou-se no poder por quase trinta De acordo com o contedo do comentrio, acima apresentado,
anos, de 1924 at 1953, ano de sua morte. Durante esse tempo, correto afirmar:
implantou uma ditadura brutal e policialesca. A seu mando mi- a) Petersburgo, a partir do incio do sculo XX, tornou-se o
lhares de pessoas, entre elas centenas de lderes socialistas, foram principal plo industrial da Europa.
fuzilados ou exilados nas terras frias na Sibria. Trotsky tambm b) A pujana de Petersburgo decorreu da subordinao da
foi uma vtima fatal da represso stalinista. Mesmo tendo fugido, Rssia ao imperialismo americano.
foi morto a golpes de marreta por um emissrio do Grande Guia c) A efervescncia das manifestaes operrias resultou do
do Socialismo. declnio da nobreza rural, aps a Revoluo Comunista.
No plano econmico, Stalin suprimiu a NEP, adotou os chama- d) Petersburgo se tornou um grande centro industrial, onde o
dos planos quinquenais . Por meio deles, a Unio Sovitica desen- processo de modernizao incrementou o fortalecimento do
volveu-se rapidamente, tornando-se uma grande potncia mundial. movimento reivindicatrio operrio.

04. (IFCE 2006) Sobre a Revoluo Russa de 1917, correto


E X E R C C I O afirmar:
a) resultou no fortalecimento do regime czarista
01. (IFCE 2005) Um dos acontecimentos mais significativos do b) contribuiu para o enfraquecimento do Partido Bolchevique
sculo XX foi a Revoluo Socialista na Rssia, em 1917, por c) os bolcheviques defendiam a necessidade de uma revoluo
colocar em xeque a ordem socioeconmica capitalista. Com res- gradual e de reformas
peito ao desencadeamento do processo revolucionrio, correto d) a derrota da Rssia, na disputa pela regio da Manchria,
afirmar que: retardou o processo revolucionrio de Outubro de 1917
a) a participao da Rssia, na Primeira Guerra Mundial, e) contribuiu para uma redefinio dos quadros polticos e
desencadeou uma srie de greves e de revoltas populares em sociais com instalao de um estado socialista
razo da crise de abastecimento de alimentos, provocando o
incio do movimento 05. (UEL 2000) O GOVERNO PROVISRIO FOI DEPOS-
b) os mencheviques tiveram um papel fundamental no TO; a maioria de seus membros est presa. O poder sovitico
processo revolucionrio, por defenderem a implantao das propor uma paz democrtica imediata a todas as naes. Ele
Teses de Abril que consistiam, dentre outras exigncias, na proceder entrega aos comits camponeses dos bens dos grandes
reforma agrria, na retirada do pas da guerra e na entrega do proprietrios, da Coroa e da Igreja... Ele estabelecer o controle
poder aos sovietes operrio sobre a produo, garantir a convocao da Assembleia
c) os bolcheviques representavam a ala mais conservadora dos Constituinte para a data marcada... garantir a todas as nacio-
socialistas, chegando a ocupar o poder com a Revoluo de nalidades que vivem na Rssia o direito absoluto de disporem
Fevereiro de 1917, atravs de Alexander Kerenski de si mesmas. (O Congresso dos Sovietes proclama a queda do
d) Stalin, a partir de outubro de 1917, estabeleceu a tese de Governo Provisrio. Apud FERRO, Marc. A revoluo russa de
que era necessria a revoluo em um s pas, em oposio a 1917. So Paulo: Perspectiva, 1974. p.126.) Com base no docu-
Trotsky, lder do exrcito vermelho mento acima e nos conhecimentos sobre a Revoluo Russa,
e) o governo revolucionrio de Stlin conseguiu superar os correto afirmar:
conflitos que existiam no seu interior, quando estabeleceu a a) O texto caracteriza o ensaio revolucionrio ocorrido na
Nova Poltica Econmica que representava os interesses dos Rssia em 1905, que teve incio com o episdio conhecido
setores mais conservadores como o Domingo Sangrento.
b) A destituio do Governo Provisrio devolveu aristocracia
02. (UECE 2005/1) Um grave erro na historiografia tradicional as terras desapropriadas durante a sublevao popular.

UECEVEST 145

Apostilas UECEVEST mod3.indb 145 06/02/2011 09:59:18


H I S T RIA GERAL II

c) O documento destaca a principal contradio da atuao a) Estabelecer prticas de planejamento estatal no Primeiro
poltica dos integrantes dos sovietes: a manuteno a Plano Quinquenal sovitico.
qualquer preo do esforo de guerra contra os alemes. b) Implantar uma economia exclusivamente comunista para
d) O Congresso dos Sovietes de operrios, soldados e vencer a crise aps a Primeira Guerra Mundial.
camponeses, que proclamou o documento acima, colocou c) Adotar algumas prticas capitalistas deplanejamento estatal
em prtica a tese Todo poder aos sovietes, defendida por aplicadas na Alemanha aps o Tratado de Versalhes.
vrios revolucionrios russos. d) Dar um passo para trs e dois para frente na Nova Poltica
e) A intolerncia russa em relao s outras nacionalidades Econmica (NEP).
pode ser percebida no documento que aclamou a derrubada e) Formar o Mercado Comum do Leste Europeu, denominado
do Governo Provisrio. de COMECON.

06. (UNIFOR 2003/2) Leia atentamente o texto redigido por 08. (PUCMG 2008) Em outubro de 1917, os bolcheviques assu-
Lnin em 1917. Uma revoluo, uma revoluo real, profun- miram o poder na Rssia. A Revoluo Russa de 1917 anunciou
da, do povo, para usar a expresso de Marx, o processo incri- o fim do capitalismo e o incio do comunismo em escala plane-
velmente complicado e penoso de morte de uma velha ordem tria. Sobre a Revoluo Russa e a consolidao do socialismo
social e o nascimento de uma nova, o ajustamento das vidas de sovitico, todas as afirmativas esto corretas, exceto:
dezenas de milhares de pessoas. Uma revoluo a mais aguda, a) Revelou-se um movimento de carter radical, visto que
mais furiosa e desesperada luta de classes e guerra civil. Nenhu- morreram milhares de homens defendendo suas posies e
ma grande revoluo da histria escapou da guerra civil. Se no impondo um sacrifcio populao russa em nome de uma
houvesse circunstncias excepcionalmente complicadas, no ha- revoluo social.
veria revoluo. Quem teme os lobos no vai floresta. (Leo b) Foi um movimento de ruptura no processo do antigo
Huberman. Histria da riqueza do homem. Trad. Rio de Janeiro: Imprio Russo. A demolio quase instantnea do regime
Zahar, 1979. p. 286) No II Congresso do Partido Operrio So- czarista significou uma mudana no destino da Rssia e da
cial-Democrata Russo, definiram-se dois grupos com diferentes Europa.
projetos e concepes de revoluo. Sobre as principais divergn- c) Revelou-se como um movimento perverso. A ascenso
cias entre os mencheviques, liderados por Martov e Plekanov, e do comunismo demonstrou um socialismo com regime
os bolcheviques, sob a direo de Lnin, correto afirmar que os autoritrio comparvel aos governos totalitrios da Europa.
mencheviques d) Foi um movimento isolado no processo de modernizao da
a) aspiravam realizar inicialmente uma revoluo burguesa Rssia empreendido pelo Czar, refletiu os anseios do grupo
com o apoio dos operrios e camponeses e os bolcheviques dos camponeses pela coletivizao da terra.
procuravam empreender diretamente uma ditadura do
proletariado com o respaldo da vanguarda revolucionria. 09. (UFG 2007) Duas revolues marcaram o mundo ocidental:
b) admitiam somente solues pacficas e democrticas a Revoluo Francesa, ocorrida em 1789, e a Revoluo Russa,
para romper com a velha estrutura social, enquanto os ocorrida em 1917. Sobre estas revolues, INCORRETO afir-
bolcheviques buscavam empreender lutas e revoltas armadas mar que, em ambas,
para defender os interesses burgueses. a) as nobrezas feudais recuperaram rapidamente o seu prestgio
c) e os bolcheviques buscavam caminhos e estratgias um e poder poltico: na Frana, a restaurao feudal ocorreu
tanto semelhantes para empreender a revoluo, a diferena durante o governo de Napoleo Bonaparte; na Rssia, no
fundamental estava na constituio social desses grupos: o governo de Stlin.
primeiro era de origem proletria e o segundo de origem b) duas rainhas de origem alem serviram como smbolo do
burguesa. descontentamento popular contra a Monarquia absolutista:
d) defendiam o liberalismo, pois acreditavam que o sistema a rainha Maria Antonieta, na Frana, e a rainha Alexandra,
capitalista constitua uma etapa fundamental no processo de na Rssia.
instituio do comunismo, e os bolcheviques encampavam c) fortes crises econmicas antecederam os processos
os princpios da social-democracia. revolucionrios, encarecendo o preo dos alimentos e acirrando
e) identificavam-se com os princpios e teses anarquistas, o descontentamento dos camponeses e das massas urbanas.
particularmente os defendidos por Bakthin, enquanto que os d) houve a execuoa dos monarcas e o aniquilamento dos
bolcheviques, menos radicais, eram simpatizantes dos ideais regimes absolutistas: na Frana, com a decapitao do rei
iluministas de Jean- Jacques Rosseau. Luis XVI; na Rssia, com o fuzilamento do czar Nicolau II.

07. (UFPEL 2006/1) A extrema runa, agravada pela m colhei- 10. (UFAC 2003) Um dos momentos mais extraordinrios, du-
ta de 1920, fez com que a mudana se tornasse necessria com rante a I Guerra Mundial, foi a ecloso de um poderoso e espon-
a mxima urgncia, diante da impossibilidade de se estabelecer tneo levante popular na Rssia czarista, culminando na chama-
com rapidez a grande indstria. [...] A luta contra a especulao da Revoluo Russa e na constituio de uma alternativa de
deve ser transformada em luta contra os roubos e contra o modo poder que desafiava a lgica dos estados capitalistas da Europa
de enganar a vigilncia, o registro e o controle do Estado. Com ocidental e da Amrica do Norte. Dentre as principais bandeiras
esse controle dirigiremos o capitalismo, em certo grau indispen- e reivindicaes dos pobres das cidades russas, dos trabalhadores
svel e imprescindvel para ns, para a vida do capitalismo de Es- agrcolas e dos operrios, responsveis diretos pela revoluo que
tado. ([...] LENIN, Sobre o imposto em espcie. 21 de abril de derrubou a secular monarquia, podemos destacar:
1921.) Ao final da Guerra Civil (1918-1921), a Rssia encon- a) Po, Emprego e Terra
trava-se completamente arrasada. A tarefa dos bolcheviques era b) Po, Paz e Terra
extraordinariamente grande e dificilmente se conseguiria implan- c) Po, Paz e Independncia Nacional
tar o socialismo nas runas e escombros. (MARQUES, Adhemar d) Po, Paz e Fim da Guerra
Martins. Histria Contempornea atravs de textos. 10 Ed. So e) Po, Terra e Independncia Nacional
Paulo: Contexto, 2003.) Os textos favorecem a compreenso do
projeto de Lenin, que foi caracterizado pela poltica de

146 UECEVEST

Apostilas UECEVEST mod3.indb 146 06/02/2011 09:59:18


HISTRIA G E R A L I I

G A B A R I T O
01. a 02. b 03. d 04. e 05. d
06. a 07. d 08. d 09. a 10. b

REFERnCIAS BIBlIOGRFICAS

AQUINO, Rubim Santos Leo de Aquino; FRANCO, Denize


de Azevedo; LOPES, Oscar Guilherme P.C. Histria das Socie-
dades. Das Sociedades Modernas s Sociedades Atuais. 36
Ed. Rio de Janeiro: Editora

ARRUDA, Jos Jobson de A. Toda a Histria: Histria Geral e


Histria do Brasil. So Paulo: tica, 2007.
AZEVEDO, Gislaine; SERIACOPI, Reinaldo. Histria Srie
Brasil. So Paulo: Editora tica, 2005.

BRAICK, Patrcia R.; MOTA, Myriam B. Histria das cavernas


ao terceiro milnio. 3 Ed., So Paulo: Moderna, 2007.

BURNS, Edward McNall; LERNER, Robert; MEACHAM,


Standish. Histria da Civilizao Ocidental. Do Homem das
Cavernas s naves espaciais. Vol . II 40 ed. So Paulo: Globo,
2000.

CAMPOS, Flvio de; MIRANDA, Renan Garcia. A Escrita da


Histria. So Paulo:Escala Educacional, 2005.

COTRIM, Gilberto. Histria Global Brasil e Geral. 7 Ed.,


So Paulo: Saraiva, 2003.

ORDOES, Marlene; QUEVEDO, Jlio. Horizonte da His-


tria. Histria para o ensino mdio. So Paulo: IBEP, 2005.

SCHMIDT, Mario. Nova Histria Crtica. So Paulo: Nova


Gerao, 2005.

VICENTINO, Claudio; DORIGO, Gianpaolo. Histria para o


ensino mdio. So Paulo: Scipione, 2006.l

UECEVEST 147

Apostilas UECEVEST mod3.indb 147 06/02/2011 09:59:19


Apostilas UECEVEST mod3.indb 148 06/02/2011 09:59:19
P R - V E S T I B U l A R

HISTRIA DO BRASIL

Apostilas UECEVEST mod3.indb 149 06/02/2011 09:59:25


Caro(a) Aluno(a),

Para facilitar o acompanhamento de tais contedos, abaixo esto indicadas as nomenclaturas utilizadas
pela UECE e pelo ENEM:

UECE ENEM
Repblica Velha I poltica dos governadores, Economia agro-exportadora
oligarquia e coronelismo; economia brasileira: a borracha na Amaznia.
agrrioexportadora; A industrializao brasileira, a
industrializao e urbanizao: urbanizao e as transformaes
remodelao e disciplinarizao do sociais e
espao urbano; trabalhistas.
Repblica Velha II Cangaceirismo, Canudos, A luta pela conquista de direitos pelos
Contestado, Revolta da Vacina, cidados: direitos civis, humanos,
Revolta da Chibata e a Sedio de polticos e sociais. Direitos sociais
Juazeiro do Norte; a Coluna Prestes. nas constituies brasileiras. Polticas
afirmativas.
Era Vargas Anos 30 e Estado Novo: Estado e A luta pela conquista de direitos pelos
sociedade: legislao trabalhista e cidados: direitos civis, humanos,
corporativismo; Revoluo polticos e sociais. Direitos sociais
Constitucionalista de So Paulo; nas constituies brasileiras. Polticas
Integralismo e Igreja Catlica; afirmativas.
Aliana Nacional Libertadora (ANL)
ea Ditaduras polticas na Amrica
Revoluo Comunista de 1935; Latina: Estado Novo no Brasil
redemocratizao e reorganizao
partidria.

Apostilas UECEVEST mod3.indb 150 06/02/2011 09:59:26


HISTRIA D O B R A S I L

REPBlICA VElHA ASPECTOS A constituio de 1891


POlTICOS E ECOnmICOS (UECE/EnEm) A primeira Constituio republicana foi promulgada em 24
de fevereiro de 1891. Estabeleceu como forma de governo o regi-
Em 15 de Novembro de 1889, foi proclamada a Repblica, me representativo, em que o povo exerce o poder indiretamente,
atravs de um golpe liderado pelo Marechal Deodoro da Fonseca. atravs de seus representantes, eleitos em pleito direto por to-
A primeira etapa da histria republicana brasileira chamada de dos os cidados do sexo masculino, alfabetizados e maiores de
Repblica Velha (1889-1930). O termo foi usado originalmente 21 anos. Ou seja, mulheres e analfabetos estavam excludos do
com o objetivo de diferenciar aquele momento contemporneo, processo. Alm de religiosos e militares de baixa patente. Quanto
do que se vive atualmente. No entanto, a palavra velha tambm forma de votao, importante salientar que o voto era aberto:
pode ser aplicada devido ao carter pejorativo, ou seja, algo an- quando uma pessoa votava, todos ficavam sabendo quais eram os
tigo, ultrapassado, superado. Afinal, este um perodo marcado seus candidatos. O Presidente da Repblica tambm seria eleito
pela corrupo, pela manipulao eleitoral, pela violncia e pelo por voto direto para um mandato de quatro anos. O Estado pas-
desrespeito opinio pblica. saria a adotar o sistema federativo, copiado dos Estados Unidos,
Denomina-se Repblica da Espada o perodo compreendido no qual os estados tm maior autonomia, podendo at mesmo
entre 1889 e 1894, porque o Brasil, nessa poca, foi governado contrair emprstimos no exterior, estabelecer impostos e organi-
por dois militares: Deodoro da Fonseca e Floriano Peixoto. zar sua poltica interna. A administrao pblica ficou estrutura-
da em trs poderes:
Deodoro da Fonseca (1889-1891) Poder Executivo Exercido pelo Presidente da Repblica,
O Governo Provisrio, cuja presidncia coube ao Marechal pelo vice-presidente e pelos ministros; nos Estados, o titular do
Deodoro da Fonseca, tratou logo de organizar o novo regime. Poder Executivo era o presidente de Estado, como era chama-
Passou a governar por decretos-leis (atos legislativos do Poder do naquela poca o governador.
Executivo), at que fosse promulgada a nova Constituio, pois Poder Legislativo Exercido pelo Congresso Nacional, for-
a de 1824 deixara de vigorar com a Proclamao da Repblica. mado por duas cmaras a Cmara Alta ou Senado e a Cmara
O governo provisrio tomou as seguintes providncias: expulso Baixa ou Cmara dos Deputados, cujos titulares eram eleitos
do Brasil de D. Pedro II e a famlia real; transformao das pro- por voto direto; em nvel estadual, o Poder Legislativo era exer-
vncias em estados; separao entre Igreja e Estado; instituio cido pelas Assembleias Legislativas.
do casamento civil e registro de nascimento; naturalizao de es- Poder Judicirio Cujo principal rgo era o Supremo Tri-
trangeiros; adoo de uma nova bandeira e convocao de uma bunal Federal, secundado por juzes e tribunais federais nas di-
assembleia constituinte. ferentes regies brasileiras; nos Estados, tal poder era exercido
pelos tribunais e pelos juzes estaduais.
Crise do encilhamento
O governo constitucional de Deodoro (1891)
Venceram as primeiras eleies republicanas, para Presidente,
Deodoro da Fonseca, e para Vice-Presidente, Floriano Peixoto.
Como Deodoro no representava a classe que detinha o poder, os
grandes latifundirios do caf, teve de enfrentar violenta oposio
no Congresso Nacional e crticas a seus projetos. No dia 3 de Se-
tembro de 1891, dissolveu o Congresso e, para isso, contou com o
apoio dos governos estaduais. Mas civis e militares de estados pol-
tica e economicamente fortes, reagiram ao golpe. O Presidente, te-
mendo uma guerra civil, renunciou em 23 de Novembro de 1891.
Como a Constituio estabelecia, assumiu o poder o vice, Floriano
Peixoto, que encontrou o pas em grave crise financeira e poltica.

Gravura que representa a especulao no mercado de aes Floriano Peixoto (1891-1894)


Ao assumir, Floriano reabriu o Congresso e substituiu os go-
A poltica econmica do governo provisrio visava a promo- vernos dos estados adeptos de Deodoro. Setores das Foras Ar-
ver uma industrializao no pas. O Ministro da Fazenda, Ruy madas exigiam do Presidente a realizao de novas eleies. A
Barbosa, props aumentar a quantidade de moeda em circulao. Constituio era ambgua no tocante renncia do Presidente
Afirmava que, ampliando o crdito, ocorreria uma natural diver- da Repblica. Em seu artigo 42 dizia: Se, no caso de vaga por
sificao da economia e o desenvolvimento da indstria e do co- qualquer causa, da presidncia ou vice-presidncia, no houver
mrcio. O decreto, assinado em Janeiro de 1890, regulamentava ainda decorridos dois anos do perodo presidencial, proceder-se-
as emisses bancrias, determinando uma reforma financeira no nova eleio. Por este artigo constitucional, o vice-presidente
Brasil. Foram feitas grandes emisses de moedas sem o equiva- deveria convocar novas eleies para a presidncia da Repblica.
lente lastro. Essa moeda seria dada, sob a forma de emprstimos Entretanto, o Marechal Floriano Peixoto, vice-presidente,
bancrios, a indivduos que quisessem iniciar uma empresa. A baseou-se no artigo 11 das disposies transitrias da Constitui-
nica exigncia do banco para conceder o emprstimo era o aval o para assumir a presidncia. Esse artigo, aps estabelecer que o
pelas aes da nova empresa. Contudo, o que houve foi uma presidente e o vice do primeiro perodo republicano seriam eleitos
corrida especulativa, pois elementos aproveitadores criavam em- pelo Congresso, afirma em seu pargrafo 22 O presidente e o vi-
presas que no existiam, seno no papel, ou utilizavam apenas ce-presidente, eleitos na forma deste artigo, ocuparo a presidn-
uma parte do capital que haviam tomado emprestado. O clima cia e a vice-presidncia durante o primeiro perodo presidencial.
de euforia tomou conta at mesmo da Bolsa de Valores, onde as A interpretao de Floriano Peixoto foi contestada por treze
empresas fictcias eram vendidas a preos completamente irreais, generais que exigiram a convocao de eleies presidenciais, e
levando os incautos a perder suas economias. Alm disso, a gran- por civis e militares que queriam a volta de Deodoro ao governo.
de quantidade de dinheiro em circulao levou a uma enorme A uns e outros, o Marechal puniu, reformando os generais e des-
onda inflacionria, que acabou por aumentar o custo de vida. terrando civis e militares para as fronteiras da Amaznia.

UECEVEST 151

Apostilas UECEVEST mod3.indb 151 06/02/2011 09:59:26


H I S T RIA DO BRASIL

Floriano Peixoto enfrentou ainda vrias revoltas contra o seu Presidente


governo, principalmente a Revoluo Federalista do Rio Gran-
de do Sul e a Revolta da Armada, ambas em 1893. Ao derrotar
as tropas federalistas e as da Armada, os governistas cometeram
muitas atrocidades e fuzilamentos sumrios nos Estados do Sul, Governadores
especialmente em Santa Catarina.
Floriano conseguiu, pela fora, manter e consolidar a Rep-
blica, com o apoio de parte do exrcito e do povo do Rio de Ja-
neiro, que havia sido beneficiado com o tabelamento dos preos Coronis
da carne e dos aluguis. Ficou conhecido como Marechal de
Ferro, por causa da sua ao repressiva. No final de seu manda- Para substitu-lo no cargo de Presidente da Repblica, Pru-
to, o Partido Republicano Paulista (PRP) decidiu lanar como dente de Moraes lanou a candidatura de outro paulista, Campos
candidato presidncia, o paulista Prudente de Moraes, repre- Sales, que governou at 1902. Campos Sales preocupou-se com
sentante dos cafeicultores. Sua eleio ps fim Repblica da o saneamento financeiro da nao. Assinou um acordo interna-
Espada e deu incio a uma nova fase da histria republicana: a cional, chamado Funding Loan, obtendo os emprstimos que
Repblica Oligrquica, tambm conhecida como Repblica dos necessitava para reequilibrar financeiramente o pas. Com isso,
Coronis, ou Repblica do Caf com Leite. resolvia por algum tempo a situao difcil do Tesouro Nacional
e do mercado cambial. Os credores impuseram condies severas.
Repblica Oligrquica (1894-1930) Bens pblicos foram alienados, impostos foram aumentados, ao
mesmo tempo em que uma parte da moeda foi tirada de circu-
lao. O Ministro da Fazenda, Joaquim Murtinho, incumbiu-se
de pr em prtica as medidas saneadoras. A reao foi violenta.
Foi um tratamento de choque em um paciente que agonizava
financeiramente. Mas, com a entrada de capitais estrangeiros,
o oramento foi finalmente equilibrado, e a confiana voltou a
conviver com as iniciativas governamentais desacreditadas desde
a poca do encilhamento.
No plano poltico, Campos Sales, para obter o apoio do Con-
gresso para o seu programa econmico, no hesitou em propor
a Poltica dos governadores, um acerto poltico entre o governo
federal e as oligarquias regionais. Por poltica, o governo federal
se comprometia a apoiar os governos estaduais, no interferindo
na poltica local, e, em troca, esses governos estaduais se com-
prometiam a apoiar o governo federal. O presidente prestigiava
Consolidando o regime republicano,os cafeicultores paulistas o reconhecimento dos deputados e senadores federais indicados
trataram de organizar o Estado de forma que atendesse a seus pelos governos dos estados. Isso explica a degola dos candidatos
interesses. A poltica nacional estava nas mos dos Estados mais de oposio. No congresso, foi criada a Comisso Verificadora,
poderosos, So Paulo e Minas. No plano federal, as bancadas que era uma comisso que averiguava a apurao e a lisura das
paulistas e mineiras eram imbatveis, enquanto aliadas. Os can- eleies. Na verdade, era um rgo a servio do executivo, pois s
didatos presidncia da Repblica eram impostos, apoiados na neomearia os candidatos que fossem aprovados pelo governo fe-
pujana dos dois Estados grandes. Os Estados mdios eram deral. Dessa forma, o coronel lanava o candidato do seu partido,
contidos pelo uso de interveno federal. acertava com o presidente do estado, e a populao confirmava,
As eleies no tinham carter competitivo. Do acordo entre j que o voto era aberto. Se, por acaso, um candidato da oposio
paulistas e mineiros era indicado o candidato eleito a Presidente conseguisse ser eleito, no chegaria sequer tomar posse, porque a
da Repblica. Processava-se a poltica do caf-com-leite, com o comisso verificadora o cassaria, acusado de corrupo.
poder tranquilamente controlado pela oligarquia cafeeira. Houve A maior beneficiada, em nvel federal foi a oligarquia pau-
casos em que ocorreram competies, geralmente provenientes lista. Por isso, no hesitaram em indicar a sugesto de um outro
da separao entre So Paulo e Minas. paulista para a Presidncia, Rodrigues Alves. Os polticos das de-
Toda a poltica era feita atravs dos partidos Republicanos mais oligarquias se sentiram prejudicados com esse arranjo, mas
estaduais. Alguns eram originrios ainda do Imprio, outras se Campos Sales buscou equilibrar a situao conseguindo o apoio
formaram aps a Repblica. No possua verdadeira representati- dos mineiros, fazendo uma aliana com Minas Gerais para que
vidade popular, pois eram redutos das oligarquias. Os votos eram o prximo presidente fosse um mineiro. Surgia assim, a poltica
conseguidos atravs da ao dos coronis, chefes de poder local caf-com-leite, que era a unio entre So Paulo e Minas Gerais.
e cabos eleitorais. Os coronis exigiam das pessoas que depen- Para as demais oligarquias, o Executivo se comprometia a honrar
diam economicamente deles que votassem no candidato que eles os compromisso com a Poltica dos Governadores.
indicassem. Essa prtica ficou conhecida como voto de cabresto, Em 1906, cumprindo o acordo estabelecido pelos mineiros,
expresso popular que significava voto imposto, obrigado. Rodrigues Alves apoiou para a presidncia a candidatura de Afonso
Por meio da fraude, da violncia ou dos favores, o coronel Pena. Mineiro, era um fiel representante da poltica do caf-com-
conseguia eleger o prefeito, os vereadores, os deputados, os se- leite, criando um mecanismo de valorizao artificial do preo do
nadores, o presidente do estado (atual governador) e at mesmo caf, chamada Caixa de Converso.A presidncia de Afonso Pena
o presidente. O prestgio do coronel dependia do nmero de desenvolveu-se de 1906 a 1909, pois faleceu subitamente antes
eleitores que tinha sob seu controle. Esse fenmeno poltico ficou de completar o mandato. Nilo Peanha, vice eleito pela chapa de
conhecido como coronelismo. Dependia de um compromisso Afonso Pena, assumiu a presidncia apenas para encerrar o manda-
assegurado entre o coronel e a populao, uma verdadeira troca to de seu antecessor e organizar novas eleies. Mesmo assim, criou
de favores e influncias. o Servio de Proteo ao ndio (SPI), sob o comando de Cndido
Mariano da Silva Rondon e as Escolas de Artfices, predecessores

152 UECEVEST

Apostilas UECEVEST mod3.indb 152 06/02/2011 09:59:27


HISTRIA D O B R A S I L

dos Centros Federais de Educao Tecnolgica. Paulistas e minei- O caf brasileiro dominou o mercado mundial e os seus maio-
ros desentenderam-se sobre a indicao para o prximo presidente. res consumidores eram os Estados Unidos e a Europa. Essa situa-
A sucesso presidencial inflamou-se pela primeira vez na histria o estimulou os cafeicultores, que tinham a seu favor a abundn-
da Repblica Velha. A poltica do caf-com-leite foi rompida cia de terras e a mo-de-obra barata, a expandir as exportaes.
graas habilidade de Pinheiro Machado. Hermes da Fonseca era Alm da populao rural, contaram com o trabalho do imigrante,
apoiado por Minas, Rio Grande do Sul e por Nilo Peanha. So que se instalou principalmente em So Paulo, onde havia uma
Paulo apoiou o candidato baiano Rui Barbosa. Rui desenvolveu poltica oficial para atra-los. Em razo do grande volume da pro-
a Campanha Civilista. Dono de uma coragem imensa e de uma duo de caf, houve, nos primeiros do Sculo XX, uma crise
oratria extraordinria, pregava as reformas com as quais pensava de superproduo e consequente queda de seu preo. Isso levou
transformar o processo poltico. Defendia o voto secreto, at ento os cafeicultores paulistas, mineiros e fluminenses a assinarem o
inexistente, ttulo eleitoral, cdigo civil e a reforma da Constituio Convnio de Taubat, em 1906. Por esse acordo, o governo se
de 1891. O apoio macio das oligarquias, a fraude e a presso fez comprometia a comprar o excedente da produo para estocar e
Hermes da Fonseca o novo presidente. melhores preos no mercado. O governo tentou tambm contro-
Desde a poca de Deodoro e Floriano, as Foras Armadas lar a expanso da lavoura cafeeira, taxando novas plantaes. O
tinham sido alijadas do poder. Esta volta deu-se no governo de preo do caf voltou a subir e os altos lucros estimularam os ca-
Hermes da Fonseca, de 1910 a 1914. Pinheiro Machado, hbil feicultores a continuar investindo no produto. Quando preo do
poltico gacho era a eminncia parda do regime. Havia fundado caf caia, o governo emitia dinheiro e fazia emprstimos externos
um partido poltico para dar respaldo ao presidente que domi- para comprar o excedente da produo. Com isso, a moeda era
nava as bancadas minoritrias no Congresso Nacional. Hermes desvalorizada, o que provocava a lata do custo de vida. Assim, o
da Fonseca conseguiu livrar-se da influncia de Pinheiro Macha- prejuzo, que deveria ser do fazendeiro, era dividido entre todos.
do e iniciou a poltica de Salvaes. Secundado por assesso- No entanto, as perspectivas de lucros levavam o cafeicultor a con-
res florianistas jacobinos, iniciou intervenes nos Estados que tinuar expandindo as suas plantaes. Iria chegar o momento em
pretendiam ser Salvacionistas. Hermes utilizava o poder inter- que o governo no conseguiria garantir essa poltica.
vencionista da Unio para quebrar a poltica dos governadores.
Atravs de expedies militares, interveio nos Estados, derruban- A borracha
do as velhas oligarquias e substituindo-as por elementos hermis- No final do Sculo XIX, com o desenvolvimento da indstria
tas. As Salvaes ocorreram nos Estados do Norte e Nordeste, automobilstica, cresceu de borracha na Europa e nos Estados
politicamente mais fracos. No Cear, o intervencionismo gerou Unidos. A valorizao do produto contribuiu para aumentara a
uma revolta sertaneja sob a liderana do Padre Ccero, conhe- sua produo no Brasil, a regio amaznica atraiu muita gente,
cido como o Coronel de Batina. O Padim Cio conseguiu porque era rica em seringueiras, rvores das quais se extrai o ltex
mobilizar foras para reconduzir ao poder a oligarquia cearense para a fabricao da borracha. O maior contingente da mo de
derrubada pelo marechal. Essas Salvaes impopularizaram o obra veio do Nordeste, por causa da proximidade da regio e da
presidente, o exrcito e o prprio Pinheiro Machado, patrono da misria provocada pela seca. Os trabalhadores eram submetidos a
candidatura Hermes. So Paulo e Minas Gerais livraram-se do condies subumanas de trabalho, em maio floresta amaznica.
intervencionismo, por serem demasiadamente poderosos. Qual- A produo, antes insignificante, aumentou muito e, em 1910,
quer tentativa nesses dois estados poderia resultar em ameaa de o Brasil era o primeiro produtor mundial de borracha. Fato que
guerra civil. O presidente no conseguiu depor os oligarcas do trouxe grande acmulo de riqueza por parte dos proprietrios
poder. Na medida em que derrubava uma oligarquia, o setor dis- de seringais, que habitavam em Manaus e Belm, cidades que
sidente dessa mesma oligarquia subia ao poder. Assim, os grupos viveram um grande surto de crescimento por esta poca. Muitos
dominantes revezavam-se no poder. Continuava a hegemonia sob ultrapassaram as fronteiras brasileiras e entraram, pelo territrio
o controle das mesmas camadas sociais. So Paulo e Minas volta- da Bolvia, na extrao do ltex das seringueiras. Havia uma em-
ram a um acordo e apoiaram a candidatura do mineiro Venceslau presa americana na regio que solicitou a interveno do governo
Brs. Voltava ao poder a poltica caf-com-leite. boliviano. Os seringueiros revoltaram-se sob a liderana de Pl-
cido de Castro, que proclamou o Estado Independente do Acre,
Aspectos econmicos em 1902, para posterior anexao ao Brasil. Um atrito maior no
Durante o perodo da Repblica Velha, a economia brasi- interessava a ambos os pases fronteirios. O Baro do Rio Branco
leira manteve as mesmas caractersticas dos perodos anteriores. assinou com a Bolvia o Tratado de Petrpolis, em 1903, estabe-
Continuou sendo agrria, monocultora e dependente do merca- lecendo: Construo da Estrada de Ferro Madeira-Mamor, li-
do externo. Essa dependncia tornava a economia muito frgil. gando a Bolvia Bacia Amaznica; indenizao de 2 milhes de
Quando os pases consumidores atravessavam uma crise, redu- libras Bolvia; cesso de territrio no Mato Grosso e a posse do
ziam as compras, e quando outro pas vendia o mesmo produto a Acre pelo Brasil. No entanto, o ciclo da borracha entrou em deca-
preo mais baixo, o Brasil perdia mercado. Alm do caf, o Brasil dncia rapidamente, principalmente devido concorrncia ingle-
exportava alguns outros produtos como a borracha, o mate, o sa. Os ingleses plantaram mudas de seringueiras em suas colnias
fumo, e importava manufaturados e gneros alimentcios. asiticas e conseguiram vender o produto a preo mais baixo.

O caf O cacau
Cultivado na regio do recncavo baiano, adquiriu impor-
tncia com o progresso da indstria do chocolate, nos Estados
Unidos e na Europa, nas primeiras dcadas do Sculo XX. Entre-
tanto, a concorrncia dos domnios ingleses na frica provocou a
queda da exportao do cacau brasileiro.

A indstria
A implantao da indstria no Brasil enfrentou muitos obst-
culos, mesmo depois da independncia.Os produtos estrangeiros,
sobretudo os ingleses, concorriam com os brasileiros, pois paga-

UECEVEST 153

Apostilas UECEVEST mod3.indb 153 06/02/2011 09:59:27


H I S T RIA DO BRASIL

vam taxas de alfndega baixas para entrar no Brasil. No perodo 03. (UECE 2009/2) A Repblica, na voz de seus propagandistas
monrquico, houve um pequeno desenvolvimento industrial fa- mais radicais, como Silva Jardim e Lopes Trovo, era apresentada
vorecido pela liberao de capital, com a abolio do trfico de es- como a irrupo do povo na poltica na melhor tradio da re-
cravos e com a elevao das tarifas alfandegrias, a partir de 1844. voluo Francesa de 1789. (Fonte: CARVALHO, Jos Murilo.
No governo provisrio da Repblica Velha, o Ministro da Fazen- Os Bestializados: o Rio de Janeiro e a Repblica que no foi.
da, Rui Barbosa, liberou a emisso de dinheiro para financiar a So Paulo: Companhia das Letras, 1987. pp. 9-19.) O fragmento
indstria. Os resultados foram pfios em virtude da crise infla- anterior refere-se
cionria. A partir de 1910, So Paulo substituiu o Rio de Janeiro, a) Repblica Brasileira que, nos seus sessenta primeiros
tornando-se o principal centro industrial brasileiro. Contriburam anos, viveu sombra do regime monrquico e do poder
para a industrializao de So Paulo: o acmulo do capital com o moderador.
caf e a ampliao do mercado consumidor decorrente do aumen- b) ao Manifesto Republicano de 1870, que considerava o
to da populao. A maioria dos imigrantes que esse estado recebeu regime republicano brasileiro incompatvel com a soberania
se dirigiu para a zona rural, mas alguns deles instalaram manufa- nacional.
turas, que se transformaram, mais tarde em grandes indstrias, c) ao regime republicano brasileiro que, embora proclamado
como por exemplo: Matarazzo, Kaplin, Filizola, etc. O desenvol- sem a iniciativa popular, despertou, entre os excludos do
vimento industrial da Repblica Velha foi favorecido pela ecloso sistema anterior, certo entusiasmo quanto s possibilidades
da Primeira Guerra Mundial. Durante esse perodo, o Brasil no de participao.
podia importar dos pases em conflito. Houve o desenvolvimento d) ao regime iniciado em 1889, que, efetivamente, representou
da indstria de bens de consumo no-durveis: tecidos, alimen- uma nova era de participao poltica para o trabalhador
tos, vesturio, calados, vidros, etc. O operariado era constitudo brasileiro, em especial para o operariado.
por imigrantes estrangeiros, principalmente espanhis e italianos.
O operariado trabalhava de dez a doze horas por dia, no tinha 04. (UECE 2008/2) Ter a qualidade de cidado brasileiro , sim,
direito a frias, nem a indenizao por acidentes de trabalho. Os ter uma denominao honorfica mas que s d direitos cvicos e
salrios eram muito baixos, por isso alimentavam-se mal e mora- no direitos polticos. Situao semelhante acontecia mesmo em
vam em habitaes precrias. Diante desse cenrio, no foi difcil paises como Inglaterra, Frana e Estados Unidos que boa parte da
o surgimento de movimentos operrios. Inicialmente, o anarco- populao no possua direitos polticos. (Fonte: GRINBERG,
sindicalismo se fez mais presente, inclusive nos dois primeiros Keila. O Fiador dos Brasileiros: cidadania, escravido e direito
congressos operrios brasileiros (1906 e 1913) e, principalmente, civil. Rio de Janeiro: Civilizao Brasileira, 2002. pp. 112/115.)
na primeira greve geral no Brasil, em 1917. Porm, em 1922, com Em relao aos direitos polticos dos brasileiros na primeira me-
a fundao do Partido Comunista, houve uma ciso dentro desse tade do sculo XIX, so feitas as seguintes afirmaes:
movimento, que, alm das disputas internas, ainda era obrigado I. Ser cidado numa sociedade escravocrata era uma distino
a conviver com a brutal represso por parte do governo, que pode almejada, mesmo que ela no trouxesse consigo direitos
ser resumida na frase atribuda ao ltimo presidente da Repblica polticos.
Velha, Washington Luis: A questo social um caso de polcia. II. O significado corrente da expresso direitos cvicos era
de reconhecer, por exemplo, a denominao honorfica de
escravos e libertos que haviam lutado por alguma causa
E X E R C C I O benfica ao Brasil.
III. A segurana pblica estava relacionada proteo dos
01. (IFCE 2009/2) Em relao proclamao da Repblica, cidados, por isso, a dificuldade de muitos em
correto afirmar-se que considerar os libertos como cidados de fato.
a) Est ligada ao desenvolvimento da cotonicultura como
principal atividade econmica do Brasil Imperial. Sobre as trs afirmaes anteriores, assinale o correto.
b) Associa-se ao crescimento das ideias anarcossindicalistas e ao a) Todas so verdadeiras.
movimento operrio. b) Apenas a I e a III so falsas.
c) Vincula-se difuso de ideias positivistas. c) Apenas a I e a II so falsas.
d) Foi um processo marcado pela afirmao dos cafeicultores d) Apenas a II e a III so falsas.
do vale do Paraba e de escravocratas, que queriam, com a
Repblica, manter a escravido. 05. (UECE 2003/1) Um elemento fundamental para a compre-
e) Associa-se ao exclusivo desejo dos coronis nordestinos de enso do funcionamento poltico da Primeira Repblica o Co-
romper com a Monarquia. ronelismo, ou seja, a dominao dos poderosos chefes polticos
locais, em geral grandes proprietrios sobre a massa de eleitores,
02. (UECE 2009/2) Pode-se afirmar que a Proclamao da Re- que, pelos laos de dependncia pessoal a favor, atrela os traba-
pblica inaugurou uma nova ordem poltica no Brasil. Neste sen- lhadores despossuidos, transformando-os em sua clientela pesso-
tido, assinale o correto. al. (NEVES, Margarida de Souza e HEIZER, Aida. A ordem
a) O Centralismo presente no perodo Imperial foi substitudo o Progresso. So Paulo: Atual, 1991. Histria em Documento.)
pelo Federalismo defendido pelas elites do oeste paulista e de Em relao ao fenmeno do coronelismo, na Primeira Repblica,
Minas Gerais. assinale a alternativa falsa.
b) De imediato, o poder poltico passou a ser controlado pelas a) A Comisso de Verificao foi um dos mais fortes
oligarquias rurais, mormente de So Paulo e Rio de Janeiro. instrumentos de combate ao poder dos coronis.
c) Os quatro primeiros governos republicanos, na primeira b) O voto de cabresto constitua uma das bases fundamentais
Repblica, corresponderam chamada Repblica da Espada, do poder dos coronis.
sob a hegemonia dos militares ligados ao exrcito. c) Os exrcitos dos capangas garantiam o exerccio do poder
d) A feio poltica da primeira Repblica explicitou-se num local aos oligarcas.
nico partido: o Partido Republicano, de mbito nacional, d) A rede de compromissos comeava pelos coronis nos
articulado e forte. municpios, passava pelos arranjos entre as oligarquias
regionais e chegava at o presidente da Repblica.

154 UECEVEST

Apostilas UECEVEST mod3.indb 154 06/02/2011 09:59:28


HISTRIA D O B R A S I L

zados por capangas, para que votassem de acordo com o interesse


do coronel. Mas recorria-se tambm a outras estratgias, como
06. (ENEM 2010) compra de votos, eleitores-fantasma, troca de favores, fraudes na
I. Para consolidar-se como governo, a Repblica precisava apurao dos escrutnios e violncia. (Disponvel em: http://his-
eliminar as arestas, conciliar-se com o passado monarquista, toriadobrasil.net/republica. Acesso em: 12 de Dezembro de 2008
incorporar distintas vertentes do republicanismo. Tiradentes adaptado). Com relao ao processo democrtico do perodo
no deveria ser visto como heri republicano radical, mas registrado no texto, possvel afirmar que:
sim como heri cvico-religioso, como mrtir, integrador, a) O coronel se servia de todo tipo de recursos para atingir seus
portador da imagem do povo inteiro (CARVALHO, Jos objetivos polticos.
Murilo de. A formao das almas: o imaginrio da Repblica b) O eleitor no podia eleger o Presidente da Repblica.
no Brasil. So Paulo: Companhia das Letras, 1990) c) O coronel aprimorou o processo democrtico ao instituir o
II. Ei-lo, o gigante da praa,/o Cristo da multido! voto secreto.
Tiradentes quem passa/deixem passar o Tito. d) O eleitor era soberano em sua relao com o coronel.
(ALVES, C. Gonzaga ou a Revoluo de Minas. In CARVALHO, Jos Murilo de. A e) Os coronis tinham influncia maior nos centros urbanos.
formao das almas: o imaginrio da Repblica no Brasil. So Paulo: Companhia
das Letras, 1990) 09. (ENEM 2009) Houve momentos de profunda crise na his-
A 1 Repblica brasileira, nos seus primrdios, precisava cons- tria mundial contempornea que representaram para o Brasil,
tituir uma figura histrica capaz de congregar diferenas e sus- oportunidades de transformao no campo econmico. A Pri-
tentar simbolicamente o novo regime. Optando pela figura de meira Guerra Mundial (1914-1918) e a quebra da Bolsa de Nova
Tiradentes, deixou de lado figuras como Frei Caneca ou Bento York (1929), por exemplo, levaram o Brasil a modificar suas es-
Gonalves. A transformao do inconfidente em heri nacional tratgias produtivas e a contornar as dificuldades de importao
evidencia que o esforo de construo de um simbolismo por de produtos que demandava dos pases industrializados. Nas trs
parte da Repblica estava relacionado primeiras dcadas do Sculo XX, o Brasil:
a) Ao carter nacionalista e republicano da Inconfidncia, a) Impediu a entrada de capital estrangeiro, de modo a garantir
evidenciado nas ideias e na atuao de Tiradentes. a primazia na indstria nacional.
b) identificao da Conjurao Mineira como o movimento b) Priorizou o ensino tcnico, no intuito de qualificar a mo-
precursor do positivismo brasileiro. de-obra nacional direcionada indstria.
c) Ao fato de a proclamao da Repblica ter sido um movimento c) Experimentou grandes transformaes tecnolgicas na
de poucas razes populares, que precisava de legitimao. indstria e mudanas compatveis na legislao trabalhista.
d) semelhana fsica entre Tiradentes e Jesus, que d) Aproveitou a conjuntura de crise para fomentar a
proporcionaria, a um povo catlico como o brasileiro, uma industrializao pelo pas, diminuindo as desigualdades
fcil identificao. regionais.
e) Ao fato de Frei Caneca e Bento Gonalves terem liderado e) Direcionou parte do capital gerado pela cafeicultura para
movimentos separatistas no Nordeste e no Sul do pas. a industrializao, aproveitando a recesso europeia e
norteamericana.
07. (ENEM 2010) O artigo 402 do Cdigo Penal Brasileiro de
1890 dizia: Fazer nas ruas e praas pblicas exerccios de agilidade 10. (ENEM 2010) As secas e o apelo econmico da Borracha
e destreza corporal, conhecidos pela denominao de capoeira- produto que no final do Sculo XIX alcanava preos altos nos
gem: andar em correrias, com armas ou instrumentos capazes de mercados internacionais motivaram a movimentao de massas
produzir uma leso corporal, provocando tumulto ou desordens. humanas oriundas do Nordeste do Brasil para o Acre. Entretanto,
Pena: Priso de dois a seis meses. at o incio do Sculo XX, essa regio pertencia Bolvia, embora
a maioria da sua populao fosse brasileira e no obedecesse au-
(SOARES, C.E.L. A Negregada Instituio: os capoeiras no Rio de Janeiro: 1850-
1890. Rio de Janeiro: Secretaria Municipal de Cultura, 1994 adaptado).
toridade boliviana. Para reagir presena de brasileiros, o governo
de La Paz negociou o arrendamento da regio a uma entidade in-
O artigo do Primeiro Cdigo Penal Republicano naturaliza medidas ternacional, o Bolivian Syndicate, iniciando violentas disputas dos
socialmente excludentes. Nesse contexto, tal regulamento expressava dois lados da fronteira. O conflito s terminou em 1903, com a
a) A manuteno de parte da legislao do Imprio com vistas assinatura do Tratado de Petrpolis, pelo qual o Brasil comprou
ao controle da criminalidade urbana. o territrio por dois milhes de libras esterlinas. (Disponvel em
b) A defesa do retorno do cativeiro e escravido pelos primeiros www.mre.gov.br.. Acesso em: 03 Nov. 2008 adaptado) Com-
governos do perodo republicano. preendendo o contexto em que ocorreram os fatos apresentados,
c) O carter disciplinador de uma sociedade industrializada, o Acre tornou-se parte do territrio brasileiro
desejosa de um equilbrio entre progresso e civilizao. a) Pela formalizao do Tratado de Petrpolis, que indenizava o
d) A criminalizao de prticas culturais e a persistncia Brasil pela sua anexao.
de valores que vincularam certos grupos ao passado de b) Por meio do auxlio do Bolivian Syndicate aos emigrantes
escravido. brasileiros na regio.
e) O poder do regime escravista, que mantinha os negros como c) Devido crescente emigrao de brasileiros que exploravam
categoria social inferior, discriminada e segregada. os seringais.
d) Em funo da presena de inmeros imigrantes estrangeiros
08. (ENEM 2009) A figura do Coronel era muito comum du- na regio.
rante os anos iniciais da Repblica, principalmente nas regies e) Pela indenizao que os imigrantes brasileiros pagaram Bolvia.
do interior do Brasil. Normalmente, tratava-se de grandes fazen-
deiros que utilizavam seu poder para formar uma rede de clien-
tes polticos e garantir o resultado das eleies. Era usado o voto G A B A R I T O
de cabresto, por meio do qual o coronel obrigava os eleitores de
seu curral eleitoral a votarem nos candidatos apoiados por ele. 01. c 02. a 03. c 04. c 05. a
Como o voto era aberto, os eleitores eram pressionados e fiscali- 06. c 07. d 08. a 09. e 10. c

UECEVEST 155

Apostilas UECEVEST mod3.indb 155 06/02/2011 09:59:29


H I S T RIA DO BRASIL

REPBLICA VELHA tivessem enorme dio do novo regime. Mas isso devia-se ao fato
REVOLTAS E DECADNCIA (UECE/ENEM) de que, aps a Proclamao da Repblica, a situao dos serta-
nejos piorou enormemente. A questo fundamental de Canudos
era a luta pela posse da terra. A luta contra o latifndio e contra
A realidade social da Repblica Velha pode ser vista sob dois
as injustias da poca.
aspectos: De um lado, uma sociedade que se modernizava rapi-
Situado no interior da Bahia, s margens do Vaza-Barris, Ca-
damente, apresentando um crescimento urbano e populacional,
nudos era uma velha fazenda abandonada. A origem do nome
que trazia consigo problemas sociais inerentes modernidade. E,
deve-se ao fato de que a populao, que ali vivia anteriormente,
de outro lado, uma sociedade tradicional, rural, na qual persis-
pitava cachimbos em canudos que existiam na beira do rio. Para
tiam as antigas e as arcaicas formas de convvio social, baseadas
se atingir o arraial do Belo Monte (foi este nome dado por Con-
nos laos de fidelidade pessoal. Em ambos os casos, formou-se
selheiro) era difcil, pois a estrada de ferro mais prxima distava
uma massa de trabalhadores que vivia em estado de extrema po-
200 quilmetros. As caatingas, as serras e a aridez do serto se
breza, sem terra, alijada do mercado de trabalho. Essa massa da
constituam em excelentes elementos de defesa. Canudos chegou
populao estava excluda da propriedade dos meios de produ-
a ter uma populao de 25 mil habitantes, onde se praticava uma
o, das decises polticas e dos direitos trabalhistas e vivia com-
economia de subsistncia, ao estilo do chamado comunismo pri-
pletamente margem do poder. Esses excludos enfrentavam a
mitivo, no qual a colheita era repartida igualmente entre todos.
situao de opresso cotidiana e responderam com rebelio, no
Para terminar a construo de uma nova Igreja para o arraial,
protesto trgico com a opresso e a misria. No caso de Canudos
Antnio Conselheiro comprara uma certa quantidade de madei-
e do Contestado, o protesto injustia social estava mesclado
ra na cidade de Juazeiro. Apesar de Conselheiro j ter pago a
com a religiosidade popular. Essas rebelies sociais ocorreram no
mercadoria, o comerciante no entregou a madeira na data con-
Brasil tanto no meio urbano as revoltas da Chibata, da Vacina
vencionada. Diante disso, Conselheiro disse que iria at Juazeiro
e as tenentistas quanto no meio rural os movimentos messi-
escolher a madeira, mas suas palavras foram maldosamente in-
nicos de Canudos e Contestado e a Sedio de Juazeiro.
terpretadas. Pressionado pelos militares, o Governador da Bahia,
enviou a 1 expedio contra Canudos. Essa 1 expedio tinha o
Revoltas rurais comando do Tenente Manoel da Silva Pires Ferreira. Enfrentando
os sertanejos armados de faces, foices, forquilhas, velhas armas
Canudos (BA 1894-1897) de fogo, a expedio no pde continuar, batendo em retirada. A
2 expedio era comandada pelo Major Febrnio de Brito. Con-
tava com seiscentos soldados, dois canhes e duas metralhadoras.
Todos acreditavam que esmagariam a resistncia sertaneja. Os
sertanejos no foram para um combate direto. Profundamente
conhecedores da regio, atacavam pelos flancos, adotando tticas
de guerrilha, fustigavam e enfraqueciam as tropas do governo.
A expedio chega a Tabuleirinho. Os soldados esto famintos e
fatigados. Os combates se sucedem. Febrnio de Brito constata
que est sem mantimentos e munies e compreende o malogro
de seu empreendimento, ordena a retirada, que foi das mais san-
grentas, pois os guerrilheiros, sob o comando de Paje, atacam
em pequenos grupos por todos os lados. Finalmente, a expedio
chegou a Monte Santo em estado lastimvel. Os sertanejos ven-
No final do Sculo XIX, eclodiu no Serto da Bahia um novo ceram novamente.
tipo de sociedade, ideologicamente impregnada de religiosidade A 3 expedio, composta por 1.200 soldados e farta muni-
e socialista na prtica. Essa experincia, extremamente perigosa, o, era comandada pelo famoso Coronel Antnio Moreira Csar
foi conduzida por um personagem controverso chamado Ant- (o corta cabea da Revoluo Federalista). A expedio partiu
nio Vicente Mendes Maciel conhecido como Antnio Con- de Queimadas a 17 de fevereiro de 1897, chegou a Angico em 2
selheiro (devido aos conselhos) nasceu no comeo de 1828, de maro; ali se dividiu em duas colunas, ficando a segunda sob
em Quixeramobim, na provncia do Cear. O pequeno Antnio o comando do Coronel Pedro Nunes Tamarindo. Essa expedio
Vicente cresceu assistindo s injustias e a toda sorte de violncias fracassou totalmente. Moreira Csar morreu em combate. A re-
que caracterizavam a sociedade nordestina de ento. Influenciado tirada foi desastrosa. Enquanto isso sucedia, os sertanejos reco-
por leituras das obras religiosas e pela ao social do bondoso lhiam os despojos. Pela estrada e pelos lugares prximos jaziam,
padre Jos Antnio de Ibiapina, Antnio Vicente, como os ca- esparsas, armas e munies.
valeiros medievais, iria lutar, sob o signo da cruz, pelos fracos e Somente a quarta expedio, comandada pelo General Ar-
oprimidos. A historiografia tradicional o apresentou como um tur Oscar, utilizando mais de oito mil homens, conseguiu que-
louco, fantico, bandido, inimigo da Repblica e da Igreja. Aban- brar a resistncia sertaneja. A fome, a sede, a falta de munio
donado pela esposa, frustrado por no ter conseguido ser padre, e o cerco total tornavam impossvel a resistncia sertaneja. Eu-
Antnio Vicente perambulava pelo serto, batizando multides clides da Cunha, enviado do Jornal O Estado de So Paulo,
de crianas e casando os amancebados. Dava conselhos prticos, no escondeu o seu espanto diante de tamanho herosmo dos
simples. Sempre trazia uma palavra de consolo para os sertanejos sertanejos: Canudos no se rendeu. Exemplo nico em toda a
espoliados pela realidade scio-poltica da Repblica Velha. Li- histria, resistiu at o esgotamento completo. Expugnado pal-
derava as comunidades na construo de cemitrios, reformas de mo a palmo, na preciso integral do termo, caiu no dia 5 (ou-
igrejas, obra de cavar cacimbas. tubro de 1897), ao entardecer, quando caram seus ltimos de-
A economia nordestina estava em franca decadncia. O lati- fensores, que todos morreram. Eram quatro apenas: um velho,
fndio, as secas, a misria e as injustias de toda sorte caracteri- dois homens feitos e uma criana, na frente dos quais rugiam,
zavam aquela sociedade agrria e semifeudal. Polticos da poca raivosamente, cinco mil soldados. Caiu o arraial a 5, no dia
apresentaram Canudos como uma tentativa de restaurar a Mo- 6 acabaram de destru-lo, desmanchando-lhe as casas, 5.200,
narquia. Isso no verdade, embora Conselheiro e os sertanejos cuidadosamente contadas.

156 UECEVEST

Apostilas UECEVEST mod3.indb 156 06/02/2011 09:59:29


HISTRIA D O B R A S I L

Revolta do contestado (PR/SC 1912-1914) verno Federal e o Sen. Pinheiro Machado, contrariados em seus
propsitos, passaram a hostilizar o chefe eleito e reconhecida-
mente legal da terra cearense. Franco Rabelo sofreu toda a espcie
de oposio e descrdito por parte do Governo Federal. O Sen.
Pinheiro Machado, de comum acordo com o Marechal Hermes,
procurava um meio para conturbar a vida poltica do Cear e
afastar de suas funes o ento presidente do Estado.
Para servir de estopim a uma rebelio, foi escolhida a cida-
de de Juazeiro do Norte, que era chefiada pelo mdico baiano
Dr. Floro Bartolomeu, sob os auspcios do Padre Ccero Romo
Batista. A revoluo comeou, praticamente, a 9/12/1913, dia
em que o Dr. Floro iniciou as hostilidades, com a deposio das
autoridades estaduais de Juazeiro, providenciando construo de
piquetes e barricadas em redor da cidade.
Conflitos armados se verificaram em todo o Estado, nota-
Na divisa entre os estados do Paran e Santa Catarina surgiu damente em Crato, Barbalha, Iguatu, Maranguape, Caucaia e
uma comunidade de trabalhadores rurais posseiros, que trabalha- Acarape. Generalizou-se a confuso e a desordem. Em Miguel
vam na extrao de mate e eucalipto para madeireiras da regio. No Calmon (hoje Ibicu), o governo cearense concentrou todos os
entanto, em 1910, a companhia norte-americana Brazil Railway seus recursos no sentido de impedir a marcha dos romeiros sobre
Company ganhou a concesso para a construo da estada de fer- a capital. Tudo foi em vo. As foras legais logo se dispersaram
ro que ligaria So Paulo ao Rio Grande do Sul. Para cumprir um com a morte do seu comandante J. da Penha, crivado de balas.
dos termos do contrato, a companhia se responsabilizou por nove Foi enviada, pela via ferroviria, uma fora tarefa de jagunos,
quilmetros de cada lado da ferrovia, o que ocasionou a desapro- rumo Fortaleza.
priao de terras dos antigos habitantes, que no possuam a pro- O Marechal Hermes da Fonseca, no dia 9 de maro, implan-
priedade legal da terra. Em 1911, a Lumber, poderosa madeireira tou no Cear o Estado de Stio, anulando assim os poderes execu-
ligada Brazil Railway, estabeleceu-se na zona contestada. Tinha tivo e legislativo, desprezando os ditames da constituio federal,
autorizao para explorar a madeira da regio, comprometendo-se ferindo a democracia e golpeando brutalmente a autonomia do
com a colonizao. Com essas novidades, os habitantes da regio Estado.
estavam profundamente prejudicados, mas foram acalmados com Com a deposio do Presidente Constitucional do Estado,
empregos na ferrovia. Porm, com o fim da construo, mais de 8 foi nomeado para substitu-lo o Gal. Setembrino de Carvalho.
mil trabalhadores ficaram desempregados e em p-de-guerra. A deposio de Franco Rabelo na vida poltica do Cear trouxe
Surgiu nessa poca, na regio do Contestado, um monge, desiluses e revoltas. De 1914 a 1930, formou-se no Estado um
Joo Maria, na realidade, desertor do exercito paranaense. Mo- ambiente revolucionrio de corajosa e decidida oposio aos que
narquista, agrupou a populao pobre, sem terra e desempregada, traam a Repblica num autntico desvirtuamento dos salutares
que via nele um profeta. Este grupo foi armado e preparado para princpios democrticos.
o combate a partir do povoado de Taquaruau. Joo Maria pre-
gava a existncia de um Reino Milenarista: o Messias destruiria Revoltas urbanas
o mal e inauguraria um reino de felicidade, um reino paradisaco
de mil anos. Nesse reino, vigoraria a Lei de Deus, todos teriam Revolta da vacina (RJ 1904)
lugar para plantar e haveria prosperidade e justia. Os fazendei-
ros da regio solicitaram e conseguiram o apoio das autoridades
na represso deste movimento, sob o comando do Coronel Joo
Gualberto, que culminou com a morte do Beato Joo Maria, ain-
da em 1912. No entanto, a vitria final foi dos campnios, que
logo em seguida constituram um novo lder, o beato Jos Maria,
que tambm desapareceu em combate. Para reprimir definitiva-
mente o movimento, foi mandado regio, o General Fernando
Setembrino de Carvalho, que desbaratou os ltimos focos de re-
sistncia dos camponeses, recorrendo brutalidade tpica destes
momentos para com os comandados do Beato Emanuel, ltimo
lder do contestado.

A sedio de Juazeiro
Por afastar da administrao do Estado uma verdadeira oli-
garquia que se vinha formando, foi eleito no pleito de 11/4/1912
presidente Constitucional do Cear o Tenente-Coronel Marcos Em 1903, a cidade do Rio de Janeiro, capital da Repblica,
Franco Rabelo. A vitria de Franco Rabelo contra o Dr. Domin- passou por uma profunda remodelao urbana, durante a ges-
gos Carneiro, candidato do Comendador Accioly, foi recebida to do Prefeito Pereira Passos e do Presidente Rodrigues Alves.
com a maior alegria e entusiasmo, no s pela populao local, O objetivo era puramente esttico, buscava-se transformar o Rio
mas tambm pela cpula do Governo Federal, que via nesse fato numa Paris dos trpicos, porque a cidade-luz passara por algo
a chance de substituir a oligarquia reinante em nosso Estado. similar no fim do sculo XIX. O primeiro passo nessa onda de
Nesse tempo o Presidente da Repblica, Marechal Hermes da ordenamento urbano era a construo da Avenida Central (hoje
Fonseca, lutava abertamente pela candidatura de Pinheiro Ma- Rio Branco) que estava destinada a ser um carto-postal, no es-
chado para substitu-lo no palcio do Catete. tilo dos bulevares parisienses, larga, arborizada e iluminada. Po-
O Cel. Franco Rabelo, no entanto, manifestou-se contrrio rm, para atingir este nvel, foi preciso desapropriar centenas de
s pretenses do Marechal Hermes da Fonseca. O Chefe do Go- habitaes pobres, tais como cortios. Alm disso, era inevitvel

UECEVEST 157

Apostilas UECEVEST mod3.indb 157 06/02/2011 09:59:30


H I S T RIA DO BRASIL

demolir vrios becos, vielas e similares, situados bem no centro da Foi impossvel contar o nmero de feridos e mortos, quando
cidade. O problema que o governo desapropriou os moradores ao final do dia 16 as foras legais retomaram o controle da cidade.
e no lhes pagou nenhuma indenizao, sob o argumento de que A chamada Revolta da Vacina conteve todos os ingredientes
os mesmos no possuiriam a propriedade legal dos imveis. Tais caractersticos de um levante popular contra um regime que no
pessoas se viram obrigadas a abandonar seus lares e a morar no deixava o menor canal efetivo de expresso legal. At a desorgani-
entorno da cidade, nos morros ainda com presena de mata vir- zao da revolta foi caracterstica: no havia, objetivamente, um
gem, dando origem s favelas. conjunto orgnico de foras populares politicamente expressivo,
No entanto, o principal fator para a Revolta da Vacina em nenhuma cidade brasileira.
foi a epidemia de varola hemorrgica que tomou conta do Rio de
Janeiro em 1904, que s neste ano havia vitimado 4.201 pessoas. Revolta da chibata (RJ 1910)
Em outubro de 1904, depois de longos debates parlamentares e
contra a opinio da oposio foi aprovado o decreto que instituiu
a vacinao obrigatria contra a varola, para todos os brasileiros
de mais de 6 meses de idade. A repulsa causada na imprensa e
na populao por esse decreto foi a expresso de toda uma srie
de descontentamentos populares e de polticos e militares contra
a dominao oligrquica. A vacina foi regulamentada no dia 9
de novembro, e no dia 10 comearam os distrbios. O governo
proibiu concentraes e comcio, mas a massa comeou a afluir
s ruas e praas, provocando interveno policial. No dia 11, um
forte policiamento provocou pequenos choques e prises, e no
dia 12, uma concentrao de 3 mil pessoas diante da sede da Liga
Contra a Vacina Obrigatria acendeu os nimos. Em 1910, eclodiu no Rio de Janeiro uma revolta de mari-
No dia 13, a revolta tomou propores gigantescas, com nheiros, mais conhecida como Revolta da Chibata. Os marinhei-
milhares de pessoas vindas das favelas, dos cortios, enchendo ros revoltaram-se contra a m alimentao, as precrias condies
a praa Tiradentes e outras, vaiando a polcia e o governo. O de trabalho e, principalmente, os castigos corporais a que eram
General Piragibe, chefe de polcia, ordenou tropa avanar de submetidos, assim como, para disciplin-los, era comum o uso
sabre desembainhado sobre a multido: ocorreram tiroteios, com da chibata. O estopim da revolta foi o aoitamento, em 22 de
mortos e feridos de ambos os lados. A populao se dispersava e Novembro de 1910, do Praa Marcelino Rodrigues de Menezes
se reaglutinava em outros pontos. Logo surgiram barricadas de (250 chibatadas).
paraleleppedos e madeira. Enquanto isso, novos movimentos se O marujo Joo Cndido, negro, gacho e semianalfabeto, as-
do no centro __ e comeam a ser quebrados combustores de ilu- sumiu a liderana do movimento que se opunha a esses castigos,
minao, vitrinas de casas etc., bondes so virados para servirem a partir de um motim do Navio Minas Gerais. Os rebeldes toma-
de trincheiras. As ruas do centro da cidade ficam tomadas de bar- ram vrios navios que estavam na Baa da Guanabara e passaram
ricadas; postos policiais so assaltados. A luta se intensifica, e as a manobr-los, ameaando inclusive bombardear a cidade, caso
tropas policiais, ajudadas pelos bombeiros, so incapazes de ven- suas reivindicaes no fossem atendidas.
cer todos os focos populares. Nos dias seguintes, tornou-se gran- Aps quatro dias, o Presidente Hermes da Fonseca aboliu os
de o nmero de mortos, de bondes incendiados (os pequenos castigos e concedeu anistia aos marinheiros. Ao deporem as ar-
comerciantes forneciam baldes de querosene como combustvel), mas, perceberam que foram enganados pelo Presidente, que ex-
delegacias assaltadas e combates violentos, com a interveno de pulsou da marinha os revoltosos, nomeou oficiais e determinou
unidades do exrcito. Entretanto, a revolta tomaria uma outra a priso de Joo Candido e seus seguidores. Eles foram lanados
dimenso com a entrada em cena de militares revoltosos. nos pores do navio Satlite e nas masmorras das Ilhas das Co-
bras. Ainda tentaram se rebelar novamente, mas, agora sem os
A revolta militar navios, tiveram suas manifestaes sufocadas na base da chibata.
A amplitude do descontentamento contra Rodrigues Alves
pode ser avaliada pela adeso dos jovens militares, seu objetivo O Tenentismo
era mais conscientemente voltado para a tomada do poder atravs A inquietao tomou conta da jovem oficialidade do exrcito.
de um golpe. Para isso, se ligam at aos monarquistas do recm- O alto escalo estava plenamente identificado com as oligarquias
fundado jornal Comrcio do Brasil, de Alfredo Varela, e tinham dominantes e o exrcito havia se constitudo no brao armado do
como um dos principais porta-vozes o militar Barbosa Lima, um Estado. A politizao dos jovens militares colocou-se como outro
dos diretores da Liga Contra a Vacinao Obrigatria. componente da poltica nacional, contestando a ordem das coisas
Na madrugada do dia 15, o Presidente Rodrigues Alves, reto- atravs do tenentismo. Realmente, o movimento dos militares
mando a ofensiva, fez bombardear por navios de guerra a escola limitava-se aos quartis, porm muitos civis aderiram ao progra-
militar, enquanto as tropas comandadas por Caetano de Faria lhe ma e s leis definidas pelos tenentes. Com isso, o movimento foi
faziam o cerco. Naquela manh, os cadetes se renderam. Cente- absorvido pelas camadas mdias urbanas bloqueadas na sua pre-
nas de prises, inclusive de civis, desbarataram o resto da revolta. tenso de ascenso poltica, econmica e social. A cristalizao de
Quanto ao movimento popular, ficou mais fcil derrot-lo. A um grupo que apresentasse novas ideias e teses geraria as crises e,
ltima grande resistncia deu-se na rua da Harmonia, onde um por fim, a derrubada da Repblica Velha na Revoluo de 1930.
conjunto de trincheiras e barricadas parecia inexpugnvel a ponto O movimento tenentista no tinha uma formao ideolgica
de ser chamado Port Arthur, em referncia ao baluarte russo definida. Recebia influncias dos movimentos polticos que abala-
no Extremo Oriente, durante a guerra com os japoneses. Foi ne- vam o mundo na dcada de 20. Mais uma vez aqueles que preten-
cessrio um violentssimo tiroteio, da bem armada tropa legal, diam reformar o pas tinham os ps aqui e a cabea l fora, voltados
contra os canos cheios de dinamite, bombas, pedras e revlveres para solues totalmente desvinculadas da realidade nacional. Os
da massa amotinada. Outra luta violenta deu-se na Sade, onde tenentes colocavam-se na posio de salvao nacional. Acredita-
se destacou um negro apelidado de Prata-Preta, que sozinho en- vam que isto poderia ser conseguido com a manuteno da pureza
frentava a fora policial com dois revlveres, atirando sem parar. republicana, ao mesmo tempo que arvoravam-se em tutores do

158 UECEVEST

Apostilas UECEVEST mod3.indb 158 06/02/2011 09:59:31


HISTRIA D O B R A S I L

povo infeliz e analfabeto, incapaz de gerir seus prprios destinos. para a manuteno da ordem pblica... o perigo social represen-
O mecanismo seria imposio de votos da elite com a excluso do tado pelos pobres estava no imaginrio poltico brasileiro de fins
voto universal, uma vez que as camadas baixas constituam-se de do sculo XIX.... (CHALHOUB, Sidney. Cidade Febril:cortios
massas de manobra em pocas eleitorais pelas oligarquias. e epidemias na corte imperial. So Paulo:Companhia das Letras,
O ideal nacional era tomado como bandeira pelos tenen- 1996. Pgina 29.) Segundo o fragmento acima, as classes pobres e
tes, que monopolizaram o nacionalismo e se identificaram com perigosas representavam perigo social para os poderes institudos
a reao contra o estado atual das coisas. A Ptria s estaria salva nos finais do sculo XIX no Brasil. Analise os supostos perigos
quando o bacharelismo vazio e inconsequente tivesse sido varrido que essas classes podiam representar:
das escolas; quando as oligarquias no pudessem mais controlar I. Um grande risco que as classes perigosas poderiam oferecer
os bacharis e deixassem de identificar seus interesses de classe seria interferir na organizao do trabalho, embora a oferta
com os nacionais. A oligarquia cafeeira, em especial a paulista, era de empregos fosse ampla, a vadiagem lhes parecia mais
responsabilizada pelo estado crtico em que se encontrava o pas. atrativa.
O combate hegemonia cafeeira atraiu o apoio das oligarquias II. O perigo social oferecido pelos pobres aparecia atravs da
dissidentes regionais que viam no movimento uma forma de acu- metfora das doenas que eles poderiam transmitir em
mular foras para a derrubada dos donos do poder. virtude das suas condies de moradia e higiene.
Os tenentes eram das mais variadas origens sociais. Provi- III. As classes pobres eram tambm perigosas na sua
nham desde as oligarquias nacionais at as camadas mdias. O fato possibilidade de reproduo, pois continuariam a se
de o movimento ter nascido no seio das Foras Armadas isolou-o reproduzir, crianas pobres que permanecem expostas aos
do apoio das camadas urbanas. Apesar de o iderio tenentista, vcios de seus pais.
ser apoiado por essas camadas, a origem elitista dos tenentes
correto o que se afirma em
fazia-os impermeveis participao de outros elementos. Por isso
a) I, II e III. c) apenas em II e III.
no obtiveram respaldo quando fizeram a Revolta do Forte de Co-
b) apenas em I. d) apenas em II.
pacabana, em 1922 e as revoltas de 1924, estouradas em So Pau-
lo, Rio Grande do Sul, Sergipe, Par, Mato Grosso e Amazonas. A
02. (UFC 2010)
Coluna Prestes no conseguiu levantar o povo contra o governo.
Morte gordura!
Em 5 de Julho de 1922, os tenentes tramaram uma revolta
morte s adiposidades cerebrais!
para depor o Presidente Epitcio Pessoa, no entanto, as unida-
Morte ao burgus-mensal!
des sediciosas foram dominadas, com exceo do Forte de Co-
ao burgus-cinema! ao burgus-tlburi!
pacabana. Isolados, os oficiais agruparam-se em uma formao
Padaria Suissa! Morte viva ao Adriano!
de voluntrios e 17 deles saram s ruas para enfrentar milhares
Ai, filha, que te darei pelos teus anos?
de soldados legalistas. A adeso de um civil, transformou-se em
Um colar... Conto e quinhentos!!!
os 18 do Forte, que foram detidos. S escaparam dois feridos.
(...) Fora! Fu! Fora o bom burgus!...
Eduardo Gomes e Siqueira Campos, os demais morreram. Esta
foi a primeira manifestao do tenentismo. Outras estavam O trecho acima, transcrito do poema Pauliceia Desvairada, de
reservadas da pra frente. Mrio de Andrade, foi recitado na Semana de Arte Moderna,
Em 5 de Julho de 1924, mais de mil militares ligados ao te- realizada de 11 a 18 de fevereiro de 1922, no Teatro Municipal de
nentismo tomaram pontos estratgicos da cidade de So Paulo, So Paulo. Sobre esse movimento, correto afirmar que:
em mais uma tentativa de derrubar a repblica oligrquica. Desta a) teve como princpio uma arte baseada na esttica romntica
feita, sob o comando do General Isidoro Dias Lopes e do Coronel e realista.
Miguel Couto, os tenentistas tentavam depor o Presidente Artur b) tentou traduzir a cultura e os problemas nacionais atravs da
Bernardes. Apesar de conseguirem controlar a cidade de So Paulo arte.
por trs semanas, os tenentistas foram novamente derrotados. Aps c) gerou uma valorizao da arte europeia em detrimento da
abandonar So Paulo, foi formada a Coluna Paulista, que se enca- arte brasileira.
minhou para a cidade de Bauru. Enquanto isso, no Rio Grande do d) foi uma tentativa de renovar as manifestaes artsticas no
Sul, outro jovem oficial estava descontente com os rumos da pol- Brasil Imprio.
tica brasileira: o Capito Luis Carlos Prestes, comandante de um e) foi um grupo de poetas e escultores que reafirmaram o
destacamento de engenharia sediado em Santo ngelo. O futuro parnasianismo no Brasil.
Cavaleiro da Esperana liderou um levante militar e dirigiu-se
para a cidade de Foz do Iguau, no Paran, onde se encontrou com 03. (UFC 2010) A travessia para o Juazeiro fez-se a marchas
a Coluna Paulista, dando origem Coluna Prestes. foradas, em quatro dias. E quando l chegou o bando dos expe-
A Coluna chegou a reunir mais de mil e quinhentos homens, dicionrios, fardas em trapos, feridos, estropiados, combalidos,
que marcharam por mais de vinte e cinco mil quilmetros, ao davam a imagem da derrota. Parecia que lhes vinham em cima,
longo de treze estados, visando a sublevar os camponeses para nos rastros, os jagunos. A populao alarmou-se, reatando o
lutarem contra os desmandos da Repblica Velha. No entanto, xodo. Ficaram de fogos acesos na estao da via-frrea todas as
mesmo no tendo sido derrotado em nenhum dos 53 combates locomotivas. Arregimentaram-se todos os habitantes vlidos dis-
em que se envolveu, contra tropas do exrcito legalista, milcias postos ao combate. E as linhas do telgrafo transmitiram ao pas
estaduais e at mesmo jagunos, os objetivos da coluna no foram inteiro o preldio da guerra sertaneja O trecho acima parte do
alcanados, por isso, depois do incio da marcha, os revoltosos, livro de Euclides da Cunha que teve sua primeira edio em 1902
exaustos, se asilaram na Bolvia, pondo fim ao mais emblemtico e relata o cotidiano de um conflito ocorrido nos primeiros anos
dos movimentos tenentistas. da Repblica. O livro de Euclides e o conflito ao qual se refere
so respectivamente:
a) Inferno Verde, Caldeiro.
E X E R C C I O b) Inferno Verde, Cabanagem.
c) Os Sertes, Canudos.
01. (UECE 2010/1) As classes pobres no passaram a ser vistas d) Os Sertes, Caldeiro.
como classes perigosas apenas porque poderiam oferecer problemas e) A guerra do fim do Mundo, Contestado.

UECEVEST 159

Apostilas UECEVEST mod3.indb 159 06/02/2011 09:59:31


H I S T RIA DO BRASIL

04. (UEL 2010) Leia os textos I e II e responda questo seguinte ra, atravs da parte mais habitada da cidade, um corredor que ia
Texto I da Prainha ao Passeio Pblico. (CARVALHO, Jos Murilo de.
preciso compreender que a vacinao um objeto de difcil Os bestializados: o Rio de Janeiro e a Repblica que no foi. So
apreenso, constituindo-se, na realidade, em um fenmeno de Paulo: Companhia das Letras, 1987, p. 37.) Sobre as obras p-
grande complexidade onde se associam e se entrechocam crenas blicas descritas na citao acima, assinale a alternativa correta.
e concepes polticas, cientficas e culturais as mais variadas. A a) Faziam parte do projeto republicano de remodelao
vacinao tambm, pelas implicaes socioculturais e morais urbanstica da capital brasileira, para destruir os vestgios
que envolve, a resultante de processos histricos nos quais so fsicos da colonizao portuguesa.
tecidas mltiplas interaes e onde concorrem representaes an- b) Eram parte do programa nacional de industrializao, que
tagnicas sobre o direito coletivo e o direito individual, sobre as pretendia transformar a cidade do Rio de Janeiro no maior
relaes entre Estado, sociedade, indivduos, empresas e pases, plo industrial da Amrica Latina.
sobre o direito informao, sobre a tica e principalmente so- c) Foram postas em prtica sob a motivao de ideais
bre a vida e a morte. (Adaptado de: PORTO, A.; PONTE, C. higienistas e de modernizao e aformoseamento do espao
F. Vacinas e campanhas: imagens de uma histria a ser contada. urbano, caractersticos daquele perodo.
Histria, Cincias, Sade. Manguinhos, vol. 10 (suplemento 2). d) Ocorreram graas s aes reivindicatrias da populao
p. 725-742. 2003.) mais pobre do Rio de Janeiro, que reclamava melhorias em
suas condies de moradia e transporte.
Texto II e) Foram realizadas com o objetivo de fixar as camadas
populares na regio central do Rio de Janeiro, impedindo
que migrassem para as reas nobres da zona norte.

06. (UECE 2009/1) A Guerra de Canudos, ocorrida entre


1896 e 1897, mobilizou metade do efetivo do Exrcito Brasi-
leiro. O momento coincidia com o fim do governo do General
Floriano Peixoto e a tomada do poder pelos civis, liderados pela
elite paulista. (Fonte: RODRIGUES, Rogrio Rosa. Por um
Novo Exrcito. Revista da Biblioteca Nacional, n.36, setembro
de 2008, pp. 58-61.) Em relao imagem construda pelo
exrcito brasileiro no perodo em tela, marque a opo correta.
a) Os artigos escritos pelo escritor Euclides da Cunha bem
como seu livro Os Sertes transformaram-se num
instrumento de excelente publicidade para o exrcito
brasileiro, em virtude da atuao do mesmo na guerra.
(O Malho. Revista 126. Rio de Janeiro, 11 fev. 1905.) b) Durante todo o tempo que durou a guerra de Canudos, o
No Brasil a vacina esteve no centro de um grande embate exrcito brasileiro gozou de grande popularidade. Porm,
social no incio do sculo XX, denominado Revolta da Vacina, continuava arcaico em relao a equipamentos blicos.
ilustrado na charge acima. c) No momento da guerra, o exrcito brasileiro ainda colhia
Sobre a Revolta da Vacina, correto afirmar que foi os louros da sua participao no movimento que levou
a) um movimento cuja base social eram os trabalhadores independncia do Brasil, embora fosse, ainda, dependente
imigrantes pobres no reconhecidos pelo Estado brasileiro das malhas administrativas do exrcito portugus.
como portadores de direitos sociais e, portanto, excludos d) Para a campanha de Canudos, os alistamentos eram feitos
da campanha de vacinao em massa proposta por Oswaldo lao, denominao popular atribuda s convocaes
Cruz. foradas pelo exrcito aos jovens brasileiros.
b) uma mobilizao popular que reivindicava ao governo
Rodrigues Alves polticas de sade pblica, em particular o 07. (UECE 2009/1) A Coluna Prestes, entre 1924 e 1927, per-
combate a doenas como febre amarela, peste bubnica e correu uma vasta extenso do interior brasileiro, na tentativa de
varola. manter a luta pelos seus ideais. Como soluo para os proble-
c) deflagrada em razo dos altos custos financeiros dos mas brasileiros, os lderes da Coluna defendiam
medicamentos e das vacinas contra a varola e a febre a) o estabelecimento de uma ditadura militar que alinhasse o
amarela, ento acessveis apenas s camadas sociais mdias Brasil s experincias do nazismo alemo.
urbanas e s elites rurais. b) uma ampla e irrestrita reforma agrria, com distribuio
d) uma reao das classes populares a um conjunto de medidas das terras dos grandes latifndios entre os camponeses.
sanitrias, entre as quais uma reforma urbana (eliminao de c) a realizao de uma revoluo comunista, seguida da
cortios, construo de ruas e avenidas largas), realizada com estatizao de todas as propriedades particulares.
truculncia por funcionrios do governo federal. d) a destruio do sistema oligrquico, acompanhada da
e) uma iniciativa dos intelectuais positivistas brasileiros para os reformulao dos costumes e das prticas polticas vigentes.
quais aquelas medidas de sade pblica, voltadas s camadas
pobres da populao, deveriam ser obrigatrias. 08. (IFCE 2009) A oposio entre o catolicismo oficial e o
rstico, praticado pelos sertanejos, fica bem patente no dilogo
05. (UFC 2009) Leia o texto a seguir. Em novembro de 1904, entre o frei Rogrio Neuhaus, franciscano de origem germnica,
data da revolta [da Vacina], o trabalho de demolio das ca- e o monge Joo Maria, ocorrido em dezembro de 1897: Joo
sas para abrir a avenida Central, executado por cerca de 1800 Maria exclamou: A minha reza vale tanto quanto uma missa!
operrios, terminara, e 16 novos edifcios estavam sendo cons- Impossvel! repliquei. Nem as oraes de Nossa Senhora
trudos. O eixo central da avenida fora inaugurado em 7 de tm o valor de uma missa, pois, nesta, Jesus Cristo vem descen-
setembro, em meio a grandes festas, j com servios de bonde e do sobre o altar. O movimento da Repblica Velha, a que o
iluminao eltrica. A derrubada de cerca de 640 prdios rasga- texto se refere,

160 UECEVEST

Apostilas UECEVEST mod3.indb 160 06/02/2011 09:59:32


HISTRIA D O B R A S I L

a) A Guerra dos Farrapos. d) Revolta da Chibata. Revoluo de 1930


b) Canudos. e) O Contestado. A questo sucessria do Presidente Washington Lus provo-
c) Caldeiro. cou uma ciso da oligarquia dominante e foi o estopim de uma
revoluo que derrubou a Repblica Velha. A partir dessa revolu-
09. (IFCE 2007/1) Vrias revoltas populares ocorreram na Repbli- o, o governo passou a ser exercido por outro bloco de poder, do
ca Velha (1889/1930). Dentre elas, uma ocorreu, porque o governo qual participaram as mais diversas faces da classe dominante:
concedeu extensas reas de terras ao grupo do norteamericano Per- burguesia industrial, comercial, financeira, aristocracia agrria
cival Farquhar, a BRAZIL RAILWAY COMPANY, que construa etc. Pode-se afirmar que nenhuma dessas faces conseguiu im-
estradas e expulsava os sertanejos de suas terras. Essa revolta foi: por a sua hegemonia sobre o processo poltico.
a) a Revoluo Federalista ocorrida no governo de Floriano Para as eleies presidenciais que escolheria o sucessor de Wa-
Peixoto. shington Lus era a vez de Minas Gerais indicar o candidato, mas
b) liderada pelo padre Ccero, que defendia os sertanejos e o presidente, ligado oligarquia de So Paulo, indicou o paulista
combatia a poltica de submisso da nossa economia ao Jlio Prestes, que garantiria a continuidade da poltica de valo-
capital norte-americano. rizao do caf. A crise mundial de 1929 afetou sobremaneira a
c) a Revolta de Canudos, que ficou mais conhecida em toda cafeicultura, pois houve queda do preo do caf.
a histria republicana, porque resistiu at o esgotamento Os mineiros que esperavam a indicao de Antnio Carlos de
completo, exemplo nico em toda a Histria. Andrade, presidente de Minas Gerais, romperam sua aliana com
d) a Guerra do Contestado, cujo lder popular era o beato Jos So Paulo e, juntamente com o Rio Grande do Sul e a Paraba,
Maria. criaram uma coligao, a aliana Liberal, que lanou a candidatu-
e) um movimento forte e organizado que provocou a queda do ra de Getlio Vargas, ex-ministro da Fazenda e presidente do Rio
governo Washington Lus. Grande do Sul. O candidato a vice-presidente era o paraibano Joo
Pessoa. A Aliana Liberal, em sua campanha poltica, concentrou
10. (UFC 2005) A histria do Brasil nas primeiras dcadas do suas foras nos grandes centros urbanos, buscando, assim, a adeso
Sculo XX foi marcada por vrios movimentos sociais de base po- das novas classes sociais emergentes: a burguesia industrial e o ope-
pular, entre eles a Revolta da Chibata, ocorrida no Rio de Janeiro rariado. Aos operrios prometia satisfazer suas reivindicaes, tais
em 1910, um levante de marujos dos encouraados Minas Gerais como: frias remuneradas, regulamentao do trabalho feminino
e So Paulo. Sobre esse movimento, assinale a alternativa correta: e infantil e ampliao do direito aposentadoria. Alm disso, suas
a) Os revoltosos foram presos e, na priso, organizaram uma nova propostas de anistia aos presos polticos e de instituio do voto
rebelio, sendo massacrados pelo Exrcito e pela Marinha. secreto trouxeram-lhe o apoio dos lderes tenentistas.
b) O movimento teve uma forte influncia anarcossindicalista A eleio geriu-se em maro de 1930. A mquina da oligar-
propugnando o fim das arbitrariedades a que eram quia funcionou tanto em So Paulo como em Minas. Sim, por-
submetidos os marinheiros. que parte dos mineiros tambm apoiou Jlio Prestes. Sua vitria
c) O governo federal acatou as reivindicaes apresentadas pelo foi incontestvel dentro dos padres da Repblica Velha. Vargas,
movimento, e os revoltosos, foram anistiados, concluindo-se Borges de Medeiros, Joo Pessoa e outros oligarcas aceitaram
assim o levante. tranquilamente a derrota. Afinal, o pacto que havia feito antes
d) As causas deste movimento foram os castigos corporais era o de respeitar o resultado das urnas. No entanto, em Julho,
sofridos pelos marinheiros e os sistema de promoes com o assassinato de Joo Pessoa, no Recife, em um crime passional,
que eram beneficiados os mestios. por um desafeto poltico, contribuiu para dar mais fora ideia
e) O marinheiro negro Joo Cndido foi considerado o nico do levante, porque, segundo frase atribuda a Antonio Carlos,
culpado pelo movimento, tendo sido julgado e condenado Faamos a Revoluo, antes que o povo a faa.
morte, como castigo exemplar. A revoluo de 3 de outubro de 1930 estourou no Rio Gran-
de do Sul, Minas e Paraba. O Governo Federal preparava-se para
enfrent-la. Decretou o Estado de Stio e convocou os reservistas.
G A B A R I T O Os governos do Nordeste caram sob a coluna revolucionria de
Juarez Tvora. Minas Gerais estava sob o domnio revolucionrio.
01. c 02. b 03. c 04. d 05. c
As tropas gachas avanavam para o Norte, sob a liderana de Gis
06. d 07. d 08. e 09. d 10. a Monteiro. Havia ameaa de conflito generalizado. Washington
Lus foi deposto por uma junta pacificadora, formada pelos oficiais
Tasso Fragosso, Mena Barreto e Isaas de Noronha, sendo detido
no Forte de Copacabana e enviado ao exlio. A junta transmitiu o
A ERA VARGAS (UECE/EnEm) poder ao chefe do movimento, Getlio Vargas, em novembro de
1930. Nos 15 anos em que governou o Brasil, Getlio foi chefe do
Governo Provisrio (1930-1934); presidente constitucional, eleito
por via indireta (1934-1937), e ditador de uma nova ordem auto-
ritria conhecida como Estado Novo (1937-1945).
Abria-se uma nova era para o pas. Apesar de no ter havido uma
ruptura no processo poltico, social e econmico brasileiro, dava-se
um passo decisivo no sentido da modernizao das instituies. Mais
uma vez as mudanas eram feitas de um modo inconstitucional.
Venceu a ala radical dos tenentes, partidria da aplicao da violn-
cia como forma de transfigurar o panorama geral do pas.

Governo provisrio (1930-1934)


Chargesobre a Revoluo de 30, quando Getlio Vargas pe Logo que assumiu o poder, Getlio Vargas suspendeu a Cons-
fim na Repblica Velha. tituio de 1891, dissolveu o Congresso Nacional, as Assembleias
Fonte: www.crv.educao.mg.gov.br Estaduais e as Cmaras Municipais e nomeou interventores para

UECEVEST 161

Apostilas UECEVEST mod3.indb 161 06/02/2011 09:59:33


H I S T RIA DO BRASIL

os estados, com amplos poderes. Dois novos ministrios foram Governo constitucional (1934-1937)
criados, o do Trabalho, Indstria e Comrcio e o da Educao e As eleies para a constituinte permitiram a volta normali-
Sade. Demonstrando seu autoritarismo, desde o incio, Vargas dade democrtica, interrompida pela Revoluo de 1930 e, pela
controlou os meios de comunicao e os sindicatos. Estes, para primeira vez, as eleies foram conduzidas por uma Justia Elei-
funcionar, precisavam da autorizao do Ministrio do Trabalho toral, criada durante o governo provisrio, com a inteno de
e a sua atividade poltica foi proibida. No seu governo, foram evitar fraudes e manipulaes, tpicas da Repblica Velha. Ou-
aprovadas algumas leis trabalhistas: regulamentao do trabalho tro fato indito foi a participao feminina nas eleies de 1934,
feminino e infantil; descanso semanal remunerado; frias remu- contando inclusive com a eleio da primeira deputada, a paulis-
neradas; e jornada de trabalho de oito horas dirias. ta Carlota Pereira. Em julho de 1934, foi promulgada a terceira
Constituio do Brasil e segunda da Repblica. So caractersti-
Revoluo constitucionalista (SP/1932) cas da Constituio de 1934:
A destituio da oligarquia paulista no passaria em brancas Atravs do voto direto e secreto, elegia-se o Poder Legislativo,
nuvens. O descontentamento da populao, ante s dificuldades composto de Senado e Cmara de Deputados.
econmicas, foi capitalizado pelos polticos. So Paulo levantou Mantinha o regime federativo, presidencial e a diviso em trs
a bandeira contra o excessivo centralismo preconizado pelos te- poderes.
nentes. A reao iniciou-se com a exigncia de um interventor O Presidente seria eleito para um mandato de 4 anos.
civil e paulista, uma vez que Joo Alberto no era nem uma coisa O 1 Presidente seria eleito indiretamente pela prpria Cons-
nem outra. O bairrismo exacerbado serviu como elemento de tituinte.
agitao popular. Ao Presidente cabia o direito de sugerir emendas Constituio.
Extinguiu-se o cargo de Vice-Presidente da Repblica.
Cada Estado possuiria 2 senadores, inclusive, o Distrito Federal.
O nmero de deputados por Estado seria proporcional po-
pulao.
Os sindicatos teriam o direito de eleger os deputados profis-
sionais.
O mandato de deputado passaria a 4 anos.
Uma srie de outras medidas esto implcitas no texto consti-
tucional, como: restries s imigraes estrangeiras; estabili-
zao de empresas nacionais e nacionalizao das estrangeiras
consideradas prejudiciais; necessidade de concordncia estatal
para a explorao de minrios; proibio de propriedade de r-
gos de divulgao a estrangeiros etc.

Polarizao ideolgica
A dcada de 30 marcou a polarizao ideolgica no mundo,
Em maro, o paulista e civil Pedro de Toledo nomeado in- notadamente na Europa. A onda esquerdizante foi combatida
terventor, conforme desejado pelos paulistas. Uma vez atendi- pela reao direitista, que se impunha sob vrias formas tanto na
dos, os paulistas levantaram a bandeira da reconstitucionalizao. Europa Ocidental como na Centro-Oriental. A onda nazi-fascis-
Visto que a Constituio de 1891 estava suspensa, exigiram a ta foi importada pelo Brasil. Em 1932 nascia a Ao Integralista
imediata convocao de uma Assembleia Constituinte, com a Brasileira (AIB), cujo lema era o Deus, Ptria, Famlia. Muitas
volta ao Estado de direito. O Partido Democrtico lanou um marchas, hinos, frases retumbantes e tudo o mais. O chefe era
manifesto constitucionalista, que repercutiu em vrios Estados. Plnio Salgado, ex-integrante do PRP. A AIB era um movimento
O Governo Federal estava em choque. Os tenentes eram contra de extrema-direita, pregando o Estado Integral, que no era outra
qualquer constitucionalizao, pois acreditavam ser uma pura e coisa, seno o Estado Corporativo, monopartidrio ditatorial e
simples volta ao passado, que tornaria nula a Revoluo de 30. de nico chefe. Acusando a esquerda de se infiltrar em todas as
Em fevereiro de 1932, o Partido Democrtico e o velho PRP camadas da populao, hasteavam a bandeira do anti-socialismo
reconciliaram-se e fundaram a Frente Unida Paulista. Mais e anticomunismo. O movimento exteriorizou-se de acordo com
uma vez a unidade ideolgica estava acima das divergncias par- o padro nazi-fascista. Os correspondentes dos camisas-negras
tidrias. Em uma das manifestaes contra o governo de Getlio, de Mussolini e da SS, de Hitler, eram os camisas-verdes. O cor-
foram mortos nas cidade de So Paulo os estudantes Martins, respondente do Fascio e da Swastika era o Sigma. Do Duce e
Miragaia, Druzio e Camargo, que deu origem sigla MMDC, do Fuher era o chefe nacional. Do Ave! e Sieg Hei! era o
que foi usada para denominar um grupo radical contra Getlio. Anau!. Houve desfiles, pancadarias e perseguies aos cidados
Em 9 de Julho de 1932 eclodiu a Revoluo Constituciona- acusados de comunistas.
lista. As foras paulistas foram comandadas pelo General Isidoro O Partido Comunista Brasileiro, por sua vez, inspirava-se
Dias Lopes. Depois de cerca de trs meses de revoluo, os paulis- no modelo sovitico-stalinista. Desde o exlio de Trotsky, aps
tas foram derrotados pelas tropas federais. Entretanto, algo havia a vitria de Stalin, a URSS dominava a Terceira Internacional
mudado. Vargas concluiu que no poderia contar com os tenen- Proletria ou Komintern. A tese estalinista do comunismo nacio-
tes se pretendesse instalar-se definitivamente no poder. Assim nal havia vencido. Stalin construa o socialismo russo calcado na
sendo, passou a apoiar-se nas altas cpulas estaduais e militares. represso e nos campos de concentrao, onde pereceram alguns
A derrota militar de So Paulo foi seguida de uma vitria poltica. milhes de inconformados. Os partidos comunistas gravitavam
Vargas viu-se impelido a convocar a Constituinte de acordo com em torno das teses e orientaes provenientes de Moscou. E Mos-
o Cdigo Eleitoral, em maio de 1933. A economia cafeeira havia cou ordenava a criao de frentes amplas para combater o lati-
sido desarticulada completamente com a Revoluo de 1932, e fndio, o imperialismo e o fascismo. O rgido esquema stalinista
o Governo Federal foi obrigado a prestar-lhe ajuda. De comum foi seguido risca no Brasil. O PCB organizou uma frente ampla
acordo com a oligarquia paulista, foi indicado um novo interven- sob a denominao de Aliana Nacional Libertadora (ANL), em
tor, Armando de Sales Oliveira. 1935. Era um conglomerado de tendncias polticas de esquerdas

162 UECEVEST

Apostilas UECEVEST mod3.indb 162 06/02/2011 09:59:33


HISTRIA D O B R A S I L

e esquerdizantes. Houve o auxlio de delegados do Komintern e procurou apoio das Foras Armadas e dos governadores. Diante
para a formao de um governo popular democrtico. O chefe do fantasma do comunismo, foi fcil conseguir a unio geral,
da ANL era Lus Carlos Prestes, ex-tenente, um dos lderes que apesar da posio de alguns, como o governador paulista Arman-
com a Coluna havia percorrido o serto ao tempo de Arthur Ber- do de Salles Oliveira, que julgavam possvel afastar o comunismo
nardes e Washington Lus. Os movimentos extremistas prolifera- sem cancelar as eleies.
vam rapidamente. Os choques eram inevitveis. Getlio estava Getlio tinha tanta certeza do sucesso do golpe, que seu minis-
atento para tirar proveito da conjuntura poltica. tro da Justia, Francisco Campos, j redigia em segredo a nova Car-
Em novembro de 1935, os comunistas, agrupados em torno ta. Com apoio militar, o golpe veio em 10 de novembro de 1937,
da Aliana Nacional Libertadora (ANL), iniciaram um levante no sem resistncias. s dez da manh, Getlio, j ditador, instaurou
Rio Grande do Norte, com desdobramentos em Pernambuco e o Estado Novo e apresentou aos ministros a nova Constituio, a
Rio de Janeiro, chamado Intento na Comunista. Vencidos, seus l- polaca, pois se baseava na Constituio da Polnia. Ela concentrou
deres foram presos, entre eles Lus Carlos Prestes. Este passaria dez todos os poderes nas mos do presidente, autoridade suprema do
anos confinado numa solitria e submetido a tratamento to desu- Estado, que condena os rgos representativos de graus superiores,
mano pela ditadura que o advogado Sobral Pinto chegou a invocar dirige a poltica interna e externa, promove ou orienta a poltica
a lei de proteo aos animais em favor de seu cliente. Enquanto a legislativa de interesse nacional e superintende a administrao do
sua companheira, a Judia-alem Olga Benrio foi deportada para pas. As principais modificaes introduzidas foram as seguintes:
a Alemanha Nazista, mesmo estando grvida de uma filha de Pres- O presidente podia dissolver o Congresso e expedir decretos-lei;
tes. Olga morreu em um campo de concentrao em 1941. Os partidos foram extintos;
Aboliu-se a liberdade de imprensa e instituiu-se a censura prvia;
O estado novo (1937-1945) Interventores passaram a governar os Estados;
Instituiu-se a pena de morte;
O mandato presidencial foi prorrogado at a realizao de um
plebiscito, que jamais aconteceria.

Para censurar a imprensa, orientar a opinio pblica e fazer a


propaganda do regime, foi criado o DIP: Departamento de Im-
prensa e Propaganda. Alm disso, O DIP tambm era o canal de
comunicao entre Vargas, o Estado Novo e o povo. A avalanche
publicitria inunda o pas atravs de todos os canais de comuni-
cao possveis. criada a Hora do Brasil, programa radiofnico
de mbito nacional. As diretrizes ideolgicas eram infiltradas por
toda a parte, tomando como tema o anticomunismo exacerbado
e a figura paternalista de Vargas, o salvador da Ptria. A represso
era feita pela polcia secreta, temida pelos seus mtodos que lem-
bravam a Gestapo do III Reich. Seu chefe era Filinto Muller, to
temido quanto a prpria polcia.
A centralizao se estendeu economia, com a criao de uma
Na caricatura, referente ao perodo 1934 - 1937, v-se o srie de institutos centrais, para planejar e controlar a produo
presidente Getlio Vargas, em frente ao Palcio do Catete, nacional: Instituto do Pinho, do Sal, do Acar e do lcool, do
espalhando cascas de banana, que podem ser interpretadas Cacau. A criao do DASP Departamento Administrativo do
como armadilhas Servio Pblico permitiu a centralizao administrativa e o con-
trole do aparelho burocrtico do Estado. Sua funo era supervi-
A represso ao movimento comunista de 1935 reforou o au- sionar o comportamento dos interventores estaduais. Constitua-
toritarismo de Vargas. Com apoio de amplos setores do Exrcito se em um repositrio de apadrinhados, apaniguados, ineptos e
e das classes dominantes e, em parte, inspirado no nazi-fascismo, gente que nada sabia fazer. Um verdadeiro cabide de empregos.
ele passou a conspirar para se perpetuar no poder. A situao in-
ternacional favorecia a soluo autoritria. Fascismo e nazismo,
na poca, pareciam a muita gente regimes dinmicos, melhores
que as democracias decadentes. A desumanidade daqueles sis-
temas, os mtodos violentos que seus lderes usavam para chegar
ao poder e manter-se nele, a implacabilidade contra os inimigos
polticos, o expansionismo, o controle absoluto sobre a vontade
dos cidados, tudo isso s seria percebido quando eclodisse a II
Guerra Mundial, que estudaremos no prximo captulo.
Quando se aproximou o fim do mandato de Vargas, ele e os
setores que o apoiavam trataram de encontrar um meio de sus-
pender as eleies marcadas para 3 de janeiro de 1938. Enquanto
Jos Amrico de Almeida, Armando de Salles Oliveira e Plnio Reproduo da primeira Carteira Profissional emitida no
Salgado se preparavam para a campanha eleitoral, Getlio articu- Brasil, em nome de Getlio Vargas. (O Globo, 22/04/2004)
lava o golpe que lhe daria poderes ditatoriais. Tal como haviam
feito os nazistas na Alemanha, os seguidores de Vargas usaram A maior preocupao do Estado Novo foi com a organizao
como pretexto a ao dos comunistas embora estivessem eles dos trabalhadores em sindicatos. J comentamos que no Esta-
quase todos presos. Chegou-se a elaborar um plano falso, o Plano do fascista italiano o corporativismo se prestava ao Estado para
Cohen, que previa uma revoluo comunista e o assassinato de controlar os operrios. Vargas utilizou-se da Carta do Trabalho
centenas de polticos brasileiros. Getlio decretou estado de guer- italiana para montar o prprio esquema de controle sobre as ca-
ra, que lhe permitia prender qualquer pessoa sem ordem judicial, madas e o patronato. Algumas leis trabalhistas, conseguidas na

UECEVEST 163

Apostilas UECEVEST mod3.indb 163 06/02/2011 09:59:34


H I S T RIA DO BRASIL

Europa custa de reivindicaes violentas, foram introduzidas no Setores da classe dominante, do capital internacional e das
Brasil como uma concesso feita pelo Estado. Assim, no amadu- Foras Armadas, no entanto, no queriam mais Getlio: ele no
receu o sentido reivindicatrio. Antes que o proletariado reivin- representava mais seus interesses e por isso retiraram o apoio que
dicasse, o governo concedeu. Mas isso implicava o fortalecimento vinham dando ao Estado Novo. Aproveitando o pretexto da no-
pessoal de Vargas que, por sua vez, era recebido como o grande meao de Benjamim Vargas, irmo do presidente, para o cargo
patrono dos trabalhadores. O patronato via na ao do governo de chefe de polcia do Distrito Federal, em 29 de outubro de
uma forma de atenuao do conflito entre capital e trabalho. Al- 1945, os chefes militares obrigaram Getlio a deixar o governo.
guns setores radicais resistiram a essas concesses, porm a maio- A presidncia foi entregue ao ministro Jos Linharres, presi-
ria entendia que, caso se mantivessem inflexveis, de uma forma dente do Supremo Tribunal Federal. Getlio retirou-se para sua
ou de outra as leis trabalhistas seriam implantadas. estncia em So Borja (Rio Grande do Sul). Nas eleies, reali-
zadas em 2 de dezembro de 1945, elegeu-se presidente o general
O Brasil na segunda guerra mundial Eurico Gaspar Dutra, ministro da guerra no governo de Getlio
At a dcada de 1940, as simpatias de Vargas e de muitos e por ele apoiado discretamente.
membros de sua equipe se dirigiam ao bloco fascista europeu,
autoritrio como o modelo poltico brasileiro. A tendncia se for-
taleceu quando a II Guerra Mundial comeou, devido ao sucesso E X E R C C I O
inicial das foras do Eixo.
Getlio demorou a decidir-se em relao guerra. A opinio 01. (UECE 2010/1) Analise as afirmaes abaixo que remetem
pblica ento passou a pressionar o governo em favor dos Alia- ao papel assumido pelo ditador Getulio Vargas, enquanto chefe
dos; os Estados Unidos tambm: entraram no conflito em 1941 de estado, durante a Segunda Guerra Mundial.
e queriam instalar uma base area no litoral do Rio Grande do I. Inicialmente simpatizante das forcas do Eixo, apos o
Norte. Vargas queria construir uma usina siderrgica. Tinha che- bombardeio de navios brasileiros, foi obrigado a fechar
gado a oportunidade. Em troca da concesso para a base area e alianas com os grupos de pases aliados.
do apoio brasileiro aos Aliados, os Estados Unidos financiariam a II. Alinhou-se contra as foras do Eixo a partir de 1939, em
siderrgica de Volta Redonda. virtude da ausncia de sintonia do seu governo com os
O afundamento dos cinco navios brasileiros, supostamente governos totalitrios.
por submarinos alemes, reforou a deciso, em 1942. O povo III. Inicialmente adotou uma poltica dbia, ora negociando
foi s ruas protestar. Vargas, pressionado, rompeu relaes com os com os Estados Unidos, ora com a Alemanha. Mas, as
pases do Eixo e declarou guerra Alemanha e Itlia, em 22 de derrotas sucessivas das foras do Eixo, entre outras coisas, o
agosto. Em meados de 1944, partiu a Fora Expedicionria Brasi- fizeram decidir-se a favor dos aliados.
leira (FEB), que se incorporou ao exrcito americano e combateu Est correto o que se afirma
no norte da Itlia. O Brasil perdeu 2.000 soldados e 37 navios. a) em I, II e III. c) apenas em II e III.
b) apenas em I e II. d) apenas em I e III.
Redemocratizao
Havia uma contradio na participao brasileira: enquan- 02. (UECE 2010/2) Sobre o papel dos Tenentes aps a Revo-
to a FEB lutava na Europa contra regimes totalitrios e ao lado luo de 1930, so feitas as seguintes afirmaes.
de pases democrticos, no Brasil havia um regime ditatorial. O I. Aps a vitria da Revoluo de 1930, os Tenentes passaram
alinhamento do Brasil com os Aliados contribuiu decisivamente a fazer parte do governo, embora, posteriormente, tenham se
para o fim do Estado Novo. decepcionado com os rumos que o movimento tomou.
As manifestaes pelo fim da ditadura e a favor de eleies II. Os Tenentes tiveram contra si a grande maioria da
comearam antes mesmo de terminar a guerra. Vargas, correndo populao de So Paulo que ideologicamente girava em
contra o relgio, fez uma reforma constitucional em fevereiro de torno das elites regionais que combatiam.
1945: regulamentou o alistamento e as eleies para a presidncia III. O Nordeste foi um dos espaos de atuao dos Tenentes.
da Repblica, os governos estaduais, o Congresso Nacional e as Muitos deles eram oriundos dessa regio onde o poder
assembleias legislativas. A seguir, anistiou os presos polticos, o dominado por pequenos grupos era flagrante.
que favoreceu o debate pblico e a organizao de partidos. O
quadro partidrio ficou assim constitudo: Est correto o que se afirma
UDN Unio Democrtica Nacional, representando a oposi- a) em I, II e III. c) em I, apenas.
o liberal. Seu candidato presidncia era o brigadeiro Edu- b) em II e III, apenas. d) em I e III, apenas.
ardo Gomes.
PSD Partido Social Democrtico, reunia a oligarquia minei- 03. (UECE 2010/1) Acerca da Constituio outorgada em 1937
ra e lanava o general Eurico Gaspar Dutra. por Getlio Vargas, podemos afirmar corretamente que
PTB Partido Trabalhista Brasileiro, liderado por Vargas, par- a) aboliu o cargo de vice-presidente da Repblica.
tidrio do seu continuismo. b) estabeleceu a diviso do pas em provncias.
PCB Partido Comunista Brasileiro, lanava a candidatura de c) instituiu a separao entre a Igreja e o Estado.
Yedo Fiza Presidncia. d) propiciou o voto feminino.
PRP Partido de Representao Popular, remanescente do in-
tegralismo e liderado por Plnio Salgado. 04. (UECE 2005/2) A dcada de 1920 terminou presenciando
uma das poucas campanhas eleitorais da Primeira Repblica em
Vargas conseguiu apoio de setores trabalhistas e dos pr- que houve autntica competio para o cargo da Presidncia.
prios comunistas, graas s medidas tomadas em benefcio dos (Fonte: CARVALHO, Jos Murilo. Marco Divisrio. In Cida-
trabalhadores e sua poltica nacionalista (criao do Conselho dania no Brasil: o longo caminho. Rio de Janeiro: Civilizao
Nacional do Petrleo, Companhia do Vale do Rio Doce etc). Brasileira, 2001, p 89 a 126.) Os dois candidatos Presidncia
Grupos se organizaram em defesa dele. O movimento ficou da Repblica no perodo em questo eram:
conhecido como Queremismo: Queremos Getlio!, era a a) Washington Lus e Getlio Vargas.
palavra de ordem. b) Washington Lus e Jlio Prestes.

164 UECEVEST

Apostilas UECEVEST mod3.indb 164 06/02/2011 09:59:34


HISTRIA D O B R A S I L

c) Hermes da Fonseca e Getlio Vargas. O fragmento acima expressa UMA viso do mundo do trabalho
d) Getlio Vargas e Jlio Prestes. nas dcadas de 1930/1940 no Brasil. Neste sentido, podemos
afirmar que o fragmento acima sugere
05. (UFC 2008) Leia o texto a seguir. a) O investimento do Governo Vargas em melhorias
Permita Va. Excia. que uma pobre e humilde funcionria significativas para os trabalhadores, contrariando os
postal suba, diretamente, presena de Va. Excia. para solicitar interesses das elites econmicas brasileiras.
sua decisiva proteco para um acto que tambm de justia. b) A dificuldade do estado Getulista em promover o controle
(...) Alis, no fao seno cumprir os desejos de Va. Excia. que da classe trabalhadora, apesar de garantir-lhe certos direitos.
j declarou que no Estado Novo no existem intermedirios en- c) O estmulo ao assistencialismo do governo federal em
tre o governo e o povo. (Citado em FERREIRA, Jorge. Traba- relao aos trabalhadores e tambm aos seus sindicatos.
lhadores do Brasil: o imaginrio popular (1930-1945). Rio de d) As presses sofridas pela classe trabalhadora atravs de seus
Janeiro: FGV, 1997, p. 26.) Na carta supracitada, enviada em sindicatos para que apoiassem Vargas, incondicionalmente.
1938 ao Presidente da Repblica, Amerida de Mattos Diniz,
uma agente postal, solicita audincia, a fim de obter nomeao
para cargo concursado. A partir do trecho reproduzido, cor-
reto afirmar que:
a) os trabalhadores foram co-construtores do trabalhismo, 08. (ENEM 2010) De Maro de 1931 a Fevereiro de 1949, fo-
obtendo, assim, parte de suas reinvidicaes. ram decretadas mais de 150 novas leis de proteo social e de
b) a completa subordinao ideolgica das massas, fenmeno regulamentao do trabalho em todos os seus setores. Todas
tpico do populismo, fazia-se notar no Estado Novo. elas tem sido simplesmente uma ddiva do governo. Desde a,
c) a remetente se mostra possuidora de uma cultura poltica em o trabalhador brasileiro encontra nos quadros gerais do regime o
tudo oposta do restante dos trabalhadores. seu verdadeiro lugar. (DANTAS, M. A fora nacionalizadora do
d) o governo e os trabalhadores no eram cmplices, o que Estado Novo. Rio de Janeiro: DIP, 1942, Apud BERCITO S.R.
obrigava o primeiro a recorrer a pelegos para controlar os Nos tempos de Getlio: da revoluo de 1930 ao fim do Estado
segundos. Novo. So Paulo: Atual, 1990)
e) a ideologia do Estado Novo relevava o papel do Congresso A adoo de novas polticas pblicas e as mudanas jurdico-ins-
Nacional e dos partidos, instrumentos fundamentais numa titucionais ocorridas no Brasil, com a ascenso de Getlio Vargas
democracia. ao poder, evidenciam o papel histrico de certas lideranas e a
importncia das lutas sociais na conquista da cidadania. Desse
06. (UECE 2009/2) Construdo a partir de um golpe poltico- processo resultou a
militar e, portanto, carente de legitimidade, o regime inaugura- a) Criao do Ministrio do Trabalho, Indstria e Comrcio,
do por Vargas em 1930, disseminou por toda a sociedade uma que garantiu ao operariado autonomia para o exerccio de
produo de cunho poltico e cultural que afirmava a necessida- atividades sindicais.
de histrica de um novo governo. (Fonte: FERREIRA, Jorge. b) Legislao previdenciria, que proibiu migrantes de
Trabalhadores do Brasil: o imaginrio popular (1930-1945). Rio ocuparem cargos de direo nos sindicatos.
de Janeiro: Fundao Getulio Vargas, 1997, p. 21-24.) Sobre as c) Criao da Justia do Trabalho, para coibir ideologias
estratgias adotadas por este novo governo em relao aos tra- consideradas perturbadoras da harmonia social.
balhadores, so feitas as seguintes afirmaes: d) Legislao trabalhista que atendeu reivindicaes dos
I. Para os trabalhadores, o Estado nos anos 30 e 40 tornou-se operrios, garantindo-lhes vrios direitos e formas de
produtor de bens materiais e simblicos, a fim de obter dos proteo.
mesmos a aceitao e o consentimento ao regime poltico. e) Decretao da Consolidao das Leis do Trabalho (CLT),
II. Com base na formulao de uma legislao social e que impediu o controle estatal sobre as atividades polticas
trabalhista, o novo governo preconizava uma poltica de da classe operria.
valorizao do trabalhador que, no mbito do seu discurso,
visava elev-lo condio de cidado. 09. (ENEM 2009) A definio de eleitor foi tema de artigos nas
III. Este regime teceu sua auto-imagem induzindo os Constituies brasileiras de 1891 e de 1934. Diz a
trabalhadores a identificarem-no como guardio de seus Constituio da Repblica dos Estados Unidos do Brasil de
interesses. 1891:
Art. 70. So eleitores os cidados maiores de 21 anos que se
correto o que se afirma alistarem na forma da lei.
a) apenas em III. A Constituio da Repblica dos Estados Unidos do Brasil de
b) apenas em I e III. 1934, por sua vez, estabelece que:
c) apenas em I e II. Art. 180. So eleitores os brasileiros de um e de outro sexo,
d) em I, II e III. maiores de 18 anos, que se alistarem na forma da lei.
07. (UECE 2009/1) Leia com ateno o fragmento da cano a Ao se comparar os dois artigos, no que diz respeito ao gnero dos
seguir: eleitores, depreende-se que
Meu pai trabalhou muito a) a Constituio de 1934 avanou ao reduzir a idade mnima
Que j nas nasci cansado para votar.
Ai patro b) a Constituio de 1891, ao se referir a cidados, referia-se
Sou um homem liquidado tambm s mulheres.
No meu barraco chove c) os textos de ambas as Cartas permitiam que qualquer
Meu terno est furado cidado fosse eleitor.
Ai patro d) o texto da carta de 1891 j permitia o voto feminino.
Trabalhar, no quero mais. e) a Constituio de 1891 considerava eleitores apenas
(Wilson Batista, Nasci Cansado, Sucesso dos anos 1930). indivduos do sexo masculino.

UECEVEST 165

Apostilas UECEVEST mod3.indb 165 06/02/2011 09:59:35


H I S T RIA DO BRASIL

10. (ENEM 2009) O autor da constituio de 1937, Francisco c) a concordncia entre Mrio de Andrade e Drummond
Campos, afirma no seu livro, O Estado Nacional, que o eleitor quanto ao carter inovador de Vargas, que fez uma ampla
seria aptico; a democracia de partidos conduziria desordem; a aliana para derrotar a oligarquia mineira.
independncia do Poder Judicirio acabaria em injustia e inefi- d) a discordncia entre Mrio de Andrade e Drummond sobre
cincia; e que apenas o Poder Executivo, centralizado em Getlio a importncia da aliana entre Vargas e o paulista Jlio
Vargas, seria capaz de dar racionalidade imparcial ao Estado, pois Prestes nas eleies presidenciais.
Vargas teria providencial intuio do bem e da verdade, alm de e) o otimismo de Mrio de Andrade em relao a Getlio
ser um gnio poltico. (CAMPOS, F. O Estado nacional. Rio de Vargas, que se recusara a fazer alianas polticas para vencer
Janeiro: Jos Olympio, 1940 adaptado). Segundo as ideias de as eleies.
Francisco Campos,
a) os eleitores, polticos e juzes seriam malintencionados.
b) o governo Vargas seria um mal necessrio, mas transitrio. G A B A R I T O
c) Vargas seria o homem adequado para implantar a 01. d 02. d 03. a 04. d 05. b 06. d
democracia de partidos.
07. c 08. d 09. e 10. e 11. d 12. a
d) a Constituio de 1937 seria a preparao para uma futura
democracia liberal.
e) Vargas seria o homem capaz de exercer o poder de modo
inteligente e correto.
REFERnCIAS BIBlIOGRFICAS
11. (ENEM 2009) A partir de 1942 e estendendo-se at o final
do Estado Novo, o Ministro do Trabalho, Indstria e Comrcio ARRUDA, Jos Jobson de A. Toda a Histria: Histria Geral e
de Getlio Vargas falou aos ouvintes da Rdio Nacional semanal- Histria do Brasil. So Paulo: tica, 2007.
mente, por dez minutos, no programa Hora do Brasil. O obje-
tivo declarado do governo era esclarecer os trabalhadores acerca BRAICK, Patrcia R.; MOTA, Myriam B. Histria das cavernas
das inovaes na legislao de proteo ao trabalho. (GOMES, A. ao terceiro milnio. 3 Ed., So Paulo: Moderna, 2007.
C. A inveno do trabalhismo. Rio de Janeiro: IUPERJ / Vrti-
ce. So Paulo: Revista dos Tribunais, 1988 adaptado). CAMPOS, Flvio de; MIRANDA, Renan Garcia. A Escrita da
Os programas Hora do Brasil contriburam para Histria. So Paulo: Escala Educacional, 2005.
a) conscientizar os trabalhadores de que os direitos sociais
foram conquistados por seu esforo, aps anos de lutas COTRIM, Gilberto. Histria Global Brasil e Geral. 7 Ed.,
sindicais. So Paulo: Saraiva, 2003.
b) promover a autonomia dos grupos sociais, por meio de uma
linguagem simples e de fcil entendimento. FAUSTO, Boris. Histria do Brasil. So Paulo: UNESP, 2000.
c) estimular os movimentos grevistas, que reivindicavam um
aprofundamento dos direitos trabalhistas. ORDOES, Marlene; QUEVEDO, Jlio. Horizonte da His-
d) consolidar a imagem de Vargas como um governante tria. Histria para o ensino mdio. So Paulo: IBEP, 2005.
protetor das massas.
e) aumentar os grupos de discusso poltica dos trabalhadores, SCHMIDT, Mario. Nova Histria Crtica do Brasil: 500 anos
estimulados pelas palavras do ministro. de histria malcontada. So Paulo: Nova Gerao, 2003.

12. (ENEM 2007) So Paulo, 18 de agosto de 1929. TEIXEIRA, Francisco M.P. Brasil Histria e Sociedade. So
Carlos [Drummond de Andrade], Paulo: tica, 2002.
Achei graa e gozei com o seu entusiasmo pela candidatura
Getlio Vargas Joo Pessoa. . Mas veja como estamos... tro- VICENTINO, Claudio; DORIGO, Gianpaolo. Histria para o
cados. Esse entusiasmo devia ser meu e sou eu que conservo o ensino mdio. So Paulo: Scipione, 2006.
ceticismo que deveria ser de voc. (...). Eu... eu contemplo numa
torcida apenas simptica a candidatura Getlio Vargas, que antes
desejara tanto. Mas pra mim, presentemente, essa candidatura
(nica aceitvel, est claro) fica manchada por essas pazes fra-
glimas de governistas mineiros, gachos, paraibanos (...), com
democrticos paulistas (que pararam de atacar o Bernardes) e
oposicionistas cariocas e gachos. Tudo isso no me entristece.
Continuo reconhecendo a existncia de males necessrios, porm
me afasta do meu pas e da candidatura Getlio Vargas. Repito:
nica aceitvel. Mrio [de Andrade]
(Renato Lemos. Bem traadas linhas: a histria do Brasil em cartas pessoais.
Rio de Janeiro: Bom Texto, 2004, p. 305). Acerca da crise poltica ocorrida em fins
da Primeira Repblica, a carta do paulista Mrio de Andrade ao mineiro Carlos
Drummond de Andrade revela

a) a simpatia de Drummond pela candidatura Vargas e o


desencanto de Mrio de Andrade com as composies
polticas sustentadas por Vargas.
b) a venerao de Drummond e Mrio de Andrade ao gacho
Getlio Vargas, que se aliou oligarquia cafeeira de So
Paulo.

166 UECEVEST

Apostilas UECEVEST mod3.indb 166 06/02/2011 09:59:36


P R - V E S T I B U l A R

MATEMTICA I

Apostilas UECEVEST mod3.indb 167 06/02/2011 09:59:42


Caro(a) Aluno(a),

O presente mdulo contm os seguintes objetos de conhecimento:

UECE ENEM
Geometria plana
Conhecimentos geomtricos
Geometria no espao

Apostilas UECEVEST mod3.indb 168 06/02/2011 09:59:42


MATE M T I C A I

GEOmETRIA PlAnA (UECE/EnEm) i) Duas retas distintas no paralelas so reversas.


j) Duas retas que no tm ponto comum so paralelas.
Conceitos primitivos e definies k) Duas retas que no tm ponto comum so reversas.
Ponto, Reta e Plano so apenas conhecimentos intuitivos de- l) Duas retas coplanares ou so paralelas ou so concorrentes.
correntes da experincia e da observao. J o espao o conjunto m) Duas retas no coplanares so reversas.
de todos os pontos. n) Se dois planos distintos tm um ponto comum, ento eles
Ponto: no possui dimenso nem existncia fsica. tm uma reta comum que passa pelo ponto.
Pontos colineares: so aqueles que pertencem a uma mesma reta. o) Dois planos distintos que tm uma reta comum, so secantes.
Pontos coplanares: so aqueles que pertencem a um mesmo plano. p) Se dois planos tm uma reta comum, eles so secantes.
Reta: constituda por infinitos pontos colineares. q) Se dois planos tm uma nica reta comum, eles so secantes.
Retas coplanares: so retas que pertencem a um mesmo plano. r) Dois planos secantes tm interseo vazia.
Sendo que duas retas sero concorrentes se caso interceptarem s) Dois planos secantes tm infinitos pontos comuns.
em um nico ponto, e paralelas se caso no interceptarem. t) Se dois planos tm um ponto comum, eles tm uma reta comum.
Retas reversas: so retas que no pertencem a um mesmo plano. u) Uma reta e um plano que tm um ponto comum so oncorrentes.
Retas perpendiculares: so retas concorrentes que forma um v) Uma reta e um plano secantes tm um nico ponto comum.
ngulo de 90 w) Uma reta e um plano paralelos no tm ponto comum.
Semi-reta: cada uma das duas pores em que fica dividida x) Se uma reta est contida num plano, eles tm um ponto comum.
uma reta por qualquer um de seus pontos. y) Se uma reta paralela a um plano, ela paralela a qualquer
Segmento de reta: toda poro de uma reta limitada por dois reta do plano.
pontos distintos. z) Por um ponto fora de um plano passa infinitas retas paralelas
Semi-plano: cada uma das duas pores em que fica dividido ao plano.
um plano por qualquer uma de suas retas.
Semi-espao: cada uma das duas pores em que fica dividi-
do o espao por qualquer um de seus planos. G A B A R I T O
Planos secantes: so planos que se interceptam, formando
uma reta. 01. A 02. E
03. a) V b) V c) F d) V e) V f) F g) V h) F
Postulados i) F j) F k) F l) V m) V n) V o) V p) F q) V
Por um ponto passam infinitas retas. r) F s) V t) V u) F v) V w) V x) V y) F z) V
Dois pontos distintos determinam uma nica reta que passa
por eles.
Trs pontos distintos e no colineares determinam um nico
plano que passa por eles
ESTUDO DOS nGUlOS
Complementares se: x + y = 90
E X E R C C I O Dois ngulos x e y so: Suplementares se: x + y = 180
01. Classifique em verdadeiro (V) ou falso (F) as alternativas abaixo: Replementares se: x + y = 360
I. Por um ponto passam infinitas retas.
II. Por dois pontos distintos passa uma reta. Complemento de x = (90 - x)
III. Uma reta contm dois pontos distintos. Observao 1: Suplemento de x = (180 - x)
IV. Dois pontos distintos determinam uma s reta. Replemento de x = (360 - x)
V. Por trs pontos dados passa uma s reta.
1 = 60 (1 grau = 60 minutos)
A sequncia correta : Observao 2: 1 = 60 (1 minuto = 60 segundos)
a) VVVVF d) VFVVV 1 = 3600 (1 grau= 3600 segundos)
b) VVVFV e) FVVVV
c) VVFVV Relao entre dois ngulos provenientes
de duas retas paralelas e uma transversal
02. Indique a alternativa falsa: Uma reta t concorrente com duas retas r e s paralelas distintas,
a) Dados dois pontos A e B, existe um plano que os contm; determinam:
b) Por um ponto fora de uma reta, existe uma nica paralela
reta dada; a b r
d c
c) Existe um, e somente um, plano que contm um tringulo dado;
e f s
d) Duas retas no coplanares so reversas; h g
e) Trs pontos distintos determinam um, e s um, plano.

03. Classifique em verdadeiro (V) ou falso (F) as alternativas abaixo: ngulos (a, c) (b, d)
Opostos pelo vrtice
a) Duas retas ou so coincidentes ou so distintas. congruentes (e, g) (f, h)
b) Duas retas ou so coplanares ou so reversas. (a, e) (b, f )
Correspondentes
c) Duas retas distintas determinam um plano. (c, g) (d, h)
d) Duas retas concorrentes tm um ponto em comum. Alternos internos (c, e) (d, f )
e) Duas retas concorrentes tm um nico ponto em comum. Alternos externos (a, g) (b, h)
f ) Duas retas que tm um ponto comum so concorrentes. ngulos Colaterais internos (c, f ) (d, e)
g) Duas retas concorrentes so coplanares. suplementares Colaterais externos (a, h) (b, g)
h) Duas retas coplanares so concorrentes.

UECEVEST 169

Apostilas UECEVEST mod3.indb 169 06/02/2011 09:59:44


M AT E MTICA I

Classificao dos ngulos Pontos importantes no tringulo


Nulo Agudo Reto Incentro Baricentro

x = 0 0 < x < 90
x = 90


Obtuso Raso
Incentro o encontro das Baricentro o encontro das
bissetrizes. medianas.
90 < x < 180 x = 180 Ortocentro Circuncentro

Estudo dos tringulos

Classificao dos tringulos quanto aos ngulos


Acutngulo Retngulo Obtusngulo
Ortocentro o encontro das
alturas. Circuncentro o encontro
c a c a das mediatrizes.
b a

OBS.:
c b b O baricentro divide as medianas numa proporo de 1:2
Tem trs ngulos Tem um ngulo Tem um ngulo
agudos. reto. obtuso. Soma dos ngulos interno de um tringulo
(menores que 90) (igual a 90) (maior que 90)
a2 < b2 + c2 a2 = b2 + c2 a2 > b2 + c2 c
a + b + c = 180
Classificao dos tringulos quanto aos lados a b

Equiltero Issceles Escaleno


A soma dos ngulos internos de qualquer tringulo 180

ngulo externo de um tringulo

c
e=a+c
Tem os trs lados Tem pelo menos
dois lados iguais Tem os trs lados a e
iguais diferentes.
e o lado diferente
chama-se base. O ngulo externo de um tringulo igual soma dos ngulos
internos no adjacentes a ele.
Segmentos importantes no tringulo
Exemplo prtico:
Bissetriz Mediana
A 30
x = 30 + 100
A
x
x = 130
100

C M B
C D B Condio de existncia e um tringulo.
AM mediana relativa ao lado Para que um tringulo possa existir, preciso que um lado
AD bissetriz relativa ao lado
BC, partindo do vrtice A e qualquer seja sempre menor que a soma dos outros dois.
BC, e divide o ngulo ao
chegando ao ponto mdio M
meio.
do lado oposto.
Altura Mediatriz a
a < b + c
A b < a + c
A r b
c c < a + b

C M B
C H B
r a mediatriz relativa BC. OBS.:
AH a altura relativa ao lado
(Mediatriz a reta perpen- O maior lado de um tringulo ope-se ao maior ngulo e
BC, e forma um ngulo de
dicular a um lado no ponto vice-versa, o maior ngulo ope-se ao maior lado.
90.
mdio)
Congruncia de Tringulos
Dois tringulos so congruentes quando eles so idnticos, ou
seja, tem os mesmos ngulos e medida de lados homlogos ou
correspondentes iguais.

170 UECEVEST

Apostilas UECEVEST mod3.indb 170 06/02/2011 09:59:45


MATE M T I C A I

Se dois tringulos so congruentes, ento eles devem obede- nmeros 2, 3, 4, 5 e 6. Determine a diferena entre o maior e o
cer a um dos casos a seguir: menor ngulo.
(LLL) lado, lado, lado a) 24 b) 34 c) 56 d) 72
(LAL) lado, ngulo, lado
(ALA) ngulo, lado, ngulo 08. (UECE 2000.2) Quando um relgio de ponteiros, marca
(LAAo) lado, ngulo, ngulo oposto exatamente 3h30min a medida do ngulo menor entre os pon-
(C.E.) cateto e hipotenusa teiros (das horas e dos minutos) :
a) 85 b) 80 c) 75 d) 70

E X E R C C I O 09. (UECE 2006.1) Se 5, 12 e 13 so as medidas em metros dos


lados de um tringulo, ento o tringulo :
01. Dado um ngulo de medida igual a 55, indique: a) issceles c) retngulo
a) Seu complemento b) equiltero d) obtusngulo
b) Seu suplemento
c) Seu replemento 10. (UECE 1999.1) As medidas, em graus, dos ngulos internos
d) O dobro de seu complemento de um tringulo formam uma progresso aritmtica e um dos n-
e) A stima parte do complemento gulos mede 30. Nestas condies a medida, em graus, do maior
f ) A quinta parte de seu suplemento ngulo do tringulo igual a:
g) O complemento de sua quinta parte a) 80 b) 85 c) 90 d) 95
h) O triplo do suplemento de sua quinta parte
11. (UECE 1999.2) No tringulo ABC, os lados AB e AC so
02. Simplifique as seguintes medidas:
congruentes e o ngulo A mede 80. Se os pontos D, E e F, res-
a) 3070 122
pectivamente, so marcados sobre os lados BC, AC e AB, de
b) 11058305
modo que CE=CD e BD=BF, ento o ngulo EDF igual a:
c) 45150247"
a) 30 b) 40 c) 50 d) 65
03. Classifique em verdadeiro (V) ou falso (F):
( ) Todo tringulo issceles equiltero. 12. (UECE 2006.1) Se o tringulo equiltero CDE exterior ao
( ) Todo tringulo equiltero issceles. quadrado ABCD, a medida do ngulo ACE igual a:
( ) Um tringulo escaleno pode ser issceles. a) 60 graus c) 135 graus
( ) Todo tringulo issceles tringulo acutngulo. b) 105 graus d) 150 graus
( ) Todo tringulo retngulo tringulo escaleno.
( ) Existe tringulo retngulo issceles. 13. (UFC 1999) Na figura abaixo, os segmentos de reta
( ) Existe tringulo issceles obtusngulo. A B , A C e C D so congruentes, um ngulo externo, e um
( ) Todo tringulo acutngulo ou issceles ou equiltero. ngulo interno do tringulo ABD.
( ) Todos os tringulos issceles so congruentes. A
( ) Todos os tringulos equilteros so congruentes.
( ) Todos os tringulos retngulos so congruentes.
( ) Todos os tringulos retngulos issceles so congruentes.
B D
( ) Todos os tringulos acutngulos so congruentes. C
Assinale a opo que contm a expresso correta de em termos
04. Determine a classificao da medida do ngulo igual ao do- de .
2
bro de seu complemento. a) = 3 d) =
a) agudo d) nulo 3
3
b) reto e) raso b) = 2 e) =
c) obtuso 3
c) =
05. (UECE 1999.1) Se um ngulo igual ao seu complementar, 2
ento o seno deste ngulo igual a: 14. (UECE 2007.2) O permetro do tringulo PQR 24cm e a
2 medida de seu menor lado 5,5cm. Se as medidas dos lados deste
a) 1/2 b) c) 3 d) 1 tringulo, em centmetros, formam uma progresso aritmtica de
2 2 razo r, podemos afirmar, corretamente, que:
a) 1,4 < r < 1,8 c) 2,2 < r < 2,6
06. (UECE 2005.1) Os pontos X, Y, Z, W, distintos e colineares, b) 1,8 < r < 2,2 d) 2,6 < r < 3,0
so tais que Y ponto mdio do segmento XW e Z o ponto
mdio do segmento YW. A razo entre as medidas dos segmentos 15. (UECE 2004.2) As retas na figura interceptam-se duas a duas
XY e XZ : no pontos P,Q e R. Considerando os valores indicados, o ngulo
1 3 igual a:
a) c)
3 4 P 103
b) 2 d) 1
3 2

07. (UECE 2000.1) Considere 5 semi-retas todas partindo do


Q R 24
mesmo ponto P num mesmo plano formando 5 ngulos cont-
guos que cobrem todo o plano, sejam medidas proporcionais aos a) 101 b) 102 c) 103 d) 104

UECEVEST 171

Apostilas UECEVEST mod3.indb 171 06/02/2011 09:59:48


M AT E MTICA I

16. (UECE 2002.2) O tringulo ABC issceles, com base BC e a c a+b c+d a c
o ponto D do lado AC, tal que AD=BD=BC. Ento a soma das = ou = ou =
medidas, em graus, dos ngulos B e C : b d b d a+b c+d
A
Semelhana de Tringulos
Dois tringulos so semelhantes quando ambos possuem
D pelo menos dois ngulos congruentes.
B
E
B C
a) 144 b) 140 c) 136 d) 132 A 60 45 C D 60 45 F

17. (UECE 2003.2) Na figura abaixo, os pontos D e E esto Se dois tringulos so semelhantes, ento seus lados e altu-
sobre o lado AC do tringulo qualquer ABC. ras homlogas e os permetros so proporcionais.
B B
v E
u w
c h a b' h a'
A 60 45 C D 60 45 F
b c'
A x m n p q C
D E a b c h a+b+c
= = = = =k
Se as letras minsculas indicam as medidas dos ngulos identifi- a' b' c' h' a'+b'+ c'
cados na figura, ento u w + x q igual a:
a) m + n c) v m n Exemplo prtico:
b) m n d) m + n v B
E
18. (UECE 2008.1) Seja EOXY um trapzio. Se existe um ponto
12 8 24
Z da base menor XY tal que ZE e ZO so respectivamente as 3 2 5
bissetrizes dos ngulos YO e EX, podemos afirmar, correta- A 80 40 C D 80 40 F
mente, que: 20 6
a) os tringulo EZY e OZX so semelhantes. 24 20 12 8 24 + 20 + 15
b) o trapzio issceles. ABC ~ DEF = = = = =4
6 5 3 2 6+5+3
c) a rea do tringulo EZO a soma das reas dos tringulos
EZY e OZX.
d) a medida da base menor a soma das medidas dos lados no OBS.:
paralelos do trapzio. Lados homlogos ou correspondentes: so lados opostos aos
mesmos ngulos.
19. (UECE 2007.1) Em um quadrado PQRS, seja M o ponto Permetro: a soma dos lados.
mdio do lado PQ e N o ponto mdio do lado PS. Se os segmen-
tos RM e QN se cortam no ponto E, a medida do ngulo RN : Base mdia de um Tringulo
a) 120 b) 110 c) 90 d) 60 Se M e N so pontos mdios dos lados AB e AC respectiva-
mente, ento:
A Importante:
Se M e N so pontos
G A B A R I T O mdios dos lados AB e AC
respectivamente, ento:
01. a) 35 b) 125 c) 305 d) 70 e) 5 f ) 25 g) 79 h) 507 M N
h
02. a) 31122 b) 11145 c) 48347 BC
03. FVFFFVVFFFFFF 04. a 05. b MN // BC e MN =
B C 2
06. b 07. d 08. c 09. c 10. c H
11. c 12. b 13. a 14. c 15. a OBS.: AM AN MN Ah 1
16. a 17. b 18. d 19. c ABC~ AMN = = = =
AB AC BC AH 2

Teorema de Tales
Base mdia do Trapzio
u v A B Sabendo que MN//AD//BC
r
Q MN= BC + AD
a c
s
M N 2
b d O P
t OP = BC - AD
2
r//s//t e u, v transversais, temos que: D C

172 UECEVEST

Apostilas UECEVEST mod3.indb 172 06/02/2011 09:59:52


MATE M T I C A I

Teorema da bissetriz interna A


cateto cateto
a c c b
a c = a = b + c
b d
B C
b d a
hipotenusa
Teorema da bissetriz externa
Resumo das relaes mtricas
b+d d
a c = 1) a = m+n 5) b = an
a c c h b 2) a = b + c 6) c = am
b d 3) ah = bc 7) bm = ch
m n
a
4) h = mn 8) cm = bh
Relaes mtricas no tringulo retngulo
A lei dos senos

c b
h
b a a b c
= =
B m n C sen sen sen
a H
c
Na figura acima, possvel visualizar trs tringulos, dois a
dois semelhantes entre si. lei dos cosenos
c h
i ) b = n cn = bh
b a
h m a = b + c 2bc.cos
ABH ~ ACH ii ) = h2 = mn
n h
c
m c
iii ) h = b bm = ch

E X E R C C I O
a b
i ) c = h ah = bc 01. (CEFET) Na figura seguinte, calcule a medida do ngulo x.
a) 30 5a - 48
b c
b) 60
ABC ~ ABH ii ) = bm = ch r
h m c) 70 x
r // t
c a d) 78 s 3a + 12
iii ) m = c c = am
2

e) 86

a b
i ) b = n b = an
2
02. (UFRO)Determine a medida do ngulo x, na figura seguinte:

b c r
ABC ~ ACH ii ) = cn = bh 140
n h
c a r // s x
iii ) h = b ah = bc

60 s
Teorema de Pitgoras
No triangulo retngulo acima, prove as seguintes frmulas: a) 40 d) 90
b) 60 e) 100
1) b = an e 2) c = am
c) 80
OBS.: 03. (UFPB) Na figura abaixo so dados AB = 12cm, AE = 4cm,
m e n so chamados de projees dos catetos e (m+n)= a AD = 6cm, ED = 5cm. O permetro do tringulo ABC, em cm, :
a) 42
Somando as frmulas 1) e 2) acima, teremos: b) 45
A

2 c) 48
+ b2 = an d) 52 E
c = am e) 56 D
b + c 2 = am + an
2
b 2 + c 2 = a( m + n )
b2 + c 2 = a a
B C
b2 + c 2 = a 2

Assim, chamamos de teorema de Pitgoras, a relao em um 04. (UFC 2002) Na figura abaixo, os tringulos ABC e ABC
tringulo retngulo, dita:
O quadrado da hipotenusa igual soma dos quadrados so semelhantes. Se AC = 4 ento o permetro de ABC dividido
AC'
dos catetos pelo permetro de ABC igual a:

UECEVEST 173

Apostilas UECEVEST mod3.indb 173 06/02/2011 09:59:55


M AT E MTICA I

a) 1/8 A 11. (FUVEST) Num trapzio issceles, a altura igual base


b) 1/6 mdia. Determine o ngulo que a diagonal desse trapzio forma
c) 1/4 B' C' com a sua base.
d) 1/2 a) 60 d) 45
e) 1 b) 30 e) 15
B c) 90
C
12. (UECE 2007.1) Os vrtices do tringulo XYZ so os pon-
05. (UECE 2005.2) Uma escada de 25m est encostada na pa- tos mdios dos lados do tringulo equiltero MPQ, cujo lados
rede vertical de um edifcio de modo que o p da escada est a medem 2m, como mostra a figura:
7m da base do prdio. Se o topo da escada escorrega 4m, quantos P
metros ir escorregar o p da escada?
a) 10m b) 9m c) 8m d) 6m
X Y
06. (C. Naval) Um tringulo est inscrito em um tringulo re-
tngulo com AB = 12cm e BC = 8cm, conforme mostra a figura
abaixo. Se no retngulo a medida do comprimento o dobro da
M Z Q
medida da largura, o seu permetro, em cm, :
a) 10 A Se h1 e h2, respectivamente, so as alturas dos tringulos XYZ e
b) 12 MPQ, ento o produto h1. h2 , em m2, igual a:
c) 18 a) 2/3 b) 3/4 c) 4/3 d) 3/2
d) 20
e) 24 13. (UFC 2003) Sejam , e os ngulos internos de um tringu-
B C lo. Se as medidas desses ngulos so diretamente proporcionais a 1,
2 e 3, respectivamente, e a bissetriz do ngulo mede duas unidades
07. (UFC 2004) Na figura ao lado, o tringulo ABC retngulo de comprimento (u.c), a medida do permetro deste tringulo :
em B. O cosseno do ngulo BC : C a) 3 ( 3 + 2) u.c d) 3 ( 3 + 1) u.c
a) 12/13 b) ( 3 + 1) u.c e) (3 3 1) u.c
b) 11/13 13 c) 3 3 u.c
c) 10/13 5
d) 6/13 14. (UECE 2005.1) Na linha poligonal PQRSTU, plana e aber-
e) 1/13 A B ta como mostra a figura, dois segmentos consecutivos so sem-
pre perpendiculares, a medida de PQ 1m e, a partir de QR,
08. (UECE 2004.2) A linha poligonal com extremidades no inclusive, os demais comprimentos dos segmentos so obtidos,
pontos P e Q formada por segmentos horizontais e segmentos dobrando o valor do segmento anterior.
verticais. Se cada segmento horizontal mede 3m e cada segmento S R
vertical mede 3,2m, a medida do segmento cujas extremidades
P Q
so P e Q : Q
a) 28m
b) 24m
c) 20m T U
d) 16m
A distncia do ponto P ao ponto U, em metros, :
a) 205 b) 215 c) 15 d) 235
P
15. (UFC 2002) Na figura abaixo, temos dois tringulos equil-
teros ABC e ABC que possuem o mesmo baricentro, tais que
09. (UECE 2002.2) Na figura abaixo, o lado maior do retngulo
AB // A' B', AC // A'C' e BC // B' C'. Se a medida dos lados de ABC
o dobro do lado menor, os vrtices do losango so os pontos
igual a 3 3 cm e a distncia entre os lados paralelos medem 2cm,
mdio dos lados do retngulo e a circunferncia tangente aos
ento a medida das alturas de ABC igual a:
lados do losango:
a) 11,5cm A' B'
b) 10,5cm A B
c) 9,5cm
d) 8,5cm
e) 7,5cm
C
Se x a medida do lado menor do retngulo, ento a medida do
raio da circunferncia :
C'
x 3 x 2 3x
a) x 5 b) c) d)
5 3 2 5 16. (UECE 2001.1) Um navio navegando em linha reta passa
sucessivamente pelos pontos A, B e C. O comandante, quando
10. (UECE 2002.1) A mediana relativo hipotenusa de um o navio est em A, observa o farol em L e calcula o ngulo LC
tringulo retngulo mede 1cm e forma com um dos catetos um como sendo 45. Aps navegar 4 milhas atinge o ponto B quando
ngulo de 30. O permetro do tringulo igual a: o ngulo de 75. Quantas milhas separam o farol do ponto B?
a) (3 + 3 )cm c) (3 3 + 1)cm a) 6 b) 7 c) 8 d) 9
b) (3 - 3 )cm d) (3 3 - 1)cm 2 2 2 2

174 UECEVEST

Apostilas UECEVEST mod3.indb 174 06/02/2011 10:00:00


MATE M T I C A I

17. (UECE 1999.1) A medida, em cm, da diagonal maior de um 3. Quatro ngulos iguais a 90
paralelogramo cujos, lados medem 6cm e 8cm e o menor ngulo 4. Diagonais cortam-se no ponto mdio
mede 60 igual a: 5. Diagonais iguais
a) 3 37 b) 2 37 c) 37 d) 37
2
Quadrado

18. (UECE 2006.1) O menor lado de um paralelogramo, cujas


diagonais medem respectivamente 8 2 m e 10m e formam entre
si um ngulo de 45, mede:
a) 13 m c) 13 2 m
4
b) 17 m d) 17 2 m 1. Lados opostos paralelos
5 2. Quatro lados iguais
3. Quatro ngulos iguais a 90
19. (UECE 1999.2) As medidas dos lados AB e BC de um trin- 4. Diagonais cortam-se no ponto mdio
gulo ABC so, respectivamente, 6cm e 12cm e o ngulo entre eles 5. Diagonais iguais
120. A medida, em cm, da bissetriz do ngulo ABC igual a: 6. Diagonais perpendiculares
a) 5 b) 4 c) 3 d) 2
Trapzio
b
a x
G A B A R I T O a + b = 180
x + y = 180
01. d 02. e 03. b 04. c 05. c b y
06. c 07. a 08. c 09. a 10. a B
11. d 12. d 13. d 14. a 15. b
1. Dois lados opostos paralelos
16. c 17. b 18. b 19. b 2. Dois lados opostos no paralelos
3. Os lados paralelos tm medidas diferentes e so chamados
de base maior e base menor

ESTUDO DOS QUADRIlTEROS Classificao dos trapzios


Issceles Escaleno Retngulo
Paralelogramo

Um lado forma
Tem os lados no Tem os lados no
ngulos de 90 com
paralelos iguais. paralelos diferentes.
1. Lados opostos paralelos as bases.
2. Lados opostos iguais
3. ngulos opostos iguais
4. ngulos consecutivos suplementares
5. Diagonais cortam-se no ponto mdio
ESTUDO DOS POlGOnOS
losango
Um polgono convexo quando a reta determina por dois
vrtices consecutivos quaisquer deixa todos os demais (n-2) vrti-
ces num mesmo semiplano, caso contrrio ser cncavo.

1. Lados opostos paralelos


2. Quatro lados iguais
3. ngulos opostos iguais
4. ngulos consecutivos suplementares
5. Diagonais cortam-se no ponto mdio Polgono Convexo Polgono Cncavo
6. Diagonais perpendiculares

Retngulo Polgono Polgono


Polgono Regular
equiltero equingulo

Tem lados iguais Tem ngulos iguais Tem lados e ngu-


1. Lados opostos paralelos
2. Lados opostos iguais los iguais |

UECEVEST 175

Apostilas UECEVEST mod3.indb 175 06/02/2011 10:00:02


M AT E MTICA I

Nomenclatura dos Polgonos Posio entre duas circunferncias


n Nome n Nome Externas Tangentes Exteriores
3 Tringulo 12 Dodecgono
4 Quadriltero 13 Tridecgono R R
5 Pentgono 14 Tetradecgono r r
6 Hexgono 15 Pentadecgono C1 C2 C1 C2
7 Heptgono 16 Hexadecgono
8 Octgono 17 Heptadecgono
C1C2 > R + r C1C2 = R + r
9 Enegono 18 Octadecgono
10 Decgono 19 Eneadecgono Secantes Tangentes Interiores
11 Undecgono 20 Icosgono
R R
Frmulas importantes para polgonos r r
Soma dos ngulos internos de Si = (n 2)180 C1 C2 C1 C2
um polgono convexo qualquer.
Soma dos ngulos externos de
Se = 360 R r < C1C2 < R + r C1C2 = R r
um polgono convexo qualquer.
Medida dos ngulos internos de (n 2 )180 Interna Concntricas
Ai =
um polgono regular. n
360 R R
Medida dos ngulos externos de Ae = r r
um polgono regular. n C1 C2 C1 C2

Nmero de diagonais de um n(n 3)


N D =
polgono qualquer. 2 0 < C1C2 < R r C1C2 = 0

Exerccio resolvido: Responda os itens a seguir:


Correspondncia entre arcos e ngulos
a) A soma dos ngulos internos de um hexgono.
S i = (6 2 )180 = 4 180 = 720 ngulo central ngulo inscrito
A A
b) A soma dos ngulos externos de um hexgono.
Se = 360 P
P
c) A medida dos ngulos internos do hexgono regular. B
Ai =
(6 2 )180 4 180 720
= = = 120
B
6 6 6 AB
= AB =
2
d) A medida dos ngulos externos do hexgono regular.
A = 360 = 60 ngulo de vrtice interior ngulo de vrtice exterior
e
6 A A
D
P D
e) O nmero de diagonais de um Hexgono. P
6(6 3) 6 3 18 C
N D = = = =6
2 6 6 C
B B
AB + CD AB CD
= =
2 2
Estudo dos crculos e
circunferncias
Potncia de Ponto
Posio entre reta e circunferncia Corda / Corda Secante / Secante Tangente / Secante
Reta externa Reta tangente Reta secante A D B P A
A
P B
C P C
D
B C
PA PB = PC PD PA PB = PC PD PA PA = PB PC

176 UECEVEST

Apostilas UECEVEST mod3.indb 176 06/02/2011 10:00:06


MATE M T I C A I

Comprimento da 04. (UECE 2004.2) O paralelogramo PQRS tal que a bissetriz


Teorema do Bico do ngulo Q intercepta o lado PS no ponto M com MS = 2m e
Circunferncia
MQ = QR = 6m. Nestas condies a medida do lado PQ :
A a) 3,0m b) 3,5m c) 4,0m d) 4,5m
R
O B 05. (UECE 2001.1) Um quadrado dividido em sete retngulo
iguais conforme indicado na figura. Se o permetro de cada um
C destes retngulos 32m, ento o permetro do quadrado :
C = 2Rp AB = BC a) 56m
b) 58m
Quadriltero Circunscritvel Quadriltero Inscritvel c) 60m
A B B d) 62m
C

O O
A
D
C D 06. (UECE 1999.2) Num quadrado de 11m de lado so traa-
das retas paralelas a um de seus lados, de modo que a distncia
AB + CD = AD + BC A + C = B + D = 180o entre duas retas consecutivas seja sempre 80cm. Se a primeira e
a ltima reta traadas distam 1,50m do lado mais prximo do
quadrado, o nmero total de retas traadas :
a) 9 b) 10 c) 11 d) 12
E X E R C C I O
07. (UECE 2001.1) Na figura esto desenhados um hexgono
01. Classifique em verdadeiro (V) ou falso (F): regular, um quadrado e um tringulo. A medida do ngulo x :
( ) Todo Quadrado Retngulo.
( ) Todo Retngulo Quadrado.
( ) Todo Paralelogramo Losango.
( ) Todo Losango Paralelogramo. x
( ) Todo Quadrado Losango.
( ) Todo Losango Quadrado.
( ) Todo retngulo Losango a) 45 b) 60 c) 6230 d) 75
( ) Todo Losango Retngulo.
( ) Todo Paralelogramo Quadrado. 08. (UECE 2006.1) No quadriculado retangular abaixo esto re-
( ) Todo Quadrado Paralelogramo. presentados quatro canteiros.
( ) Todo Retngulo Paralelogramo.
( ) Todo Paralelogramo Retngulo. IV
I
( ) O losango pode ser um Quadrado.
( ) As diagonais do retngulos so perpendiculares.
( ) As diagonais dos Quadrados so perpendiculares.
( ) As diagonais dos Losangos so perpendiculares.
( ) A interseo das diagonais dos Retngulos ponto mdio II III
das diagonais.
( ) As diagonais dos Paralelogramos no cruzam-se ao meio.
( ) As diagonais dos Paralelogramos so iguais. Se os permetros dos canteiros I, II e III, so, respectivamente,
( ) As diagonais dos Trapzios so iguais. 60m, 64m e 56m, ento o permetro do canteiro IV :
( ) As diagonais dos Quadrado e Retngulos so iguais. a) 58m b) 60m c) 62m d) 68m
( ) As diagonais dos losangos tem sempre medidas diferentes.
( ) O Trapzio tem sempre dois lados iguais. 09. (UNIFOR-98.2) Dois crculos distintos de raios iguais e con-
( ) Os ngulos da base de um Trapzio isscele tem medidas tidos em um mesmo plano:
diferentes. a) podem no ter tangentes comuns.
( ) Dois ngulos consecutivos quaiquer de um Trapzio so b) no podem ter uma nica tangente comum.
suplementares c) tm apenas duas tangentes comuns.
( ) Um quadriltero que tem lados opostos paralelos ter d) podem ter mais do que quatro tangentes comuns.
ngulos opostos congruentes. e) no podem ter trs tangentes comuns.
( ) As diagonais dos losangos so bissetrizes de seus ngulos.
10. (UECE 2005.2) Na figura, o retngulo DGHI, o tringulo
02. Determine respectivamente a soma dos ngulos internos, a soma equiltero DEF e o quadrado ABCI tm, todos, permetro igual
dos ngulos externos, a medida do ngulo interno, a medida do ngu- a 24cm. H G
lo externo e o nmero de diagonais dos polgonos regulares a seguir:
F
a) Octgono. c) Pentadecgono E
b) Dodecgono d) Icosgono
I C
03. (ETFCE) O nmero de diagonais de um polgono regular, D
cuja medida do ngulo interno 165 :
a) 54 b) 90 c) 170 d) 252
A B

UECEVEST 177

Apostilas UECEVEST mod3.indb 177 06/02/2011 10:00:08


M AT E MTICA I

Se o ponto D ponto mdio de CI, o permetro da figura fechada 18. (UECE 2000.2) Na figura temos trs circunferncias tan-
ABCDEFGHIA igual a: gentes, duas a duas, cujos centros A, B e C so vrtices de um
a) 48cm b) 49cm c) 50cm d) 51cm tringulo retngulo em C e as duas circunferncias maiores pos-
suem raios com a mesma medida R. A linha tangente a duas
11. (UECE 2006.2) As medidas dos ngulos internos , , , circunferncias e secante terceira e P o ponto de interseo da
de um quadriltero convexo esto em progresso aritmtica, sendo reta com AB. A medida do segmento AP :
45 a menor medida. O valor da soma sen + sen + sen+ sen : a) R 2 c) ( 3 1)R
b) R 3 d) (3 2 )R A C
a) 2 3 + 6 c) 3 2 + 6
3 2
P
b) 2 3 + 6 d) 3 2 + 6
B
2 3

12. (UECE 2006.2) A medida do raio da circunferncia inscrita


no tringulo retngulo cujos catetos medem, respectivamente, 19. (UECE 2004.1) Uma janela tem a forma vista na figura abai-
3m e 4m : xo, constituda da um quadrado de 60cm de lado acoplado a um
a) 2,0m b) 1,8m c) 1,2m d) 1,0m arco de uma circunferncia de 50cm de raio (menor que um semi-
crculo). A altura mxima da janela (distncia do ponto mdio da
13. (UFC) A figura ao lado mostra quatro rodas circulares, tangen- base da janela ao ponto mais alto de sua parte superior), em cm, :
tes duas a duas, todas de mesmo raio r e circundadas por uma cor- a) 65
reia ajustada. Determine o comprimento da correia, em termos de r. b) 70
OBS: despreze a espessura da correia. c) 90
d) 80

20. (UFC 2004) Na figura, a razo entre o permetro da regio


hachurada e o permetro da circunferncia de centro em O :
a) 1/3
a) 8R + Rp d) 8R + 2Rp +4
b) 4R + Rp e) 2R + 4Rp b)
4
c) 8R + 4Rp
c) p/4 0
14. (UECE 2008.2) O ponto P externo a uma circunferncia e +4
d)
sua distncia ao centro da circunferncia 13m. A secante traada 2
de P intercepta a circunferncia nos pontos Q e R, de modo que PQ e) 2
mede 9cm e PR mede 16m. A medida do raio da circunferncia :
a) 4m b) 5m c) 6m d) 7m

15. (UECE 2008.2) Sobre trs circunferncias localizadas em um G A B A R I T O


mesmo plano, com centros no colineares, podemos afirmar cor-
retamente que: 01. * 02. ** 03. d 04. c 05. a
a) a interseo das trs circunferncias um conjunto vazio. 06. c 07. d 08. d 09. b 10. c
b) a interseo das trs circunferncias um conjunto com, no 11. c 12. d 13. d 14. b 15. b
mximo, um elemento. 16. a 17. a 18. a 19. b 20. d
c) a interseo das trs circunferncias um conjunto com, 01. VFFVVFFVFVVFVFVVVFFFVFFFFVV
exatamente, dois elementos.
02. a) Octgono 1080 / 360 / 135 / 45 / 20
d) a interseo das trs circunferncias um conjunto com,
b) Dodecgono 1800 / 360 / 150 / 30 / 54
exatamente, trs elementos.
c) Pentadecgono 2340 / 360 / 156 / 24 / 90
d) Icosgono 3240 / 360 / 162 / 18 / 170
16. (UECE-98.2) Em um plano, dois crculos de raios 6,5 m e
0,5 m esto no mesmo lado de uma reta e a tangenciam nos pon-
tos A e B. Se a medida do segmento AB 8m, ento a distncia
entre os centros dos dois crculos igual a:
a) 10m b) 9 m c) 8 m d) 7 m ESTUDO DAS REAS
17. (UECE 2000.1) Na figura, a reta MN tangente circunfern- Tringulos
cia em P, a secante MQ passa pelo centro O da circunferncia e a
medida do ngulo QMP 40. A medida do ngulo NPQ igual a: Qualquer Retngulo

M P
a
h c
O
Q
b b
A= b.h A= b.c
a) 65 b) 60 c) 55 d) 50 2 2

178 UECEVEST

Apostilas UECEVEST mod3.indb 178 06/02/2011 10:00:11


MATE M T I C A I

Equiltero Funo dos lados Crculos e suas Partes


rea do Crculo rea da Coroa Circular
a
c R
h R
b
r

A = p( p a )(p b )(p c )
A = . 3
4 a+b+c
*P= 2 A = R . p A = (R p)p
Funo de dois lados e um Funo dos lados e do raio rea do Setor Circular rea do Segmento Circular
ngulo entre eles inscrito

R
R
a
a r
c
R R
b
b
1 a+b+c . R . l.R
A = a . b . sen A = .r A= ou A =
2 2 360 2 A = ASETOR ATRINGULO

Funo dos lados e do raio


circunscrito

c E X E R C C I O
a R
01. (UECE-98.1) A rea de um tringulo igual a 8cm. Se um
dos lados dos tringulo mede 2 cm, ento a altura, em cm, do
b tringulo, relativa a este lado igual a:
a) 6 b) 7 c) 8 d) 9
a .b.c
A= 02. (UFPI) Se o permetro de um quadrado 1m, sua rea
4R
igual a:

Quadrilteros a) 1 m2 b) 1 m2 c) 1 m2 d) 1 m2
4 9 16 25
Paralelogramo Retngulo
03. (UNIFOR-98.1) Na figura seguinte, tem-se o tringulo ABC
h h inscrito em um semicrculo de centro 0 e raio de medida 4 cm. A
razo entre as reas dos tringulos ABO e AOC, nessa ordem, :
b A
A = b.h A = b.h
B C
Losango Quadrado
O
5
d a) 2 d)
3 6
D 3
b) e) 1
4
4
D.d c)
A= A =
2 5

Trapzio
04. (UECE 1999.1) Num tringulo ABC, AB = 3cm, AC=4cm e
b sua rea 3cm. Nestas condies, o ngulo A igual a:
a) 90 b) 60 c) 45 d) 30
h
05. (UECE 2005.1) Num losango, cada lado mede 15cm e uma
B das diagonais mede 18cm. A rea deste losango, em cm2, :
a) 216 b) 236 c) 260 d) 270
(B + b) . h
A=
2 06. (UNIFOR-96.2) A rea, em metros quadrados, de um tra-
pzio cujas bases medem 12m e 8m e cujos ngulos da base maior
medem 60 :

UECEVEST 179

Apostilas UECEVEST mod3.indb 179 06/02/2011 10:00:16


M AT E MTICA I

a) 20( 3 + 4 3 ) d) 16 2 A B
b) 20 3 e) 2( 2 + 3 ) P Q
c) 20 2

07. (UECE 2008.2) O permetro de um tringulo mede 24m e


sua hipotenusa mede 10m. A medida da rea deste tringulo :
a) 8m2 b) 12m2 c) 14m2 d) 24m2
D C
08. (UECE 2007.2) As diagonais de um losango medem 12m M
e 16m. A medida da rea do quadriltero, cujos vrtices so os 3 3
a) 16 1 c) 16 1 +
pontos mdios dos lados do losango, igual a:
6 4 6 4
a) 32 m2 b) 36 m2 c) 42 m2 d) 48 m2
3
b) 16 1 d) 16 1 + 3
09. (UECE 2001.2) Na figura ABCD um quadrado de lado 3 4 3 4

1cm, E o ponto mdio da diagonal BD e F o ponto mdio do
segmento BE. A rea do tringulo BFC em cm2 : 15. (UECE 2002.2) Na figura o retngulo ABCD est inscrito
a) 1/12 B C no crculo de centro O, os segmentos FE e FB medem 20cm e
b) 1/8 F 40cm, respectivamente, e OE paralelo a BC. A medida da rea
c) 1/4 E sombreada, em cm2, :
d) 3/8
E
A D
A B
10. (UECE 2006.1) Na figura, a rea da regio sombreada igual a: F
1m 1m
O

D C
1m 1m

a) 100 (25p 48) c) 100 (25p 16)


1m 1m b) 100 (25p 24) d) 100 (25p 8)

16. (UECE 2004.1) Na figura, o tringulo ABC est inscrito na


a) 0,85m2 b) 1,15m2 c) 1,50m2 d) 1,75m2 semicircunferncia cujo raio mede 2cm. Para cada posio do
vrtice A, ao longo do arco BC, a soma das reas sombreadas
11. (UECE 2006.1) Na figura, o retngulo ABCD foi dividido assume um valor. O menor destes valores, em cm, :
nas 4 partes X, Y, Z e W.
A B A
X
W

Y B C
Z
D C a) 3( 3) c) 2( 3)
b) 3( 2) d) 2( 2)
Se X e Y so quadrados de reas 81m e 144m , respectivamente, e
2 2

Z um tringulo com 102m2 de rea, ento a rea da regio W : 17. (UECE 2005.1) Na figura, os vrtices do quadrado PQRS
a) 327m2 b) 316m2 c) 309m2 d) 282m2 esto sobre a circunferncia que o circunscreve, os lados do qua-
drado medem 1m e o arco PR pertence circunferncia centrada
12. (UECE 1999.1) Sejam H1 e H2 dois hexgonos regulares de em S.
lados iguais a 4m e 6m, respectivamente. Se C1 e C2 so, respecti-
vamente, os crculos circunscritos a H1 e H2, ento a razo entre P Q
as reas de C1 e C2 igual a:
a) 3/2 b) 9/4 c) 4/9 d) 2/3

13. (UECE 2008.2) Desejamos construir uma calada em volta


de dois lados consecutivos de um terreno retangular. A calada
exterior ao terreno e tem largura constante. Se duas das di-
S R
menses do terreno so 20m e 30m, respectivamente, e a rea da
calada mede 77,25m2, ento sua largura mede:
a) 2,00m b) 1,75m c) 1,50m d) 1,25m A rea da regio pigmentada, em m2 , :
a) 0,3 b) 0,5 c) 0,7 d) 0,9
14. (UECE 2002.1) Na figura, o lado do quadrado ABCD mede
4cm e M, ponto mdio do lado CD, o centro do arco de cir- 18. (UECE 2003.1) Considere um tringulo com rea a 2 m2 e
cunferncia PQ tangente ao lado AB. A medida, em m, da rea com dois de seus lados (b1 e b2) medindo 2m e 4m. Se o ngulo
sombreada : interno 3 formado por b1 e b2 for menor que 90, a rea do
tringulo obtido do tringulo inicial, duplicando o ngulo 3 :

180 UECEVEST

Apostilas UECEVEST mod3.indb 180 06/02/2011 10:00:19


MATE M T I C A I

a)
4
3 m2 c) 2 3 m2 a) = 5 d) = 2
3 2
b) 4 3 m2 d)
2
3 m2 b) = 3 e) = 2
3 2 3
19. (UECE 2006.2) Na figura, as duas circunferncias so tan- c) = 4
gentes, o centro da circunferncia maior um ponto da circun-
ferncia menor e o dimetro da circunferncia maior mede 4cm. 24. (UECE 2005.2) Na figura as semi-retas r e s so tangentes
ao crculo de raio 1m. Se = 60, a rea da regio pigmentada
igual a: s

G
R
O
R
A rea da regio hachurada igual a: r
F
H
a) p2cm2 b) 2p2cm2 c) 2p cm2 d) p cm2
3
a) 3 m 2 c) + m 2
20. (SANTA CASA-SP) Um lago circular de 20m de dimetro 3 3 6
circundado por um passeio, a partir das margens do lago, de 2m
3 2
de largura. A rea do passeio representa a seguinte percentagem b) m d) + 3 m 2
3 6 3
da rea total do lago:
a) 10% d) 32%
b) 20% e) 44% 25. (UECE 2003.1) Na figura, ABCD um quadrado cujo lado
c) 15 % mede 20cm e O seu centro. Os segmentos OE, OF, OG, OH, e
OI dividem o quadrado em cinco regies com reas iguais. Se AE
21. (UECE 2004.2) A figura ao lado representa trs crculos con- mede 7cm ento o produto AF.CG, em cm,
cntricos de raios 3m, 4m e 5m, respectivamente. Que porcenta- F
gem da rea do crculo maior representa a rea cinza? A B
a) 25
b) 36 G
c) 30 E
d) 32 O

D I H C
a) 108 b) 126 c) 135 d) 153
22. (UECE 2004.1) Na figura as trs circunferncias so tangen-
tes no ponto P e seus raios so expressos, em cm, por nmeros 26. (UECE 2003.2) Considere o retngulo ABCD com rea
naturais consecutivos. Se a medida da rea limitada pela circun- igual a 6cm. Sejam: C no prolongamento do lado BC para
ferncia menor for igual medida da rea compreendida entre a direita tal que CC'= BC ; D no prolongamento de CD para baixo
circunferncia intermediria e a maior ento a soma dos dime- tal que DD'= 2CD' ; A no prolongamento de DA para esquerda
tros das trs circunferncias igual a: tal que AA'= 3 AD ; B no prolongamento de AB para cima tal
a) 36cm que BB'= 4 AB. Nestas condies a rea do quadriltero ABCD,
b) 30cm em cm, :
c) 24cm a) 150 B C
d) 18cm P b) 132
c) 114
d) 108
A D

23. (UFC) Considere a figura abaixo na qual: 27. (UECE 2007.1) O retngulo LMNP est dividido em trs
1. A rea do semicrculo c1 quatro vezes a rea do semicrculo c2. quadrados (q1, q2 e q3) e um retngulo (r).
2. A reta r tangente a c1 e a reta s tangente a c1 e c2.
r L P
q1 r
q3
C1 q2
M N
s

C2 A razo entre as medidas do lado menor e do lado maior de r .
A razo entre as reas de r e de LMNP :
Ento, podemos afirmar corretamente que: a) 1/2 b) 1/16 c) 1/20 d) 1/24

UECEVEST 181

Apostilas UECEVEST mod3.indb 181 06/02/2011 10:00:23


28. (UECE 2004.2) Se o retngulo PQRS abaixo tem rea igual ferncia menor, que passa pelo centro de S. A medida da rea da
a 756 m2 e formado por 7 retngulos congruentes ento o pe- regio limitada pela circunferncia menor :
rmetro de PQRS, em m, : 15
a) 25 b) c) 10 d) 20
a) 114 P Q 2 2 2 2
b) 112
c) 110 38. (UECE 2008.1) Na figura abaixo, os tringulos PQR e RST
d) 105 so equilteros e congruentes e a medida de cada um de seus
lados x metros. O ponto M a interseo dos segmentos PS e
S R QR e os pontos P, R e T so colineares.

29. (UECE 2005.2) Um quadrado transformado em um retn- Q S


gulo aumentando-se um de seus lados de p% e diminuindo o ou-
tro em p%. Se sua rea ento diminuda em 1%, o valor de p :
a) 1/2 b) 1 c) 5 d) 10 M
P T
30. (UECE 2007.1) A medida, em cm2, da maior rea possvel R
de um retngulo inscrito em uma circunferncia cuja medida do
raio 1cm : Assinale a alternativa na qual se encontra a rea, em metros qua-
a) 2 b) 3 c) 2 3 d) 3 2 drados, do tringulo PMT.
3 2 3 2 3 2
31. (UECE 2007.1) As retas r e s so paralelas, a distncia entre a) 3 x 2 b) x c) x d) x
elas 7m e o segmento AB, com A r e B s, perpendicular a 2 3 4
r. Se P um ponto em AB que o segmento AP mede 3m e X e Y
so pontos em r e s, respectivamente, de modo que o ngulo XPY 39. (UECE 2006.2) No desenho abaixo h uma representao gr-
mede 90, a menor rea possvel do tringulo XPY, em m2, : cos x
fica parcial da funo f(x) = , definida no intervalo [0, [ ,
a) 21 b) 16 c) 14 d) 12 1 + cos x
e um trapzio retangular OPQR sombreado, no qual os vrtices P
32. (UECE 2007.1) No retngulo XYZW, os lados XY e YZ me- e Q pertencem ao grfico de f(x).
dem, respectivamente, 8m e 6m.
W Z y

O Q
R
X Y
M 0 P x
Se M o ponto mdio do lado XY, ento a medida, em m, da
rea da regio sombreada : Sabendo que o vrtice R tem ordenada 1/3, a rea do trapzio,
a) 22 b) 20 c) 18 d) 16 em unidades de rea, :
a) 7 b) 5 c) 7 d) 5
33. (UECE 2007.2) Em um retngulo XYWZ, seja M, o ponto 18 18 36 36
mdio do lado XY, e seja N, o ponto de interseo da diagonal
XW com o segmento ZM. Se a medida da rea do tringulo XMN 40. (UFC 2002) Considere a figura abaixo, na qual:
1m2, ento a medida da rea do retngulo XYWZ igual a: o seguimento de reta AB tangente circunferncia em A;
a) 16m2 b) 14m2 c) 12m2 d) 10m2 o segmento de reta AC dimetro da circunferncia ;
o comprimento do segmento de reta AB igual metade da
34. (MACK-SP) Em um trapzio, os lados paralelos medem comprimento da circunferncia .
16cm e 44cm, e os lados no paralelos, 17cm e 25cm. A rea do Ento a rea do tringulo ABC dividido pela rea de igual a:
trapzio, em cm2, : C
a) 250 b) 350 c) 450 d) 550 e) 650

35. (UECE 2007.2) Gilberto agricultor e deseja aumentar a


rea de sua roa, que tem a forma de um quadrado, em 69%. Se
a roa, depois de ampliada, continua tendo a forma de um qua-
drado, ento a medida do lado do quadrado da roa inicial deve B
ser aumentada em: A
a) 18% b) 22% c) 26% d) 30% a) 1/2 d) 4/3
b) 2/3 e) 5/3
36. (UECE 2008.2) Se aumentarmos, na mesma proporo, o c) 1
comprimento dos lados d um quadrado, sua rea ter um au-
mento de 69%. Nestas condies, a porcentagem de aumento de 41. (UFC 2003) Sejam e os ngulos de um tringulo retn-
cada lado foi: gulo. Se sen = sen e se a medida da hipotenusa 4cm, a rea
a) 20% b) 30% c) 34,5% d) 69% desse tringulo (em cm) :
a) 2 d) 12
37. (UECE 2008.1) A corda PQ de uma circunferncia S, cuja b) 4 e) 16
medida do dimetro 10m, o dimetro de uma outra circun- c) 8

Apostilas UECEVEST mod3.indb 182 06/02/2011 10:00:26


MATE M T I C A I

42. (ENEM 2010) Em canteiros de obras de construo civil a) d)


comum perceber trabalhadores realizando medidas de compri-
mento e de ngulos e fazendo demarcaes por onde a obra deve
comear ou se erguer. Em um desses canteiros foram feitas al-
gumas marcas no cho plano. Foi possvel perceber que, das seis b) e)
estacas colocadas, trs eram vrtices de um tringulo retngulo e
as outras trs eram os pontos mdios dos lados desse tringulo,
conforme podem ser visto na figura, em que as etacas foram in-
dicadas por letras. c)

45. (ENEM 2009) O governo cedeu terrenos para que famlias


construssem suas residncias com a condio de que no mnimo
94% da rea do terreno fosse mantida como rea de preserva-
o ambiental. Ao receber o terreno retangular ABCD, em que
A regio demarcada pelas estacas A, B, M e N deveria ser calada BC
AB = , Antnio demarcou uma rea quadrada no vrtice A,
com concreto. Nessas condies, a rea a ser calada corresponde 2
a) mesma rea do tringulo AMC. para a construo de sua residncia, de acordo com o desenho, no
b) mesma rea do tringulo BNC. qual AE = AB lado do quadrado.
c) metade da rea formada pelo tringulo ABC. 5
d) ao dobro da rea do tringulo MNC.
e) ao triplo da rea do tringulo MNC.

43. (ENEM 2010) A loja Telas & Molduras cobra 20 reais por
metro quadrado de tela, 15 reais por metro linear de moldura,
mais uma taxa fixa de entrega de 10 reais. Uma artista plstica
precisa encomendar telas emolduras a essa loja, suficientes para
8 quadros retangulares (25cm 50cm). Em seguida, fez uma se-
gunda encomenda, mas agora para 8 quadros retangulares (50cm Nesse caso, a rea definida por Antnio atingiria exatamente o
100cm). O valor da segunda encomenda ser limite determinado pela condio se ele
a) o dobro do valor da primeira encomenda, porque a altura e a) duplicasse a medida do lado do quadrado.
a largura dos quadros dobraram. b) triplicasse a medida do lado do quadrado.
b) maior do que o valor da primeira encomenda, mas no o c) triplicasse a rea do quadrado.
dobro. d) ampliasse a medida do lado do quadrado em 4%.
c) a metade do valor da primeira encomenda, porque a altura e e) ampliasse a rea do quadrado em 4%.
a largura dos quadros dobraram.
d) menor do que o valor da primeira encomenda, mas no a 46. (ENEM 2002) As cidades de Quito e Cingapura encontram-
metade. se prximas linha do equador e em pontos diametralmente
e) igual ao valor da primeira encomenda, porque o custo de opostos no globo terrestre. Considerando o raio da Terra igual a
entrega ser o mesmo. 6370 km, pode-se afirmar que um avio saindo de Quito, voando
em mdia 800 km/h, descontando as paradas de escala, chega a
44. (ENEM 2002) Um terreno com o formato mostrado na figu- Cingapura em aproximadamente
ra foi herdado por quatro irmos e dever ser dividido em quatro a) 16 horas. d) 32 horas.
lotes de mesma rea. Um dos irmos fez algumas propostas de b) 20 horas. e) 36 horas.
diviso para que fossem analisadas pelos demais herdeiros. Dos c) 25 horas.
esquemas abaixo, onde lados de mesma medida tm smbolos
iguais, o nico em que os quatro lotes no possuem, necessaria- 47. (ENEM 2008) O tangram um jogo oriental antigo, uma
mente, a mesma rea : espcie de quebra-cabea, constitudo de sete peas: 5 tringu-
los retngulos e issceles, 1 paralelogramo e 1 quadrado. Essas
peas so obtidas recortando-se um quadrado de acordo com o
esquema da figura 1. Utilizando-se todas as sete peas, possvel
representar uma grande diversidade de formas, como as exempli-
ficadas nas figuras 2 e 3.

Figura 1 Figura 2 Figura 3


Se o lado AB do hexgono mostrado na figura 2 mede 2 cm,
ento a rea da figura 3, que representa uma casinha, igual a

UECEVEST 183

Apostilas UECEVEST mod3.indb 183 06/02/2011 10:00:28


M AT E MTICA I

a) 4 cm2. d) 14 cm2. 51. (ENEM 2000) Em uma empresa, existe um galpo que precisa
b) 8 cm2. e) 16 cm2. ser dividido em trs depsitos e um hall de entrada de 20 m2, con-
c) 12 cm2. forme a figura abaixo. Os depsitos I, II e III sero construdos para
o armazenamento de, respectivamente, 90, 60 e 120 fardos de igual
48. (ENEM 2009 Prova Anulada) Dois holofotes iguais, situa- volume, e suas reas devem ser proporcionais a essas capacidades.
dos em H1 e H2, respectivamente, iluminam regies circulares,
ambas de raio R. Essas regies se sobrepem e determinam uma
regio S de maior intensidade luminosa, conforme figura.

A largura do depsito III dever ser, em metros, igual a:


Area do setor circular: Asc= R em radianos. a) 1. b) 2. c) 3. d) 4. e) 5.
2
A rea da regio S, em unidades de rea, igual a 52. (ENEM 2003) Na literatura de cordel, os textos so impressos,
em geral, com 8, 16, 24 ou 32 pginas de formato 10,5 cm x 15,5
2R 3 R R
a) - d) cm. As razes histricas que explicam tal fato esto relacionadas
3 2 2 forma artesanal como so montadas as publicaes e ao melhor
R aproveitamento possvel do papel disponvel. Considere, abaixo, a
(2 - 3 3 )R confeco de um texto de cordel com 8 pginas (4 folhas):
b) e)
12 3

R R
c) -
12 8

49. (ENEM 2009) O quadro apresenta informaes da rea


aproximada de cada bioma brasileiro.
biomas rea
rea / total
continentais aproximada
Brasil
brasileiros (km ) 2

Amaznia 4.196.943 49,29%


Cerrado 2.036.448 23,92% Utilizando o processo descrito acima, pode-se produzir um exem-
Mata Atlntica 1.110.182 13,04% plar de cordel com 32 pginas de 10,5 cm x 15,5 cm, com o
Caatinga 844.453 9,92% menor gasto possvel de material, utilizando uma nica folha de
Pampa 176.496 2,07% a) 84 cm x 62 cm d) 42 cm x 62 cm
Pantanal 150.355 1,76% b) 84 cm x 124 cm e) 21 cm x 31 cm
c) 42 cm x 31 cm
rea Total Brasil 8.514.877
Disponvel em: www.ibge.gov.br. Acesso em: 10 jul. 2009 (adaptado). 53. (ENEM 2004)
VENDEDORES JOVENS
comum em conversas informais, ou mesmo em noticirios, o
Fbrica de LONAS Vendas no Atacado
uso de mltiplos da rea de um campo de futebol (com as medi-
10 vagas para estudantes, 18 a 20 anos, sem experincia.
das de 120 m x 90 m) para auxiliar a visualizao de reas con-
Salrio: R$ 300,00 fixo + comisso de R$ 0,50 por m2 vendido.
sideradas extensas. Nesse caso, qual o nmero de campos de
Contato: 0xx97-43421167 ou atacadista@lonaboa.com.br
futebol correspondente rea aproximada do bioma Pantanal?
a) 1.400 d) 1.400.000 Na seleo para as vagas deste anncio, feita por telefone ou cor-
b) 14.000 e) 14.000.000 reio eletrnico, propunha-se aos candidatos uma questo a ser
c) 140.000 resolvida na hora. Deveriam calcular seu salrio no primeiro ms,
se vendessem 500 m de tecido com largura de 1,40 m, e no se-
50. (ENEM 2000) Um marceneiro deseja construir uma escada gundo ms, se vendessem o dobro. Foram bem sucedidos os jo-
trapezoidal com 5 degraus, de forma que o mais baixo e o mais vens que responderam, respectivamente,
alto tenham larguras respectivamente iguais a 60 cm e a 30 cm, a) R$ 300,00 e R$ 500,00.
conforme a figura: b) R$ 550,00 e R$ 850,00.
c) R$ 650,00 e R$ 1000,00.
d) R$ 650,00 e R$ 1300,00.
e) R$ 950,00 e R$ 1900,00.

54. O periscpio bsico emprega dois espelhos, paralelos, a certa


distancia um do outro. Os raios luminosos atingem o primeiro
Os degraus sero obtidos cortando-se uma pea linear de madeira espelho de modo que os ngulos a e b so iguais, que os reflete
cujo comprimento mnimo, em cm, deve ser: para segundo espelho; da so novamente refletidos para o visor.
a) 144. b) 180. c) 210. d) 225. e) 240. O trajeto completo da luz tem a forma aproximada de um Z. Os

184 UECEVEST

Apostilas UECEVEST mod3.indb 184 06/02/2011 10:00:30


MATE M T I C A I

periscpios so acessrios fundamentais dos submarinos, usados a) 696.938 km d) 700.000 km


para captar imagens acima da gua. b) 796.838 km e) 596.938 km
c) 590.000 km

58. (UNB ADPTADO ENEM) Entre as infinidades de for-


mas retangulares existentes, h uma que, desde os tempos mais
antigos, parece causar nas pessoas uma agradvel sensao visual
esttica de beleza. Essa forma, chamada de retngulo ureo o
contorno imaginrio do Partenon, construdo na Grcia, no s-
Com relao aos ngulos de instalao dos espelhos do telescpio culo V a.C.. Para se definir essa forma geomtrica especial, tome
abaixo podemos afirmar que a medida do ngulo c : um retngulo, e divida-o em um quadrado e um retngulo menor,
a) 45 b) 20 c) 80 d) 50 e) 30 como na figura abaixo. O retngulo ABCD ureo, se o retngu-
lo CDEF que se obtm pelo processo descrito acima semelhante
55. (ENCEJA 2005 ADAPTADO ENEM) A figura a seguir ao retngulo maior ABCD. Baseado no texto, a razo entre x e y :
representa o projeto de um jardim quadrado, onde haver diver- x
sos tipos de plantas. De acordo com estas informaes, a parte: y
A E x-y D

y y

B F C
a) Dos coqueiros corresponde a da gramada.
b) Gramada corresponde a do jardim. a) 1 5 d) 5 1
c) Das roseiras corresponde a 1/8 do jardim. 2
d) Dos coqueiros corresponde a 1/16 do jardim. 1 + 5 5 +1
b) e)
e) Dos coqueiros corresponde a 1/8 do ip. 2
56. A bancada de uma pia, que tem o formato e as dimenses c) 2 5
dadas na figura abaixo, deve ser fixada na parede de um banheiro
de modo que o lado maior fique encostado em uma das paredes. 59. (UFSCAR 2007 ADAPTADA ENEM) Os satlites de co-
Ento podemos afirmar que: municao so posicionados em sincronismo com a Terra, o que
F E D C significa dizer que cada satlite fica sobre o mesmo ponto da super-
fcie da Terra. Considere um satlite cujo raio da rbita seja igual a
7 vezes o raio da Terra. Na figura P e O representam duas cidades
na Terra, separadas pela maior distncia possvel em que um sinal
60cm

60cm

100cm 100cm
pode ser enviado e recebido, em linha reta, por esse satlite.

A 100cm B
a) Se a bancada for de mrmore custando R$ 100,00 o m,
ento a bancada custar R$ 118,00.
b) A bancada s poder ser fixada se a largura da parede for
maior ou igual a 2,60 m.
c) Para que a gua da torneira caia no ponto de encontro das
diagonais do retngulo ABDE a distanciado ponto final da
torneira at a parede devera ser de 40 cm.
d) O permetro da bancada ser maior que 570 cm.
e) Se a mo-de-obrado do pedreiro custa R$ 100,00, o m da Se R a medida do raio da Terra, para ir de P at Q, passando
bancada R$ 100,00 e outros materiais necessrios R$ 300,00 pelo satlite, o sinal percorrer, em linha reta, a distncia de:
o custo da obra ser R$ 510,00. a) 6( 3 )R. d) 10( 3 )R.
b) 7( 3 R. e) 11( 2 )R.
57. (UNICAMP ADAPTADA ENEM) Num eclipse total
do Sol, o disco lunar cobre exatamente o disco solar, o que com- c) 8( 3 )R.
prova que o ngulo sob o qual vemos o sol o mesmo sob o qual
vemos a Lua. Considerando que o raio da Lua 1.738 kme que 60. (UNIFOR 2002 ADAPTADO ENEM) A estrela repre-
a distancia da Lua ao Sol 400 vezes a da Terra Lua, calcule o sentada na figura abaixo foi usada para representar a superfcie
raio do Sol. Considere a distancia da Lua ao Sol como a distancia de um municpio em um mapa, e suas pontas foram obtidas das
do centro da Lua ao centro do Sol). interseces dos prolongamentos dos lados de um hexgono re-
Sol gular de 0,2 cm de lado.
Terra Lua

r
R

UECEVEST 185

Apostilas UECEVEST mod3.indb 185 06/02/2011 10:00:31


M AT E MTICA I

Se esse mapa foi feito na escala 1:2 500 000, isto , cada I cm no Exemplo:
mapa corresponde a 2 500 000 cm de medida real, a rea real da
superfcie do municpio, em quilmetros quadrados, igual a:
a) 150 3 d) 50 3
b) 75 3 e) 25 3
c) 125/2 3

61. (UFRN 2003 ADAPTADO ENEM) Miguel pintar um


painel retangular com motivos geomtricos. As duas regies des-
tacadas, a regio I (FGKM), contida no quadrado FGLM, e a
regio 2 (HILK), contida no paralelogramo HILM, conforme Elementos de um poliedro convexo
figura a seguir, sero pintadas de vermelho. Sabe-se que a tinta Faces So os polgonos.
utilizada para pintar uma regio qualquer depende proporcional- Arestas So os lados dos polgonos. Vrtices - So os vrtices
mente de sua rea. dos polgonos.

Relaco de Euler
vA+F=2
Para um poliedro convexo com v vrtices, F faces e A arestas.

Poliedros de Plato
Se Miguel gastasse na pintura da regio I, 3/7 da tinta vermelha Denomina-se poliedro de Plato o poliedro que satisfaz a s
de que dispe, podemos afirmar que: seguintes condies:
a) O restante de tinta vermelha daria, exatamente, para a Todas as faces tem o mesmo nmero de arestas. De cada vrtice
pintura da regio 2. parte o mesmo nmero de arestas.
b) O restante de tinta vermelha seria insuficiente para a pintura Existem apenas cinco poliedros de Plato: Tetraedro - possui
da regio 2. 4 faces triangulares. Hexaedro - possui 6 faces quadrangulares.
c) A regio 2 seria pintada e ainda sobrariam 3/7 de tinta vermelha. Octaedro - possui 8 faces triangulares. Dodecaedro - possui 12
d) A regio 2 seria pintada e ainda sobraria 1/7 de tinta vermelha. faces pentagonais.
Icosaedro - possui 20 faces tringulares.

G A B A R I T O OBS.:
Para um poliedro de Plato cujas faces so polgonos regulares
01. c 02. c 03. e 04. d 05. a 06. b congruentes existem somente cinco tipos de poliedros regulares.
07. d 08. d 09. b 10. c 11. d 12. c
13. c 14. a 15. a 16. d 17. b 18. c Poliedro Regular:
um poliedro de Plato cujas faces so polgonos regulares
19. d 20. e 21. b 22. c 23. d 24. a congruentes, existem somente cinco tipos de poliedros regulares.
25. c 26. d 27. c 28. a 29. d 30. a
31. d 32. d 33. c 34. c 35. d 36. b Prismas
37. a 38. d 39. d 40. c 41. b 42. e
43. b 44. e 45. c 46. c 47. b 48. a
49. e 50. d 51. d 52. d 53. c 54. a
55. c 56. b 57. a 58. b 59. c 60. b
Prisma reto: tem aresta lateral perpendicular ao plano das bases.
61. d Prisma oblquo: as arestas laterais so oblquas aos planos das
bases.
rea lateral: a soma das reas de todas as faces laterais.
rea total: a soma das reas de todas faces do prisma.
GEOmETRIA ESPACIAl Volume do Prisma: dado por V = Sb.h onde Sb a rea de
sua base e h sua altura.
POlIEDROS (UECE/EnEm)
So slidos geomtricos limitados por polgonos de tal modo Paraleleppedo
que esses polgonos tenham dois a dois um lado comum. Paraleleppedo o prisma de seis faces onde todas so parale-
Exemplos: logramos. Paraleleppedo retngulo, tambm ortoedro ou bloco
retangular.

Volume do paraleleppedo Reto-Retngulo

Poliedros Convexos
Um poliedro convexo se est situado por completo em um
dos semi-espaos determinados por qualquer um dos planos que
contm cada face desse poliedro. Caso contrrio no-convexo.
onde a, b e c so arestas laterais.

186 UECEVEST

Apostilas UECEVEST mod3.indb 186 06/02/2011 10:00:33


MATE M T I C A I

rea total: AT = 2(ab + ac + bc). est sendo abasteci da razo constante de 50/min. O tempo
Diagonal do paraleleppedo de aresta a, b, e c necessrio, em horas, para preencher esta piscina :
a) 26 b) 24 c) 22 d) 20 e) 18

OBS.: 06. (UECE/98) A razo entre os volumes de dois cubos . Em


(a+b+c)2 = a2 + b2 + c2 + 2(ab+ac+bc) relao s arestas dos cubos podemos dizer que:
(a+b+c)2 = D2 + AT a) so iguais.
b) uma delas o dobro da outra.
Cubo c) uma delas o triplo da outra:
O cubo um caso particular do paraleleppedo retngulo; no d) uma delas o qudruplo da outra.
cubo todas as seis faces so quadradas e suas trs dimenses so iguais.
07. (UNIFOR/98) Um paraleleppedo retngulo tal que suas
dimenses, dadas em cm, so termos consecutivos de uma P.G.
de razo . Se o volume do paraleleppedo 1000 cm, sua rea
total, em cm, e:
a) 780 d) 390
b) 760 e) 380
No cubo, temos: c) 640
Volume V: V = B . H V = a . a . a v = a3
rea S: a soma das reas dos seis quadrado; 08. (UFC) Sabendo-se que ao aumentarmos de 3 cm a diagonal.
Logo temos: D de um cubo, a rea total desse cubo aumenta de 78 cm2, ento
podemos afirmar que D, em cm mede:
S = 6.a2 a) 2 b) 3 c) 4 d) 5 e) 6
Diagonal D: usamos a frmula obtida para o paraleleppedo 09. (PUC-RS) Se um prisma quadrangular regular tem rea total
retngulo, fazendo a = b = c; temos, portanto: igual a 10 vezes a rea da base, ento a razo entre sua altura e a
aresta da base :
a) 1/2 b) 1 c) 3/2 d) 2 e) 3

10. (UECE) A altura de um prisma hexagonal regular mede 15


E X E R C C I O cm. Se o volume desse prisma 360 cm ento uma das arestas
da base, em cm, mede:
01. (UFC-04) Um poliedro convexo s tem faces triangulares e
a) 4 b) 5 c) 6 d) 7
quadrangulares. Se ele tem 20 arestas e 10 vrtices, ento, o n-
mero de faces triangulares :
11. Um paraleleppedo retngulo tem volume V I e arestas de
a) 12 b) 11 c) 10 d) 9 e) 8
medidas a, b e c centmetros. Outro paraleleppedo retngulo
tem volume V 2 e arestas cujas medidas correspondem s do an-
02. (UFC 2000) Um poliedro convexo de nove vrtices possui
terior multiplicadas por um nmero real positivo k. Nessas con-
quatro ngulos tridricos e cinco ngulos tetradricos. Ento o
nmero de faces desse poliedro : dies, a razo igual a:
a) 12 b) 11 c) 10 d) 9 e) 8
a) k3 d)
03. (UFC 2008) O nmero de faces de um poliedro convexo
com 20 vrtices e com todas as faces triangulares igual a: b) e) k
a) 28 b) 30 c) 32 d) 34 e) 36
c) k2
04. (UNIFOR/99) Considere um prisma reto feito de vidro mui-
to fino, com as dimenses dadas na figura e contendo lquido 12. (CESESP-PE) Considere um prisma reto cuja aresta lateral e a
at uma altura de 16 cm. Imagine que esse prisma seja colocado aresta da base tm medidas iguais e cuja base um losango de dia-
sobre uma superfcie horizontal, apoiado sobre uma das faces la- gonais 6cm e 8cm. Determine o volume e a rea lateral do prisma.
terais de menor rea.

a) 120 cm c) 200 cm
b) 150 cm d) 240 cm
Nessas condies, a altura do lquido ser de:
a) 10cm d) 8,5cm
b) 9,5cm e) 8cm G A B A R I T O
c) 9cm
01. e 02. d 03. e 04. e 05. b 06. d
05. (UFC-98) Uma piscina na forma de um paraleleppedo re- 07. a 08. d 09. d 10. a 11. a 12. a
tngulo de 9m de comprimento, 4m de largura e 2m de altura

UECEVEST 187

Apostilas UECEVEST mod3.indb 187 06/02/2011 10:00:35


M AT E MTICA I

Pirmides Lado Aptema

Classificao e elementos T

H R

Tetraedro regular
uma pirmide em que todas as faces so tringulos equilteros.

Considerando a figura acima:


o aptema lateral do tetraedro a altura de um tringulo equi-
ltero ou seja, .

rea da base Ab = P.a o ponto H sendo o centro de um tringulo equiltero baricen-


rea total AT=Al+Ab tro: logo .

Volume
rea de base
Uma pirmide dita regular quando sua base for um po-
lgono regular e a projeo ortogonal do vrtice coincidir como rea total
centro da base.
As pirmides so classificadas de acordo com o nmero de Volume
lados dos polgonos da base.
Pirmide triangular a base um tringulo.
Pirmide quadrangular a base um quadriltero. Tronco de pirmide
Pirmide pentagonal a base um pentgono. Considere o tronco de pirmide da figura abaixo:
Pirmide hexagonal a base um hexgono.

Tringulo equiltero

B = rea da base maior


Quadrado b = rea da base menor
H = altura da pirmide AXYZW
h = altura da pirmide AXYZW
k = altura do tronco
v = volume da pirmide AXYZW
V = volume da pirmide AXYZW
t = aptema do tronco
Hexgono regular

Propriedade

Volume

188 UECEVEST

Apostilas UECEVEST mod3.indb 188 06/02/2011 10:00:39


MATE M T I C A I

E X E R C C I O 08. (PUC - PR) Seccionando-se um cubo de 1m de aresta por


um ponto que passa pelos pontos mdios I e J das arestas EF e
01. Considere a pirmide quadrangular regular indicada na figu- FG e pelo vrtice B do cubo, conforme a figura, obtemos uma
ra. Calcule: pirmide. Razo entre o volume dessa pirmide e o do slido
geomtrico resultante da seco do cubo igual a:
a) 1/11
b) 1/7
c) 1/23
d) 1/3
e) 1/8

a) a medida do aptema da base.


b) a medida do aptema da pirmide.
c) a medida da aresta lateral.
d) a rea total da pirmide. 09. Uma pirmide quadrada tem todas as arestas medindo 2 cm.
02. Numa pirmide regular de base quadrangular, a medida do Ento, a sua altura, em cm, mede:
permetro da base 40cm. Sabendo que a altura da pirmide a) 1 d) 2
12cm, re lateral dessa pirmide : b) e) 3
a) 200 cm2 c) 260cm2 c)
b) 230cm2 d) 300cm2
10. Considere o tetraedro regular MPQR, de aresta a, represen-
03. Calcule a rea lateral de uma pirmide triangular regular, cuja tado na figura. Determine a rea do tringulo M N P, que N
aresta lateral mede 13cm e o aptema da pirmide mede 12cm. ponto mdio de QR.
a) 1000cm2 c) 220cm2
b) 200cm 2
d) 240cm2 a)

04. A figura nos mostra um cubo de aresta igual a 2cm. Tomando-


se como base o quadrado ABCD e como vrtice o ponto V (centro b)
da face ABCD do cubo), obtm-se um pirmide. A rea total
dessa em cm2 , e: c)
a) 4 +
b) 4(1 + )
a2 2
c) +5 d)
d) 5(1 + ) 5

05. O slido da figura composto de uma paraleleppedo re-


tngulo, de dimenses 40cm, 16cm e 16cm, e duas pirmides 11. Uma pirmide regular de altura 12cm tem como base um
quadrangulares regulares igual de altura 15cm, externas ao pa- quadrado de lado 10cm. Sua rea lateral, em centmetros qua-
raleleppedo e com as bases respectivamente coincidentes com drado, :
as faces quadradas do paraleleppedo. A rea total desse slido : a) 360 b) 260 c) 180 d) 100 e) 65
a) 4863 cm2
b) 3600 cm2 12. Um tronco de pirmide regular tem como bases tringulos
c) 3648 cm2 equilteros cujos lados medem, respectivamente, 2cm e 8 cm. A
d) 4215 cm2 aresta lateral mede 5cm. O volume deste tronco :
a) 7 cm c) 9 cm
06. O volume de um tetraedro regular de aresta a 18 cm. A b) 7 cm d) 9 cm
medida de sua aresta :
a) 6 cm c) 4 cm 13. O aptema de um tronco de pirmide regular mede 10dm,
b) 5 cm d) 3 cm as bases so quadrados de lados, respectivamente, 8dm e 20dm.
O volume desse tronco :
07. Uma pirmide triangular regular tem 12cm de altura. Deter- a) 1000 dm3 c) 1664 dm3
minar a que distncia do vrtice deve -se traar um plano paralelo b) 1306 dm 3
d) 1771 dm3
base, de modo que a rea da seco obtida seja da rea da base.
a) 8 cm
b) 7 cm
c) 6 cm G A B A R I T O
d) 5 cm 01. * 02. c 03. a 04. b 05. c 06. a 07. c
08. c 09. b 10. a 11. b 12. b 13. c
* a) a = 6 cm b)
c) d)

UECEVEST 189

Apostilas UECEVEST mod3.indb 189 06/02/2011 10:00:43


M AT E MTICA I

CIlInDRO 03. (ETI) Em um recipiente cilndrico cujo raio da base dm


foram introduzidos 4 litros de gua pura. Portanto, a altura, em
dm, atingida pelo lquido no recipiente de:
a) d)

b) c) 20

04. (ULBRA) Um cilindro equiltero tem rea da base 25p cm2.


Seu volume, em cm3, .
a) 500 p c) 200 p
b) 250 p d) 100 p

05. (AMAN) Aumentando-se de 20% o raio da base de um cilindro


circular reto e reduzindo-se sua altura em 10%, temos que o volume:
a) No variar. d) Aumentar de 44%.
b) Aumentar de 34% e) Aumentar de 33,6%.
rea da Base: Ab=pR2 rea lateral: AL=2pRh c) Aumentar de 29,6%.
rea total: At=2pR(h+R) Volume: V=pR2h
06. (UFPA) Dois cilindros equilteros A e B tem os raios da base
Seces transversal iguais a e R1 e R2, respectivamente. A razo R1/R2 igual a 1/2.
a interseco do cilindro com um plano paralelo s duas Ento, a razo entre os volumes de A e B :
bases. Esta seco ser um crculo. a) 1/16 c) 1/2
b) 1/8 d) 1/4
Seco meridiana
a interseco do cilindro com um plano que contm o eixo 07. Um cilindro circular reto com raio da base r = 5cm, e altura
do cilindro. Num cilindro circular reto, a seco meridiana re- 2r, seccionado por um plano paralelo ao seu eixo, a uma distn-
sulta um retngulo. cia x = 3cm desse eixo. A rea dessa seco :
a) 50 cm2 c) 70 cm2
b) 60 cm2 d) 80 cm2

08. (FUVEST - SP) Um cano cilndrico, com 4cm de dimetro e


50m de comprimento est acoplado a uma caixa d gua de forma
cbica com 1 m de lado. Num certo instante, a caixa est cheia de
gua e o cano vazio. Solta-se a gua pelo cano at que fique cheio.
Qual o valor aproximado da altura da gua na caixa no instante
em que o cano ficou cheio?
Cilindro equiltero a) 90cm d) 96cm
um cilindro circular reto cuja seco meridiana um qua- b) 92cm e) 98cm
drado. c) 94cm
Note que h = 2r
09. (UECE) O volume de um cilindro circular reto .
rea lateral: Se a altura desse cilindro mede , ento a rea total do cilin-
Al = 2r . (2r) dro, em cm2, :
h = 2r Al = 4r a) 72 p c) 92 p
b) 84 p d) 96 p
rea total:
At = 2r . (2r + r) 10. Se o volume e a rea lateral de um cilindro circular reto so,
At = 6r respectivamente, 200p cm3 e 80p cm2, ento a rea total desse
cilindro :
a) 110p cm2 c) 130p cm2
E X E R C C I O b) 120p cm 2
d) 140p cm2
01. A rea da superficie total de um cilindro equiltero, cujo raio 11. (UNIFOR) Considere um cilindro reto cujo raio da base
das bases mede 8dm, : mede r cm e cuja altura mede 1 cm. Aumentando-se o raio em
a) 384p dm2 c) 265p dm2 3cm o volume aumenta em x cm3; por outro lado, se a altura
b) 348p dm 2
d) 256p dm2 fosse acrescida de 3cm o volume tambm aumentaria de x cm3.
O volume do cilindro original, em cm3, :
02. (PUCRS) O volume do cilindro equiltero cuja seo meri- a) 4p d) 12p
diana tem rea igual a x igual a: b) 6p e) 15p
a) c) c) 9p

12. (ESAN) Um cilindro reto, cuja base um crculo de raio r =


b) d) 3m, tem 108p m, de volume. Ento a rea total desse cilindro :

190 UECEVEST

Apostilas UECEVEST mod3.indb 190 06/02/2011 10:00:46


MATE M T I C A I

a) 126p m2 c) 72p m2 reas e Volumes


b) 81p m2 d) 90p m2

13. A rea lateral de um cilindro circular reto 180p cm2. Se o


volume desse cilindro 180 5 cm3, ento a rea de base desse
cilindro em cm, :
a) 10p c) 16p
b) 15p d) 20p

G A B A R I T O
01. a 02. c 03. d 04. b 05. c 06. b 07. d rea da base Ab= pR2 rea total At= pR(g+R)
08. c 09. b 10. c 11. c 12. c 13. d rea lateral Al = pRg Volume V=1/3pR2h

Cone Equiltero
um cone cuja seco meridiana um tringulo equiltero.
COnE g = 2r
h=r
Definio
rea lateral: AL = pr . 2 r = 2p r2 .
Seja o crculo de centro O e raio r, contido no plano .
Cone a reunio de todos os segmentos com um extremo em
rea total: AT = 2p r2 + pr2 = 3p r2 .
V e outro extremo no crculo.
Volume:
Elementos

Tronco de cone

Base o crculo de O e raio r.


Vrtice o ponto V. Propriedades
Geratrizes so os segmentos com extremos em V e num ponto 2 3
da circunferncia de base. h r g h b h v
Altura a distncia h do vrtice V ao plano da base. H R G H B H V
Eixo a reta determinada pelo vrtice V e pelo centro O do
crculo. Onde, B a rea da base maior e b a rea da base menor. V
o volume do cone maior e v o volume do cone menor.
Classificao
Area Lateral AL=pk(r+R)
Cone circular oblquo Area Total AT= AL +B+b
o cone de eixo oblquo ao plano
da base. Volume

E X E R C C I O
01. Um cone circular reto tem 12cm de altura e 13cm de geratriz.
O volume desse cone :
Cone circular reto a) 98pcm3 c) 100pcm3
o cone de eixo perpendicular ao b) 99pcm 3
d) 101cm3
plano da base.
tambm chamado cone de revo- 02. (UECE) O raio da base de um cone circular reto mede 5 cm.
luo. Se a rea total desse cone 90p cm2, ento o volume desse cone,
em cm3, :
Relao: g2 = h2 + r2 a) 75p c) 100p
b) 80p d) I50p

UECEVEST 191

Apostilas UECEVEST mod3.indb 191 06/02/2011 10:00:49


M AT E MTICA I

03. (FUVEST-SP) Deseja-se construir um cone circular reto com 10. Um recipiente cnico, com altura 2 e raio da base 1, em
4cm de raio da base e 3cm de altura. Para isto, recorta-se, em car- unidades de medida, contm gua at a metade de sua altura.
tolina, um setor circular para a superfcie lateral e um crculo para Inverte-se a posio do recipiente, o vrtice fica para baixo. Nessa
a base. A medida do ngulo central do setor circular : situao a distncia do nvel da gua ao vrtice do recipiente ,
a) 144o e) 336o em unidades de medidas:
b) 192 o
d) 288o a) d)
c) 240 o

b) e)
04. Se a rea da superfcie de um cone equiltero 48pcm2. En-
to seu volume : c)
a) c)
11. Um cone circular reto cortado em duas partes por um plano
b) d) paralelo sua base e que passa pelo ponto mdio de sua altura. Se v e
V volumes da menor e da maior dessas respectivamente, ento vale:
a) 1/9 d) 2/7
05. A figura abaixo representa um cilindro do qual foi retirado um b) 1/8 e) 2/9
cone. O volume da parte do slido que permanece onde estava : c) 1/7
a)
12. (ITA) A geratriz de um cone circular reto forma com o eixo
b) deste cone um ngulo de 45. Sabendo-se que o permetro de sua
seco meridiana mede 2cm, podemos afirmar que a rea total
c) deste cone vale:
a) d)
d) b) e)
c)
06. O raio da base, a altura e a geratriz de um cone circular reto
constituem, nessa ordem, uma progresso aritmtica de razo
igual a 1. Logo o volume desse cone , em unidades de volume:
a) 4p c) 10p
G A B A R I T O
b) 8p d) 12p 01. c 02. c 03. d 04. b 05. a 06. d
07. A rea da superfcie total do slido indicado na figura : 07. a 08. b 09. d 10. d 11. b 12. b

6cm
ESFERA
15cm 10cm
Seja um ponto O e um segmento de medida r. Esfera o
conjunto de todos os pontos A do espao, tais que a medida do
a)
segmento OA menor ou igual a r.
b)
c)
d)

08. Com um setor circular, cujo ngulo central mede 120, cons-
tri-se um cone circular reto de raio igual a 3cm. O volume do
cone assim obtido : Superfcie esfrica o conjunto dos pontos A tal que a me-
a) 9 cm c) 9 cm dida do segmento OA igual a R
b) 18 cm d) 18 cm

09. (UECE 2008.1) Um cone reto, cuja medida da altura h


seccionado, por um plano paralelo base, em duas partes: um
h
cone cuja medida da altura e um tronco de cone, conforme
5
a figura. A razo entre as medidas dos volumes do cone maior e
do cone menor : rea da superfcie esfrica
S = 4p R2

Volume

a) 15 b) 45 c) 90 d) 125

192 UECEVEST

Apostilas UECEVEST mod3.indb 192 06/02/2011 10:00:55


MATE M T I C A I

Seco 03. (FAAP-SP) O comprimento de uma caldeira cilndrica termi-


A interseco da esfera com todo plano que lhe seja secante nada por duas semi-esferas l = 4m. Se a rea total da caldeira
um crculo. Veja a relao mtrica determinada por um plano 4pm2, ento a altura e o raio da parte cilndrica medem, respec-
secante numa esfera: tivamente:
Raio da seco: r a) 2m e 3m
Distncia de O a O: d b) 3m e 2m
Raio da esfera: R c) 1/2m e 3m
R2 = d2 + r2 d) 2m e 1 m

Quando a seco contm o centro da esfera, crculo obtido


chamado crculo mximo, pois seu raio igual ao raio r da esfera. 04. (UFC) Calcule, em cm3, o volume de um dado fabricado a
Seja um crculo obtido pela seco de um plano secante. Plos partir de um cubo de aresta igual a 4cm, levando em conta que os
desse crculo so os extremos do dimetro perpendicular ao plano buracos representados dos nmeros, presentes em suas
desse crculo. 1
faces, so semi-esferas de raio 3 cm.
7
rea da calota esfrica a) 64 cm2
Seja uma esfera de raio R. A rea da calota esfrica de altura h : b) 62 cm2
c) 60 cm2
A = 2pRh d) 58cm2
e) 56 cm2

05. (AESF) O volume da esfera inscrita num cubo de aresta 20


cm :
a) d)
rea do fuso esfrico
Seja uma esfera de raio R. A rea do fuso esfrico de medida
a (em radianos) :
b) e)
A = 2R
Para em rd Para graus
R c)
A=
90

06. (UNIFOR) Um pino de ao macio tem a fonna de um cilin-


Volume Da Cunha Esfrica dro circular reto acoplado a uma semi-esfera cujo dimetro mede
Seja uma esfera de raio R. Demonstra-se que o volume da 3 cm confonne mostra a figura abaixo. Se a parte cilndrica tem
cunha esfrica de medida a graus : 6 cm de altura, o volume desse pino, em centmetros cbicos, :
a) 7p
R b) 16p
V=
270
Para em rd Para em graus c)
2R
V=
3
d)

e) 72p
E X E R C C I O
07. (AFA) Qual o volume, em cm3, da esfera inscrita em um
01. dada uma esfera de raio 10cm. Um plano a secciona essa cone reto, cuja altura e dimetro da base so, respectivamente,
esfera a uma distncia de 6cm do centro da mesma. O raio de l6cm e 24cm?
seco : a) 27p c) 288p
a) 8cm d) 6cm
b) 7cm e) 5cm b) d) 686p
c) 6cm

02. (UFMG) O slido da figura formado por um hemisfrio 08. (UFC 2008) Duas esferas de raios iguais a r so colocadas no in-
cujo raio mede 3cm e um cone circular cuja altura H. Se o terior de um tubo de ensaio sob a fonna de um cilindro circular reto
volume do slido 48pcm3, determine a altura H. de raio da base r e altura 4r. No espao vazio compreendido entre as
a) 5cm esferas, a superficie lateral e as bases, superior e inferior, do tubo de
b) 8cm ensaio, coloca-se um lquido. Ento, o volume desse lquido :
c) 10cm a) d) 2pr3
d) 12cm
e) 15cm b) e) 4pr3

c)

UECEVEST 193

Apostilas UECEVEST mod3.indb 193 06/02/2011 10:00:59


M AT E MTICA I

09. (UECE 2008) Uma esfera est circunscrita a um cubo cuja a) 12 cm. d) 1.216 cm.
medida da aresta 2 m. A medida do volume da regio exterior b) 64 cm. e) 1.728 cm.
ao cubo e interior esfera c) 96 cm.
a) c)
15. (ENEM 2010) Uma fbrica produz barras de chocolates no
b) d) formato de paraleleppedos e de cubos, com o mesmo volume. As
arestas da barra de chocolate no formato de paraleleppedo medem
10. A razo entre a rea da esfera e a rea do cubo inscrito nessa 3 cm de largura, 18 cm de comprimento e 4 cm de espessura. Ana-
esfera : lisando as caractersticas das figuras geomtricas descritas, a medida
a) c) das arestas dos chocolates que tm o formato de cubo igual a
a) 5 cm. d) 24 cm.
b) 6 cm. e) 25 cm.
b) d) c) 12 cm.

11. (UECE 2007.1) Como mostrar a figura, o cilindro reto est 16. (ENEM 2010) Dona Maria, diarista na casa da famlia Teixei-
inscrito na esfera de raio 4cm. ra, precisa fazer caf para servir as vinte pessoas que se encontram
numa reunio na sala. Para fazer o caf, Dona Maria dispe de
uma leiteira cilndrica e copinhos plsticos, tambm cilndricos.
4cm
Com o objetivo de no desperdiar caf, a diarista deseja colocar
a quantidade mnima de gua na leiteira para encher os vinte
copinhos pela metade. Para que isso ocorra, Dona Maria dever.
Sabe-se que o dimetro da base e a altura do cilindro possuem a
mesma medida. O volume do cilindro :
a) 18 2 cm c) 24 2 cm
b) 32 2 cm d) 36 2 cm

12. O volume do fuso esfrico cujo ngulo 30 e raio igual a


5cm : a) encher a leiteira at a metade, pois ela tem um volume 20
vezes maior que o volume do copo.
4
a) c) b) encher a leiteira toda de gua, pois ela tem um volume 20
7 vezes maior que o volume do copo.
b) d) c) encher a leiteira toda de gua, pois ela tem um volume 10
vezes maior que o volume do copo.
d) encher duas leiteiras de gua, pois ela tem um volume 10
vezes maior que o volume do copo.
e) encher cinco leiteiras de gua, pois ela tem um volume 10
vezes maior que o volume do copo.
13. (ENEM 2010) A siderrgica Metal Nobre produz diversos
17. (ENEM 2010) Uma metalrgica recebeu uma encomenda
objetos macios utilizando o ferro. Um tipo especial de pea feita
para fabricar, em grande quantidade, uma pea com o formato de
nessa companhia tem o formato de um paraleleppedo retangu-
um prisma reto com base triangular, cujas dimenses da base so
lar, de acordo com as dimenses indicadas
6 cm, 8 cm e 10 cm e cuja altura 10 cm. Tal pea deve ser vazada
na figura que segue.
de tal maneira que a perfurao na forma de um cilindro circular
reto seja tangente s suas faces laterais, conforme mostra a figura.

O produto das trs dimenses indicadas na pea resultaria na me-


dida da grandeza
a) massa. d) capacidade.
b) volume. e) comprimento
c) superfcie.
O raio da perfurao da pea igual a
14. (ENEM 2010) Um porta-lpis de madeira foi construdo no
a) 1 cm. d) 4 cm.
formato cbico, seguindo o modelo ilustrado a seguir. O cubo de
b) 2 cm. e) 5 cm.
dentro vazio. A aresta do cubo maior mede 12 cm e a do cubo
c) 3 cm.
menor, que interno, mede 8 cm. O volume de madeira utiliza-
do na confeco desse objeto foi de
18. (ENEM 2010) Em um casamento, os donos da festa serviam
champanhe aos seus convidados em taas com formato de um
hemisfrio (Figura 1), porm um acidente na cozinha culminou
na quebra de grande parte desses recipientes. Para substituir as
taas quebradas, utilizou-se um outro tipo com formato de cone
(Figura 2). No entanto, os noivos solicitaram que o volume de
champanhe nos dois tipos de taas fosse igual.

194 UECEVEST

Apostilas UECEVEST mod3.indb 194 06/02/2011 10:01:03


MATE M T I C A I

Considere: 21. (ENEM 1999) Assim como na relao entre o perfil de um


corte de um torno e a pea torneada, slidos de revoluo resul-
Vesfera = 4 R tam da rotao de figuras planas em torno de um eixo. Girando-
3 se as figuras abaixo em torno da haste indicada obtm-se os sli-
dos de revoluo que esto na coluna da direita.
Vcone = 1 Rh
3
Sabendo que a taa com formato de hemisfrio servida com-
pletamente cheia, a altura do volume de champanhe que deve ser
colocado na outra taa, em centmetros, de
a) 1,33. d) 56,52.
b) 6,00. e) 113,04.
c) 12,00.

19. (ENEM 2010) Uma empresa vende tanques de combustveis


de formato cilndrico, em trs tamanhos, com medidas indicadas
nas figuras. O preo do tanque diretamente proporcional
medida da rea da superfcie lateral do tanque. O dono de um
posto de combustvel deseja encomendar um tanque com menor
custo por metro cbico de capacidade de armazenamento.

A correspondncia correta entre as figuras planas e os slidos de


revoluo obtidos :
Qual dos tanques dever ser escolhido pelo dono do posto? (Con- a) 1A, 2B, 3C, 4D, 5E.
sidere 3) b) 1B, 2C, 3D, 4E, 5A.
a) I, pela relao rea/capacidade de armazenamento de 1/3 c) 1B, 2D, 3E, 4A, 5C.
b) I, pela relao rea/capacidade de armazenamento de 4/3 d) 1D, 2E, 3A, 4B, 5C.
c) II, pela relao rea/capacidade de armazenamento de 3/4 e) 1D, 2E, 3B, 4C, 5A.
d) III, pela relao rea/capacidade de armazenamento de 2/3
e) III, pela relao rea/capacidade de armazenamento de 7/12 22. (ENEM 2009) Um arteso construiu peas de artesanato
interceptando uma pirmide de base quadrada com um plano.
20. (ENEM 2007) A diversidade de formas geomtricas espaciais Aps fazer um estudo das diferentes peas que poderia obter, ele
criadas pelo homem, ao mesmo tempo em que traz benefcios, causa concluiu que uma delas poderia ter uma das faces pentagonal.
dificuldades em algumas situaes. Suponha, por exemplo, que um Qual dos argumentos a seguir justifica a concluso do arteso?
cozinheiro precise utilizar exatamente 100 mL de azeite de uma lata a) Uma pirmide de base quadrada tem 4 arestas laterais e a
que contenha 1 200 mL e queira guardar o restante do azeite em interseo de um plano com a pirmide intercepta suas arestas
duas garrafas, com capacidade para 500 mL e 800 mL cada, deixan- laterais. Assim, esses pontos formam um polgono de 4 lados.
do cheia a garrafa maior. Considere que ele no disponha de instru- b) Uma pirmide de base quadrada tem 4 faces triangulares e,
mento de medida e decida resolver o problema utilizando apenas a quando um plano intercepta essa pirmide, divide cada face
lata e as duas garrafas. As etapas do procedimento utilizado por ele em um tringulo e um trapzio. Logo, um dos polgonos
esto ilustradas nas figuras a seguir, tendo sido omitida a 5 etapa. tem 4 lados.
c) Uma pirmide de base quadrada tem 5 faces e a interseo
de uma face com um plano um segmento de reta. Assim,
se o plano interceptar todas as faces, o polgono obtido nessa
interseo tem 5 lados.
d) O nmero de lados de qualquer polgono obtido como
interseo de uma pirmide com um plano igual ao
nmero de faces da pirmide. Como a pirmide tem 5 faces,
o polgono tem 5 lados.
e) O nmero de lados de qualquer polgono obtido
interceptando-se uma pirmide por um plano igual ao
Qual das situaes ilustradas a seguir corresponde 5 etapa do nmero de arestas laterais da pirmide. Como a pirmide
procedimento? tem 4 arestas laterais, o polgono tem 4 lados.

a) d) 23. (ENEM 2005) Os trs recipientes da figura tm formas dife-


rentes, mas a mesma altura e o mesmo dimetro da boca. Neles
so colocados lquido at a metade de sua altura, conforme in-
dicado nas figuras. Representando por V1, V2 e V3 o volume de
b) e) lquido em cada um dos recipientes, tem-se

c)
V1 V2 V3

UECEVEST 195

Apostilas UECEVEST mod3.indb 195 06/02/2011 10:01:04


M AT E MTICA I

a) V1 = V2 = V3 d) V3 < V1 < V2 de 500 litros de gua. Suponha que, um certo dia, aps uma
b) V1 < V3 < V2 e) V1 < V2 = V3 campanha de conscientizao do uso da gua, os moradores das
c) V1 = V3 < V2 900 casas abastecidas por esse reservatrio tenham feito econo-
mia de 10% no consumo de gua. Nessa situao,
24. (ENEM 2009 Anulada) Em uma praa pblica, h uma
fonte que formada por dois cilindros, um de raio r e altura h2,
e o outro de raio R e altura h2. O cilindro do meio enche e, aps
transbordar, comea a encher o outro.

Se R = r e h2 = h1/3 e, para encher o cilindro do meio foram a) a quantidade de gua economizada foi de 4,5 m.
necessrios 30 minutos, ento, para se conseguir encher essa fon- b) a altura do nvel da gua que sobrou no reservatrio, no final
te e o segundo cilindro, de modo que fique completamente cheio, do dia, foi igual a 60 cm.
sero necessrios: c) a quantidade de gua economizada seria suficiente para
a) 20 min d) 50 min abastecer, no mximo, 90 casas cujo consumo dirio fosse de
b) 30 min e) 60 min 450 litros.
c) 40 min d) os moradores dessas casas economizariam mais de R$
200,00, se o custo de 1 m de gua para o consumidor fosse
25. (ENEM 2009 Anulada) Um artista plstico construiu, com igual a R$ 2,50.
certa quantidade de massa modeladora, um cilindro circular reto e) um reservatrio de mesma forma e altura, mas com raio da
cujo dimetro da base mede 24 cm e cuja altura mede 15 cm. An- base 10% menor que o representado, teria gua suficiente
tes que a massa secasse, ele resolveu transformar aquele cilindro para abastecer todas as casas.
em uma esfera.
Volume da esfera: Vesfera = 4R . 28. (ENEM 2009) Uma fbrica produz velas de parafina em for-
3 ma de pirmide quadrangular regular com 19 cm de altura e 6 cm
Analisando as caractersticas das figuras geomtricas envolvidas, de aresta da base. Essas velas so formadas por 4 blocos de mesma
conclui-se que o raio R da esfera assim construda igual a: altura - 3 troncos de pirmide de bases paralelas e 1 pirmide na
a) 15 d) 3 60 parte superior -, espaados de 1 cm entre eles, sendo que a base
superior de cada bloco igual base inferior do bloco sobrepos-
b) 12 e) 6 30
to, com uma haste de ferro passando pelo centro de cada bloco,
c) 24 unindo-os, conforme a figura.
26. (ENEM 2006) Uma artes confecciona dois diferentes tipos
de vela ornamental a partir de moldes feitos com cartes de papel
retangulares de 20cm X 10cm (conforme ilustram as figuras abai-
xo). Unindo dois lados opostos do carto, de duas maneiras, a
artes forma cilindros e, em seguida, os preenche completamente
com parafina.

Se o dono da fbrica resolver diversificar o modelo, retirando a


pirmide da parte superior, que tem 1,5 cm de aresta na base,
mas mantendo o mesmo molde, quanto ele passar a gastar com
parafina para fabricar uma vela?
a) 156 cm3. d) 216 cm3.
b) 189 cm .
3
e) 540 cm3.
c) 192 cm .
3

29. (ENEM 2009 Anulada) Um chefe de cozinha utiliza um


instrumento cilndrico afiado para retirar parte do miolo de uma
Supondo-se que o custo da vela seja diretamente proporcional laranja. Em seguida, ele fatia toda a laranja em seces perpen-
ao volume da parafina empregado, o custo da vela do tipo I, em diculares ao corte feito pelo cilindro. Considere que o raio do
relao ao custo da vela de tipo II ser: cilindro e da laranja sejam iguais a 1 cm e 3 cm, respectivamente.
a) O triplo d) A metade
b) O dobro e) A tera parte
c) igual

27. (ENEM 2009) A figura ao lado mostra um reservatrio de


gua na forma de um cilindro circular reto, com 6 m de altura.
Quando est completamente cheio, o reservatrio suficiente
para abastecer, por um dia, 900 casas cujo consumo mdio dirio A rea da maior fatia possvel

196 UECEVEST

Apostilas UECEVEST mod3.indb 196 06/02/2011 10:01:06


MATE M T I C A I

a) duas vezes a rea da seco transversal do cilindro. REFERnCIAS BIBlIOGRFICAS


b) trs vezes a rea da seco transversal do cilindro.
c) quatro vezes a rea da seco transversal do cilindro. BOYER, Carl B. Historia da Matemtica. So Paulo, Edgard
d) seis vezes a rea da seco transversal do cilindro. Blucher, 1974.
e) oito vezes a rea da seco transversal do cilindro.
SMOLE, Ktia C. S.& Maria Ignez S.V. Matemtica Ensino
30. (ENEM 2009) Uma empresa que fabrica esferas de ao, de 6 Mdio Volumes 1, 2 e 3. So Paulo: Saraiva 2003.
cm de raio, utiliza caixas de madeira, na forma de um cubo, para
transport-las. Sabendo que a capacidade da caixa de 13.824 IEZZI, Gelson ...[ e tal ]. Matemtica: Vol nico. So Paulo.
cm, ento o nmero mximo de esferas que podem ser transpor- Atual, 1997.
tadas em uma caixa igual a IEZZI, Gelson ...[ e tal ]. Matemtica: Vol nico. So Paulo.
a) 4. d) 24. Atual, 1998.
b) 8. e) 32.
c) 16. ALENCAR FILHO, Edgard de. Iniciao Lgica
Matemtica. So Paulo: Nobel, 2002.
31. (ENEM 2001) Em muitas regies do Estado do Amazonas,
o volume de madeira de uma rvore cortada avaliado de acordo BEZERRA, Manoel Jairo. Matemtica para o ensino mdio:
com uma prtica dessas regies: volume nico.So Paulo: Scipion, 2001.
I. D-se uma volta completa em torno do tronco com um
LIMA, Elon Lages... [ et al.]. Temas e Problemas. Rio de
barbante.
Janeiro: SBM, 2001.
LIMA, Elon Lages... [ et al.]. Temas e Problemas e
Elementares. Rio de Janeiro: SBM, 2005.
LIMA, Elon Lages... [ et al.]. A Matemtica do Ensino Medio
Temas. Rio de Janeiro: SBM, 2006.
II. O barbante dobrado duas vezes pela ponta e, em seguida,
seu comprimento medido com fita mtrica. 1 dobra 2
dobra

III. O valor obtido com essa medida multiplicado por ele


mesmo e depois multiplicado pelo comprimento do tronco.
Esse o volume estimado de madeira.
Outra estimativa pode ser obtida pelo clculo formal do volume
do tronco, considerando-o um cilindro perfeito.
A diferena entre essas medidas praticamente equivalente s
perdas de madeira no processo de corte para comercializao.
Pode-se afirmar que essas perdas so da ordem de
a) 30%. d) 12%.
b) 22%. e) 5%.
c) 15%.

32. (UECE 2004.2 Adaptado Enem) A figura, construda em pa-


pelo plano, com rea igual a 33 m, formada por um quadrado
cujo lado mede x metros e por quatro retngulos com lados medindo
2 e x metros. A caixa paraleleppedica, obtida dobrando os retngu-
los nas linhas pontilhadas, limita no seu interior um volume igual a:
a) 18 m
b) 21 m
c) 24 m
d) 27 m

G A B A R I T O
01. a 02. c 03. d 04. b 05. c 06. c
07. c 08. c 09. a 10. d 11. c 12. b
13. b 14. d 15. b 16. c 17. b 18. b
19. d 20. d 21. d 22. c 23. b 24. c
25. d 26. b 27. b 28. b 29. e 30. b
31. b 32. a

UECEVEST 197

Apostilas UECEVEST mod3.indb 197 06/02/2011 10:01:07


Apostilas UECEVEST mod3.indb 198 06/02/2011 10:01:07
P R - V E S T I B U l A R

MATEMTICA II

Apostilas UECEVEST mod3.indb 199 06/02/2011 10:01:17


Caro(a) Aluno(a),

O presente mdulo contm os seguintes objetos de conhecimento:


UECE ENEM
Fatorial -
Analise combinatoria Conhecimento de estatistica e probabilidade
Numeros binominais -
Triango de pascal -
Binomio de newton -
Estatistica Conhecimento de estatistica e probabilidade.

Apostilas UECEVEST mod3.indb 200 06/02/2011 10:01:17


MATE M T I C A I I

FATORIAl (UECE/EnEm) Arranjos simples


Seja B = {b1, b2 ..., bn} um conjunto com n ele-mentos (n N).
Definies Denomina-se arranjo simples dos n elementos de B, tomados
Seja n um nmero natural, n 2. Define-se fatorial de n, que in- p a p, qualquer agrupamento de p elementos distintos, escolhi-
dicamos por n! como o produto dos nmeros naturais consecutivos dos entre os elementos de B (p N e p n).
n . (n 1) . (n 2)... 3.2.1, isto : Indica-se: Anp ou A np
n! = n.(n 1).(n 2)...3.2.1 OBS.:
Arranjo o tipo de agrupamento em que um grupo dife-
Por definio: 0! = 1 e 1! = 1
rente de outro pela ordem ou pela natureza dos elementos com-
ponentes.
E X E R C C I O Frmula do nmero de arranjos
01. (UECE) O valor de 12! (12 + 1)! : n!
12! A n,p =
(n p)!
a) 24 b) 12 c) 6 d) 3
Arranjos com repetio
02. (UECE) Sabendo que 1(1!) + 2(2!) + 3 . (3!) + ... + n(n!) = Seja B = { b1, b2 ..., bn} um conjunto com n elementos (n N).
(n + 1)! 1, n IN*. Pode-se afirmar que: 1(1!) + 2(2!) + 3 . (3!) Denomina-se arranjo com repetio dos n elementos de B to-
+ ... + 14(14!) + 1 igual a: mados p a p, qualquer agrupamento (sequncia) de p elementos
a) 20 c) 24 no necessariamente distintos, escolhidos entre os elementos de
b) 22 d) 26 B (p N e p n ou p > n).
03. (UECE - 04.2) O nmero de divisores positivos do nmero Indica-se: (AR)n,p
75.600 : Frmula do nmero de arranjos com repetio:
a) 4! + 5! c) 4!
b) 2! + 3! + 4! d) 5! (AR)n,p = np

04. (UFC) Se n inteiro positivo, ento o valor de n que satisfaz:


n! + 1 + n! + 2 + ... + n! + n = n + 49n :
2
E X E R C C I O
2
01. (UFC) A quantidade de nmeros pares de 4 algarismos distin-
a) 24 d) 3 tos que podemos formar com os algarismos 1, 2, 4, 5, 7, 8 e 9 :
b) 4 e) 5 a) 20 d) 360
c) 6 b) 60 e) 420
c) 240
05. (UECE) Se S = 1! + 2! + 3! +... + 2000!. Ento o digito das
unidades de S : 02. (UFC) No sistema decimal, quantos nmeros inteiros positi-
a) 1 c) 5 vos, com 3 algarismos, tm todos os algarismos distintos?
b) 3 d) 7 a) 648 d) 84
b) 120 e) 900
c) 720
06. Determine n de modo que 1 + 2 + 3 + 4 + ... + n = 1 .
(n + 1)! 240
03. (UECE) Se 5.An,3 = 2.An1,4, ento An,2 igual a:
a) 30 c) 56
b) 42 d) 72

G A B A R I T O 04. (UECE) A quantidade de nmeros inteiros positivos menores


que 400 que podemos formar, utilizando somente os algarismos
01. b 02. d 03. d 04. b 05. b 06. 06 1, 2, 3, 4 e 5, de modo que no figurem algarismos repetidos, :
a) 36 c) 61
b) 56 d) 85

05. (UECE) Uma empresa possui 8(oito) scios, dos quais sero
AnlISE COmBInATRIA (UECE/EnEm) escolhidos 2(dois) para os cargos de presidente e vice-presidente.
Se n o nmero de maneiras distintas como pode ser feita a es-
Princpio fundamental da contagem colha, ento n igual a:
Se um acontecimento pode ocorrer por vrias etapas sucessi- a) 56 c) 72
vas e independentes de tal modo que: b) 64 d) 80
P1 o nmero de possibilidades da 1 etapa.
P2 o nmero de possibilidades da 2 etapa. 06. (UNIFOR) Trs homens e trs mulheres vo ocupar 3 de-
Pk o nmero de possibilidades da k-sima etapa. graus de uma escada para tirar uma foto. Essas pessoas devem
se colocar de maneira que em cada degrau fique apenas um casal.
Ento: P1 , P2 ... Pk o nmero total de possibilidades de o Nessas condies, de quantas maneiras diferentes elas podem se
acontecimento ocorrer. arrumar?

UECEVEST 201

Apostilas UECEVEST mod3.indb 201 06/02/2011 10:01:19


M AT E MTICA II

a) 1.080 d) 288 a) 128 d) 60


b) 720 e) 144 b) 120 e) 48
c) 360 c) 72

05. De quantas maneiras diferentes podem-se colocar seis pessoas


ao redor de uma mesa de forma circular se duas delas devem ficar
G A B A R I T O sempre juntas (uma ao lado da outra)?
01. d 02. a 03. c 04. c 05. a 06. d

G A B A R I T O
01. D 02. 48 03. 48 04. E 05. 48
PERmUTAES SImPlES (UECE/EnEm)
Seja B = {b1, b2, ..., bn} um conjunto com n elementos (nN).
Denomina-se permutao simples dos n elementos de B todo
arranjo dos n elementos de B, tomados n a n.
Indica-se: COmBInAES SImPlES (UECE/EnEm)
Pn = An,n
Seja B = {b1, b2, ..., bn} um conjunto com n elementos (n
OBS.:
N).
Permutao: o tipo de agrupamento ordenado no qual, em
Denomina-se combinao simples dos n elementos de B, to-
cada grupo, entram todos os elementos.
mados p a p, qualquer subconjunto de p elementos do conjunto B.
Frmula das permutaes simples:
Indica-se:
Pn = n(n 1)(n 2) ...1 = n! C n ,p ou C np

Permutao com repetio OBS.:


Permutaes com elementos repetidos Combinao o tipo de agrupamento em que um grupo di-
O nmero de permutaes possveis com n elementos, dentre ferente de outro apenas pela natureza dos elementos componentes.
os quais um certo elemento se repete n vezes, igual ao fatorial
de n dividido pelo fatorial. Frmula das combinaes simples:

Se tivermos n elementos, dos quais: n!


C n ,p =
so iguais a A. p !(n p)!
so iguais a B.
so iguais a C. Probabilidade (EnEm)
Experimentos aleatrios so aqueles que podem apresentar
O nmero de permutaes distintas dos n elementos ser: resultados distintos mesmo quando praticados em condies
n! idnticas.
Pn , , =
! ! ! Ex.: No lanamento de uma moeda para cima, no podemos pre-
ver qual face cair voltada para cima.
Permutao circular
O nmero de permutaes circulares de n elementos dada por: Espao amostral o conjunto de todos os resultados poss-
p c
= (n 1)! veis para um experimento aleatrio. No caso da moeda temos
n
S = {cara, coroa}.

Evento qualquer subconjunto do espao amostral.


E X E R C C I O Evento complementar de A o evento A = S A:
01. (UNIFOR) Quantos so os anagramas da palavra PROVA Probabilidade chama-se probabilidade de um evento A S,
que terminam com a letra A?
ao nmero: p(A) = n(A)
a) 720 d) 24 n(S)
b) 120 e) 4
c) 60
Propriedades
0 p(A) 1
02. (UFC) Uma famlia com 5 pessoas possui um automvel de
p(A) + p(A) = 1
5 lugares. Sabendo que somente duas pessoas sabem dirigir, de
quantos modos podero se acomodar para uma viagem?
Probabilidade de uma reunio de dois eventos Sejam A e B
eventos contidos em S:
03. (UFC) Quantos so os nmeros de 5 algarismos distintos,
P(A B) = P(A) + P(B) P(A B)
todos eles mpares, nos quais os dois maiores algarismos esto
sempre juntos?
Probabilidade da interseo de eventos independentes Se-
jam A e B eventos independentes contidos em S:
04. Dois rapazes e quatro moas formam um a fila para serem
P(A B) = P(A) . P(B)
fotografados. Se deve ficar um rapaz em cada extremo da fila,
quantas disposies diferentes essa fila pode ter?

202 UECEVEST

Apostilas UECEVEST mod3.indb 202 06/02/2011 10:01:23


MATE M T I C A I I

a) 60 c) 40
E X E R C C I O b) 50 d) 30
01. (UFC) Onze clubes disputam um campeonato em que cada 09. (UECE) Cinco homens e uma mulher pretende utilizar um
clube joga uma s vez com cada um dos outros. O nmero de jogos banco de cinco lugares. De quantas maneiras diferentes podem
realizados neste campeonato : sentar-se, de modo que a mulher nunca fique em p?
a) 53 d) 56 a) 600 c) 2400
b) 54 e) 57 b) 1200 d) 2520
c) 55
10. (UECE) No sistema decimal de numerao, os nmeros in-
02. (UNIFOR) Um editor pode publicar apenas 4 livros de um teiros entre 100 e 999 que possuem algarismos diferentes consti-
grupo de 9. De quantas maneiras diferentes ele pode selecionar tuem em conjunto com n elementos. O valor de n :
esses quatro? a) 720 c) 576
a) 180 d) 56 b) 648 d) 202
b) 126 e) 24
c) 104 11. (UFC) Do conjunto D={2,3,4,5,6,7,8,9,10} escolhe-se, aleatoria-
mente, um subconjunto de dois elementos distintos. A probabilidade
03.(UNIFOR) As retas r e s so distintas e paralelas enter si. So de que os nmeros do conjunto escolhido sejam primos entre si :
dados 5 pontos distintos na reta r e 4 pontos distintos na reta s. a) 11/18 d) 7/9
Quantos so os tringulos determinados pelos pontos dados? b) 2/3 e) 5/6
c) 13/18
04. (UNIFOR) Cinco moas e sete rapazes candidataram-se para
estrelar um comercial de TV, mas apenas duas moas e trs ra- 12. (UECE) O nmero mximo de planos que podem ser deter-
pazes formaro a equipe. Quantas equipes distintas podero ser minado por 5 pontos no espao :
formadas com esses candidatos? a) 20 c) 12
a) 420 d) 120 b) 15 d) 10
b) 350 e) 36
c) 260 13. (UECE) De quantos modos podemos dividir 8 pessoas em 2
grupos, um com 5 e o outro com 3 pessoas?
05. (UNIFOR) Joo e Maria fazem parte de uma turma de 10 a) 80 c) 56
crianas, 6 das quais sero escolhidas para participar de uma pea b) 64 d) 48
a ser encenada em sua escola. Considerando todos os grupos que
podem ser escolhidos, em quantos deles Joo e Maria estariam 14. (UECE) Os casais Jos e Maria, Joo e Isabel e Pedro e Lui-
presentes? za, ao procurarem assentos na sala de exibio do Cine ORION
a) 3760 d) 70 encontraram dois lugares vizinhos em cinco filas diferentes. Sem
b) 1680 e) 28 separar os integrantes de cada casal, o nmero de formas distintas
c) 140 de tomarem assento naquele recinto :
a) 150 c) 480
06. (CESPE UNB) Um professor props dividir sua turma em b) 300 d) 950
7 grupos de alunos; os elementos de um dos grupos ficariam no
centro de uma circunferncia, e os demais grupos, posicionados 15. (UECE) O nmero de modos diferentes de escolher quatro ele-
em 6 locais bem determinados sobre a circunferncia, teriam a mentos (nmeros) distintos no conjunto {-6, -5, -4, -3, -2, -1, 1, 2,
incumbncia de questionar os elementos do grupo do centro a 3, 4, 5, 6} de tal forma que o produto destes nmeros seja positivo, :
respeito de um assunto pr agendado. A figura abaixo ilustra a a) 324 c) 225
posio dos 7 grupos. b) 255 d) 216

16. (UECE) Seja P o conjunto cujos elementos so os nmeros


inteiros positivos com cinco dgitos obtidos com as permutaes
dos algarismos 2, 3, 4, 8 e 9. Se dispomos os elementos de P em
ordem crescente, o nmero de ordem de 43928,:
a) 58 c) 59
b) 57 d) 60
Nesse caso, a quantidade de formas possveis e distintas de se or-
ganizar os grupos dos questionadores e questionados ser igual a: 17. (UECE) Dos 21 vereadores de uma Cmara Municipal, 12
a) 5040 c) 720 so homens e 9 so mulheres. O nmero de Comisses de vere-
b) 840 d) 120 adores, constitudas com 5 membros, de forma a manter-se sem-
pre 3 participantes de um sexo e 2 do outro, igual a:
07. (UECE) O Colgio Arraia organizou um torneio no qual cada a) 10.364 c) 12.436
participante enfrenta uma nica vez todos os demais. Se houve 780 b) 11.404 d) 13.464
disputas, quantos so os participantes do torneio?
a) 36 c) 40 18. (UECE) Um estudante tem que selecionar 5 disciplinas, entre
b) 38 d) 42 12 ofertadas para o prximo semestre, e uma delas tem que ser Ge-
ografia ou Histria, as quais esto includos entre as 12 ofertadas.
08. (UECE) Quantos nmeros mpares, cada um com trs alga- De quantas maneiras o estudante pode escolher estas disciplinas:
rismos, podem ser formados com os algarismos 2, 3, 4, 6 e 7, se a a) 330 c) 540
repetio de algarismos repetida? b) 462 d) 792

UECEVEST 203

Apostilas UECEVEST mod3.indb 203 06/02/2011 10:01:23


M AT E MTICA II

19. (UFC) Oito pessoas, sendo 5 homens e 3 mulheres, sero a) 23+25 c) 23 . 25


organizados em uma fila. A probabilidade das pessoas do mesmo b) 2724 d) 2724
sexo ficarem juntas :
1 5 28. (UECE) Participei de um sorteio de oito livros e quatro
a) d)
28 18 DVDs, todos distintos, e ganhei o dinheiro de escolher dentre
estes, trs livros e dois DVDs. O nmero de maneiras distintas
b) 1 e) 1 que eu posso fazer esta escolha :
18 38 a) 32 c) 336
c) 3 b) 242 d) 192
28
29. Um grupo de 3 mulheres e 3 homens dentre os quais Jos e
20. (UFC) Considere o espao amostral constitudo pelos nmeros Maria, que so casados entre si, devem formar uma fila indiana
de 3 algarismos distintos, formados pelos algarismos 2, 3, 4 e 5, em que no haja dois homens juntos. A quantidade de filas, em
assinale a opo em que consta a probabilidade de que ao escolher- que Jos Maria no esto separados por qualquer outra pessoa, :
mos um destes nmeros, aleatoriamente, este seja mltiplo de 3. a) 24 d) 120
a) 1/3 d) 2/3 b) 40 e) 720
b) 1/3 e) 3/4 c) 72
c) 1/2

21. (UFC) Assinale a alternativa na qual consta a quantidade de


nmeros inteiros formados por trs algarismos distintos, escolhi-
dos dentre 1, 3, 5, 7 e 9 e que so maiores que 200 e menores 30. (ENEM) Doze times se inscreveram em um torneio de fu-
que 800. tebol amador. O jogo de abertura do torneio foi escolhido da
a) 30 d) 48 seguinte forma: primeiro foram sorteados 4 times para compor o
b) 36 e) 54 Grupo A. Em seguida, entre os times do Grupo A, foram sortea-
c) 42 dos 2 times para realizar o jogo de abertura do torneio, sendo que
o primeiro deles jogaria em seu prprio campo, e o segundo seria
22. (UFC) A quantidade de nmeros inteiros, positivos e mpa- o time visitante. A quantidade total de escolhas possveis para o
res, formados por trs algarismos distintos, escolhidos dentre os Grupo A e a quantidade total de escolhas dos times do jogo de
algarismos 0, 1, 2, 3, 4, 5, 6, 7, 8 e 9, igual a: abertura podem ser calculadas atravs de
a) 320 d) 360 a) uma combinao e um arranjo, respectivamente.
b) 332 e) 384 b) um arranjo e uma combinao, respectivamente.
c) 348 c) um arranjo e uma permutao, respectivamente.
d) duas combinaes.
23. (UFC) O nmero de maneiras segundo as quais podemos e) dois arranjos.
dispor 3 homens e 3 mulheres em trs bancos fixos, de tal forma
que em cada banco fique um casal, sem levar em conta a posio 31. (ENEM) Um casal decidiu que vai ter 3 filhos. Contudo,
do casal no banco, : quer exatamente 2 filhos homens e decide que, se a probabilidade
a) 9 d) 32 fosse inferior a 50%, iria procurar uma clnica para fazer um tra-
b) 18 e) 36 tamento especfico para garantir que teria os dois filhos homens.
c) 24 Aps os clculos, o casal concluiu que a probabilidade de ter exa-
tamente 2 filhos homens
24. (UFC) O nmero mximo de ponto de interseo entre 10 a) 66,7%, assim ele no precisar fazer um tratamento.
circunferncia distintas : b) 50%, assim ele no precisar fazer um tratamento.
a) 100 d) 32 c) 7,5%, assim ele no precisar fazer um tratamento.
b) 90 e) 20 d) 25%, assim ele precisar procurar uma clnica para fazer um
c) 45 tratamento.
e) 37,5%, assim ele precisar procurar uma clnica para fazer
25. (UECE 2005.1) Com um grupo de 15 pessoas, do qual fazem um tratamento.
parte Lcia e Jos, o nmero de comisses distintas que se podem
formar com 5 membros, incluindo, necessariamente Lcia e Jos, : 32. (ENEM) Em um concurso de televiso apresentam-se ao
a) 3003 c) 455 participante, trs fichas voltadas para baixo, estando representada
b) 792 d) 286 em cada uma delas as letras T, V e E. As fichas encontram-se ali-
nhadas em uma ordem qualquer. O participante deve ordenar as
26. (UECE - 06.1) Bruno faz 1 (um) jogo na SENA, apostando fichas ao seu gosto, mantendo as letras voltadas para baixo, ten-
nos 6. (seis) nmeros 8, 18, 28, 30, 40 e 50; automaticamente, tando obter a siglaTVE. Ao desvir-las, para cada letra que esteja
Bruno tambm estar concorrendo quina (grupo de 5 nme- na posio correta ganhar um prmio de R$ 200,00. A proba-
ros), quadra (grupo de 4 nmeros) e o terno (grupo de 3 nme- bilidade de o participante no ganhar qualquer prmio igual a:
ros), a partir do grupo inicialmente apostado. Se n o nmero de a) 0 d) 1/2
quinas, q o nmero de quadras e P o nmero de ternos incluidos b) 1/3 e) 1/6
na aposta de Bruno, ento n + q + P igual a: c) 1/4
a) 12 c) 60
b) 41 d) 81 Texto para a questo 33.
A vida na rua como ela O Ministrio do Desenvolvimento So-
27. (UECE) Se um conjunto x possui 8 elementos, ento o n- cial e Combate Fome (MDS) realizou, em parceria com a ONU,
mero de subconjuntos de x que possuem 3 ou 5 elementos : uma pesquisa nacional sobre a populao que vive na rua, tendo

204 UECEVEST

Apostilas UECEVEST mod3.indb 204 06/02/2011 10:01:25


MATE M T I C A I I

sido ouvidas 31.922 pessoas em 71 cidades brasileiras. Nesse le- pelos efeitos das queimadas, o atendimento hospitalar no
vantamento, constatou-se que a maioria dessa populao sabe ler setor de pediatria seja reforado.
e escrever (74%), que apenas 15,1% vivem de esmolas e que,
entre os moradores de rua que ingressaram no ensino superior, 36. (ENEM) Dados do Instituto de Pesquisas Econmicas Apli-
0,7% se diplomou. Outros dados da pesquisa so apresentados cadas (IPEA) revelaram que no binio 2004/2005, nas rodovias
nos quadros abaixo federais, os atropelamentos com morte ocuparam o segundo lu-
gar no ranking de mortalidade por acidentes. A cada 34 atrope-
lamentos, ocorreram 10 mortes. Cerca de 4 mil atropelamentos/
ano, um a cada duas horas, aproximadamente
Disponvel em: http://www.ipea.gov.br. Acesso em: 6 jan. 2009.
De acordo com os dados, se for escolhido aleatoriamente para in-
vestigao, mais detalhada um dos atropelamentos ocorridos no
binio 2004/2005, a probabilidade de ter sido um atropelamento
sem morte
a) 2/17 d) 3/5
b) 5/17 e) 12/17
c) 2/5

G A B A R I T O
33. No universo pesquisado, considere que P seja o conjunto das 01. c 02. b 03. 70 04. b 05. d 06. b 07. c
pessoas que vivem na rua por motivos de alcoolismo/drogas e 08. b 09. a 10. b 11. a 12. d 13. c 14. c
Q seja o conjunto daquelas cujo motivo para viverem na rua a 15. b 16. c 17. d 18. c 19. a 20. c 21. b
decepo amorosa. Escolhendo-se ao acaso uma pessoa no grupo
pesquisado e supondo-se que seja igual a 40% a probabilidade de 22. a 23. e 24. b 25. d 26. b 27. b 28. c
que essa pessoa faa parte do conjunto P ou do conjunto Q, en- 29. b 30. a 31. e 32. b 33. a 34. a 35. e
to a probabilidade de que ela faa parte do conjunto interseo 36. e
de P e Q igual a
a) 12%. d) 36%.
b) 16%. e) 52%.
c) 20%.
nmEROS BInOmInAIS (UECE/EnEm)
34. (ENEM) A probabilidade de o concorrente ganhar exata-
mente o valor de R$ 400,00 igual a: Definies
a) 0 d) 2/3 Chamamos de nmero binomial de numerador n e denomi-
b) 1/3 e) 1/6 nador (ou classe) p ou simplesmente nmero binomial n sobre p
c) 1/2 todo nmero da forma:

35. (ENEM) A queima de cana aumenta a concentrao de di- com {n, p} N e p n.


xido de carbono e de material particulado na atmosfera, causa
alterao do clima e contribui para o aumento de doenas respi- Se : n p 0:
ratrias. A tabela abaixo apresenta nmeros relativos a pacientes
internados em um hospital no perodo da queima da cana.
n = numerador
p = classe ou denominador

Nmeros binomiais complementares.


Def.: so complementares se p + q = n.

prop.: se p + q = n, ento .
Escolhendo-se aleatoriamente um paciente internado nesse hos-
pital por problemas respiratrios causados pelas queimadas, a Nmeros binomiais consecutivos.
probabilidade de que ele seja uma criana igual a
a) 0,26, o que sugere a necessidade de implementao de Def.: so consecutivos (porque as suas classes
medidas que reforcem a ateno ao idoso internado com so nmeros naturais consecutivos e os numeradores so iguais).
problemas respiratrios.
b) 0,50, o que comprova ser de grau mdio a gravidade dos Relao de Stiel:
problemas respiratrios que atingem a populao nas regies
das queimadas.
c) 0,63, o que mostra que nenhum aspecto relativo sade Sendo {n, p, q} N, p n e q n, tem-se que:
infantil pode ser negligenciado.
d) 0,67, o que indica a necessidade de campanhas de
conscientizao que objetivem a eliminao das queimadas.
e) 0,75, o que sugere a necessidade de que, em reas atingidas

UECEVEST 205

Apostilas UECEVEST mod3.indb 205 06/02/2011 10:01:28


M AT E MTICA II

E X E R C C I O
Observe que cada nmero binomial est localizado na

01. Seja n um nmero natural tal que , ento: linha n e coluna p. Calculando cada binomial, podemos reescre-
ver o tringulo da seguinte forma:
a) n = 5 d) n = 2
b) n = 4 e) n.r.a.
c) n = 3

02. A equao

a) no admite solues
b) admite uma soluo entre 1 e 5
c) admite uma soluo entre 5 e 12
d) admite uma soluo entre 12 e 20
e) admite uma soluo maior que 20

NOTAS:
03. Calcule n sabendo que: O primeiro elemento de cada linha igual a 1, pois todo bino-
mial de denominador zero igual a 1.
O ltimo elemento de cada linha igual a 1, pois todo bino-
mial de numerador igual ao denominador igual a 1.
04. O smbolo , {n,p} N e p n representa um nmero
Propriedades do tringulo de Pascal
binomial. Os valores de p de modo que so: P1 Em qualquer linha, dois binomiais equidistantes dos
extremos so complementares e, portanto, iguais. Consideremos,
a) p = 4 d) p = 3 ou p = 5 como exemplo, a sexta linha (linha 5):
b) p = 4 ou p = 6 e) p = 5
c) p = 4 ou p = 5

05. Os nmeros n N, nesta ordem


esto em PA. Calcule n.
P2 A soma de dois binomiais consecutivos de uma mesma
linha igual ao binomial situado imediatamente abaixo do bino-
mial da direita. ( a relao de Stiel).
TRInGUlO DE PASCAl (UECE)
Introduo
Tambm conhecido como tringulo de Tartaglia, uma ta-
bela de nmeros binomiais, assim colocada:
((n+1) Coluna)
(1 Coluna)
(2 Coluna)
(3 Coluna)
(4 Coluna)

Assim:
(1 Linha) 1
1 1
(2 Linha)
1 2 1
(3 Linha) 1 3 3 1
(4 Linha) 1 4 6 4 1
1 5 10 10 5 1
(5 Linha) . . . . . .
. . . . . .
((n+1) Linha) P3 A soma de todos os binomiais da linha de numerador
n igual a 2n.

206 UECEVEST

Apostilas UECEVEST mod3.indb 206 06/02/2011 10:01:33


MATE M T I C A I I

Note que:


1 + 2 + 3 + 4 = 10

De fato, da observao do tringulo, podemos concluir gene-


ricamente, que:
1 + 3 + 6 = 10

Considerando uma diagonal genrica, escrevemos:


P4 A soma dos elementos de uma coluna, desde o primeiro
at um determinado elemento, igual ao binomial situado ime-
diatamente direita e abaixo do ltimo elemento considerado.
P6 Para n par:

P7 Para n mpar:

........................
Observando duas colunas do tringulo, temos que:
E X E R C C I O

1 + 2 + 3 + 4 = 10 01. A soma , vale:
e

a) c)


1 + 3 + 6 = 10 b) d)

Para uma coluna genrica podemos escrever:


02. (UFC) Se , calcule .

P5 A soma dos elementos de uma diagonal, desde o elemento 03. A sequncia 1, 8, x, y, 70, y, x, 8, 1 uma linha do tringulo
da primeira coluna at um determinado elemento, igual ao bino- de Pascal. Determine x e y.
mial situado imediatamente abaixo do ltimo elemento considerado:
04. Calcule o valor de x na equao abaixo:

05. (ENEM) A escrita Braile para cegos um sistema de smbolos


no qual cada carter um conjunto de 6 pontos dispostos em
forma retangular, dos quais pelo menos um se destaca em relao
aos demais. Por exemplo, a letra A representada por.
a) 12
b) 31
c) 36
d) 63
........................ e) 720

UECEVEST 207

Apostilas UECEVEST mod3.indb 207 06/02/2011 10:01:39


M AT E MTICA II

G A B A R I T O 06. O valor de m tal que = 729 :


01. c 02. 33 03. x=28 e y=56 04. 12 05. d
a) 14 b) 9 c) 6 d) 7 e) 8

07. (UECE) Se n = , ento o nmero

BInmIOS DE nEWTOn (UECE) binominal igual a:

Binmio de newton a) 20 b) 35 c) 48 d) 56
(x + a) = 1 ................................... linha 0 1
(x + a)1 = x + a .............................. linha 1 1 1 08. (UECE) O termo mdio (ou central) do desenvolvimento
(x + a)2 = x2 + 2ax + a2 .................. linha 2 1 2 1
(x + a)3 = x3 + 3ax2 + 3a2x + a3 ....... linha 3 1 3 3 1 de igual a:

(x + a)n = ? ................... linha n a) 252 b) 254 c) 256 d) 258

09. (UECE) No desenvolvimento , k o termo


Assim, desenvolvido, teremos o binmio de Newton (segun-
do as potncias decrescentes de x): independente de x. O valor de k2 3k + 1 :
a) 18 b) 19 c) 20 d) 21

10. (UECE) O coeficiente de x2.a3 no desenvolvimento de (2x + 3a)5 :


T1 T2 T3 a) 243 b) 720 c) 810 d) 1080

11. (UECE) O coeficiente de x2 no desenvolvimento


:
a) 32 c) 64
Tp+1 Tn+1 b) 60 d) 120
Ou, em forma de somatrio, o mesmo binmio fica:
12. (UECE 03.1) Se i = , ento o quarto termo do desenvol-
(x + a)n =
vimento de (1 + i)6 :
a) 15i b) 15i c) 20i d) 20i
IMPORTANTE: Frmula do termo geral:
13. (UECE) O termo mdio no desenvolvimento de :

a) 126 c) 252
b) 126x5 d) 252x5
E X E R C C I O
14. (UECE) Se os coeficientes do segundo, terceiro e quarto ter-
01. (UECE-97.2) O coeficiente de x2 no desenvolvimento de mo do desenvolvimento de (1 + x)n forma, nesta ordem, uma
: progresso aritmtica, ento o valor de n igual a:
a) 112 c) 168 a) 8 b) 7 c) 6 d) 5
b) 140 d) 224
15. (UFC) Sabendo-se que a soma dos coeficientes do desenvol-
02. O 4 termo do desenvolvimento de : vimento de (x + 1)p, 2048, determine o valor de p.

a) 19x4 d) 20x3 16. (UFC) Se ento o valor de m :


b) 20x4 e) 19x5
c) 21x5 a) 2 b) 4 c) 6 d) 8 e) 10
03. (UFC) Sabendo-se que a soma dos coeficientes dos termos 17. (UFC) O coeficiente de x no polinmio P(x)=(x 1).(x + 3)5 :
do desenvolvimento de (x + y)n 4096, determine o valor de n. a) 30 b) 50 c) 100 d) 120 e) 180

18. (UFC) Sendo n e r inteiros positivos e x 0, valor de r para


04. (UNIFOR) No desenvolvimento do binmio ,o
termo independente de x
o qual o 10 termo do desenvolvimento de indepen-
a) 24 b) 12 c) 8 d) 6 e) 4
de de x :
a) 16 b) 17 c) 18 d) 19 e) 20
05. (UFC-92.1) O valor da expresso: (1+sen2)5 5(1+sen2)4 +
10(1+sen2)3 10(1+sen2)2 + 5(1+sen2) 1 igual a:
a) (sen2)5 d) 0
b) (1 + sen2)5 1 e) (sen2)5 + 1. 19. (ITA) A respeito das combinaes an= e bn =
c) 1
temos que, para cada n = 1, 2, 3, ..., a diferena an bn igual a:

208 UECEVEST

Apostilas UECEVEST mod3.indb 208 06/02/2011 10:01:44


MATE M T I C A I I

a) d) G A B A R I T O
01. a 02. d 03. 12 04. a 05. a 06. c
b) e)
07. d 08. a 09. b 10. d 11. b 12. d
c) 13. c 14. b 15. 11 16. b 17. e 18. c
19. e 20. 27 21. 02 22. a 23. b 24. b
25. d 26. 216 27. a 28. a
20. Calcule a soma dos coeficientes numricos do desenvolvi-
mento (4x y)3.

21. Os nmeros reais x e y so tais que x y = 1 e


nOES DE ESTATSTICA (UECE/EnEm)
.
Introduo
Durante a campanha para a eleio presidencial no Brasil, a im-
Ache o produto P = xy.
prensa divulgava, semanalmente, pesquisas sobre a preferncia popu-
lar quanto aos candidatos. Um leigo poderia duvidar dos resultados
22. Qual o termo central de (x 3)6?
dessas pesquisas com o argumento: impossvel em apenas sete dias,
a) 540x3 d) 540x3
indagar a toda populao brasileira sua preferncia eleitoral.
b) 3240x 3
e) 540x4
A estatstica uma cincia que torna possvel concluso sobre uma
c) 3240x 3
populao, sem a necessidade de consultar todos os seus membros.
23. No desenvolvimento de (x + 3)6 , o nmero de termos com
coeficiente par : Universo estatstico ou populao estatstica
a) 2 b) 3 c) 4 d) 5 e) 6 Numa pesquisa, ou coleta de dados, chama-se universo esta-
tstico ou populao estatstica o conjunto de todos os elemen-
24. O coeficiente de x5 no desenvolvimento de (x2 4x + 4)4 : tos que podem oferecer dados pertinentes ao estudo em questo.
a) 12 d) 56 Exemplo 1
b) 448 e) 2888 O Ministrio da Fazenda deseja saber o preo mdio da cesta
c) 8945 bsica de alimentos no Brasil.
O universo estatstico ou populao estatstica , nesse caso,
25. O valor numrico do polinmio x4 4x3y + 6x2y2 4xy3 + y4 o conjunto de todas as casas comerciais que vendem produtos de
cesta bsica.
quando: igual a: Exemplo 2
feita uma pesquisa de opinio a respeito da preferncia so-
a) 2/5 c) 8/5 bre os vrios candidatos presidncia da Repblica.
b) 4/5 d) 16/5 O universo estatstico ou populao estatstica , nesse caso, o
conjunto de todos os eleitores brasileiros.
26. Calcule o valor numrico de .
Amostra
As vezes, o universo estatstico muito vasto ou impossvel
27. (UFC) O smbolo indica a combinao de n objeto k a k.
pesquisar todos os seus elementos. retirado desse universo um
subconjunto chamado amostra, e a pesquisa feita nessa amostra.
O valor de x y quando e
A escolha da amostra feita obedecendo-se a alguns critrios
tcnicos, para que as concluses obtidas retratem com fidelidade
igual a:
o universo estatstico.
a) 0 d) 25
Exemplo 3
b) 1 e) 125
No exemplo 1, o da cesta bsica, a escolha da amostra deve
c) 5
obedecer aos critrios:
a) so pesquisados, por exemplo, 30 supermercados por Estado,
28. (UECE) O quadro nmerico a seguir conhecido como o
sendo que 10 deles se localizam em bairros de classe A, outros
tringulo de Pascal.
10 em bairros de classe B e outros 10 em bairros de classe C;
1 linha 1 b) os preos so pesquisados num mesmo intervalo de tempo e
2 linha 1 1 o menor possvel.
3 linha 1 2 1 Podem, ainda, ser includos outros critrios para a escolha da
4 linha 1 3 3 1 amostra, sempre com o intuito de torna-la imparcial.
5 linha 1 4 6 4 1
Rol
6 linha 1 5 10 10 5 1
............................................. denominada rol uma sequncia: (a1, a2, a3 ... an ) dos dados
e assim sucessivamente numricos de uma amostra, tal que: I. ai ai+1 para todo i, 1 i n.
Ou ento: II.ai ai+1 para todo i, 1 i n.
Observando a lgica do quadro anterior, podemos concluir que
a soma dos segundo elemento da 2009 linha com o penltimo Amplitude de um rol
elemento da linha imediatamente anterior : Sejam b e a o maior e o menor valor de um rol, respectiva-
a) 4015 e) 4019 mente, numa unidade u. Chama-se amplitude do rol a direrena
b) 4017 d) 4021 b a, na unidade u.

UECEVEST 209

Apostilas UECEVEST mod3.indb 209 06/02/2011 10:01:48


M AT E MTICA II

Exemplo 4 Intervalos de classes Frequncia


As idades de 8 alunos, escolhidos aleatoriamente durante uma (estaturas em metros) (n de pessoas)
aula, so: 17, 19, 18, 20, 16, 21, 20 e 22 anos. Escrever esses 1,65 1,70 5
dados em rol e calcular a amplitude de cada rol.
Resoluo: 1,70 1,74 3
a) (16, 17, 18, 19, 20, 21, 22) 1,74 1,84 5
Amplitude do rol: 22 anos 16 anos = 6 anos 1,84 1,88 2
b) (22, 21, 20, 20, 19, 18, 17, 16)
Amplitude do rol: 22 anos 16 anos = 6 anos Amplitude de um intervalo de classe
Sejam, respectivamente, b e a o maior e o menor valor de
Distribuio de frequncia um intervalo de classe, numa unidade u. Chama-se amplitude do
Foi escolhida aleatoriamente uma amostra de 15 homens intervalo, na unidade u, a diferena b a.
adultos de uma certa regio do Brasil, e mediu-se a estatura de Exemplo 5
cada um. As medidas, em metros, foram as seguintes: 1,85; 1,72; A tabela seguinte mostra uma distribuio de frequncia dos
1,68; 1,83; 1,70; 1,69; 1,84; 1,80; 1,73; 1,76; 1,74; 1,65; 1,67; volumes, em mL, de uma amostra de garrafas de refrigerante.
1,88; 1,81. Intervalos de classes Frequncia
Escrevendo esses dados em rol e agrupando-os em sequncias, (volumes em mL) (n de pessoas)
como, por exemplo: (1,65; 1,67; 1,68; 1,69; 1,70); (1,72; 1,73; 280 285 58
1,74); (1,76; 1,80; 1,81; 1,83; 1,84); (1,85; 1,88), cada uma
285 292 112
das sequncias obtidas desse agrupamento chamada classe da
amostra, e o nmero de elementos de cada classe e denominado 292 300 202
frequncia da classe. 300 304 100
Ao se distribuir as estaturas em classes, indicando-se a frequ-
ncia de cada classe, fez-se o que se chama de distribuio de Determinar a amplitude de cada intervalo de classe.
frequncia da amostra. Resoluo:
A distribuio de frequncia pode ser apresentada sob forma A amplitude de:
de tabela: 280 285 285mL 280mL = 5 mL
285 292 292mL 285mL = 7 mL
Estaturas em metros Nmero de pessoas 292 300 300mL 292mL = 8 mL
1,65 1,70 5 300 304 304mL 300mL = 4mL
1,72 1,74 3
Exemplo 6
1,76 1,84 5
Um professor de Educao Fsica mediu as massas de 15 atle-
1,85 1,88 2 tas, obtendo os seguintes resultados, em kg: 70,6; 60,0; 69,5; 72,0;
Os smbolos 1,65 1,70, 1,72 1,74, 1,76 1,84 e 1,8 1,88 re- 75,0; 59,4; 71,0; 70,6; 73,0; 68,5; 71,5; 74,8; 66,0; 68,4; 79,4.
presentam intervalos reais fechados e so chamados de intervalos Dar uma distribuio de frequncia dessa amostra, com inter-
de classes. Por exemplo, o intervalo de classe 1,65 1,70 possui valos de amplitude 4,0kg.
todas as medidas, em metros, maiores ou iguais a 1,65 e menores Resoluo:
ou iguais a 1,70. Inicialmente, calcula-se a diferena entre o maior e o menor
valor da amostra, isto : 79,4kg 59,4kg = 20,0kg
Observaes Tal resultado a amplitude de um rol formado com esses
1) Os extremos dos intervalos de classe no precisam pertencer dados. A seguir, divide-se a amplitude do rol pela amplitude de
amostra. Por exemplo, a distribuio de frequncia anterior cada intervalo de classe, ou seja, 20,0kg : 4,0kg = 5, obtendo-se
pode ser apresentada sob a forma: assim o nmero de intervalos de classes.
Intervalos de classes Frequncia Assim, a distribuio de frequncia :
(estaturas em metros) (n de pessoas) Intervalos de classes Frequncia
1,64 1,71 5 (massas em kg) (n de atletas)
1,72 1,75 3 59,4 63,4 2
1,76 1,85 5 63,4 67,4 1
1,85 1,90 2 67,4 71,4 6
71,4 75,4 5
Note-se que os valores: 1,71; 1,75 e 1,90 so extremos de
intervalos de classes e no so elementos da amostra. 75,4 79,4 1

2) Os intervalos de classes podem ser abertos, isto , no Classe unitria


possuir um de seus extremos. Em certas distribuies de frequncia, conveniente conside-
Por exemplo, o smbolo: rar cada classe como um nico nmero real. Esse tipo de classe
1,65 1,70 representa valores maiores ou iguais a 1,65 e chamado classe unitria.
menores que 1,70. Exemplo 7
1,70 1,74 representa valores maiores que 1,70 e menores ou Numa prova de Matemtica, o aproveitamento dos alunos
iguais a 1,74. descrito pela tabela:
1,70 1,76 representa valores maiores que 1,70 e menores Intervalos de classes Frequncia
que 1,76. (notas) (n de alunos)
2,5 3
A distribuio de frequncias anterior pode ser apresentada 4,0 5
sob a forma: 5,0 8

210 UECEVEST

Apostilas UECEVEST mod3.indb 210 06/02/2011 10:01:49


MATE M T I C A I I

5,5 9 4 40 10%
6,5 8 5 80 20%
7,0 6 6 20 5%
8,0 5 Total 400 100%
10,0 2 Esses resultados podem ser apresentados graficamente.
Note-se que as classes dessa distribuio, 2,5; 4,0; 5,0; 5,5;
6,5; 7,0; 8,0; e 10,0, so unitrias. Grfico de barras verticais
As frequncias so dispostas num eixo vertical.
Frequncia relativa percentual (F%)
Numa distribuio por frequncia, chama-se frequncia re-
lativa percentual de uma classe o nmero:
F
F% = .100%
Ft

Onde F a frequncia da classe considerada e Ft a frequn-


cia total, isto , Ft a soma das frequncias de todas as classes da
distribuio.
Exemplo 8
Uma amostra de alunos de um certo colgio apresentou as
seguintes idades: 25; 17; 20; 19; 18; 16; 17; 23; 22; 18; 17; 21;
21; 16; 17; 19; 18; 19; 20 e 27 anos.
Construir uma tabela com a distribuio de frequncia, com Grfico de barras horizontais
classes unitrias, e calcular F, Ft e F%. As frequncias so dispostas num eixo horizontal.
Resoluo:
Classes (idades) F F%
16 2 10%
17 4 20%
18 3 15%
19 3 15%
20 2 10%
21 2 10%
22 1 5%
23 1 5%
25 1 5%
27 1 5%

Ft = 20 Grfico por setores


Divide-se um crculo em setores, com medidas dos arcos di-
Anlise da distribuio de frequncia retamente proporcionais s frequncias. Dividindo-se 360 em
A fase mais importante de um levantamento estatstico a partes diretamente proporcionais s frequncias 80, 60, 120, 40,
anlise de sua distribuio de frequncia, pois percebe-se a partir 80 e 20, obtm-se, respectivamente, 72, 54, 108, 36, 72 e
da a tendncia do universo estatstico. 18. Assim, o grfico por setores :
Para uma boa interpretao desses resultados, convm estudas
alguns conceitos que levaro s concluses mais corretas possveis
sobre o universo estatstico.

Representaes grficas
Os dados colhidos numa amostra podem ser representados
graficamente. Os grficos dividem-se em duas categorias: os de
informao e os de anlise.
Neste curso, sero estudados apenas os grficos de informao,
que visam a dar ideias apenas quantitativas dos dados colhidos.
Exemplo 9
Um dado foi lanado 400 vezes, e em cada lanamento foi
registrado o nmero de faces voltadas para cima. A tabela a seguir
mostra a frequncia de cada face e a porcentagem correspondente.
Frequncia relativa
Classes (face) Frequncia
percentual
1 80 20%
medidas de tendncia central
2 60 15%
Em certas distribuies de frequncia, os valores tendem a se
3 120 30% centralizar, isto , os valores intermedirios da varivel apresen-

UECEVEST 211

Apostilas UECEVEST mod3.indb 211 06/02/2011 10:01:50


tam frequncias maiores que os valores extremos. Chama-se a isso Definio
tendncia central de uma distribuio de frequncia. A mdia aritmtica ponderada dos nmeros x1, x2, x3, ... xn,
Essa tendncia pode ser medida numericamente pelas me- respectivamente, o nmero:
didas de tendncia central ou medidas de posio, que so a
mdia aritmtica, a mediana e a moda. x k + x 2 k 2 + x 3 k 3 + ... + x n k n
x= 1 1
k1 + k 2 + k 3 + ... + k n
Mdia aritmtica
Seja o conjunto A = {x1, x2, x3, ..., xn}. Exemplo 11
Chama-se mdia aritmtica dos valores x1, x2, x3, ...,xn o n- Uma empresa distribuiu um prmio para 5 funcionrios. O
mero x tal que: valor do prmio para cada um foi pago da seguinte maneira: 10%
x 1 + x 2 + x 3 + ... + x n do salrio atual para cada ano trabalhado na empresa. Os valores
x= dos salrios desses funcionrios so: R$ 8.600,00; R$ 7.000,00;
n
R$ 5.400,00; R$ 9.000,00 e R$ 6.000,00, e seus tempos de casa
so: 8, 12, 10, 3 e 7 anos, respectivamente. Qual o valor mdio
Observao dos prmios desses funcionrios?
Chama-se desvio ou afastamento de um elemento xi do con- Resoluo:
junto (x1, x2, x3, ..., xn), em relao medida aritmtica x, o n- O valor procurado a mdia aritmtica ponderada dos valo-
mero d tal que: res R$ 860,00; R$ 700,00; R$ 540,00; R$ 900,00 e R$ 600,00,
d = xi x com pesos respectivamente iguais a: 8, 12, 10, 3 e 7.
Isto :
860.8 + 700.12 + 540.10 + 900.3 + 600.7
Exemplo 10 x=
8 + 12 + 10 + 3 + 7
Dada a distribuio de frequncia a seguir, calcular a mdia
aritmtica das notas dessa prova. x = R$ 689,50
Classes (notas da prova Frequncia Resposta: R$ 689,50
final de Matemtica) (n de alunos)
3,0 6 Mediana (Md)
4,0 7 Sejam os valores escritos em rol:
5,0 6 x1, x2, x3, ...,xn
6,0 11 1) Sendo n mpar, chama-se mediana o termo xi tal que o
8,0 7 nmero de termos da sequncia que precedem xi igual ao
10,0 3 nmero de termos que o sucedem, isto , xi o termo mdio
da sequncia (xn) em rol.
Resoluo: 2) Sendo n par, chama-se mediana o valor obtido pela mdia
x = 6 .3 + 7 .4 + 6 .5 + 11 .6 + 7 .8 + 3 .10 aritmtica entre os termos xj e xj +1, tais que o nmero de
40 termos que precedem xj igual ao nmero de termos que
sucedem xj + 1, isto , a mediana a mdia aritmtica entre os
Assim, termos centrais da sequncia (xn) em rol.
x = 5,7
Exemplo 12
Os desvios das notas 3,0; 4,0; 5,0; 6,0; 8,0 e 10,0 so, res- Determinar a mediana do conjunto de dados:
pectivamente: {12, 3, 7, 10, 21,18, 23}
d1 = 3,0 5,7 = - 2,7 Resoluo:
d2 = 4,0 5,7 = - 1,7 Escrevendo os elementos do conjunto em rol, tem-se: (3, 7,
d3 = 5,0 5,7 = - 0,7 10, 12, 18, 21, 23). A mediana o termo mdio desse rol.
d4 = 6,0 5,7 = - 0,3 Logo: Md = 12
d5 = 8,0 5,7 = - 2,3 Resposta: Md = 12
d6 = 10,0 5,7 = - 4,3
Exemplo 13
Mdia aritmtica ponderada Determinar a mediana do conjunto de dados:
As provas bimestrais de certo colgio tm pesos diferentes. Os {10, 12, 3, 7, 18, 23, 21, 25}
dois primeiros bimestres tm peso 1, o terceiro tem peso 2 e o Resoluo:
quarto bimestre tem peso 3. Escrevendo os elementos do conjunto em rol, tem-se: (3, 7,
Em termos de mdia aritmtica, isso significa que, se as notas 10, 12, 18, 21, 23, 25). A mediana a mdia aritmtica entre os
de matemtica de certo aluno nos quatro bimestres forem 6, 5, dois termos centrais do rol.
7 e 8, respectivamente, a mdia aritmtica dever ser calculada 12 + 18
entre os nmeros 6, 5, 7, 7, 8, 8, 8, isto : Logo: Md = = 15
2
6 + 5 + 7 .2 + 8 .3 Resposta: Md = 15
x= =7
7
Moda (Mo)
Essa mdia chamada mdia aritmtica ponderada dos n- Num conjunto de nmeros: x1, x2, x3, ..., xn, chama-se moda
meros 6, 5, 7 e 8 com fatores de ponderao (pesos) 1, 1, 2 e 3, aquele valor que ocorre com maior frequncia.
respectivamente.
Observao:
A moda pode no existir e, se existir, pode no ser nica.

Apostilas UECEVEST mod3.indb 212 06/02/2011 10:01:53


MATE M T I C A I I

Exemplo 14 Resoluo:
O conjunto de dados 3, 3, 8, 8, 8, 6, 9, 31 tem moda igual a a) A mdia de pontos por jogo :
8, isto , Mo = 8
x = 22 + 18 + 13 + 24 + 26 + 20 + 19 + 18 x = 20
Exemplo 15 8
O conjunto de dados 1, 2, 9, 6, 3, 5 no tem moda. b) A varincia de pontos por jogo :
Exemplo 16 (22 20)2 + (18 20)2 + (13 20)2 + (24 20)2 + (26 20)2 + (20 20)2 + (19 20)2 + (18 20)2
O conjunto de dados 1, 5, 5, 5, 6, 7, 8, 8, 8 possui duas mo- 2 =
8
das, 5 e 8, e chamado bimodal.
2 = 14,25
medidas de disperso Exemplo 18
Duas distribuies de frequncia com medidas de tendncia Outro jogador B, da mesma equipe do jogador do exemplo
central semelhantes podem apresentar caractersticas diversas. anterior, apresentou nos mesmos jogos, o desempenho mostrado
Necessita-se de outros ndices numricos que informem sobre o pela tabela abaixo:
grau de disperso ou variao dos dados em torno da mdia ou
de qualquer outro valor de concentrao. Esses ndices so cha- Jogo n de pontos
mados medidas de disperso. 1 30
2 10
Varincia 3 20
Define-se a densidade demogrfica de uma regio como a ra- 4 8
zo de seu nmero de habitantes para sua rea. 5 6
Segundo dados da ONU, de 1990, a Europa o continente
mais povoado do mundo, com 66 hab/km2. Porm existem regi- 6 42
es europeias com densidades demogrficas muito maiores que 7 10
66 hab/km2 por exemplo, a Alemanha, com 201 hab/km2 en- 8 34
quanto outras tm densidades demogrficas muito menores que Resoluo:
66 hab/km2, como as regies montanhosas dos Crpatos e Blcs, a) A mdia de pontos por jogo :
onde h menos de 10 hab/km2. 30 + 10 + 20 + 8 + 6 + 42 + 10 + 34
Ora, quando se diz que a densidade demogrfica da Europa x = x = 20
8
de 66 hab/km2, deve-se entender tal medida como uma mdia
aritmtica e, portanto, uma medida de tendncia central, que b) A varincia :
no caracteriza as discrepncias entre as densidades demogrficas (30 20)2 + (10 20)2 + (20 20)2 + (8 20)2 + (6 20)2 + (42 20)2 + (10 20)2 + (34 20)2
2 =
das vrias regies europeias. 8
2 = 165
H um ndice que mede a disperso dos elementos de um
conjunto de nmeros em relao sua mdia aritmtica, e que Comparando-se os exemplos 17 e 18, percebe-se que a mdia
chamado de varincia. Esse ndice assim definido: dos jogadores a mesma, ou seja, 20 pontos por jogo; porm a
Seja o conjunto de nmeros x1, x2, x3,...,xn, tal que x a sua varincia do jogador A(14,25), menor que a varincia do joga-
mdia aritmtica. Chama-se varincia desse conjunto, e indica- dor B(125). Isso quer dizer que o nmero de pontos do jogador
se por 2, o nmero: A, em cada jogo, esteve mais prximo da mdia 20 do que o
nmero de pontos do jogador B.
(x )
n
2
i x Concluso:
2 = i =1 Se dois conjuntos A e B tm varincia A2 e B2, com A< B,
n isso significa que os elementos de A se dispersaram menos em
isto : relao mdia aritmtica do que os elementos de B.

2
=
(x1 ) (
2
) ( 2
x + x 2 x + x 3 x + ... + x n x) 2
( ) 2 Desvio padro
Como vimos no item anterior, a varincia um ndice que
n mede a disperso dos elementos de uma amostra de medidas em
relao sua mdia aritmtica. Embora sendo um ndice de muita
Exemplo 17 utilidade a varincia pode apresentar dificuldades quanto sua in-
Em oito jogos, o jogador A, de bola-ao-cesto, apresentou o terpretao, pois a unidade em que se apresentam os elementos da
seguinte desempenho, descrito na tabela abaixo: amostra. Por exemplo, se os elementos da amostra se apresentam
na unidade L (litro), ento a varincia se apresenta na unidade
Jogo n de pontos L2 (litro ao quadrado). Como interpretar essa unidade? Pode-se
1 22 contornar essa dificuldade definindo-se um novo ndice: o desvio
2 18 padro que nada mais do que a raiz quadrada da varincia.
3 13
Definio
4 24
5 26 Seja o conjunto de nmeros x1, x2, x3,...,xn, tal que x a sua
6 20 mdia aritmtica. Chama-se desvio padro desse conjunto, e
7 19 indica-se por , o nmero:

(x )
n
2
8 18 x
i
a) Qual a mdia de pontos por jogo? = i =1

b) Qual a varincia do conjunto de pontos? n

UECEVEST 213

Apostilas UECEVEST mod3.indb 213 06/02/2011 10:01:56


M AT E MTICA II

isto :

=
(x 1 ) (
2
) (
2
)
x + x 2 x + x 3 x + ... + x n x
2
( )2
01. (ENEM) Depois de jogar um dado em forma de cubo e de
n faces numeradas de 1 a 6, por 10 vezes consecutivas, e anotar o
nmero obtido em cada jogada, construiu-se a seguinte tabela de
Exemplo 19 distribuio de frequncias.
As estaturas dos jogadores de uma equipe de basquetebol so:
2,00m; 1,95m; 2,10m; 1,90m e 2,05m. Calcular:
a) A estatura mdia desses jogadores.
b) O desvio padro desse conjunto de estaturas.
Resoluo:
a) Sendo x a estatura mdia, temos:
2,00 + 1,95 + 2,10 + 1,90 + 2,05
x= x = 2,00m
5 A mdia, mediana e moda dessa distribuio de frequncias so,
respectivamente
b) Sendo o desvio padro, tem-se: a) 3, 2 e 1 d) 5, 4 e 2
b) 3, 3 e 1 e) 6, 2 e 4
(2,00 2,00)2 + (1,95 2,00)2 + (2,10 2,00)2 + (1,90 2,00)2 + (2,05 + 2,00)2 c) 3, 4 e 2
=
5
02. (ENEM) Suponha que a etapa final de uma gincana escolar
consista em um desafio de conhecimentos. Cada equipe escolhe-
= 0,005m 0,07m ria 10 alunos para realizar uma prova objetiva, e a pontuao da
equipe seria dada pela mediana das notas obtidas pelos alunos. As
OBSERVAO: provas valiam, no mximo, 10 pontos cada. Ao final, a vencedora
Do mesmo modo que a varincia, o desvio padro tambm foi a equipe mega, com 7,8 pontos, seguida pela equipe Delta,
mede a disperso dos elementos da amostra em relao mdia com 7,6 pontos. Um dos alunos da equipe Gama, a qual ficou na
aritmtica. Assim, se duas amostras A e B tm desvios padro A terceira e ltima colocao, no pde comparecer, tendo recebido
e B, respectivamente, tal que A < B, ento os elementos de A nota zero na prova. As notas obtidas pelos 10 alunos da equipe
se dispersam menos, em relao mdia aritmtica, do que os Gama foram 10; 6,5; 8; 10; 7; 6,5; 7; 8; 6; 0. Se o aluno da equi-
elementos de B. pe Gama que faltou tivesse comparecido, essa equipe
a) teria a pontuao igual a 6,5 se ele obtivesse nota 0.
Exemplo 20 b) seria a vencedora se ele obtivesse nota 10.
A tabela mostra os desempenhos de dois alunos A e B nas c) seria a segunda colocada se ele obtivesse nota 8.
provas bimestrais de matemtica durante um certo ano: d) permaneceria na terceira posio, independentemente da
nota obtida pelo aluno.
Aluno Notas e) empataria com a equipe mega na primeira colocao se o
A 6 4 5 5 aluno obtivesse nota 9.
B 6 6 4 4
03. (ENEM)Em sete de abril de 2004, um jornal publicou o
Qual dos dois alunos teve desempenho mais regular durante ranking de desmatamento, conforme grfico, da chamada Ama-
esse ano? znia Legal, integrada por nove estados.
Resoluo:
Sendo x A e x B as notas mdias de A e B, respectivamente,
tem-se:
6+ 4+5+5
xA = =5
4
6+6+4+4
xB = =5
4

Os dois alunos tm a mesma mdia de notas. Para saber qual


deles teve o desempenho mais regular, basta comparar os desvios
padro de seus conjuntos de notas.
Sendo A e B os desvios padro de A e B respectivamente,
temos: Considerando-se que at 2009 o desmatamento cresceu 10,5%
em relao aos dados de 2004, o desmatamento mdio por estado
A =
(6 5)2 + (4 5)2 + (5 5)2 + (5 5)2 =
1
= 0,5 em 2009 est entre
4 2 a) 100 km e 900 km. d) 3300 km e 4000 km.
b) 1000 km e 2700 km. e) 4100 km e 5800 km.

B =
(6 5)2 + (6 5)2 + (4 5)2 + (4 5)2 =1
c) 2800 km e 3200 km.
4 04. (ENEM) O grfico apresenta a quantidade de gols marcados
Como A < B, conclui-se que o aluno A teve desempenho pelos artilheiros das Copas do Mundo desde a Copa de 1930 at
mais regular. a de 2006.

214 UECEVEST

Apostilas UECEVEST mod3.indb 214 06/02/2011 10:01:59


MATE M T I C A I I

c) superior a 400 milhes de dlares, mas infeiror a 500


milhes de dlares.
d) superior a 500 milhes de dlares, mas inferior a 600
milhes de dlares.
e) superior a 600 milhes de dlares.

07. (ENEM) Na tabela, so apresentados dados da cotao mensal


do ovo extra branco vendido no atacado, em Braslia, em reais, por
caixa de 30 dzias de ovos, em alguns meses dos anos 2007 e 2008.

Disponvel em: http://www.suapesquisa.com. Acesso em: 23 abr. 2010 (adaptado).

A partir dos dados apresentados, qual a mediana das quantidades


de gols marcados pelos artilheiros das Copas do Mundo?
a) 6 gols d) 7,3 gols
b) 6,5 gols e) 8,5 gols
c) 7gols De acordo com esses dados, o valor da mediana das cotaes
mensais do ovo extra branco nesse perodo era igual a
05. (ENEM) O quadro seguinte mostra o desempenho de um a) R$73,10. d) R$83,00.
time de futebol no ltimo campeonato. A coluna da esquerda b) R$81,50. e) R$85,30.
mostra o nmero de gols marcados e a coluna da direita informa c) R$82,00.
em quantos jogos o time marcou aquele nmero de gols.

G A B A R I T O
01. b 02. d 03. c 04. b 05. e 06. d 07. d

REFERnCIAS BIBlIOGRFICAS
MEC- Parmetros curriculares Nacionais Ensino Mdio
PCNEM, Braslia, Ministrio da Educao 1999.
Se X, Y e Z so, respectivamente, a mdia, a mediana e a moda
desta distribuio, ento GIOVANN. J. Ruy, Bonjorno, J. Roberto Matemtica, So
a) X = Y < Z. d) Z < X < Y. Paulo, FTD, 2000.
b) Z < X = Y. e) Z < Y < X.
c) Y < Z < X. DANTE, Luiz Roberto Didatico da Resoluo de
Problemas de Matemtica, So Paulo Atica, 1991.
06. (ENEM) Brasil e Frana tm relaes comerciais h mais de
200 anos. Enquanto a Frana a 5- nao mais rica do planeta,
o Brasil a 10-, e ambas se destacam na economia mundial. No
entanto, devido a uma srie de restries, o comrcio entre esses
dois pases ainda no adequadamente explorado, como mostra
a tabela seguinte, referente ao perodo 2003-2007.

Os dados da tabela mostram que, no perodo considerado, os va-


lores mdios dos investimentos da Frana no Brasil foram maio-
res que os investimentos do Brasil na Frana em um valor
a) inferior a 300 milhes de dlares.
b) superior a 300 milhes de dlares, mas inferior a 400
milhes de dlares.

UECEVEST 215

Apostilas UECEVEST mod3.indb 215 06/02/2011 10:02:01


Apostilas UECEVEST mod3.indb 216 06/02/2011 10:02:01
P R - V E S T I B U l A R

FSICA I

Apostilas UECEVEST mod3.indb 217 06/02/2011 10:02:09


Caro(a) Aluno(a),

Para facilitar o acompanhamento de tais contedos, abaixo esto indicadas as nomencla-


turas utilizadas pela UECE e pelo ENEM:

UECE ENEM
Esttica dos slidos e equilbrio dos corpos O movimento, o equilbrio e a descoberta
extensos. de Leis Fsicas.
Conservao da energia Energia, trabalho e potncia.
Conservao da quantidade de movimento O movimento, o equilbrio e a descoberta
de Leis Fsicas.

Apostilas UECEVEST mod3.indb 218 06/02/2011 10:02:09


FSICA I

ESTTICA DOS SlIDOS E EQUIlBRIO Y(m) 1,0m


DOS CORPOS EXTEnSOS (UECE/EnEm)
1,0m
P1
Centro de massa
Chama-se centro de massa de um sistema fsico, o ponto
onde se admite concentrada, para efeito de clculo, toda a massa P2
do corpo. P3
0 P4 X(m)

As coordenadas X e Y do centro de massa so:


Cm Cm 1,0 . 0 + 2,0 . 2,0 + 3,0 . 4,0 + 4,0 . 1,0 20
X= = = 2,0m
1,0 + 2,0 + 3,0 + 4,0 10
1,0 . 4,0 + 2,0 . 2,0 + 3,0 . 1,0 + 4,0 . 0 11
Y= = = 1,1m
Se o corpo rgido for constitudo de um material homogneo, 1,0 + 2,0 + 3,0 + 4,0 10
o centro de massa conincidir com o centro geomtrico.
Entretanto, se o corpo no for constitudo de um material
Velocidade do Centro de Massa de um Sistema de Partculas:
homogneo, o centro de massa ficar deslocado para regio em
que houver maior concentrao de massa.
Pb Al P1 P3
Pn V cm
Cm
Cm
Cm P4
P2
1 1
Ppb>Pal Sejam um sistema de partculas P1, P2, P3, ..., Pn, com massas
respectivamente iguais a m1, m2, m3, ...,mn.
O centro de massa nem sempre um ponto pertencente ao
A velocidade do centro de massa dada pela seguinte expres-
corpo, por exemplo, num anel circular homogneo e de espessura
so vetorial:
constante.
m1 V1 + m 2 V2 + ... m n V n
V cm =
m1 + m 2 + ... m n
Cm
Se m1v1, m2v2, ..., mnvn, correspondem s respectivas quanti-
dades de movimento:
Posio do Centro de massa de um Sistema de Partculas Q
V cm = total
m total
Y
Y1 P1
Esttica dos Slidos
Y2 P2

Y3 P3

0 X1 X3 X2 X a
Sejam X e Y ou Xcm e Ycm, a abscissa e a coordenada do
Joo Teimoso
centro de massa do sistema Oxy.
O Centro de massa desse sistema de partculas dado pelas
Equilbrio esttico: A condio para um ponto material es-
mdias ponderadas:
tar em equilbrio esttico em relao a um referencial que as
m1 x 1 + m 2 x 2 + m 3 x 3 + ... + m n x n resultantes das foras que nele atuam seja nula.
X=
m1 + m 2 + m 3 + ... + m n Equilbrio de translao: Um corpo est em equilbrio de
translao quando seu centro de massa est em repouso ou em
m1 y 1 + m 2 y 2 + m 3 y3 + ... + m n y n MRU em relao a um determinado referencial, ou ainda, a re-
Y= sultante das foras externas atuante seja zero.
m1 + m 2 + m 3 + ... + m n

Exemplo: Quatro partculas P1, P2, P3 e P4, de massas respectiva- F ext = m.a cm a cm = 0 F ext =0
mente iguais a 1,0Kg, 2,0Kg , 3,0Kg e 4,0Kg encontram-se so-
Esttica de rotao: A condio de equilbrio de rotao de
bre um mesmo plano e posicionadas em relao a um referencial
um corpo extenso sob a ao de um conjunto de foras coplana-
Oxy, conforme a figura:
res que a soma algbrica dos momentos/torques em relao a

UECEVEST 219

Apostilas UECEVEST mod3.indb 219 06/02/2011 10:02:12


FSICA I

qualquer eixo perpendicular seja nula: Um ponto material est em equilbrio se a


=0 soma das foras que agem sobre ele nula.
Essa condio tambm pode ser expressa considerando que
a soma (em mdulo) de todos os torques horrios seja igual F = 0 R = 0
soma de todos os torques anti-horrios:

=
h ah
Mtodo da Linha Poligonal
Se um ponto material se encontra em equilbrio, a linha po-
ligonal das foras que agem sobre ele fechada.
Concluso: Para que um corpo permanea totalmente em
equilbrio:
F4
Fext = 0 F1 F1
F2
F3
= 0
F4
Teorema das Trs Foras F2 F3
Se um corpo estiver em equilbrio sob a ao exclusiva de
trs foras, estas devero ser, necessariamente, concorrentes
num nico ponto ou paralelas: Um caso especial
F3 No caso especfico de equilbrio de um ponto material sob a
F1 F2 ao de trs foras, a linha poligonal determina o tringulo.

P3 P1 P2 F1 B
F3 F1

P1 P2 P3
A
F2
C
F1 F2 F3
F2

Centro de Gravidade de um Corpo


Denominamos Centro de Gravidade (CG) o ponto onde
atua a fora peso de um corpo. Em campo gravitacional uni- Como as trs foras representam os lados de um tringulo, as
forme, o centro de gravidade coincide com o centro de massa. relaes entre as suas intensidades obedecem s propriedades dos
tringulos. Aplicando a lei dos senos, temos:
CG CG
F1 F F
= 2 = 3
sen sen sen
P P ou
Barra Placa
F1 F F
= 2 = 3
O centro de gravidade de uma placa triangular homognea senA senB senC
(tringulo escaleno), est no ponto de encontro de suas medianas
(baricentro). O baricentro divide as medianas na proporo 2:1.
Mtodo dos Componentes Vetoriais
CG Consideraremos um ponto material em equilbrio sob a ao
a de trs foras F1, F2 e F3.
2a

Equilbrio esttico e dinmico F2 F1


Um automvel parado em uma estrada encontra-se em equi-
lbrio esttico, e um automvel em movimento, com veloci-
dade vetorial constante, em uma pista horizontal, encontra-se
em equilbrio dinmico. Em qualquer um deles, as foras esto
equilibradas, o que significa dizer que a fora resultante nula.
Iniciaremos com as situaes nas quais os corpos podem ser
representados por um nico ponto. Esse ponto o centro de
massa do corpo e nele podemos imaginar que esteja concentra-
da toda a massa do corpo. Nessas condies, esse ponto recebe
o nome de ponto material. F3

220 UECEVEST

Apostilas UECEVEST mod3.indb 220 06/02/2011 10:02:15


FSICA I

Para estabelecer as relaes entre as intensidades dessas trs


foras, utilizando o mtodo dos componentes vetoriais, devemos F
inicialmente obter os componentes vetoriais de cada fora nos
eixos retangulares x e y: A +
F1x = F1 . cos F2x = F2 . cos F3x = 0
F1y = F1 . sen F2y = F2 . sen F3y = F3 O polo

F 1y F1 d
F2 Linha de ao de F
F 2y
O momento da fora F em relao ao ponto O, ou a tendn-

cia de rotao que a fora F produz na barra em relao a esse
F 2x F 1x ponto, dado por:

F3 M=F.d

Nessa expresso, F a intensidade da fora e d a distncia da


Se um ponto material est em equilbrio, ento, obrigatoria- linha de ao da fora ao eixo de rotao. A distncia d recebe o
mente, devemos ter equilbrio, tanto na direo horizontal (eixo nome de brao da fora.
x), como na direo vertical (eixo y). No caso em que a fora no perpendicular ao segmento de
F x = 0 F1 . cos F2 . cos = 0 reta que une o ponto de aplicao da fora com o plo, podemos
calcular o momento de duas maneiras:
atravs dos componentes vetoriais da fora
F y = 0 F1 . sen + F2 . sen F3 = 0
Fy F
+
Versores A B
Esse mtodo somente apresenta interesse quando as foras Linha de
so delimitadas por um quadriculado em escala. Nesse caso, ao de F x Fx O polo
possvel visualizar diretamente os componentes e escreve-los em
funo de padres vetoriais. a
Como todo padro, os vetores de base, chamados de versores, Linha de ao de F y
so unitrios. Qualquer vetor pode ser escrito como a soma de ou-
tros dois, e vamos apenas considerar vetores que estejam no mesmo Fx = F. cos
plano. Para tanto, basta definir os dois versores ortogonais e .
Fy = F.sen

M = M(Fx ) + M(Fy )
j M = Fa . sen
M = 0 + (F . sen ) a
F1
atravs do brao de fora
F3
C Linha de
ao de F
F
d
F2 +
A B

O polo
a
Na figura, temos:
^ ^ No tringulo ABC, obtemos:
F1 = 4 i + 4 j d
sen = d = a sen
^
F = F
^
F2 = 3 i 5 j 1 + F 2 + F 3 = 0 (equilbrio) a
^ ^ E, o momento da fora dado por:
F3 = 7 i + 2 j
M = Fd M = Fa . sen
OBS.:
momento de uma Fora O momento de uma fora em relao a um ponto uma
Consideraremos uma fora de intensidade F, aplicada no grandeza vetorial, possuindo mdulo, direo e sentido. Mas,
ponto A de uma barra que pode girar livremente em torno do como utilizaremos somente foras coplanares, basta adotar uma
ponto O chamado de plo. conveno de sinais para os sentidos dos momentos.

UECEVEST 221

Apostilas UECEVEST mod3.indb 221 06/02/2011 10:02:17


FSICA I

Equilibro de um corpo rgido o corpo permanece em repouso nessas condies, a fora


Quando um corpo rgido, sujeito ao simultnea de vrias de atrito esttico igual componente Pt do peso, paralela
foras coplanares, encontra-se em equilbrio, temos: rampa:
Fa = Pt = P . sen
F = 0ou MRU);
equilbrio de translao (centro de massa em repouso
o corpo encontra-se na iminncia de deslizamento a fora
M = 0docorpo).
equilbrio de rotao (em relao a qualquer ponto
de atrito esttico mxima e igual componente Pt do peso:
Fa(mx) = P . sen

Plano inclinado sem atrito eN = P . sen


Um bloco apoiado numa superfcie plana, no horizontal,
tem a tendncia de deslizar para baixo. O deslizamento pode Sendo N = P . sen e P = mg, escrevemos:
ocorrer ou no, dependendo do coeficiente de atrito entre as su-
perfcies. Se o deslizamento ocorrer, ser numa direo paralela emg . cos = mg . sen e = tg
ao plano, em movimento retilneo, com sentido para baixo.
Como os vetores fora resultante e acelerao tm sempre a o corpo acelera plano abaixo nesse caso, a fora de atrito
mesma direo e o mesmo sentido, a fora resultante dever ser dinmico menor do que a componente Pt do peso do corpo.
para baixo, numa direo paralela ao plano. A figura abaixo repre- Portanto, h uma fora resultante, dada por:
senta um bloco apoiado em um plano inclinado perfeitamente
FR = Pt Fa(din)
liso e as foras peso e normal agindo sobre ele.
ma = mg . sen dmg . cos
N
a = g(sen d . cos )
A
FR = Pt Pt
a
o de Pn E X E R C C I O
e
dir C 01. (Fuvest-SP) A figura I indica um sistema composto por duas
P
roldanas leves, capazes de girar sem atrito, e um fio inextensvel
B que possui dois suportes em suas extremidade. O suporte A pos-
sui um certo nmero de formigas idnticas, com 20 miligramas
Com base no tringulo ABC, podemos escrever:
cada. O sistema est em equilbrio. Todas as formigas migram
Pn = P . cos ento para o suporte B, e o sistema movimenta-se de tal forma
que o suporte B se apia numa mesa, que exerce uma fora de 40
Pt = P . sen milinewtons sobre ele, conforme ilustra a figura II
Por imposio da rampa, na direo da fora normal, a resul-
tante nula. Portanto:
A
Pn = N N = P . cos A B
Figura I Figura II B
E, na direo paralela ao plano, temos:
Determine:
a) o peso de cada formiga.
FR = Pt = P . sen
b) o nmero total de formigas.
ma = mg . sen a = g . sen
02. Um copo de peso P sustentado por trs fios inestensveis. Sa-
bendo que a intensidade da trao no fio AB de 80N, determine:
OBS.:
A acelerao de um corpo colocado num plano inclinado per- 30 C
feitamente liso (sem atrito) no depende da massa do corpo, mas 1
sen 30 =
somente da inclinao da rampa e do campo gravitacional local. 2
A
B 3
Plano inclinado com atrito cos 30 =
2
Havendo atrito, acrescentamos a fora de atrito, contrria ao
3
deslizamento, ou contrria tendncia de movimento. tg 30 =
3
N
Fa
a) o valor do peso P.
Pt b) a intensidade da trao no fio BC.
a
o de Pn 03. (Masck-SP) O sistema mostrado na figura est em equilbrio.
e A esfera pesa 150N e o corpo W, 100N. A roldana e o fio que
dir
P une W com a esfera so ideais. No h atrito. A reao do apoio
horizontal sobre a esfera vale:
Com base na figura, vamos analisar trs situaes:

222 UECEVEST

Apostilas UECEVEST mod3.indb 222 06/02/2011 10:02:20


FSICA I

a) 70N 08. (UECE 2009.2) Uma caixa cbica, sem o tampo superior, cons-
b) 60N tituda de cinco lminas metlicas iguais, finas, uniformes e de mes-
c) 100N 60 mo material, conforme a figura a seguir.
d) 40N
e) 150N W z
a

04. (FGV-SP) Os trs blocos mostrados na figura permanecem


em repouso graas fora de atrito entre o bloco de 10N e a a
superfcie de apoio. y
10N
x a
No sistema de referencia da figura, as coordenadas x, y e z do
100N 80N
centro de massa da caixa so:
a) (a/2, a/2, 2a/5). c) (2a/5, 2a/5,a/5).
b) (a/2, a/2, a/2). d) (4a/5, a/5 a/5).
Podemos afirmar que o valor da fora de atrito :
a) 10N d) 60N
09. (UECE 2009.2) Trs esferas puntiformes, colineares, cada
b) 20N e) 100N
uma de massa m, esto presas uma as outras e a um eixo de ro-
c) 40N
tao por meio de trs hastes rgidas, cada uma de comprimento
l, inextensvel e de massa desprezveis em relao s massas das
05. (UFP-SP) Um carrinho de pedreiro mantido parado, em
esferas.O conjunto (eixo, haste, esfera, haste, esfera, haste, esfera),
equilbrio, na posio da figura. O peso total do carrinho 800N.
conforme visto na figura a seguir, gira no plano da prova (plano
A fora vertical feita pelo pedreiro em N, de:
horizontal), em torno desse eixo de rotao, com velocidade an-
a) 160
gular constante. A tenso T nas haste 1, 2 e 3 , respectivamente:
b) 320
c) 480 eixo m m m
d) 533 1 2 3
e) 800
P
a) 6 ml, 5 ml 3 ml. c) 3ml, 2ml e ml.
b) ml, 2ml e 3ml. d) 3ml, 5ml e 7ml.
60 40
06. (UERJ) O esquema abaixo, utilizado na elevao de peque-
nas caixas, representa uma barra AB rgida, homognea, com
comprimento L e peso desprezvel, que est apoiada e articulada 10. (ENEM 2010) Durante uma obra em um clube, um grupo
no ponto O. Na extremidade A, aplicada, perpendicularmente de trabalhadores teve de remover uma escultura de ferro maci-
barra, uma fora constante de mdulo F. Na extremidade B, o colocada no fundo de uma piscina vazia. Cinco trabalhadores
coloca-se uma caixa W, que equilibra a barra paralela ao solo. amarraram cordas escultura e tentaram pux-la para cima, sem
sucesso. Se a piscina for preenchida com gua, ficar mais fcil
Se a extremidade A dista 3 L do ponto O, o valor do peso da para os trabalhadores removerem a escultura, pois a
carga W : 4
a) escultura flutuara. Dessa forma, os homens precisaro fazer
F W fora para remover a escultura do fundo.
O b) escultura ficar com peso menor. Dessa forma, a intensidade
A B da fora necessria para elevar a escultura ser menor.
c) gua exercer uma fora na escultura proporcional a sua massa,
e para cima. Esta fora se somar fora que os trabalhadores
a) F b) 2F c) 3F d) 4F e) 5F fazem para anular a ao da fora peso da escultura.
d) gua exercer uma fora na escultura para baixo, e esta
07. (UFSE) Os dois blocos mostrados na figura possuem pesos passar a receber uma fora ascendente do piso da piscina.
iguais a 10 N cada. Despreze o atrito e considere g = 10 m/s , Esta fora ajudar a anular a ao da fora peso na escultura.
sem 30 = 0,50 e cos 30 =0,86. e) gua exercer uma fora na escultura proporcional ao seu
volume, e para cima. Esta fora se somar fora que
os trabalhadores fazem, podendo resultar em uma fora
ascendente maior que o peso da escultura.

30
01. * 02. * 03. c 04. b 05. b
A acelerao dos corpos , em m/s: 06. c 07. a 08. a 09. a 10. e
a) 2,5 d) 7,0
b) 5,0 e) 7,5 01. a) 2,0.104N 02. a) 80 3 N b) 160 3 N
c) 6,5 b) n=100 3 3

UECEVEST 223

Apostilas UECEVEST mod3.indb 223 06/02/2011 10:02:21


FSICA I

Conservao da Energia (UECE/ENEM) Exemplo: A figura representa as intensidades de uma fora apli-
cada em um corpo na direo de seu deslocamento. Determine
Introduo o trabalho realizado pela fora para deslocar o corpo entre as po-
Seja qual for a forma assumida, a energia representa a capa- sies:
cidade de fazer algo acontecer ou funcionar. Podemos dizer que a) d=0 e d=10m
energia a capacidade de realizar um trabalho. Se alguma coisa b) d=10m e d=25m
pode realizar um trabalho, direta ou indiretamente, por meio de c) d=25m e d=30m
alguma transformao, porque essa coisa tem uma forma de d) d=30m e d=35m
energia. e) d=0 e d=35m
F(N)
Trabalho (W)
20

W = F.d
30
0 10 25 35 d(m)

WAB = Fx .d

WAB = F.d.cos
-20

No Sistema Internacional de Unidades (S. I), W= N.m ou Resoluo:


joule (J), em homenagem ao fsico ingls James Prescott Joule a) Neste caso, a fora varivel. Logo, o trabalho dado pela
(1818-1889). rea do tringulo:
W0-10 = 10.20 = 100J
2
1N.m = 1J b) Agora, a fora constante. Logo, o trabalho dado pela
rea do retngulo>
Exemplo: Determine o trabalho realizado pela fora de intensi- W10-25 = 15 . 20 = 300J
dade 20N quando o corpo sofre um deslocamento horizontal de
5m, nos dois casos: c) Caso de fora varivel:
a) b) 60 W25-30 = 5 . 20 = 50J
2
d) Caso de fora varivel: a fora F tem sentido contrrio ao do
Resoluo: deslocamento:
a) Como a fora tem a mesma direo do deslocamento, temos: W30-35 = = 50J

W = F . d W = 20 . 5 = 100J e) O trabalho total a soma de todos os trabalhos parciais:


W0-35 = 100 + 300 + 50 50 = 400J
b) A fora no tem a mesma direo do deslocamento. Logo:
Potncia (Pot)
W = F . d cos W = 20 . 5 . cos 60 W = 20 . 5 . = 50 J Uma mquina caracterizada no pelo trabalho que efetua,
mas pelo trabalho que pode efetuar em determinado tempo, ou
Respostas: seja, sua potncia (Pot).
a) 100J; b) 50J

O trabalho de uma fora F, constante ou no, pode ser obtido


atravs de um grfico.

Fora X deslocamento
A unidade de potncia no S.I: W (Watt)
F
J Obs. :1CV = 735 W
1W = 1
s 1HP = 746 W
deslocamento
0 x
Rendimento ( )
A = WF Putil
= P = Putil + Pdissipada
P total total
F O rendimento o quociente entre duas grandezas de mesma
unidade, portanto, ele adimensional, isto , no tem unidade.
O rendimento tambm pode ser expresso em porcentagem.
deslocamento Exemplo: Um motor consome 2KW quando realiza um trabalho
0 x
de 2800J em 7s.

224 UECEVEST

Apostilas UECEVEST mod3.indb 224 06/02/2011 10:02:23


FSICA I

a) Determine a potncia dissipada por este motor. Energia Potencial Gravitacional (Epg)
b) Calcule o rendimento deste motor.
Trabalho da fora peso: Wp
Resoluo:
a) A potncia total do motor a potncia que ele consome E pg = m.g.h
(Pt = 2KW = 2000W) e a potncia til dada por:
A Energia Potencial Gravitacional tambm dada em Joule (J).

r
Logo, a potncia dissipada igual a: g

Pt = Pu + Pd 2000 = 400 + Pd Pd = 1600J

b) = = = 0,2 ou = 20%
Exemplo: Uma garota ergue verticalmente uma pedra de 200N,
a uma velocidade constante de 2m/s, at uma altura de 8m em
Resposta: relao ao solo. Considere g=10m/s.
a) 1600J; b) 20% a) Qual o valor da fora desenvolvida pela garota?
b) Qual o trabalho realizado pela garota: O que ocorre com
Energia Cintica (Ec) esse trabalho?
W = F.d W = m.a.d c) Calcule a energia potencial da pedra em relao ao solo.
Mas :
V2
Resoluo:
V 2 = V02 + 2.a.d d = (fazendo Vo = 0) a) Como a velocidade constante e o deslocamento em linha
2a
reata (M.R.U.) FR = O, logo: F = P F = 200N
Logo : b) Wp = m.g.h Wp = 20.10.8 Wp = 1600J.
V2 mv 2 Transforma-se em energia potencial gravitacional.
W = ma W =
2a 2 c) Epg= m.g.h Epg = 20.10.8 Epg = 1600J

mv 2 Energia Potencial Elstica (Epe)


Ec =
2
Trabalho da fora elstica: WFel
A unidade EC a mesma de trabalho, J (joule). Lei de Hooke:
Fel = K.x
Teorema da Energia Cintica
W = F.d W = m.a.d Fel
Mas : k.x

V 2 = V02 + 2.a.d ad =
(V 2
V0 2 )
2
Logo : X (deformao)

W=m
(
V V0 2

W =
mv 2

2
)
mv02 A = Wf el W el = F .
x
2 2 2 2
x
W = E C(final ) E C(inicial ) W = E C W = E pe = K.x.
2
Kx 2
Exemplo: Um projtil de massa 20g, com velocidade de 250m/s E pe =
2
dirigida horizontalmente, atinge uma placa de madeira e penetra
30cm nela. Determine o mdulo da fora mdia de resistncia Exemplo: Determine a energia potencial elastica armazenada em
imposta pela madeira ao movimento da bala. uma mola de costante elastica K = 500N/m, quando ela disten-
didade 40cm.
Resoluo:
Resoluo:
Dados: k = 500N/m e x = 40cm = 0,4m.

Epel = = Epel = 40J

Resposta: 40J

Energia mecnica (Em)


A energia no se cria nem se destri, apenas se transforma de
um tipo em outro, em quantidades iguais.
O sinal negativo indica que uma fora de resistncia. Em = Ec + E pg + E pel
Resposta: F 2083,3N

UECEVEST 225

Apostilas UECEVEST mod3.indb 225 06/02/2011 10:02:25


FSICA I

Sistema Conservativo EMf = EMi 02. (UECE 2009.1 1 Fase) Uma escada rolante foi proje-
A energia mecnica total permanece constante. tada para transportar 10 pessoas por minuto do primeiro para
o segundo andar de um shopping center. A escada tem 12m de
E m = E c + E p = cte comprimento e uma inclinao de 30 com a horizontal. Supon-
do que cada pessoa pesa 800N, o consumo de energia da escada
rolante, com capacidade mxima, ser:
Sistema Dissipativo EMf = EMi + Ed a) 800W c) 4000W
b) 400W d) 80W
E m = E c + E p + E dissipada
03. (UECE - 2008.1/1 fase) Assinale a alternativa que, de acor-
Exemplo: Um corpo de 400g de massa lanada verticalmente do com a fsica newtoniana, contm apenas grandezas (fsicas)
para baixo, com velociade de 8m/s, de uma altura de 4m em que no dependem do referencial adotado.
relao ao solo. Considere g = 10m/s. a) Trabalho e energia cintica
a) Desprezando o atrito com o ar, qual o valor da velocidade b) Fora, massa e a celarao
do corpo ao atingir o solo? c) Massa, energia cintica e acelerao
b) Supondo que 20% de sua energia mecnica seja dissipada d) Temperatura e velocidade
pelo atrito com o ar durante a queda, qual o valor da
velodidade do corpo quando estiver a 1,2m do solo? 04. (UECE 2009.2) Considere duas montanhas russas vizinhas,
de mesmo formato e sustentadas sobre um terreno plano, con-
Resoluo: forme a figura.
a) Desprezando o atrito com o ar e utilizando o princpio da A
conservao da energia mecnica, temos:
Ef = Ei Ecf + Epf = Eci + Epi B

P1 P2 P3 P4

Suponha que os pilares de sustentao P1 e P2 sejam de mesmo


b) Como 20% da energia mecnica inicial dissipada pelo tamanho de P3 e P4 tenham 70% do tamanho de P1. Despreze
atrito, temos: Ef = 0,8 Ei, pois: (100% 20% = 80% ou 0,8) quaisquer atritos. Os objetos A e B, puntiformes, partem simul-
Logo: Ecf + Epf = 0,8 (Ecf + Epf ) taneamente do incio de cada montanha sob ao unicamente da
gravidade. Sobre a diferena entre a energia potencial gravitacio-
nal, em relao ao nvel do solo, dos objetos A e B, ao percorre-
rem as montanhas russas, assinale o correto.
a) A diferena entre a energia potencial gravitacional do corpo
A e a do corpo B 70% da energia do corpo A.
b) A diferena entre a energia potencial gravitacional do corpo
A e a do corpo B 30% da energia do corpo A.
c) A diferena entre a energia potencial gravitacional do corpo
A e a do corpo B 0% da energia do corpo A.
d) A diferena entre a energia potencial gravitacional do corpo
A e a do corpo B 50% da energia do corpo A .
E X E R C C I O
01. (UECE.2009.1 2 Fase) A figura abaixo mostra o perfil 05. (UNIFOR) Um corpo de massa 2,0 kg abandonado, no
de uma pista de skate, feita do mesmo material do ponto U ao ponto A de uma pista sem atrito. No ponto B este corpo se choca
ponto Y. com a mola de constante elstica 320N/m, em sua configurao
natural. Adotando g=10m/s,pode-se afirmar que a mxima de-
U formao sofrida pela mola , em metros, de:
X

V Y

Uma jovem skatista parte do ponto U com velocidade nula, passa


pelos pontos V, X e chega ao ponto Y com velocidade nula. Com a) 0,25 d) 1,5
base no exposto, assinale o correto. b) 0,50 e) 2,0
a) A energia cintica em V igual energia potencial c) 0,75
gravitacional em U
b) A energia cintica em V igual energia potencial 06. (UECE 2010.1 1 Fase) Um corpo de massa 2 Kg parte do
gravitacional em X. repouso e cai na vertical. O ar exerce no corpo uma fora de resis-
c) A energia cintica em V igual energia potencial tncia do ar o dobro do modulo da velocidade do corpo em cada
gravitacional em Y. instante. Considerando que a acelerao da gravidade 10 m/s, o
d) A energia cintica em V maior que a energia potencial trabalho da fora resultante que age no corpo, da posio inicial at
gravitacional em X. o ponto onde sua velocidade ser metade da velocidade terminal, :
a) 35J. b) 15J. c) 25J. d) 50J.

226 UECEVEST

Apostilas UECEVEST mod3.indb 226 06/02/2011 10:02:27


FSICA I

A fora que atua sobre o peso e produz o deslocamento vertical


da garrafa a fora
07. (ENEM 2010) Deseja-se instalar uma estao de gerao de a) de inrcia. d) centrpeta.
energia eltrica em um municpio localizado no interior de um b) gravitacional. e) elstica.
pequeno vale cercado de altas montanhas de difcil acesso. A cida- c) de empuxo.
de cruzada por um rio, que fonte de gua para consumo, irri-
gao das lavouras de subsistncia e pesca. Na regio, que possui 10. (ENEM 2005) Observe a situao descrita na tirinha abaixo.
pequena extenso territorial, a incidncia solar alta o ano todo.
A estao em questo ir abastecer apenas o municpio apresenta-
do. Qual forma de obteno de energia, entre as apresentadas,
a mais indicada para ser implantada nesse municpio de modo a
causar o menor impacto ambiental.
a) Termeltrica, pois possvel utilizar a gua do rio no sistema
de refrigerao.
b) Elica, pois a geografia do local prpria para a captaao
desse tipo de energia.
c) Nuclear, pois o modo de resfriamento de seus sistemas no
afetaria a populao.
d) Fotovoltaica, pois possvel aproveitar a energia solar que
chega superfcie do local.
e) Hidreltrica, pois o rio que corta o municpio suficiente
para abastecer a usina construda.

08. (ENEM 2006) A figura a seguir ilustra uma gangorra de brin-


quedo feita com uma vela. A vela acesa nas duas extremidades
e, inicialmente, deixa-se uma das extremidades mais baixa que a
outra. A combusto da parafina da extremidade mais baixa pro- (Francisco Caruso & Luisa Daou, Tirinhas de Fsica, vol. 2, CBPF, Rio de Janeiro,
voca a fuso. A parafina da extremidade mais baixa da vela pinga 2000.)
mais rapidamente que na outra extremidade. O pingar da parafina Assim que o menino lana a flecha, h transformao de um tipo
fundida resulta na diminuio da massa da vela na extremidade de energia em outra. A transformao, nesse caso, de energia
mais baixa, o que ocasiona a inverso das posies. Assim, enquan- a) potencial elstica em energia gravitacional.
to a vela queima, oscilam as duas extremidades. Nesse brinquedo, b) gravitacional em energia potencial.
observa-se a seguinte sequncia de transformaes de energia: c) potencial elstica em energia cintica.
d) cintica em energia potencial elstica.
e) gravitacional em energia cintica.

G A B A R I T O
a) energia resultante de processo qumico energia potencial 01. d 02. a 03. b 04. b 05. b
gravitacional energia cintica 06. c 07. a 08. e 09. d 10. c
b) energia potencial gravitacional energia elstica energia
cintica
c) energia cintica energia resultante de processo qumico
energia potencial gravitacional
d) energia mecnica energia luminosa energia potencial COnSERVAO DA QUAnTIDADE
gravitacional DE mOVImEnTO (UECE/EnEm)
e) energia resultante do processo qumico energia luminosa
energia cintica Impulso ( )
Quando uma fora constante atua em um corpo durante um
09. (ENEM 2005) Observe o fenmeno indicado na tirinha ao lado. curto intervalo de tempo diremos que o corpo recebeu um im-
pulso. No (S.I), I = N.s.
I = F T

Exemplo: A intensidade de uma fora horizontal aplicada sobre


[ um corpo de massa igual a 35kg, incialmente em repouso, varia
com o tempo conforme indica o grfico.
F(N)
20

10

(Adaptado. Luisa Daou & Francisco Caruso, Tirinhas de Fsica, vol. 2, CBPF, Rio 0 4 6 t(s)
de Janeiro, 2000.)

UECEVEST 227

Apostilas UECEVEST mod3.indb 227 06/02/2011 10:02:29


FSICA I

a) Qual a intensidade do impulso dessa fora no intervalo de 0 Exemplo: Determine durante quanto tempo deve agir uma fora
a 6s? de intensidade 40N sobre um corpo de massa 4kg, para que sua
b) Qual a velocidade do corpo no instante 6s? velocidade passe de 20m/s para 80m/s.
Resoluo: Resoluo:
a) A intensidade do impulso dada numericamente pela soma Isolando o corpo, temos:
das reas do retngulo e do trapzio. Vf= 80m/s
Dados: Vi= 20m/s
F= 40N
m= 4kg

Usando o teorema do impulso, vem:

b) A fora mdia que age sobre o corpo de 0 a 6s dada por:

Resposta: 6s

Quantidade de Movimento de um Sistema


A acelerao mdia nesse intervalo de tempo : Q t = Q 1 + Q 2 + Q 3 + ... + Q n

As foras internas podem provocar variaes nas quantidades


de movimento de cada partcula de um sistema, mas no pro-
vocam variaes na quantidade de movimento total do sistema.

Exemplo: Trs pontos materiais, com massas m1= 2kg, m2= 5kg e
Portanto, V = Vo + a.t
m3= 8kg, tem velocidades respectivamente iguais a V1= 10m/s, V2=
6m/s e V3= 20m/s, num certo instante, conforme indica a figura.

Respostas:
a) 70N; b) 2,0m/s

Quantidade de Movimento (Q) m1 m2 m3


uma grandeza fsica que relaciona a massa de um corpo Determine a intensidade da quantidade de movimento desse sis-
com sua velocidade, para caracterizar o estado de movimento tema, no instante considerado.
deste corpo. A quantidade de movimento (ou momento linear)
de um corpo de massa m, que se move com uma velocidade v, Resoluo:
definida pela expresso: A itensidade para cada ponto material :
Q = mv Q1= m1 V1 = 2 . 10 = 20kg.m/s
Q2= m2 V2 = 5 . 6 = 30kg.m/s
m Q3= m3 V3 = 8 . 20 = 160kg.m/s
No S.I.: Q = kg.
s
Colocando a origem dos vetores no ponto A, temos:
Exemplo: Um carro de 900kg de massa, parte do repouso com
acelerao de 2m/s, com direo horizontal e sentido para a di-
reita. Determine as caractersticas da quantidade de movimento
desse carro no instante 15s.
Resoluo:
Clculo da velocidade no instante 15s: A
V= Vo + a.t V= 0 + 2 . 15 V= 30m/s
A quantidade de movimento tem as seguintes caractersticas:
Intensidade: Q= m.V Q= 900.30 Q= 27000kg.m/s Fazendo = + , temos =30 + 160 = 190 kg . m/s
Direo: horizontal
Sentido: da esquerda para a direita A quantidade de movimento do sistema :

Teorema do Impulso
F = m.a
v vo
F=m
t
F.Dt = mV mVo A
I = Qf Qi

I = Q

228 UECEVEST

Apostilas UECEVEST mod3.indb 228 06/02/2011 10:02:31


FSICA I

Princpio da Conservao da Quantidade de movimento 2m/s 2m/s


Se for nula a resultante das foras externas que atuam em um b) antes A B
sistema de particulas, a quantidade de movimento total deste siste-
depois A B parados
ma se conserva, em outras palavras, a quantidade de movimento de
um sistema de corpos isolados de foras externas constante. Este Resoluo:
princpio aplicado geralmente em exploses, disparos e choques,
onde as foras internas so muito mais intensas que as externas. a)

b)
Exemplo: Um peixe de 8,6kg, nadando para a direita a 1m/s,
engole um peixe de 0,4kg, que nada na sua direo a 3,5m/s, ou
seja, em sentido contrrio. Tipo de Colises
Determine o mdulo da velocidade do peixe maior imediata- Perfeitamente elstica: A energia cintica final igual ener-
mente aps engolir o menor. gia cintica inicial.
Q antes = Qdepo is Vaf = Vap e = 1
Resoluo:
Utilizando o princpio da conservao da quantidade de mo- Parcialmente elstica: H perda de energia cintica, sendo a
vimento, temos: energia cintica final menor que a inicial.
Sendo:
Q antes = Qdepo is Vaf < Vap 0 < e <1
Q1 quantidade de movimento do peixe maior
Q2 Quantidade de movimento do peixe menor Inelstica: Se os dois corpos que colidem caminharem juntos
aps a coliso. Pode ocorrer que toda a energia cintica do siste-
ma se transforme e os corpos fiquem em repouso aps a coliso:
Q antes = Qdepo is e = 0 Vaf = 0

Exemplo: Um carro de 800 Kg, parado num sinal vermelho,


abalroado por trs por outro carro, de 1200 Kg com uma veloci-
dade de 72 Km/h. Imediatamente aps o choque os dois carros se
movem juntos. Calcule a velocidade aps a coliso.
Resposta: 0,8m/s
Resoluo:
Colises Qf = Qi (mA+mB).V = mA .VA +mB .VB
Em uma coliso, a quantidade de movimento total se con- m
serva. (1200 + 800).V = 1200.20 + 800.0 V = 12
s
Coeficiente de Restituio (e) Exemplo: Em uma mesa de sinuca, a bola branca, de massa m,
movendo-se com velocidade v = 2,0 m/s, atinge a bola amarela
Velocidade Relativa de Afastamento (tambm de massa m), que se encontrava em repouso. Supondo
e=
Velocidade Relativa de Aproximao que o choque seja central e elstico, determine as velocidades das
duas bolas bolas aps a coliso.
Vaf
e=
Vap

OBS.:
O coeficiente de restituio um nmero puro, pode variar Em qualquer coliso, como as que ocorrem entre bolas de
entre 0 e 1 (0 e 1) e depende dos materiais que os corpos bilhar, h conservao da quantidade de movimento.
so constitudos.
Quando as velocidades dos corpos tiverem sentidos opostos Sejam V1 e V1 as velocidades das bolas branca e amarela, aps
soma, e mesmo sentido a velocidade relativa a diferena das a coliso. A quantidade de movimento do sistema (das duas bo-
velocidades. las), antes do choque, era mv, pois apenas a bola branca estava em
movimento. Como sabemos, em qualquer coliso h conservao
Exemplo: Calcule o coeficinte de restituio dos seguintes choques: da quantidade de movimento total e, ento, podemos escrever

10m/s 4m/s m v = mV1 + mV1


a) antes A B
J que o choque central, os vetores v , V1 e V1 tem a mesma
1m/s 5m/s direo e, portanto, a relao anterior poder ser escrita escalar-
depois A B
mente, isto mv = mV1 + mV2 ou v = V1 + V2 donde V1 + V2 = 2,0.

UECEVEST 229

Apostilas UECEVEST mod3.indb 229 06/02/2011 10:02:34


FSICA I

Alm disso, tratando-se de uma coliso elstica, a energia ci- a) a energia mecnica dos blocos se conserva.
ntica do sistema se conserva. b) a quantidade de movimentos dos blocos no se conserva.
Logo: c) a velocidade do bloco mais leve, aps o choque, de
1 mv = 1 mV + 1 mV ou v = V + V donde aproximadamente 2,0 m/s.
2 2
1
2
2 1 2 1 d) a velocidade do bloco mais leve, aps o choque, de
aproximadamente 1,0 m/s.
V1 + V2 = 4,0
Obtivemos, assim, duas equaes, relacionando as incgnitas 03. (UECE 2009.1 2Fase) Uma partcula de massa M e veloci-
V1 e V2: dade de mdulo V colide com uma superfcie plana, fazendo um
V1 + V2 = 2,0 e V1 + V2 = 4,0 ngulo de 30 com a mesma. Aps a coliso a partcula refletida
Da primeira equao, tiramos V1 = 2,0 V2 e, substituindo com uma trajetria cuja direo tambm faz um ngulo de 30
com a superfcie, como ilustrado na figura. Considerando que o
na segunda, vir
mdulo da velodicade da partcula continua o mesmo, aps a co-
(2,0 V2)2 + V2 = 4,0
liso, a alterao na quantidade de movimento da partcula na di-
Resolvendo esta equao, obtemos V2 = 2,0 m/s e, como V1 = reo perpendicular parede devido coliso, em mdulo, igual a:
2,0 V2, conclumos que V1 = 0. a) 0 30
b) MVsen30
c) 2MVsen30
d) 2MV

30

04. (UECE-2004.2) Um projtil de 20g de massa entra horizon-


talmente numa tbua fixa, de modo que, imediatamente antes de
No dispositivo mostrado na figura, todas as esferas (de ao) nela penetrar, sua velocidade de 800m/s e ao sair da tbua, a ve-
tm a mesma. Levantando-se a esfera de uma das extremidades locidade tambm horizontal, de 600m/s. Sendo de 0,01s o tem-
e deixando-a colidir com as demais, como em (a), observamos po que o projetil permaneceu no interior da tbua, a intensidade
que ela entra em repouso e apenas a bola da outra extremidade mdia da fora que a tbua exerce sobre o projetil, em newtons, :
adquire uma velocidade igual da esfera iniciante(b). a) 400
b) 300
Assim, vemos que, em virtude da coliso, a bola branca entra c) 200
em repouso e a bola amarela adquire uma velocidade igual que a d) 100
bola branca possua antes do choque. possvel que voc j tenha
visto este fato ocorrer em um jogo de bilhar.
05. (UNIFOR) Um corpo de massa 1,5kg, inicialmente em re-
pouso, sofre a ao de uma fora nica, de direo constante, cujo
mdulo varia com o tempo de acordo com o grfico. No instante
E X E R C C I O t=4,0s a velocidade do corpo , em m/s.
01. (UECE 2007.1 2 Fase) Por transportar uma carga extre- a) 2,0
mamente pesada um certo caminho trafega a uma velocidade de b) 5,0
10m/s Um rapaz beira da estrada brinca com uma bola de tnis. c) 10
Quando o caminho passa, ele resolve jogar a bola na trazeira d) 15
do mesmo. Sabendo-se que a bola atinge a traseira do caminho e) 20
perpendicularmente, com velocidade 20m/s, em relao ao solo,
qual a velocidade horizontal final da bola aps o choque? Consi- 06. (UECE.2006.2) A figura um grfico aproximado da fora
dere um choque perfeitamente elstico. exercida durante um choque de uma bola de tnis com uma parede.
a) 10m/s c) 20m/s
b) zero d) 30m/s

02. (UECE 2010.1 - 2 Fase) A figura ao lado mostra dois ins-


tantneos de dois blocos cujas massas so MI = 2,0 kg e MII =
3,0 kg, que deslizam, na mesma direo e sentido, sobre uma
superfcie horizontal sem atrito, e se chocam entre si. Um ins-
tante antes da coliso, e outro aps a coliso. Pelas informaes
contidas na figura, podemos afirmar que A massa da bola 40g, sua velocidade inicial igual a 32 m/s e
sua direo perpendicular a parede. Ela recua sem perder velo-
cidade e tambm perpendicular a parede. Determine, em N, a
fora mxima exercida pela parede sobre a bola.
a) 0,40 c) 0,20
b) 0,32 d) 0,64

07. (UECE 2009.2) Duas Massas puntiformes chocam frontal-


mente. Uma delas tem velocidade inicial de mdulo V e massa
M, e a outra tem velocidade inicial nula e massa m. Considere o
caso em que a massa m pode ser considerada desprezvel se com-
parada massa M. Nessa situao o mdulo da velocidade final
da menor massa aps o choque, :

230 UECEVEST

Apostilas UECEVEST mod3.indb 230 06/02/2011 10:02:36


FSICA I

Obs: Considere o choque perfeitamente elstico. Fsica Fundamental (FTD)


a) 2V. c) V/2. Autores: Bonjorno e Clinton
b) V. d) V/4.
Site da UECE / UFC / CEFET-Ce.
08. (UECE.2009.1 1 Fase) Um grupo de alunos, no labora-
trio de fsica, afirma que observaram uma coliso perfeitamente
elstica entre duas esferas metlicas bem polidas, em uma super-
fcie horizontal, que resultou nas duas esferas terminarem em
repouso. Nenhuma fora externa horizontal estava agindo nas
esferas no instante a coliso. Sobre o fato, assinale o correto.
a) As velocidade escalares iniciais das duas esferas eram iguais e
suas massas eram idnticas.
b) As velocidade escalares iniciais das duas esferas eram
diferentes e suas massas eram, tambm diferentes.
c) As velocidade escalares iniciais das duas esferas eram iguais,
mas suas massas no necessariamente eram idnticas.
d) A coliso no pode ter ocorrido como afirmado pelo grupo.

09. (UNIFOR 2007.1) Um carrinho de madeira, de massa 4 Kg,


move-se numa superfcie horizontal de atrito desprezvel com ve-
locidade de 0,6 m/s. Deixa-se cair verticalmente sobre o carrinho
um bloco de argila de massa 2 Kg que, aps a coliso, segue co-
lado com o carrinho. A velocidade do conjunto imediatamente
aps o impacto , em m/s.
a) zero d) 0,40
b) 0,20 e) 0,50
c) 0,30

10. (UECE 2009.2) Um objeto puntiforme desliza sob a ao da


gravidade sob uma semi-esfera cuja seo plana fixada ao solo.
Considere que o objeto parte do ponto mais alto, mediante a apli-
cao de uma perturbao muito pequena, de modo que se possa
considerar como nula sua velocidade inicial. Considere desprez-
veis todos os atritos e suponha constante a acelerao da gravidade.
No instante em que o objeto perde contato com a semi-esfera, a
frao da energia mecnica correspondente energia cintica
a) 1/3. c) 1/2.
b) 2/3. d) 1/4.

G A B A R I T O
01. b 02. c 03. c 04. a 05. e
06. d 07. a 08. d 09. d 10. a

REFERnCIAS BIBlIOGRFICAS
Curso de Fsica (Scipione)
Autores: Antnio Mximo e Beatriz Alvarenga.

Temas de Fsica (FTD)


Autores: Bonjorno e Clinton

Fsica Bsica (Atual)


Autores: Nicolau e Toledo

Fsica (tica)
Autores: Wilson Roberto e Carlos Barros

Apostila Fsica (IBEP)


Autor: Ivan Gonalves dos Anjos

UECEVEST 231

Apostilas UECEVEST mod3.indb 231 06/02/2011 10:02:36


Apostilas UECEVEST mod3.indb 232 06/02/2011 10:02:36
P R - V E S T I B U l A R

FSICA II

Apostilas UECEVEST mod3.indb 233 06/02/2011 10:02:43


Caro(a) Aluno(a),

Para facilitar o acompanhamento de tais contedos, abaixo esto indicadas as nomencla-


turas utilizadas pela UECE e pelo ENEM:

UECE ENEM
Introduo a ptica geomtrica Oscilao, ondas, pticas e radiao.
Espelhos esfricos Oscilao, ondas, pticas e radiao.
Refrao da luz Oscilao, ondas, pticas e radiao.
Lentes esfricas Oscilao, ondas, pticas e radiao.

Apostilas UECEVEST mod3.indb 234 06/02/2011 10:02:43


FSICA II

InTRODUO A OPTICA GEOmTRICA Classificao dos meios ticos


(UECE/EnEm) meio transparente: um meio no qual a luz se propaga em traje-
trias regulares e que permite a visualizao ntida dos objetos.
ptica geomtrica Meio translcido: um meio no qual a luz se propaga em traje-
a parte da Fsica que estuda a trajetria da luz, no se preo- trias irregulares e no propicia visualizao ntida dos objetos.
cupa com a sua natureza. Meio opaco: um meio no qual a luz no se propaga e que no
permite a visualizao dos objetos.
Raio de luz Meio homogneo: um meio onde suas propriedades so as
um ente imaginrio utilizado para representar a direo e o mesmas em qualquer elemento de volume desse meio.
sentido de propagao da luz por uma linha orientada. Meio isotrpico: um meio em que as propriedades fsicas inde-
pendem da direo na qual foram medidas.

A cor dos corpos


Feixe de luz A cor apresentada por um corpo, ao ser iluminado, depende
do tipo de luz que ele reflete difusamente. Assim, corpos de dife-
rentes cores, sendo iluminados por luz branca, teremos:
Lu Lu Lu Lu
Feixe de luz: um conjunto de raios de luz. z bra z bra z bra z bra
nc nc nc nc
Um feixe luminoso pode ser: a a a a

Corpo Corpo Corpo Corpo


branco vermelho amarelo preto
O corpo vermelho, se refletir difusamente a luz vermelha e
cnico convergente cnico divergente cilndrico paraleo absorver as demais; o corpo amarelo, se refletir difusamente a luz
amarela e absorver as demais, e assim por diante. E negro, se o
Fonte de luz corpo absorver todas as luzes.
So corpos que podem ser vistos, ou seja, emitem luz. Atente ao fato de que um corpo amarelo parecer negro
quando for iluminado por luz diferente da branca e da amarela,
Classificao das fontes de luz pois essa luz ser devidamente absorvida.
Lu Lu Lu Lu
z za z z
Quanto emisso de luz bra
nc
ma
rela
ver
d
azu
l
a e
Fonte primria ou corpo luminoso: emite luz prpria.
Ex: O Sol, as estrelas, uma lmpada ligada, uma vela acesa, um
vaga-lume, um interruptor, metal aquecido ao rubro etc.
Corpo Corpo Corpo Corpo
Fonte secundria ou corpo iluminado: no emite luz prpria, parece parece parece parece
reflete luz de uma fonte primria. amarelo amarelo negro negro
Ex: A Lua, os planetas, um caderno, uma caneta, uma cadeira,
uma pessoa, um quadro etc. Os princpios da tica geomtrica
Princpio da propagao retilnea da luz: nos meios homog-
Quanto dimenso neos, isotrpicos e transparentes a luz se propaga em linha reta.
Fonte pontual ou puntiforme: suas dimenses so desprez- Princpio da independncia dos raios luminosos: ao se pro-
veis em ralao ao ambiente considerado. pagar em um meio, cada raio de luz se comporta independen-
Ex: Uma pequena lmpada num salo. temente de qualquer outro raio de luz.
Fonte extensa: suas dimenses no so desprezveis em relao Princpio da reversibilidade: a trajetria seguida por um raio
ao ambiente considerado. de luz no depende do sentido em que o percurso feito.
Ex: Uma lmpada fluorescente num quarto.
Eclipse do sol e eclipse da lua
Tipo de luz A ocorrncia de eclipse do sol e de eclipse da lua explicada
pelo princpio da propagao retilnea da luz
Classificao da luz quanto cor
Luz monocromtica: a luz de uma nica cor, ou seja, no
decorrente de uma composio de cores.
Ex: A luz amarela emitida pelo vapor de sdio incandescente.
Luz policromtica: a luz resultante da composio de luzes
monocromticas.
Ex: A luz branca emitida pelo Sol constituda por uma infini-
dade de luzes monocromticas, as quais podem ser divididas em
sete cores principais:

Vermelho Alaranjado Amarelo Verde Azul Anil Violeta

UECEVEST 235

Apostilas UECEVEST mod3.indb 235 06/02/2011 10:02:43


F S I C A II

Cmara escura de orifcio


uma aplicao prtica do princpio da propagao retilnea
da luz e funciona como uma mquina fotogrfica rudimentar

H h

D d

H D
= A regio do espao visvel por reflexo no espelho fica ento
h d
determinada pela retas que tangenciam as bordas do espelho que
passam por 0 (imagem do observador).
1 lei da reflexo
O raio incidente I, a reta normal superfcie no ponto de Translao do espelho plano
incidncia N e o correspondente raio refletido R devem ser co- Quando um espelho plano sofre uma translao o desloca-
planares (pertencem ao mesmo plano). mento (velocidade) sofrido pela imagem igual ao dobro do des-
locamento (velocidade) sofrido pelo espelho.
N
I R DI = 2 . dE VI = 2 . VE

Rotao do espelho plano

i r

2 lei da reflexo
O ngulo de incidncia i, entre o raio incidente e a reta normal,
igual ao ngulo de reflexo r, entre o raio refletido e a reta normal.
^ ^
i = r
O desvio angular sofrido pelo raio refletido corresponde ao
Espelho plano: formao de imagens dobro do deslocamento angular sofrido pelo espelho.

=2.

Associao de espelhos planos

Caractersticas:
A distncia do objeto ao plano do espelho igual distncia da
imagem ao plano do espelho.
O tamanho da imagem coincide com o tamanho do objeto Em alguns casos possvel calcular o nmero N de imagens
independentemente da distncia do objeto ao espelho. obtidas numa associao de dois espelhos planos que formam en-
Os espelhos planos formam imagens direitas, ou seja, a imagem tre si um ngulo .
no sofre inverso.
Para um espelho plano, o objeto e a imagem tm naturezas 360
opostas. N= 1
Objeto Real Imagem virtual
Objeto virtual Imagem real
N o nmero de imagens formadas.
A imagem fornecida por um espelho plano troca o lado direi- o ngulo entre os espelhos, em graus.
to pelo lado esquerdo. A imagem que sofre reverso dita
imagem enantiomorfa.
E X E R C C I O
Campo visual
a regio do espao visvel por reflexo em um espelho. O 01. Nas figuras I, II e III, esto representados fenmenos fsicos
campo visual de um espelho depende do tipo e do tamanho do que podem ocorrer quando um feixe de luz incide na superfcie
espelho, da posio em que esse espelho se encontra, e, tambm, de separao entre dois meios de ndices de refrao diferentes.
da posio ocupada pelo observador. Em cada uma delas, esto mostradas as trajetrias desse feixe.

236 UECEVEST

Apostilas UECEVEST mod3.indb 236 06/02/2011 10:02:45


FSICA II

a) independente quando se propaga.


b) reversvel quando se propaga.
c) se propaga em linha reta.
d) se propaga em linha curva.
e) contorna os objetos ao se propagar.

Considerando-se essas informaes, CORRETO afirmar que 08. Admita que o Sol subitamante morresse, ou seja, sua luz
ocorre mudana no mdulo da velocidade do feixe de luz apenas deixasse de ser emitida. 24 horas aps esse evento, um eventual
no(s) fenmeno(s) fsico(s) representado(s) em sobrevivente, olhando para o cu, sem nuvens, veria:
a) I. c) I e II. a) a luz e estrelas.
b) II. d) I e III. b) somente estrelas.
c) somente estrelas.
02. A figura representa um feixe de raios paralelos incidentes d) uma completa escurido.
numa superfcie S e os correspondentes raios emergentes e) somente os planetas do sistema solar.

09. O p de uma rvore de 5,0 m de altura est situado


distncia de 3,0 m de um espelho plano vertical. A respeito da
imagem formada por esse espelho, pode-se dizer que ser:
a) real e medir 5,0 m.
b) virtual e medir 3,0 m.
Esta figura ilustra o fenmeno ptico da c) real e medir 2,5 m.
a) disperso. c) refrao d) virtual e medir 5,0 m.
b) reflexo difusa d) difrao e) virtual e medir 1,5 m.

03. Uma cmara escura de orifcio fornece a imagem de um pr- 10. A figura mostra um espelho plano que pode girar em torno
dio, o qual se apresenta com altura de 5cm. Aumentando-se de de um eixo contendo seu centro C. Estando na posio E1, o
100m a distncia do prdio cmara, a imagem se reduz para espelho capta a luz proveniente de uma fonte pontual A, fixa no
4cm de altura. Qual a distncia entre o prdio e a cmara, na anteparo, refletindo-a de volta ao ponto de partida. O espelho so-
primeira posio? fre, ento, uma rotao equivalente a um ngulo de 15, passando
a) 100 m d) 400 m para a posio E2. Neste caso, a luz recebida de A reflete-se para
b) 200 m e) 500 m o ponto B. A distncia vale:
c) 300 m
a) 3
.d
04. Considere as seguintes afirmativas: 3
I. A gua pura um meio translcido.
II. O vidro fosco um meio opaco. b) 3 .d
III. O ar um meio transparente.
c) 2
Sobre as afirmativas acima, assinale a alternativa correta. .d
2
a) Apenas a afirmativa I verdadeira. d) 2 . d
b) Apenas a afirmativa II verdadeira.
c) Apenas a afirmativa III verdadeira. e) 2 .d
d) Apenas as afirmativas I e a III so verdadeiras.
e) Apenas as afirmativas II e a III so verdadeiras.
11. Pedro, que utiliza seu relgio na mo esquerda, coloca-se a
05. Ana Maria, modelo profissional, costuma fazer ensaios foto- trs metros de um espelho plano. O garoto levanta a mo esquer-
grficos e participar de desfiles de moda. Em trabalho recente, ela da. Analise as afirmaes a seguir:
usou um vestido que apresentava cor vermelha quando ilumina- I. Pedro v sua imagem a seis metros de si.
do pela luz do sol. Ana Maria ir desfilar novamente usando o II. A imagem invertida, isto , est com os ps para cima.
mesmo vestido. Sabendo-se que a passarela onde Ana Maria vai III. A imagem levanta a mo que no possui relgio.
desfilar ser iluminada agora com luz monocromtica verde, po- IV. A imagem tem a mesma altura do garoto.
demos afirmar que o pblico perceber seu vestido como sendo
a) verde, pois a cor que incidiu sobre o vestido. Assinale a nica alternativa correta:
b) preto, porque o vestido s reflete a cor vermelha. a) I e III. c) Apenas I.
c) de cor entre vermelha e verde devido mistura das cores. b) II e IV. d) I e IV.
d) vermelho, pois a cor do vestido independe da radiao
incidente. 12. Voc pula de um trampolim alto sobre uma piscina de
gua completamente calma no fundo da qual existe um
06. Durante a final da Copa do Mundo, um cinegrafista, desejan- espelho plano. Se sua imagem refletida nesse espelho
do alguns efeitos especiais, gravou cena em um estdio completa- aproxima-se de voc, num dado instante, a 16 m/s, com que
mente escuro, onde existia uma bandeira da Azurra (azul e bran- velocidade voc se aproxima do espelho?
ca) que foi iluminada por um feixe de luz amarela monocromtica. a) 16 m/s. d) 4 m/s.
Quando a cena foi exibida ao pblico, a bandeira apareceu: b) 8 m/s. e) 64 m/s.
a) verde e branca. c) preta e branca. c) 32 m/s.
b) verde e amarela. d) preta e amarela.
13. O estudo da luz e dos fenmenos luminosos sempre atraiu os
07. O fenmeno conhecido como eclipse solar evidencia que a luz: pensadores desde a antiga Grcia. Muitas so as aplicaes dos es-

UECEVEST 237

Apostilas UECEVEST mod3.indb 237 06/02/2011 10:02:47


F S I C A II

pelhos e lentes, objetos construdos a partir dos estudos realizados espelho denominado cncavo e, quando a superfcie refletora
em ptica. A figura representa um periscpio, instrumento que a externa, o espelho convexo.
permite a observao de objetos mesmo que existam obstculos
opacos entre o observador e uma regio ou objeto que se deseja ob-
servar. Considere que, nesse periscpio, E1 e E2 so espelhos planos.

Principais elementos geomtricos


comuns aos espelhos esfricos

R o raio de curvatura
C o centro de curvatura
F o foco do espelho
E.P o eixo principal do espelho
A respeito do periscpio e dos fenmenos luminosos que a ele V o vrtice do espelho
podem ser associados so feitas as afirmativas: a distncia focal
I. A colocao de espelhos planos, como indicada na figura, Espelho esfrico de GaussSo os espelhos esfricos que
permite que a luz proveniente da rvore atinja o observador obedecem s condies de nitidez de Gauss.
comprovando o princpio da propagao retilnea da luz. Condies de nitidez de Gauss: os raios incidentes sobre o
II. O ngulo de incidncia do raio de luz no espelho E1 espelho devem ser paralelos ou poucos inclinados em relao ao
congruente ao ngulo de reflexo nesse mesmo espelho. eixo principal e prximos dele.
III. Como os espelhos E1 e E2 foram colocados em posies
paralelas, os ngulos de incidncia do raio de luz no espelho Propriedades dos espelhos esfricos de gauss
E1 e de reflexo no espelho E2 so congruentes entre si. Todo raio de luz que incide paralelamente ao eixo principal
Dessas afirmativas. Est correto apenas o que se l em: reflete-se numa direo que passa pelo foco principal.
a) II d) II e III
b) I e II e) I, II e III
c) I e III

14. Os dois espelhos planos perpendiculares E e F da figura abai-


xo conjugam do objeto A trs imagens B, C e D.

Todo raio de luz que incide numa direo que passa pelo foco
principal reflete-se paralelamente ao eixo principal.

Se os espelhos E e F se transladam com velocidade de mdulo 3,0


cm/s e 4 cm/s, respectivamente, a imagem D se movimenta com
velocidade de mdulo igual a:
a) 30 cm/s d) 7,0 cm/s
b) 20 cm/s e) 10 cm/s Todo raio de luz que incide numa direo que passa pelo
c) 5,0 cm/s centro curvatura reflete-se sobre si mesmo.

ESPELHOS ESFRICOS (uece/enem)


Definio
Espelho esfrico uma calota esfrica na qual uma de suas
superfcies refletora. Quando a superfcie refletida a interna, o

238 UECEVEST

Apostilas UECEVEST mod3.indb 238 06/02/2011 10:02:48


FSICA II

Todo raio de luz que incide sobre o vrtice do espelho reflete-se


simetricamente em relao ao eixo principal.

A imagem de um objeto real sempre virtual direita e menor


do que o objeto.

Estudo Analtico dos Espelhos Esfricos:

Formao das imagens


Espelho Esfrico Cncavo {O Objeto Real / I Imagem.
1 Caso: Objeto alm do centro de curvatura

f a distncia focal.
p a distncia do objeto ao vrtice do espelho.
p a distncia da imagem ao vrtice do espelho.
a altura do objeto.
A imagem forma-se no centro de curvatura e o foco principal a altura da imagem.
do espelho e real, invertida e menor do que o objeto. A o aumento linear transversal.
2 Caso: Objeto sobre o centro de curvatura
Equao dos pontos conjugados:

Equao do aumento linear transversal: -

Refencial de Gauss para os Espelhos


Espelho Cncavo: f > 0
A imagem forma-se no centro de curvatura e real, invertida Espelho Convexo: f < 0
e mesmo tamanho que o objeto.
Imagem Real: p > 0 e i < 0
3 Caso: Objeto entre o centro de curvatura e o foco principal Imagem Virtual: p < 0 e i > 0

Imagem Direita: A > 0


Imagem Invertida: A < 0

A imagem forma-se alm do centro de curvatura e real, in- E X E R C C I O


vertida e maior que o objeto.
01. Um objeto real colocado sobre o eixo principal de um espe-
4 Caso: Objeto sobre o foco principal lho esfrico cncavo a 4 cm de seu vrtice. A imagem conjugada
desse objeto real e est situada a 12 cm do vrtice do espelho,
cujo raio de curvatura :
a) 2 cm d) 5 cm
b) 3 cm e) 6 cm
c) 4 cm

02. Um objeto de 8 cm de altura, colocado perpendicularmente


Imagem de natureza imprpria (a imagem se forma no in- 10 cm frente de um espelho convexo de 6 cm de distncia focal,
finito) fornece uma imagem:
a) real, de 12 cm de altura, direita.
5 Caso: Objeto entre o foco principal e o vrtice b) real, de 3 cm de altura, invertida.
c) virtual, de 12 cm de altura, invertida.
d) virtual, de 3 cm de altura, direita.
e) virtual, de 3 cm de altura, invertida.

03. Um pequeno objeto colocado perpendicularmente sobre o


eixo principal e a 12cm do vrtice de um espelho esfrico cn-
A imagem forma-se atrs do espelho e virtual, direita e cavo, cujo raio de curvatura 36cm. A imagem conjugada pelo
maior do que o objeto. espelho :
a) real, invertida e maior que o objeto
Espelhos Esfricos convexos: b) virtual, direita e maior que o objeto

UECEVEST 239

Apostilas UECEVEST mod3.indb 239 06/02/2011 10:02:50


F S I C A II

c) virtual, direita e menor que o objeto o objeto for deslocado para 10cm do espelho, a nova imagem
d) real, invertida e menor que o objeto aparecer a uma distncia:
a) 10 cm d) 30 cm
04. Quando colocamos um pequeno objeto real entre o foco b) 15 cm e) infinita
principal e o centro de curvatura de um espelho esfrico cncavo c) 20 cm
de Gauss, sua respectiva imagem conjugada ser:
a) real, invertida e maior que o objeto. 10. A figura representa uma seta objeto real colocada perpendi-
b) real, invertida e menor que o objeto. cularmente ao eixo principal de um espelho cncavo e a imagem
c) real, direita e maior que o objeto. a ela conjugada, sendo . A distncia de ao centro do
d) virtual, invertida e maior que o objeto. espelho vale:
80cm
05. Selecione a alternativa falsa relativa aos espelhos esfricos de O
pequena abertura.
a) Em um espelho cncavo, todo raio incidente, paralelo ao C F V
eixo principal, reflete-se passando pelo foco; I
b) Em um espelho cncavo, todo raio incidente, passando pelo,
foco, reflete-se paralelamente ao eixo principal. a) 40cm d) 50cm
c) Em um espelho cncavo, todo raio incidente, passando pelo b) 30cm e) 10cm
centro, reflete-se sobre si mesmo. c) 20cm
d) A imagem de um objeto real formada em um espelho
convexo sempre virtual. 11. Um objeto linear de altura h est assentado perpendicularmen-
e) A imagem de um objeto real formada em um espelho te no eixo principal de um espelho esfrico, a 15cm de seu vrtice.
cncavo sempre real. A imagem produzida direita e tem altura de h/5. Este espelho
a) cncavo, de raio 15 cm.
06. Uma pequena bola solta de uma altura, e cai verticalmente b) cncavo, de raio 7,5 cm.
de A para um ponto B situado sobre o eixo principal de um espe- c) convexo, de raio 7,5 cm.
lho cncavo de 3m de distncia focal. Se Vo representa o mdulo d) convexo, de raio 15 cm
da velocidade mdia da bolinha no seu percurso de A a B, e V1 e) convexo, de raio 10 cm.
o mdulo da velocidade mdia de sua imagem produzida pelo
referido espelho, vlida a relao: 12. A distncia entre um objeto e a imagem que um espelho
a) V1 = 2 . Vo esfrico lhe conjuga mede 15 cm. Sendo ambos reais, com o ob-
b) V1 = 3 . Vo jeto apresentando altura que o dobro da imagem, determine a
c) V1 = 1,5 . Vo natureza do espelho e a sua distncia focal.
d) V1 = Vo a) Espelho convexo e distncia focal de 10,0 cm.
e) V1 = 4 . Vo b) Espelho cncavo e distncia focal de 3,3 cm.
c) Espelho convexo e distncia focal de 3,3 cm.
d) Espelho cncavo e distncia focal de 10,0 cm.

e) Espelho cncavo e distncia focal de 1,5 cm.

07. Um objeto colocado a 10 cm de um espelho forma uma ima- 13. Uma seta objeto colocada perpendicularmente ao eixo prin-
gem real de mesmo tamanho que o objeto. Desloca-se o objeto cipal de um espelho cncavo, ficando a uma distncia de 16cm
para 4 cm de distncia do espelho. Sobre o espelho e a imagem, de sua imagem. Sabendo que a imagem direita e 3 vezes maior,
na nova posio, CORRETO afirmar que o raio de curvatura do espelho, em cm, vale:
a) o espelho cncavo e a imagem real, invertida e maior a) 12 d) 16
que o objeto. b) 13 e) 20
b) o espelho cncavo e a imagem virtual, direita e maior c) 15
que o objeto.
c) o espelho plano e a imagem virtual, direita e de mesmo
tamanho que o objeto.
d) o espelho convexo e a imagem virtual, direita e maior
que o objeto. REFRAO DA LUZ (UECE/ENEM)
e) o espelho convexo e a imagem virtual, direita e menor
que o objeto. Definio de refrao
a variao na velocidade da luz ao mudar de meio de pro-
08. Deseja-se a imagem de uma lmpada, ampliada 5 vezes, so- pagao.
bre uma parede situada a 12cm de distanciado objeto. Quais as
caractersticas e a posio do espelho esfrico que se pode utilizar? ndice de refrao absoluto:
Ele dever ser: O ndice de refrao absoluto de um meio o quociente entre
a) convexo, com 5cm de raio, a 3cm da lmpada. a velocidade da luz no vcuo e a velocidade da luz neste meio.
b) cncavo, com 5cm de raio, a 3cm da lmpada.
c) convexo, com 24cm de raio, a 2cm da lmpada.
d) cncavo, com 6cm de raio, a 4cm da lmpada.
e) convexo, com 6cm de raio, a 4cm da lmpada. ndice de refrao relativo:
O ndice de refrao relativo entre o meio 1 e o meio 2 :
09. (Pucmg 99) Um objeto situado a 20cm de um espelho cn-
cavo forma uma imagem real de tamanho igual ao do objeto. Se

240 UECEVEST

Apostilas UECEVEST mod3.indb 240 06/02/2011 10:02:52


FSICA II

As leis da refrao Vamos considerar um raio de luz monocromtico que atra-


vessa um prisma de vidro imerso no ar.
1a lei da refrao
O raio de luz incidente, o raio de luz refratado e a linha nor-
mal so coplanares.

2a lei da refrao ( lei de Snell-Descartes)


O produto do ndice de refrao no qual se encontra o raio de
luz pelo seno do ngulo que este raio forma com a linha normal
constante. O desvio angular total sofrido pelo raio de luz atravessar o
n1. sen i = n2 . sen r prisma :
= i1 + i2 A
i1 ngulo de incidncia
Reflexo total ngulo limite i2 ngulo de emergncia
A ngulo de abertura do prisma
A = r1 + r2
r1 ngulo de refrao na face 1
r2 ngulo de refrao na face 2

Disperso da luz
Disperso da luz (branca) consiste na separao das luzes co-
loridas monocromticas que constituem uma luz policromtica.

Quando um raio de luz passa de um meio mas refringente


para um meio menos refringente formando um ngulo de inci-
dncia maior que um certo ngulo limite, este raio sofrer refle-
xo total.

No ar, todos as luzes coloridas propagam-se, praticamente,


com a mesma velocidade. Ao penetrar o bloco de vidro, a luz
violeta sofre a maior desvio, visto que possui o maior ndice de re-
frao, por outro lado, a luz vermelha, por sofrer o menor ndice
de refrao, a que menos se desvia da trajetria inicial.
Dioptro plano A disperso da luz ocorre em virtude davariao do ndice de
o conjunto constitudo por dois meios transparentes e a refrao das diferentes cores para um dado meio
superfcie de separao entre eles.

E X E R C C I O
01. O fundo de uma piscina, para quem olha do lado de fora
dela, parece mais prximo da superfcie da gua, devido :
a) disperso c) refrao
b) difrao d) interferncia

02. Um raio luminoso incide sobre a superfcie da gua, confor-


me a figura a seguir.
A imagem tem natureza virtual e forma-se no mesmo meio Qual alternativa representa o que acontece com o raio?
em que est o objeto.
a) b)
ar
gua
p posio aparente do objeto (imagem)
p posio real do objeto
n ndice de refrao do meio onde se encontra o observador
n ndice de refrao do meio onde se encontra o objeto c) d) e)

Prisma
O prisma tico uma associao de dois planos, cujas super-
fcies diptricas no so paralelas.

UECEVEST 241

Apostilas UECEVEST mod3.indb 241 06/02/2011 10:02:53


F S I C A II

03. O ndice de refrao da gua 4/3 e o do vidro 3/2. A em um prisma ptico, o que caracteriza o fenmeno da disper-
razo entre a velocidade da luz no vidro e a velocidade da luz so. Pelo que se conclui da ilustrao, o prisma de vidro (ou de
na gua ser: material semelhante) e est imerso no ar.
a) 9/8 d) 2

b) e) 8/9

c)

04. (UECE) Um peixe encontra-se a 100cm da superfcie da


gua, na mesma vertical que passa pelo olho do observador. O
ndice de refrao da gua 4/3. A imagem do peixe, conjugada Cada frequncia do espectro da luz branca sofre um desvio dife-
pelo dioptro gua ar e vista pelo observador, : rente na travessia do prisma, permitindo a obteno de um fei-
a) real, situada na gua, profundidade de 75cm. xe policromtico no qual se distinguem as cores fundamentais
b) virtual, situada no ar, 20cm da superfcie da gua. presentes, tambm, num arco-ris. A respeito do fenmeno da
c) virtual, situada na gua, profundidade de 75cm. disperso da luz no prisma, analise as alternativas abaixo e aponte
d) real, situada na gua, profundidade de 4/3 m. a correta:
a) A cor que mais se desvia a violeta, pois ao refratar-se do
05. Um raio de luz incide de um meio menos refringente 1 para ar para o vidro, sofre menor variao de velocidade de
outro mais refringente 2, sendo o ndice de refrao relativo propagao que as demais cores.
Observa-se que parte da luz se reflete e parte se refra- b) A cor que menos se desvia a violeta, pois ao refratar-se do
ta, e que o raio refletido perpendicular ao refratado. O valor do ar para o vidro e do vidro para o ar, sofre maior variao no
ngulo de incidncia : comprimento de onda que as demais cores.
a) 15 d) 60 c) O diferente desvio sofrido por cada uma das cores
b) 30 e) 75 componentes do espectro da luz branca determinado
c) 45 pelo ndice de refrao que o vidro apresenta para cada
frequncia, isto , para a luz violeta ele apresenta maior
06. (ITA) Com respeito ao fenmeno do arco-ris, pode-se afir- ndice de refrao que para a luz vermelha.
mar que: d) Na travessia do prisma, a cor de maior frequncia sofre o
I. Se uma pessoa observa um arco-ris sua frente, ento o Sol menor desvio, enquanto que a cor de menor frequncia sofre
est necessariamente a oeste. o maior desvio.
II. O Sol sempre est direita ou esquerda do observador. e) O desvio sofrido por cada uma das cores componentes
III. O arco-ris se forma devido ao fenmeno de disperso da luz do espectro da luz branca determinado pela variao de
nas gotas de gua. frequncia que cada uma delas sofre na refrao do ar para
Das afirmativas mencionadas, pode-se dizer que: os vidro e do vidro para o ar.
a) todas so corretas;
b) somente I falsa; 10. Um pescador deixa cair uma lanterna acesa em um lago a
c) somente a III falsa; 10,0 m de profundidade. No fundo do lago, a lanterna emite um
d) somente II e III so falsas; feixe luminoso formando um pequeno ngulo q com a vertical
e) somente I e II so falsas. (veja figura).

07. (U.F. Pelotas-RS) Um raio luminoso monocromtico passa


do vcuo para um meio material de ndice de refrao igual a
4/3. Sendo a velocidade de propagao da luz no vcuo igual a
3,00 . 105km/s, podemos afirmar que a velocidade da luz no meio h
material de:
a) 4,00 . 105 km/s d) 2,00 . 105 km/s
b) 2,25 . 10 km/s
5
e) 3,25 . 105 km/s
c) 3,00 . 10 km/s
5


08. (MACKENZIE-SP) Para haver reflexo total, preciso apenas: Considere: tg sen e o ndice de refrao da gua n = 1,33.
a) uma superfcie de separao entre dois meios. Ento, a profundidade aparente h vista pelo pescador igual a
b) que a luz se propague do meio de menor para o de maior a) 2,5 m c) 7,5 m
ndice de refrao. b) 5,0 m d) 8,0 m
c) um espelho plano.
d) que a luz se propague de um meio mais refringente para
um meio menos refringente, com um ngulo de incidncia
superior a um certo ngulo limite.
e) que a luz se propague do meio mais refringente para um
meio menos refringente, produzindo um ngulo de refrao
LENTES ESFRICAS (uece/enem)
superior ao ngulo limite.
Definio de lentes esfricas
09. Um dos discos clssicos do rock, o lbum The Dark Side of A lente esfrica uma associao de dois dioptros na qual pelo
the Moon, do grupo ingls Pink Floyd, lanado em 1973, traz menos um deles , obrigatoriamente, um dioptro esfrico, e o
em sua capa uma bonita figura da luz branca sendo decomposta outro pode ser um dioptro esfrico ou um dioptro plano.

242 UECEVEST

Apostilas UECEVEST mod3.indb 242 06/02/2011 10:02:54


FSICA II

Elementos geomtricos de uma lente esfrica pouco inclinados em relao ao eixo principal e pouco afastados
de seu centro tico.
Representao:

C1 e C2, os centros de curvatura das faces;


R1 e R2, os raios de curvatura das faces;
V1 e V2, os vrtices das faces; Propriedades das lentes esfricas
e, a espessura da lente. Comportamento de alguns raios luminosos ao atravessarem
uma lente esfrica delgada
nomenclatura das lentes esfricas Todo raio luminoso que atravessa uma lente, passando pelo seu
Cita-se em primeiro lugar o nome da face com o maior raio centro tico, no sofre desvio.
da curvatura, em seguida pelo nome da face com menor raio de
curvatura.
Lentes de bordas finas

Todo raio luminoso que incide numa lente, paralelamente ao


seu eixo principal, emerge na direo do foco principal imagem.
Lentes de bordas grossas

Comportamento tico de uma lente esfrica Todo raio luminoso que incide numa lente, numa direo que
Qualquer lente pode ter um comportamento convergente ou passa pelo foco principal objeto, emerge paralelamente ao eixo
divergente, pois tal comportamento no depende s da lente. De- principal.
pende tambm do meio onde se situa a lente.
a) nLENTE > nMEIO

Estudo grfico das lentes esfricas


Faremos a determinao grfica (geomtrica) da imagem de
b) nLENTE < nMEIO pequenos objetos frontais s lentes delgadas.
Objeto real colocado diante de uma lente divergente:

Condies de nitidez de Gauss


Neste curso, estudaremos apenas as lentes esfricas delgadas,
isto , sua espessura muito pequena quando comparada aos
raios de curvatura das faces esfricas.
Vamos considerar que os raios de luz incidentes na lente sero

UECEVEST 243

Apostilas UECEVEST mod3.indb 243 06/02/2011 10:02:55


F S I C A II

O centro tico da lente 5 caso: Objeto colocado entre o foco principal objeto F e o
F ponto de foco principal objeto centro tico O da lente.
Fi ponto de foco principal imagem
A ponto objeto antiprincipal
Ai ponto imagem antiprincipal

Objeto real colocado diante de uma lente convergente.


1 caso: Objeto colocado alm do ponto objeto antiprincipal A:

A imagem obtida de natureza virtual, direta e maior do que


o objeto.

Estudo analtico das lentes esfricas

A imagem obtida est situada entre o foco F e o ponto anti-


principal A e real, invertida e menor que o objeto.

2 caso: Objeto colocado sobre o ponto objeto antiprincipal A.

p abscissa do objeto;
p, abscissa da imagem;
i ordenada da imagem;
o ordenada do objeto;
f distncia focal.
A imagem obtida agora est situada exatamente sobre o ponto Imagem direita o e i tm o mesmo sinal
antiprincipal A e real, invertida e de mesmo tamanho que o Imagem o e i tm sinais opostos
objeto. Lente convergente f > 0
3 caso: o objeto colocado entre o ponto objeto antiprincipal Lente divergente f < 0
A e o foco principal F.
Equao do aumento linear transversal.

i p,
A= =
o p

Equao dos pontos conjugados (equao de Gauss).

1 1 1
= +
f p p,
A imagem est situada alm do ponto antiprincipal A e real,
invertida e maior do que o objeto. Vergncia da lente esfrica delgada
Vergncia (ou convergncia) de uma lente esfrica delgada o
4 caso: objeto colocado sobre o foco principal objeto F. inverso de sua distncia focal.

1
V=
f

Equao dos fabricantes de lentes (equao de Halley)


Equao que relaciona a distncia focal da lente, os raios de
curvatura das faces e os ndices de reflexo absoluta, da lente e do
meio envolvente.

1 nl 1 1
= 1 +
Temos uma imagem imprpria, pois os raios emergentes da f nm R 1 R 2
lente so paralelos.

244 UECEVEST

Apostilas UECEVEST mod3.indb 244 06/02/2011 10:02:57


FSICA II

Associao de lentes esfricas a) espelho convexo.


Num sistema de lentes justapostas, a lente equivalente possui b) raio de curvatura do espelho vale 5cm.
vergncia igual soma algbrica das vergncias das lentes justapostas. c) distncia focal do espelho vale 2,5cm.
d) imagem do objeto virtual.
Vequi = V1 + V2 + V3 + ... e) imagem est situada a 30cm do espelho.

07. Um objeto real colocado diante de uma lente delgada conver-


gente de distncia focal 10cm. A distncia entre o objeto e a lente
de 30cm. A distncia da imagem lente , em cm, igual a:
a) 12cm d) 25cm
b) 15cm e) 30cm
c) 20cm

08. Um objeto real colocado perpendicularmente ao eixo ptico


e a 1,80m de uma lente convergente, d uma imagem real cinco
vezes menor. A distncia focal dessa lente mede:
E X E R C C I O a) 30cm c) 15cm
b) 20cm d) 10cm
01. (MACKENZIE) Considerando uma lente biconvexa cujas
faces possuem o mesmo raio de curvatura, podemos afirmar que: 09. (Fatec 2005) Um espelho esfrico cncavo tem distncia fo-
a) o raio de curvatura das faces sempre igual ao dobro da cal 3,0m. Um objeto de dimenses desprezveis se encontra sobre
distncia focal; o eixo principal do espelho, a 6,0m deste. O objeto desliza so-
b) o raio de curvatura sempre igual metade do recproco de bre o eixo principal, aproximando-se do espelho com velocidade
sua vergnca; constante de 1,0 m/s. Aps 2,0 segundos, sua imagem
c) ela sempre convergente, qualquer que seja o meio envolvente; a) ter se aproximado 6,0m do espelho.
d) ela s convergente se o ndice de refrao do meio b) ter se afastado 6,0m do espelho.
envolvente for maior que o do material da lente; c) ter se aproximado 3,0m do espelho.
e) ela s convergente se o ndice de refrao do material da d) ter se afastado 3,0m do espelho.
lente for maior que o do meio envolvente. e) ter se aproximado 12,0m do espelho.

02. Um objeto, colocado entre o centro e o foco de uma lente 10. (FUND. CARLOS CHAGAS) Uma lente, feita de mate-
convergente, produzir uma imagem: rial cujo ndice de refrao absoluto 1,5, convergente no ar.
a) virtual, reduzida e direita Quando mergulhada num lquido transparente, cujo ndice de
b) real, ampliada e invertida refrao absoluto 1,7, ela:
c) real, reduzida e invertida a) ser convergente;
d) virtual, ampliada e direita b) ser divergente;
c) ser convergente somente para a luz monocromtica;
03. Um objeto O real e retilneo de 20cm de altura posto a d) se comportar como uma lmina de faces paralelas;
60cm de uma lente delgada e convergente de 10cm de distncia e) no produzir nenhum efeito sobre os raios luminosos.
focal. O objeto est perpendicular ao eixo da lente. A imagem de
O _________, __________ e mede ________cm. 11. O fato de uma lente ser convergente ou divergente depende:
Marque a opo que completa, na ordem e corretamente, as la- a) apenas da forma da lente;
cunas do enunciado. b) apenas do meio onde ela se encontra;
a) real, invertida, 4 c) virtual, invertida, 4 c) do material de que feita a lente e da forma da lente;
b) real, direita, 12 d) virtual, direita, 12 d) da forma da lente, do material de que feita a lente e do
meio onde ela se encontra;
04. (Mackenzie 96) Um objeto real colocado sobre o eixo prin- e) n.d.a.
cipal de um espelho esfrico cncavo a 4 cm de seu vrtice. A
imagem conjugada desse objeto real e est situada a 12 cm do
vrtice do espelho, cujo raio de curvatura :
a) 2 cm. d) 5 cm.
b) 3 cm. e) 6 cm.
G A B A R I T O
c) 4 cm. Introduo a optica geomtrica
01. d 02. b 03. d 04. c 05. b
05. Um objeto colocado a uma distncia de 12cm de uma lente
06. d 07. c 08. b 09. d 10. a
delgada convergente, de 8cm de distncia focal. A distncia, em
centmetros, da imagem formada em relao lente : 11. d 12. b 13. e 14. e
a) 24 d) 8
Espelhos esfricos
b) 20 e) 4
c) 12 01. e 02. d 03. b 04. a 05. e
06. c 07. b 08. a 09. e 10. a
06. (Unirio 2000) Um objeto colocado diante de um espelho. 11. c 12. d 13. a
Considere os seguintes fatos referentes ao objeto e sua imagem:
I. o objeto est a 6cm do espelho; Refrao da luz
II. o aumento transversal da imagem 5;
III. a imagem invertida. 01. c 02. e 03. e 04. c 05. d
A partir destas informaes, est correto afirmar que o(a): 06. e 07. b 08. d 09. c 10. c

UECEVEST 245

Apostilas UECEVEST mod3.indb 245 06/02/2011 10:02:58


F S I C A II

Lentes esfricas
01. e 02. d 03. a 04. e 05. a 06. e
07. b 08. a 09. b 10. b 11. d

Referncias Bibliogrficas
Tpicos de Fsica 3: Eletromagnetismo Ricardo Helondoca,
Gualter Jos Biscuola. 18 ed. So Paulo: Saraiva 2001.

Fsica Volume nico Caio Sergio Calcada, Jos Luiz Sampaio

Curso de Fsica Volume Antnio Mximo, Beatriz Alvarenga

246 UECEVEST

Apostilas UECEVEST mod3.indb 246 06/02/2011 10:02:59


P R - V E S T I B U l A R

QUMICA GERAL

Apostilas UECEVEST mod3.indb 247 06/02/2011 10:03:03


Caro(a) Aluno(a),

Para facilitar o acompanhamento de tais contedos, abaixo esto indicadas as nomencla-


turas utilizadas pela UECE e pelo ENEM:
UECE ENEM
Ligaes interatmicas e intermoleculares
geometria molecular
Materiais, suas prioridades e usos.
polaridade de ligaes e de molculas
ligaes intermoleculares
Funes inorgnicas gua
Materiais, suas propriedades e usos
(caractersticas, propriedades e usos dos
Funes inorgnicas (H2, O2, N2, Cl2, S) substancias moleculares).
Relaes da qumica com suas tecnologias, a
sociedade e o meio ambiente.
Materiais, suas propriedades e usos:
Halognios, Metais alcalinos, Au, Fe, Cu, Al.

Apostilas UECEVEST mod3.indb 248 06/02/2011 10:03:04


QUMI C A G E R A L

lIGAES QUmICAS (UECE/EnEm) Ns representamos simplificadamente assim:

ligaes qumicas
Apenas os Gases Nobres (He, Ne, Ar, Xe, Kr, Rn), nas condi-
es ambientes, apresentam tomos estveis isolados, isto , no
unidos a outros tomos. Observe com ateno:
Assim, para entender como funcionam as ligaes entre os Na+ e Cl- so chamados ONS.
tomos, temos que analisar a eletrosfera dos gases nobres, j que Os ons Na+ e Cl- possuem 8 eltrons na ltima camada.
eles so a chave para entender a questo.
Obedecem, portanto, Regra do Octeto.
Esses ons so formados, de modo geral, quando se encon-
Distribuio Eletrnica dos Gases nobres
tram juntos tomos de metais e ametais.
Gs Nobre K L M N O P METAIS + AMETAIS
Hlio 2 Elementos muito Elementos muito
Nenio 2 8 eletropositivos. 1 a 3 eltrons eletronegativos. 5 a 7 eltrons
Argnio 2 8 8 na camada de valncia. na camada de valncia.
Criptnio 2 8 18 8 Tendncia a perder eltrons Tendncia a ganhar eltrons
Xennio 2 8 18 18 8 e formar ctions. e formar nions.
Radnio 2 8 18 32 18 8
Transferncia de eltrons
Tudo indica que possuir uma eletrosfera igual de um gs no-
bre, isto , possuir 8 eletrons a camada de valncia (com exceo ctions + nions
do tomo de Hlio) faz com que o tomo fique estvel. Logo, os
tomos no-estveis se unem uns aos outros a fim de adquirirem composto inico
essa configurao de estabilidade. Concluses
ons positivos so formados quando metais perdem seus el-
ligao inica ou eletrovalente trons da camada de valncia. Embora esta caracterstica seja co-
a ligao que se estabelece atravs de foras de natureza mum a todos os metais, somente os metais representativos al-
eletrosttica entre ons - da o seu nome: ligao inica. canam a configurao de um gs nobre ao perder seus eltrons
ons so tomos ou grupos de tomos carregados eletricamente. de valncia. Os metais de transio tambm formam ons, mas
Quando o on positivo, chamado ction. no seguem obrigatoriamente a regra do octeto e se apresentam
Quando o on negativo, chamado nion. com uma ou mais valncias, o mesmo ocorrendo com alguns
elementos da famlia IVA.
Vamos considerar um tomo de sdio: Os ametais e o hidrognio, na presena de metais, tendem a
11 prtons 11+ 11
Na K L M ganhar eletrons em quantidade suficiente para formar ons nega-
11 eltrons 11- 2 8 1 tivos com configurao eletrnica semelhante dos gases nobres.
=0
Sua configurao no estvel, pois apresenta um eltron na A tabela abaixo mostra, de modo genrico, as valncias de
ltima camada. Estabilizar-se- perdendo 1 eltron. ctions e nions.
IA
Vamos considerar agora um tomo de cloro: ~
1 IIA IIIA IVA VA VIA VIIA
17 prtons 17+ 17
Cl K L M
17 eltrons 17- 2 8 7 Metais de transio +3 ~
4 3
~ ~
2 ~
1
+2
=0 +1 ons positivos +4
+2 +1 +2 +3 +4
Sua configurao tambm no estvel, pois apresenta sete
eltrons na ltima camada. Estabilizar-se- ganhando um eltron.
ons positivos
A estabilidade entre o Na e o Cl pode ser atingida da seguinte +2 +3 +4
maneira:

Exemplo
1. Um elemento X, cujo nmero atmico 12, se combina com
um elemento Y, situado na famlia VA da tabela peridica, e re-
sulta num composto inico cuja frmula provvel ser:
a) XY d) X2Y3
b) XY2 e) X3Y2
c) X2Y

Soluo:
Inicialmente, devemos fazer a distribuio eletrnica do ele-
mento X:
perde
12
X - 1s2
/ 2s2
2p6
/ 3s2
X2+
2e -

UECEVEST 249

Apostilas UECEVEST mod3.indb 249 06/02/2011 10:03:05


Q U M I CA GERAL

Como elemento Y est na famlia VA, ele apresenta 5 eltrons Exemplos:


na sua camada de valncia: a) Brometo de clcio: Ca+2 Br -1 CaBr2
recebe b) xido de potssio: K+1 O -2 K2O
Y (VA) Y3 - c) Sulfeto de brio: Ba+2 S-2 BaS
3e -
Um composto que possuir pelo menos uma ligao inica
Frmula inica: X3Y2 ser considerado composto inico.

Ligaes covalente
2. (FUVEST-SP-MOD) Considere os ons: Ca2+, e OH. So as ligaes em que os dois tomos se unem por meio
A combinao desses ons pode resultar na hidroxiapatita, mineral de emparelhamento de eltrons. Os eltrons que formam o par
presente em ossos e dentes. A frmula qumica pode ser represen- de ligao apresentam spins contrrios, resultando, desse fato,
tada por Cax(PO4)3OH. O valor x nesta frmula : uma fora atrativa de natureza magntica. O par eletrnico pode
a) 1 d) 4 originar-se de um eltron proveniente de cada tomo (covalncia
b) 2 e) 5 normal) ou de um mesmo tomo (covalncia dativa). Quando os
c) 3 dois tomos se unirem, formaro uma molcula.

Soluo: Ligao covalente normal


Como sabemos que a somatria das cargas deve ser igual a Quando cada tomo contribui com um eltron para estabele-
zero e que pela frmula temos: cer o par eletrnico, a ligao covalente normal.
A ligao covalente ocorre, principalmente, entre tomos com
a tendncia de receber eltrons (tomos que possuem quatro ou
x ons Ca2+ x . (+2) mais de quatro eltrons na ltima camada), isto , ela ocorre,
3 ons 3 . (-3) principalmente, com tomos de no-metais.
1 on OH- 1 . (-1) Vamos considerar dois tomos de hidrognio:
Somatria das cargas:
[x . (+2)] + [3 . (-3)] + [1 . (-1)] = 0
2x - 9 - 1 = 0
x=5

Propriedades dos compostos inicos


No possuem molculas e, sim, associaes compactas de
ons. A configurao dos dois no estvel, pois apresentam 1 el-
Nas condies-ambiente so slidos cristalinos. tron na ltima camada.
Apresentam elevados pontos de fuso e ebulio. Ambos se estabilizaro ganhando 1 eltron.
Ora, de que forma ambos podem ganhar um eltron?
A estabilidade entre os dois pode ser atingida da seguinte ma-
neira:

Observao
Os ons se distribuem alternadamente nos vrtices de polie-
dros regulares.

Os tomos de hidrognio ligam-se atravs de um par de el-


trons, chamado ligao covalente, que passar da por diante a
pertencer a ambos os tomos, satisfazendo desse modo suas ne-
cessidades eletrnicas.
Retculo cristalino do Na+ e Cl- Est formada, assim, uma verdadeira molcula.

Para determinarmos a frmula do composto inico, basta co- Observao


nhecermos as cargas dos ons presentes na ligao. Note que, numa molcula, o nmero de prtons igual ao
nmero de eltrons.
Esquema de formulao: A+x B-y AyBx Na molcula, a soma das cargas zero.

Onde: A metal x+ carga do ction Frmulas de Lewis.


B ametal y- carga do nion Podemos representar as substncias moleculares atravs das
chamadas frmulas eletrnicas ou frmulas de Lewis. Essas fr-

250 UECEVEST

Apostilas UECEVEST mod3.indb 250 06/02/2011 10:03:07


QUMI C A G E R A L

mulas so constitudas pelos smbolos dos tomos ao redor dos


quais se dispem os eltrons da camada de valncia.
Exemplos:
H + H H : H
Par de eltrons covalentes
(A + B) Represente na frmula estrutural plana
O par eletrnico pode ser representado por um trao (Fr-
mula de Couper). Ex.:
[ ] [ O = O O ] O3
H Cl H: 1s1
1
Cl : 1s2 2s2 2p6 3s2 3p5 [ ] [ O = S O ] SO2
17

Frmula eletrnica

Propriedades dos compostos covalentes


Frmula estrutural: H + Cl H - Cl Formam molculas, unidades discretas de matria sem carga
eltrica.
CO2 C: 1s2 2s2 2p2 So mais numerosos, pois abrangem quase a totalidade dos
6
O: 1s2 2s2 2p4 compostos orgnicos.
8
Tm P. F. e P. E. baixos, predominando os estados lquido e
Frmula eletrnica: gasoso.

Solubilidade em gua
Solveis: os muito polares
Insolveis: os apolares

Conduo de eletricidade
Frmula estrutural: O = C = O No conduzem: puros
Conduzem: os polares ionizveis
HCN 1
H: 1s1
C: 1s2 2s2 2p2 ligaes simples, dupla e tripla
6
N: 1s2 2s2 2p3 As ligaes covalentes podem ser classificadas quanto ao n-
7
mero de pares eletrnicos compartilhados entre os tomos de
Frmula eletrnica acordo com o quadro.

Ligao
A B 1 par eletrnico entre os tomos. Sigma ()
simples
2 pares eletrnicos entre os Ligao 1 lig. sigma
A=B
tomos dupla 1 lig. p
3 pares eletrnicos entre os Ligao 1 lig. sigma
AB
tomos tripla 2 lig. p
Frmula estrutural: H - C N
Relacionando os elementos das famlias VIIA, VIA, VA, IVA
NOTE BEM com as ligaes covalentes e as possveis coodenadas, podemos
A Molcula de Gs Oxignio (O2) montar o esquema a seguir:
Observou-se experimentalmente que as molculas de gs
oxignio, principalmente na fase de agregao lquida, so for-
FAMLIAS VIIA VIA VA IVA
temente paramagnticas. Devem, portanto, ter eltrons desem-
parelhados. Camada de X X X X
valncia
ligao covalente coordenada ou dativa. 7e ~
6e ~
5e ~
4 e~
um tipo de ligao covalente que ocorre quando um dos
tomos envolvidos j atingiu a estabilidade, com 8 eltrons na ca- Ligaes X X X X
mada de valncia, e o outro tomo participante necessita ainda de covalentes
1 covalente 2 covalentes 3 covalentes
2 eltrons para completar sua camada de valncia. O tomo que 4 covalentes
j adquiriu a estabilidade por meio de ligaes anteriores compar-
tilha um par de eltrons seus com o outro tomo, ainda instvel. X X
X X

TOMOS: A (estvel) B ( necessita de 2 eltrons) Possveis 1, 2 ou at 1 ou at 2 s 1 dativa


ligaes dativas 3 dativas dativas no faz
dativas

A B por no
apresentar
eltrons livres

UECEVEST 251

Apostilas UECEVEST mod3.indb 251 06/02/2011 10:03:11


Q U M I CA GERAL

ligao metlica d) uma ligao tripla e duas ligaes simples.


feita por uma nvem de eltrons semi-livres (teoria da nu- e) duas ligaes duplas e uma ligao simples.
vem eletrnica ou do mar de eltrons).
Segundo esta teoria, o metal seria um aglomerado de tomos, 06. (F. F. C. L. Belo Horizonte-MG) Examinando-se a frmula
nutrons e ctions, mergulhados numa nuvem ou mar de el- eletrnica do cido fosfrico (H3PO4), encontra-se:
trons livres (deslocalizados). a) 4 ligaes inicas.
Ocorre entre: b) 3 ligaes covalentes.
c) 1 ligao dativa.
Metal + Metal d) 2 ligaes dativas.
e) eltrons desemparelhados.
Representamos os metais atravs de seus smbolos sem indicar
a atomicidade que indeterminada, isto , ao invs de represen- 07. (Fatec-SP) A condutibilidade eltrica dos metais explicada
tarmos a unio de tomos de ferro (Fe) por Fen, onde n indica a admitindo-se:
atomicidade indeterminada, utilizamos apenas o seu smbolo Fe. a) ruptura de ligaes inicas.
Outros exemplos: b) ruptura de ligaes covalentes.
Zinco: Zn c) existncia de prtons livres.
Alumnio: Al d) existncia de eltrons livres.
e) existncia de nutrons livres.

08. (UCS) Dentre os seguintes metais, qual NO participa da


composio de ligas metlicas?
a) Ferro. d) Mercrio.
Propriedades dos metais b) Sdio. e) Prata.
Bons condutores de corrente eltrica. c) Ouro.
Alto ponto de fuso e de ebulio.
Brilho caracterstico 09. (UCS) A frmula do carbonato de clcio CaCO3. Nesta
Dcteis e Maleveis substncia h tomos unidos por ligaes:
a) inicas, apenas. d) covalente e metlicas.
b) covalentes, apenas. e) inicas e covalentes.
E X E R C C I O c) metlicas, apenas.

01. (UFSE) Todos os tomos esto com suas eletrosferas iguais s 10. (PUC-SP) Considere as informaes a respeito de uma subs-
de gases nobres na molcula representada por: tncia X:
a) CF d) CF4 I. Em condies ambientais slida.
b) CF2 e) CF5 II. Dissolve-se em gua.
c) CF3 III. Possui alto ponto de fuso.
IV. No estado slido no conduz eletricidade.
02. (F. C. Chagas-BA) Considerando suas posies na tabela V. Conduz eletricidade em soluo aquosa ou quando fundida.
peridica, hidrognio e enxofre devem formar o composto de
O mais provvel que X seja:
frmula:
a) um composto inico que se dissocia em gua.
a) HS d) H2S3
b) um composto molecular polar que se ioniza em gua.
b) HS2 e) H3S2
c) um metal que reage com a gua.
c) H2S
d) uma substncia apolar que se dissocia em gua.
e) um composto molecular polar que se dissocia em gua.
03. (Osec-SP) A ligao covalente estabelecida entre dois ele-
mentos qumicos ser tanto mais polar quanto maior for a dife-
rena entre as x desses elementos.
Completa-se corretamente esta afirmao substituindo x por: G A B A R I T O
a) massas atmicas. d) densidades.
b) eletronegatividades. e) cargas nucleares. 01. d 02. c 03. b 04. d 05. a
c) temperaturas de fuso. 06. c 07. d 08. b 09. e 10. a

04. (Unifor-CE) O elemento qumico metlico A combina-se


com o oxignio formando o composto A2O3. O nmero atmico
do metal pode ser:
a) 12 d) 31
b) 20 e) 37 GEOmETRIA mOlECUlAR (UECE)
c) 30
As molculas so formadas por tomos unidos por ligaes
05. (U. F. Uberlndia-MG) O fosgnio COCl2 , um gs, pre- covalentes e podem apresentar na sua constituio, desde 2 at
parado industrialmente por meio da reao entre o monxido de milhares de tomos. A disposio espacial dos ncleos desses
carbono e o cloro. A frmula estrutural da molcula do fosgnio tomos ir determinar diferentes formas geomtricas para as
apresenta: molculas.
a) uma ligao dupla e duas ligaes simples. Como podemos prever a geometria de uma molcula?
b) uma ligao dupla e trs ligaes simples. Atravs da teoria da repulso dos pares eletrnicos da cama-
c) duas ligaes duplas e duas ligaes simples. da de valncia.

252 UECEVEST

Apostilas UECEVEST mod3.indb 252 06/02/2011 10:03:13


QUMI C A G E R A L

Os pares de eltrons existentes ao redor do tomo central de uma Pode-se determinar a polaridade de uma molcula pelo vetor
molcula, tendero a se afastar ao mximo, pois como todas as cargas momento dipolar resultante ( r ), isto , pela soma dos vetores
so do mesmo sinal (negativo), eles se repelem mutuamente. de cada ligao polar da molcula.
Assim, se houver 2 pares de eltrons ao redor do tomo central,
eles tendero a ficar de lados opostos, graas a esta repulso. Se Molcula apolar : r = 0 Molcula polar : r 0
forem 3 pares, eles iro se dispor nos vrtices de um tringulo equi-
ltero e, se forem 4 pares, eles iro para os vrtices de um tetraedro. Para determinar o vetor r deve-se considerar 2 fatores:
a escala de eletronegatividade, que nos permite determinar a
X X X orientao dos vetores de cada ligao polar;
(Disposio linear ) a geometria da molcula, que nos permite determinar a dispo-
(Disposio trigonal) sio espacial desses vetores. Ex.:
(Disposio tetradrica) H+ - Cl- H Cl r 0
H x Cl Toda molcula biatmica linear H- Cl ( linear)
Molcula Polar
Geometria linear
O= C= O O = C= O (linear)
( 2 pares) (segmento de reta) 180
O-=C+=O OCO r = 0
O
Molcula Apolar vetores se
C Geometria linear cancelam
H H
(3 pares) (tringulo equiltero) Trigonal plana ( 120) N de Existe(m)
tomos par(es) de
em eltrons
Geometria ngulo Exemplos Polaridade
torno do livres no
tomo tomo
central central
02 No Linear-180 CO2; HCN Polar ou apolar
02 Sim Angular-varivel. H2O; SO2 Polar
03 No Trigonal plana-120 BF3; SO3 Apolar
03 Sim Piramidal NH3; PH3 Polar
04 No Tetradrica CH4; SiCl4 Apolar
04 Sim Gangorra SF4 Polar
POlARIDADE DE lIGAES E DE
05 No Bipiramidal triangular PCl5 Apolar
mOlCUlAS (UECE/EnEm) Pirmide de base
05 Sim IF5 Apolar
quadrada
Eletronegatividade 06 No Octadrica SF6 Apolar
tomos de diversos elementos qumicos apresentam diferen-
tes tendncias para atrair os eltrons.
Eletronegatividade a capacidade que os tomos de um de-
terminado elemento possuem para atrair eltrons.
A eletronegatividade uma propriedade peridica. E X E R C C I O
Ordem crescente de eletronegatividade: 01. (FSJT) Se o momento dipolar do cloreto de berlio nulo,
Metais < H = P < C = S = I < Br < Cl = N < O < F (mais qual a estrutura de sua molcula?
eletronegativo) a) linear Cl Cl Be
No se define eletronegatividade para os Gases Nobres, uma b) linear Cl Be Cl
vez que eles no apresentam tendncia a receber ou compartilhar c) angular Cl
eltrons. / \
Cl Be
Ligao entre tomos de d) angular Be
mesma eletronegatividades ligao covalente apolar / \
Ex.: H2, Cl2 ... Cl Cl
e) linear Be Cl Cl
Ligao entre tomos de
diferentes eletronegatividades ligao covalente polar 02. (FMU) Considerando-se os compostos
Ex.: HCl, HF ... 1. SiH4
2. CO2
Toda ligao inica uma ligao polar, e a polarizao 3. CCl4
de uma ligao inica sempre maior do que a de uma ligao 4. HCl
covalente. 5. H2O
As molculas podem ser classificadas quanto a sua polaridade
em 2 grupos: polares ou apolares. Como j vimos, numa mol- Quais molculas so polares?
cula de HF ocorre o aparecimento de plos, e ela recebe o nome a) 3 e 5 d) 1 e 5
de molcula polar. b) 4 e 5 e) 3 e 4
A polarizao da ligao apresenta uma direo, um sentido e c) 2 e 3
uma intensidade. Assim, podemos representar a polarizao por
um vetor. 03. NH3, H2O e CH4 so, respectivamente, molculas:

UECEVEST 253

Apostilas UECEVEST mod3.indb 253 06/02/2011 10:03:16


Q U M I CA GERAL

a) polar, polar, apolar d) polar, apolar, apolar InTERAES InTERmOlECUlARES


b) polar, polar, polar e) apolar, apolar, apolar EnTRE mOlCUlAS (UECE/EnEm)
c) apolar, apolar, polar

As substncias covalentes podem ser slidas, lquidas ou ga-
04. A ligao C H praticamente apolar devido a uma pequena
sosas; isto prova que entre suas molculas podem existir foras de
diferena de eletronegatividade entre os tomos. Nessas condi-
intensidades maiores ou menores. So exatamente essas foras
es, apolar a molcula:
ou ligaes entre molculas que iremos estudar.
a) H3C CH2 OH d) H3C O CH3
b) H3C CH2 NH2 e) H3C CH3
c) H3C CH2 Cl Classificao
Atualmente, as atraes entre molculas so classificadas da
05. (PUC) Qual das substncias abaixo tem molcula apolar line- seguinte forma:
ar e apresenta ligaes duplas? 1 tipo: van der Waals ou Foras de London: (Dipolo indu-
a) HCl b) H2O c) N2 d) CO2 e) NH3 zido - Dipolo induzido).
2 tipo: Associao de dipolos (Dipolo permanente - Dipolo
06. (ITA) Qual das afirmaes falsa? permanente).
a) C6H6 pouco solvel em H2O. 3 tipo: Pontes ou Ligao de Hidrognio.
b) NH3 uma substncia covalente apolar.
c) O iodo mais solvel em CCl4 que em H2O. As ligaes entre molculas comumente ocorrem no estado
d) A molcula Cl2 apolar. lquido ou slido. No estado gasoso mais frequente encontrar-
e) Naftaleno bastante solvel em benzeno. mos molculas isoladas.
Quanto maior for a intensidade das foras intermolecu-
07. (UFC) Fugir da poluio das grandes cidades, buscando ar lares, maior ser a energia necessria para provocar mudanas
puro em cidades serranas consideradas osis em meio fumaa, de estado e, consequentemente maiores sero os PF e PE dos
pode no ter o efeito desejado. Resultados recentes obtidos por compostos moleculares.
pesquisadores brasileiros mostraram que, em consequncia do
movimento das massas de ar, dixido de enxofre (SO2) e di-
xido de nitrognio (NO2) so deslocados para regies distantes
e de maior altitude. Curiosamente, estes poluentes possuem
propriedades similares, que se relacionam com a geometria mo- Estado Slido Estado Lquido Estado Gasoso
lecular. Assinale a alternativa que descreve corretamente essas
propriedades. 1 tipo
a) Trigonal plana; polar, sp3
b) Tetradrica; apolar, sp3 Fora de van der Waals
c) Angular; apolar, sp2 As foras de van der Waals so um tipo de interao da maior
d) Angular; polar, sp2 importncia, uma vez que afetam todos os tipos de tomos e
e) Linear; apolar, sp molculas, sem exceo. So de grande importncia na determi-
nao das propriedades dos lquidos (PE, densidade, viscosidade,
08. (UECE) O hidrxido de clcio, Ca(OH)2 usado na prepara- etc.) e afetam, embora em menor grau, as propriedades dos sli-
o de argamassa (areia + cal) na construo civil, obtido indus- dos e dos gases.
trialmente atravs do seguinte processo qumico: Para tomos como os dos gases nobres e molculas apolares
como as dos hidrocarbonetos, as nicas foras intermoleculares
CaCO3 CaO + CO2 so as de van der Waals. Logo, quanto maior o nmero de el-
CaO + H2O Ca(OH)2 trons de que a molcula dispe, mais polarizvel ser e portanto
maior ser a atrao de van der Waals.
Da pode-se afirmar, corretamente: Para os hidrocarbonetos encontramos um aumento gradual
a) As ligaes qumicas que formam o Ca(OH)2 so somente de ponto de ebulio com o aumento do peso molecular.
inicas
b) A frmula estrutural do CaCO3 : 2 tipo
O
Associao de dipolos ( dipolos permanentes)
Quando dois tomos de diferentes eletronegatividades ligam-
O C
se por covalncia, o partilhamento dos eltrons da ligao feito
de modo desigual, com o tomo mais eletronegativo recebendo
O Ca uma maior parte da densidade eletrnica total da ligao. O par-
c) O hidrxido de clcio um composto apolar tilhamento desigual d origem a dipolos eltricos na ligao. Tais
d) Se o Ca(OH)2 absorver o CO2 do ar atmosfrico, formar dipolos de ligao e de par de eltrons livres recebem o nome
novamente o CaCO3. genrico de momento de dipolo (). Algumas vezes so chama-
dos de dipolos permanentes. Na fase lquida, as molculas ten-
dem a se orientar de modo a permitir atraes mtuas. Como a
movimentao trmica tende a contrabalanar esta tendncia de
G A B A R I T O ordem, um lquido um sistema apenas parcialmente ordenado.
01. b 02. b 03. a 04. e 05. d Uma vez que as atraes ora descritas tm que ser vencidas
06. b 07. d 08. d para que o lquido seja vaporizado, as molculas polares tendem
a apresentar maior ponto de ebulio do que as molculas no
polares equivalentes.

254 UECEVEST

Apostilas UECEVEST mod3.indb 254 06/02/2011 10:03:17


QUMI C A G E R A L

Os dipolos so indicados por setas cuja ponta na direo do central, o que no ocorre no estado lquido. Dessa maneira o
lado negativo: CH3 Br. mesmo nmero de molculas ocupa maior volume no estado s-
Muitas vezes as cargas inicas parciais so representadas pela lido que no lquido. por isso que o gelo menos denso que a
letra : positivo e negativo : +CH3-F -, CH3-O-CH3 . gua lquida. O cristal slido mantido por pontes de hidrognio.

3 tipo

Ponte de hidrognio ou ligao de hidrognio


Quando um tomo de hidrognio liga-se por covalncia a um
tomo mais eletronegativo mantm uma afinidade residual por ou-
tro tomo eletronegativo. Por exemplo, um tomo de hidrognio
pode atuar como uma ponte entre dois tomos de oxignio, ligan-
do-se a um deles por ligao covalente e ao outro por foras ele-
trostticas. Esse tipo de atrao eletrosttica um caso especial da
atrao dipolo- dipolo e tem grande importncia prtica. a cha- Representao esquemtica das
mada ligao hidrognio ou ponte de hidrognio. Usualmente molculas de gua no estado slido
uma ligao hidrognio representada por uma linha interrompida
em vez da linha contnua como fazemos para ligaes covalentes:
H R E X E R C C I O
O O
R H 01. Durante a evaporao da gasolina, so rompidas:
H H a) ligaes covalentes.
O O
R R b) ligaes inicas.
Ligao hidrognio intermolecular c) interaes tipo pontes de hidrognio.
d) interaes tipo dipolo instantneo-dipolo induzido.
A intensidade dessas ligaes (~5 Kcal/mol) muito menor
do que a das ligaes covalentes ( 50 a 100 Kcal/mol) mas muito 02. Gelo seco o nome popular do slido. Nas condies am-
maior do que a das interaes de van der Waals (0,05 a 0,5 Kcal/ bientes esse material sofre sublimao, isto , passa diretamente
mol, para os gases nobres). As ligaes hidrognio mais fortes e do estado slido para o gasoso. Nesse processo so rompidas: CO2
mais importantes so aquelas nas quais os tomos eletronegativos a) ligaes covalentes.
envolvidos so do grupo do O, N ou F. b) ligaes inicas.
Ligaes hidrognio fortes: F-H ---F, O-H---N, F-H---N c) interaes tipo pontes de hidrognio.
Ligaes hidrognio fracas: N-H---O, N-H---N, S-H---S, d) interaes tipo dipolo instantneo-dipolo induzido.
S-H---O, C-H---O
03. (PUC-RS) Na tentativa de se explicar a interao entre tomos
As ligaes hidrognio intermoleculares causam aumen- de cobre e estanho no bronze, emprega-se o modelo de ligao:
tos dramticos nos pontos de ebulio dos lcoois em rela- a) de van der Waals. d) metlica.
o aos pontos de ebulio de compostos de peso molecu- b) covalente polar. e) covalente apolar.
lar semelhante. Por exemplo: CH3CH2CH2OH (PE=98C); c) inica.
CH3OCH2CH3(PE=8C).
Entre os diversos casos de interao molecular, h um que 04. (UCS) Na transformao de gua liquida em vapor dgua h
merece um estudo particular, pois , sem dvida, o mais impor- predominantemente:
tante a ponte de hidrognio ou ligao de hidrognio. a) formao de ligaes intermoleculares.
Examine os valores tabelados: b) formao de ligaes intramoleculares.
c) formao de ligaes por pontes de hidrognio.
Massa Ponto de d) ruptura de ligaes intramoleculares.
Composto N de eltrons
molecular Ebulio (oC) e) ruptura de ligaes por pontes de hidrognio.
H2O 10 18 +100
H2S 18 34 - 61 05. (PUC-SP) Os pontos de ebulio dos halognios so, res-
H2Se 36 81 - 41 pectivamente: flor -118,0 oC; cloro -34,6 oC; bromo 58,8 oC
H2Te 54 129,5 -2 e iodo 184,4 oC. A explicao para esses valores situados numa
faixa to ampla reside na existncia de foras intermoleculares
Hidretos do grupo 6A sensivelmente diferentes nas quatro substncias. As ligaes entre
molculas de halognios se fazem atravs de:
Essa caracterstica da gua explicada pela existncia de pon- a) foras de London, resultantes de momentos dipolares
tes de hidrognio entre as molculas H2O. Essas ligaes so bem instantneos, que diminuem com o aumento do tamanho
mais fracas que as ligaes covalentes entre os tomos de H e O do tomo.
da molcula de H2O. No entanto, so mais fortes do que as inte- b) foras resultantes da existncia de momentos dipolares
raes dipolo-dipolo existentes no H2S ou no H2Se. permanentes, que aumentam com o tamanho do tomo.
Por isso, nas condies ambientes, H2O lquido e os demais c) foras resultantes da existncia de momentos dipolares
hidretos do grupo 6A so gases. permanentes, que diminuem com o tamanho do tomo.
d) foras resultantes do compartilhamento de eltrons
Representao esquemtica das molculas de gua no estado determinando a formao de um enlace covalente.
lquido. e) foras de London, resultantes de momentos dipolares
A gua no estado slido (gelo) forma uma estrutura hexagonal instantneos, que aumentam com o aumento do tamanho
em que as molculas se dispem de tal modo que h um vazio do tomo.

UECEVEST 255

Apostilas UECEVEST mod3.indb 255 06/02/2011 10:03:18


Q U M I CA GERAL

06. A escolha de solventes na indstria depende de suas proprie- Par conjugado cido-base
dades fsicas, as quais esto correlacionadas com a natureza das Par de espcies qumicas que se diferem por nmero de (H+).
interaes intermoleculares de seus constituintes. Com relao a Exemplo: NH3 + H2O NH41+ + OH1-
tais informaes, correto afirmar:
a)Os constituintes das essncias dos perfumes so, em Pares conjugados cido1 / Base1
geral, dissolvidos em lcoois que so compostos polares; cido2 / Base2
trata-se, portanto, de substncias apolares.
b) As substncias gordurosas so, geralmente, apolares e so Espcie anfiprtica ou anftera
removidas pelos detergentes, por meio da interao da Espcies qumicas que podem doar ou receber prtons (H+).
extremidade inica de suas estruturas com as gorduras. Exemplos:
c) Os lcoois, os cidos carboxlicos e as aminas apresentam H2O + HCl H3O + Cl
interaes do tipo ponte de hidrognio; logo, so solveis H2O + NH3 NH4 + OH
em hidrocarbonetos.
d) A previso da polaridade das substncias feita por Teoria de lewis 1923 teoria eletrnica
meio da geometria de suas molculas e da diferena de cido
eletronegatividade entre os tomo ligantes. toda espcie qumica (molcula ou on) capaz de receber
e) As ligaes na molcula do dixido de carbono (CO2) so par de eltrons no compartilhado de outra espcie qumica.
apolares.
Base
toda espcie qumica (molcula ou on) capaz de doar pares
de eltrons no compartilhado a outra espcie qumica.
G A B A R I T O Exemplo: ..
01. d 02. d 03. d 04. e 05. e 06. d H+ + NH3 NH41+
cido de Lewis base de Lewis

Exemplos de cidos e bases de Lewis:


FUnES InORGnICAS I (UECE/EnEm) H1+ , Cl1+ , NO12+ , CH13+ , Cu 2+ , Fe 2+
1444444424444444 3
cidos ction
Teorias sobre cidos e bases
BF3 , AlCl 3, FeCl 3, AlBR 3
1444442444443
Teoria de Arrhenius 1889 teoria da soluo aquosa Molculas
cido So Eletroflicos
toda substncia que, por adio de gua, aumenta a con-
centrao de ons H+, chamado de hidrnio ou hidroxnio. Cl1, I1, F1, CN1, OH1, CH13
144444424444443
Exemplos:
HCl + H2O H3O+ + Cl- Bases nions
NH H 2 O, R NH 2
143,44
424444 3
Base
toda substncia que, por adio de gua, aumenta a con- Molculas
centrao de ons OH-, chamado de hidrxido ou hidroxila. So Nucleoflicos
Exemplos: NaOH, KOH, Ca(OH)2.
Com base nas teorias supracitadas, podemos concluir:
H2O
NaOH(s) Na+(aq) + OH-(aq) cido cido
H2O Arrhenius Brnsted
Ca(OH)2(s) Ca2+(aq) + 2OH-(aq)

Teoria de Brnsted-lowry 1923 teoria protnica.


cido
cido
toda espcie qumica (molcula ou on) que doa prtons Lewis
(H+).
Exemplo: Todo cido de Arrhenius cido de Brnsted e cido Lewis.
+ Todo cido de Brnsted cido de Lewis, mas nem sempre
HCl + H O H O + Cl
2 3
cido de Arrhenius.
Nesta reao o HCl e o H3O+ so cidos, pois doam prtons Um cido de Lewis nem sempre pode ser cido de Brnsted e
(H ).
+ Arrhenius.

Base Determinao terica da fora cida (Pauling)


toda espcie qumica (molcula ou on) que recebe prtons Para determinarmos a fora de um cido sem termos mo
(H+). o valor de , podemos nos guiar pelas seguintes regras prticas.
+
HCl + NH NH + Cl
3 4
Para hidrcidos:
Nesta reao, o NH3 e Cl so bases, pois recebem prtons So fortes: HCl, HBr, HI.
(H ).
+ moderado: HF.
So fracos: HCN, H2S.

256 UECEVEST

Apostilas UECEVEST mod3.indb 256 06/02/2011 10:03:21


QUMI C A G E R A L

Para oxicidos (HxEOy):


E X E R C C I O
nmero de oxignios
nmero de hidrognios ionizveis 01. (PUC) Segundo Brnsted-Lowry, um cido uma base con-
jugada, diferem entre si por:
0 cido Fraco a) um prton d) um par de eltrons
Se (y x) for 1 cido Moderado b) uma hidroxilia e) uma ligao covalente
2 cido Forte c) um hidroxnio
3 cido Forte
02. (UFB) Entre as afirmativas abaixo, relacionadas com cidos e
Exemplos: bases, a nica correta :
HClO4 a) a base conjugada de um cido forte base forte;
H2SO4 b) a base conjugada de um cido fraco uma base forte;
H3PO4 c) um cido e sua base conjugada reagem para formar sal e gua;
H3BO3 d) o cido H2O funciona como a sua prpria base conjugada;
H3PO3 e) n.d.a.
H3PO2
03. (ITA) cido uma substncia capaz de receber 1 par de el-
ATENO! trons. A definio acima corresponde proposta de:
O cido carbnico (H2CO3) uma importante exceo a essa a) Arrhenius d) Lewis
regra. Ele um cido fraco, ao contrrio do que prev a regra b) Brnsted e) Ostwald
(3 2 = 1; moderado). Trata-se de um cido instvel, que sofre c) Lavoisier
decomposio em gs carbnico e gua:
H2CO3 CO2 + H2O 04. Ag+ um cido:
a) de Arrhenius d) nas trs teorias
Classificao das bases b) de Brnsted e) Ag+ no um cido
Os critrios de classificao de uma base so os seguintes: c) de Lewis

Quanto ao nmero de ons hidrxidos: 05. NH3 uma base:


Monobase - possui 1 grupo OH-. a) de Arrhenius d) nas trs teorias
Exemplo: NaOH, KOH etc. b) de Brnsted e) NH3 no uma base
c) de Lewis
Dibase -possui 2 grupos OH-.
Exemplo: Ca(OH)2, Mg(OH)2 etc. 06. (PUC) Um cido de Lewis deve ter:
a)hidrognio ionizvel
Tribase possui 3 grupos OH-. b)oxignio em sua molcula
Exemplo: Al(OH)3, Fe(OH)3 etc. c)baixa densidade eletrnica
d) larga densidade eletrnica
Tetrabase possui 4 grupos OH-. e) carter inico
Exemplo: Pb(OH)4, Sn(OH)4 etc.
07. Entre as bases dadas a seguir, indique quais so praticamente
Quanto solubilidade em gua: insolveis em gua:
Solveis bases de metais alcalinos (1A) e hidrxido de amnio I. KOH;
(NH4OH). II. Mg(OH)2;
Exemplo: KOH, NaOH etc. III. NaOH;
IV. Al(OH)3;
Parcialmente solveis bases dos metais alcalino-terrosos (2A). V. Fe(OH)2;
Exemplo: Mg(OH)2, Ca(OH)2 etc. VI. LiOH
a) V e VI. d) II, IV, V.
Praticamente insolveis todas as demais bases.
b) IV e VI. e) I, III, VI.
Exemplo: AgOH, Fe(OH)3 etc.
c) II, III, IV.
Quanto ao grau de dissociao ():
08. Sabor adstringente o que percebemos quando comemos
O grau de dissociao de uma base o que mede a sua fora.
uma banana verde (no-madura). Que substncia a seguir teria
sabor adstringente?
So chamadas:
a) CH3COOH d) C12H22O11
Bases fortes
b) NaCl e) H3PO4
Os hidrxidos que se dissociam totalmente em soluo. So
c) Al(OH)3
bases fortes os hidrxidos dos metais alcalinos (1A) e metais alca-
lino-terrosos (2A).
09. Considere a ionizao total do cido fosfrico (H3PO4):
Exemplos: KOH, NaOH, Ba(OH)2, etc.
H3PO4 3H1+ + PO43-
Identifique a alternativa que apresenta uma base que, na sua dis-
Bases fracas
sociao total, produz o mesmo nmero de ons, por frmula,
Os hidrxidos que se dissociam apenas ligeiramente em solu-
que o cido fosfrico:
o. So bases fracas os demais hidrxidos (AgOH, Fe(OH)2 etc.)
a) Hidrxido de sdio.
e soluo de hidrxido de amnio (NH4OH).
b) Hidrxido de clcio.

UECEVEST 257

Apostilas UECEVEST mod3.indb 257 06/02/2011 10:03:22


Q U M I CA GERAL

c) Hidrxido de prata. essencial na qumica orgnica, participando de maneira


d) Hidrxido de alumnio. relevante no ciclo energtico dos seres vivos, sendo essencial na
e) Hidrxido de brio. respirao celular dos organismos aerbicos. Um tomo de oxi-
gnio combinado com dois de hidrognio forma uma molcula
10. O mrmore , basicamente, formado de carbonato de clcio de gua.
(CaCO3) e, quando extremamente puro, apresenta a cor branco- Na sua forma molecular, O2, um gs a temperatura ambien-
neve. o caso do mrmore carrara usado por Michelangelo em te, incolor, inspido, inodoro, comburente, mas no combustvel
suas esculturas. A funo qumica a que pertence o carbonato de e pouco solvel em gua.
clcio : Existe, tambm, uma forma molecular formada por trs
a) Base. d) xido cido. tomos de oxignio, O3, denominada oznio, cuja presena na
b) Sal. e) xido bsico. atmosfera protege a Terra da incidncia de radiao ultravioleta
c) cido. procedente do Sol.

Caractersticas principais
Nas condies normais de presso e temperatura o oxignio
G A B A R I T O se encontra no estado gasoso formando molculas diatmicas
01. a 02. b 03. d 04. c 05. c de formula O2. formado durante a fotossntese das plantas e,
posteriormente utilizado pelos animais na respirao. O oxignio
06. c 07. d 08. c 09. d 10. b
lquido e slido tem uma ligeira colorao azulada e, em ambos
os estados, muito paramagntico. O oxignio lquido se obtm
usualmente a partir da destilao fracionada do ar lquido, junto
com o nitrognio. Reage praticamente com a totalidade dos me-
CARACTERSTICAS, PROPRIEDADES E tais, exceto com os metais nobres como ouro, platina e outros,
USOS DAS SUBSTnCIAS mOlECUlARES provocando a corroso. incolor. O oxignio tem trs istopos
estveis e dez radioativos. Todos os radioistopos do oxignio tem
(H2, O2, n2, Cl2, S) (EnEm) uma meia-vida de menos de trs minutos.
O Oznio (O3) se forma mediante descargas eltricas a partir do
HIDROGnIO oxignio molecular (durante as tormentas eltricas, por exemplo).
O hidrognio o primeiro elemento da Tabela Peridica e No oxignio lquido j foi encontrado, em pequenas quanti-
apresenta caractersticas nicas. O hidrognio bastante reativo. dades, uma dupla molcula de oxignio: (O2)2.
Por exemplo, ele queima no ar formando gua, em uma reao
com o oxignio e libera grande quantidade de energia. Abundncia e obteno
A estrutura do tomo de hidrognio se assemelha de certo o elemento mais abundante da crosta terrestre (estimado em
modo a dos metais alcalinos e tambm a dos halognios. Em 46,7%) e, dos oceanos (em torno de 87% como componente da
muitas reaes os halognios adquirem um eltron formando gua). o segundo em abundncia na atmosfera (cerca de 20%).
ons negativos. No tpico do hidrognio formar on negativo, Os xidos de metais, silicatos (SiO44-) e carbonatos (CO32-) so
embora ele forme hidretos inicos M+ H- (por exemplo LiH e encontrados com frequncia em rochas e no solo. Na atmosfera
CaH2) com alguns poucos elementos eletropositivos. existe como oxignio molecular, O2, dixido de carbono (CO2)
e, em menor proporo como monxido de carbono (CO), oz-
Istopos de hidrognio nio (O3), dixido de nitrognio (NO2), monxido de nitrognio
O hidrognio natural constitudo por trs istopos: o prtio (NO), dixido de enxofre (SO2) e outros.
que corresponde a 99,986 % 11H, o deutrio 0,014% do istopo Nos planetas exteriores mais afastados do Sol e em cometas
2
1
D e o trtio 7 x 10-16% do istopo 31T, sendo que as proprie- se encontra gua congelada e outros compostos de oxignio. Por
dades do hidrognio so essencialmente as propriedades de seu exemplo, no planeta Marte h dixido de carbono congelado.
istopo mais leve. O espectro deste elemento tambm foi detectado em algumas
poucas estrelas.
Usos do hidrognio
A obteno feita a partir da destilao fracionada por baixa
Em maaricos oxignio-hidrognio para cortar ou soldar temperatura do ar atmosfrico (criogenia). No mesmo processo
metais, pode-se chegar a temperatura de 3000oC. so obtidos o nitrognio e o argnio.
Na produo de amnia na indstria pelo processo de Haber.
Na hidrogenao de leos vegetais com a obteno de gorduras. Ao biolgica
Assim como utilizado na produo de margarina, como O oxignio respirado pelos organismos aerbicos, liberado
mostra a reao abaixo: pelas plantas no processo de fotossntese, participa na converso
CH3.(CH2)n.CH = CH.COOH + H2 CH3.(CH2)n.CH2.CH2.COOH de nutrientes em energia intracelular.
Usado tambm para reduzir nitrobenzeno anilina (na in- A reduo do nvel de oxignio provoca a hipoxemia e, a falta
dstria de corantes), e na reduo cataltica do benzeno (a pri- total ocasiona a anoxia, podendo provocar a morte do organismo.
meira etapa na produo do nilon-66) A hemoglobina o pigmento que d a cor aos glbulos ver-
Na produo do metanol: melhos (eritrcitos) e tem a funo vital de distribuir o oxignio
catalisador pelo organismo.
CO + 2H2 CH3OH
Precaues
OXIGnIO O oxignio pode ser txico a elevadas presses parciais.
O oxignio representa aproximadamente 20% da composi- Alguns compostos de oxignio como o oznio, o perxido
o da atmosfera terrestre. um elemento qumico de smbolo de hidrognio e radicais hidroxila so muito txicos. O corpo
O, nmero atmico 8 (8 prtons e 8 eltrons) com massa at- humano desenvolveu mecanismos de proteo contra estas es-
mica 16 u. pcies txicas. Por exemplo, a glutacin atua como antioxidante,

258 UECEVEST

Apostilas UECEVEST mod3.indb 258 06/02/2011 10:03:22


QUMI C A G E R A L

como a bilirrubina que um produto derivado do metabolismo Considera-se que foi descoberto formalmente por Daniel Ru-
da hemoglobina. therford em 1772 ao determinar algumas de suas propriedades. En-
As atmosferas ricas em oxignio, em presena de materiais tretanto, pela mesma poca, tambm se dedicaram ao seu estudo
combustveis, so suscetveis de provocar incndios que se propa- Scheele que o isolou, Cavendish, e Priestley. O nitrognio um gs
gam com grande rapidez, podendo provocar exploses. Do mesmo to inerte que Lavoisier se referia a ele como azote (zoe), que uma
modo, pode ocorrer quando existem fontes de oxignio como clo- palavra francesa que significa imprprio para manter a vida. Al-
ratos, percloratos, dicromatos e outros. Estes compostos com alto guns anos depois, em 1790, foi chamado de nitrognio, por Chatpal.
poder oxidante podem, tambm, provocar queimaduras qumicas. Foi classificado entre os gases permanentes desde que Faraday
no conseguiu torn-lo lquido a 50 atm e -110 C. Mais tarde,
Caractersticas principais em 1877, Pictet e Cailletet conseguiram liquefaz-lo.
Nome, smbolo, nmero atmico Oxignio, O, 8 Alguns compostos de nitrognio j eram conhecidos na Idade
Mdia: os alquimistas chamavam deaqua fortis o cido ntrico e
No-metal, representativo
Classe, srie qumica aqua regia a mistura de cido ntrico e clordrico, conhecida pela
(Calcognio)
sua capacidade de dissolver o ouro.
Grupo, perodo, bloco 16 (VIA), 2, p
Densidade, dureza 1,429 kg/m3 (273K), (ND) Caractersticas principais
Aparncia e cor Incolor Como elemento (N) tem uma elevada eletronegatividade ( 3
na escala de Pauling ) e 5 eltrons no nvel mais externo (camada
Propriedades fsicas de valncia), comportando-se como on trivalente na maioria dos
Ponto de fuso 50,35 K (-222,8 C) compostos que forma. Condensa a aproximadamente 77 K (-196
Ponto de ebulio 90,18 K (-182,97 C) C) e solidifica a aproximadamente 63 K (-210 C).
Ocorre como um gs inerte (N2), no-metal, incolor, inodoro
Volume molar 17,36 10-6 m3/mol e inspido, constituindo cerca de 4/5 da composio do ar atmos-
Presso de vapor no definida frico, no participando da combusto e nem da respirao.
Velocidade do som 317,5 m/s (293 K)
Entalpia de vaporizao 3,4099 kJ/mol Caractersticas gerais
Entalpia de fuso 0,22259 kJ/mol Nome, smbolo, nmero atmico Nitrognio (Azoto), N, 7
Estado da matria gs (paramagntico) No-metal , representativo
Classe, srie qumica
(famlia do nitrognio)
Istopos mais estveis Grupo, perodo, bloco 15 ( VA ), 2, p
iso AN meia-vida MD ED (MeV) PD Densidade, dureza 1,2506 kg/m3 (273K), (ND)
16
O 99,762% estvel com 8 nutrons Aparncia e cor Incolor
17
O 0,038% estvel com 9 nutrons
18
O 0,2% estvel com 10 nutrons Propriedades fsicas
Volume molar 13,5410-6 m3/mol
Aplicaes Ponto de fuso 63,15 K (-210,15 C)
A principal utilizao do oxignio como oxidante devido a
Ponto de ebulio 77,35 K (-196,15 C)
sua elevada eletronegatividade, superada somente pela do flor.
Por isso, o oxignio lquido usado como comburente nos mo- Entalpia de vaporizao 2,7928 kJ/mol
tores de propulso dos foguetes, apesar de que nos processos in- Estado da matria gasoso
dustriais o oxignio para a combusto obtido diretamente do ar. Entalpia de fuso 0,3604 kJ/mol
Outras aplicaes industriais so na soldadura e na fabricao Velocidade do som 334 m/s (298,15 K ou 0 C)
de ao e metanol. Presso de vapor no definida
A medicina usa o oxignio administrando-o como suplemen-
to em pacientes com dificuldades respiratrias. Istopos mais estveis
Tambm engarrafado para ser respirado em diversas prticas
desportivas como, por exemplo, o submarinismo ou em ativida- iso AN meia-vida MD ED (MeV) PD
des profissionais para acessar a locais de pouca ou nenhuma ven-
tilao ou em atmosferas contaminadas. O oxignio provoca uma
13
N sinttico 9,965 minutos 2 200 C
13

resposta de euforia quando inalado. No sculo XIX era utilizado 14


N 99,634% estvel
misturado com o xido nitroso como analgsico. 15
N 0,366% estvel
Atualmente, esta mistura ressurgiu para evitar a dor em tra-
tamentos dentrios. O Oxignio-15, radioativo com emisso Abundncia e obteno
de psitron usado no exame PET do crebro em medicina O nitrognio o componente principal da atmosfera terrestre
nuclear. ( 78,1% em volume ). obtido, para usos industriais, pela des-
tilao do ar lquido. O elemento est presente na composio
nITROGnIO de substncias excretadas pelos animais, usualmente na forma de
Nas condies ambientes (25 C e 1 atm) o nitrognio en- ureia e cido rico.
contrado no estado gasoso, obrigatoriamente em sua forma mole- Tem-se observado compostos que contm nitrognio no es-
cular biatmica (N2), formando cerca de 78% do ar atmosfrico. pao exterior. O istopo 14N se cria nos processos de fuso nu-
Alm do nome nitrognio, chamado ainda de azoto em clear das estrelas.
pases que seguem o portugus europeu. um elemento qumico
com smbolo N, nmero atmico 7 e nmero de massa 14 (7 Aplicaes
prtons e 7 nutrons), representado no grupo (ou famlia) 15 A mais importante aplicao comercial do nitrognio na
(antigo 5A) da tabela peridica. obteno do gs amonaco pelo processo Haber. O amonaco

UECEVEST 259

Apostilas UECEVEST mod3.indb 259 06/02/2011 10:03:23


Q U M I CA GERAL

usado, posteriormente, para a fabricao de fertilizantes e cido cloro gasoso amarelo-esverdeado e se combina facilmente com
ntrico. usado, devido a sua baixa reatividade, como atmosfera quase todos os outros elementos.
inerte em tanques de armazenamento de lquidos explosivos, du-
rante a fabricao de componentes eletrnicos (transistores, dio- Aplicaes
dos, circuitos integrados, etc.) e na fabricao do ao inoxidvel. O cloro um composto qumico importante na purificao
O nitrognio lquido, obtido pela destilao do ar lquido, se usa de gua, em desinfetantes, gua sanitria e no gs de mostarda.
em criogenia, j que na presso atmosfrica condensa a -196 C. O cloro tambm amplamente utilizado na fabricao de
Outra aplicao importante o seu uso como fator refrigerante, muitos produtos e itens direta ou indiretamente, ou seja, na pro-
para o congelamento e transporte de alimentos, conservao de duo de produtos de papel, anti-sptico, corantes, alimentos, in-
corpos e clulas reprodutivas sexuais e femininos ou quaisquer seticidas, tintas, produtos de petrleo, plsticos, medicamentos,
outras amostras biolgicas. txteis, solventes e muitos outros produtos de consumo.
Entre os sais do cido ntrico esto includos importantes usado para matar as bactrias e outros micrbios de abaste-
compostos como o nitrato de potssio (nitro ou salitre empre- cimento de gua potvel.
gado na fabricao de plvora) e o nitrato de amnio como fer- O cloro est envolvido no encalhe de polpa de madeira para
tilizante. fabricao de papel, usado industrialmente na gua sanitria e
Os compostos orgnicos de nitrognio como a nitroglicerina tambm para remover tinta de papel reciclado.
e o Trinitrotolueno (TNT) so muito explosivos. A hidrazina e O cloro possui propriedades de um composto orgnico quan-
seus derivados so usados como combustvel em foguetes. do substitudo por hidrognio (borracha sinttica), por isso
Na medicina nuclear, o 13N (l-se nitrognio 13), radioativo amplamente utilizados em qumica orgnica, na produo de clo-
com emisso de positron, usado no exame PET. rato, clorofrmio, tetracloreto de carbono, e na bromo extrao.

Aes biolgicas O cloro no ambiente


O nitrognio o componente essencial dos aminocidos e Na natureza, encontrado apenas combinado com outros
dos cidos nuclicos, vitais para os seres vivos. As leguminosas elementos, principalmente de sdio sob a forma de sal comum
so capazes de desenvolver simbiose com certas bactrias do solo (NaCl), mas tambm na carnalita, e silvinita. Cloretos com-
chamadas de Rizbios. Estas bactrias absorvem o azoto direta- pem grande parte do sal dissolvido nos oceanos da Terra: cerca
mente do ar, sendo este transformado em amonaco que logo de 1,9% da massa de gua do mar de ons cloreto.
absorvido pela planta. Na planta o amonaco reduzido a nitrito A quantidade de cloreto no solo varia de acordo com a dis-
pela enzima nitrito redutase e logo em seguida reduzido a ni- tncia do mar. A mdia nos solos superior cerca de 10 ppm. As
trato pela enzima nitrato redutase. O nitrato posteriormente plantas contm quantidades diferentes de cloro, uma microu-
utilizado pela planta para formar o grupo amino dos aminocidos trient essenciais para as plantas superiores, onde se concentra nos
das protenas que, finalmente, se incorporam cadeia trfica. Um cloroplastos.
bom exemplo deste processo observado na soja, sendo esta uma
cultura que dispensa adubao nitrogenada. Os efeitos do cloro na sade
O cloro um gs altamente reativo. um elemento que
CLORO ocorre naturalmente. Os maiores utilizadores de cloro so em-
presas que fazem dicloreto de etileno e outros solventes clorados,
Propriedades Qumicas do Cloro policloreto de vinila (PVC), resinas, clorofluorcarbonos e xido
Nmero atmico 17 de propileno. Empresas de papel usam cloro para branquear o
papel. gua e estaes de tratamento de guas residuais usam o
Massa atmica 35,453 g.mol -1 cloro para reduzir os nveis de microrganismos na gua que po-
Eletronegatividade de acordo 3.0 dem transmitir doenas aos seres humanos (desinfeco).
com Pauling3.0 A exposio ao cloro pode ocorrer no local de trabalho ou
Densidade 3.21*10 -3 g.cm -3 at 20 C no ambiente aps a sua emisso para o ar, gua ou terra. Pessoas
3,21 * 10 -3 g.cm -3 a 20 C que usam produtos contendo cloro e substncias qumicas em
Ponto de fuso -101 C -101 C piscinas no so normalmente expostas ao cloro em si. O cloro
Ponto de ebulio -34.6 C -34,6 C geralmente encontrados apenas em ambientes industriais.
O cloro entra no corpo quando respirado com o ar contami-
Raio de Van der waals 0.127 nm 0,127 nm
nado ou quando consumidos com alimentos ou gua contami-
Raio Inico 0.184 (-2) nm ; 0,184 nm (-2); 0,029 nm (+6) nados. Ele no permanece no corpo, devido sua reatividade.
0.029 nm (+6) O efeito do cloro na sade humana depende da quantidade
Isotopes Istopos4 4 de cloro que est presente, bem como a durao e a frequncia
Distribuio Eletrnica [Ne] 3s 2 3p 5 [Ne] 3s 2 3p 5 de exposio.
Respirar pequenas quantidades de cloro por perodos curtos
O cloro foi descoberto em 1774 por Carl Wilhelm Scheele, de tempo afeta o sistema respiratrio humano. Os efeitos variam
que erroneamente pensava que continha oxignio. Seu nome foi de tosse e dor no peito, a reteno de gua nos pulmes. O cloro
dado por Humphry Davy, em 1810, que insistiu que ele era na irrita a pele, os olhos e o sistema respiratrio. Estes efeitos no
realidade um elemento qumico. so susceptveis de ocorrer em nveis de cloro, que normalmente
O cloro puro tem a forma fsica de um gs diatmico verde. so encontrados no meio ambiente.
O nome cloro derivado da chloros, o que significa verde, referin-
do-se a cor do gs. O gs de cloro duas vezes e meia mais pesado Os efeitos ambientais do cloro
que o ar, tem um odor desagradvel intensamente sufocante, e O cloro se dissolve quando misturado com gua. Ele tam-
extremamente venenoso. Em sua forma lquida e slida um bm pode escapar da gua e entrada na atmosfera sob certas con-
oxidante poderoso, agente de branqueamento e de desinfeco. dies. A maioria dos lanamentos diretos de cloro para o meio
Este elemento uma parte da srie de halognio formando ambiente so atravs do ar e da superfcie da gua.
sais. extrado de cloretos atravs da oxidao e eletrlise. O Uma vez no ar ou na gua, o cloro reage com outros produtos

260 UECEVEST

Apostilas UECEVEST mod3.indb 260 06/02/2011 10:03:23


QUMI C A G E R A L

qumicos. Ele se combina com material inorgnico na gua para O mtodo Frasch consiste em cavar poos, introduzindo-se
formar sais de cloreto e com o material orgnico na gua para nas rochas sedimentares trs tubos concntricos. Pelo tubo exter-
formar produtos qumicos orgnicos clorados. no injetada gua superaquecida, destinada a fundir o enxofre
Plantas e animais no so susceptveis ao armazenamento de em seu depsito subterrneo; no tubo de menor dimetro injeta-
cloro. No entanto, estudos de laboratrio mostram que a exposi- se ar sobre forte presso para forar o enxofre lquido a subir
o repetida ao cloro no ar podem afetar o sistema imunolgico, superfcie, atravs da tubulao intermediria.
o sangue, o corao e o sistema respiratrio dos animais. Em alguns pases o enxofre retirado dos gases destilados do
O cloro provoca danos ambientais em baixos nveis. O clo- petrleo, gases residuais da fabricao de coque e da hulha, desti-
ro especialmente prejudicial para os organismos que vivem na laes de folhetos pirobetuminosos, e de minrios como a pirita
gua e no solo. ferrfera e a anidrita.
Aproximadamente oitenta por cento do enxofre produzido no
EnXOFRE mundo consumido na fabricao de acido sulfrico. A grande
Gregos e romanos usavam o enxofre em fumigaes e tam- importncia do acido sulfrico prende-se ao fato de ser empregado
bm se valiam dos vapores de sua combusto no branqueamento em quase todos os ramos do processo industrial, como na
de tecidos. Misturado com salitre e carvo, foi empregado no fabricao de fertilizantes, de cidos e sais minerais, de corantes
fabrico da plvora pelos chineses do sculo XI e introduzido na e de inseticidas, assim como em explosivos, em metalurgia, no
Europa por volta do sculo XIV. refino de petrleo, como catalisador em numerosas indstrias de
O enxofre um elemento qumico no metlico, de smbolo produtos orgnicos, na indstria petroqumica, etc
S, pertencente ao grupo VIa da tabela peridica (grupo dos cal- Propriedades fsicas e qumicas do enxofre:
cognios), juntamente com o oxignio, o selnio, o telrio e o Nmero atmico: 16
polnio.
Peso atmico: 32,064
Propriedades e ocorrncia:
O enxofre apresenta a propriedade conhecida como alotropia, Ponto de fuso: forma amorfa 50 C
isto , pode cristalizar-se de maneiras diferentes. A forma alotr- Ponto de fuso: forma cinza 112,8 C
pica mais estvel temperatura ambiente o enxofre ortorrm- Ponto de ebulio: 444,6 C
bico (S), devido sua estrutura cristalina octadrica. Apresenta Densidade (20 C):
colorao de um amarelo caracterstico, frgil e insolvel em
gua, mais solibiliza em dissulfeto de carbono. O enxofre mo- enxofre monoclnico: 1,96g/cm3
noclnico (S), segunda variedade alotrpica desse elemento, enxofre ortorrmbico: 2,07g/cm3
estvel entre 95,5 e 119 C, e obtm-se a partir do vagaroso res- Estados de oxidao: -2, +4, +6
friamento do enxofre fundido. Configurao eletrnica: 1s22s22p63s23p4
O enxofre encontrado na natureza em estado livre ou em
compostos, sobretudo na forma de sulfetos e sulfatos. Existem de-
psitos de enxofre livre nas regies vulcnicas, onde ocorre em nas CARACTERSTICAS, PROPRIEDADES E
fissuras das lavas ou em crateras de vulces extintos (tipo solfatri- USO DOS HAlOGnIOS (EnEm)
co). A ao qumica de materiais betuminosos ou ao redutora de
microrganismos sobre o sulfato de clcio gera o enxofre tipo gesso. Os halognios (do grego hal, sal, e gen, produzir), famlia
O enxofre de natureza vulcnica representa menos de 5% da dos elementos qumicos no metlicos flor, cloro, bromo, iodo
populao mundial, que provm quase totalmente dos depsitos e astato, compem o grupo VIIA (17) da tabela peridica. Esses
sedimentares tercirios, deformados pela intruso do sal, como elementos apresentam comportamento qumico muito parecido
acontece na Louisiana e no Texas, nos Estados Unidos, e em Vera e formam compostos com propriedades semelhantes. Todos pro-
Cruz, no istmo de Tehuantepec, no Mxico. Nesse tipo de dep- duzem sais de sdio, entre os quais o cloreto de sdio, ou sal de
sito o enxofre encontrado impregnando calcrios porosos, na cozinha, o mais conhecido.
periferia dos domos salinos algumas vezes tambm petrolferos. A capacidade de reao ou de combinao com outros ele-
Os principais compostos naturais de enxofre so a galena (sul- mentos to grande nos halognios que raramente so encontra-
feto de chumbo), a blenda (sulfeto de zinco), o cinbrio (sulfeto dos em estado livre na natureza. Aparecem sobretudo na forma
de mercrio), a estibinita (sulfeto de antimnio), a pirita de cobre de sais dissolvidos na gua do mar, ou em extensos depsitos sa-
(sulfeto de cobre), o sulfato de clcio (gesso) e o espato pesado linos originados em eras geolgicas remotas pela evaporao de
(sulfeto de brio). O enxofre tambm encontrado em tecidos e mares interiores. A nica exceo o astato, que se apresentam
estruturas animais e vegetais como a cebola, o alho, a mostarda, na forma de istopos radioativos de vida efmera, razo pela qual
muitos leos, o plo, os ovos, as protenas. Nas guas de muitas ocorre naturalmente.
fontes, e na forma de gases sulforosos. Em condies ambientais normais, o flor e o cloro ocorrem
em estado gasoso; o iodo e o astato, em estado slido; e o bromo,
Extrao e aplicaes em estado lquido. Outras propriedades fsicas, como ponto de
Quando o enxofre de natureza sedimentar, pode ser extra- fuso e ebulio, densidade e peso atmico, variam em ordem
do por processos diversos. O proveniente da Siclia, por exemplo, crescente do flor para o iodo. O calor especfico quantidade
obtm-se aquecendo o minrio em fornos especiais, para vola- de calor necessria para aquecer em 1 C um grama de substncia
tiliz-lo e recolh-lo em cmaras de sublimao, em estado de varia de forma inversa.
pureza. Quando o enxofre se acha infiltrado em formaes muito A caracterstica qumica fundamental dos halognios seu
profundas, como ocorre no golfo do Mxico, capeado por cama- poder como agente oxidante. Essa caracterstica permite que o
das de areia sem coeso, de lodo, e de arenito com petrleo e gs tomo do halognio aceite mais um eltron em sua configurao,
natural, no so possveis de aplicao os mtodos tradicionais de para atingir um arranjo eletrnico mais estvel. No processo, o
minera por poos e galerias. Emprega-se, nesse caso, o mtodo tomo adquire carga eltrica negativa e torna-se um on. Os ons
desenvolvido por Herman Flasch, que deu grande impulso ao haleto, relativamente grandes e incolores, possuem alta estabili-
desenvolvimento das indstrias que dependem do enxofre. dade, especialmente no caso dos fluoretos e cloretos. A energia

UECEVEST 261

Apostilas UECEVEST mod3.indb 261 06/02/2011 10:03:23


Q U M I CA GERAL

de ionizao dos halognios, energia necessria para remover um A colorao diferente da chama deve-se a excitao dos eltrons,
eltron do elemento, mxima no flor e mnima no iodo. O que passam de um subnvel a outro mais alto por causa da ener-
astato, devido natureza radioativa, forma poucos compostos, gia (E) fornecida pela chama. Ao retornar ao seu nvel inicial, o
que so instveis. eltron emite energia na forma um onda de luz de determinado
Nas condies normais, o flor (F2) um gs amarelo; o cloro comprimento, que corresponde a uma certa cor do espectro. Essa
(Cl2) um gs amarelo-esverdeado; o bromo (Br2) um lquido energia se relaciona com o nmero de onda atravs da relao
castanho que passa facilmente a vapor; o iodo (I2) uma substn- de Einstein:
cia slida cor vileta-escuro e com brilho metlico. E=h
As solues aquosas de iodo apresentam uma colorao par-
da. O iodo bastante solvel em lcool, apresentando colorao Abaixo esto os elementos listados com as cores correspon-
semelhante a da soluo aquosa. Em solventes no polares ou dentes e seus comprimentos de onda.
pouco polar, o iodo dissolve-se conservando a cor violeta prpria Cor Comprimento de onda (nm)
das molculas de I2. Em presena de amido, o iodo d uma co-
Li Vermelho-carmim 670,8
lorao azul.
O bromo forma solues cuja cor varia de dourada a marrom, Na Amarelo 589,2
dependendo da sua concentrao. A soluo aquosa de bro- K Violeta 766,5
mo chamada gua de bromo. Rb Vermelho-violeta 780,0
A gua de cloro uma soluo do cloro em gua. O cloro Cs Azul 455,5
reage lentamente com a gua de acordo com a seguinte equao:
Cl2 + H2O HClO + HCl
Caractersticas, Propriedades e
A molcula de F2 tem uma atrao por eltrons to forte, que uso dos ELEMENTOS: Ouro, Ferro,
se torna perigoso o trabalho com o elemento. Ele reage violenta- Cobre, Alumnio (ENEM)
mente com gua, vidro quente e com a maioria dos metais. Flor
normalmente armazenado em tanques de metal, uma ligao Liga metlica
metlica com Ni, Cu e Fe. Estes so um dos poucos metais no Apesar da grande variedade de metais existentes, a maioria
atacados pelo F2. no empregada em estado puro, mas em ligas com propriedades
alteradas em relao ao material inicial, o que visa, entre outras
Caractersticas, Propriedades e coisas, a reduzir os custos de produo.
As indstrias automobilsticas, aeronuticas, navais, blicas e
uso dos Metais Alcalinos (ENEM) de construo civil so as principais responsveis pelo consumo
de metal em grande escala. So tambm representativos os setores
Os elementos do Grupo I possuem uma qumica bastante ho- de eletrnica e comunicaes, cujo consumo de metal, apesar de
mognea, mostrando de modo bem claro os efeitos do tamanho quantitativamente inferior, tem importncia capital para a eco-
crescente dos tomos em suas propriedades fsicas e qumicas. Por nomia contempornea. Ligas metlicas so materiais de proprie-
terem bastantes semelhanas possuem talvez a qumica menos des- dade semelhantes s dos metais e que contm pelo menos um
complicada de toda a tabela peridica. Todos so metais formam metal em sua composio. H ligas formadas somente de metais
ctions monovalentes e so bons condutores de eletricidade, moles e outras formadas de metais e semimetais (boro, silcio, arsnio,
e muito reativos. Isso pode se justificar por possurem na cama- antimnio) e de metais e no-metais (carbono, fsforo).
da de valncia em eltron fracamente ligado ao tomo, formando interessante constatar que as ligas possuem propriedades
assim compostos inicos. Seus hidrxidos so bases muito fortes. diferentes dos elementos que as originam. Algumas propriedades
O Ltio difere razoavelmente dos outros elementos do grupo, so tais como diminuio ou aumento do ponto de fuso, au-
como em todos os grupos tpicos tabela peridica, o primeiro mento da dureza, aumento da resistncia mecnica.
elemento apresenta certas diferenas em relao aos demais. O Ligas metlicas mais comuns no cotidiano:
sdio e o potssio so o stimo e oitavo elementos mais abun- Ao constitudo por Fe e C.
dantes da crosta terrestre em peso e ocorrem dissolvidos como Ao inoxidvel constitudo por Fe, C, Cr e Ni.
sais em grande quantidade na gua do mar. O cloreto de sdio o Ouro de Joias constitudo por Au (75%), Ag e/ou Cobre
composto mais usado, sendo notria a sua importncia e extra- (25%) para o ouro 18K. O ouro 24K ouro puro.
do principalmente da gua do mar. Quanto aos outros, Rubdio, Amlgama dental (utilizada em obturao) constituda por
Csio e Frncio no possuem mtodos de extrao viveis sendo Hg, Ag e Sn.
este ltimo por causa de seu decaimento radioativo. Bronze constitudo por Cu e Sn.
O sdio reage violentamente com a gua produzindo chama Lato (utilizado em armas e torneiras) constitudo por Cu
se estiver concentrado num pedao de bom tamanho, de um gro e Zn.
de arroz, por exemplo. A equao a seguinte:
2Na + 2H2O 2NaOH + H2 As ligas metlicas
O sdio metlico logo perde o brilho quando exposto ao ar As ligas metlicas podem ser classificadas em basicamente
seco. Isso devido diferena de eletronegatividade, o que os faz ter dois tipos de ligas: ligas ferrosas e ligas no ferrosas.
uma grande afinidade qumica. O xido formado segue a equao:
2Na + O2 Na2O Ligas Ferrosas
So aquelas onde o ferro constituinte principal. Essas ligas
A reao com o sulfato de cobre forma sulfato de sdio: so importantes como materiais de construo em engenharia. As
Na + CuSO4 Na2SO4 + Cu ligas ferrosas so extremamente versteis, no sentido em que elas
podem ser adaptadas para possuir uma ampla variedade de pro-
Um teste pode ser usado para verificar a cor dos metais alcali- priedades mecnicas e fsicas. A desvantagem dessas ligas que
nos colocando-se um cloreto de um metal alcalino sob a ao de elas so muito suscetveis corroso. Aos: so ligas ferro-carbono
uma chama, onde cada metal emite uma chama de cor diferente. que podem conter concentraes apreciveis de outros elemen-

262 UECEVEST

Apostilas UECEVEST mod3.indb 262 06/02/2011 10:03:24


QUMI C A G E R A L

tos de liga. As propriedades mecnicas so sensveis ao teor de Processos da fuso


carbono, que normalmente inferior a 1%. 1. Aos com baixo Fundem-se quantidades adequadas dos componentes da liga,
teor de carbono, essas ligas contem geralmente menos que 0,25% a fim de que estes se misturem perfeitamente no estado lqui-
de C. como consequncia essas ligas so moles e fracas, porm do. A fuso feita em cadinhos de ferro, de ao ou de grafite,
possuem uma ductilidade e uma tenacidade excepcionais; alm em fornos de revrbero ou em fornos eltricos. A massa fundida,
disso, so usinveis soldveis e, dentre todos os tipos de ao, so homognea, resfriada lentamente em formas apropriadas. So
os mais baratos de serem produzidos. Aplicaes tpicas para este tomadas precaues especiais para evitar a separao dos com-
tipo de liga incluem os componentes de carcaas de automveis ponentes da liga durante o resfriamento, para evitar a oxidao
e chapas usadas em tubulaes, edificaes e latas estanhadas. 2. dos metais fundidos, para minimizar as perdas dos componentes
Aos com mdio teor de carbono: esses aos possuem concentra- volteis, etc. Esse processo tambm pode ser efetuado na super-
es de carbono aproximadamente de 0,25 e 0,60%p de carbo- fcie de um corpo. Assim, mergulhando-se folhas de ferro em
no. As maiores aplicaes destas ligas se encontram em rodas de estanho fundido, forma-se na sua superfcie uma liga de ferro e
trens, engrenagens, virabrequins e outras peas de alta resistncia estanho. Obtm-se, assim, a folha-de-flandres,tambm chamada
que exigem uma combinao de elevada resistncia, resistncia lata.Compresso
abraso e tenacidade. 3. Aos com alto teor de carbono: esses O processo de compresso consiste em submeterem-se mis-
aos apresentam em mdia uma concentrao de carbono e 0,60 turas em propores adequadas dos componentes a altssimas
a 1,4%p. so mais duros, mais resistentes e, porem, os menos presses. Esse processo de importncia na preparao de ligas
dcteis dentre todos os aos de carbono. Esses aos so usados de alto ponto de fuso e quelas cujos componentes so imiscveis
geralmente como ferramentas de corte, bem como para a fabrica- no estado lquido.
o de facas, laminas de serras para metais, molas e arames com
alta resistncia. Processo Eletroltico
O processo eletroltico consiste na eletrlise de uma mistura
Ligas no ferrosas apropriada de sais, com o fim de se efetuar deposio simultnea
So ligas que no possuem como constituinte principal o ele- de dois ou mais metais sobre ctodos
mento ferro. Ligas de cobre: o cobre, quando no se encontra na
forma de ligas, to mole e dctil que muito difcil de ser usi- Processo de Metalurgia Associada
nado. As ligas de cobre mais comuns so os lates, onde o zinco, O processo de metalurgia associada consiste na obteno de
na forma de uma impureza substitucional, o elemento de liga uma liga constituda de dois ou mais metais, submetendo-se ao
predominante. Ligas de cobre-zinco com concentraes aproxi- mesmo processo metalrgico uma mistura de seus minrios.e etcs
madamente de 35%p de zinco so relativamente moles, dcteis
e facilmente submetidos deformao plstica a frio. As ligas de Oxidao
lato que possuem um maior teor de zinco so mais duras e mais A maioria dos metais tende a se oxidar quanto expostos ao
resistentes. Os bronzes so ligas de cobre com vrios outros ele- ar, especialmente em ambientes midos. Entre os vrios proce-
mentos, incluindo o estanho, alumnio, o silcio e o nquel. Essas dimentos empregados para evitar ou retardar a oxidao, os mais
ligas so relativamente mais resistentes do que os lates, porm comuns so a aplicao de pinturas protetoras, a formao de ligas
ainda possui um elevado nvel de resistncia a corroso. Alguns com outros elementos que reduzam ou eliminem tal propenso e a
outros exemplos de ligas no ferrosas so as ligas de alumnio, que conexo a plos eltricos que impeam a ocorrncia do fenmeno.
so caracterizadas por uma densidade relativamente baixa, con- interessante o caso do alumnio, que, em presena do oxignio,
dutividade eltrica e trmica elevada, e uma resistncia corroso forma uma delgada pelcula de xido que detm a oxidao.
em alguns ambientes comuns, com a atmosfera ambiente. Liga
de magnsio caracterizada pela baixa densidade do magnsio Alguns tipos de ligas metlicas
que a mais baixa dentre todos os metais estruturai; dessa forma Ao
suas ligas so usadas onde um peso leve considerado importan- O ao a liga de ferro e carbono onde a porcentagem deste
te, como por exemplo, em componentes de aeronave. ltimo varia de 0,008% a 2,11%. Nos aos utilizados pela in-
dstria geralmente essa porcentagem fica entre 0,1 a 1,0%. Em
Importncia na indstria certos aos especiais, o carbono pode chegar a 1,5%. So tam-
Apesar da grande variedade de metais existentes, a maioria no bm constituintes normais do ao o silcio (0,2%) e o mangans
empregada em estado puro, mas em ligas com propriedades alte- (1,5%). O enxofre e o fsforo so impurezas indesejveis, e seus
radas em relao ao material inicial, o que visa, entre outras coisas, teores no devem ser maiores do que 0,05%. Quando se adi-
a reduzir os custos de produo. As indstrias automobilsticas, cionam outras substncias, para aperfeioamento das qualidades
aeronuticas, navais, blicas e de construo civil so as princi- do ao, obtm-se ligas denominadas aos especiais. Os principais
pais responsveis pelo consumo de metal em grande escala. So aos especiais contm um ou mais dos seguintes metais: nquel,
tambm representativos os setores de eletrnica e comunicaes, vandio, tungstnio, molibdnio, titnio, cobalto ou mangans.
cujo consumo de metal, apesar de quantitativamente inferior, tem
importncia capital para a economia contempornea. Ligas me- Lato
tlicas so materiais de propriedade semelhantes s dos metais e O lato uma liga de cobre e zinco, tem cor amarelada e
que contm pelo menos um metal em sua composio. H ligas utilizado na fabricao de objetos de uso domstico, como tachos
formadas somente de metais e outras formadas de metais e semi- e bacias, de instrumentos musicais de sopro e de jias fantasia.
metais (boro, silcio, arsnio, antimnio) e de metais e no-metais
(carbono, fsforo). interessante constatar que as ligas possuem
propriedades diferentes dos elementos que as originam. Algumas
propriedades so tais como diminuio ou aumento do ponto de
fuso, aumento da dureza, aumento da resistncia mecnica. REFERnCIAS BIBlIOGRFICAS

Processos BRADY, J.E.; HUMISTON, G.E.; Qumica Geral, Livros Tc-


As ligas metlicas podem ser obtidas por diversos processos: nicos e Cientficos (LTC) Editora S.A.; Rio de Janeiro, 1995.

UECEVEST 263

Apostilas UECEVEST mod3.indb 263 06/02/2011 10:03:24


Q U M I CA GERAL

BUENO, W. et al..; Qumica Geral. So Paulo. Editora Mc-


Graw-Hill do Brasil LTDA., 1978.

CAVALCANTI, E.S.B.; Apostila de Qumica Geral, UECE, 2005.

FELTRE, R.; Qumica, V.1, Editora Moderna , So Paulo, 1996.

RUSSELL, J. B., Qumica Geral, Makron Books, So Paulo, 1994.

PERUZZO, T.M; CANTO, E.L.do; Qumica Na Abordagem


do cotidiano,V.1,Editora Moderna , So Paulo, 1996.

USBERCO, J. ; SALVADOR, E. Qumica Geral, V.1, Editora


Saraiva, 2000.

http://www.mundodoquimico.hpg.ig.com.br/vidrarias.htm

http://www.cdcc.sc.usp.br/elementos/

http://www.coladaweb.com/questoes/quimica/disele.htm

http://www.vestibular1.com.br/apostilas/apostilas_exercicios.htm

http://knol.google.com/k/hidr%C3%B3xido-de-s%C3%B3dio#

http://www.tabela.oxigenio.com/nao_metais/elemento_quimi-
co_enxofre.htm

http://www.scribd.com/doc/21348141/Quimica-Inorganica-02-
Metais-Alcalinos-e-Alcalinos-Terrosos

264 UECEVEST

Apostilas UECEVEST mod3.indb 264 06/02/2011 10:03:24


P R - V E S T I B U l A R

QUMICA ORGNICA

Apostilas UECEVEST mod3.indb 265 06/02/2011 10:03:29


Caro(a) Aluno(a),

Para facilitar o acompanhamento de tais contedos, abaixo esto indicadas as nomen-


claturas utilizadas pela UECE e pelo ENEM:
UECE ENEM
Propriedades e mtodos de preparao Transformaes qumicas
Reaes, equaes e clculos qumicos Transformaes qumicas

Apostilas UECEVEST mod3.indb 266 06/02/2011 10:03:29


QUMICA O R G N I C A

REAES ORGnICAS (UECE/EnEm) H H H NO2


H2O
H3 C C H + H O NO2 H3C C + H O NO2 H3C C H
Intermedirios de reaes qumicas

Obteno de intermedirios de reaes partindo-se da mol- CH3 CH3 CH3

cula do metano: nitro isopropano

Ciso Homoltica: Obtm-se o radical metila e o radical


hidrognio Sulfonao - reao com cido sulfrico concentrado (H2SO4
CH3 : H CH3 H = HO - SO3H). Veja o exemplo:
H H H SO3H
Ciso Heteroltica: a ligao covalente quebrada de modo
H2O
H3C C H + H O SO3H H3C C + H O SO3H H3C C H
heterogneo, um dos tomos fica com o par eletrnico e
obtem-se o on carbnio (+CH3) e o carbnion (CH3) CH3 CH3 CH3

CH3 : H CH3 : H cido isopropil-sulfnico

CH3 : H CH3 : H
Pirlise ou craque Os alcanos, quando convenientemen-
te aquecidos, sofrem ruptura homoltica na cadeia, simtrica ou
Tipos de reaes orgnicas no, resultando outros alcanos e alquenos de cadeias menores.
Essa reao muito usada para se obter gasolina (mistura de oc-
Reaes de substituio em alcanos e em cicloalcanos tanos) a partir de querosene e leo diesel (alcanos com mais de
Esses compostos so estveis e s reagem em condies enr- 16 carbonos). O alcano passa atravs de uma cmara aquecida
gicas, ou seja, so pouco reativos; da serem chamados de parafi- a cerca de 400 a 600o C, utilizando geralmente um catalisador
nas (parum anis = baixa afinidade). formado por xidos metlicos. Essas reaes tambm produzem
Trata-se da transformao qumica em que ocorre troca de certa quantidade de hidrognio.
ligantes entre as molculas dos reagentes (reao de dupla troca). Ruptura da cadeia e Rearranjo no radical e liberao
Observe o esquema geral: formao de radicais livres de hidrognio nascente

H H H H H H H H
C H + AB C B + HA
H C C C C H H C C + C C H

H H H H H H H H
Esse tipo de reao caracterstico, entre outros compostos,
nos alcanos e cicloalcanos com anis estveis.
H H
Vamos analisar as seguintes substituies nos alcanos: H H
C C + H C C H
Halogenao reao radicalar com halognio; a ordem de H H
H H
reatividade F2> Cl2> Br2> I2; as reaes que envolvem o flor so
violentas, e as de iodo praticamente no ocorrem. Por exemplo: Reao de Substituio nos cicloalcanos Nos anis com
Ruptura homoltica do Br2 cinco ou mais carbonos, ocorre substituio de hidrognio por
e formao de bromo radical halognio. Por exemplo:
calor ou luz CH2 CH2
Br Br Br + Br H H
H2C C + Cl Cl H2C C + HCl
Ataque do brometo ao hidrognio do alcano H H H
e raptura homoltica da ligao CH H Cl2 Cl
H C C C Br + HBr H2C CH2 H2C CH2
H H H
H C C C H + 2 Br
H H H clorociclopentano
H Br H
H H H Reao de adio aos cicloalcanos
H C C C H + HBr
Por serem compostos cclicos saturados, os cicloalcanos no
H H H deveriam sofrer reaes de adio. Mas se verifica que os primei-
ros compostos dessa srie podem ser abertos, o que possibilita a
Em alcanos com trs ou mais tomos de carbonos, o hidro- ocorrncia de reao de adio. Veja o quadro:
gnio a ser substitudo , de preferncia, o do carbono menos Ciclano Reao com H2
Reao com X2
Reao com HX
hidrogenado (Regra de Markovnikov para a substituio). Assim, (Cl2 ou Br2)
a ordem de reatividade dos hidrognios : CH2 120C
Ni
H3C C CH3 Cl C C C Cl
H2 H2 H2
H3C C C Cl
H 2 H2
H2C CH2 H2
H ligado a C tercirio > H ligado a C secundrio > H ligado a C primrio H2C CH2 200C Br C C C C Br cond.
H3C C C CH3 H3C C C C Br
Ni H2 H2 H2 H2 H2 H2 energ. H2 H2 H2
H2C CH2
Por exemplo: CH2
300C
H2C CH2 H3C (CH2)3 CH3 reao de substituio no ocorre
H H H H H H Ni
H2C CH2

H3C C C C H + Cl Cl H3C C C C H + HCl CH2


H2C CH2
Cl2 no ocorre reao de substituio no ocorre
H H H H Cl H H2C CH2
CH2
carbonos com trs hidrognios 2cloropropano

O quadro mostra que, conforme o anel aumenta, se torna mais


Nitrao - reao com cido ntrico concentrado (HNO3 =
difcil sua quebra, ou seja, quanto maior o anel, mais estvel ele .
HO NO2); por exemplo:

UECEVEST 267

Apostilas UECEVEST mod3.indb 267 06/02/2011 10:03:31


Q U M I CA ORGNICA

Reaes de adio eletroflica aos alquenos


J aprendemos que uma ligao dupla carbono-carbono for-
mada por uma ligao forte e uma ligao mais fraca. As rea-
es que esperaramos que pudessem ocorrer com os alquenos, so
aquelas nas quais a ligao , mais fraca, quebrada e substituda
por duas novas ligaes mais fortes. Uma reao deste tipo
chamada uma reao de adio. Ela pode ser representada como:

Observa-se nestas reaes de adio que se produz apenas um


produto. Este produto obedece a regra de Markovnikov que diz
que: o hidrognio de um halogeneto de hidrognio adicionar-se-
Quais seriam os melhores reagentes para estas reaes de adi- dupla ligao no tomo de carbono mais hidrogenado, e o halognio
o a ligaes duplas carbono-carbono? Sabemos que a ligao adicionar-se- ao carbono menos hidrogenado.
tem capacidade de doar eltrons (base de Lewis, reagente nucle- Reao de hidratao - reao de adio de gua na presen-
oflico), assim evidente que os reagentes que procuram a vizi- a de cidos, com a formao de lcoois. Como exemplo, veja a
nhana de eltrons ( reagentes eletroflicos ou cidos de Lewis), hidratao do eteno:
em reaes qumicas, devero adicionar-se facilmente ligao H H
O
dupla de um alqueno. A adio eletroflica uma reao caracte- H3C H3C
rstica de alquenos. C CH2 H+ C CH2 O H3C CH CH2
H H2O H
Hidrogenao reao de adsoro na superfcie do metal, H
H H
H
muito empregada na preparao industrial de alcanos: a cada
OH
carbono da dupla ligao (que se torna simples) se liga um hi- H+
drognio com estereoqumica cis. Por exemplo: H3C CH CH2
H
2 propanol

Adies de radicais livres: brometo de hidrognio quan-


do, perxidos, luz ou outros iniciadores de reaes via radicais
livres, esto presentes nas reaes de adio de cidos a alquenos,
Reao de halogenao reao de adio de halognios pro- a reao radicalar e obtm-se um produto anti-Markovnikov,
duzindo dialetos vicinais trans (tomos de halognio ligados a ou efeito perxido.
carbonos vizinhos); tendo como intermedirio o on halnio.
Podem ser empregados cloro (Cl2),bromo (Br2) ou iodo ( I2),
nesta ordem de reatividade: Cl2> Br2 > I2.

Polimerizao Dentre os principais produtos da qumi-


ca orgnica industrial de hoje esto os Polmeros, compostos de
alto peso molecular com unidades estruturais repetidas formados
Reao de adio de cidos Os cidos prticos, H-Z, so pela combinao de monmeros simples.
casos tpicos de reagentes eletroflicos que se adicionam a al-
quenos. Quando no meio apropriado, cidos prticos fortes
como o cido sulfrico, os halogenetos de cidos e o cido
trifluoroactico reagem rapidamente com alquenos. Os cidos
mais fracos como a gua e o cido actico no se adicionam por
si s, porm tais reaes ocorrem quando catalisadas por traos
de um cido forte.
Epoxidao A oxidao de alquenos pode levar a compos-
tos, contendo oxignio ligado a um dos carbonos da ligao du-
pla. Vrios tipos diferentes de agentes oxidantes so empregados,
dependendo do produto desejado.

Exemplos de reaes de adio de cidos:

rpido
Ozonlise A reao entre alquenos e oznio ocorre rapi-
damente a baixas temperaturas, quebrando a ligao dupla. O
produto um composto carbonilado (aldedo ou cetona).

268 UECEVEST

Apostilas UECEVEST mod3.indb 268 06/02/2011 10:03:34


QUMICA O R G N I C A

O O


H C C H + 2[O] H C C H
KMnO4/OH
Um ozonideo
Nox= 1 Nox= +1
H etino etanodial (dialdedo)
agente
CH3CH2CH CH2 CH3CH2C + O
redutor H
H
Oxidao enrgica de alcenos e alcinos Para uma oxida-
Reao de Diels-Alder- Cicloadio o mais profunda dos alcenos e dos alcinos, usa-se uma soluo
concentrada e cida de permanganato de potssio (KMnO4/H+)
sob aquecimento. Veja:
os alcenos submetidos a essa soluo sofrem ruptura na ligao
dupla e, conforme a posio dessa ligao na cadeia, temos:
se o carbono da dupla no possui hidrognio, forma-se cetona;
se o carbono da dupla possui um hidrognio, forma-se cido
carboxlico;
se o carbono da dupla possui dois hidrognios, forma-se gs
Adio aos carbonos da tripla ligao As mesmas reaes carbnico.
que ocorrem com os alquenos ocorrem tambm com os alquinos, Por exemplo:
sendo que estes ltimos so mais reativos, e pode ocorrer a quebra Nox= 0 Nox= +2
de um ou dois pares eletrnicos da ligao tripla:
H3C C C CH3 + 3[O] H3C C O+O C CH3
KMnO4/H+
H CH2 OH CH3
Nox= 1 Nox= +3
Nox= +4
A ligao tripla reage de maneira semelhante ligao dupla,
exceto pelas adies de 2 mol de um reagente ligao tripla, nas re- H 3C C C H + 5[O] H3C C O + O C OH
aes de hidrogenao cataltica, halogenao, adio de cidos, etc. KMnO4/H+
H H OH OH
Nox= 1 Nox= 2 Nox= +3
Hidrogenao Na hidrogenao parcial quebra-se um par CO2 + H2O
eletrnico da ligao tripla, que se converte em dupla; na total,
quebram-se os dois pares, e a ligao tripla converte-se em sim- os alcinos submetidos quela soluo sofrem quebra na ligao
ples. Por exemplo: tripla. Se o carbono da ligao tripla no possuir hidrognio,
ele se converte em carboxila; se possuir, ele se converte em CO2:
O
Reao de oxidao branda de alcenos e alcinos Essa
oxidao feita com uma soluo diluda, neutra ou levemente H3C C C H + 4[O] H3C C + CO2
bsica, de permanganato de potssio (KMnO4), conhecida como KMnO4/H + OH
reativo de Baeyer (KMnO4/OH-). Veja:
Os alcenos sob a ao do reativo de Baeyer produzem dilcoois Reao de substituio eletroflica nos compostos
vicinais: aromticos
OH OH
O benzeno e outros compostos aromticos sofrem bromao,
H2C CH2 + [O] + H2O H2C CH2 clorao e iodao na presena de um catalisador adequado. J a
fluorao direta do benzeno no prtica nas condies usuais
KMnO4/OH porque o flor um agente oxidante to poderoso que reage ex-
Nox= 2 Nox= 1 plosivamente com a maior parte dos compostos orgnicos.
eteno etanodiol Existem quatro tipos de reaes de substituio eletroflica
(dilcool vicinal) em aromticos que so de importncia na qumica orgnica de
snteses, que so:
Os alcinos sob a ao do reativo de Baeyer produzem dicetonas, Halogenao
cetonas-aldedos ou dialdedo, no caso do etino. Por exemplo:
Br2 + FeBr3 Br+ + FeBr4
O O
lento H
H3C C C CH3 + 2[O] H3C C C CH3 Br+ +
KMnO4/OH
Br+
H
Nox= 0 Nox= +2
2 butino butanodiona (dicetona) H Br
O O Br+ + Br + HBr
H
H3C C C H + 2[O] H3C C C H Nitrao
KMnO4/OH
Nox= 0 Nox= 1 Nox= +2 Nox= +1 H2SO4 + HNO3 HSO4 + H O NO2
propino cetopropanol
(cetona-aldeido) H

UECEVEST 269

Apostilas UECEVEST mod3.indb 269 06/02/2011 10:03:37


Q U M I CA ORGNICA

H2ONO2 H2O + NO2 Havendo uma segunda substituio no anel, o grupo R (con-
forme sua estrutura) comandar o hidrognio do anel que ser
H
substitudo. Assim, temos:
H
:O: :N: :O: NO2 Grupos meta-dirigentes grupos retiradores de eltrons (por
: induo,-I, ou por ressonncia,-R) comandam a substituio
:
:
do hidrognio da posio meta,a reao s ocorre na presena
de cido de Lewis:
H
H NO2 NO2 NO2
NO2 B + HB cido
+ Br Br + HBr
de
H Br2 Br
Lewis
Sulfonao nitrobenzeno
2H2SO4 SO3 + H3O + SO4
Se houver quantidade suficiente de bromo, as duas posies
H meta so substitudas:
SO3 SO3 NO2 NO2
H
cido
+ 2Br2 + 2HBr
H de
Lewis Br Br
SO3 SO3
H + BH Grupos orto/para-dirigentes - grupos doadores de eltrons
B (por induo,+I, ou por ressonncia,+R) comandam a
substituio dos hidrognios das posies orto e/ou para, a
SO3 SO3H reao s ocorre na presena de cido de Lewis:
+ BH + B CH3 CH3

H cido Cl
Alquilao de Friedel-Crafts reao com haleto de alquila, + Cl Cl + HCl
na presena de um cido de Lewis, para gerar o eletrfilo forte, de
Cl2 Lewis
seguida da introduo de grupo alquila no anel benznico:
metilbenzeno
CH3CH2Br AlCl CH3CH2 Br AlCl3 CH3CH2 + AlCl3
Se houver excesso de cloro, a substituio total (das duas
H posies orto e da posio para):
AlCl3Br CH3CH2
CH3CH2 + CH3CH2 + HBr
H CH3 CH3

Acilao de Friedel-Crafts - reao com haleto de acila, na cido Cl Cl


+ 3Cl2 + 3HCl
presena de um cido de Lewis, para gerar o eletrfilo forte, on de
aclio, seguida da introduo de grupo acila no anel benznico: Lewis
O Cl
R C
Cl
+ AlCl3 [R C O R C O [+ AlCl3

Um ion aclio Nos quadros, abaixo, esto representados os principais grupos


O meta-dirigentes e orto/para-dirigentes.
C CH2CH3 Principais meta dirigentes
O AlCl3
CH3CH2C + (retiradores de eltrons/ -I, -R)
Cl
-NO2
Grupos orientadores no anel benznico -SO3H
Considere um composto com um anel benznico e que j -COOH
sofreu uma substituio, tendo entrado nesse anel um grupo R. -CONH2
Em relao posio de R, as outras posies do anel so deno- -CHO
minadas orto, meta e para: -CN
Principais orto-para dirigentes
R (doadores de eltrons / +I, +R)
orto orto -NH2
-OH
OR
meta meta -R (alquil)
para -X (Cl, Br, I)

270 UECEVEST

Apostilas UECEVEST mod3.indb 270 06/02/2011 10:03:39


QUMICA O R G N I C A

Oxidao de aromticos H Br
O
lcool
H C C H + 2KOH H C C H + 2KBr + 2HOH
CH3 C
3[O] altas
2H2O
+ OH + H2O Br H temp.
KMnO4
Nox= 3 Nox= +3 Reaes de lcoois

Reaes de compostos halogenados Converso de lcoois em halogenetos de alquila


Os compostos orgnicos halogenados so muito usados H3C CH2OH + HI H3C CH2I + HOH
como intermedirios em snteses. Eles tm pouco interesse em si
mesmos, mas podem ser convertidos em uma grande variedade H2O
de compostos e podem ser obtidos por um grande nmero de di- Converso de lcoois em steres
ferentes processos. Assim, eles tm uma grande importncia pr- O O
H+
tica; por isto, vamos estudar algumas de suas reaes qumicas. H3C C + H3C OH H3C C + HOH
A maior parte das reaes dos halogenetos orgnicos pode ser OH metanol O CH3 H2O
enquadrada em duas classes: reaes de substituio e reaes
de eliminao. cido etanico etanoato de metila (ster)

Substituio nucleoflica Essa reao entre cido e lcool, sob catlise cida, se chama
Consistem na troca de um tomo ou grupo de tomos da mo- Esterificao de Fischer.
lcula por outro tomo ou grupo de tomos. Em particular estas re-
aes so importantes na sntese de uma variedade de molculas or- Desidratao converso de lcoois em alquenos e teres
H H
gnicas, inclundo lcoois, aminas, alquinos, steres, teres e nitrilas. H H
170C
RX + Z RZ + X H C C H C C + HOH
H2SO4(conc)
Halogeneto Nuclefilo Grupo de sada H H H2O
H OH
de alquila
Havendo mais de um carbono vizinho ao que contm o gru-
Esta reao ocorre atravs de dois tipos de mecanismos, o me- po hidroxila (OH), deve-se eliminar o hidrognio ligado ao car-
canismo Sn1 e o mecanismo Sn2, que esto exemplificados abaixo: bono vizinho menos hidrogenado (Regra de Saytze):
Mecanismo Sn1 H H H H
170C
H3C C C CH3 HC C C CH3 + HOH
H2SO4(conc.) 3
2-buteno H2O
OH H
2-butenol
Se um lcool tratado com cido sulfrico a uma temperatura
menor do que a necessria para desidrat-lo a alqueno e se h excesso
de lcool, a desidratao ocorre tendo como resultado um ter.

Mecanismo Sn2

Oxidao de lcoois
Os lcoois se oxidam em presena de permanganato de pots-
Eliminao sio (KMnO4) ou de dicromato de potssio (K2Cr2O7), em qual-
chamada de reao de eliminao a transformao qumica quer meio, ou quando expostos ao oxignio do ar em presena de
de halogenetos de alquila, em que ocorre a formao de alquenos. catalisador (cobre ou platina). Sendo lcool:
dialeto vicinal primrio, produz aldedo;
lcool secundrio, produz cetona.
H2C CH2 + Zn H2C CH2 + ZnBr2
OH
eteno O
Br Br H3C C H + [O] H3C C + H 2O
1, 2-dibromoetano H
H Nox= 1 Nox= +1
Eliminao de halogenetos lcool primrio aldedo
Monohalogenetos ou diahalogenetos vicinais sofrem elimi-
nao de halogenetos (HX) quando tratados com bases fortes, H
temperaturas elevadas e em meio alcolico, produzindo alcenos
ou alcinos, respectivamente: H3C C CH3 + [O] H3C C CH3 + H2O
H H OH Nox= 0 O Nox= +2
lcool
H C C H + KOH H C C H + KBr + HOH lcool secundrio cetona
altas
temp. H H H2O
H Br Os lcoois tercirios no se oxidam.

UECEVEST 271

Apostilas UECEVEST mod3.indb 271 06/02/2011 10:03:42


Q U M I CA ORGNICA

Reaes de aldedos e cetonas Hidrlise de derivados de cidos


As reaes mais caractersticas dos compostos carbonilados
so as adies ligao dupla carbono-oxignio, que so Reaes
de Adio Nucleoflicas:

Adies de Alcois carbonilas de aldedos e cetonas

Preparao de anidridos atravs da desidratao de ci-


dos carboxlicos- quando aquecidos na presena de agen-
tes desidratantes, os cidos carboxlicos desidratam-se pro-
duzindo anidridos:
O
H3C C C
H2 OH

Condensao com amonaco e derivados
- H2O C6H5 OH H2SO4(conc.)
C6H5 H C6H5 OH C NH outros
H
C O + NH3 C
H produtos H3C C C
H NH2 Aldimina H2 O

Condensao Aldlica cido propanico


O
2 H3C C C
H2 O + HOH (desidratao intermolecular)
H3C C C H2O
Oxidao de aldedos H2 O
O O anidrido propanico
H3C C + [O] H3C C O O
H OH C C
Nox= +1 Nox= +3 OH
O + HOH (desidratao
etanal cido etanico OH H2SO4(conc.)
C C H2O intramolecular)
Reaes de cidos carboxlicos e seus derivados O O
A carboxila um dos grupos funcionais mais importantes. A cido ortoftlico anidrido ortoftlico
qumica da carboxila inclui reaes que envolvem (1) a remoo
do prton por base, (2) o ataque do grupo carboxila ou do on
carboxilato como nuclefilo a outras substncias, (3) o ataque de
outro nuclefilo ao carbono da carboxila e (4) o ataque de uma E X E R C C I O
base ao hidrognio ativado pela carbonila do cido carboxlico
01. Pretende-se fazer a clorao do propano (C3H8). Qual o pro-
ou derivado. Combinaes destes tipos de reaes so comuns:
duto obtido (haleto) em maior quantidade?
a) cloreto de n-propila
b) 1-cloropropano
c) cloreto de isopropila
d) a reao no ocorre
e) a reao ocorre, mas no se obtm haleto
Os cloretos de cidos, anidridos, cetenos, steres e aminas
02. Numa reao de 2-metilbutano com Cl2, ocorreu a substitui-
so chamados, frequentemente, derivados de cidos, uma vez que
o de hidrognio. Qual o composto clorado obtido em maior
podem ser convertidos a cidos carboxlicos ou obtidos a partir
quantidade?
destes compostos. Considera-se, s vezes, as nitrilas como deriva-
a) 1,2,3-tricloropentano
dos de cido.
b) 1-cloro-2-metilbutano
A maior parte das rea es de cidos carboxlicos e seus deri-
c) 1-cloro-3-metilbutano
vados envolve substituio nucleoflica, no carbono da carboni-
d) 2-cloro-2-metilbutano
la, catalisada por cido.
e) 2,2-dicloropentano

03. (Fuvest SP) Fenol (C6H5OH) encontrado na urina de


pessoas expostas a ambientes poludos por benzeno (C6H6). Na
transformao de benzeno em fenol ocorre:
Preparao de amidas e nitrilas
a) substituio no anel aromtico.
b) quebra na cadeia carbnica.
c) rearranjo no anel aromtico.

272 UECEVEST

Apostilas UECEVEST mod3.indb 272 06/02/2011 10:03:46


QUMICA O R G N I C A

d) formao de cicloalcano. a) NO2, Cl, Br.


e) polimerizao. b) CF3, NO2, COOH.
c) CHO, NH2, CH3.
04. (Cesgranrio) Considere as seguintes reaes orgnicas: d) SO3H, NO2, COOH.
e) CH3, Cl, NH2.

10. (UFC-CE) So apresentados, abaixo, quatro compostos per-


tencentes a vrias funes orgnicas:
CH3OH CH3NH2 CH2 = CH2 C6H10
A alternativa que corresponde a dois compostos capazes de sofrer
reao de adio :
a) metanol e metilamina.
Podemos classific-las, respectivamente, como:
b) metailamina e eteno.
a) adio, substituio, eliminao.
c) ciclo-hexeno e metilamina.
b) adio, eliminao, substituio.
d) ciclo-hexeno e metanol.
c) eliminao, adio, substituio.
e) eteno e ciclo-hexeno.
d) eliminao, substituio, adio.
e) substituio, adio, eliminao.
11. Os lipdeos podem ser classificados como leos ou gorduras:
nos leos predominam cadeias insaturadas e nas gorduras predo-
05. (Mack) No sistema de equaes a seguir, as substncias A e B
minam cadeias saturadas. Com base nessa afirmao, um leo
so, respectivamente:
transformado em gordura, atravs de um processo, no qual ocor-
re reao de:
a) saponificao d) desidratao
b) hidrlise cida e) hidrogenao cataltica
c) esterificao
a) metano e bromo-metano
b) etano e bromo-etano
12. Na reao do H2C = CH CH3 com HCl h formao em
c) eteno e bromo-etano
maior quantidade do:
d) propeno e 2-bromo-propeno
a) 1-cloropropano. d) 1,3-dicloropropano.
e) eteno e etino
b) 2-cloropropano. e) 2,2-dicloropropano.
c) 1,2-dicloropropano.
06. (UFF) Os fenis so compostos orgnicos oxigenados. Mui-
tos so utilizados como desinfetantes para uso domstico e alguns
13. (Fuvest-SP) Dois hidrocarbonetos insaturados, que so isme-
so os principais constituintes do carvo mineral. Indica-se, a se-
ros, foram submetidos, separadamente, hidrogenao cataltica.
guir, as estruturas de componentes do carvo mineral, alguns dos
Cada um deles reagiu com H2 na proporo, em mols, de 1:1,
quais esto presentes no desinfetante conhecido por creolina.
obtendo-se, em cada caso, um hidrocarboneto de frmula C4H10.
Os hidrocarbonetos que foram hidrogenados poderiam ser:
a) 1-butino e 1-buteno.
b) 1, 3-butadieno e ciclobutano.
c) 2-buteno e 2-metilpropeno.
d) 2-butino e 1-buteno.
e) 2-buteno e 2-metilpropano.

14. (UEPG-PR) Quando se faz borbulhar o propino em soluo


de H2SO4 (em presena de sulfato de mercrio como catalisa-
Nesta reao, os produtos principais so os que apresentam suas
dor), e a esse hidrocarboneto se adiciona uma molcula de gua
estruturas indicadas por:
ocorre tautomerizao e o principal produto dessa reao :
a) I e II d) II e IV
a) cido actico. d) 2-propanol.
b) I e IV e) III e IV
b) 2, 2-propanodiol. e) propanona.
c) II e III
c) propanaldedo.
07. (UFF) O grupo amino (-NH2), ligado ao anel benznico, nas
15. (Covest) lcoois no so utilizados somente como combust-
reaes de substituio eletroflica aromtica um orientador:
veis, mas tambm na produo de derivados do petrleo, como,
a) apenas orto d) orto e meta
por exemplo, alquenos. Qual dos lcoois abaixo produzir prope-
b) meta e para e) orto e para
no a partir da desidratao em meio cido?
c) apenas meta
a) 2-metil-2-propanol. d) 2-butanol.
b) etanol. e) 2-metil-1-propanol.
08. (Puc-RJ) Qual a massa de nitrobenzeno obtida na nitrao de
c) 2-propanol.
5g de benzeno, supondo-se um rendimento da reao de 100%?
Dados: (massas molares) C =12 g/mol; H =1g/mol; N =14 g/
mol; O = 16 g/mol
a) 7,88g c) 6,98 G A B A R I T O
b) 5,77 d) 4,32
01. c 02. d 03. a 04. a 05. b
09. (UFBA) Das alternativas a seguir, a que contm somente gru- 06. d 07. e 08. a 09. d 10. e
pos orientadores meta : 11. e 12. b 13. c 14. e 15. c

UECEVEST 273

Apostilas UECEVEST mod3.indb 273 06/02/2011 10:03:47


Q U M I CA ORGNICA

REFERNCIAS BIBLIOGRAFICAS
ALLINGER, N.L., CAVA, M.P., DE JONGH, P.C., JOHN-
SON, C., LEBEL, N., STEVENS, C.L., Qumica Orgnica,
Ed. Guanabara dois.

CAREY, F.C. & SUNDBERG, R.J., Advanced Organic Che-


mistry, 3a. Ediao.

CAVALCANTI, E.S.B.; Apostila de Qumica Orgnica,


UECE, 2005.

FELTRE, R. ; Qumica , V.3, Editora Moderna, So Paulo, 1996.

PERUZZO, T.M; CANTO, E.L.do; Qumica na Abordagem


do cotidiano, Editora Moderna, So Paulo, 1996.

SOLOMONS, T.W.G. & FRYHLE, C.B., Qumica Orgnica,


Ed. LTC, 7a. Edio.

USBERCO, J. ; SALVADOR, E. Qumica Orgnica I, Editora


Saraiva, 2000.

VOLLHARDT, K.P.C. & SCHORE, N. E., Qumica Orgni-


ca, Ed. BOOKMAN, 4Edio.

http://www.scribd.com

http://www.cienciaquimica.hpg.com.br

http://www.oxigenio.com

http://translate.googleusercontent.com/translat

274 UECEVEST

Apostilas UECEVEST mod3.indb 274 06/02/2011 10:03:47


P R - V E S T I B U l A R

FSICO-QUMICA

Apostilas UECEVEST mod3.indb 275 06/02/2011 10:03:56


Caro(a) Aluno(a),

Para facilitar o acompanhamento de tais contedos, abaixo esto indicadas as


nomenclaturas utilizadas pela UECE e pelo ENEM:
UECE ENEM
Eletroquimica (Eletrlise / Leis de Faraday) Transformaes qumicas e energia
Estudo dos gases Transformaes qumicas e energia

Apostilas UECEVEST mod3.indb 276 06/02/2011 10:03:56


FSICO - Q U M I C A

ElETROQUmICA (UECE/EnEm) 3) ons


Simples: Fe3+
A variedade de reaes qumicas que envolvem oxidao-re- Nox = carga do on
duo no nosso cotidiano surpreendente. Parece que vivemos Composto: SxO42-
das pilhas e baterias que movimentam as calculadoras, carros, Nox total = carga do on
brinquedos, lmpadas, rdios, televises e muitas outras coisas. x -8 = -2 x= 6
Para combater a corroso, polimos a prataria, pintamos as
grades de ferro e galvanizamos os pregos. Reaes de xido-reduo
Circuitos de computadores so cobertos por finas camadas de Reao de xido-reduo aquela que ocorre com transfern-
ouro ou prata aplicadas por eletrodeposio. cia de eltrons, mas como ns no conseguimos enxergar os el-
A revelao fotogrfica utiliza reaes qumicas que envolvem trons o Nox nos mostra o que ocorre com os eltrons, portanto,
transferncia de eltrons. se ocorrer variao do Nox teremos uma reao de xido-reduo.
As plantas transformam energia em compostos atravs de uma Exemplos:
srie de reaes chamadas de cadeia de transporte de eltrons. 2 Mg(s) + O2(g) 2 MgO(s)
Os testes de glicose na urina, ou de lcool no ar expirado, so 0 0 +2 2
feitos com base em intensas mudanas de cor, atravs de reaes (NH4)2Cr2O7(s Cr2O3(s) + N2(g) + 4 H2O(v)
que tambm envolvem a transferncia de eltrons. -3 +1+ 6 2 +3 2 0 +1 -2
Quando voc acende uma lanterna eltrica ou d partida no
carro, ocorre uma reao qumica muito importante: a reao de No se esqueam:
oxireduo. Reao de xido-reduo
Pilha um dispositivo que produz corrente eltrica a partir
de uma reao de oxirreduo. Transferncia de eltrons
Eletrlise um processo no qual uma corrente eltrica pro-
duz uma reao de oxirreduo. Variao do Nox

Definies importantes Vamos analisar o que ocorreu na primeira reao com cada
A) Oxidao = perda de eltrons = aumento do Nox = agente um dos elementos:
redutor = nodo 2 Mg(s) + O2(g) 2 MgO(s)
B) Reduo = ganho de eltrons = diminuio do Nox = agente 0 0 +2 2
oxidante = ctodo
oxidao reduo
n de Oxidao (nox)
a carga que surge quando os elementos esto combinados: No Mg o Nox aumentou perdeu eltrons oxidou.
NaCl Na+ Cl (sdio doou 1 e e o cloro recebeu 1e) No O o Nox diminuiu recebeu eltrons reduziu.
Quando a ligao for covalente e houver diferena de eletro-
negatividade ir tambm surgir uma carga, como por exemplo, a Quando uma substncia encontra a outra, uma sofre reduo
ligao entre H e Cl, o cloro mais eletronegativo que o hidro- e provoca na outra a oxidao, por outro lado, a outra substncia
gnio; o cloro fica negativo pois puxa o par de eltrons com mais sofre oxidao e provoca a reduo. Formando o que conhecemos
intensidade que o hidrognio, que fica positivo. por agente oxidante e agente redutor.
H+ Cl
O objetivo agora que voc consiga determinar o Nox atravs Agente redutor
das regras que sero expostas a seguir. Em um exerccio podere- a substncia que contm o elemento que sofre oxidao, a
mos encontrar para determinar o Nox uma substncia simples ou substncia que provoca a reduo:
uma substncia composta ou um on, agora vamos apresentar as redutor - Mg
regras para determinar o Nox de cada um dos itens.
Agente oxidante
Regras para determinar o nox a substncia que contm o elemento que sofre reduo, a
1) Toda substncia simples: Nox = zero substncia que provoca a oxidao:
2) Substncia composta oxidante - O2
metais alcalinos (Na e K) e Ag: Nox= +1
metais alcalinos terrosos Tabela de nox Fixo
(Mg e Ca) e Zn: Nox =+2 Elementos Nox
alumnio:Nox=+3 Famlia 1 A +1
hidrognio
Famlia 2A +2
hidrognio + ametal: Nox= +1
hidrognio + metal: Nox = - 1 Zn, Cd +2
oxignio Ag +1
na maioria dos compostos: Nox=-2 Al +3
nos perxidos (O22): Nox= -1 (exemplo H2O2) Halognios (7A) -1
calcognios (O, S): Nox= -2 Calcognios (6A) - 2 (no final da frmula)
Halognios (F, Cl, Br, I): Nox = -1
Hidrognio +1 (na maioria)
Exemplo: Qual o Nox do carbono na molcula abaixo? -1 (com 1A ou 2A)
Na2+1 CX O3-2 Oxignio -2 (na maioria)
Carga Total: +2 +x -6 = 0 x= +4 +1 ou +2 (com flor)
Assim o Nox do C = +4 -1/2 (perxidos)

UECEVEST 277

Apostilas UECEVEST mod3.indb 277 06/02/2011 10:03:57


F S I C O -QUMICA

Pilha de Daniell
constituda de dois recipientes, onde cada um contm uma E X E R C C I O
placa metlica mergulhada em uma soluo aquosa de seus pr-
01. Determine o Nox do fsforo nas espcies PCl3, PCl5, H3PO4.
prios ons (eletrodo).
Para que ocorra a produo de corrente eltrica, os eletrodos
02. D o n de oxidao do cloro nas substncias:
esto ligados atravs de um fio condutor e uma ponte salina ou
Cl2, HCl, HClO, HClO2, HClO3, HClO4
parede porosa, unindo os dois recipientes.
Ex: pilha de Zn e Cu
03. (UFES) Considere o composto qumico fosfato de clcio,
Circuito externo tambm chamado de ortofosfato de clcio. Em relao a ele, mar-
e e que a opo incorreta:
dados: Ca (MA=40u); P (MA=31u); O (MA=16u).
Ponte salina de K2 SO4 a) sua frmula contm 13 tomos
Zn Cu b) a massa de seu mol 310g.
nodo() SO42 K+ (+) Ctodo c) o n de oxidao do fsforo + 5.
d) o composto um sal.
e) o clcio substitui os dois hidrognios ionizveis do cido de
origem.
Zn2+ Cu2+
K SO4 2
04. Das reaes abaixo, quais so de xido-reduo?
Zn SO42
2+
Cu2+ SO42 I. I2(g) 2 I(g)
II. NaCl + AgNO3 AgCl + NaNO3
III. 2Na + O2 Na2O
Concluses
a) I, II e III.
Plo negativo - nodo b) I e II somente.
Ocorre oxidao c) II e III somente.
Perde eltrons: Zn Zn2+ + 2e d) III somente.
Fornece eltrons ao circuito -eltrico externo e) Nenhuma delas de oxirreduo.
Ocorre corroso do Zn
Ocorre aumento da concentrao da soluo de Zn2+ 05. (UnB-DF) O marca-passo uma pequena bateria colocada
sob a pele de pessoas com certos tipos de problemas cardacos.
Plo positivo - Ctodo Esta pilha eletroqumica libera pulsos eltricos que regulam as
Ocorre reduo: Cu2+ + 2e Cu(S) batidas do corao. A reao responsvel pela produo de cor-
Recebe ou ganha eltrons rente eltrica : HgO(s) + Zn(s) + H2O(l) Zn(OH)2(aq) + Hg(l)
Retira eltrons do circuito eltrico externo Julgue os itens:
Ocorre diluio da soluo 1) O HgO o agente redutor.
Ocorre eletrodeposio 2) O nome da substncia HgO xido de mercrio II.
3) A substncia Zn(OH)2 pertence funo hidrxido.
Equao da pilha 4) O Zn o agente oxidante
nodo / / Ctodo 5) Para cada tomo de zinco que reage, um eltron
(Me/Men+) / / (Men+/Me) transferido no processo.

Diferena de potencial (ddp) de uma pilha 06. (FUVEST) Considere as seguintes reaes qumicas:
oxid I. SO2 + H2O2 H2SO4
Me Men+ + ne- II. SO2+H2O H2SO3
red III. SO2+NH4OH NH4HSO3
Podem-se classificar como reao de oxidorreduo, apenas:
E = ERED + EOXID a) I d) I e III
b) II e) II e III
c) III
G = -nFE
07. (UFU-MG) Entende-se por corroso de um material a sua
Logo: deteriorao ou destruio, causada por uma reao qumica com
E > 0 indica que a reao espontnea; o meio no qual se encontra. Essas reaes so de oxidao e redu-
E < 0 indica que a reao no-espontnea. o. Na reao qumica de oxidao e reduo, representada pela
equao: Al + 3AgNO3 Al(NO3)3 + 3Ag
OBS.: a) o alumnio o oxidante, porque oxidado;
As reaes de todas as pilhas so espontneas. b) o alumnio o redutor, porque oxidado;
Exemplos de potencial de reduo c) a prata do nitrato de prata o oxidante, porque ela
Cu 2+ + 2e Cu Ered = +0,34V oxidada;
Ag+ + e Ag Ered = +0,80V d) a prata do nitrato de prata o redutor, porque ela
Mg + 2e Mg
2+
Ered = - 2,37V reduzida;
Fe 2+ + 2e Fe Ered = - 0,44V e) o alumnio e a prata do nitrato de prata so redutores,
porque eles so oxidados.

278 UECEVEST

Apostilas UECEVEST mod3.indb 278 06/02/2011 10:03:58


FSICO - Q U M I C A

A eletrlise gnea exige eletrodos inertes que possuam elevado


G A B A R I T O ponto de fuso. Geralmente so usados a platina ou grafita.
01.+3, +5, +5 02.* 03. e 04. d A eletrlise do NaCl um processo economicamente impor-
05. 2 e 3 06. a 07. b tante. O NaCl se funde temperatura de 808 C.
NaCl(slido) NaCl(lquido)
02. zero, -1, +1, +3, +5 e +7
Atravs de dissociao.

NaCl Na+ + Cl
mETAl DE SACRIFCIO
Gerador
Ions em movimento:
Podemos revestir o ferro com uma fina camada de outro ma-

e Eletrodos e + Na+, Cl
terial que possua maior poder redutor que ele, isto , que tenha de carvo
um potencial de oxidao maior que o ferro.
Esse metal parte para o sacrifcio e no permite a oxidao
do ferro. usado outro metal que sofra oxidao mais facilmente
que o ferro. nodo
Na+
Por isso chamado de sacrifcio: o metal se sacrifica (se oxida) Ctodo Cuba eletroltica
Cl

no lugar do ferro, protegendo-o. Metais como zinco e magnsio Na Gs cloro Cl2
podem ser sacrificados para proteger o ferro. Eles se oxidam no
lugar do ferro.
Os ons Cl1- se dirigem para o nodo (plo positivo), perdem
Equao de nernst seus eltrons e so transformados em gs cloro, Cl2.
E 2,303RT log Q 2 Cl Cl2 + 2e (oxidao)
E=
nF
Os ons Na1+ se dirigem para o ctodo (plo negativo), rece-
a 25 C: bem um eltron e so transformados em sdio metlico (Na0). A
equao foi multiplicada por 2 para igualar o nmero de eltrons
E 0,059 log Q na reduo e na oxidao.
E=
n 2 Na+ + 2e 2Na0 (reduo)

OBS.: A equao global da eletrlise dada pela soma das reaes de


No equilbrio Q = Kc ou Kp. dissociao do sal e das reaes que ocorrem nos eletrodos.
2NaCl 2Na+ + 2Cl
Eletrlise (UECE/EnEm) 2Cl Cl2 + 2e- (oxidao)
A eletrlise um processo eletroqumico, caracterizado pela 2Na+ + 2e 2Na0 (reduo)
ocorrncia de reaes de oxi-reduo em uma soluo conduto- Reao global: 2NaCl Cl2 + 2Na0
ra quando se estabelece uma diferena de potencial eltrico entre
dois (ou mais) eletrodos mergulhados nessa soluo. Vale lembrar Portanto, a eletrlise do NaCl tem como produtos o gs clo-
que a denominao soluo eletroltica, empregada para desig- ro e sdio metlico.
nar qualquer soluo aquosa condutora de eletricidade, deriva A eletrlise gnea permite a obteno do alumnio a partir
justamente desse processo. da bauxita (Al2O3). Em condies normais a bauxita funde a
Alm de sua larga aplicao industrial, a eletrlise se revela 2050C. Com a utilizao da criolita (Na3AlF6) como fundente,
bastante adequada e interessante para demonstraes em feiras esta temperatura cai para 1000 C.
de cincias, pois no requer montagens complicadas e pode ser Al2O3 2Al3+ + 3O2
observada visualmente (junto aos eletrodos) enquanto ocorre.
constituda de um gerador e um recipiente chamado cuba No plo negativo...
eletroltica. 4Al3+ + 12e- 4Al0

+ No plo positivo...
6O2- 3O2 + 12e

Eletrodo Eletrodo Equao global...


negativo positivo 2Al2O3 4Al3+ + 6O2-
4Al3+ + 12e 4Al
Cuba Soluo 6O2- 3O2 + 12e
eletroltica A B
inica
Reao global: 2Al2O3 4Al0 + 3O2
O gs oxignio formado na oxidao reage com o carbono do
Classificao eletrodo de grafita produzindo CO2.

Eletrlise gnea OBS.:


O termo gnea vem do latim igneu: ardente Tanto na eletrlise como na pilha os fenmenos de reduo
a reao qumica provocada pela passagem de corrente el- e oxidao que ocorrem no ctodo e no nodo respectivamente,
trica atravs de um composto inico fundido (xidos ou sais). permanecem constantes.

UECEVEST 279

Apostilas UECEVEST mod3.indb 279 06/02/2011 10:03:59


F S I C O -QUMICA

Eletrlise aquosa Oxidao (perda de eltrons) no nodo


Uma substncia qualquer pode gerar ons em soluo aquo-
sa. Neste caso, os ons que podem vir a sofrer eletrlise podem descarrega-se mais
se originar dessa substncia ou da prpria auto-ionizao da OH (da gua) facilmente que os
gua. Apesar da auto-ionizao da gua ocorrer em baixa ex- nions oxigenados
tenso, ela oferece esta possibilidade de um ou outro on vir a Exemplo: NO3, PO3,
se descarregar. SO43 e fluoretos
Apenas uma espcie de on positivo ou negativo descarrega
por vez. Cada on possui uma voltagem adequada para se descar- descarrega-se menos
regar. O par ction/nion por apresentar menor voltagem para facilmente que os
descarregar ser o que vai descarregar primeiro. nions no-oxigenados
A auto-ionizao da gua fornece os ons H1+ (H3O1+) e OH1-. Exemplo: Cl, Br e I
H2O H+ + OH
OBS.:
ou ainda Descarga do H+
2H2O H2O + H+ + OH H3O+ + OH 2H+ + 2e H2
Descarga de OH
No caso dos ctions, metais pouco reativos descarregam pri- 2OH O2 + H2O + 2e
meiro, pois possuem maior tendncia a aceitar os eltrons de vol-
ta e se descarregar. leis de Faraday
Metais altamente reativos como metais alcalinos e alcalino Por volta de 1912, o fsico ingls Robert Millikan conseguiu
terrosos se descarregam depois. determinar experimentalmente a carga de um eltron.
O H+ ou H3O+ se descarregam antes do alumnio, de metais A carga de um eltron igual a 1,6 x 1019C (C=Coulomb),
alcalinos e alcalinos terrosos. Mas perdem em prioridade de des- que a unidade utilizada para cargas eltricas.
carga para os demais ctions. Em qumica, uma grandeza muito importante o mol.
Quanto menor for a eletronegatividade do nion para pren- 1e 1,6 x 1019C (carga de 1e)
der o eltron, maior a facilidade de descarga do mesmo. 6,02 x 10 23
x;
Ocorre com competitividade de ons. Pois numa mesma so- x= 6,02 x 1023 x 1,6 x 1019C
luo, alm dos ons do composto, temos os ons provenientes da
x= 96 500C
ionizao da gua (cidos, bases ou sais).
CA C+ + A
Assim, um mol de eltron transporta 96 500C.
H2O H+ + OH
Portanto,
Existem dois tipos de eletrlise aquosa: eletrlise com eletro-
dos inertes (grafite e platina) e a eletrlise aquosa com eletrodos 96 500C = 1F = carga de 1 mol de eltrons
ativos.
Ex: soluo aquosa de NaCl Em 1832, Michael Faraday descobria a relao que existe en-
tre a quantidade de matria depositada nos eletrodos e a quanti-
Gerador dade de eletricidade que passa numa soluo.
Faraday descobriu experimentalmente que a massa deposita-
e Eletrodos e

+
de carvo da nos eletrodos, para um determinado eletrlito, diretamente
proporcional carga que passa pela soluo.
Eletrodo A corrente eltrica que passa por um fio condutor constitu-
negativo da por um fluxo de eltrons. A intensidade da corrente simbo-
H2O Cl2 lizada por i e expressa em ampres (A). Sendo Q, a quantidade de
H2 Na+(aq) carga em Coulombs (C)
nodo
Ctodo Cl(aq) A cor rosa da fenolf- Q = i.t
Na Cl2 H+ OH talena indica a presea
(concentrado) de ons OH(aq) Gaiola de Faraday
Num corpo neutro, as cargas eltricas positivas e negativas
A eletrlise aquosa do cloreto de sdio produz hidrognio e distribuem-se pelo corpo.
cloro gasosos.

+ +
Tabela de ctions e nions (facilidade de descarga) +
Reduo (ganho) de eltrons no ctodo +
+ + +
descarrega-se mais +
H+ (da gua) facilmente que os ons de Se eletrizarmos o corpo, as cargas em excesso repelem-se mu-
metais alcalinos tuamente e concentram-se na periferia do corpo, na sua superf-
metais alcalino-terrosos cie exterior.
Mn2+, Al3+

descarrega-se menos
facilmente que os ons dos
outros metais

280 UECEVEST

Apostilas UECEVEST mod3.indb 280 06/02/2011 10:04:00


FSICO - Q U M I C A

Passado um curto tempo inicial aps a eletrizao, o corpo Equivalente eletroqumico (Eq): a quantidade de substn-
fica em equilbrio eletrosttico, no havendo movimentos de car- cia eletrolisada ou depositada, quando se faz passar uma carga de
gas eltricas a nvel macroscpico. 1 C na soluo.
Estes fatos comprovaram experimentalmente Michael Faraday E
ao encerrar-se no interior de uma gaiola condutora, onde verificou Eq =
F
no haver manifestao de fenmenos eltricos no seu interior.
Uma gaiola de Faraday, para alm de ser condutora, no ne-
cessita ser contnua, podendo ser constituda por uma rede met-
lica. Desta configurao lhe veio o nome de gaiola. A verificao EXERCCIO RESOlVIDOS
do seu comportamento eltrico pode ser feita colocando pndu- 01. Na reao 2H+ + 2e H2(g), a participao de 1mol de
los eltricos nas suas paredes interiores e exteriores e eletrosc- eltrons fornecer qual volume de gs hidrognio, medido nas
pios no seu interior. No havendo cargas eltricas no seu interior, CNTP (22,4L):
verifica-se que, ao eletrizar a gaiola por contato com um gerador Resoluo:
eletrosttico, os pndulos exteriores se desviam das paredes, acu- 2H+ + 2e H2(g)
sando a sua eletrizao, enquanto os interiores permanecem im- 2mols 2mols 1mol
veis, assim como as folhas dos eletroscpios, comprovando a no 1mols 1mols 1/2mol
existncia de cargas eltricas no interior da gaiola.
Duas leis de Faraday sintetizam as observaes experimentais. 1 mol ------------- 22,4 L
mol ------------- V V=11,2 L
1 lei de Faraday
Nos condutores em equilbrio a eletricidade distribuda 02. Uma pea de bijuteria recebeu um banho de prata (pratea-
apenas na superfcie externa ; no seu interior no h trao de o) por meio de um processo eletroltico. Sabendo-se que nessa
eletricidade. deposio o Ag+ reduz-se a Ag e que a quantidade de carga envol-
1 lei A massa que se forma em um eletrodo diretamen- vida no processo de 0,01F, qual a massa de prata depositada?
te proporcional quantidade de carga que atravessa o circuito. Resoluo:
Logo: m=k.Q Ag+ + e Ag
m = massa de substncia produzida; k = constante de propor-
cionalidade; Q= quantidade de carga em C 1mol 1mol
1F --------- 108g
2 lei de Faraday 0,01F -------- mAg mAg = 1,08g
No equilbrio eltrico a fora eltrica no interior dos condu-
tores completamente fechados e desprovidos de corpos eletriza- 03. Numa pilha de flash antiga, o eletrlito est contido numa lata
dos nula. de zinco que funciona como um dos eletrodos. Que massa de zinco
A gaiola de Faraday foi adotada para proteger instrumentos e oxidada a Zn+2 durante a descarga desse tipo de pilha, por um
aparelhos de grande sensibilidade colocados no seu interior. Tam- perodo de 30 minutos, envolvendo uma corrente de 5,36 10-1A?
bm serve para garantir a segurana de instalaes perigosas como Resoluo:
paiis e locais de preparao de explosivos. A proteo de edi- i = 5,36 101A
fcios contra descargas atmosfricas outra aplicao da gaiola. t= 1800s Q = i . t Q = 5,36 . 101 . 1800 Q=965C
Devido a esta funo de proteo, a gaiola tambm conhecida
como cran eletrosttico. 65
E.i.t . 5,36 . 101 . 1800
2 lei A massa que se forma em um eletrodo diretamente m= m= 2
proporcional ao equivalente de oxi-reduo da substncia. Logo:
96500 96500
m=k.E
m = massa de substncia produzida; k= constante de propor- m= 0,325g
cionalidade; E= equivalente de oxi-reduo.
Equivalente de oxi-reduo: a massa de um elemento que Zn Zn+2 + 2e ou
perde ou ganha um mol de eltrons. Veja o exemplo: 1 mol do 1 mol 2 mol
metal cobre (Cu) perde 2 moles de eltrons e forma um mol de 65g --------------- 2 96500C
ctions bivalentes (Cu+2), conforme o processo : mZn --------------- 965C
Cu Cu+2 + 2e- mZn = 0,325g
1mol 2mols
massa molar 04. Determinar o equivalente-grama de uma substncia formada
E= em um eletrodo, sabendo que, ao passar uma corrente de 9,65A
n de carga
de intensidade durante 8 minutos e 20 segundos, formam-se 1,4g
Dessa forma, teremos para o cobre o seguinte raciocnio: con- de substncia.
siderando que a massa molar do Cu 63,5 e que o Nox +2, Resoluo:
ento: m = 1,4g i = 9,65A t = 500g
63,5 E. i . t E . 9,65 . 500
E= 31,75g E= ? m= 1,4=
2 96500 96500
E = 28g
Levando em considerao o enunciado das duas leis de Fara-
day, a massa de substncia produzida em um eletrodo poder ser 05. Qual a quantidade de eletricidade obtida em uma pilha de
calculada por meio das seguintes relaes: Daniell pela oxidao de 0,2612g de zinco? Qual a intensidade
E.Q E.i.t da corrente produzida, sabendo-se que a pilha funcionou durante
E= e m= 25 minutos e 44 segundos? Dado: Zn=65,3u
96500 96500

UECEVEST 281

Apostilas UECEVEST mod3.indb 281 06/02/2011 10:04:01


F S I C O -QUMICA

Resoluo: estudo modificando-se apenas duas das grandezas e a outra se


Zn Zn+2 + 2e mantm constante.

1mol 1mol Lei de Boyle, Lei de Charles e Gay-Lussac
65,3g ----- 2 96500C Lei de Boyle-Mariotte: A temperatura constante, o volume
0,2612 --- Q Q = 772C ocupado por uma quantidade fixa de um gs inversamente pro-
t = 1544s Q = i t 772 = i 1544 0,5A = i porcional sua presso.
P.V = k = constante
06. Um rdio de pilha ficou ligado durante a partida de um jogo A lei de Boyle-Mariotte pode ser representada por um grfico
de futebol. Nesse perodo sua cpsula de zinco sofreu um desgaste presso-volume. Neste grfico, as abscissas representam a presso
de 0,3275g tendo originado uma corrente de 0,3216 A. Qual foi de um gs, e as ordenadas, o volume ocupado.
a durao da narrao do jogo, em minutos? Considere a massa V
atmica do zinco igual a 65,5u.
Resoluo:
Zn Zn+2 + 2e V
65,5g ----- 2 96500C
0,3275g ----- Q Q =965C V
Q = i t 965 = 0,316 t t = 3000s t = 50 minutos 2

P
P 2P
ESTUDO DOS GASES (UECE/ENEM)
A curva obtida uma hiprbole, cuja equao representativa
A matria pode assumir trs estados fundamentais ou fases: PV = constante. Portanto, podemos representar:
fase slida: possui volume e forma constantes. As molculas P1 . V1 = P2 . V2
esto mais prximas e apresentam o menor nvel energtico.
fase lquida: possui volume constante e forma varivel. As Lei de Charles e Gay-Lussac
molculas esto mais separadas que na fase slida e apresentam presso constante, o volume ocupado por uma massa fixa de
um nvel energtico intermedirio. gs diretamente proporcional temperatura absoluta.
fase gasosa: possui volume e forma variveis. As molculas Esta transformao gasosa, onde a presso mantida constante,
esto mais separadas e apresentam um nvel energtico mais chamada de transformao isobrica.
elevado, a energia cintica mdia das molculas a medida da Graficamente, encontramos:
temperatura.
V
Os gases apresentam caractersticas importantes:
no possuem volume prprio;
tm uma grande compressibilidade e uma grande expansibilidade; 2V
possuem uma grande difusibilidade.

Transformaes gasosas V
Todo gs constitudo de partculas (molculas) que esto
em contnuo movimento desordenado. Esse movimento de um
grande nmero de molculas provoca colises entre elas e, por T
T 2T
isso, sua trajetria no retilnea num espao aprecivel, mas sim
caminham em ziguezague. Essas colises podem ser consideradas A reta obtida representada pela equao:
perfeitamente elsticas. V = (constante) . T ou V/T = constante
O estado em que se apresenta um gs, sob o ponto de vista Com isso, ficamos com:
microscpico, caracterizado por trs variveis: presso, volume
V1 V2
e temperatura. So denominadas variveis de estado. =
T1 T2
Volume
A volume constante, a presso exercida por uma determinada
1mL = 1 cm3 1L = 1000mL 1dm3 = 1Litro massa fixa de gs diretamente proporcional temperatura absoluta.
Esta transformao gasosa, onde o volume mantido constante,
Presso denominada de transformao isocrica, isomtrica ou isovo-
1 atm = 760 mmHg = 760 Torr lumtrica.
Graficamente, encontramos:
OBS.: P
A presso atmosfrica ao nvel do mar corresponde a 1atm ou
760 mmHg e decai com a altitude.
2P
Temperatura
TK = 273 + TC
P
Leis fsicas dos gases
Uma dada massa de gs sofre uma transformao quando T
ocorrem variaes nas suas variveis de estado. Comeamos o T 2T

282 UECEVEST

Apostilas UECEVEST mod3.indb 282 06/02/2011 10:04:02


FSICO - Q U M I C A

A reta obtida representada pela equao: Sendo n a quantidade em mols de cada gs, podemos concluir:
P = (constante) .T ou P/T = constante nN2 = nO3 = nHe
Com isso, ficamos com:
P1 P2 Determinou-se experimentalmente o volume ocupado por 1
= mol de qualquer gs nas CNTP e foi encontrado o valor aproxi-
T1 T2
madamente igual a 22,4 L. Portanto, podemos dizer que:
Gs perfeito ou ideal Volume molar nas CNTP = 22,4L/mol
Obedece rigorosamente s Leis Fsicas dos Gases em quais-
quer condies de temperatura e presso. um gs que obedece Equao de Clapeyron
s equaes p.V/T = k e p.V = n.R.T, com exatido matemtica. As leis de Boyle e Charles/Gay-Lussac podem ser combinadas
com a lei de Avogadro para relacionar volume, presso, tempera-
Gs real tura e quantidade em mols de um gs.
No segue o comportamento do gs ideal, principalmente em Tal relao chamada de equao de estado de um gs. Ela
presses muito altas e/ou em temperaturas baixas, porque ocorre pode ser encontrada das seguintes formas:
alta reduo de volume e as partculas, muito prximas, passam a
interferir umas no movimento das outras. I. lei de Boyle-mariotte
Um gs real aproxima-se do comportamento de um gs ideal V proporcional a 1/P quando T e n so constantes.
medida que diminui a presso e aumenta a temperatura. II. Lei de Charles/Gay-Lussac
Equao geral dos gases V proporcional a T onde P e n so constantes.
Esta equao utilizada quando ocorre transformao gasosa P proporcional a T onde V e n so constantes.
em que as trs variveis de estado (P, V e T) se modificam simul- III. Lei de Avogadro
taneamente. V proporcional a n quando T e P so constantes. Agrupan-
Ela obtida por meio da relao matemtica entre as transfor- do as quatro expresses encontramos:
maes gasosas estudadas anteriormente. V proporcional a 1/P. (T). (n) ou
V = R .(1/P).(T). (n), onde R representa a constante de pro-
I. Isotrmica porcionalidade e chamada de constante universal dos gases. A
P1 . V1 = P2 . V2 equao de estado pode ento ser representada por:
P.V = n.R.T
II. Isobrica
V1 V2 P1 . V1 P2 . V2 Esta equao tambm denominada de equao de Cla-
= =
T1 T2 T1 T2 peyron, em homenagem ao fsico francs que a determinou.
A constante R pode assumir vrios valores dentre os quais des-
III. Isocrica tacamos:
P1 P2 R = 0,082atm . L . mol-1 . K-1
= e
T1 T2
R = 62,3mmHg . L . mol-1 . K-1
Grfico da Transformao Gasosa
P Estado inicial 1 T1 Densidade de um gs
A densidade gasosa pode ser trabalhada sob duas formas: a
Transformao isotrmica densidade absoluta e a densidade relativa.
P1 4
Estado intermedirio
3 Densidade Absoluta
Transformao usibrica A densidade absoluta uma relao entre a massa e o volume
P2 2 Estado final 2 T2 ocupado por um gs, em determinadas condies de temperatura
e presso.
1
m
d=
V1 V V2 V V

Condies Normais de Temperatura e Presso (CNTP, CN Esta densidade pode ser encontrada, levando-se em conside-
ou TPN) rao a presso e a temperatura absoluta, partindo-se da equao
So definidas como condies normais de temperatura e pres- de estado do gs ideal:
so quando o gs submetido a uma presso de 1 atm e tempe- P V = n. R. T
ratura de 0 C. Portanto, podemos colocar: m
P = 1 atm = 760 mmHg P. V = R. T
M
T = 0 C = 273 K
m
lei de Avogadro P- M = R. T
V
Volumes iguais de gases quaisquer, mesma temperatura e pres-
so, encerram o mesmo nmero de molculas. Sabendo que: d = m/V
PM=dRT
P.M
d=
R.T
A unidade utilizada o g/L.

UECEVEST 283

Apostilas UECEVEST mod3.indb 283 06/02/2011 10:04:03


F S I C O -QUMICA

Trabalhando-se nas CNTP (P = 1 atm e T = 273 K), encon- em todas as direes e a altas velocidades. Desta maneira, rapi-
tramos a seguinte equao: damente elas ocuparo todo o recipiente, adotando ento o seu
1.M volume. Durante o movimento, elas iro colidir entre si e tam-
d= bm com as paredes do recipiente. Cada coliso sobre as paredes
0,082 . 273 ir exercer uma pequena fora sobre ela. Como a presso de-
M finida pela fora exercida sobre uma determinada rea (P = F /
d= A), a presso total ser dada pelo somatrio de todas as colises
22,4 sobre a rea total do recipiente. A relao entre as diversas gran-
dezas que definem o estado gasoso foram descritas pelas: Lei
Densidade Relativa de Avogadro, Lei de Boyle e Lei de Charles. Buscando uma lei
A densidade relativa encontrada atravs da relao entre as geral para descrever o Estado Gasoso foi proposta inicialmente a
densidades absolutas de dois gases, medidas nas mesmas condi- Lei dos Gases Combinada, associando as Leis de Boyle e Char-
es de temperatura e presso. les, e finalmente a Lei dos Gases Ideais, a qual conseguiu reunir
as trs leis bsicas. Utilizando ainda os conceitos de presso e
P . M1 P . M1 volume parciais, pela Lei de Dalton, pode-se utilizar e aplicar
d1 = estas leis para misturas gasosas.
R.T d1 R.T
=
P . M2 d2 P . M2 Resumindo
d2 =
R.T R.T Nas transformaes gasosas podemos modificar tanto a tem-
peratura, o volume ou a presso. Para cada tipo de transformao
existe uma lei:
d1 M1 M1
= ou d1,2 = Transformao Lei Nome da Lei
d2 M2 M2 Isotrmica P1V1 = P2V2 Boyle-Mariote
Esta relao indica quantas vezes um gs mais denso ou Isovolumtrica P1/T1 = P2/T2 Gay-Lussac
menos denso que outro gs. Por exemplo, uma bexiga com gs Isobrica V1/T1 = V2/T2 Charles
hidrognio mantm-se suspensa no ar porque o gs hidrognio
menos denso que o ar. Difuso e Efuso dos Gases (Lei de Grahan)
Difuso disperso gradual de uma substncia em outra.
Mistura Gasosa
A mistura entre dois ou mais gases sempre constitui um sis- Lei da Efuso de Graham
tema homogneo. Velocidade de efuso 1/ M
Consideremos inicialmente dois recipientes contendo, o pri- Velocidade de efuso 3RT/M
meiro, gs nitrognio (N2) e o segundo, gs hlio (He).
Os dois gases so misturados em um terceiro recipiente, con- Velocidade de efuso A / velocidade de efuso B =MB/MA
forme o esquema representado abaixo. Velocidade de efuso velocidade mdia das molculas no gs
Velocidade mdia das molculas A / velocidade mdia das
N2 inicial He inicial molculas B =MB/MA
PN VN TN PH VH TH Velocidade de efuso aumenta com a raiz quadrada da
2
nN
2 2 e
nH
e e temperatura
2 e Velocidade de efuso em T2 / velocidade de efuso em T1
= T2/ T1
Velocidade mdia das molculas T2 / velocidade mdia das
molculas T1 =T2 / T1
P V T VELOCIDADE MDIA DAS MOLCULAS EM UM
n = nN + nH GS T/M
2 e
O volume de todas as molculas do gs desprezvel, se com-
Mistura (N2 + He) parado ao volume total no qual o gs est contido;
As foras atrativas e repulsivas entre as molculas de gs so
A teoria cintica dos gases desprezveis.
O modelo utilizado para descrever o estado gasoso a Teoria Para dois gases nas mesmas condies de presso e tempera-
Cintica dos Gases. Esta teoria baseia-se nos seguintes postulados: tura tem-se que:
As unidades elementares encontram-se em rpido e constante
movimento em linha reta; VelocidadeA dA
As distncias entre elas so muito grandes em relao ao seu = dB
tamanho; VelocidadeB
Pode ser desprezado a interao entre as unidades elementares; ou
Quando ocorrem colises entre as unidades elementares, a VelocidadeA MA
energia conservada; =
A temperatura uma medida da energia cintica mdia das VelocidadeB MB
unidades elementares.
Temperatura Crtica
Alm da temperatura, duas outras grandezas so funda- Existe uma temperatura, denominada temperatura crtica (2)
mentais na descrio de um gs, que so a presso e o volume. acima da qual, por maior que seja a presso, a substncia encontra-
Quando um gs colocado em um recipiente fechado, as suas se no estado gasoso. Por isso costume fazer uma distino entre
unidades elementares estaro se movimentando em linha reta, gs e gs e vapor:

284 UECEVEST

Apostilas UECEVEST mod3.indb 284 06/02/2011 10:04:04


FSICO - Q U M I C A

gs uma substncia no estado gasoso, acima da temperatura 03. Uma equipe tenta resgatar um barco naufragado que est a 90
critica. m de profundidade. O poro do barco tem tamanho suficiente
vapor uma substncia no estado gasoso abaixo da temperatura para que um balo seja inflado dentro dele, expulse parte da gua
crtica. e permita que o barco seja iado at uma profundidade de 10
m. O balo dispe de uma vlvula que libera o ar, medida que
Desse modo, os diagramas de estado ficam com os aspectos o barco sobe, para manter seu volume inalterado. No incio da
das Fig. I (substncias que se expandem na fuso) e da Fig. II operao, a 90 m de profundidade, so injetados 20.000 mols de
(substncias que se contraem na fuso). Nessas figuras, C o ar no balo. Ao alcanar a profundidade de 10 m, a porcentagem
ponto crtico, definido pela temperatura crtica(2) e pela presso do ar injetado que ainda permanece no balo : (Presso na su-
crtica pc. perfcie do mar = 1 atm; No mar, a presso da gua aumenta de
1 atm a cada 10 m de profundidade. A presso do ar no balo
sempre igual presso externa da gua).
a) 20 % d) 80 %
b) 30 % e) 90 %
c) 50 %

04. Considere os gases NH3 e CO2 nas mesmas condies de


presso e temperatura. Podemos afirmar corretamente que a rela-
o entre as velocidades de difuso dos mesmos, vNH3/vCO2,
igual a: (Massas Molares: C = 12 g.mol1; O = 16 g.mol1; N = 14
g.mol1; H = 1 g.mol1)
a) 2,0 d) 0,6
b) 1,6 e) 1,0
c) 1,4

05. Usando os conceitos relacionados ao estudo dos gases, pode-


mos afirmar CORRETAMENTE que:
a) atravs da Lei de Boyle, possvel comprovar que, a uma
temperatura constante, o volume ocupado por uma massa
fixa de um gs diretamente proporcional presso.
b) de acordo com a Teoria Cintica Molecular dos gases, um gs
formado por molculas em constante movimento e, em um
gs ideal, no h atrao nem repulso entre as molculas.
c) pela Lei de Charles, para transformaes isobricas,
E X E R C C I O o volume de um gs inversamente proporcional
temperatura absoluta.
01. Algumas companhias tabagistas j foram acusadas de adicio- d) pela Lei de Gay-Lussac, proposta em 1802, quando uma
narem amnia aos cigarros, numa tentativa de aumentar a libe- massa varivel de um gs sofre transformao isocrica, a
rao de nicotina, o que fortalece a dependncia. Suponha que presso do gs ser diretamente proporcional temperatura
uma amostra de cigarro libere 2,0 104mol de amnia, a 27C e absoluta do sistema.
1 atm. (Dado: R = 0,082 atm L / K x mol). e) pela Hiptese de Avogadro, gases diferentes, nas mesmas
O volume de NH3gasoso, em mL, ser, aproximadamente condies de volume, de presso e de temperatura, sempre
a) 49 d) 0,049 apresentaro diferentes nmeros de molculas.
b) 4,9 e) 0,0049
c) 0,49 06. Quanto aos gases, CORRETO afirmar que:
a) sob presso de uma atmosfera e temperatura ambiente, um
02. Considere o texto a seguir: mol de qualquer gs ocupa o volume de 22,4 litros
Cavendish ficou intrigado pelo gs que era produzido quan- b) a equao de estado que relaciona volume, temperatura,
do certos cidos reagiam com metais. [...] Descobriu que esse presso e massa de um gs chamada equao de Clapeyron
novo gs tinha uma densidade de apenas 1/14 da do ar. Observou c) nas transformaes isomtricas, o volume varia, enquanto a
tambm que, quando uma chama era introduzida numa mistura temperatura e a presso permanecem constantes
desse gs com ar, o gs pegava fogo. Por isso chamou-o de ar d) a 1 atm, 760 mmHg e 273 K, o volume de um mol de gs
inflamvel dos metais. [...] Cavendish pensou que o ar inflam- depende de sua posio na tabela peridica
vel vinha de fato dos metais, no do cido. Como a maioria dos e) a expresso PV = nRT representa a lei de Boyle
qumicos, seus contemporneos, ele tambm aceitava a teoria do
flogstico, acreditando que os metais eram uma combinao de 07. Em determinadas condies, o dixido de nitrognio (NO2)
cinza metlica e flogstico. Isso, juntamente com a leveza e in- pode ser formado a partir de nitrognio (N2) e oxignio (O2).
flamabilidade excepcionais do ar inflamvel, o levou conclu- Considere um recipiente de 24 L com mbolo mvel em que h
so sensacional de que havia conseguido isolar o flogstico. (Paul uma mistura estequiomtrica de nitrognio (N2) e oxignio (O2),
Strathern, O sonho de Mendeleiev) ou seja, a presso parcial de nitrognio no recipiente metade da
presso parcial do oxignio. A quantidade mxima que pode ser
As informaes contidas no texto permitem concluir que o gs obtida do gs dixido de nitrognio, mantidas as condies de
observado por Cavendish era o temperatura e presso,
a) O2 d) CO2 a) 8 L. d) 24 L.
b) N2 e) CH4 b) 12 L. e) 46 L.
c) H2 c) 16 L.

UECEVEST 285

Apostilas UECEVEST mod3.indb 285 06/02/2011 10:04:05


F S I C O -QUMICA

08. Um frasco de 250 mL contm nenio a uma presso de 0,65


atm. Um outro frasco de 450 mL contm argnio a uma presso
de 1,25 atm. Os gases so misturados a partir da abertura de uma
vlvula na conexo que liga os dois recipientes. Considerando
o volume da conexo desprezvel e, ainda, o sistema mantido a
uma temperatura constante, a presso final da mistura de gases ,
aproximadamente,
a) 1,03 atm. d) 2,80 atm.
b) 1,90 atm. e) 5,6 atm.
c) 2,06 atm.

09. Os gases do estmago, responsveis pelo arroto, apresentam


composio semelhante a do ar que respiramos: nitrognio, oxi-
gnio, hidrognio e dixido de carbono. Nos gases intestinais,
produzidos no intestino grosso pela decomposio dos alimentos,
encontra-se tambm o gs metano. Considerando cada gs indivi-
dualmente, qual seria a ordem esperada de liberao destes para o
ambiente, em termos de suas velocidades mdias de difuso no ar?
a) N2, O2, CO2, H2, CH4
b) H2, N2, O2, CH4, CO2
c) H2, CH4, N2, O2, CO2
d) CO2, O2, N2, H2, CH4
e) CH4, CO2, N2, O2, H2

10.A calibrao dos pneus de um automvel deve ser feita perio-


dicamente. Sabe-se que o pneu deve ser calibrado a uma presso
de 30 lb/pol2em um dia quente, a uma temperatura de 27C.
Supondo que o volume e o nmero de mol injetados so os mes-
mos, qual ser a presso de calibrao (em atm) nos dias mais
frios, em que a temperatura atinge 12C? Dado: Considere 1
atm = 15 lb/pol2.
a) 1,90 atm. d) 0,89 atm.
b) 2,11 atm. e) 14,3 atm.
c) 4,50 atm.

G A B A R I T O
01. b 02. c 03. a 04. b 05. b
06. b 07. c 08. a 09. c 10. a

REFERnCIAS BIBlIOGRFICAS
COMPLETAMENTE QUMICA
AUTORA: MARTHA REIS
Vol II, 3 edio. So Paulo: FTD, 2001.

CURSO COMPLETO DE QUMICA.


AUTOR: ANTNIO SARDELLA
2 edio. So Paulo: tica, 1996.

http://www.profpc.com.br/gases.htm

http://www.vestibular1.com.br/revisao/r254.htm

http://quimica-exercicios.blogspot.com/2009/09/estudo-dos-
gases.html

http://www.scribd.com/doc/19542660/Apostila-de-Fisica-Para-
Vestibular

286 UECEVEST

Apostilas UECEVEST mod3.indb 286 06/02/2011 10:04:05


P R - V E S T I B U l A R

BIOLOGIA 1

Apostilas UECEVEST mod3.indb 287 06/02/2011 10:04:10


Caro(a) Aluno(a),

Para facilitar o acompanhamento de tais contedos, abaixo esto indicadas as nomen-


claturas utilizadas pela UECE e pelo ENEM:
CONTEDO UECE ENEM
Tecido Muscular/
Histologia Molculas, Clulas e Tecidos
Tecido Nervoso
Identidade dos seres vivos:evoluo
Sistema Digestrio/ Anatomofisiologia
e padres anatmicos e fisiolgicos
Sistema Circulatrio Comparada
observados nos seres vivos
Identidade dos seres vivos:evoluo
Sistema Respiratrio/ Anatomofisiologia
e padres anatmicos e fisiolgicos
Sistema Excretor Comparada
observados nos seres vivos
Identidade dos seres vivos:evoluo
Sistema Nervoso/
Sistemas Integradores e padres anatmicos e fisiolgicos
Sistema Endcrino
observados nos seres vivos

Apostilas UECEVEST mod3.indb 288 06/02/2011 10:04:11


BI O LO G I A 1

TECIDO mUSCUlAR musculatura estriada que se recorre para mover os membros superio-
res e inferiores. H filamentos de actina e de miosina entrelaados, o
O tecido muscular faz parte do corpo humano e caracterizado que confere o tpico aspecto estriado desse tecido muscular.
por sua contratilidade, ou seja, pela capacidade de se contrair segun-
do alguns estmulos, utilizando ATP (molcula orgnica responsvel Tecido muscular estriado cardaco
pelo armazenamento de energia em suas ligaes qumicas) como Ocorre nas paredes do corao, formando o miocrdio. A forma
forma de energia e por sua excitabilidade, ou seja, capacidade de res- de suas fibras alongada e cilndrica e elas se ramificam e se unem
ponder a um estmulo nervoso. As clulas desse tecido so de origem de espao a espao. Esse tecido tem contrao rpida, involuntria
mesodrmica, e sua diferenciao se d atravs da sntese de protenas e rtmica, possuindo clulas que se associam por discos intercalares.
especficas com uma organizao determinada, tais como os diferen-
tes tipos de actinas, miosinas e protenas motoras filamentosas. Essas
clulas, aps sofrerem diferenciao, assumem a forma de fibras; nes-
tas, as estruturas celulares recebem nomes especiais:
Estrutura Fibra muscular
Membrana Plasmtica Sarcolema
Citoplasma Sarcoplasma
Retculo Endoplasmtico Retculo Sarcoplasmtico
Mitocndrias Sarcossomas Contrao muscular
De acordo com o modelo de Huxley, a contrao muscular
Os tecidos musculares so diferenciados por suas caracters- consequncia de uma srie de eventos:
ticas morfo-funcionais. Existem trs tipos principais de tecidos O impulso nervoso alcana a clula muscular por meio de uma
musculares: Tecido Muscular Liso, Tecido Muscular Estriado Es- sinapse especial, chamada sinapse neuro-muscular ou placa
queltico e Tecido Muscular Estriado Cardaco. motora. H liberao de mediadores qumicos nas placas mo-
toras que excitam as clulas musculares estriadas e o mediador
Classificao do tecido muscular qumico a acetilcolina.
O mediador qumico atinge a membrana plasmtica da clula
Tecido muscular liso muscular, que recebe o nome especial de sarcolema. Uma vez
Tambm denominado clula muscular lisa, apresenta-se a clula muscular tendo sido excitada, um potencial de ao
geralmente alongado e fusiforme, com as extremidades agudas, desencadedo na clula muscular.
possui apenas um ncleo e de posio central. Suas clulas apre- Esse potencial de ao propaga-se por todo o sarcolema e tam-
sentam apenas estrias longitudinais e no possuiem estrias trans- bm pelo retculo sarcoplasmtico (RS), nome que recebe o
versais. encontrado nas paredes das vsceras: estmago, intesti- retculo endoplasmtico dessas clulas. Com o estmulo, as
no, etc, e tambm nas extremidades dos vasos sanguneos, para membranas do RS tornam-se permeveis aos ons clcio. An-
a manuteno da presso arterial. Sua contrao lenta e no teriormente armazenados nas cisternas do RS, os ons clcio
depende da vontade, ou seja, involuntria. penetram nos sarcmeros e colocam-se em contato com as mo-
lculas de actina e de miosina.
Tecido muscular estriado esqueltico Na presena de clcio, as molculas de miosina adquirem ati-
vidade cataltica (atividade ATPsica) e comeam a degradar
molculas de ATP, convertendo-as em ADP.
Com a energia liberada pela hidrlise do ATP, as molculas de
miosina deslizam sobre as de actina, encurtando os sarcmeros.
Com o encurtamento dos sarcmeros, as miofibrilas como um
todo encurtam, diminundo o comprimento da clula inteira.

encontrado aderido ao esqueleto. A forma de suas fibras alon-


gada, cilndrica e suas extremidades so arredondadas. Cada fibra
do msculo estriado apresenta vrios ncleos de posio perifrica.
Apresenta estriaes transversais e longitudinais. Sua contrao r-
pida, mas possui pouca capacidade de se manter contrado. O tipo
de controle da fibra voluntrio. Cada msculo estriado esqueltico
constitudo por milhes de clulas contrteis. O citoplasma dessas
clulas repleto de unidades contrteis filamentares chamadas mio-
fibrilas, que se dispem longitudinalmente no interior das clulas.
Por sua vez, cada miofibrila formada por unidades longitudinais OBS.: Energtica da Contrao Muscular.
que se repetem: os sarcmeros. Cada um deles possui de 2 a 3mm
de comprimento, de tal forma que cada clula muscular chega a No processo da respirao celular, as clulas musculares origi-
alcanar at vrios centmetros de comprimento. Outro organide nam molculas de ATP, a partir da oxidao da glicose. Durante
encontrado em abundncia, no citoplasma das clulas musculares, o repouso, pequena quantidade desse ATP consumida, sendo a
a mitocndria, o que fcil de se explicar, pois a mitocndria a maior parte armazenada na forma de fosfocreatina, originrias da
sede da respirao celular, processo que origina o ATP consumido juno da creatina com um grupamento fosfato rico em energia,
durante a contrao muscular. Em cada sarcmero, encontram-se proveniente da molcula do ATP:
dois tipos de protenas, em um arranjo uniforme e caracterstico. a ATP + creatina ADP + fosfocreatina

UECEVEST 289

Apostilas UECEVEST mod3.indb 289 06/02/2011 10:04:12


B I O LO GIA 1

Durante os perodos de repouso do msculo, ocorre armaze- o. As Clulas de Schwann podem envolver um neurnio to-
namento de fosfocreatina no citoplasma das clulas musculares. talmente (mielinizado) ou parcialmente (no mielinizado).
Na contrao, a clula muscular consome essa fosfocreatina acu-
mulada nos perodos de repouso: Micrglia (= Fagcitos)
fosfocreatina + ADP creatina + ATP Ingerem e destroem bactrias e clulas; ao morrerem, segre-
ATP ADP + fosfato + energia (contrao) gam protenas que atraem macrfagos do sistema imunitrio. No
final da limpeza, estes ou permanecem, ou tambm se transfor-
Quando o trabalho muscular muito intenso, e apenas a res- mam em micrglia. So clulas pequenas, com poucos prolonga-
pirao celular aerbica no suficiente para suprir todo o ATP mentos, geralmente muito ramificadas.
consumido na contrao, as clulas musculares passam a execu-
tar, alm da respirao aerbica, a fermentao lctica. Embora Os Neurnios
bem menos rentvel que o outro processo, a fermentao permite So clulas alongadas, dotadas de um corpo celular (ou pe-
que mais ATP seja produzido, sem demanda extra de oxignio. ricrio) e numerosos prolongamentos. Esses prolongamentos do
Entretanto, o acmulo de cido lctico no tecido muscular tem neurnio podem ser de dois tipos:
alguns inconvenientes. Pode causar muita dor muscular, fadiga Dendritos: (do grego dendron, rvore) prolongamentos bas-
(incapacidade de responder adequadamente aos estmulos) e ci- tante ramificados que tm a funo de captar estmulos;
bras. Vale ressaltar, entretanto, que h outras causas de cibras. Axnio: prolongamento terminado em ramificaes que delimi-
tam pequenas dilataes, onde se abrigam os neuro-hormnios
(responsvel pela transmisso do impulso nervoso). Em toda sua
extenso, o axnio envolvido pela bainha de Schwann. Em
muitos axnios, as Clulas de Schwann determinam a formao
Tecido nervoso da bainha de mielina (invlucro lipdico que atua como isolante
eltrico e facilita a transmisso do impulso nervoso). Entre uma
O tecido nervoso atua com uma estrutura sensvel que, ao ser clula de Schwann e outra existe uma regio de descontinuidade
estimulado, conduz os impulsos nervosos de maneira rpida e, s da bainha de mielina, que acarreta a existncia de uma constrio
vezes, por distncias grandes. O tecido nervoso (origem ectodr- ou estrangulamento denominada ndulo de Ranvier.
mica) a mais alta expresso de especializao para o desempenho
de duas propriedades da matria viva: irritabilidade, que permite
s clulas responderem a um estmulo interno ou externo e con-
dutibilidade, que permite conduzir uma onda de excitao (im-
pulso nervoso) por toda a clula. O sistema nervoso constitudo
de neurnios (clulas nervosas) e de uma variedade de clulas de
manuteno, sustentao e nutrio denominadas neurglias.
neurnios astrcidos
Tecido nervoso macrglia oligodendrcitos
neurglia clulas de
micrgria shwann
As fibras nervosas
So formadas pelos prolongamentos dos neurnios (dendri-
Neurglia tos ou axnios) e seus envoltrios. Cada fibra nervosa envol-
Inicialmente considerada como uma simples cola, da a eti- vida por uma camada conjuntiva denominada endoneuro. As
mologia do seu nome; A neurglia , efetivamente, constituda fibras nervosas organizam-se em feixes. Cada feixe envolvido
por clulas que transmitem informao. Podem estar ativamente por uma bainha conjuntiva denominada perineuro. Vrios feixes
envolvidas em vrias doenas degenerativas como o mal de Al- agrupados paralelamente formam um nervo. O nervo tambm
zheimer, por exemplo. Geralmente arredondadas, so de 10 a 15 envolvido por uma bainha de tecido conjuntivo chamada epi-
vezes mais numerosas que os neurnios. neuro. Quando partem do encfalo, os nervos so chamados de
Estudos recentes tm mostrado a grande importncia dos gli- cranianos; quando partem da medula espinhal denominam-se ra-
citos; acredita-se, por exemplo, que, sem essas clulas, os neurnios quidianos. De acordo com a direo da transmisso do impulso
no conseguiriam estabelecer conexes uns com os outros. Nos pri- nervoso, os nervos podem ser:
meiros anos de vida, o tamanho do encfalo aumenta multiplicao Sensitivos: quando transmitem os impulsos nervosos dos r-
dos glicitos, j que o nmero de neurnios se mantm praticamen- gos receptores at o SNCentral;
te o mesmo (cerca de 100 bilhes) desde o nascimento. H quem Motores: quando transmitem os impulsos nervosos do SN-
correlacione esse aumento de glicitos durante a infncia com o Central para os rgos efetores.
grande aumento no nmero de conexes (sinapses) entre neurnios,
fundamental ao desenvolvimento fsico e mental do ser humano. A Conduo do Impulso Nervoso.

Macrglia
Astrcitos: de forma estrelada, so responsveis pela sinalizao
celular. A comunicao neurnio-astrcito d-se em ambas as
direes; os ps dos astrcitos ligam neurnios e vasos sangu-
neos, exercendo funo nutritiva.
Oligodendrcitos e Clulas de Schwann: fabricao da mielina
a partir de lpidos e protenas. Os neurnios do SNcentral so
revestidos por oligodendrcitos; os neurnios do SNperifrico
so revestidos por Clulas de Schwann, consideradas as nicas
clulas de sustentao do SNPerifrico; as Clulas de Schwann
tambm tm caractersticas fagocitrias no caso de uma infec-

290 UECEVEST

Apostilas UECEVEST mod3.indb 290 06/02/2011 10:04:13


BI O LO G I A 1

Quando, em repouso, o neurnio apresenta carga eltrica ex- afirmar, corretamente, que as protenas actina e Miosina esto
terna positiva e interna negativa. Diz-se, ento, que o neurnio envolvidas no processo de:
em repouso est polarizado. Diante de um estmulo nervoso ade- a) transporte de oxignio no tecido sanguneo
quado a permeabilidade da membrana ao sdio aumenta, o que b) cobertura protetora da pele
acarreta um fluxo desses ons para o interior do neurnio, deter- c) contrao muscular
minando uma inverso da polaridade, o ambiente interno torna- d) sinapse nas terminaes nervosas
se positivo e o ambiente externo torna-se negativo. Num segundo
momento, a membrana torna-se permevel ao potssio, que migra 02. Considerando as caractersticas do tecido muscular, some as
para o meio externo, possibilitando o retorno ao potencial primiti- caractersticas corretas:
vo de repouso. Assim, a membrana torna-se novamente positiva 01. O tecido muscular de origem mesodrmica.
no lado externo e negativa no lado interno. A inverso de polarida- 02. Todos os tecidos musculares so formados por um s tipo de
de da membrana determina o surgimento de potencial de ao (al- fibra muscular.
terao eltrica durante a passagem do impulso) que se alastra ao 04. O tecido muscular esqueltico tem estreitas relaes
longo do neurnio, de forma a gerar um impulso nervoso. medi- funcionais com tecido sseo e nervoso.
da que o impulso nervoso se propaga, ocorrem sucessivas inverses 08. Nos mecanismos de contrao, a nvel celular, esto
de polaridade e sucessivos retornos ao potencial de repouso. A necessariamente envolvidos o retculo sarcoplasmtico e as
presena de bainha de mielina acelera a velocidade da conduo do mitocndrias.
impulso nervoso. Nas regies dos ndulos de Ranvier, a onda de 16. O sarcmero s encontrado em fibras musculares estriadas.
despolarizao salta diretamente de um ndulo para outro, no
acontecendo em toda a extenso da regio mielinizada (a mielina 03. (UFC) A carne escura das pernas e coxas de galinhas constitu-
isolante). Fala-se em conduo saltatria e com isso h um consi- da, principalmente, por um certo tipo de fibra muscular diferente
dervel aumento da velocidade do impulso nervoso daquela da sua carne branca, o que torna essas fibras adaptadas a
diferentes tipos de atividades. Observe as trs afirmativas abaixo.
As sinapses I. a carne escura das pernas e coxas tem fibras musculares
So articulaes terminais estabelecidas entre um neurnio e ricas em mioglobina, fornecendo oxignio s mitocndrias
outro (interneurais) ou entre um neurnio e uma fibra muscular durante esforos musculares prolongados;
(neuromuscular) ou entre um neurnio e uma clula glandular II. a carne branca dos msculos peitorais tem fibras musculares
(neuroglandulares). Entre um neurnio e outro existe um mi- relativamente pobres em mioglobina, sendo suas contrao
croespao, denominado sinapse, na qual um neurnio transmite rpida, mas no mantida por muito tempo;
o impulso nervoso para o outro atravs da ao de mediadores III. as fibras musculares lentas esto adaptadas realizao
qumicos ou neuro-hormnios. de trabalho contnuo, possuindo menor quantidade de
mitocndrias e pouca irrigao sangunea.
Assinale a alternativa correta.
a) apenas a afimativa I est correta.
b) apenas as afimativas II e III esto corretas.
c) apenas I e III esto corretas.
d) apenas a afirmativa III est correta.
e) apenas as afirmativas I e II esto corretas.

04. (UECE) Os discos intercalados so estruturas especiais que


prendem as clulas do tecido constituinte do seguinte rgo:
a) perna. c) lngua.
Os neuro-hormnios esto contidos em microvesculas pre- b) estmago. d) corao.
sentes na extremidade do axnio. Como os neuro-hormnios
so capazes de transmitir o impulso nervoso, acham-se presentes 05. (PUC-PR) Com relao conduo do impulso nervoso nos
apenas nas extremidades dos axnios, conclui-se que o sentido neurnios podemos afirmar:
de propagao do impulso ao longo do neurnio tem o seguinte I. A fibra nervosa s responde ao estmulo que ultrapassar o
trajeto: Dendrito Corpo Celular Axnio. Aps a transmis- limiar de excitao.
so do impulso, a acetilcolina (liberada na sinapse) decomposta II. A transmisso do impulso nervoso um processo
pela ao da enzima colinesterase, impedindo a passagem cont- eletroqumico e no somente eltrico.
nua do impulso nervoso. III. A conduo do impulso nervoso ocasiona, respectivamente,
uma migrao do K+ para o interior e Na+ para o exterior
OBS.: Afeces do Sistema Nervoso. do axnio.

O sistema nervoso suscetvel a infeces provocadas por Assinale a alternativa correta.


uma grande variedade de bactrias, vrus e outros microorganis- a) apenas as afimativas II e III estam corretas.
mos (meningite, poliomielite e encefalite). Em certas afeces, b) todas as afimativas esto corretas.
como a neuralgia, a enxaqueca ou a epilepsia, pode no haver c) apenas I e III esto corretas.
nenhuma evidncia de dano orgnico. Outra doena, a paralisia d) apenas a afirmativa III est correta.
cerebral, est associada a uma leso cerebral. e) apenas as afirmativas I e II esto corretas.

06. (U.F. Juiz de Fora-MG) O crack uma droga que atua no


crebro, alterando a fisiologia das sinapses nervosas, o que pode
E X E R C C I O levar a paradas cardacas e convulses. Sobre as sinapses entre
neurnios incorreto afirmar:
01. (UECE 2008.1) Alm de participar da construo do corpo a) Possuem mediadores qumicos responsveis pela transmisso
dos organismos, as protenas exercem diversas funes. Podemos do impulso nervoso entre dois neurnios.

UECEVEST 291

Apostilas UECEVEST mod3.indb 291 06/02/2011 10:04:15


B I O LO GIA 1

b) Possuem receptores moleculares especficos na membrana b) Apresentar clulas fusiformes, sem estriaes, com ncleo
ps-sinptica, onde se ligam os mediadores qumicos. nico e central.
c) Correspondem a locais onde h continuidade do citoplasma de c) Possuir discos intercalares que aumentam a coeso.
um neurnio com o citoplasma de outro. d) Possuir placa motora e clulas dotadas de vrios ncleos
d) Possuem mediadores qumicos denominados perifricos.
neurotransmissores, que ficam armazenadas em vesculas. e) Apresentar ou no estriaes quando visto ao microscpio
e) Podem diferir quanto ao tipo de neurotransmissor presente. ptico.

07. As informaes abaixo resumem observaes relacionadas 11. (UERJ) A fora de contrao da fibra muscular estriada
com as fontes de energia para a contrao muscular: definida pela tenso desenvolvida pelo filamentos de miosina e
I. Um msculo que no possua ATP e fosfocreatina no se actina do sarcmero e sofre influncia do grau de superposio
contrai. desses filamentos.
II. Um msculo que no possua fosfocreatina e recebe ATP,
continuar a se contrair at que todo o ATP seja gasto.
III. Um msculo que no possua ATP, mas recebe fosfocreatina,
no se contrai.
IV. Um msculo sem ATP, mas que recebe fosfocreatina e
ADP, forma ATP e continuar a se contrair enquanto tiver
fosfocreatina e ADP.
Desses dados, conclui-se que:
a) ADP e fosfocreatina so indispensveis para a contrao
muscular;
b) ADP e ATP tm o mesmo papel na contrao muscular; (GUYTON, A. C. & HALL, J. E. Tratado de Fisiologia Mdica Rio de Janeiro:
c) A fosfocreatina inibe a ao do ATP; Guanabara Koogan, 1997.)
d) Fosfocreatina e ATP somam seus efeitos, determinando a
contrao muscular; De acordo com o grfico, podemos dizer que a molcula de mio-
e) o ATP indispensvel para a contrao muscular. sina apresenta uma interao mais eficiente com a actina entre os
seguintes segmentos:
08. (MACKENZIE-SP - modificada) a) O e A d) C e D
b) A e B e) O e D
c) B e C

12. (UFV-MG) Nos organismos pluricelulares, as clulas orga-


nizam-se em tecidos classificados como: epiteliais, conjuntivos,
musculares e nervosos. Considerando-se estes tecidos, assinale a
alternativa INCORRETA:
a) As fibras colgenas so constituintes da substncia
intercelular da maioria dos tecidos conjuntivos.
b) O tecido muscular liso formado por clulas fusiformes,
mononucleadas e de contrao rpida e voluntria.
c) O tecido nervoso constitudo por dois tipos celulares
principais: os neurnios e as clulas de neurglia.
d) Os tecidos epiteliais so caracterizados pela ausncia, ou
quase ausncia, de substncias intercelulares.
e) Os tecidos adiposo, sseo e sanguneo, embora apresentem
A respeito da sinapse representada acima, correto afirmar que: funes e morfologias bem distintas, tm a mesma origem
a) a liberao das substncias presentes nas vesculas determina embrionria.
a passagem de impulso de um neurnio para outro.
b) as substncias presentes nas vesculas so produzidas 13. (UFV) Preocupados com a boa forma fsica, os frequentado-
exclusivamente nas clulas desse sistema. res de uma academia de ginstica discutiam sobre alguns aspectos
c) possvel haver contato fsico entre os neurnios. da musculatura corporal. Nessa discusso, as seguintes afirmati-
d) s est presente no sistema nervoso central. vas foram feitas:
e) o impulso nervoso passa da membrana ps-sinptica para a I. O tecido muscular estriado esqueltico constitui a maior
pr-sinptica. parte da musculatura do corpo humano.
II. O tecido muscular liso responsvel direto pelo
09. (Fazu-MG) A regio de encontro entre dois neurnios e entre desenvolvimento dos glteos e coxas.
neurnios e rgos, onde ocorre a transmisso qumica de impul- III. O tecido muscular estriado cardaco, por ser de contrao
sos eltricos, denominada: involuntria, no se altera com o uso de esterides
a) sinapse. d) neurglia. anabolizantes.
b) desmossomos. d) bainha de mielina. Analisando as afirmativas, pode-se afirmar que:
c) axnio. a) apenas II e III esto corretas.
b) apenas I est correta.
10. (FAFEOD) incorreto afirmar que um msculo de contra-
c) apenas II est correta.
o involuntria poder:
d) I, II e III esto corretas.
a) Localizar-se na parede do intestino delgado, no tero ou no
e) apenas I e II esto corretas.
corao.

292 UECEVEST

Apostilas UECEVEST mod3.indb 292 06/02/2011 10:04:16


BI O LO G I A 1

14. (UECE) Com relao s clulas nervosas, correto afirmar:


a) Nos vertebrados, alm dos neurnios, o sistema nervoso
constitudo por clulas gliais, cuja funo dar sustentao
aos neurnios.
b) Os dendritos so prolongamentos dos neurnios, cuja
funo transmitir para outras clulas os impulsos nervosos
produzidos pelo corpo celular.
c) Os axnios so genericamente chamados de fibras celulares,
cuja funo conectar os corpos celulares.
d) O impulso nervoso, ou sinapse nervosa, transmitido de um
neurnio para outro com o auxlio dos mediadores qumicos.

15. (UECE) Sobre a morfologia do neurnio, foram feitas as se-


guintes afirmaes:
I. O corpo celular contm o ncleo com o nuclolo.
II. Os dendritos so prolongamentos citoplasmticos do corpo Digesto na boca
celular e podem apresentar grande nmero de ramificaes. Por meio da mastigao, os dentes reduzem o alimento a peque-
III. O axnio uma nica expanso citoplasmtica do corpo nos pedaos, processo chamado de digesto mecnica, misturando-
celular, ramificada em sua poro final. o saliva. A saliva (secretada pelas glndulas salivares) contm a
IV. A bainha de mielina envolve o axnio e os dendritos. amilase salivar ou a ptialina, enzima que digere o amido (quebra-o
em molculas de maltose). O pH da boca em torno de 7, ideal
So corretas as afirmaes: para a ao da ptialina. A lngua responsvel pela insalivao e
a) I, II, III e IV. pela deglutio que empurra o bolo alimentar para o esofago. Um
b) I, II e III, somente. pouco antes desse momento, h um mecanismo de reflexo que fe-
c) I, II e IV, somente. cha a laringe, para que no passe alimento para as vias respiratrias.
d) II, III e IV, somente. O esfago um tubo musculoso que liga a faringe ao estmago.
Est localizado entre os pulmes e atrs do corao.
16. (UNIRIO) Sabemos que a fibra nervosa formada pelo ax-
nio e dobras envoltrias de diferentes clulas no SNC e no SNP, Digesto no estmago
que so, respectivamente: O estmago, localizado no lado esquerdo superior do abdome,
a) oligodendrcitos e astrcitos fibrosos. abaixo das costelas, serve de reservatrio alimentar. Nele, o bolo
b) oligodendrcitos e Clulas de Schwann. alimentar misturado com a secreo estomacal (suco gstrico: ci-
c) astrcitos protoplasmticos e micrglia. do clordrico, pepsina e renina). Na regio de comunicao entre
d) astrcitos protoplasmticos e astrcitos fibrosos. o esfago e o estmago, existe um anel muscular denominado de
e) Clulas de Schwann e micrglia. crdia. Anel muscular como esse (esfncter), tambm est presente
na sada do estmago. A pepsina faz a digesto das protenas com a
quebra da ligao peptdica; a renina tem como funo coagular as
protenas do leite, fazendo com que este permanea mais tempo no
G A B A R I T O estmago, favorecendo uma digesto mais completa. Ela mais co-
01. 02. 03. 04. 05. 06. c mumente encontrada em estmagos de crianas e recm-nascidos.
07. e 08. a 09. a 10. d 11. c 12. b O pH do estmago em torno de 2. O bolo alimentar transfor-
ma-se em uma massa acidificada e semi-lquida, denominada de
13. b 14. a 15. b 16. b quimo.O esfncter muscular de comunicao entre o estmago e o
intestino delgado chamado de piloro.

Digesto no intestino delgado


O intestino delgado um tubo de pouco mais de 6m de com-
SISTEmA DIGESTRIO primento e pode ser dividido em trs regies: duodeno (25cm),
jejuno (5m) e leo (1,5m). O intestino delgado apresenta vilosida-
des e as clulas do mesmo apresenta as chamadas microvilosidades.
Introduo
A digesto do quimo se d predominantemente no duodeno e no
A digesto o processo pelo qual grandes molculas org-
jejuno, onde lanado o suco entrico (soluo rica em enzimas di-
nicas adquiridas atravs da alimentao so quebradas (pelas
gestivas: enteroquinase (tripsinognio tripsina e peptidases). No
enzimas) em molculas menores, mais simples e mais solveis,
duodeno, atua tambm o suco pancretico (tripsina, quimiotripsi-
de tal forma que possam ser absorvidas pelas clulas do orga-
na, lpase pancretica, amilases, ribonucleases, desoxiribonucleases,
nismo. Os vertebrados apresentam sistema digestrio comple-
etc.). No suco pancretico encontramos tambm o bicarbonato de
to e a digesto completamente extracelular. O tubo digestivo
sdio; ele tem como funo elevar o pH do quimo para ficar em
humano apresenta as seguintes regies: boca, faringe, esfago,
torno de 8. A bile, secreo produzida pelo fgado, tambm age no
estmago, intestino delgado, intestino grosso e nus. A parede
duodeno e tem como funo a emulsificao de lipdios, transfor-
do tubo digestivo possui a mesma estrutura bsica da boca ao
mado grandes molculas de gordura em gotculas microscpicas,
nus (mucosa, submucosa, muscular e adventcia). O Peristal-
facilitando a ao da lpase pancretica. A absoluta maioria dos nu-
tismo a contrao da camada muscular em ondas rtmicas,
trientes absorvida pela mucosa do intestino delgado.
responsvel pela movimentao do alimento no trato digestivo.
A digesto humana envolve processos mecnicos e qumicos. A Intestino grosso
mastigao, deglutio e os movimentos peristlticos so exem- Mede cerca de 0,5m e dividi-se em trs partes: ceco, clon e
plos de processos mecnicos; a ao das enzimas um exemplo reto. Na extremidade fechada do ceco encontramos o apndice
de processo qumico. cecal. O clon tem a forma de uma letra U invertida e dividi-

UECEVEST 293

Apostilas UECEVEST mod3.indb 293 06/02/2011 10:04:17


B I O LO GIA 1

se em quatro regies: clon ascendente, transverso, descendente e Corao


sigmide. O reto termina no nus. No intestino grosso, parte da
gua e dos sais minerais da massa de resduos absorvida. Na re-
gio final do clon, a massa de resduos ou massa fecal se solidifica,
transformando-se nas fezes (fibras de celulose e outros componen-
tes no digeridos). A defecao ocorre devido distenso provoca-
da pela presena das fezes estimulando as terminaes nervosas do
reto e fazendo com que o esfncter interno anal relaxe-se involun-
tariamente. Entretanto, a defecao s ir ocorrer quando houver o
relaxamento do esfncter anal mais externo; esse, voluntariamente.
Os processos digestivos so controlados pelo sistema nervoso
autnomo e por hormnios.

Esquema da dupla circulao nos mamferos

Apresenta quatro cavidades internas: cmaras cardacas. As


Sistema circulatrio duas cmaras superiores so os trios (trio direito AD e trio
esquerdo AE); as duas inferiores, os ventrculos (ventrculo direi-
Componentes do sistema circulatrio to VD e ventrculo esquerdo VE). O AD comunica-se com o
Sangue: fluido formado por clulas; VD por meio da vlvula tricspide. O AE comunica-se com o VE
O sangue veneso pobre em oxignio e rico em gs carbnico. por intermdio da bicspide ou mitral. A funo dessas vlvulas
O sangue arterial rico em oxignio e pobre em gs carbnico. garantir a circulao do sangue no corao em um nico sentido,
Vasos Sanguneos: tubos onde o sangue circula; sempre dos trios para os ventrculos. A contrao de uma cmara
Corao: rgo musculoso cuja contrao impulsiona o san- cardaca denominada sstole e seu relaxamento, distole.
gue, fazendo-o circular no interior dos vasos sanguneos.
Ciclo cardaco
O incio do ciclo marcado pela sstole dos trios, que bom-
beiam sangue para o interior dos ventrculos (o sangue chega
aos trios direito e esquerdo por meio das veias cavas e das veias
pulmonares, respectivamente). Os ventrculos direito e esquerdo
entram em sstole bombeando o sangue, respectivamente, para
as artrias pulmonar e aorta. Durante a sstole ventricular a tri-
cspide e a mitral se fecham, evitando o retorno de sangue para
os trios. A frequncia de batimentos cardacos controlada pelo
ndulo sinoatrial ou marcapasso. O ndulo atrioventricular atua
como distribuidor do sinal gerado pelo marcapasso. A presso ar-
terial a presso exercida pelo sangue contra a parede das artrias.
A presso sistlica da ordem de 120 mmHg e a diastlica fica
Funes do sistema circulatrio: em torno de 80 mmHg.
Transporte de nutrientes;
Transporte de O2 e CO2; Circulao pulmonar e sistmica
Transporte de hormnios; O sangue passa duas vezes pelo corao. Impulsionado pelo
Transporte de clulas e de anticorpos do sistema imunolgico; ventrculo direito, o sangue vai para os pulmes, de onde retorna
Remoo de excrees. novamente ao corao (circulao dupla). Circulao Pulmo-
nar (ou pequena circulao): o sangue chega ao trio direito do
Conceitos corao por duas grandes veias, a veia cava superior e a veia cava
Artrias: vasos que levam sangue do corao para os rgos e inferior. Do trio direito o sangue passa para o ventrculo direito,
tecidos do sangue. Elas apresentam parede relativamente es- que o bombeia para a artria pulmonar, que leva o sangue para
pessa, constituda por trs camadas de tecido, denominadas os pulmes. Nos pulmes o sangue adquire oxignio e libera gs
tnicas. As artrias que partem do corao ramificam-se pro- carbnico. Circulao Sistmica (ou grande circulao): dos
gressivamente em artrias menores, atingindo todas as partes pulmes o sangue rico em O2 retorna ao corao pelas veias pul-
do corpo, sendo chamados de arterolas. monares e penetra no trio esquerdo. Do trio esquerdo o sangue
Veias: vasos que levam sangue dos rgos e tecidos de volta ao passa para o ventrculo esquerdo que o bombeia para a artria
corao. Suas paredes tambm so constitudas por trs tni- aorta, levando o sangue oxigenado a todos os sistemas do corpo.
cas. As veias de maior dimetro apresentam vlvulas, que ser-
vem para impedir o refluxo do sangue, garantindo a circulao Sangue arterial
sangunea de carter unidirecional. A maioria das artrias transporta sangue rico em O2; as ex-
Capilares Sanguneos: Vasos muito finos que ligam as arter- cees so as artrias pulmonares, que levam sangue pobre em
olas s vnulas. O2. Sangue Venoso: A maioria das veias transporta sangue rico
O homem tem sistema circulatrio fechado (sangue nunca em CO2; as excees so as veias pulmonares, que trazem sangue
abandona o interior dos vasos sanguneos). Impulsionado pelo oxigenado dos pulmes ao corao.
corao, o sangue caminha pelo interior dos vasos (artrias)
que se ramificam em artrias menores (arterolas) e se ligam Sistema linftico
aos capilares. Os capilares se conectam (no lado oposto) com constitudo por uma ampla rede de vasos linfticos que se
as vnulas que se unem e formam as veias, levando o sangue de distribuem por todo corpo. Os capilares linfticos esto mergu-
volta ao corao. lhados entre as clulas dos tecidos e tm por funo a drenagem

294 UECEVEST

Apostilas UECEVEST mod3.indb 294 06/02/2011 10:04:18


BI O LO G I A 1

do excesso de lquido tissular (extravasado dos capilares sangu- 06. (UNIFESP) No tubo 1 existe uma soluo contendo clulas
neos) e reconduzi-lo circulao. Caso essa funo, por algum de fgado de boi. Em 2, h uma soluo de clulas extradas de
motivo, cessar e o lquido tissular se acumular, observaremos um folhas de bananeira.
edema linftico. Gnglios Linfticos: so rgos de defesa do or-
ganismo humano. Os gnglios filtram a linfa e a deixam livres de
vrus, bactrias etc. As nguas so os gnglios linfticos inchados,
por inflamao.
Voc deseja eliminar completamente todos os constituintes dos
Doenas cardiovasculares envoltrios celulares presentes em ambos os tubos.
Arteriosclerose; Para isso, dispe de trs enzimas digestivas diferentes:
Angina do peito (dores no peito ao menor esforo cardaco); C: digere carboidratos em geral.
Infarto do miocrdio (isquemia brusca no msculo cardaco); L: digere lipdios.
Isquemia Cerebral; P: digere protenas.
Hipertenso. Para atingir seu objetivo gastando o menor nmero possvel de
enzimas, voc deve adicionar a 1 e 2, respectivamente:
Preveno de doenas cardiovasculares a) 1 = C; 2 = P.
Evitar o tabagismo, alimentao saudvel, prtica de exerc- b) 1 = L; 2 = C.
cios fsicos etc. A constituio gentica predispe certas pessoas c) 1 = C e P; 2 = C e L.
a desenvolverem doenas cardiovasculares. Os genes, entretanto, d) 1 = C e P; 2 = C, L e P.
atuam junto com fatores ambientais que, comprovadamente, de- e) 1 = L e P; 2 = C, L e P.
sencadeiam doenas.
07. (FATEC) A figura a seguir esquematiza o corao de um ma-
mfero, com suas cmaras (representadas por letras), veias e art-
E X E R C C I O rias (representadas por algarismos).
01. Durante o processo de digesto de alimentos pelo homem,
observa-se uma variao do pH ao longo do aparelho digestivo.
Considerando essa variao, podemos dizer que o pH:
a) na boca cido, no estmago alcalino e neutro no
intestino.
b) na boca e no estmago cido, tornando-se prximo ao
neutro no intestino.
c) na boca alcalino, no estmago neutro e no intestino
cido.
d) na boca prximo ao neutro, no estmago torna-se cido e
no intestino volta a ser alcalino.
e) tende a apresentar uma tendncia geral acidificao.
Identifique cada parte do corao e assinale a alternativa que
02. (Fazu MG) O alimento movido ao longo do trato gas- apresenta a correspondncia correta.
trointestinal por um processo proveniente da contrao da cama- a) O sangue rico em O chega nas cmaras A e B.
da muscular circular; a onda progride e espreme o alimento para b) O sangue rico em CO chega na cmara B.
baixo e/ou para frente de maneira semelhante a sada de creme c) Os vasos identificados por I, II e III so, respectivamente,
dental de um tubo. Tal processo de motilidade denomina-se: veia cava superior, artria pulmonar e artria aorta.
a) Peristaltismo d) Homeostase d) O vaso indicado por IV traz sangue arterial dos pulmes ao
b) Digesto e) Secreo corao.
c) Absoro e) O vaso indicado por III leva o sangue arterial do corao
para o corpo.
03. (FGV) Chamamos de artrias todo vaso sanguneo que:
a) chega ao corao. d) chega s aurculas. 08. Um indivduo fez uma refeio da qual constavam, basica-
b) conduz sangue venoso. e) conduz sangue arterial. mente, trs substncias: A, B e C. Durante a digesto ocorreram
c) parte do corao. os seguintes fatos: na cavidade bucal, iniciou-se a digesto de B;
ao chegar ao estmago iniciou-se a digesto de C, e a digesto
04. (UFRN) Qual dos seguintes produtos de secreo no tem de B foi interrompida; no duodeno, ocorreu digesto de A, B
ao enzimtica: e C. A partir desses dados correto afirmar que A, B e C so,
a) suco gstrico. d) saliva. respectivamente:
b) suco entrico. e) bile. a) lipdeos, protena, carboidrato.
c) suco pancretico. b) protena, carboidrato, lipdeos.
c) lipdeos, carboidrato, protena.
05. (UFSE) Tripsina, pepsina e ptialina so enzimas digestivas d) carboidrato, lipdeos, protena.
produzidas, respectivamente, no: e) protena, lipdeos, carboidrato.
a) fgado, pncreas e estmago.
b) pncreas, estmago e glndulas salivares. 09. O esquema representa um corte longitudinal do corao de
c) pncreas, glndulas salivares e estmago. um mamfero. O sangue que deixa o ventrculo direito (VD) e o
d) estmago, glndulas salivares e fgado. que deixa o ventrculo esquerdo (VE) seguiro, respectivamente,
e) fgado, estmago e pncreas. para:

UECEVEST 295

Apostilas UECEVEST mod3.indb 295 06/02/2011 10:04:19


B I O LO GIA 1

SISTEmA RESPIRATRIO

a) aurcula direita e aurcula esquerda.


b) veia cava pulmonar.
c) ventrculo esquerdo e pulmes.
d) pulmes e artria aorta.
e) pulmes e ventrculo direito

10. Nos mamferos, pode-se encontrar sangue venoso:


a) na aurcula direita, na artria pulmonar e na veia cava.
b) no ventrculo direito, na veia pulmonar e na veia cava.
c) na aurcula direita, na veia pulmonar e artria aorta.
d) na aurcula esquerda, na artria pulmonar e na veia cava. A respirao um processo bioqumico intracelular durante
e) no ventrculo esquerdo, na veia pulmonar e na artria aorta. o qual ocorre liberao de energia contida em substncias orgni-
cas. H trocas gasosas com o meio externo atravs de superfcies
11. A funo das vlvulas existentes nas veias : respiratrias do corpo dos animais. Os gases importantes para a
a) retardar o fluxo sanguneo. respirao so: oxignio (O2) e gs carbnico (CO2). Suas trocas
b) impedir o refluxo do sangue. gasosas ocorrem por meio de difuso. importante ressaltar a
c) acelerar os batimentos cardacos. necessidade do meio aquoso. Sem uma superfcie umedecida, as
d) retardar as pulsaes. trocas gasosas tornam-se ineficientes. Logo abaixo, temos alguns
e) reforar as paredes dos vasos. conceitos importantes:
Pigmentos Respiratrios: so substncias capazes de se com-
12. Relacione as colunas: binar com o O2 aumentando a capacidade de transporte desse
1. irrigao do miocrdio ( ) veia pulmonar gs pelo corpo. Nos humanos esse pigmento a hemoglobina.
2. conduz sangue arterial ( ) pequena circulao Hemoglobina (Hb): protena, localizada na hemcia, constitu-
3. leva O2 para os tecidos ( ) artria pulmonar da por 4 cadeias polipeptdicas associadas a um grupamento
4. Conduz sangue venoso ( ) grande circulao qumico chamado grupo heme, que contm ferro.
5. retira CO2 de circulao ( ) coronria 1Hb + 4O2 Hb (O2)4
A correta sequncia numrica da segunda coluna, de cima para bai-
xo, de conformidade com a primeira : Transporte de CO2
a) 2-5-4-3-1 c) 5-3-2-4-1 5 - 7%: dissolve-se no plasma sanguneo.
b) 1-3-4-5-2 d) 4-2-3-1-5 23%: associao a grupos amina da hemoglobina e outras pro-
tenas do sangue.
13. Quando o corao bombeia sangue com toda a fora: 70%: penetra nas hemcias e transformado em cido carbnico.
a) as artrias se distendem em virtude da presso
b) as veias se contraem. Componentes do Sistema Respiratrio
c) a presso arterial mnima. Um par de pulmes, fossas nasais, boca, faringe, laringe, tra-
d) ele encontra-se em distole. queia, brnquios, bronquolos e alvolos pulmonares.
e) o fluxo sanguneo diminui. Fossas Nasais: so duas cavidades paralelas que comeam nas
narinas e terminam na faringe. Elas so separadas pelo septo
nasal. Tm funes de umedecer e aquecer o ar que penetrar
nas vias respiratrias. Esto presentes nas fossas nasais clulas
G A B A R I T O sensoriais olfativas.
Faringe: Canal comum do sistema digestrio e respiratrio; se
01. 02. 03. 04. 05. comunica com a boca e com as fossas nasais. A faringe conduz
06. e 07. d 08. c 09. d 10. a o ar at a laringe dividida em Naso Faringe e Faringe.
11. b 12. a 13. a Laringe: um tubo que continua a faringe, por volta do osso
de hiide. Na laringe encontramos a glote (entrada da laringe)
e a epiglote (vlvula que impede que os alimentos penetrem as
vias respiratrias). O revestimento interno da laringe contm
pregas chamadas de cordas vocais.
Traqueia, Brnquios e Bronquolos: A traqueia um tubo de
10 cm de comprimento (continuao da laringe) que se bifurca
dando origem aos brnquios, que conduzem ar aos pulmes.
Tanto a traqueia como os brnquios possuem clulas formadoras
de muco. Nos pulmes, os brnquios se ramificam em bronqu-
olos. Cada bronquolo termina nos alvolos pulmonares. Nestes,
ocorre as trocas gasosas entre o sangue e o ar (hematose).

296 UECEVEST

Apostilas UECEVEST mod3.indb 296 06/02/2011 10:04:20


BI O LO G I A 1

Alvolos Pulmonares: pequenas bolsas de paredes finas reco- SISTEmA EXCRETOR


berto por capilares sanguneos. Essa proximidade permite que
ocorra difuso de gases entre o sangue e o ar: ao chegar aos
capilares alveolares, o sangue rico em CO2 e pobre em O2;
depois de passar pelos capilares dos alvolos, o sangue torna-se
rico em O2 e pobre em CO2 sendo este processo chamado de
Hematose. A hematose realizada devido s diferenas de pres-
ses parciais dos diversos gases entre o ar alveolar e o sangue. O
ar alveolar possui uma composio um pouco diferente do ar
atmosfrico, o que altera as presses parciais dos gases em ter-
mos de ar alveolar quando comparados com o ar atmosfrico.
Pulmo: envolvido por duas membranas (pleura); um r-
go esponjoso, com aproximadamente 25 cm de comprimento
e 700g de peso. O pulmo direito ligeiramente maior que o
esquerdo e est dividido em trs lbulos; o esquerdo est divi-
dido em dois lbulos.
Diafragma: A base de cada pulmo se apia no diafragma,
uma membrana espessa formada por camadas musculares; o
diafragma separa a cavidade torcica da abdominal.

A excreo um processo pelo qual os animais eliminam


substncias nitrogenadas txicas produzidas durante o metabolis-
mo celular. As substncias nitrogenadas excretadas pelos animais
provm, principalmente, da degradao de aminocidos ingeri-
dos no alimento. Os principais tipos de excretas so: amnia (in-
vertebrados e peixes de gua doce); ureia e cido rico. O sistema
excretor humano constitudo por um par de rins, um par de
ureteres, pela bexiga urinria e pela uretra.
Rins: Localizam-se na parte posterior do abdome, logo abai-
Ventilao pulmonar xo do diafragma, um de cada lado da coluna vertebral. O rim
Inspirao: Contrao do diafragma e msculos intercostais; possui uma cpsula fibrosa que protege o crtex (poro mais
Aumento de volume da caixa torcica e diminuio interna. externa do rim) e a medula (poro mais interna do rim). A
Com isso, h sada de ar dos pulmes. principal funo dos rins a de filtrar o sangue (remoo dos
resduos nitrogenados). Alm da funo excretora, os rins so
Expirao: Relaxamento do diafragma e msculos intercostais. responsveis tambm pela osmorregulao (manuteno da
Diminuio do volume da caixa torxica e aumento da pres- tonicidade do sangue). O sangue chega ao rim atravs da ar-
so interna. Com isso, h sada de ar dos pulmes. tria renal, que se ramifica originando um grande nmero de
arterolas. Essas arterolas ramificam-se no interior da cpsula
do glomrulo renal, formando um enovelado de capilares cha-
mados de glomrulo renal.
Nfrons: Na regio do crtex renal, esto os nfrons, que so
estruturas microscpicas responsveis pela filtrao de sangue
e pela remoo das excrees. O nfron uma longa estrutura
tubular que possui, em uma das extremidades, uma expanso
em forma de taa chamada cpsula do glomrulo renal. Esta
cpsula se conecta com o tbulo do proximal, continua pela
ala rfrica e pelo tbulo distal, desembocando em um ducto
coletor (tubo coletor de urina).

Filtrao do Sangue.
Os capilares do glomrulo deixam extravasar diversas subs-
tncias presentes no sangue: gua, sais minerais, ureia, glicose,
aminocidos etc. Essas substncias passam entre as clulas da pa-
rede da cpsula do glomrulo renal e constituem o filtrado glo-
merular (urina inicial). A urina inicial caminha pelo tbulo con-
tornado proximal, ala rfrica e pelo cartorcido distal, de onde
lanado em um ducto coletor. Durante esse percurso a parede
dos tbulos renais reabsorve a maior parte da gua e substncias

UECEVEST 297

Apostilas UECEVEST mod3.indb 297 06/02/2011 10:04:23


B I O LO GIA 1

presentes na urina inicial, que passam para o sangue dos capilares 08. (UFPI) H uma desordem hormonal chamada diabetes in-
do nfron. Esses capilares formam a arterola eferente, pela qual o sipidus, que causada por uma falha na produo do hormnio
sangue deixa a cpsula do glomrulo renal. anti-diurtico (ADH). Assinale a alternativa que descreve corre-
Ureteres: Os ductos coletores de urina se unem para formar tamente as consequncias dessa desordem.
canais cada vez mais grossos. A fuso desses ductos forma um a) Aumento na produo de urina e diminuio do volume dos
canal nico chamado de ureter, que sai do rim em direo fluidos corporais.
bexiga urinria. b) Falta de sede e diminuio do volume dos fluidos corporais.
Bexiga: Acumula a urina produzida nos rins. dotada de mus- c) Aumento na concentrao de glicose sangunea e perda de peso.
culatura lisa e elstica. d) Diminuio da concentrao de insulina sangunea e
Uretra: um tubo que parte da bexiga e termina, na mulher, aumento da glicose sangunea.
na regio vulvar; e no homem, na extremidade do pnis. e) Aumento da concentrao de insulina sangunea e
diminuio da glicose sangunea.

E X E R C C I O 09. Um homem com um clculo renal localizado em um de seus


rins, passado algum tempo, consegue eliminar o clculo. At ser
01. (UnB-DF) Assinale a alternativa que apresenta uma estrutura eliminado, o clculo passou sucessivamente pelo:
comum aos sistemas respiratrio e digestivo. a) rim, ureter, bexiga e uretra.
a) Brnquios. d) Esfago. b) rim, bexiga, ureter e uretra.
b) Faringe. e) Laringe. c) rim, uretra, bexiga e ureter.
c) Pulmo. d) rim, bexiga, uretra e ureter.
e) rim, uretra, ureter e bexiga.
02. (Fuvest-SP) A obstruo dos bronquolos impede que o oxi-
gnio atinja: 10. Em decorrncia de baixa ingesto de gua pelo organismo,
a) a faringe. d) a traqueia. pode-se prever que:
b) o esfago. e) os alvolos. a) diminua a presso osmtica do sangue.
c) a laringe. b) os tbulos renais fiquem mais permeveis gua.
c) diminua a taxa de hormnio antidiurtico liberado na
03. (Fuvest-SP) Uma pessoa excreta mais ureia quando come mais: circulao.
a) amido. d) gordura. d) aumente a secreo de aldosterona e diminua a de hormnio
b) protena. e) sacarose. antidiurtico.
c) glicose. e) a urina se torne muito diluda.
04. (UFRN) Durante a respirao, quando o diafragma se con- 11. Brnquias e pulmes so rgos cuja estrutura reflete a fun-
trai e desce, o volume da caixa torcica aumenta, por conseguinte o que exercem. O contedo dessa afirmao baseia-se, princi-
a presso intrapulmonar: palmente, no fato de ambos apresentarem:
a) diminui e facilita a entrada de ar. a) estrutura ramificada, que possibilita grande superfcie de
b) aumenta e facilita a entrada de ar. contato com a gua ou com o ar atmosfrico.
c) diminui e dificulta a entrada de ar. b) estrutura compacta, que acarreta grande proteo das dobras
d) aumenta e dificulta a entrada de ar. por onde os gases se difundem.
e) aumenta e expulsa ar dos pulmes. c) grande nmero de canais, o que faz com que o gs oxignio
v diretamente para as clulas de todo o corpo.
05. Os rins, alm da importante funo excretora, contribuem de d) rica vascularizao, que permite ao organismo a eliminao
maneira eficiente na (o): rpida do gs oxignio.
a) manuteno do equilbrio endcrino. e) extensa rede de leuccitos, que estimula a maior captao de
b) metabolismo dos acares. gases da gua ou do ar atmosfrico.
c) manuteno da composio sangunea.
d) manuteno da temperatura. 12.
e) metabolismo dos lipdios.

06. (UFPR) O transporte de oxignio no organismo humano se faz:


a) atravs dos leuccitos.
b) atravs do plasma sanguneo.
c) tanto pela hemoglobina plasmtica como pela existente
no interior das hemcias quando a taxa de hemoglobina
normal.
d) atravs da hemoglobina existente nas hemcias.
e) na dependncia de boa funo plaquetria.
Sobre este desenho, so feitas trs afirmativas:
07. (UFRS) O deslocamento de um atleta para locais de grande al-
I. Em 4, o ar passa em direo aos pulmes, aps ter sido
titude pode acarretar-lhe algumas alteraes no organismo. A curto
aquecido em 1.
prazo, provoca modificaes da atividade respiratria e, a longo pra-
II. Em 6, o oxignio do ar penetra nos vasos sanguneos, sendo
zo, produz alteraes sanguneas. Assim, este indivduo apresenta
o fenmeno conhecido como hematose.
a) hiperventilao e diminuio do nmero de hemcias.
III. Em 8, o gs carbnico proveniente do sangue passa para o ar.
b) hiperventilao e aumento do nmero de hemcias.
c) hipoventilao e diminuio do nmero de hemcias. Indique:
d) hipoventilao e aumento do nmero de hemcias. a) se somente I for correta.
e) isoventilao e manuteno do nmero de hemcias. b) se somente II for correta.

298 UECEVEST

Apostilas UECEVEST mod3.indb 298 06/02/2011 10:04:24


BI O LO G I A 1

c) se somente I e II forem corretas. Na sinapse, regio de contato entre dois neurnios, h uma
d) se somente I e III forem corretas. pequena distncia entre as duas clulas envolvidas, isto , no
e) se I, II e III forem corretas. h continuidade entre as membranas celulares. A passagem do
impulso nervoso nessa regio feita por substncias denominadas
13. A ingesto de bebidas alcolicas inibe a liberao do hormnio neuro-hormnios ou mediadores qumicos como a adrenalina e
responsvel pelo aumento da permeabilidade das membranas das a acetilcolina.
clulas dos tbulos renais. Com isso, diminuda a reabsoro:
a) passiva de gua, o que diminui a concentrao sangunea e
concentra a urina.
b) passiva de gua, o que aumenta a concentrao sangunea e
dilui a urina.
c) passiva de gua, o que diminui a concentrao sangunea e
dilui a urina.
d) ativa de gua, o que aumenta a concentrao sangunea e
dilui a urina.
e) ativa de gua, o que diminui a concentrao sangunea e
concentra a urina.

14. (PUC-SP) Considere as seguintes etapas do processo respira-


trio no homem: A passagem do impulso nervoso entre dois neurnios
1- Produo de ATP nas mitocndrias
2- Ocorrncia de hematoses ao nvel dos alvolos. Nos vertebrados, o sistema nervoso ocupa posio dorsal e
3- Transporte de oxignio aos tecidos pelas hemcias. est protegido pela caixa craniana e pela coluna vertebral. Com-
A ordem em que essas etapas se realizam, a partir do momento preende o sistema nervoso central (SNC), o sistema nervoso peri-
em que um indivduo inspira ar do ambiente : frico (SNP) e o sistema nervoso autnomo (SNA).
a) 1-2-3. d) 3-1-2.
b) 2-1-3. e) 3-2-1. S. n. Central
c) 2-3-1. o sistema nervoso central formado pelo encfalo e pela me-
dula espinhal. Ambos so protegidos por estruturas sseas, o en-
cfalo pela caixa craniana e a medula pelas vrtebras.
Alm disso, tanto o encfalo quanto a medula so envolvidos
G A B A R I T O por membranas, as meninges, denominadas dura-mter, aracni-
de e pia-mter. Entre a aracnide e a pia-mter circula o liquido
01. 02. 03. 04. 05.
cefalorraquidiano, que protege e nutre o sistema nervoso.
06. d 07. b 08. a 09. a 10. b
11. a 12. d 13. b 14. c Encfalo
O encfalo se divide em crebro, cerebelo, ponte e bulbo ra-
quidiano.
Crebro: a parte mais volumosa do encfalo. Divide-se em
duas metades denominadas hemisfrios cerebrais. Sua superf-
SISTEmA nERVOSO cie externa chamada de crtex cerebral. O crebro o centro
da motricidade voluntria, inteligncia, vontade, memria,
O sistema nervoso juntamente com o sistema endcrino atua imaginao, conscincia, criatividade e das sensibilidades ol-
na coordenao e na integrao das funes das clulas, dos teci- fativas, tteis, auditivas, visuais e gustativas. Nos vertebrados
dos, dos rgos e aparelhos, de modo que trabalhem harmonio- inferiores, de peixes at aves, os hemisfrios cerebrais tm su-
samente como uma unidade. O sistema nervoso formado pelo perfcie lisa. Por isso esses animais so chamados de lisencfa-
tecido nervoso sendo que suas principais clulas so os neur- los. Nos mamferos, principalmente os primatas. a superfcie
nios, especializados em receber e transmitir impulsos nervosos. cerebral dotada de uma srie de circunvolues, que aumen-
O impulso nervoso de natureza eltrica e resulta de alteraes tam consideravelmente essa superfcie e comportam um maior
nas cargas eltricas das superfcies interna e externa da membrana nmero de neurnios. Por essa razo, os mamferos so deno-
plasmtica da clula nervosa. Essas alteraes, que constituem o minados girencfalos.
impulso nervoso, sempre comeam nos dendritos, e caminham
do corpo celular para o axnio.

Esquema do crebro humano, mostrando suas


circunvolues.

Cerebelo: Situa-se logo abaixo do crebro. bem desenvolvi-


do em peixes bons nadadores, nas aves e nos mamferos. o
controlador da tonicidade, vigor muscular e equilbrio corporal.

UECEVEST 299

Apostilas UECEVEST mod3.indb 299 06/02/2011 10:04:26


B I O LO GIA 1

Ponte: Est localizada abaixo do crebro, diante do cerebelo. neurnios. Quanto posio destas substncias, notamos que
Em todos os vertebrados, com exceo dos mamferos, funcio- no crebro a substncia cinzenta perifrica, enquanto que na
na como centro da viso. Nos mamferos, sua funo de servir medula interna; j a substncia branca interna no crebro e
de passagem aos estmulos nervosos que vo ao crebro. perifrica na medula.
Bulbo raquidiano: Localiza-se acima da medula espinhal e
abaixo da ponte. Como centro nervoso, o bulbo controla o S. N. Perifrico
ritmo cardiorespiratrio e alguns atos reflexos, como os da de- O S. N. Perifrico formado por uma rede de nervos, que
glutio, suco, mastigao, vmito, tosse, secreo lacrimal podem ser cranianos, quando partem do encfalo e raquidianos,
e o piscar. quando partem da medula espinhal. Os peixes e os anfbios apre-
sentam dez pares de nervos cranianos. Os rpteis, as aves e os
mamferos apresentam doze pares de nervos cranianos. Da me-
dula espinhal partem 31 pares de nervos raquidianos. Os nervos
fazem a comunicao dos centros nervosos com rgos sensoriais
(receptores) e com os msculos e glndulas (efetores). De acordo
com a direo da transmisso do impulso nervoso, os nervos se
classificam em:
Nervos Sensitivos: Transmitem ao crebro as impresses co-
lhidas na superfcie do corpo.
Nervos Motores: Conduzem a resposta elaborada pelos cen-
tros nervosos a um rgo efetor, geralmente um msculo.
Nervos Mistos: Levam e trazem estmulos dos centros nervo-
Medula espinhal sos.
a continuao do bulbo; cilndrica, achatada e desce pelo
interior da coluna vertebral. A medula conduz impulsos sensiti- S. N. P. Autnomo
vos para o crebro e traz impulsos motores. Exercem tambm a formado por nervos que funcionam independentemente da
funo de centro nervoso responsvel por muitos atos reflexos, nossa vontade. Por exemplo: o corao, o estmago, o intestino,
principalmente os relacionados com o instinto de conservao e a secreo de algumas glndulas so comandadas pelo S. N. P.
defesa. O caminho do impulso nervoso no ato reflexo denomi- Autnomo sem influncia da nossa vontade. O S. N. P. Autno-
nado arco reflexo. mo divide-se em dois tipos e cada rgo por ele controlado recebe
nervos dos dois sistemas:
Arco reflexo S. N. P. A. Simptico: Funciona como um chicote, ativando
Arco reflexo uma ao nervosa medular inconsciente e mui- o rgo.
to rpida. Como exemplo, temos o reflexo rotuliano ou pate- S. N. P. A. Parassimptico: Funciona como um freio, ini-
lar. Ele acontece quando se aplica um golpe com um martelo bindo a ao do rgo.
de borracha no joelho de uma pessoa, excitando as extremidades
nervosas (dendritos) dos neurnios sensitivos (aferentes). Imedia- Os nervos do simptico originam-se na regio mediana da
tamente, os axnios desses neurnios transmitem a informao medula; os nervos parassimpticos saem do bulbo e da extremi-
adquirida at a medula do S.N. Central., penetrando-lhe pela dade final da medula. O efeito de cada um desses sistemas varia
regio dorsal. Na medula esto os neurnios associativos, que de rgo para rgo. O corao, por exemplo, estimulado pelo
transformam a informao adquirida em ordem de ao. No caso simptico e inibido pelo parassimptico. J com a musculatura
do reflexo rotuliano, os neurnios sensoriais transmitem o impul- do tubo digestivo ocorre o contrrio. O sistema nervoso simpti-
so nervoso diretamente para os neurnios motores, que partem co atua na transformao do glicognio armazenado no fgado e
da regio ventral da medula e vo estimular os msculos da coxa, nos msculos em glicose, liberando-a no sangue.
o que provoca o movimento da perna que estava dobrada e li-
vremente pendente. O impulso nervoso tambm transmitido
pela medula at o crebro, onde ao ser interpretado, confere ao
indivduo a conscincia da pancada.

Sistema Endcrino
Os hormnios so substncias qumicas, produzidas por c-
Existe no S. N. Central dois tipos de substncias formando os lulas isoladas ou por glndulas chamadas endcrinas que, libera-
rgos: cinzenta e branca. A substncia cinzenta formada pela das no sangue, vo agir distncia, estimulando ou inibindo as
concentrao dos corpos celulares dos neurnios, enquanto que funes de certos rgos. O rgo sobre o qual um determinado
a substncia branca formada pela concentrao dos axnios dos hormnio age chamado rgo-alvo.

300 UECEVEST

Apostilas UECEVEST mod3.indb 300 06/02/2011 10:04:30


BI O LO G I A 1

Funes das principais glndulas endcrinas e mistas humanas Os rgos do sistema nervoso que controlaram a primeira e a
Glndula Hormnio Efeitos Principais segunda reao foram, respectivamente:
Desenvolvimento de a) medula e crebro.
b) cerebelo e crtex.
STH (Somatotrfico) cartilagens, ossos e msculos
(crescimento corporal do c) medula e hipotlamo.
indivduo). d) crebro e medula.
TSH (Tireotrfico) Estimulante da tireide. e) crebro e neurnio.
ACTH (Adreno- Estimulante do crtex das
corticotrfico) adrenais 02. (UFC 2008) O momento do vestibular, sem dvida, causa
Pro lactina Estimulante da produo de nos candidatos uma mistura de sensaes como prazer, por estar
Adeno- leite nas glndulas mamrias. prxima a to sonhada aprovao; emoo, por vivenciar uma
-Hipfise
Na mulher: estimula a grande scolha, e medo de cometer um equvoco ao responder as
maturao do folculo ovariano questes. Essas sensaes estimulam o sistema nervoso, ocasio-
e contribui para a regularidade
FSH (Folculo- menstrual. No homem: nando taquicardia e aumento da frequncia respiratria. Assina-
estimulante) le a alternativa que apresenta a glndula que foi estimulada e o
estimula clulas intersticiais
de Leydig a produzirem hormnio produzido como consequncia das sensaes citadas
hormnios masculinos. no texto.
LH (Luteinizante) Na mulher: estimula a a) Supra-renal e adrenalina
formao do corpo lteo. b) Tireide e addrenalina
Hipotlamo Contraes musculares do c) Tireide e caicitonina
(seus tero (ocasio do parto) e d) Hipfise e adrenalina
hormnios so Ocitocina das glndulas mamrias para e) Pineal e meiatonina
acumulados ejeo do leite.
e liberados
pela neuro- ADH (Antidiurtico) Controla a excreo de gua 03. Ao localizar sua presa, um animal apresenta taquicardia, di-
hipfise). pelos tbulos renais. latao de pupila e tremor geral do corpo. Esse fato se deve
T3 (triiodotironina) e Estimulante do metabolismo liberao de:
T4 (tetraiodotironina celular (consomem iodo para a) adrenalina, pelo sistema nervoso simptico.
Tireide ou tiroxina) a sua produo). b) adrenalina, pelo sistema nervoso parassimptico.
Dificulta a retirada de clcio c) acetilcolina, pelo sistema nervoso simptico.
Calcitonina dos ossos pelo paratormnio. d) acetilcolina, pelo sistema nervoso parassimptico.
Mantm nveis normais de e) acetilcolina, pelo sistema nervoso central.
clcio no sangue, retirando
Paratireides Paratormnio dos ossos e estimulando a
sua absoro no intestino e 04. O cretinismo e o nanismo representam hipofunes das se-
reabsoro nos tbulos renais. guintes glndulas:
Equilbrio hidrossalino a) tireide e paratireides.
Aldosterona (reabsoro tubular de Na+ b) adeno-hipfise e tireide.
e Cl). c) adeno-hipfise e paratireides.
Crtex das
Controle do metabolismo dos d) supra-renais e tireide.
Cortisol aucares a partir das protenas e) tireide e hipfise.
Adrenais e gorduras.
Liberao de hormnios 05. O diabetes insipidus e o diabetes mellitus resultam, respecti-
Andrognios masculinos e femininos na vamente, de deficincia:
circulao.
a) do lobo posterior da hipfise e do pncreas.
Aumento da presso arterial e
Medula Das Adrenalina do ritmo cardaco. Aumenta b) do pncreas e do lobo posterior da hipfise.
Adrenais c) do crtex adrenal e do pncreas.
o acar no sangue.
Baixa a glicose no sangue ao d) do pncreas e do crtex adrenal.
Pncreas Insulina aumentar o armazenamento e) da parte excrina e da parte endcrina do pncreas.
(glndula de glicognio no fgado.
mista) Estimula a quebra do 06. Qual das alternativas indica corretamente os componentes do
Glucagon glicognio no fgado. Sistema Nervoso Central (SNC) e do Sistema Nervoso Perifrico
Ovrios Estrognios Caracteres sexuais secundrios (SNP)?
(glndulas femininos.
Gnglios Medula
mistas) Progesterona Atuao sobre o tero. Encfalo Nervos
Nervosos Espinhal
Testculos
(glndulas Testosterona Caracteres sexuais secundrios a) SNC SNC SNC SNP
masculinos. b) SNC SNP SNC SNC
mistas)
c) SNC SNP SNC SNP
d) SNP SNC SNP SNC

E X E R C C I O 07. Quando voc termina de jogar uma partida de futebol, com


90 minutos de durao, voc nota que h um aumento do nme-
01. Imagine as seguintes situaes: ro de batidas de seu corao por minuto. O responsvel por isso
1. Voc vai tomar uma injeo e fica com o brao distendido, o sistema nervoso:
recebendo a picada da agulha sem nenhuma reao; a) somtico. d) perifrico.
2. Voc estava distrado e algum picou-lhe o brao com um b) autnomo simptico. e) autnomo somtico.
alfinete; a reao foi um salto. c) autnomo parassimptico.

UECEVEST 301

Apostilas UECEVEST mod3.indb 301 06/02/2011 10:04:30


B I O LO GIA 1

08. Considere as seguintes funes de controle do sistema end- c) controle da atividade esqueltica.
crino: d) controle da fala.
I. Concentrao de clcio e fsforo. e) controle da respirao e circulao.
II. Crescimento geral do corpo.
III. Atividade das gnadas. 16. (PUC-MG) O esquema a seguir representa um processo de
IV. Metabolismo do acar no corpo dos mamferos. regulao endcrina por fatores internos e externos ao organismo.
As glndulas que correspondem a estas funes so respectiva-
mente:
a) paratireides, hipfise, hipfise e pncreas.
b) tireide, hipfise, hipfise e pncreas.
c) paratireides, hipfise, hipfise e timo.
d) supra-renal, hipfise, timo e pncreas.
e) hipfise, supra-renal, pncreas e tireide.

09. A ocorrncia de apatia e falta de apetite num ser humano,


bem como a presena de dilatao na altura da garganta (bcio),
podero indicar que o mesmo est com hipofuno da glndula:
a) timo. d) pncreas.
b) paratireide. e) nenhuma das anteriores.
c) tireide. De acordo com o esquema e seus conhecimentos, correto afir-
mar, EXCETO:
10. Se analisarmos o sangue de uma pessoa em situao de emer- a) Estmulo e inibio atuam coordenadamente opondo-se a
gncia ou perigo, ou num momento de raiva ou susto, podere- grandes variaes na concentrao plasmtica de tiroxina em
mos identificar o aumento do hormnio: indivduos normais.
a) tiroxina. d) ocitocina. b) Tiroxina pode atuar como hormnio que contribui para nos
b) corticotrofina. e) adrenalina adaptarmos ao frio.
c) gonadotrofina. c) A reduo nos nveis de TSH sempre indicador de
hipotireoidismo.
11. Um dficit de gua no sangue estimula certas clulas no hipo- d) A carncia nutricional de iodo pode determinar a reduo na
tlamo que, por sua vez, levam a hipfise a liberar: produo de tiroxina e o aumento de volume da tireide.
a) Ocitocina.
b) Adrenalina. 17. (PUC-SP) Nas alternativas a seguir so mostrados trs itens:
c) Secretina. I. parte do sistema nervoso estimulada;
d) Hormnio antidiurtico. II. substncia liberada;
e) Hormnio luteinizante. III. efeito sobre o ritmo cardaco.
12. Um paciente adulto procurou um endocrinologista porque Qual das alternativas apresenta corretamente o que se d com
estava com baixo peso, metabolismo basal muito alto, nervosis- uma pessoa em situao de perigo?
mo e globo ocular saliente (exoftalmia). A disfuno hormonal a) I - simptico; II - acetilcolina; III - aumento
que poderia ser responsvel pelo quadro apresentado pelo pacien- b) I - parassimptico; II - acetilcolina; III - diminuio
te envolve: c) I - simptico; II - adrenalina; III - aumento
a) o pncreas. d) a tireide. d) I - parassimptico; II - adrenalina; III - diminuio
b) a paratireide. e) a supra-renal. e) I - simptico; II - acetilcolina; III - aumento.
c) a adrenal.

13. Para se observar corpos de neurnios, o melhor seria fazer


cortes histolgicos com material colhido a partir de:
G A B A R I T O
a) nervos raquianos. d) medula ssea. 01. 02. 03. 04. 05.
b) nervos cranianos. e) crtex cerebral. 06. c 07. b 08. a 09. c 10. e
c) massa branca da medula.
11. d 12. d 13. e 14. e 15. e
14. (Fuvest-SP) Qual dos seguintes comportamentos envolve
maior nmero de rgos do sistema nervoso?
a) levantar a perna quando o mdico toca com martelo no
joelho do paciente. REFERnCIAS BIBlIOGRFICAS
b) salivar ao sentir o aroma de comida gostosa.
c) piscar com a aproximao brusca de um objeto. AMABIS e MARTHO. Biologia das Clulas. Volume 1. 2 edi-
d) retirar bruscamente a mo ao tocar num objeto muito o. So Paulo. Editora Moderna, 2004.
quente.
e) preencher uma ficha de identificao.

15. (UFRS) O crebro faz o controle de vrias funes do orga-


nismo. Do ponto de vista evolutivo, as estruturas mais antigas do
crebro dos vertebrados so responsveis por:
a) controle da memria.
b) processamento visual.

302 UECEVEST

Apostilas UECEVEST mod3.indb 302 06/02/2011 10:04:31


P R - V E S T I B U l A R

BIOLOGIA 1I

Apostilas UECEVEST mod3.indb 303 06/02/2011 10:04:36


Caro(a) Aluno(a),

Para facilitar o acompanhamento de tais contedos, abaixo esto indicadas as


nomenclaturas utilizadas pela UECE e pelo ENEM:
UECE ENEM
Platelmintos e nematelmintos Grandes linhas da evoluo dos seres vivos.
Tipos de ciclo de vida.
Moluscos e aneldeos Grandes linhas da evoluo dos seres vivos.
Tipos de ciclo de vida.
Aritropodes e equinodermos Grandes linhas da evoluo dos seres vivos.
Tipos de ciclo de vida.
Cordados Grandes linhas da evoluo dos seres vivos.

Apostilas UECEVEST mod3.indb 304 06/02/2011 10:04:37


BIO LO G I A 1 I

PlATElmInTOS E nEmATElmInTOS recolhe os excretas e a gua, jogando-os para o meio exterior


(UECE/EnEm) atravs de pros excretores.

So vermes de corpo achatado, triblsticos, acelomados, com Classificao
simetria bilateral. Podem ser de vida livre ou parasitas, mas a maio- Turbelrios: Platelmintos de vida livre, com epitlio ciliado.
ria parasita. Novidades evolutivas em relao aos celenterados: Ex: Planria (Dugesia tigrina);
Simetria Bilateral;
Trs folhetos germinativos (triblsticos); Trematdeos: Vermes parasitas com epiderme no-ciliada e
Incio da cefalizao do sistema nervoso. uma ou mais ventosas.
Ex: Fasciola hepatica / Schistosoma mansoni;
Caractersticas gerais
Acelomados Cestdeos: Formas parasitas com corpo dividido em proglotes.
Triploblsticos (triblsticos); Ex: Taenia solium.
Simetria Bilateral;
Cefalizao; Doenas causadas por Platelmintos
Sistemas excretor e digestivo;
Corpo achatado dorsoventralmente; Esquistossomose
Sistema nervoso: um par de gnglios anteriores com cordes. O esquistossomo (Schistosoma mansoni) o verme causador
da esquistossomose, uma verminose bastante perigosa e comum
Sistema digestivo (planria) em reas com saneamento precrio. Os esquistossomos tm se-
Incompleto: tem boca, mas no tem nus; xos separados; a fmea mede cerca de 1,5 cm de comprimento e
Carnvora; o macho, cerca de 1 cm. O esquistossomo macho possui um ca-
Boca na regio ventral; nal onde a fmea se abriga na poca da reproduo, o chamado
Faringe prottil: se desloca exteriormente. canal ginecforo. O verme vive geralmente nas veias que ligam
o intestino ao fgado das pessoas. A presena desses vermes e de
Etapas da digesto uma grande quantidade de ovos pode provocar o rompimento
Digesto inicia-se extracorprea; dessas veias. Alm disso, ocorre um aumento no volume abdo-
Aps a digesto extracorprea, o alimento sugado para o in- minal devido ao crescimento desproporcional do fgado e do
testino onde ocorre liberao de enzimas que fazem a digesto bao, causado pelo vazamento de plasma atravs das veias rom-
extracelular; pidas. Por isso, a esquistossomose tambm conhecida como
Posteriormente ocorre absoro de partculas pelas clulas di- barriga-dgua. Entre outros sintomas, alm do aumento do
gestivas: digesto intracelular; volume do abdome, podem ocorrer dores abdominais, clicas,
Finalizando, ocorre a distribuio das partculas para os tecidos nuseas, inflamao do fgado e enfraquecimento do organis-
por difuso; mo. Inicialmente, o verme pe seus ovos nas veias do intestino
do hospedeiro. Esses ovos atravessam as paredes das veias e do
intestino e so eliminados juntamente com as fezes. Os ovos
que caem na gua transformam-se em larvas, os miracdios. Es-
tes penetram no corpo de um caramujo e ali transformam-se
em larvas com cauda, chamadas cercrias. Depois de formadas,
as cercrias saem do caramujo e passam novamente para a gua.
As cercrias, ento, podem penetrar a pele humana, atingindo a
corrente sangunea e, finalmente, as veias que ligam o intestino
Sistema nervoso (planria) ao fgado, onde se desenvolvem e se transformam em vermes
Ganglionar ventral. adultos, fechando o ciclo.
Ocelos (sensorial).

Sistema reprodutor
Reproduo Assexuada: Fragmentao, regenerao (Quando
ocorre, a primeira parte a se formar onde reside o gnglio
cerebride).

Regenerao
Reproduo Sexuada: os platelmintos podem ser monicos,
como as planrias e tnias, ou diicos, como os esquistossomos. Ciclo de vida do Schistosoma mansoni

Tenases
Sistema Excretor
Localizado nas laterais, constituido por protonefrdio: atravs As Tnias possuem um corpo segmentado composto por
dos batimentos dos flagelos, a clula-flama ou solencito anis (progltides). O conjunto de progltides chamado de

UECEVEST 305

Apostilas UECEVEST mod3.indb 305 06/02/2011 10:04:39


B I O LO GIA 1I

estrbilo. As progltides grvidas, isto , aquelas que possuem Doenas causadas por Nematelmintos
ovos e so maduras localizam-se no segmento terminal da tnia.
Habitualmente, para efeitos de esquematizao, divide-se o corpo Ascaridase
da tnia em 3 zonas: o esclex ou cabea, o pescoo e o estrbilo. Doena causada pelo verme Ascaris lumbricoides, vulgarmente
Entre as tnias existentes, trs tm o homem como hospedeiro denominado lombriga, cujo corpo alongado e cilndrico, com as
definitivo, so elas: a Taenia solium (porco), Taenia saginata (car- extremidades afiladas. O comprimento varia entre 15 e 35 cent-
ne bovina) e Diphyllobothrium latum (peixe). metros. Os machos apresentam a cauda enrolada e so menores
que as fmeas. A dimenso do corpo desses vermes varia de acordo
com o seu nmero e intensidade do parasitismo. O nmero pode
chegar a 600 exemplares num mesmo hospedeiro. A transmisso
desta verminose d-se por ingesto de ovos embrionados, atravs
de mos sujas de terra, por alimentos ou gua contaminados. Cada
fmea pe mais de 200 mil ovos por dia. Portanto, se considerar-
mos o grande nmero de pessoas portadoras da verminose e, prin-
cipalmente, as condies precrias de higiene e saneamento, fcil
perceber a facilidade de se contrair a doena. Ao evacuar no solo e
ao ingerir alimentos e gua contaminados, as crianas expem-se
com maior facilidade, desrespeitando, assim, as mais elementares
regras de higiene. Os ovos ingeridos liberam larvas que rompem a
parede intestinal, caindo na circulao, iniciando um percurso pelo
fgado, corao e pulmes. Dos pulmes, as larvas passam para as
Ciclo da Taenia solium vias respiratrias (brnquios, traqueias, laringe) e depois para o tubo
digestivo. No tubo digestivo, evoluem para as formas adultas que se
Obs.: reproduzem, reiniciando o ciclo. O nico hospedeiro o homem.
Neurocisticercose: A cisticercose humana doena gravs-
sima, pois os cisticercos se localizam no sistema nervoso central
(neurocisticercose), nos olhos, msculos e nas vceras. Nestes lo-
cais, podem permanecer at 30 anos, determinando crises con-
vulsivas, cefaleias, vmitos, alteraes de viso, hidrocefalia e at
mesmo a morte. Os ovos das tnias so muito resistentes inati-
vao atravs de substncias qumicas, mas podem ser destrudos
pela fervura acima de 90C. Dessa forma, os cuidados higinicos
so importantes para se evitar a transmisso dessa doena. H
enfermidades contra as quais, at o presente momento, nada se
pode fazer para extermin-las; outras, no entanto, como a cisti-
cercose devem e podem ser eliminadas de nossa populao.

Nematelmintos (=Asquelmintos)

Caractersticas gerais:
Corpo alongado, cilndrico e afilado (extremidades);
Cutcula resistente; Ciclo do Ascaris lumbricoides
Pseudocelomados;
Triblsticos; Ancilostomose
Simetria bilateral. uma doena causada pelo Ancylostoma duodenale, como
tambm pelo Necator americanus. Ambos so vermes nematel-
Estrutura corporal mintos (asquelmintos), de pequenas dimenses, medindo entre
No apresentam: sistemas respiratrio, circulatrio ou esquel- 1 e 1,5cm. A doena tambm conhecida popularmente como
tico. Os vermes de vida livre transportam os gases respiratrios amarelo, doena do jeca-tatu, mal-da-terra, anemia-dos-
por difuso; j os parasitas so anaerbios. mineiros, opilao, etc. As pessoas portadoras dessa verminose
so plidas, com a pele amarelada, pois os vermes vivem no in-
Sistema digestivo testino delgado e, com suas placas cortantes ou dentes, rasgam as
Completo (boca e nus) paredes intestinais, sugam o sangue e provocam hemorragias e
Primeiros da escala zoolgica a apresentarem sistema digestivo anemia. A pessoa se contagia ao manter contato com o solo con-
completo; a boca circundada por papilas ou lbios (algumas taminado por dejetos. As larvas filariides penetram ativamen-
espcies possuem dentes). te atravs da pele (quando ingeridas, podem penetrar atravs da
mucosa). As larvas tm origem nos ovos eliminados pelo homem.
Sistema nervoso Os vermes adultos vivem no intestino delgado do homem. De-
Ganglionar. pois do acasalamento, os ovos so expulsos com as fezes (a fmea
do Ancylostoma duodenale pe at 30 mil ovos por dia, enquan-
Sistema excretor to a do Necator americanus pe 9 mil). Encontrando condies
Tbulos, em forma de H (existe um poro excretor). favorveis no calor (calor e umidade), tornam-se embrionados 24
Renetes horas depois da expulso. A larva assim originada denomina-se
rabditide. Abandona a casca do ovo, passando a ter vida livre
Sistema reprodutor no solo. Depois de uma semana, em mdia, transforma-se numa
Seres diicos (reproduo sexuada). larva que pode penetrar atravs da pele do homem, denominada

306 UECEVEST

Apostilas UECEVEST mod3.indb 306 06/02/2011 10:04:41


BIO LO G I A 1 I

larva filariide infestante. Quando os indivduos andam descal- sexuais. Aps cerca de oito meses da infeco inicial (perodo pr-
os nestas reas, as larvas filariides penetram na pele, migram patente), comeam a produzir microfilrias que surgem no sangue,
para os capilares linfticos da derme e, em seguida, passam para assim como em muitos rgos. O mosquito infectado quando
os capilares sanguneos, sendo levadas pela circulao at o co- pica um ser humano doente. Dentro do mosquito as microfilrias
rao e, finalmente, aos pulmes. Depois, perfuram os capilares modificam-se ao fim de alguns dias em formas infectantes, que mi-
pulmonares e a parede dos alvolos, migram pelos bronquolos e gram principalmente para a cabea do mosquito.
chegam faringe. Em seguida, descem pelo esfago e alcanam o
intestino delgado, onde se tornam adultas.

Ancylostoma duodenale O mosquito do gnero Aedes pode transmitir filariose

Oxiurose
A oxiurose ou enterobiose uma parasitose intestinal causada
por um pequeno verme, cujo nome cientfico Enterobius vermicu- E X E R C C I O
laris ou Oxyuros. A fmea apresenta cauda reta e afilada e atinge 10
milmetros; j o macho, com cauda recurvada, atinge 4 milmetros. 01. (UECE 2007.1) A Taenia solium um organismo bastante
O Oxyuros de distribuio ampla, sendo provavelmente o mais conhecido quando se cuida da sade humana. Evolutivamente,
comum e disseminado dos helmintos que parasitam o homem, podemos afirmar, corretamente, que este organismo pratica uma
ocorrendo mesmo em populaes que apresentam condies sa- forma de nutrio
nitrias satisfatrias. Os Oxyuros so vermes do tipo cilndrico a) ingestora como a de outros animais.
(nematelmintos) do tipo scaris, porm muito pequenos, medem b) absorvedora consequente da sua atividade de parasita.
de 5 a 12 mm de comprimento, brancos, com a cabea obtusa e c) absorvedora consequente de sua atividade de fungo.
vesicular. Ocupam a parte final do reto, causando intenso prurido d) ingestora como a de outros protozorios.
anal, dores abdominais e irritabilidade. Os Oxyuros encontram-se
ocasionalmente nas crianas e nos jovens. Entre as vrias formas 02. (UFC 2007) Ao longo da histria evolutiva do Reino Animal,
de transmisso, predomina as numerosas modalidades de auto- os mecanismos fisiolgicos foram se aperfeioando em cada t-
infeco, a deposio de larvas nas roupas de cama e de uso pessoal. xon. No que se refere excreo, o mecanismo mais primitivo de
Os ovos penetram pela boca, atravs dos dedos e dos alimentos, eliminao de excretas utilizado pelos metazorios formado por:
e os embries passam para os intestinos, onde vivem. As fmeas, a) brnquias, j que a vida originou-se no ambiente aqutico.
fertilizantes, encontram-se em grande nmero nas regies de ceco b) nefrdios, que comunicam a cavidade corporal com o meio
e no clon e passam com os ovos para as fezes, parecendo pequenas externo.
linhas brancas. noite as fmeas saem para o nus, onde causam c) clulas-flama, que possuem uma extremidade que se abre na
irritao. As crianas coam-se e os ovos e os vermes introduzem-se superfcie corporal.
nas unhas; e se levam, por hbito, os dedos boca, facilmente os d) glomrulos, ligados aos vasos sanguneos, que reabsorvem
engolem e assim se reinfectam. As medidas profilticas compreen- gua, ons e outras substncias.
dem principalmente a educao sanitria, como lavar as mos antes e) tbulos de Malpighi, onde ocorre passagem dos resduos da
de ingerir e preparar os alimentos. digesto diretamente para o exterior.

03. (UFS) Comparando-se os ciclos de vida de platelmintos e


nematides parasitas do homem, pode-se afirmar que estgios
larvais de vida livre ocorrem somente nos gneros
a) Schistosoma e Ancylostoma
b) Ancylostoma e Ascaris
c) Ascaris e Taenia
d) Taenia e Wuchereria
e) Wuchereria e Schistosoma
Ovos de Enterobius vermicularis vistos ao microscpio
04. O parasitismo uma associao em que uma das espcies
Filariose vive sobre a outra ou no seu interior, alimentando-se dela. Em
A filariose a doena causada pelo parasita Wuchereria bancrof- relao s parasitoses analise as proposies a seguir, com relao
ti que se aloja nos vasos linfticos causando linfedema. Essa doena algumas caractersticas deste tipo de associao, escrevendo (V)
tambm conhecida como elefantase, devido ao aspecto de perna se a afirmativa for verdadeira ou (F) se for falsa.
de elefante do paciente. Tem como transmissor os mosquitos dos ( ) O piolho, que vive e se reproduz no cabelo humano um
gneros Culex, Anopheles, Mansonia ou Aedes, presentes nas regies bom exemplo de ectoparasita.
tropicais e subtropicais. Quando o nematdeo obstrui o vaso lin- ( ) Tnia(Solitria) um parasita heteroxeno em contraposio a
ftico o edema irreversvel, da a importncia da preveno com Ascaris (lombriga), que um parasita monoxeno, cujo ciclo
mosquiteiros e repelentes, alm de evitar o acmulo de guas para- evolutivo inteiramente realizado em um s hospedeiro.
das em pneus velhos, latas, potes e outros. As larvas so transmiti- ( ) Quando um parasita caracterstico de uma certa regio,
das pela picada dos mosquitos. Da corrente sangunea elas dirigem- mantendo-se em equilbrio durante longos perodos de
se para os vasos linfticos, onde se maturam nas formas adultas tempo, fala-se em endemia.

UECEVEST 307

Apostilas UECEVEST mod3.indb 307 06/02/2011 10:04:43


B I O LO GIA 1I

( ) Esquistossomose, doena de Chagas e malria so trs 09. O doente que apresenta cisticercose:
doenas parasitrias de grande repercusso no Brasil. a) foi picado por Triatoma.
( ) Um caso de parasitismo muito conhecido caracteriza b) nadou em gua com caramujo contaminado.
certos peixes, chamados rmoras, que grudam-se atravs de c) andou descalo em terras contaminadas.
ventosas superfcie ventral de tubares. d) comeu carne de porco ou de vaca com larvas de tnia.
e) ingeriu ovos de tnia.
05. A esquistossomose no Brasil compromete a sade de milhares
de brasileiros, incapacitando-os para o trabalho e, s vezes, obri- 10. Formularam-se algumas hipteses sobre o motivo de um me-
gando-os a uma vida socioeconmica precria. causada pelo nino ter contrado ascaridase.
parasita Schistosoma mansoni, que se transmite ao homem e aos I. Andou descalo sobre terra.
animais atravs: II. Nadou em lagoas com caramujos.
a) do banho em rios e lagos contaminados com os miracdios III. Comeu carnes mal-cozidas.
do parasita. IV. Levou boca mo suja de terra.
b) do banho em rios e lagos contaminados com o ovo do V. Bebeu gua no potvel.
parasita.
c) do contato com guas dos rios e lagos contaminados com as So procedentes as hipteses:
larvas adultas do parasita. a) I e II. d) III e V.
d) do contato com guas de rios e lagos contaminados com as b) I e III. e) IV e V.
cercrias do parasita. c) II e IV.
e) da picada do inseto Anopheles.
11. Na evoluo dos metazorios, clulas nervosas so encontra-
06. Uma criana foi internada em um hospital com convulses e das primeiro de forma difusa e s depois centralizam-se em um
problemas neurolgicos. Aps vrios exames, foi diagnosticada gnglio cerebride. Essa afirmao pode ser fundamentada a par-
cisticercose cerebral. A me da criana iniciou, ento, um pro- tir do estudo da anatomia comparada dos:
cesso contra o aougue do qual comprara carne todos os dias, a) cnidrios e platielmintes
alegando que este lhe forneceu carne contaminada com o verme b) plastielmintes e aneldeos
causador da cisticercose. A acusao contra o aougue: c) porferos e aneldeos
a) No tem fundamento, pois a cisticercose transmitica d) artrpodos e moluscos
pela ingesto de ovos de tnia eliminadas nas fezes de
hospedeiros. 12. (Mackenzie) A elefantase uma verminose provocada por
b) No tem fundamento, pois a cisticercose contrada quando um nematide, seu principal sintoma o inchao de ps e pernas.
a criana nada em lagoas onde vivem caramujos hospedeiros Esse inchao provocado:
do verme. a) pelo acmulo de vermes nos vasos linfticos, impedindo
c) Tem fundamento, pois a cisticercose transmitida pelo a reabsoro da linfa, que se acumula nos espaos
consumo de carne contaminada por larvas encistadas, os intercelulares.
cisticercos. b) pelo entupimento de vasos sanguneos, causado pela
d) Tem fundamento, pois a cisticercose transmitida pelo coagulao do sangue na tentativa de expulsar os vermes.
consumo de ovoso da tnia, os cisticercos, que ficam c) pelo acmulo de vermes nos capilares sanguneos,
alojados na carne do animal hospedeiro. dificultando o retorno do sangue.
d) pela reao do sistema imunolgico a presena dos vermes.
07. Considere o esquema abaixo sobre o ciclo de vida do Schis-
tosoma mansoni: As trs formas (I, II, III), sob as quais o parasita 13. Analise as seguintes medidas profilticas:
pode ser encontrado na gua, correspondem, respectivamente, a: I. Uso de sapatos e eliminao higinica das fezes humanas.
II. Cozinhar totalmente a carne de porco.
III. Destruio de caramujos planorbdeos.
As medidas I, II e III previnem, respectivamente:
a) o amarelo, a tenase e a esquistossomose.
b) a ascaridase, a cisticercose e a esquistossomose.
c) a ascaridase, a esquistossomose e a cisticercose.
d) o amarelo, a cisticercose e a esquistossomose.
a) miracdio, cercria e ovo. e) a elefantase, a tenase e a barriga-dgua.
b) ovo, miracdio e cercria.
c) cercria, miracdio e ovo. 14. (F. Objetivo-SP) Se uma determinada regio estivesse ocor-
d) ovo, cercria e miracdio. rendo alta incidncia de parasitismo por Ancylostoma duodenale,
e) miracdio, ovo e cercria. qual das medidas abaixo, entre outras, deveria ser tomada?
a) Eliminar os mosquitos vetores.
08. Obtiveram-se as seguintes informaes sobre o ciclo vital de b) Alertar a populao para o uso de calados.
um determinado parasita humano: c) Exame mais acurado nos matadouros, procura de cistos do
I. Seu hospedeiro intermedirio no um artrpode; parasita.
II. Apresenta mais de um estgio larval; d) Destruir caramujos das lagoas.
III. Sua fase adulta apresenta dimorfismo sexual. e) Orientar a populao quanto aos cuidados com a gua ou
com vegetais contaminados.
O parasita em questo poderia ser:
a) Taenia solium. d) Ancylostoma duodenale. 15. (FUVEST-SP) A falta de instalaes sanitrias adequadas
b) Schistosoma mansoni. e) Trypanosoma cruzi. diretamente responsvel pelo aumento da incidncia das seguin-
c) Plasmodium vivax. tes doenas endmicas:

308 UECEVEST

Apostilas UECEVEST mod3.indb 308 06/02/2011 10:04:43


BIO LO G I A 1 I

a) esquistossomose, doena de Chagas, malria. caramujos e bivalves, o manto secreta uma concha calcrea dura.
b) esquistossomose, amarelo, tenase. O corpo mole, possuindo geralmente um exoesqueleto (con-
c) bcio endmico, amarelo, tenase. cha), formado a partir das glndulas presentes no manto (que
d) malria, doena de Chagas, bcio endmico. todos possuem). O corpo divide-se na maioria dos casos em ca-
e) doena de Chagas, malria, amarelo. bea, onde h uma concentrao de massas ganglionares nervosas
formando os olhos; massa visceral, local onde est a maior parte
16. (UFRS) A restrio do tamanho e da forma em planrias est dos rgos; e p, que um rgo musculoso utilizado na locomo-
diretamente relacionada: o, que assume vrias formas.
a) ausncia de um sistema circulatrio.
b) ao sistema nervoso difuso.
c) presena de protonefrdeos.
d) ao hermafroditismo.
e) possibilidade de regenerao.

G A B A R I T O
01. 02. 03. 04. 05. 06. a
07. b 08. b 09. c 10. e 11. a 12. a
13. a 14. b 15. a 16.a Sistema respiratrio
Branquial ou pulmonar.

Sistema digestivo
comum apresentarem uma cavidade bucal com uma mem-
brana recoberta por pequenos dentes, chamada de rdula, com
mOlUSCOS E AnElDEOS (UECE/EnEm) a qual raspam os alimentos antes de degluti-los (ausente nos
bivalves).
Do grego molllis, que significa mole. Os moluscos so o
segundo maior grupo de animais em nmero de espcies, sendo Sistema circulatrio
suplantado apenas pelos artrpodes e, em nmero de indivduos, Aberto: formado por um corao musculoso dorsal, artrias,
pelos artrpodes e pelos nematelmintos. Originaram-se h 600 veias, e hemocelas. Presente na maioria dos mosluscos.
milhes de anos e h 100 milhes de anos j tnhamos as 7 classes Fechado: presente nos cefalpodes (polvo, lula, spia, etc), no
de moluscos atuais. Atravs dos tempos, os moluscos evoluram e qual o lquido sanguinio circula sempre pelo interior de vasos.
encontraram novos hbitats. H cerca de 400 milhes de anos al-
guns moluscos, primeiro os bivalves, comearam a habitar a gua Sistema excretor
doce. H cerca de 300 milhes de anos alguns gastrpodes ini- Realizada por um par de metanefrideos, cujas aberturas ciliadas
ciaram a sua migrao para o ambiente terrestre e de gua doce. retiram as excrees da cavidade pericardia, eliminando-as
Atualmente os moluscos vivem em todas as partes do mundo. atravs dos poros excretores.
Dos oceanos mais profundos at as montanhas mais altas.
Sistema nervoso
Caractersticas gerais do tipo ganglionar com vrios pares de gnglios: cerebrides,
Eumetazorios; pedais, pleurais e viscerais, de onde partem nervos para todo o
Triblsticos; organismo.
Celomados;
Protostmios, quanto origem da boca; Sistema reprodutor
Todos os moluscos apresentam uma simetria bilateral, apesar Os escafpodos so unissexuados, possuindo uma gnada que
de, em alguns casos (como o caracol) o corpo do indivduo produz gametas masculinos e femininos, uma fecundao ex-
sofrer uma toro durante o desenvolvimento embrionrio; terna e desenvolvimento indireto com a larva trocfora.
A maioria marinha, ao longo das praias ou em guas rasas, Os gastrpodes so hermafroditas, apresentando uma glndula
outros habitam a gua doce, e os caracis e lesmas so terrestres; denominada ovotestis capaz de produzir vulos e espermato-
Muitos so de vida livre, alguns geram larvas parasitas em pei- zides, apresentando fecundao interna com cpula recproca
xes, e outros servem de hospedeiros intermedirios de vermes e desenvolvimento direto.
parasitas; Nos bivalves, os sexos so separados e o desenvolvimento
So de grande importncia econmica os mariscos: ostras, lulas indireto; nos marinhos, vulos e espermatozides so descar-
e outros, que servem de alimento humano; as conchas de bival- regados na gua (fecundao externa) ocorrendo uma larva
ves de gua doce, que so cortadas em botes, e alguns bivalves denominada vliger. Nos bivalves de gua doce da famlia
(Ostrea paradisiaca) que produzem prolas. Unionideae, a fecundao interna, formando-se uma larva
denominada gloqudeo, que possui uma concha bivalve e um
Estrutura corporal longo fio larval.
O corpo revestido por um tecido denominado manto, ca- Nos cefalpodes, polvos e lulas, os sexos so separados. Um
racterstica exclusiva deste filo, com funes de proteo, respi- dos braos do macho funciona como aparelho copulador, sen-
rao e secreo. A estrutura chamada manto o que separa os do especialmente transformado e designado por brao hetero-
moluscos dos outros animais, funcionando como uma cobertura ctilo. Introduzindo na cavidade paleal (do manto) da fmea
para os tecidos moles e delicados; portanto, o manto nada mais o heteroctilo serve para inocular o espermatforo (pacote de
que uma expanso de tegumento. Em alguns moluscos, como os espermatozides). O desenvolvimento direto.

UECEVEST 309

Apostilas UECEVEST mod3.indb 309 06/02/2011 10:04:44


B I O LO GIA 1I

Classificao canais; em alguns arquianeldeos o celoma est completamente


ausente. O celoma pode estar dividido numa srie de comparti-
Classe Aplacophora mentos por septos. Em geral, cada compartimento corresponde
So moluscos pouco conhecidos, no possuem concha e vi- a um segmento e inclui uma poro dos sistemas nervoso e cir-
vem nos mares em profundidades de 3000m ou mais. Ex.: Cha- culatrio, permitindo-lhes funcionar com relativa independn-
etoderma canadensis. cia. Cada segmento est dividido externamente em um ou mais
anis, coberto por uma cutcula segregada pela epiderme e,
Classe Monoplacophora internamente, possui um fino sistema de msculos longitudi-
Vivem exclusivamente nas profundezas marinhas. Ex: Neo- nais. Essas caractersticas so parcialmente comuns aos nema-
pilina. tdeos e aos artrpodes e, por isso, eles foram durante algum
tempo colocados no mesmo grupo sistemtico, o filo Articulata;
Classe Polyplacophora mas, estudos recentes revelaram que essas caractersticas devem
Vivem exclusivamente no ambiente marinho (guas rasas). ser consideradas como convergncias evolutivas. A maioria dos
Moluscos cuja concha constituda por oito placas encaixadas aneldeos possui, em cada segmento, um par de cerdas, porm
os poliquetas (as minhocas marinhas) possuem ainda um par de
sobre o dorso. Ex: Chiton.
apndices denominados parpodes (ou falsos ps). Na extre-
midade anterior do corpo, antes dos verdadeiros segmentos - a
Classe Scaphopoda
cabea - encontra-se o protostmio onde esto os olhos e outros
So moluscos exclusivamente marinhos, cuja concha lembra rgos dos sentidos. Por baixo, encontra-se o peristmio, onde
uma pequena presa de elefante, oca e aberta nas duas extremidades. est a boca. A extremidade posterior do corpo o pigdio, onde
Ex: Dentalium. est localizado o nus e os tecidos que do origem a novos seg-
mentos durante o crescimento.
Classe Pelecypoda (=Bivalvia)
Moluscos aquticos, marinhos ou de gua doce. Sistema digestivo
Ex: mexilhes, ostras. O tubo digestivo bastante especializado, devido variedade
das dietas, uma vez que muitas espcies so predadoras, outras
Classe Gastropoda detritvoras, outras se alimentam por filtrao, outras ainda in-
Podem viver na gua doce, no mar ou no ambiente terrestre. gerem sedimentos, dos quais o intestino tem de separar a parte
Ex: caramujos, caracis e lesmas. nutritiva; finalmente, as sanguessugas alimentam-se de sangue de
outros animais, por suco.
Classe Cephalopoda
Vivem exclusivamente no mar. Sistema respiratrio
Ex: lulas, polvos e spias. Respirao tegumentar ou cutnea indireta (pela pele, indo
para o sistema circulatrio primeiro). O oxignio passa por difu-
Aneldeos so pela epiderme para a corrente sangunea. Pela corrente san-
Vulgarmente chamam-se aneldeos aos vermes segmentados, gunea o oxignio ser distribudo para os tecidos. O dixido de
com o corpo formado por anis, do filo Annelida como a mi- carbono ser eliminado por difuso dos vasos sanguneos para o
nhoca e a sanguessuga. Existem mais de 15.000 espcies desses meio externo;
animais em praticamente todos os ecossistemas, terrestres, ma-
rinhos e de gua doce. Encontram-se aneldeos com tamanhos Sistema circulatrio
desde menos de um milmetro at a mais de 3 metros. Foram os primeiros a surgirem com Sistema Circulatrio Fe-
chado (circula apenas dentro dos vasos sanguneos). O sistema
vascular composto por um vaso sanguneo dorsal que leva o
sangue no sentido da cauda e outro ventral, que o traz na
direo oposta. Atravs da contrao do vaso sanguneo dorsal,
o sangue atinge os 4 coraes. Os coraes bombeiam o sangue
para o vaso sanguneo ventral, que o distribu para todo corpo.

Sistema excretor
So 2 metanefrdeos por anel. O excreta principal a ureia e
o nefrstoma recolhe os excretas da cavidade celomtica; os ex-
cretas sero eliminados atravs do nefrodiporo que est na epi-
derme. A ureia eliminada transformada por bactrias no solo
em amnia, fundamental para a sntese de protenas das plantas;
Caractersticas gerais
Sistema nervoso
Triblsticos;
O sistema nervoso formado por um cordo ventral, a par-
Celomados;
tir do qual saem nervos laterais em cada segmento. J capaz
Corpos divididos em anis ou metmeros;
de organizar seus movimentos. Gnglio o corpo celular dos
Terrestres ou aquticos;
neurnios. Cada anel tem 2 gnglios nervosos (secundrios aos
Simetria bilateral.
cerebrais). Possui, tambm, clulas tcteis (tatorreceptoras e qu-
miorreceptoras);
Estrutura corporal
Os aneldeos so animais de corpo alongado, segmentado,
Sistema reprodutor
triploblsticos, protostmios e celomados, ou seja, com a cavi-
Realizam reproduo sexuada. As minhocas so hermafrodi-
dade do corpo cheia de um fluido onde o intestino e os outros
tas e a fecundao cruzada. Ocorre troca de espermatozides
rgos se encontram suspensos. Os oligoquetas e os poliquetas
entre as minhocas; aps a troca de espermatozides o clitelo se-
possuem celomas grandes, mas, nas sanguessugas, o celoma est
creta o casulo. Ocorre postura de vulos no interior do casulo.
preenchido por tecidos e reduzido a um sistema de estreitos

310 UECEVEST

Apostilas UECEVEST mod3.indb 310 06/02/2011 10:04:45


BIO LO G I A 1 I

Classificao quanto a quantidade de cerdas no corpo 04. possvel identificar nos moluscos trs partes bsicas: cabea,
Classe Oligochaeta (poucas cerdas). p e massa visceral. O que mais pode ser dito corretamente sobre
Ex: Pheretina hawayana (minhoca). os moluscos?
I. A rdula uma estrutura exclusiva dos moluscos a qual
Classe Polychaeta (muitas cerdas). formada por dentes quitinosos dispostos em fileiras
Ex: Nereis virens (nereida). horizontais.
II. A massa visceral envolvida pela concha e formada
Classe Hirudnea (sem cerdas). pelos rgos da digesto, circulao, excreo, respirao e
Ex: Hirudo medicinalis (sanguessuga). reproduo.
III. O p uma regio achatada, fortemente musculosa,
formando sola muscular, o qual utilizado para rastejar,
E X E R C C I O nadar ou cavar.
IV. Os moluscos possuem sistema digestrio incompleto,
01. (UFC 2009) Evidncias moleculares, baseadas em sequn- contendo apenas boca.
cias de RNA, sugerem o parentesco entre moluscos e aneldeos. V. O sistema circulatrio fechado, com sangue fluindo apenas
Esses dados reforam a hiptese de que esses grupos apresentam no interior de vasos.
um ancestral comum. O parentesco entre esses grupos pode ser
evidenciado tambm levando-se em considerao caractersticas Marque alternativa que traz os itens verdadeiros:
biolgicas tais como: a) I, II e V. d) III, IV e V.
a) protostomia, cordo nervoso dorsal e desenvolvimento direto. b) I, II e III. e) I, II, III, IV, V.
b) metameria, presena de celoma e desenvolvimento indireto. c) I, IV e V.
c) presena de celoma, simetria bilateral e clivagem espiral.
d) pseudoceloma, simetria bilateral e respirao branquial. 05. (UECE 2009.1) Nos moluscos, o rgo constitudo de uma
e) protostomia, clivagem espiral e metameria. membrana epidrmica, que possui glndulas responsveis pela
secreo da concha o(a)
02. (UECE 2010.1) O filo Mollusca compreende imensa diver- a) rdula. c) manto.
sidade biolgica dentro do reino animal, perdendo apenas em b) papo. d) p.
nmero de espcies conhecidas para os artrpodes. So predomi-
nantemente marinhos, podem viver nadando livremente ou fixos 06. (PUC) Os moluscos que apresentam a cavidade paleal adap-
a substratos. Embora exista grande nmero de espcies, todos tada respirao area so os:
apresentam caractersticas comuns que definem o grupo. Com a) polvos. d) caracis.
relao aos moluscos analise as afirmaes abaixo b) ostras. e) mexilhes.
I. Polvos so moluscos que apresentam crebros bastante c) aranhas.
desenvolvidos, olhos dotados de cristalino capazes de
formar imagens, semelhantes aos dos vertebrados, e sistemas 07. (CESCEM-SP) A qual dos animais abaixo refere-se a descri-
sensoriais altamente especficos que possibilitam respostas o a seguir?
rpidas aos estmulos ambientais, tornando-os excelentes O sangue circula em vasos; o intestino recebe os alimentos di-
predadores. geridos e os distribui s clulas do corpo; na epiderme, absorve
II. Lulas so cefalpodes que vivem no mar e respiram por oxignio do ar e desprende gs carbnico; transporta os excretas
meio de brnquias, porm, diferentemente de seus parentes nitrogenados a uma srie de tubos que desembocam em poros na
prximos, no possuem concha, e por isso seu corpo macio superfcie do corpo.
muito apreciado na culinria especializada em frutos do mar. a) Planria. d) Aranha.
III. Bivalves so organismos envolvidos em conchas constitudas b) Minhoca. e) Sapo.
por duas valvas que tem por finalidade proteger o molusco c) Gafanhoto.
de seus predadores naturais. Internamente, o corpo desses
animais constitudo por um p muscular e sifes inalante 08. (UFRGS) O que Platelmintos e Moluscos possuem em comum?
e exalante, que promovem a passagem da gua pelas lminas a) O sistema respiratrio.
branquiais, estrutura relacionada a absoro de oxignio, b) A presena de celoma.
alimentos e poluentes dissolvidos. c) O tipo de sustentao.
IV. Todos os gastrpodes apresentam o corpo protegido por d) O sistema digestivo.
conchas, que em sua maioria so espiraladas; o corpo desses e) A ocorrncia de cefalizao.
animais apresenta tentculos sensoriais e rdula, estruturas
09. Na evoluo do sistema digestrio dos animais aparecem, su-
relacionadas, respectivamente, percepo de estmulos
cessivamente, seres sem tubo digestrio, organismos com tubo di-
ambientais e locomoo desses animais.
gestrio incompleto e seres com tubo digestrio completo. Exem-
So corretas as afirmaes plos de animais com essas caractersticas so, respectivamente,
a) somente I, II, e IV. c) somente II e IV. a) esponja, medusas e anmonas
b) somente I e III. d) somente II, III e IV. b) planrias, esponjas e minhocas
c) esponjas, planrios e minhocas
03. A ocorrncia de um sistema circulatrio fechado, sangue com d) medusas, canmonas e moluscos
hemoglobina, trs folhetos embrionrios formando um celoma
verdadeiro e corpo metamerizado so caractersticas que apare- 10. Lula, ostra, mexilho e polvo tm em comum:
cem em conjunto pela primeira vez em: a) sistema nervoso formado por grande massa ganglionar cerebral.
a) Moluscos. d) Platelmintos. b) concha externa.
b) Aneldeos. e) Vertebrados. c) estmago com grande ceco.
c) Insetos. d) cromatforos.
e) respirao branquial.

UECEVEST 311

Apostilas UECEVEST mod3.indb 311 06/02/2011 10:04:45


B I O LO GIA 1I

11. A produo de prolas requer a introduo artificial de pe- ARTRPODES E EQUInODERmOS


quenas partculas estranhas no manto. Este circunda o corpo es- (UECE/EnEm)
tranho e secreta camadas sucessivas de ncar sobre ele. Os animais
so mantidos em cativeiro por muitos anos at que as prolas Os artrpodes (Filo Arthropoda) so o maior grupo de ani-
sejam formadas. Os animais utilizados nesse processo pertencem, mais existentes no mundo e incluem os insetos, aracndeos,
respectivamente, ao filo e classe: crustceos, quilpodes e diplpodes. Esto descritas cientifica-
a) Mollusca e Gastropoda. d) Mollusca e Cephalopoda. mente mais de um milho de espcies de artrpodes, ou seja,
b) Arthropoda e Crustacea. e) Mollusca e Pelecypoda. mais de 4/5 de todas as espcies existentes, desde formas mi-
c) Arthropoda e Insecta. croscpicas do plncton, com menos de um quarto de milme-
tro, at animais de grandes dimenses. O nome vem do grego
12. (UFRGS) As plataformas martimas de prospeco e extrao arthros, articulao e podos, ps; isto , ps articulados. Os
de petrleo desempenham uma funo ecolgica pouco conhe- artrpodes existem em todos os ambientes da terra: no mar,
cida do pblico. Constituem um hbitat para organismos que na gua doce, no meio terrestre e no ar. Existem ainda muitas
vivem fixos ou junto a algum substrato, como corais, algas, mo- formas parasitas e simbiticas. H registos fsseis de artrpodes
luscos e crustceos. Assinale representantes dos filos a que perten- desde o perodo Cambriano. Os artrpodes tm uma carapaa
cem, respectivamente, os grupos de animais acima sublinhados. protetora externa, que o seu esqueleto, formada por uma subs-
a) medusa, craca, lula. tncia resistente e impermevel, chamada quitina, endurecida
b) estrela-do-mar, lesma, tatu-de-jardim. por conter muito carbonato de clcio. Ao crescer, o artrpode
c) hidra, ourio-do-mar, camaro. abandona o esqueleto velho, pequeno e fabrica outro, maior.
d) anmona, ostra, siri. Esse fenmeno chamado muda ou ecdise. Ela ocorre vrias
e) medusa, pepino-do-mar, siri. vezes para que o animal possa atingir o tamanho adulto. Os
artrpodes, no entanto, no possuem apenas patas articuladas,
13. Nas minhocas, a fecundao e o desenvolvimento so: mas tambm, antenas e peas bucais.
a) interna / direto.
b) externa / indireto. Caractersticas gerais
c) interna / indireto. O corpo sempre revestido por um exoesqueleto endurecido
d) externa / direto. contendo quitina (um polissacardeo), que trocado periodica-
e) interna / indireto com larva trocfora. mente, permitindo o crescimento do animal;
So segmentados, mas a metameria mais evidente na fase em-
14. Quanto reproduo, as minhocas (Classe Oligochaeta) so: brionria, pois no adulto h tendncia fuso de segmentos,
a) monicas, isto , cada animal apresenta tanto rgos sexuais originando partes definidas do corpo, como cabea, trax e ab-
masculinos como femininos. dome (entretanto, a segmentao do adulto aparece claramente
b) monicas, isto , cada animal apresenta apenas rgos nos apndices, na musculatura e no sistema nervoso);
sexuais masculinos ou femininos. Apresentam patas e outros apndices articulados (da o nome
c) diicas, isto , cada animal apresenta tanto rgos do filo), formados por vrios segmentos ou artculos, unidos
masculinos como femininos. por juntas mveis, facilitando bastante a locomoo;
d) diicas, isto , cada animal apresenta apenas rgos So dotados de simetria bilateral, adaptativa para animais que
masculinos ou femininos. exploram seu ambiente;
Possuem uma cavidade corprea, o celoma, mas extremamente
15. (UFRN) Assinale a alternativa que relaciona corretamente o reduzido;
animal ao seu respectivo rgo excretor: Apresentam, em sua maioria, alto grau de concentrao e de-
a) Protozorios: glndulas verdes. senvolvimento do sistema nervoso central e dos rgos sensiti-
b) Celenterados: tbulos de Malphigi. vos, o que permitiu, em alguns grupos, a existncia de padres
c) Platelmintos: vacolos contrteis. de comportamento complexos, inclusive organizaes sociais.
d) Insetos: clula-flama.
e) Aneldeos: nefrdios. Estrutura corporal
Exoesqueleto: A existncia de um esqueleto externo duro
16. (PUC-SP) Os aneldeos so animais com o corpo formado formado por um polissacardeo denominado quitina uma das
por muitos segmentos ou metmeros e que apresentam como ca- razes do sucesso alcanado pelos artrpodes. Ao contrrio da
racterstica obrigatria: cutcula fina e flexvel dos aneldeos, os artrpodes possuem o
a) habitat aqutico. exoesqueleto composto por uma cutcula grossa, responsvel pela
b) sistema excretor com um par de nefrdios por segmento. rigidez do corpo. Sua poro externa impermevel, sendo com-
c) respirao branquial. posta de protenas e de cera. A poro interna, mais espessa,
d) hermafroditismo. formada por camadas de quitina e contm ainda pigmentos e car-
e) um par de cerdas por segmento. bonato de clcio. totalmente acelular e secretado pela epiderme
subjacente, estando ligado a ela.
17. (VUNESP-SP) Trocfora e vliger so dois tipos de larvas de: Msculos: A utilidade do exoesqueleto se estende loco-
a) aneldeos. c) equinodermas. moo. Enquanto o movimento dos aneldeos depende dos
b) moluscos. d) insetos. msculos que atuam sobre o liquido celomtico, nos artrpodes
os msculos atuam sobre unidades esquelticas rgidas adjacen-
tes, funcionando como um sistema de alavancas. O exoesque-
G A B A R I T O leto que fornece os pontos para a insero muscular, com a
ao antagnica, ou seja, cada estrutura corporal que se contrai
01. 02. 03. 04. 05. 06. d sob ao de um msculo, distende-se pela ao de outro. Isso
07. b 08. e 09. c 10. e 11. e 12. d permite movimentos elaborados que facilitam a explorao do
13. d 14. 15. e 16. b 17. b ambiente.

312 UECEVEST

Apostilas UECEVEST mod3.indb 312 06/02/2011 10:04:46


BIO LO G I A 1 I

Sistema digestivo transporte gasoso. Insetos, quilpodes e diplpodes possuem o


De forma geral, o tubo digestivo dos artrpodes completo. sangue totalmente incolor, denominado hemolinfa, desprovido
Algumas variaes podem surgir, dependendo do animal. Entre de pigmentos e responsvel apenas pelo transporte alimentar,
os insetos, por exemplo, o tubo digestivo formado por: boca; uma vez que o oxignio chega diretamente aos tecidos pelo sis-
faringe muscular; esfago curto; associado a glndulas salivares, tema traqueal. Isso pode explicar o fato de, apesar da circulao
produtoras de secrees que umedecem o alimento; papo grande aberta, com menor presso sangunea e fluxo mais lento, os mo-
para armazenamento; moela para triturao mecnica; estmago, vimentos serem rpidos. H uma completa independncia entre
ligado a cecos gstricos glandulares, que secretam sucos para a respirao e circulao nos insetos.
digesto qumica; intestino, rea de maior absoro alimentar;
reto, onde feita a absoro final de gua; e nus. Sistema excretor
O sistema excretor dos artrpodes retira excretas nitrogena-
dos das lacunas sanguneas e, atravs de diferentes estruturas, eli-
minam para o meio exterior.

Sistema reprodutor
Os artrpodes, em geral, so diicos. As formas terrestres tm
fecundao interna, utilizando apndices modificados na copu-
lao. J as formas aquticas podem realizar fecundao externa
Tubo digestivo de um inseto ou interna. A maioria das formas apresenta estgio larval, sendo o
estgio adulto atingido atravs de metamorfose. Mecanismos de
Sistema Respiratrio corte precedem a copulao em diversas formas.
Nos artrpodes, podem ser encontrados trs tipos diferentes
de estruturas respiratrias: Classificao
Brnquias: so tpicas das formas que predominam nos ecos- Os artrpodes podem ser classificados em cinco classes prin-
sistemas aquticos, os crustceos. So constitudas por fila- cipais, usando como critrio o nmero de patas.
mentos muito finos, repletos de vasos sanguneos e realizam as
trocas gasosas diretamente da gua. O nmero de brnquias N de patas Classe Exemplos
varivel com o tipo de crustceo que se est estudando. 6 Insetos Barata, mosquito
Traqueias: formam um sistema de tubos areos, revestidos por 8 Aracndeos Aranha, escorpio
quitina, que conduzem o ar diretamente aos tecidos do corpo. 10 Crustceos Camaro, siri
O fluxo do ar regulado pela abertura e pelo fechamento de 1 par por seg. Quilpodes Lacraia
poros especiais situados no exoesqueleto, denominados estig-
2 pares por seg. Diplpodes Piolho-de-cobra
mas. Existem em insetos, aracndeos, quilpodes e diplpodes.
Na respirao traqueal, o sangue no participa; todo o trans- Classe dos Insetos
porte gasoso feito pelas traqueias Os insetos formam a maior classe do Reino Animal, so mais
de 800.000 espcies conhecidas. Os insetos vivem espalhados por
todo o mundo. Desde as regies polares at as zonas tropicais,
passando por rios, mares e oceanos. Embora a aparncia dos in-
setos seja muito variada, o corpo de todos eles dividido em
cabea, trax e abdome. Na cabea h um par de antenas e 1
par de mandbula. Todos os insetos possuem 3 pares de patas.
Nem todos os insetos tm asas. A maioria desses animais alcana
a maturidade atravs da metamorfose. No possvel dizer que
um inseto encontrado sobre uma planta qualquer sem maior im-
portncia econmica seja uma praga. A maior parte das esp-
cies de insetos (cerca de 98%) no se enquadram nessa categoria.
Na realidade, essas espcies todas fazem parte de um delicado
e importante equilbrio biolgico natural, cuja perturbao pelo
homem pode resultar no aparecimento de pragas.
Respirao traqueal dos insetos.
Classe dos Aracndeos
Filotraqueia (=pulmes foliceos): so estruturas exclusivas As aranhas, por exemplo, so aracndeos que apresentam
dos aracndeos, sempre existindo aos pares. Cada pulmo fo- cores variadas, e, quanto ao tamanho, vo do quase invisvel a
liceo uma invaginao (reentrncia) da parede abdominal olho nu at espcies de mais de 20 cm. Todas as aranhas so
ventral, formando uma bolsa onde vrias lamelas paralelas predadoras e sua dieta inclui os insetos, tais como: moscas, mos-
(lembrando as folhas de um livro entreaberto), altamente vas- quitos, grilos, gafanhotos, baratas, etc. Apesar de poucas aranhas
cularizadas, realizam as trocas gasosas diretamente com o ar possurem a capacidade de intoxicar o homem, todas as aranhas
que entra por uma abertura do exoesqueleto. so venenosas. Alm das aranhas, nessa classe so estudados os
escorpies, cujo corpo igual ao da aranha, com uma nica di-
Sistema circulatrio ferena: o abdome dividido em duas partes, pr-abdome e ps-
O sistema circulatrio dos artrpodes do tipo aberto, no abdome. No ps-abdome, encontra-se a glndula que produz o
qual o sangue deixa os vasos e passa a fluir por espaos livres veneno que o animal injeta na vtima com um aguilho. O maior
entre os tecidos, as lacunas ou hemoceles. O corao muscular de todos os escorpies pode atingir at 21 cm e o menor chega
fica situado dorsalmente e bombeia o sangue para todo o corpo. no mximo a 12 mm quando adulto. Os escorpies se destacam
Aracndeos e crustceos possuem sangue quase incolor, conten- entre os aracndeos por terem uma durao de vida que vai alm
do hemocianina como pigmento respiratrio, responsvel pelo de uma estao. Sua longevidade vai dos 2 aos 6 anos. O maior

UECEVEST 313

Apostilas UECEVEST mod3.indb 313 06/02/2011 10:04:47


B I O LO GIA 1I

tempo de vida registrado para um escorpio chega at 8 anos. vivendo em rochas ou semi-enterrados na areia; ocorrem em lu-
Podem viver tanto em lugares desertos quanto nas matas. gares em grande nmero, porm, no formam colnias, algumas
espcies so fixas e outras mveis, nenhuma delas parasitas. A
Classe dos Crustceos estrutura exclusiva dos equinodermos o sistema vascular aqufe-
Os crustceos fazem parte dessa classificao por serem possui- ro ou sistema ambulacrrio. Trata-se de uma parte especializada
dores de pernas articuladas, mas sem espinha dorsal. Como exem- do celoma, responsvel pela locomoo do animal.
plo dessa classe, poderemos citar o camaro, a craca, a lagosta, o
pitu, o siri e o caranguejo, dentre outros. Podemos encontrar crus- Sistema digestivo
tceos em praticamente todos os ambientes do mundo, desde as Geralmente completo, dividindo-se em boca, esfago, est-
fossas abissais dos oceanos, at glacirios e lagoas temporrias dos mago, intestino e nus. Como anexos ocorrem glndulas diges-
desertos. O nome da classe vem do fato de terem um exoesqueleto tivas.
de quitina endurecido pelo acmulo de carbonato de clcio, (do
latim, crusta = carapaa dura). A maior parte destes animais vive Sistema respiratrio
no mar, mas existem alguns caranguejos que so capazes de viver A respirao pode ser realizada atravs do sistema ambulacr-
tambm na terra. O tatuzinho, encontrado nos jardins tambm rio, por ppulas, brnquias; no grupo holoturoidea, pelas rvores
pertence a esta classe e diferenciam-se dos demais artrpodes man- respiratrias cloacais ou hidropulmo.
dibulados por possurem dois pares de antenas.
Sistema excretor
Classe dos Quilpodes No existem rgos excretores diferenciados, os catablitos
Os quilpodes so animais velozes, carnvoros, sua alimenta- so absorvidos por amebcitos e eliminados atravs das ppulas
o faz-se base de larvas e besouros. Como exemplo, podemos (brnquias pequenas e moles) da parede intestinal, dos ps am-
citar as lacraias e as centopeias. Atualmente existem mais de 581 bulacrrios e, ainda, da rvore respiratria. O nitrognio elimi-
espcies descritas. O seu corpo alongado e achatado e divide- nado sob a forma de amnia (NH3).
se em duas partes principais: cabea e tronco, com um nmero
varivel de segmentos, apresentando um par de patas por anel ou Sistema circulatrio
segmento; so animais que no se enrolam. A maioria das esp- No h um verdadeiro sistema circulatrio, este representa-
cies mede menos de 5 cm de comprimento mas algumas atingem do por um sistema hemal formado por lagunas.
os 25 cm. Os quilpodes possuem 1 par de antenas e seu sistema
respiratrio traqueal. Possuem garras na cabea utilizadas para
Sistema nervoso
injetar o veneno na presa, veneno este que no letal para o ho-
formado por um anel peridigestivo e por cordes nervosos
mem, porm causa muita dor.
radiais.

Sistema reprodutor
bem desenvolvido com glndulas mltiplas. Os sexos so
separados, mas no ocorre dimorfismo sexual externo, a fecunda-
o se d na gua e o desenvolvimento indireto, isto , apresenta
larvas especiais.

Quilpode Classificao

Classe Crinoidea (=Crinides)


Classe dos Diplpodes
Incluindo o embu e piolho-de-cobra os diplpodes ou milpe- Nessa classe, encontramos animais conhecidos vulgarmente
des (mil - ps), como tambm so chamados esses animais de lenta como lrios-do-mar. Possuem o corpo caliciforme, munidos ou
locomoo, herbvoros e que se enrolam em espiral. Possuem um no de pednculo. As formas pedunculadas possuem uma haste
corpo cilndrico, com 1 par de antenas e 2 pares de patas locomo- com a qual se fixa a um suporte qualquer. No topo do pednculo
toras por segmento (que podem variar de 20 a 100) e seu sistema encontram-se peas calcreas formando o clice no interior do
respiratrio traqueal. Esses animais habitam de preferncia lugares qual se abrigam as partes moles do animal.
midos, podem ser encontrados embaixo de pedras e folhas mortas
Classe Echinoidea (=Equinides)
ou dentro de troncos apodrecidos; alimentam-se de vegetais mortos.
Rene espcies apresentando forma hemisfrica globosa, re-
presentado pelos ourios-do-mar, e pelas formas discides acha-
tadas como a bolacha-da-praia. Nos ourios-do-mar, o corpo
formado externamente por uma grande carapaa dividida em
uma parte central denominada roseta apical situada no dorso do
animal, e a corona que compreende o resto da carapaa.

Classe Asteroidea (=Asterides)


Diplpode
Enquadram nessa classe, os animais conhecidos como estrelas-
Equinodermos do-mar. Geralmente possuem 5 braos, mas existem algumas es-
pcies com um nmero maior, porm, sempre mltiplo de cinco.
Caractersticas gerais
No dorso, distingue-se uma parte central denominada disco, do
So metazorios que apresentam larvas com simetria bilateral
qual partem cinco prolongamentos idnticos denominados braos.
e adultos com simetria pentarradiada, onde a cabea no se acha
diferenciada do corpo. Este coberto por uma delicada epiderme Classe Ophiuroidea (=Ofiurides)
ciliada, envolvendo um esqueleto constitudo por placas calc- Animais conhecidos como serpentes-do-mar, apresentam o
reas fixas ou mveis. Possuem habitat exclusivamente marinho, corpo estrelado, mas diferem dos asterides, por apresentarem

314 UECEVEST

Apostilas UECEVEST mod3.indb 314 06/02/2011 10:04:47


BIO LO G I A 1 I

o disco central bem diferenciado dos braos. Estes, em numero 05. (UECE 2010.2) Visitando a coleo de fsseis da UECE,
de 5, so cilndricos, delgados, simples ou ramificados. Quando Paulinha identificou os seguintes:
vivos, apresentam movimentos serpenteados, da o seu nome.

Classe Holothuroidea (=Holoturides)


So conhecidos popularmente como pepinos-do-mar; ao
contrrio dos outros equinodermatas, eles apresentam o corpo
cilndrico e alongado, com tegumento mole abaixo do qual se
acham espalhadas placas calcreas microscpicas que funcionam
como endoesqueleto.

E X E R C C I O
01. (UECE 2010.2) Muitos animais passam por um processo co-
nhecido como ecdise. Assinale a alternativa que contm somente
animais que substituem periodicamente seus exoesqueletos du-
rante o seu crescimento.
a) cobras, ostras e caranguejos
b) aranhas, baratas e caranguejos
c) caranguejos, corais e aranhas
d) baratas, cobras e aranhas

02. (UECE 2010.2) Um monitor da disciplina de zoologia do Chegando em casa resolveu brincar com Pedrinho e montou
curso de Cincias Biolgicas recebeu do seu professor, para utili- um quadro pedindo-lhe para identificar, pelo algarismo romano
zar em uma aula prtica, uma tabela contendo animais que deve- correspondente, a nica linha que relacionava corretamente os
riam ser colocados em seus respectivos filos. fsseis a organismos animais que possuam esqueletos internos
Animal Filo e externos.
Caranguejo
Minhoca
Caracol
Polvo
Barata

A resposta que completaria corretamente a tabela, de cima para


baixo,
a) Crustcea, Annelida, Mollusca, Cephalopoda, Insecta.
b) Arthropoda, Annelida, Mollusca, Mollusca, Insecta.
c) Arthropoda, Annelida, Mollusca, Mollusca, Arthropoda.
d) Crustcea, Annelida, Gastropoda, Cephalopoda, Insecta.

03. A cabea e o trax so fundidos, constituindo o cefalot-


rax[...]O ltimo metmero abdominal apresenta um par de apn- A associao correta a do algarismo
dices achatados, os urpodes, que, juntamente com uma pea a) I. c) III.
terminal afilada denominada tlson, constituem a cauda, utiliza- b) II. d) IV.
da para nadar.
Amabis & Martho, 2004 06. (UECE 2009.1) Maria Paula e Pedro combinaram fazer um
sorteio de estruturas que deveriam estar presentes no animal que
O texto refere-se a qual grupo de animais do Filo Arthropoda?
iriam consumir em uma saborosa refeio. Dentre as estruturas
a) insetos d) crustceos
sorteadas apareceram: glndulas coxais, maxilpedes e brnquias.
b) aracndeos e) diplpodes
Para esta refeio escolheram um caranguejo. Quanto escolha,
c) quilpodes
podemos afirmar, corretamente, que
a) equivocada, pois no existe um animal com as trs estruturas
04. Um certo parasita, que causa uma doena humana, aloja-se
sorteadas.
no estmago e depois na glndula salivar do hospedeiro trans-
b) acertada, pois o caranguejo possui as trs estruturas sorteadas.
missor. A seguir, no ciclo de transmisso da doena para o ho-
c) equivocada, pois o animal com as trs estruturas sorteadas
mem, o parasita invade a corrente sangunea, depois o fgado,
a aranha.
onde se multiplica, atingindo novamente a corrente sangunea.
d) equivocada, pois o animal com as trs estruturas sorteadas
O parasita, o hospedeiro transmissor e a doena descritos so,
a lagosta.
respectivamente:
a) Trypanosoma gambiensis/ Glossina palpalis/ Doena do Sono
07. A respirao e a circulao nos insetos sustentam a alta de-
b) Trypanosoma cruzi/ Triatoma infestans/ Doena de Chagas
manda metablica desses animais durante a vo. Alm disso, a
c) Leishmania brasiliensis/ Phlebotomus intermedius/
respirao fraqueal uma importante adaptao dos insetos para
Leishmaniose
a vida terrestre. Sobre as relaes fisiolgicas entre os processos
d) Plasmodium vivax/ Anopheles/ Malria
respiratrio e circulatrio nos insetos, correto afirmar:
e) Wuchereria bancrofti/ Culex fatigans/ Filariose

UECEVEST 315

Apostilas UECEVEST mod3.indb 315 06/02/2011 10:04:48


B I O LO GIA 1I

a) o sistema circulatrio aberto contm hemocionina, c) indicar seu perodo de reproduo.


pigmento respiratrio que facilita o transporte de oxignio d) referir-se sua classificao taxonmica.
do sistema traqueal para os tecidos. e) referir-se sua nocividade.
b) o sistema traqueal conduz oxignio diretamente para os
tecidos e o dixido de carbono em direo oposta, o que 13. (UFRGS) Os aracndeos so animais terrestres que ocorrem
forma a respirao independente de um sistema circulatrio em uma grande diversidade de habitats. Em relao a estes ani-
c) o sistema traqueal conduz oxignio da hemolinfa para os mais correto afirmar que:
tecidos, o que torna a respirao dpendente de uma sistema a) possuem antenas.
circulatrio b) possuem trs pares de patas.
d) o sistema circulatrio fechado contm hemoglobina e c) apresentam excreo por nefrdios.
fundamental para o transporte de oxignio do sistema d) possuem quelceras.
traqueal para os tecidos e) so monicos.

08. (ACAFE-SC) So representantes do filo Echinodermata: 14. (OMEC-SP) Assinale a alternativa que aponta o erro co-
a) crinides, ourio-do-mar, corais. metido na caracterizao do filo Echinodermata: So animais
b) corais, holotrias, crinides. exclusivamente marinhos, de organizao pentarradiada, com
c) euglena, ourio-do-mar, crinides. larvas de simetria bilateral, esqueleto calcrio externo, triblsticos
d) ourio-do-mar, ofiros, holotrias. e deuterostmios.
e) ostras, ofiros, crinides. a) animais exclusivamente marinhos.
b) larvas de simetria bilateral.
09. Os animais conhecidos popularmente como taturas, co- c) esqueleto calcrio externo.
muns nas praias do litoral gacho, pertencem ao grupo dos Crus- d) triblsticos.
tceos. Qual das alternativas apresenta somente animais que fa- e) deuterostmios.
zem parte deste grupo taxonmico?
a) ostra caramujo lula 15. (UFGRS) Numa expedio cientfica para o litoral, um gru-
b) siri tatuzinho-de-jardim camaro po de estudantes separou os animais capturados nos trs grupos
c) craca lagostim marisco seguintes:
d) centopeia mexilho lacraia Grupo 1 Grupo 2 Grupo3
e) ourio-do-mar caranguejo anmona
estrelas-do-mar e taturas e
mexilhes e lulas
ourios camares
10. (Unesp) Existem no filos animais estruturas que os caracteri-
zam e os identificam. Coanacitos, cnidoblastos, culas-flama ou Qual das caractersticas abaixo foi usada para efetuar a separao
solencitos, sistema ambulacrrio e rdula so algumas estruturas dos grupos?
encontradas, respectivamente, em: a) O tipo de simetria.
a) platelmintos, celenterados, equinodermos, platelmintos e b) A presena de um celoma.
moluscos c) A presena de brnquias.
b) porferos, celenterados, equinodermos, platelmintos e d) O tipo de sustentao do corpo.
moluscos e) A presena de antenas.
c) porferos, celeterados, platelmintos, equinodermos e
moluscos, 16. Em relao a grupos de invertebrados, considere as caracte-
d) porferos, celenterados, platelmintos, moluscos e rsticas citadas abaixo.
equinodermos 1 - presena de dois pares de antenas.
e) equinodermos, moluscos, porferos, celeterados e 2 - corpo metamerizado.
platelmintos 3 - hbitat exclusivamente marinho.
4 - presena de exoesqueleto.
11. Os carrapatos so insetos geralmente pequenos, com cabea, 5 - locomoo atravs de sistema ambulacrrio.
trax e abdome fundidos. Na maioria das espcies desses artr-
podes, a ecloso do ovo origina uma larva que se transforma em Assinale a correspondncia correta entre o grupo animal e suas
ninfa e, posteriormente, em adulto com quatro pares de patas. caractersticas.
Quando adultos, so ectoparasitas e alimentam-se de sangue. a) Aneldeos - 2 e 5.
Este atexto sobre o carrapato apresneta ERRO ao: b) Moluscos - 2 e 4.
a) referir-se sua classe taxonmica c) Crustceos - 3 e 4.
b) referir-se aos seus estdios de desenvolvimento d) Insetos - 1 e 4.
c) descrever a sua diviso do corpo e) Equinodermos - 3 e 5.
d) referir-se ao nmero de patas do adulto

12. Agosto e setembro so os meses em que mais aparecem es-


corpies, pois justamente nesses meses que eles tm seu perodo
de reproduo. O inseto se refugia nos mais diferentes lugares, G A B A R I T O
como amontoados de madeira, pedras, entulhos, fendas na pare- 01. 02. 03. 04. 05. 06. a
de ou atrs de mveis. Diariamente, pelo menos uma pessoa pica- 07. b 08. d 09. b 10. c 11. a 12. d
da por escorpio atendida no Centro de Toxologia do hospital
Joo XXII. (Estado de Minas, 17/08/1990) 13. d 14. c 15. d 16. e
Essa matria sobre o escorpio incorre em erro ao:
a) descrever seu comportamento.
b) indicar seu hbitat.

316 UECEVEST

Apostilas UECEVEST mod3.indb 316 06/02/2011 10:04:49


BIO LO G I A 1 I

CORDADOS (UECE/EnEm) habituados: peixes (com exceo das mixinas), anfbios, rpteis,
aves e mamferos incluindo o Homem pertencem a este gru-
O grupo dos Cordados (filo Chordata) inclui os Protocor- po. Outras caractersticas adicionais so a presena de um sistema
dados que so cordados sem coluna vertebral e crnio, e os Ver- muscular geralmente simtrico a simetria bilateral tambm
tebrados que possuem crnio e quase sempre a coluna vertebral uma caracterstica dos vertebrados e de um sistema nervoso
(apenas os peixes da classe Myxini no possuem esta estrutura). central, formado pelo crebro e pela medula espinhal localizados
Os protocordados se dividem em 2 subfilos: Cefalocordados, re- dentro da parte central do esqueleto (crnio e coluna vertebral).
presentados pelo anfioxo e os Urocordados, animais ssseis que
vivem grudados em rochas submersas e em grandes algas. Peixes
Os peixes so animais vertebrados, aquticos pecilotrmicos
Caractersticas gerais e que possuem os membros transformados em nadadeiras sus-
Triblsticos; tentadas por raios sseos, esqueleto sseo ou cartilaginoso. As
Deuterostmios; principais adaptaes a este meio de vida so o corpo fusiforme,
Simetria bilateral; as guelras ou brnquias com que respira o oxignio dissolvido na
Segmentao (metamricos). gua (embora os dipnicos usem pulmes), os membros transfor-
mados em barbatanas e, na sua maior parte, o corpo coberto de
Caractersticas Exclusivas escamas. Revestimento: pele com escamas e muco (para deslizar
Esse filo se caracteriza por possuir indivduos que, pelo menos melhor na gua).Locomoo: por nadadeiras. Bexiga natatria
no estgio embrionrio, apresentam trs caractersticas exclusivas permite que suba e desa. Temperatura do corpo: so pecilotr-
do grupo (entre os animais estudados): micos, ou seja, animais cuja temperatura do corpo muda de acor-
Notocorda; do com o ambiente. Respirao: por brnquias, retiram oxignio
Fendas Faringeanas; da gua. Reproduo: existem machos e fmeas. Na maioria a fe-
Tubo Nervoso Dorsal. cundao externa, na gua. Desenvolvimento do ovo: externo,
Cauda ps-anal so ovparos. Metamorfose: existe s em alguns casos. Corao
dividido em 2 cavidades: um trio e um ventrculo.
notocorda
uma estrutura em forma de basto, um pouco flexvel e que Anfbios
se encontra ao longo da regio dorsal do embrio, entre o tubo Os anfbios (anfi= duas ; bio= vida) so animais vertebrados,
nervoso e o tubo digestivo. Na maioria dos cordados a notocorda pecilotrmicos que no possuem bolsa amnitica agrupados na
substituda pela coluna vertebral (vertebrados). Nos cordados classe Amphibia. A caracterstica mais marcante dos seres vivos da
invertebrados (protocordados) a notocorda pode persistir nas for- classe o seu ciclo de vida dividido em duas fases: uma aqutica
mas adultas. e outra terrestre. Esto identificadas cerca de 3000 espcies vi-
vas de amfbios. Revestimento: pele lisa, mida, com muco (para
Fendas Faringeanas deslizar melhor na gua). Locomoo: por patas ou se rastejam.
So fendas que surgem durante o desenvolvimento embrio- Temperatura do corpo: so pecilotrmicos, ou seja, animais cuja
nrio de todos os cordados. Essas fendas so pares e se situam dos temperatura do corpo muda de acordo com o ambiente. Respira-
dois lados do embrio. Existem arcos de sustentao (arcos bran- o: girino por brnquias. Adulto por pulmes e atravs da pele
quiais) que mantm as fendas abertas. Em muitos cordados (ter- fina, com vasos sanguneos (respirao cutnea). Dependem da
restres) essas fendas se fecham nos adultos e os arcos branquiais gua para respirao. Reproduo: existem machos e fmeas. Fe-
do origem a estruturas diversas como mandbula, cartilagens da cundao externa. Ovparos (animais cujos ovos se desenvolvem
faringe e ossculos da orelha interna. Em outros (aquticos), elas fora do corpo da me, no ambiente). Metamorfose: larva o gi-
continuam abertas e se desenvolvem formando as brnquias des- rino. Corao dividido em 3 cavidades: 2 trios e um ventrculo.
ses animais (por esse motivo tambm so chamadas de fendas
branquiais). Rpteis
Os rpteis so animais vertebrados tetrpodes e ectotrmicos
Tubo nervoso Dorsal (no possuem temperatura corporal constante). So todos amnio-
Diferentemente dos no-cordados que possuem um cordo tas (animais cujos embries so rodeados por uma membrana ami-
nervoso macio, os cordados possuem um tubo nervoso e este nitica). Os rpteis atuais so representados por quatro ordens:
dorsal e no ventral. Nos vertebrados, esse tubo nervoso sofre Ordem Crocodilia crocodilos
uma dilatao na regio anterior, dando origem ao encfalo e a Ordem Rhynchocephalia tuataras
parte restante constitui a medula espinhal. Ordem Squamata lagartos
Ordem Testudinata tartarugas
Cauda ps-anal
Os embries dos cordados tm uma regio que se prolonga Revestimento: pele com escamas, placas ou carapaas. Loco-
alm do nus a cauda. O desenvolvimento e a funo da cauda moo: patas para andar ou nadar, os que no tem patas rastejam.
variam nos diferentes grupos. Ela pode servir para a natao, para Temperatura do corpo: so pecilotrmicos, ou seja, animais cuja
apoiar o corpo, como arma de defesa e para apreenso de objetos. temperatura do corpo muda de acordo com o ambiente. Respira-
Em algumas espcies, como na espcie humana, a cauda desa- o: pulmonar (por pulmes). Reproduo: sexos separados (ma-
parece completamente durante o desenvolvimento embrionrio. cho e fmea). Fecundao interna na fmea. Ovparos (ovos com
cascas). Metamorfose: desenvolvimento direto (nenhum caso de
metamorfose). Corao dividido em 3 cavidades: 2 trios e um
Subfilo Vertebrata
ventrculo.
O Subfilo Vertebrata ou Craniata composto por todos os
animais com coluna vertebral ou espinha dorsal formada por
OBS.:
vrtebras serialmente ordenadas que protegem a medula espinhal.
Algumas cobras so vivparas, isto , os ovos se desenvolvem
So comumente chamados de vertebrados ou craniatas. A maio-
no organismo materno.
ria dos animais com maior grau de organizao a que estamos

UECEVEST 317

Apostilas UECEVEST mod3.indb 317 06/02/2011 10:04:49


B I O LO GIA 1I

Aves de anos, achado no noroeste da Austrlia, agora o exemplo mais


As aves so animais vertebrados tetrpodes, endotrmicos, antigo de uma me grvida vivpara.
ovparos, caracterizados principalmente por possuirem penas, Cincia Hoje, jul. 2008.
apndices locomotores anteriores modificados em asas, bico a) A presena de um saco vitelino no embrio fssil seria uma
crneo e ossos pneumticos. So reconhecidas aproximadamen- caracterstica segura para determinar o fssil como vivparo.
te 9.000 espcies de aves no mundo. Revestimento: penas, que b) A concluso de que o fssil de um animal vivparo veio da
participam do vo e da manuteno da temperatura do corpo. observao de que o embrio estava se desenvolvendo dentro
Locomoo: A maioria voa e para isso os ossos so leves (ocos) e do corpo da me.
a musculatura do peito bem desenvolvida e rica em vasos san- c) Uma das caractersticas que levaria concluso de que
guneos. Temperatura do corpo: homeotrmicos (temperatura se tratava de um animal vivparo seria a presena de um
corporal interna constante; precisam de grandes quantidades de resqucio de cordo umbilical.
alimento para fornecer energia para manter a temperatura corpo- d) O fssil em questo poderia tambm ser de um animal
ral). Respirao: pulmonar (por pulmes). Reproduo: existem ovovivparo, pois, assim como os vivparos, os ovovivparos
machos e fmeas e a fecundao interna. So Ovparos e tem apresentam nutrio maternal durante o desenvolvimento
casca. Metamorfose: desenvolvimento direto, mas precisam dos embrionrio.
pais at aprender a voar e conseguir alimentos sozinhos. Corao
dividido em 4 cavidades: 2 trios e 2 ventrculos. 02. (UFC) At recentemente, o aparecimento das baleias era
um dos mistrios mais inexplicveis enfrentados pelos bilogos
mamferos
evolucionrios. Sem plos e membros posteriores, e incapazes de
Os mamferos (nome cientfico: Mammalia) pertencem
ir terra sequer para um gole de gua fresca, os cetceos atuais
classe dos animais vertebrados caracterizados pela presena de
so um desvio dramtico da norma dos mamferos (Scientific
glndulas mamrias nas fmeas, que produzem leite para alimen-
American Brasil, n.1, 2002, pg. 64.). A caracterstica marcante
tao dos filhotes (ou crias), e a presena de plos ou cabelos.
que inclui os cetceos na classe dos mamferos, apesar de sua apa-
So animais homeotrmicos, ou seja, de sangue quente.O cre-
rncia pisciforme, est relacionada ao fato desses animais:
bro controla a temperatura corporal e o sistema circulatrio, in-
a) respirarem o ar.
cluindo o corao (com quatro cmaras). Os mamferos incluem
b) serem triblsticos.
5.500 espcies (incluindo seres humanos), distribudo em aproxi-
c) serem celomados.
madamente 1.200 gneros, 152 famlias e at 46 ordens, embora
d) apresentarem circulao fechada.
isso possa variar de acordo com o esquema de classificao. Re-
e) terem reproduo sexuada.
vestimento: pelos. Locomoo: a maioria por patas em ambiente
terrestre, mas h adaptaes para nadar ou voar. Temperatura
03. No corao com duas e quatro cavidades no acontece mis-
do corpo: homeotrmicos. Os mamferos tm mecanismo para
tura de sangue venoso com sangue arterial. Nos primeiros circu-
abaixar ou aumentar a temperatura, quando esta se altera. Res-
lam somente sangue venoso, enquanto nos segundos circulam
pirao: pulmonar (por pulmes). Reproduo: sexos separados
o sangue venoso separado do sangue arterial. J no corao de
e fecundao interna. So Vivparos. Metamorfose: desenvolvi-
trs cavidades h mistura de sangue venoso com sangue arterial
mento direto. Podem ser classificados em monotremados, mar-
no ventrculo. Dentre os animais abaixo, aqueles que possuem
supiais e placentrios. Corao dividido em 4 cavidades: 2 trios
corao com trs cavidades so:
e 2 ventrculos.
a) mamferos/aves.
b) peixes/anfbios.
Monotremados
c) rpteis/anfbios.
uma ordem de mamferos primitivos que possuem patas,
d) rpteis/aves.
bico e apenas uma sada de excretas (similar a cloaca). classi-
ficado mamfero por causa da presena de glndulas mamrias,
04. (UFPA) Alm dos mamferos, os animais que apresentam
apesar de ele no possuir tetas, a fmea da espcie fica de costas e
corao com quatro cavidades e circulao sangunea do tipo
os filhotes lambem o leite que espalha na sua barriga. Os repre-
completa, sem qualquer mistura entre sangue venoso e arterial:
sentantes do grupo so somente o curioso ornitorrinco (aqutico)
a) rpteis.
e as quidnas (terrestres).
b) aves.
c) anfbios.
Marsupiais
d) peixes sseos.
Possuem esse nome por causa da presena de uma bolsa deno-
e) peixes cartilaginosos.
minada marspio, que ajuda no desenvolvimento do filhote. O
filhote nasce prematuro, pois no tem nada que o segure dentro
05. O carter compoartilhado que aproxima, evolutivamente, o
da barriga da me; ento ele nasce e continua o desenvolvimento
filo Echinodermata do filo Chordata :
dentro do marspio. Gambs, cucas, cangurus e demnios da
a) a presena de notocorda na fase embrionria
tasmnia so exemplos desses animais.
b) o desenvolvimento de um endoesqueleto
Placentrios c) a origem do tudo digestivo pela protosmia
Possuem dentro do tero a placenta, que ajuda o filhote a d) o aparecimento do celoma pela esquizocelia
ficar dentro da barriga da me por mais um tempo. o maior
grupo dos mamferos, com macacos, ces, bovinos, etc. 06. (UNIMEP-SP) Assinale a alternativa em que existem seres
no pertencentes ao grupo dos cordados:
a) cobra, salamandra e jacar.
b) estrela-do-mar, aranha e lagosta.
E X E R C C I O c) homem, gorila e chimpanz.
d) tubaro, baleia e leo-marinho.
01. (UFC 2009) Leia o texto a seguir e assinale a alternativa cor- e) anfioxo, sapo e tartaruga.
reta. Um fssil extremamente bem conservado, de 380 milhes

318 UECEVEST

Apostilas UECEVEST mod3.indb 318 06/02/2011 10:04:50


BIO LO G I A 1 I

07. (UFC 2008.2) Em muitos animais verificamos algumas es- b) os animais ectotrmicos, como as aves, dependem de uma
truturas semelhantes em sua anatomia ou funo. Um exemplo fonte externa de calor, bem como do auxlio de penas para
disso so as asas das aves e dos insetos, ambas usadas para vo. manter a temperatura corprea constante.
Estas estruturas so conhecidas como: c) a necessidade de ambientes midos para a sobrevivncia dos
a) homlogas, assim como os plos dos mamferos e as penas anfbios consequncia principalmente de sua ectotermia.
das aves. d) os animais endotrmicos so capazes de manter a
b) homlogas, assim como as nadadeiras das baleias e dos temperatura corprea constante atravs da produo interna
tubares. de calor.
c) anlogas, assim como as asas dos morcegos e o brao humano. e) a endotermia e a homeotermia so caractersticas
d) anlogas, assim como as patas anteriores do co e do cavalo. compartilhadas por mamferos e peixes cartilaginosos.
e) anlogas, assim como as patas dos caranguejos e dos rpteis.
15. Determinados animais constituem uma transio entre os
08. So caractersticas exclusivas das aves: invertebrados e os vertebrados. No possuem esqueleto sseo e,
a) endotermia, penas e corao com 4 cavidades sim, apenas notocorda:
b) endotermia, sacos areos e penas a) Batrquios. d) Insetos.
c) corao com 4 cavidades b) Amebas. e) Celenterados.
d) sacos areos e penas c) Protocordados.

09. No ano 2000, pesquisadores da Universidade da Carolina do 16. (UECE 2009.1) Asas de aves e asas de insetos possuem ori-
Norte (EUA) divulgaram, pela primeira vez, a descoberta de um gem embrionria diferentes e exercem a mesma funo. Portanto,
corao fossilizado, com quatro cavidades, pertencente a um di- so consideradas rgos
nossauro, Willo, que viveu h 66 milhes de anos. A preservao a) vestigiais. c) homlogos.
dessa rara evidncia anatmica veio somar-se s feies osteolgi- b) anlogos. d) mimticos.
cas observadas nos esqueletos fsseis de dinossauros. Isso refora
a hiptese filogentica de que, dos grupos citados nas alternativas, 17. (UECE 2009.1) Mamferos so animais extremamente in-
os mais estreitamente relacionados com os dinossauros so: teressantes, existindo cerca de quatro mil espcies conhecidas,
a) os lagartos e os crocodilos. distribudas pelo mundo. Esses animais encontram-se classifica-
b) os anfbios e as aves. dos em dois grandes grupos, onde os ornitorrincos e as quid-
c) os crocodilos e as aves. nas (exclusivos da Austrlia) pertencem Subclasse Prototheria
d) as tartarugas e os mamferos. e todos os outros animais esto colocados na Subclasse Theria.
e) os lagartos e os mamferos. Com relao aos mamferos, so Marsupiais e Placentrios, res-
pectivamente:
10. (UFC) O carter compartilhado que aproxima, evolutiva- a) morcego e baleia. c) cassaco e morcego.
mente, o filo Echinodermata do filo Chordata : b) rato e cachorro. d) canguru e gamb.
a) a presena de notocorda na fase embrionria.
b) o desenvolvimento de um endoesqueleto. 18. (UECE 2009.2) Pedro foi ao zoolgico, onde deveria listar
c) a origem do tubo digestivo pela protostomia. animais, somente homeotrmicos. A sua lista estaria corretamen-
d) o aparecimento do celoma pela esquizocelia. te formulada se constassem nela os seguintes animais:
e) o desenvolvimento de um tubo nervoso dorsal. a) golfinho; tatu; jacar; papagaio.
b) tejo; arara; tartaruga; preguia.
11. (UFC) Assinale a alternativa que apresenta o componente do c) peixe-boi; foca; tucano; hipoptamo.
corpo dos animais cujo constituinte principal a queratina: d) pelicano; jaboti; jibia; crocodilo.
a) Camada drmica da pele de mamferos.
b) Endoesqueleto de condrctes e ostectes. 19. (UECE 2009.2) Tanto nas aves quanto nos mamferos o te-
c) Escamas da pele de serpentes e lagartos. gumento e seus anexos contribuem para o(a)
d) Cutcula dos diplpodes e quilpodes. a) manuteno do nvel de oxignio no sangue.
e) Concha dos gastrpodes e bivalves. b) regulao do teor de ureia no sangue.
c) manuteno da temperatura corporal constante.
12. (UFRN) Na maioria dos peixes sseos o rgo responsvel pela d) controle do teor de gua no organismo.
manuteno do equilbrio hidrosttico com o meio denominado:
a) bexiga natatria. d) brnquias. 20. (UECE 2009.2) Analise as afirmaes abaixo, classificando-
b) nadadeira caudal. e) linha lateral. as em verdadeiras (V) ou falsas F.
c) clsper. ( ) Nos mamferos o corao encontra-se dividido em quatro
cavidades: dois trios e dois ventrculos. Desta forma, o
13. A moela uma bolsa muscular presente no tubo digestivo de sangue oxigenado no se mistura com o sangue rico em gs
muitas aves onde: carbnico.
a) ocorre a absoro de alimento ( ) As baleias, os golfinhos e o peixe-boi, so exemplos de
b) vivem microorganismos que digerem celulose mamferos aquticos e, portanto, as mes amamentam suas
c) o alimento triturado juntamente com as pedras engolidas crias em baixo da gua.
d) o alimento armazenado antes de ir para o estmago ( ) A caracterstica principal que identifica um mamfero
a presena de uma notocorda bem desenvolvida na fase
14. (UFC 2008) A respeito do processo de controle da tempera- embrionria, a qual ir se transformar em tubo nervoso na
tura corprea nos vertebrados, correto afirmar que: vida adulta.
a) os rpteis, animais ectotrmicos, mantm sua temperatura ( ) Dentre os mamferos encontram-se animais que possuem
corprea sempre elevada devido presena das escamas membros locomotores modificados em asas, patas e
drmicas, que funcionam como isolante trmico. nadadeiras.

UECEVEST 319

Apostilas UECEVEST mod3.indb 319 06/02/2011 10:04:50


B I O LO GIA 1I

( ) Mamferos so animais exclusivamente vivparos, diicos,


com fecundao interna e desenvolvimento direto.
Assinale a opo que contm a sequncia correta, de cima para
baixo.
a) V, V, V, F, V c) F, V, F, V, F
b) F, F, V, F, V d) V, V, F, V, F

21. (UECE 2010.1) Joo Alberto andava pelo campo. Por um


descuido foi picado por uma cobra. Identificou a serpente e ime-
diatamente foi a um posto de sade, j apresentando diplopia,
ptose palpebral, dilatao da pupila e insuficincia respiratria
aguda. Contudo, foi tratado com soro antielapdico, recuperan-
do-se do agravo sua sade. Podemos afirmar que a cobra que
picou Joo Alberto foi
a) jibia. c) cascavel.
b) jararaca. d) coral.

G A B A R I T O
01. 02. 03. 04. 05. 06. b
07. e 08. d 09. c 10. b 11. c 12. a
13. c 14. d 15. c 16. b 17. c 18. c
19. c 20. d 21. d

REFERnCIAS BIBlIOGRFICAS
AMABIS e MARTHO. Biologia dos Organismos. Volume 2.
2 ed. So Paulo: Moderna, 2004.

CSAR & CEZAR. Biologia 2. So Paulo: Saraiva, 2002.

LOPES, SNIA. Bio 2. So Paulo: Saraiva, 2002.

SOARES, JOS LUIS. Dicionrio etimolgico e


circunstanciado de Biologia. So Paulo: Scipione, 1993.

Revistas
*Cincia Hoje
Excelente revista brasileira (mensal), responsvel pelas publica-
es de divulgao cientfica da Sociedade Brasileira para o Pro-
gresso da Cincia (SBPC). Apresenta artigos de qualidade sobre
os resultados de pesquisas realizadas no Brasil e no Exterior, em
todas as reas de conhecimento. Artigos disponveis on-line:
http://www.ciencia.org.br

*Scientific American Edio Brasil


Edio (mensal), em portugus, da famosa revista americana
Scientific American, com artigos de excelente qualidade sobre
Biologia, Fsica, Antropologia etc.

320 UECEVEST

Apostilas UECEVEST mod3.indb 320 06/02/2011 10:04:51

Das könnte Ihnen auch gefallen